Sie sind auf Seite 1von 610

.. .

'-··
/ .�. :. -·.
. .
�,'-

,•: ,· : .

)
' ..
'

COURSE IN
................. "'�
./ ' :· :- ...t�':
, ·.
..... ·.
.
,'.,,
.

.i; -' :

j I

� j • !

"K ....... , ..,........0··· ..ri·f�i··


...U.�;.�......
..C../.'..:j ..( �.J../. L.. . . . ........... .
..J:J.1'.�/ ..�\...... (0r:\L ..... ·· .J'Q.(U o I
.. ..)< iJi CL � e .... ' "!!,..hi o .,__
..��.. ��-........ �P. (8.-/ 3
....... r.1o.\;bo.... c1�. . .. . ... ... . . . 2004
0

o •" f. 1 1 • 0 0 O ' O O O O Of," f I t O I I" 0 • • 0 •to I, o O � • • I�� I ' •

f """'""'""""''"'"""""·"•··
I O O • 0 0 0 t o O ' o O. 0 0 I o O O O I O I I O o O " • • o' > '' ' 4 • ,a I• t ti �
A HIGHER
SCHOOL CERTIFICATE COURSE
IN MATHEMATICS

YEAR TWELVE

3
UNIT
COURSE

by

J. CORONEOS, B.Sc., Dip. Ed.


ALL RIGHTS RESERVED

Printed and bound in Australia


by The Book Printer, Victoria
PREFACE

The present volume for 6th form students completes the


3 unit Course for the Higher School Certificate Exam-
ination. The plan in this book has been to present a
textbook for all groups.

The following features are incorporated in this volume.

ITEM]. The great ni,nber of worked��s on each..:fQE_ic.


(The working through of these by the student will ben­
efit him or her greatly. The more mature student will
take advantage of this to work ahead and proceed with­
out teaching for long periods of time.)

ITEM 2. The tremendous variety of exercises.


(It is not intended that every exercise will be worked
by each student, but of course as many exercises as
possible should be done in the allotted time, This
allows for the greatly differing abilities between the
weakest and strongest students.
These exercises have been carefully graded and the
n�ber of exercises per set enables plenty of valuable
practice for the students. Those exercises which are
unmarked are within the capabilities of all students.
Those marked with tare supplementary exercises and in­
tended for further practice; and those marked with•
are more difficult and intended for the best students.)
ITEM 3. The Students' Guid.e - listing the suggested period
g.?_locations.
(This is a valuable guide to
(a) the· amount of time which is suggested should be
spent on each topic
and (b) the exercises which each group of students,
(A, B or C) may be expected to cover in the
allotted time.
The allotted time takes into account the 60 - 90minutes
which should be the minimum time spent by a
student on Mathematics each night.)

:TEM 4. Thorowh and c0111Plete revision of the entire


Course topic by topic.
(This revision section lays emphasis on the understand­
ing and derivation of the basic results in the Course.
It gives a systematic re-appraisal of the whole Course
and is not merely a collection of uncorrelated and ir­
relevant exercises. It is suggested that the revision
exercises could be tackled as assignments during the
year, or cOODnenced as soon as the basic Course has
been completed. )

fJJ:J1_5, _'f.!ie Answer Seation.


(The magnitude of this section indicates that the
answers are not just answers hints, methods of
solution, sketches are supplied in detail. Every help
possible to the student is intended. Obviously, in a
work of this scope, some errors or omissions are un­
avoidable and some allowance in this regard is appreci­
ated.)

ITEM 6. The attention to detail tfu>oughout the Textbook.


(No topic in the Syllabus has been glossed over. Eaah
item of the Syllabus has been dealt with carefully and
thoroughly, The scope of the exercises is so wide that
practically every conceivable aspect of each topic or
item is treated.)

Specimen ·Papers and Complete Worked Solutions.


Associated with this book (and by the same author) are
sets of Speaimen Papers. These consist of 11 Specimen
Papers C and 11 Specimen Papers B E2cause of space
consideration, they were not able to be incorporated in
this text. (The 22 papers and answers occupy nearly 100
pages,)
Complete worked solutions are available for both of
these sets of Speaimen Papers.

Chapters l to 7 should be completed within 22 weeks, that


is by either the end of the second term or the beginning of
the third term. This will allow the rest of the time before
the Higher School Certificate Examination a period of at
least 6 - .8 weeks to be spent on the revision sets and
the specimen papers.

J. Coroneos,
580 Old South Head Road,
Rose Bay 2029.
Postal address: P.O. Box 25
Rose Bay 2029.
Phone: 371-8530.
C O N T E N TS
CHAPTER 1. THE INVERSE TRIGONOMETRIC FUNCTIONS
Inverse Sine Function - Definition, Sketch, Domain, Range,
Derivative, Similar Treatments for the Inverse Cosine and
Inverse Tangent Functions - NI.Dllerical Evaluations
Derivatives including Applications - Integrals including
Applications to Areas, Volumes ••••••••••• � •••••••• page 1

CHAPTER 2. ANALYTICAL GEOMETRY IN THREE DIMENSIONS


Review of Geometric Results - Points in Space - Direction
Angles, Direction Cosines, Direction Ratios - Angle be­
tween Two Lines - Equations of Lines - Change of Origin· -
Ratio-Parametric Form for the Equation of a Line - Sur­
faces. (Spheres, Paraboloids, Cylinders, Planes). Equations
of Planes - Parallel and Perpendicular Planes Angle
Between 2 Planes - Angle Between a Line and a Plane
Regions •.....•.•.•.......•.•••.••..•••..••.•••.•.• page 35

CHAPTER 3. APPLICATIONS OF CALCULUS TO THE PHYSICAL WORLD


Motion in a Straight Line - Displacement,Average Velocity
Velocity and Acceleration as Derivatives - Determin­
ation of Velocity and Position given the Acceleration and
Initial Conditions, for the cases
d2x d 2x d2x
(i)
W= constant; (ii)
Wd=2xf(t) (iii) di"2" = g(x)
Simple Harmonic Motion, from -:.72' = -n2 x - Solution of
dt
the Equation - Exercises Motion of a Particle under
Hooke's Law - Basic Ideas, Exercises - Parabolic Motion
under Gravity - Projectiles - Parametric and Cartesian
Equations of the Path •••••••••••••••••••••••••••••• page 87

CHAPTER 4. FURTHER TRIGONOMETRY AND CALCULUS


Extension of (a+B) Results to 26, 36, t Results - Trig-
onometric Equations types involving use of Identities,
26, 36 results; a cose + b sine= c Expressing
a cose + b sine= c in the forms R cos(6±a), Asin(6±B)
Derivatives of Cosec x, Sec x, Cot x - Applications
Review of the Trigonometric Integrals possible at this
stage .......•.................•.•.•..•.•.....•••..• page 171

CHAPTER 5. FURTHER POLYNOMIALS


Basic Ideas - Degree, Monie Polynomial. Sketching Poly­
nomials - Zeros of a Polynomial The Division Trans­
formation - Remainder and Factor Theorems - Deductions
from the Factor Theorem - Relations between the Roots and
Coefficients of Polynomial Equations Locating Real
Roots of p(x) = 0 - Approximation Methods to the Real
Roots of p(x) = 0 ••• (i) Halving the Interval
(ii) Newton's Method ••••••••••••••••••••••••••••••• page 226
CHAPTER 6. THE BINOMIAL THEOREM
Pascal's Triangle - Expansion of (l+x)n, (a+x)n for small
positive integral values of n 'lbe Result
nc n
+ c • n+l n
c - 'lbe Formula for c - The Binomial
r- 1 r r r
Theorem and Applications - Combinatorial Interpretation
of the Binomial Coefficients ••••••••••••••••••••••• page 273
CHAPTER 7. THEORY OF PROBABILITY
Basic Ideas - Definition of the Probability of an Event -
Sets and Events - Algebra of Events - 'lbe Total Probabil-
ity Law P(A U B) • P(A) + P(B) - P(AB), and ·the Law
P(A U B) z P(A) + P(B) for Mutually Exclusive Events -
P(AB) = P(A) • P(B) - Exercises on these Laws - Counting
Procedures in a Finite Sample Space Ordered and Un­
ordered Selections Applications to Probability
Exercises The Binomial Distribution and Binomial
Probabilities - Applications of these - Random Digits -
Expectation - E(X) • np for a Binomial Variable ••••• page 299
APPENDIX. SEQUENCES AND LIMITS OF SEQUENCES
Definitions - Graphical Representation of Sequences
'lbe Limit of a Sequence Determining Limits of
Sequences • . . . . . . . . . • . • . . . . . . . . • . . .• . . . . . • . • . . . . . . . . . page 401

REVISION - SECTION BY SECTION OF THE ENTIRE COURSE


The Real Number System - Functions, Graphs and Locus -
Trigonometry - 'lbe Straight Line - Sequences and Series -
Introductory Calculus - The Quadratic Polynomial and the
Parabola - Geometric Applications of t�e Derivative -
Applications of Integration - Logarithmic and Exponential
Functions -Trigonometric Functions - Inverse Trigonometric
Functions - Three Dimensional Analytical Geometry
Applications of Calculus to the Phyf!ical World - Further
Trigonometry and Calculus - Further Polynomials - The
Binomial 'lbeorem - 'lbeory of. Probability - Change of
Coordinate -Systems Transformations - Mathematical
Induction - Limits of Sequences •••••••••••••••••• page 416
ANSWERS •••••.••.•••••••...•••••..••••••••••.•••.••• page 53 1
STUDENTS' GUIPE TO THE
HIGHER SCHOOL CERTIFICATE COURSE IN MATHEMATICS
FORM VI COURSE
BY
J. CORONEOS

The following Guide to Exercises has been prepared to


assist you in studying this Course.

The same classifications as in the Form V book have been


used (C) the weaker students
(B) the reasonable mathematicians
(A) the top students,

SUGGESTIONS
1, A careful study of the items mentioned in the Preface of
this book should be made by you.

2. The period allo.cation for any set of exercises includes


(a) Time to aover the requisite theory and worked
examples. These should be studied and worked
throUffl! by you. Then you will be prepared for the
alass work to foll()I,),
(b) Time to cover certain exercises in class during
normal periods.
(c) Rornta•vork from the set, (The sets of exercises
have been carefully graded for this reason, How­
ever, it is probably preferable for you to practice
the section just treated in class, rather than un­
dertake any home study exercises involving
radically new ideas, unless you fall into classifi­
cation ( p,) or (B).
You will realise that without extra work at home,
your �!athematics will not improve and with the
higher matriculation requirements this is
essentia l, You are encouraged to work ahead of the
class, if possible, and to complete as many exrr�
cises from each set as you can in the time
allocated.
A student taking thie Course in Mathematics
should spend a minimum of 60 - 90 minutes eaah
night on this subjeat.

3. The nwnber of exercises to be completed in each set


varies, depending on your classification (.jJ, (3) or
(C), The exercises set for group (C) could well serve
as the minimum course for all students, Group (n-)
students should cover all exercises recommended for
Rroup (C ), as well as those extra exercises stated,
Group (A) students should cover all the exercises
recommended for groups (B) and (C), as well as those
extra exercises stipulated,

4. Remember, suaaess or failure in the cour se examination


in the Higher Sahool Certifiaate is, in the final
reakoning, up to you, and just how muah real effor>t
you put into your work.
We have supplied what we feel is a first-rate Textbook
from which you can study and obtain your mathematics
practice.
You have now to rely on yourself and on your teacher to
continue and complete the task we all want to see
achieve4 namely that you obt�in as high a mark as
possible in .liathema tics in the Higher School Certificate
Examination.
STUDENTS I GUIDE

One of the most vital features of this text Is the great number
of worked examples, and the amount of space devoted to trying to
make the subject matter as clear as possible. It Is your respons­
ibility as a student for the most important examination of your I lfe
so far, to study the text material carefully by yourself, You should
work through each of the worked examp I es, and d I rected exerci·ses, by
yourself. [This can only be done with pen and paper simply
reading the text and examples is no good, You will not fully under­
stand the iqe�s without resort to pen and paper.] As a senior
.student, you cannot expect your teacher to spend valuable c la.ss time
doing your task for you,

EXERCISES TO BE DONE BY
TOPIC SET PERIODS GROUP lC)
IB) IA)
CHAPTER l
Titeory - TI,e Inverse Trig, - 2-3 -- -- --
Functions; Definition,
sket�h, domain, range
sin- (-x) • -sin-1x etc,
Derivatives.
Numerical Exercises lA 2 1,3,5(1- 2, 5 4
iii),6,7 (iv,v)
Exercises on Derivatives 1B 3-4 1,2,4-6, 7,12, 3,18
8-11,13, 15,17
14,16 •
Integration (Study lC .4 l,2(a-h), 4,7 8
standard results) 3,5,6,9
Applications of lD 4 1-6,9, 7(i,ii) 7
Integration (Study 10-12 8 (iii)
examples)

CHAPTER 2
Review of Geometry; Basic 2A 2-3 1-11 -- --
Ideas; Distance between
2 points, Midpoint
Directions in Space, 2B a-4 1-23 24,25 --
Angle between 2 Lines
(Study theory+ examples)
Equations of Lines; 2C 4 1-21 22,23 24
Parallel and Perpendicular
Lines; Change of Origin
(Study theory)
Surfaces (Spheres, para- 2D 2-3 l-4(l) 4(m,n, --
boloids, cylinders) o)
Equations of Planes 2E 3-4 1-7,9,10 8 --
Parallel and Perpen- 2F 3 1-10 -- --
dicular Planes; Angle
between 2 planes;
Angle between Line and
a Plane
STUDENTS I GUIDE
EXERCISES TO BE DONE BY.
TOPIC SET PERIODS CROUP <°C) CB) CA)
Regions (Study examples) 2G 2-3 (a)-+(v)
Further Exercises 2H 3 1-16,19 17,18

CHAPTER 3
Displacement - Average 3A 2 1-3.,5,6 4,7
Velocity
Velocity as a Derivative 3B 1-2 1-4 5
Acceleration as a Derivative 3C 1-2 l,2 3,5- 4
Directed Exercises 3D 4-5 JJork
through examples 1-8
Exercises on Derivatives 3E 3 1-9 10 --

Determining Velocity and


Position, given
d2 x
(i) dt7' • a constant
d2 x
(ii) dt7' • f(t) 3F 3-4 117,9-15, 8,16 16(1)
[Work through examples 1-9] 17 (ii-iv)
18-21
· d2·x
(iii) dt7' • g(x) 3G 3-4 1-4,6-'12 5,B-16
[Work through examples]
Simple Harmonic Motion 3H 5 1-9(1), 16-12 9(11)
(Definition, Solution of 13-15
x• -n 2 x)
Work through examples
Hooke's Law 3.1 3-f+ 1-4 5,6 7
(Ideas + examples)
P1(ljectiles. 3J 4 1-6,8,9 7
(Discussion, Derivation of
Parametric Equations,
Equation of Path, etc,)
Work through examples 1-6

CHAPTER 4
26, 36, t results 4A 3 1,2,4-8 3
(Theory+ worked examples)
Eliminations 4B 1-2 1,2(1-xii) 4
Proofs using 26,36,t results 4C 2-3 1-11,11, 12,13,
15,16
TRIGONOMETRIC EQ.UATIONS. [Note in sets 4D, 4E, 4F, 4G, you
should only proceed CUI far as is necessary to understand
the method. You are definitely not expected to work these
questions right OtJt using tables, A few only need be done
completely to ensure the correct procedure is thoroughly
understood,-]

Eqns using trig, identities 4D 2-3 l(a-u),2


sin 2 e+cos26 • 1 etc.
Eqns using 26, 36, t 4E 3 1-11
a • B results
STUDENTS' GUIDE
EXERCISES TO &E DONE BY

--
TOPIC SET PERIODS flH
GROUP fC) fA)
Trig. Eqns 4F 2-3 l(a-z), --
2(1),
3(1,11)
a cos8+ b sin8 • c 4G 3-4 1,2,5-8 10,11 12
Derivatives of cosec x, 4H 3 1-9 -- 10
sec x, cot x
Applications of Derivatives 4I 2-3 1-3,5,6 4,10 11
Trig Integrals (Theory+ 4J 3-4 1(1-v, l(vi) l(iii)
examples). omit 111) 10 2(vi)
Note B, C groups omit 2(1-v), 12(a) 12(b)
questions marked*) 3-9
Applications of Integration 4K 3 1,351,111) 2,3(11) --
4-10

CHAPTER 5
Definitions, basic ideas 5A 2-3 1-3,5,7-10 4,6 --
on Polynomials.
Sketchin g graphs of SB 2-3 1-8 9,10 --
Polynomial�
Division Transformation - 5C 2-3 1-4 5 --
Division of Polynomials
Further division trans- 5D 1 1-4 .._ --
formations
The Remainder and Factor 5E 2-3 1-6,6-17 7 --
Theorems
[This section will have been
treateq in Form 4 by
Advanced Level Students.]
Deductions from the Factor 5F 3-4 1-8 9,10 --
Theorem (Study of theory
and examples)
Roots and Coefficients of 5G 3 1,2,4-14 3,15,16 --
Polynomial Equations
Locating Roots of p(x)•O SH 1 1-3 -- --
Halving the Interval 51 2 l-3(1v) 3(v,vi) --
Newton 's Method 5J 4-5 l-4(1v), 4(v,vi) 9
(Theory·+ working of 5-8
examples)

CHAPTER 6
Pascal's Triangle, Use in 6A 3 �.2(a-j), 2(k,n), --
expansions of (l+x)n, 3,5-9, 4,10,
n small positive integer 11-14,16, 15,18
17
Expansions (a+x)n 6B 2-3 l(a-k),2, 12,16 15
3-7, 9(a)
10,13
STU�NTS' GUIDE
TOPIC
I I l I
The Pascal Triangle Relations, Induction proof for n Cr,
General Statement of the Binomial Theorem.
The results should be known but it is best to omit the
proofs with group C, and treat lightly with Group B
students.

Use of Binomial Theorem 6C 5 l-4(a) 4(a)


(including theory above, (i-iii),5, (iv),
and examples) 6(1,ii), 4(b),
7-11, 6(iii)
12(1-iv), 12(v­
13(1,ii) viii)
14-18 13(iii)
19-24
Applications of Binomial 6D 2 1,2,4,6, 5,8, 3
Theorem 7(i),9(i}, 9(11)
10 11

CHAPTER 7
Introduction - Basic Ideas, 7A 4 1-7(1),8 7(ii)
Definition of Probability

.
(Work through examples 1-6)
Exercises on the Definition
Algebra of Events - (Theory, 7B 4 1-5, 7-9, 6,12, 10
work through examples 1-8) 11,13,15, 14,17
P(AUB) Rule 16
Two Stage Experiments 7C 3-4 1-9,11-13 10,14,
(Theory; and work 15,16 17
examples 1-2)
Examples involving both 7D 5 1-3,4(1, 10-12, 4(ill)
laws (Work through ii), 5-9, 14,17, 16,
examples 1-4) 13,15, 21 22
18-20,
23-28
Counting Procedures - 7E 2-3 1-17,19 18
Ordered Selections
Ordered selections with 7F 1 1-6
repetitions
Unordered Selections 7G 2-3 1-13
Binomial Distribution 7R 2-3 1 -18
(Introduction - basic
ideas, results, work
through examples. Proof 2-3
·to be treated lightly
with Groupe C,B)
Binomial Distribution - 71 3 1-12 13
Exercises
Further binomial 7J 1-2 1-2 3,4
distribution types
STUDENTS' GUIDE

J TOPIC SET PERiqOS EXERCISES TO BE DONE BY


GROUP (C) (B) (A)
Random Digits, Expectation 7-K· 3 1-6,8-1() 7 --
(Theory+ examples;
E(X) • np)

APPENDIX
Definitions, types of plA 2 1-5 6 7
sequences, grapho i
Limit of a Sequence, A plB 3-4 1-6 7-10 11,12
discussion (bookwork
and definition not
required for examin-
ation),
Determining limits of
sequences

Note. The Revision Papers

Period allocations for the revision sets, topic by topic, have not
been attempted, as these sets can be worked through by you in your own
time, or as suggested by your teacher,
Specimen papers are available by the same author, and these may be
attempted under examination conditions or worked through by you, either
in your own time, or as directed by your teacher, These have not been
included in this text because of space considerations (11 Sets of
Paper C and 11 Sets of Paper B together occupy approximately lOO
pages, including answers),
It i3 obvious that you should do all the revision sets, and all the
specimen papers to be fully prepared for the Higher School Certificate
Examination in Mathematics.
Complete worked soZutions are available for these specimen papers,
CHAPTER

THE INVE RSE TRIGONO�ETRIC FUNCTIONS

I. I NTPOOUCT I 01�

Corresponding to each of the six trigonometric functions


y = sin x, y = cos x, y = tan x, y = sec x, y = cosec x, y = cot x,
there may be obtained the associated -,inverse trigonometric
function. !lefore proceeding with this point, we swnmarise
the idea of an inverse function. (This has been discussed in
Coroneos "A Higher School Certificate Course in Mathematics
Form 5 3 unit Course ", pages 45 7-459.)

For the function y = f(x), then on interchanging x, y in


the equation of this function, �e obtain the inverse function
x = f(y), {which may be,written as y = y(x)}, pr•ovided that
the range of y is appropriately selected.
(a) Thus, for the function. y = ex, /
the inver11e function is x = eY,
which may �e re-written (on /
taking natural logarithms) as
logex = y logee, i.e. y=logex.
The sketch of y = eX and of the
inverse function y = logex is
shown. +----*'-------�
)(
[fiote that (i) no restrictions
are necessary in this case on
the range of y since y=logex /
is a function, i.e. lo5 x is single-valued for each
possible value of x, for each x > 0.
(ii) a new notation (namely logex) had to be
employed to write the inverse function �f y = e x in the
form y = y(x).]
(b) However, for the function = x2 then the equation x = y2
y
(obtained by interchanging x, y in y = x2 ) represents a
relation, since for each x ) 0, there are two values
of y.
1
THE INVERSE SINE FUNCTION
In this case, we define the
domain of x in y = x 2 , and y
the range of y in x = y 2
(i.e. in y±/;) so that an
inverse function of y x2
is determined.
Thus for y = x 2 , and x .:'. o;
then the inverse function
is defined as y rx
(i.e. the range of y is re­
stricted to y.:'. 0).
Also for y x 2 , and x< O;
=
then the inverse function �
is defined as y -rx /
��><
(i.e. the range of y is re­
stricted to y< 0). The sketch illustrates these fact&

In each case (a), (b)·given above, the function y = f(x)


and the inverse function x = f(y), appropriately defined
1.Jhere necessary (in the form y = y{x) say), are represented
graphically as reflections of each other in the line y = x.

The notes above will now be applied to the inverse trig­


onometric functions, beginning with the inverse sine function.
2. THE INVERSE SiNE FUNCTION
fl. DEFINITION
For the function y = sin x, the equation determined by
inlerchanging the variables x, y is x = sin y. This is not a
function, since for any possible value of x, then y is multi­
valued, not single-valued; i.e. x = sin. y is a relation.
[For example, consider the case when x = \, i.e. when
sin y = \. In this case, there are many ahgles y whose sine
is \.]
Here we have two problems:
(i) To introduce a new notation, since if x = sin y, then
how is y to be expressed in terms of x,
(ii) To define a range of values of y such that the inverse
sine of x is a function, not a relation.
The answers to these two problems are respectively
(i) y = sin-1 X y < 2!.
(ii)
- 2
11
-2�
The reasons for these answers will now be investigated.
A, The reason for the choice of the notation y = sin-1 x to
denote the inverse trigonometric function of y = sin x is
allied to the use of a similar notation for all inverse
functions. Thus if y = f(x,y), then the inverse function
is usually denoted by f-1:(x,y). In this case, the
2
SKETCH OF x sin y
inverse function to y= sin x is denoted by y= sin-1 x.
(a) Do not confuse the notation sin-1 x with the
notation (sin x)-1 � 1/sin x= cosec x.
(b) In some books, instead of sin- 1 x, the notation
arc sin x is -used. It stands

Q ..
for the length of an arc of a
circle of unit radius, cut off e,s"'x.
by 2 radii which include an I
angle whose sine is x.- A
In this case, *arc AB r6 where r J. unit,
a sin-Ix,
and *arc AB

B. The reason for the choice of the range - i y f


will if
be seen clearly on examination of the graph of the in­
verse relation x= sin y. The sketch of the inverse
func�ion (y= sin-1 x) to y= sin x is developed in
3 steps. {Step 1 gives the sketch of the function
y = sin x; step 2 gives the sketch of the inverse
relation x= sin y; step 3 gives the sketch of
y= sin-1 x.}
Step 1. Sketch of y = sin x.

_,., ·11 ZIT HI •


•I -�
Step 2. Sketch of x sin y.
(This is the curve y= sinx
set along the y axis in­
stead of along the x axis.
The curve x= sin y is the
reflection of the curve
y= sin x in the line
y= x.)
The multiplicity of values
•I< " for y such that si.n y= x
is illustrated in the case
when x= -\.

When x= �. i.e. when


sin y= Yi, 1111 -711
then y= ..., - ,
-6- -6-
11 511 1311
-� 6' 6' -6-
,

3
SKETCH OF y = sin-lx
The section of the curve x = Sin y
taken by convention for the inverse
function denoted by y = sin-1 x is in-
dicated. � �s a ge is the �losed
interval f
; ;J :

This· is the
section taken
by convention
for y = sin-lx
{Note that
(i)
2
� f,
when .::..!!. -
< y then for any value x such that
-1 i X S 1, there is one and on one value of
.
(ii) other possible ranges could be B
Tl )1!1
f-)TT -TT
,or J
2, 2 -2- 2
etc. In each case, for each x in the domain !xi i 1, a
unique value of y is determined.

11)(
(iii) a range such as [0,TT) is unsuitable since
for O � x < 1, then there are 2 values of
y, and for -1 ix< O, there are no
values of y, }
Step 3. Sketch of y = sin-1 x
y
The section of x = sin y, such that
-TT Tl
+"!r

z is described by the new


2
2�Ys
notation y sin-1 x.
Thus y si-n-1 x is the inverse -X
funct;j.on to y = sin x, and its graph is -I
�..__--.,+,""'----+-�
For y = sin -1 x,
I )(.
shown in the sketch.
the domain is given by -1 � x � 1 and
the range is given by -; 5: sin-1x � f.
In other worda, the only replace- -i
ment values of x possible for the -Y
function y = sin-1 x are real numbers from -1 to +1, both
inclusive; and the possible values of y are restricted to
real nwnbers in the range -; to + ;, both inclusive.

B. POINTS TO NOTE
Nill1ERICAL INVERSE SINES
2!.
(a)
From the graph, sin-1(�) *L'1 *OA
6
4
RELATION BETWEEN y = sjn-lx AND x = sin y
and sin -l (-�) c *RS "' *OB = -; _____ 'I � __ _

Similarly, sin-l <fz) .. *OC .!!.


4
C

and sin - l (-1)


-11
,/7}_
*OD =
4 ·x )L

From the sketch


sin- i (x) *PQ *OF ----- - -wii- - - - - -
and sin-1 (-x) = *GH *OJ= -*OF

Hence sin-1(-x) �---


1 /3
- sin- ( 2)
-11 -)(

=3
and sin-1(-1) = 1
- sin- (1) 1

- - -- - -
. .,
.,Y�

The special values sin""1 (0), sin-l (±1), sin-l (±�),


sin-1 (± sin- l ( t
ii>, 1> and certain others ( combinations
of these) may be determined exaatly, without reference to
tables. However, in other cases, approximate values may be
calculated by the use of sine tables followed by radian
tables.
For example, sin-1 (1) � sin-1(·6667)
3
{f 41 ° 49' from sine tables}
·7298 from radian tables
and sin-1(-·2189) =-sin-1 (·2189)
{f -12 ° 39'} -·2208
i.e. sin-1 (1) f ·7298 and sin-1(-·2189) = -·2208
[It would be definitely incorrect to say sin-1(1) 41 ° 49' or
3
that sin-1(-·2189) � -12 ° 39'.J

(b) RELATION BETWEEN x = sin y and y = sin-1 x


We have already stated that x = sin y and y = sin-1x are
synonymous, except in so far as the first (i.e. x = sin y) is
multi-valued, and the second (i.e. y = sin-1 x) is single­
valued for each possible x.
If y = sin- 1 x, then it foU01,1s irrmediately that x = sin y.
[Thus fa sin-1(1) and as is obvious f = sin f.J
However, if x = sin y, then y = sin-1 x follOIJs if and
5
GENERAL SOLUTION OF x sin y

r
d -11 < < TT
only if -1 :5 x _< 1 an
2 - y 5_ 2, i. es• if I x I _< 1 and
I sin-1xl s [Thus f= sin ;, but ;; sin-1 sin ce(f),
- <
; S sin -1(),) - TT · ]
2
Hence, the result y = sin-1 Y (where lxl � II can
always be interpreted as x = sin y, but the result
x = sin y is only identical to y= sin-lx, provided

-
interval [ ;, ;J.
that y (i.e. sin-1 xi is restricted to the closed

*(c) SOLUTION OF THE EQUATION x = sin y, lxl� 1


Consider firstlu the case where
0 :5 x 5 1, say x = \.
Here if l, = sin y, there are many
values possible for y. The positive )(

values of y (measured i n radians)


are given by
TT 511 TT
y = 6' 6' (211 + 6), (211 + 22!.)
6 '
4 TT 511
( 11 + 6); (411 + 6),
The negative values of y are given ')(
-71r -llrr ]]!_
by y = 6 ' 6 ' -(2 TT + 6 )' -(2TT- �IT)= - !1!,.
6
l111 711 4 llrr
-(211 + --), 6 -(411 + 6), ( 11 + -6 )' ...
- - ..
'That is, the values of y, where sin y = !,, are given by
-1911 - -1111 -711 511 -
TT 6' 1311 - 1711
-
y = •••• , -- 6 , -6 , -6 - , .6' -,
6 6 , . ... ••• •••• ( 1)
-1
Consider secondly the case where -1 S x < O, say x = 7z•
-1
Here if f2 = si n y, again y may take a multiplicity of values.
These values (in radians) are given by
TT TT 511 711
y = (11 + 7;)' (211 - 7;)' (211 + 4)' (211 + 4)'... if y > 0
-TT -311 TT 311
and y = 4 , 4 , -(211 + ), -(211 + 4), if y < 0
4
-1111 -911 -311 -11 511 l.!!.. 13"
i.e. Y · • ·' --4-, -4-, -4-, · 4' 4' -4-, • • • · · •• '· •• (2)

The genqral solution of the equation sin y = x, where


lxl i 1, is given by y = n11 + (-l)n sin-Ix, where n is in­
tegral.
For example, if sin y \, then y n11 + (-l)n sin-1(�)
n11 + (-l)
n
<f>.
Thus for n -3,-2,-l,O,l,2,3,•..• ,then the correspond-
ing values of y are give n by

6
ALTERNATIVE DEFINITION OF THE INVERSE SINE
y ... ,

i.e. y ... • 7T
- 37! - - n
7!,
6'' Z +7T6'
7T
7T
-7! - 6
7T
' 6'
7T - 6
'
Z n + 6' 3n - 6'
i.e. y ... • -19n -lln -77! 2!. 5n 13 n 17n
6 • 6 • 6 • 6' 6' 6 • 6 •
{See equation (1) above}

Simil<a>ly, if sin y =�.then the general solution for


1
y is y = nn+(-1)n sin- l ( - ) = nn+(-1)n (.::ll.) for integral n.
/2" 4
Check that these solutions give the results in equa.tion (2)
above.

*(d) ALTERNATIVE DEFINITION OF THE INVERSE SINE


Of all the solutions for y of sin y = x, given lxl � 1,
there will be one and only one nwnerically smallest value of
y; i.e. such that its absolute value is less than the abso­
lute value of any other solution.

(1) above to be
equation (2).
f. and for sin y = ?t,
In the case of sin y = \, this can be seen fran equation
this will be
These numerically smallest values of
-z,
y
from
are
called the pri�cipal values of y, and the notations sin- 1(\),
sin-1( �) are reserved exclusively for these special values.
1 -1 -
l
Thus sin-(�)= 7T
and sin- l ( ) = 7!
6 12 '""""i;·

Thus if sin y = x (�1he,e y is an ang I e measured in


radians, and x is a variable real number such that
-I� x .$ I, i.e. lxl .s ll, then for each given
value of y, there is a wzique value of x, but for
each possible x, then y is multi-valued.
However, if 11e use the notation y = sin- 1 (::c), then
we mean that y is a particular angle whose sine is
x, and this particular angle, the numerically
smallest value of y, is cal led the principal value
of y. Thus for each x, then y is single-valued,
i.e. y = sin- 1 x is a function.

(e) 1
Since sin-(-x) = -sin- 1 (�). sin- 1 (x) is an odd
function.
[An odd function f (x) is defined as one in which (-x)=-f
f (x);
an even function is one in wnich f (-x) = f (x). Some examples
of odd functions are x , x 3 . sin2x, tan x; whilst some even
functions are x 2 , x 4, cos x, ex .]
7
DERIVATIVE OF sin- X

_g_. DERIVATIVE OF sin-1 x


Let y = sin-1 x, by above
.�

11/l
x =sin y.
Differentiating with respect to y,
dx
= cos y.
dy dx
Using the fact that ..!h'.. =1 I
.x
dx dy
.fil'..=_l_
dx cos y
Now since cos 2y + sin2 y =1,
cosy t 11-sin2y
-�
±ll-x 2 , because X = sin y.
llowever, from the sketch ofy =sin-lx, (except at x=±l,
where the tangent is vertic'al), the tangent to the curve
makes an acute angle with the x axis, i.e.
.fil'.. 1 1
f
> O.
Thus = = t ak ing
· cos y = +� only.
dx cosy +h-x2•

I
i.e. ---9.c sin- l x) =
dx �

{Note (i) the domain of the derivative of sin-1 x, i.e. of


is -1 < x < 1 and its range � l,' since
/l:x2•
1
�- . > 1

(ii) that !(sin-1 x)yields a function entirely lllllike


d
the original function; this is not unusual, as
d 1
witness {or example (loge x)
dx
b dx =sin-I
X

(iii) J X + C,J

Further work on the inverse sine function and its


derivative wi I I be deferred unti I after discussion
of the other two inverse trigonometric functions.

2. THE IMVERSE COSINE

1, DEFINITION
The discussion of the inverse function to y =cos x fol­
lows the same pattern as that of y =sin-1 x.
For the function y =cos x, where -1 S y S l, the inverse
relation is x =cos y. A convention for the range of y in
x =cosy leads to the definition y =cos-1 x, (also called
arc cos x), as the ·invers� trigonometric function to y =cosx,

8
y = cos- 1 x
This range is best understood by sketching the curve re­
presented by the equation x = cos ·Y·
Step 1.

•I.

•I
·y
Step 2. Sketch of x = cosy
This is the curve y = cos x
set along the ; axis instead of
a·long the x axis. It is the re­
flection of the curve y = cos x
in the line y = x,
�-c--��---::--���-:;;�
By convention, the range of
y taken so that for any value of
x in the domain -lSxSl, there is
one and only one value of y, is
0 S y S n , i.e. 0 S -cos-lx � n,

Note that
(i) although other closed in- .. -x -.,..t- -f>•
tervals such.as [n, 2n], - -
[-n, O], [-2n,-n] etc. lead
to uniqueness of y for each
value of x, the interval
adopted by convention as the range is (0, n].

(ii) the range_[-;. f] selected for y =


is unsuitable here for y = cos-1 x.
See sketch.
n----.
Step 3. Sketch of y = cos-1x
The sketch o.f y = cos-1x is
shown. It consists of the sec­
tion of x = cosy taken for
O.:':y.S n,
For cos-lx, the domain of x
is lxl :: 1 and the range is ...
given by O .:S cos-lx � n. +-.�,�---....,.t-----""!;"--.

B. POINTS TO NOTE
(a) !-HJHF:RICAL HiVERSE COS INES
From the graph, cos-l (!,) *I..:·!
*OA
= 2!.
9
RELATION BETWEEN y cos- 1x AND x cosy
and cos-1(-\) *RS O
* ll
*OC - *BC
O
* C- *OA (since *OA B
* C, by symmetry)
TT - TT 2TT
3 3

=TT

From the sketch, II

cos-1(x) = *DE *OJ


and cos-1(-x)= *FG O
* H = O * C- H * C
*OC - O
* J, since *HC=*OJ,
by symmetry

Thus cos-1(-x) = TT - cos-1 x

-Ii =TT - cos-1( IJ) TT -


For example, cos-1(-)
TT 511
- =-·
2 2 6 6
and cos-1(-1) =TT - cos-1(1) TT - 0 = TT
-1 =cos-1
Also cos-1(_l) - cos- l (-)
12 12
- [TT - <-;}> cos-1 <-;}>]
TT
= 4 - [TT - =2
-TT
f]
The special values cos-1(0), cos-1(±1), cos-1(±f),
1
cos- (± -;}>, cos-1(± 1> and certain combinations of these
may be determined exactly, without reference to tables.
However, in other cases, approximate values may be
calculated by the use of cosine tables followed by radian
taules.
For example, cos-1(%) cos-1(·25) � {75 ° 31'} � 1·3180
1
and cos- (-·6789) TT - cos-1(·6789) H 180 ° - 47 ° 15'}
=3·1416 - ·8247 =2•3169
1
1. . e . cos -1(· ) = 1·3180 and cos-1(-·6789) =2·3169
4
(b) RELATION BETWEEN x =cosy andy =cos-1x
Now x =cos y and y =cos-1x are synonymous, except in
so far that x =cos y is multi-valued and y =cos-1 x• is
single-valued for each possible value of x.
If y = cos- 1x, then it foU()l,)sthat x = cosy
[Thus 3 = cos-1(�), and as is obvious cos { = f.J
ll01..1ever, if x = cos y, then y = i:?os- 1x foZlows if and
on Zy if -1 S x s 1 and O s y S 11, i.e. if Ix I .:: 1 and
O .S cos-1x S TT. [Thus 1 =cos Sn but 21!. I cos-1(1) , since
2 3' 3 2
OS cos-1 <f> .S TT.]
10
SOLUTION OF x =cosy
Hence the result y = cos -Ix, where !xi i I,
ca n always be interpreted a s x =cosy, but the re­
sul t x =cosy· is only identica l to y = cos-Ix
provided tha t y (i.e. cos-Ix) is restricted to the
closed interval [0,11).

*(c) SOLUTION OF THE EQUATION x =cosy, jxj S 1


Consider firstly thP case when OS x s 1, say x =!,.
l-n1en cos y �. the values of y are given by
11 11 TT Sn 11 Sn
y =3' (211 - 3), (211 + 3), (2 11 + -j), (411 - 3), (411 + 3), ..
if y > 0
5 11 .11 Sn
and y -(211 - ;), -(211 - 3), -(211 + 3), -(211 + 3), ....
if y < 0
lh -711 -511 -TT TT Sn 711 1111 1 311
i.e. y = ••• , -3-, -3-, -3-, 3' l: 3' 3' --3, -3-, ••••(l)

Consider secoi1dly the case when -1 :, x < O, say x = -'2.


l�b.en cos y =- \;, the values of y are given by
TT TT 2TT 411 2TT
y = (11 - 3 ), (11 + 3), (211 + 3), (211 + 3), (411 + 3) .....
'.
if y > 0
411 211 211 411
and y = -(211 - 3), -(211 - 3), -(211 + 3), -(211 + 3 ), ....
if y < 0
-1011 -811 -411 -211 211 411 811 1011
i.e. y , ,
--3- -3- -3- . 3 , ' 3' 3' -3 -, ...... <2>
_J
The general solution of the equation cos y = x, where
!xi � 1, is given by y = 2nn t cos-I(x), for integral n.
For example, if cos y F '2, then y = 2nn t cos-1(�)=2nn ti
Thus for n = .•••, -2, -1, O, 1, 2, .•.
t hen y TT t TT ' 2 TT �2!.
TT -2 TT t 3'
.•• , -4TT t 3' 3' 4 TT t )'
TT

3
-1311 -1111 -711 -Sn -11 11 Sn 711 1111 1311
i.e. Y •• ·' 3 ' --3-, -3-, -3-, 3' 3' 3' 3' -3-, -3-···
{See equation (1) above.}

Similarly if cos y a-�,


TT
then y = 2nn t cos-1(-�) = 2nn t (11 - 3), for integral n.
Check that these solutions give the results in equation (2)
above.
*(d) ALTERNATE DEFINITIONOF THE INVERSE COSINE
In the case of the inverse sine, an alternative defin­
ition employed the idea of the nWlerically smallest value
(the principal value) of y such that x • sin y, for a given x
in the domain Ix! S 1. This particular value of y was denoted
i>y sin-Ix.
This idea is not so applicable in the case of the in­
verse cosine; for example when cos y=�. there are two
11
DERIVATIVE OF cos-lx
nwnerioaZZy smaZZest values of y, namely
-71 and +11 {see
3 3
equation (1) above}; and when cos y =-!,;, again there are
-211 +2rr
tw0 numerio�ZZy smaZZest va Zues o� y, na�ely � - and � - {see
equation (2) above}. 3 3

Fer this idea to �e applicable, the positive value of y


2
is taken for cos -lx. _T .:us cos - i (!2) = and cos-1 (-!,;) = ;.
j
S2• DERIVATIVE OF cos-1x
Let y =cos-1 x, X = COS y
. dx
T nus =- sin y
dy
and hence .QY = ,- �
dx sin y
Now since cos 2 y + sin2y 1,
sin y ± 11 cos2y
±ri-:-7,

since x =cos y -��'--......�-+>�0t-��+->��


However, from the sketch of y =cos-lx (except at x ±1,
where the tangent is vertical), the tangent to the curve
makes an obtuse angle with the x axis, i.e.�< O.
Thus 1; =- s�n y =- +/�-xZ' taking sin y +�.

i.e.

{,'late (i) the domain of the derivative of cos-1 x, i.e. of


l < 1,
is -1 and its range !: -1, since
- 7'i?°' < X

1 s -1
- 7i-'.'?
(ii) that --5!.(cos- lx) yields a function entirely un],H.e
dx
l
the original function; namely - �
(iii) --5!.(cos-1x) - __l(sin- 1 x)
dx dx

(iv) f ( � 2 dx = sin-1x + C1 =- cos-lx + c 2 •


l x
The latter result is the one employed in the later
work on simple iiarmonic motion.}

3. THE INVERSE TANGENT


A• D[ F I N I TION
The discussion of the inverse function to y tan X
follows the same pattern as that of y =tan-1x.
12
THE INVERSE TANGENT
For the function y = tan x, Yhere -oo < y < oo, the in­
verse relation is x = tan y. A convention for the range of y
in x = tan y leads to the definition y = tan-• x, (also called
arc tan x), as the inverse trigonometric function to y = tanx.
This range is best-understovd by sketching the curve repre­
sended by the equation x =·tan y
.
�- Sketch of

I
I
� X

of X =
This is the curve
y = tan x set along
the y axis instead of
along -the x axis . It -
is the reflection of
the curve y = tan x in
the line y = x.
Ily convention, -
the range of y taken
so that for any·value
of x in the domain���---�-'""?��� This is the
< x < oo, there is -X section taken
one and only one value L>y convention
as y = tan-lx
of y, is
-11
-2 <
y <
11
2
_rr
z:_ - - -
i e -lT < tan-1x < 2!.
. . 2 2

{Note that although - - - -


ot'.1er open intervals
11
such as <2• 11!.).
2 ,
12!. - Jr, -11
(11!. ) (
2' 2 ' 2 ' 2 )
lead to a uniqueness of y
for each value of x, the interval
• -11 11
adopted by convention as the ranli,e is ( , )
2 2}

13
SKETCH OF y = tan-Ix
Sketch of y = tc:n-1x

TI1e sketch of y = tan-1x -- --- -- - -


is shown. It consists of the
section of x = tan y taken
-11 < 11
for 2 Y < 2·
the iomain of x is -oo < x < oo
(i.e. all real x), and the
range is given by

f,
·Y
- 1 1 11
; < tan- x < i.e. ltan- xl <-
2

B. POINTS TO NOTE
(a) ;�L'MERICAL INVERSE TANGENTS

From the graph,


11
*PQ *CA = -
3
and
tan-1 (-13) = *RS *OB -*OA
- 11
=3
From the sketch,

tan-l (-x) *UI = *OD


- *OC by symmetry
1
- tan- x

-1
F o r example, tan- 1(-) tan-1 (--1) -11
13 13 6
-11
and tan- 1(-1) - tan- i (1)
·';I
4
1
The special values tan- 1(±/J), tan-1 (±
), tan-1 (± 1),
13
tan-1 (C) and certain combinations of these may be determined
exaatZy , without the use of tables.

However, in other cases, approximate values may be cal-


culated by the use of tangent tables followed by radian
tables,
-3) = l 3
For example, tan- l ('""s - tan- ( ) tan-1 (·6)
5
{� 30 ° 58'} � - ·5405

(b) RELATION i3ETWEEN x = tan y and y = tan- 1x

Now x =
tan y is multi-valued and y = tan-1x is single­
valued for each po·ssiblc X,
14
SOLUTION OF x = tan y
If y = tan-1x, it foU(Jl,)s that x = tan y
11 11
[Thus
3
= tan-1(/3-), and as is obvious tan( )
3
= C J
,3
.
H(Jl,)eVer if x = tan y, then y = tan-1x follows .if and
-11 -11 -1 11
on Zy -i'f
2 < y < 2, 1.. e . if 2 < tan x < 2.
1T •

4 4
[Thus 13 = tan ;, but ; -1- tan-1 (/3), since -; < tan-: 1 (/J)<fl

Hence the result y = tan-lx, for al I real x,


can always be interpreted as x = tan y, but the re­
sult x = tan y is only identical to y = tan -lx
provided that y (i.e. tan-1xl is restricted to the
-11 11 1
open interval 12, 2

*(c) SOLUTION OF THE EQUATION x = tan .y


h'hen x ;: O, say x = 13, then the roots of tan y f3 are
11 11 11 4 11 .
y = , (rr + ), (211 + ), (211 + ), ... if y > Q
3 3 3 3
and y
411 TT
-(211 - -), -(211 - -), -(211 + - 211 ), -(211 + �)
ell
3 3 3 3
-1111 -811 -511 -211 TT 411 711 if y < o
i .e . y . .. ' --3-, --3 , --3 , -3-, 3' 3' 3' .....(5)

h'hen x < O, say x = -13, then the roots of tan y = -13 are
y =
TT
3
TT
), (211 - ), (211 +
(rr -
3
211
3
), (211 +
5
if y > o ;>, . . .
411
and y -(211 - 2!.) ' -(211 - Zn), -(211 + 1!.) -(211 + -),
3 . 3 3 ' 3 if y < 0
-!Orr -711 -411 -11 211 511 811
i.e. y . . ., --3-, -3-, --3 , -:I·
3• 3• 3' ...
••••• (6)

The general solution of the equation x = tan y, for all


real x, is given by y = n11 + tan-1x, for integral n,
For example, if tan y = /3,
then y = n11 + tan-1(/J) = n11 + {, for integral n,
Hence for n ••• , -2,-1,0,l,2, •••
11 11 11 11 11
y ••• , -211 + 3' -11 + 3' 3' 11 + 211 + '
3' 3
Y -511 -211 11 411 711
···• --, --, , 3,3, {See equation (5) above}
3 3 3

Verify that if tan y = -/3, then y = n11 - {, and that the


solutions check with the results of equat-ion (6) above.

*(d) ALTERNATIVE DEFINITION OF TEE INVERSE TANGENT


The nl4Tleriaally smallest value of y such that x = tan y,
15
DERIVATIVE OF y = tan-lx

is called the prinaipaZ value of y, and denoted by tan- lx.


Thus for the cases tan y "' fi and tan y .. -fi, the re-

3, 3;
11 -71
spective smallest nULter ical values of y are see
equations (S), (6) above. i.e. tan-1 (/J) =3 and tan-l(-r'3)•1

(e) Since tan-1(-x) = - tan-1(x), then tan-lx is an odd


function of x.

£. DERIVATIVE OF tan-lx If
'II/�
Let y = tan-1x, ••• x = tan y

dx d 1 ·+��- ....,.....::::"-;�----===-c::3��·
Thus
dy
= sec 2 y, and hence .!!Y - -,-,..,..­
dx - sec"y
Now since sec 2y 1 + tan 2 y, ·,Y·
··\I
sec y1 + x 2 , since.-�x�=--'t�a_n_,,_
2
y_.��-�-----.
Thus
.!!I, _ 1 _ 1
dx - sec2y - l+x2
I.e. d(t an
I .
· -1x ) _

(Note (i) that there is no restriction on the domain of x for


-
I
T+x2"
dx

the derivative of tan-1x, i.e. of � 2, and that


1
its range is· determined by O <
X 1
2 .S 1.
J.+x
1 >
S.
. ince O, the tangent at all points on
J+x2
y = tan-1x makes an acute angle with the x axis.
1
(ii) that _.Q.(tan-1x) = 1' 'ch 1·s a function entire-
dx l+x2' �.u
ly unlike the original function.

(iii) f 1!�2
= tan-lx + c.)

SlJ,1MARY
LJOMAIN DERIVArlVE AND OTHER
FIJNCTION & SKETCH AND RANGE l'HEGRAL POINTS
- -I .s X .s I _d(s in-1x) = ...,,b,-,
i.e. !xi .S I X
d '11-x-
lxl< I
sin-1 x+C
·sin-1(-xl
- sin x
-I .s X .s d(cos-1x> = �.
i.e. lxl s dX 1"1-x-
lx I< I

-.x
f 1� 1 x-2 = -cos-
=

s in-l x+C2
1x+C1

cos -1 (-x)
-I 0 =11-c s-1x
16
NUMERICAL EXERCISES
DOMAIN DERIVATIVE AND OTHER
FUNCTION & SKETCH ANO RANGE INTEGRAL PO�NTS
y
(tan- x
< X < 00 d I I

dx ) = l+x2
- 2!. <tan -Ix<�
2 2
ie Jtan-IxJ <¥-J dx
_
�= tan Ix+C
tan-I(-x)
=-tan-Ix

APPL! CATIONS A. NUMERICAL EVALUATIONS


EXAMPLE
( i) If a sin-ic.:...'.·) - cos-1(.:..!.)
2 12
find a withou+ tables.
(ii) Determine without tables (a"l cos {sin-1<-�f>}
(bl tan-1( 12 cos l.!!.i (cl sin(2 tan-1.t>
4
\fethod
(i) 1'7oting thc>.t sin-1(- x) = -sin-1(x), cos-I(-x) =11-cos-I(x),
tan-1(-x) = -tan-1(x)
then a= {- sin-l(t)} - {11 - cos-1(-;fz')} - {-tan-1(/3)}
-71
{- .E.} - {11 - .:!!.} - {- .:!!.} = 1 on sim�lification
6 4 3 12
('') ().. { . -1(-40)}
1.1. a [•,CW COS Sl.n
41

Let a

sin a

cos(-a)
9
cos a = from t'.1e sketch.
41'

(b) tan-1(12. �),


1
711 11
noting cos = cos(211 - )
4 4
cos 411 = l
12
tan-1(1) = .!!.
4

17
NUMERICAL EXERCISES
DOMAIN DERIVATIVE ANO OTHER
FUNCTION & SKETCH INTEGRAL PO�NTS
AND RANGE

-tan�-1x - -oo < < oo d I


-(tan-1x) = -2

V
�y
dx l+x
X

- .! <tan-1x <:!!..
2 2

17
1
ie ltan- xl<f
o dx -1
I �= tan x+C
x
tan-1(-x)
- - -�11- - =-tan-1x

APPLICATIONS A. NUMERICAL EVALUATl(XIIS

EXAMPLE
( i) If a sin-ic.:...'.-) - cos-1(.:...!..l
2 12
find a withou� tables.
(ii) Determine without tables (a") cos {sin-1(-��)}
(bl tan-1(/2 cos l!!..i (c) sin(2 tan - 1 t>
4
,fethod
(i) �7oting that sin-1 (-x) = -sin-1(x), cos-1(-x) =11-cos -1 (x),
tan-1(-x) = -tan-1(x)
then a = {- sin-1(-f)} - {11 - cos- 1(�)} - {-tan-1(/3)}
-
{- .I!.} - {11 - .!!.} - {- .!!.} = ?r. on sim�lification
6 4 3 12
('') a ,.
1.1. () . -1(-40)}
c,c,w cos { sin
41
Let a

sin a

cos(-a)
cos a =
4
i, from t:1e sketch.

tan-1(,'2 . ),
(b)
/�
711
noting cos
4
= cos(211 - .!!.)
4
cos 4TT = 1
12
tan-1 (1) = .!!.
4

17
EXERCISES SET lA

(ii) lj, = 3

(iii) n = 2

5. Evaluate, without tables,


(i) sin(sin- l a>;
3 sin {sin <-:;> };
-1 -5
sin

(iii) sin(cos-1 0·6); cos (tan-1 �); sin


1

*(iv) sin (2 cos-1 1>;


sin (2 sin-1 0·28);
cos (2 sin-1 {>;
cos {2 cos-1(0·4)}
*(v) -1)};
tan {2 tan- l 2 }; tan {2 tan- 1 (�
5
1
tan {2 sin- 0•6}; sin {2 tilll-l }}
ti. (i) Determine the first two positive and first two
1
negative values of y such that sin y = �- (Give
answers in radians.) Which of these is the value
1
of sin- (4)?
1
(ii)�a,8,y,6 are the roots of the tan x =
equation
T3
for O � x � 4w, and a> 8> y> 6. Which of these
is tan-I <-7t)? Find the value of I tan-�) - yj.
(iii) If 81, 82 are the solutions of cos 0 "' . ; in the
closed interval defined by jej � w, determine the
value of { (8 1+ 82) - cos-1(�)}.
7. (i) State the values of a (say a 1, a2) such that
sin a = -\ and a is in the open interval -2w<a<O.
Hence calculate the value of {a1 +a2+sin-1 (-�)}.
' -13
(ii) Find the roots of the equation cos 8 = � - for
2
-2w .5 8 .5 2w." What is the value of
-13"
(a) cos- l (- -) -13
(b) w - cos- l <-2->?
2
(iii) Determine the greatest value x1 and the least
value x2 such t at tan2 x = , w �­
Evaluate {tan-1 (1) - tan-1(-1) + x1 - x2}.

19
DIFFERENTIATION
APPLICATIONS OF THE ABOVE RESULTS: g,,
DIFFERENTIATION

EXAMPLE I. Find the equation ------- 'IYl 'i ______ _


of the tangent and of the
normal to y = sin• 1 x at the
point v,here x = -!,.. "
'-----------------' -�":.._____,,_.;..._,��::::......�...i.,.��
;'.!ethod
Now since for y = f(x) = sin-1x,
then f= f' (x) = - �x2,.
11
the
3radient of the tangent where
x = -\, (i.e. at A) is f'(-�).

J1
1
-
The value of f I (-�) = { 1 !,)2 = =
2
l-( - 73
Also, when x = -�,
the ordinate of A = f(-�) = sin-1(-�)
= -sin-1(!,!}
since sin-1(-x)=-sin-l(x)
= .::.!!.

Thus the equation of the tangent at A is given by


11 2 1
y + = 13(x + ). and the equation of the normal at A is
6 2
11 13 1
given by y + = - (x + ). These equations may be simpli-
6 �2� 2
fied, if desired, to give 12x - 6/3y + 6 - 1311 = 0 and
6/3x + 12y + 3/3 + Z11 = 0 respectively.

EXAMPLE 2. Derivative of sin-1Caxl, cos-1(ax),


tan-l(axl where a is constant.

.'4ethod. To find t (sin-1ax), laxl < 1

Let y = sin-1ax, and put u = ax


du
y = sin-1 u = a
dx
fil'.. = fil'.. , du = 1 a,
Now
dx du dx �
i.e. � ( si. n -1ax) - 1
dx - li-(ax)2' a
Similarly, show that
1
�( cos-1ax) - - 1 2 a,. -
dx {l-(ax) ' d!(tan lax) c
l+(ax) 2 • a;
Thus for example
(i) ;[sin-1(Sx) J 1 d
( x)
11-(5x)2 dx S 7f='rrir2
(ii) �[cos -I (eX ) J = -1 d X
-(e ) -eX
dx il-(exP dx �
(iii) �[tan-1(x 3 ) J 1 �(x 3 ) 3x 2
d l+(x 3)2 dx �
20
EXAMPLES ON DERIVATIVES

EXAMPLE 3. F·1nd Ql. ·1n each of the fol low·1ng cases


dx
-
( i) y · -le Ixll
s In c·i .1) y e tan 13x
x7
Ciii) y = log(cos-12x) Ci v) y

(v) y

Method
(i) When y =
.4Y_
dx -

.-£(tan- l Jx)
dx
_l__ _£(3x)_ etan-13x/(1+9x 2 )
l+()x)2 • dx -3 /

x7
( iii) When y = log( ),
COS -lz X
we first apply the logarithm laws,
i.e. y log x 7 - log(cos-12x)
i.e. y 7 log x - log(cos-12x)
.4Y = 1 l d J
Thus
dx 7.; - cos-lzx . dx (cos- 2x)
= l - 1 ( -1 . 2) = l + --=,-,:-"'2--==r-
x cos-12x ll-4x' x cos-12x. •l-4x'

(iv)

(cos- x)

(v) y = x3 sin-1(x 2 )
.4Y - x 3 �(sin- 1 x 2 ) + sin-1 x 2 • �(x 3 ), using result
dx - d d
d'
x3 2x + sin- 1 x 2 .(3x 2 ) for (uv)
•l-x� •
� dx

= x2 (71 ��� + 3 sin-lx 2 J

21
FURTHER EXAMPLES ON DIFFERENTIATION
(Note it is implied in all exercises en differentiation etc,
that the domain of x is such that the operation of differen­
tiation is valid. Thus when we write sin-lx, we imply
-1 S. x S. l; thus if we write cos-17x, then we imply
1 implied
-1 � 7x � 1. Similarly for d!(sin- x) = 11:x2• it is
that -1 < x < 1, Le. Jxl < l;
whilst for ....!!(cos-1 !)
., 1/i- , (::-,:--l), it is implied that
x
dX X l _ !

l�I
x2
< 1, i.e. Jxl >l i.e. x > l OZ' x < -1.]

EXAMPLE 4.
If f(x) tan-1( log x) and g(x) log(tan- lx),
evaluate f' (el . g 1 (-1 >

Method. Hhen f(x) tan-1 (1og x), f I (X)


1 !
l+(log x)2 X
1 1
and when g(x) log(tan-lx), g I (X)
tan-ix l+x2
1 .l _ _l .l -1
Hence f' (e)
l+(log e)2 e =1+1 2 ' e = 2e
(Note log e "' 1)
1 1 1 1 "' -2
and g I (-1) tan-• (-1) l+(-lt = -rr/4 · 2 rr
1 -2 "' -1
Thus f' (e) g '( -1) =- -
2e • Tl err

EXAMPLE 5. Prove that the derivatives of


y = tan-1(sec x + tan x) and z = tan-1 (tan �)
2
are
11
equa I, and hence that y - z =
4'
l,fetJ:od.
1
X ow _ild =
d
d (sec x + tan x)
dx l+(sec x + tan x) 2 x
1
{sec x tan x
1 + Sec2x + 2 sec X tan x + tan2x • + sec2x},
-1
(cos x)2 • - sin x
., _l _ • � sin x = sec x tan x. J
cos x
EI· - 1 on simplification, noting sec i x "' l+tanl x
dx - 2
Also:!:=
1
l+(tan x/2)2 dx
1
d X _ 1
(tan 2> = _ _ + t-=a=-n-2 _x_/2_
l
sec2 rt
=2 on simplification.

Thus EI '" dz and • ....!!(y - z) =O


dx dx" • • dx
22
EXERCISES SET lB
Henae y - z = a constant C, since d (C) o.
dx
i.e. tan-l(sec x + tan x) - tan-1(tan t>
C ••••••••••••• (M)
To determine C, substitute an appropriate value of x in
(M).
Here let x = D, • •• tan-1(sec O + tan O) - tan-1(tan 0) C
i.e. tan-1(1) - tan-1(0) = C, i.e. C = o = f- f
i.e. tan-l(sec x + tan x) - tan-1(tan {> = {

EXAMPLE 6. Sketch the graph of Ci) y sin(sin-lx)


(ii) y = tan(tan-1x) *(iii) y sec(sec-1x)
State any restrictions necessary on x in each case.
y
(i) Let ex = sin-1x, where -1 S. x .$ 1
• •. sin ex = x
Thus y = sin(sin-1x) = sin ex= x
-X X
i.e. sin(sin-lx) = x1 provided -lS.x.Sl
In cases (ii), (iii) prove that tan(tan-lx) = x, for all
real x and that sec(sec-1x) = x, for lxl � 1.
'I

� •Sc.c.(5ec 1 x)
,y/
·-:
-x X -)( \;"'" 0 X

-Y / _,-Y
EXERCISES SET lB EXERCISES ON DIFFERENTIATION
1. Find� in each of the following cases.
(i) y=sin-13x; y=sin-1}; y=sin-1 x 2 ; y = sin-1/x

y=cos-1}; y=cos-1x 3; cos-1 (:\)


X

(iii) y=tan-14x; y=tan-1 !; y=twi-lx 7; tan-1(1)


f if
X

2. Find
(i) y = sin-1(-cos x); y = tan-1(tan x); y = x sin-1x

x sin-1 x,. y = x2
(ii) y a x 3 tan- 1x;,y = sin-Ix; y = x tan-Ix
23
EXERCISES SET 18
(iii) y = sin-1 (1 -x2 ); y .. tan-1 (1-hc); y cos-1 (/1 -:-x);

y • tan-I(�)
X
(iv) y=(sin-lx)S· y =�·C,y=ltan - x y l · -- 1=--.-�
• cos-• x • � • (tan-Ix )4
sin- I (x) I, -1 1
(v) ·y e
= -
;�=sin (e ) ; Y""log(tan x);
x

.1 / y = tan-1(1og x)
1 1
(vi) �=logx .tan- x;\ y=e sin- x; y=log(cos-l x)2 ;
x

(vii) y=lsin 13x ; y= ( cos-1 !!. ) 8; y= ( tan-lx2 ) -3 '•


2 x
y=tan- I (e-7 )
*3. Find� when
(i) ·y=x . sin -1x+li-x2 ; y=x . cos-12 x-!,il-4x 2 ;
x3
= log( )
Y tan-lx
(ii) 1
y=log(e .sin - 3x); y={l+x )tan-l x -x; y = x sin- x
x 2 3 1 2

(iii) ·y,,,sin-1 ri::icT, O<x <l; y = tan-I x + tan- 1 ..!.;


X
y = (log x ).(sin-l x)3
7

- x -1 2 2

y=sin 1 < z+ x>l; y= e- .cos- 1 2x;


x
(iv)
x 1>·
y log(/tan-12x); y = cos(sin-1x)

4. If f(x) = sin_1.{, find f'(x ), f"(x). Pence determine

the value of f( "3) , f' ( "3) , f" ( "3) .


5. Given that f(x) = sin-1 (1og x) and g(x) = log(sin-1 x),
-'�
find f'(x ), g' (x) and hence calculate f' (e ·) • g' (-J ).
2
l
6. Det�m.ine the gradient of the tangent to the curve
(i) y=sin(cos-1 x) at the point where (a) x � (b) x =-!, =

x
(ii) y=tan-' (e ) at the point where (a) x=O (b) x=log 2
7. (i) If f(x) = x tan-1x ·· !,log(l+x2 ), calculate f'(l).
(i.i) If y=log(sin -1x) , 2
determine the value of* when

X = 2' x2 1
(iii)· If F(x) = -.- --
1 evaluate F' (-)
sin - x' 2
tan-1x
8. For the curve represented by y = e , find the
gradient of the tangent to the curve at the point where
it cuts the y c1xis. I!ence find the equation of this tan­
gent, and the �1 of the intercept cut off the

r
tangent on the axes. �
Find the equation of the tangent and the normal to the

I
curve
1
(i) y=(sin-l x)2 at the point on the curve where x = - 7i
1
-'-(ii) y=cos-1x2 at the point on it where X = f'i
(Do not multiply out results.)
24
EXERCISES SET lB
(� (i) If y sin-1 t, x = tan-1 t find the value of .QY.
when t = \. dx
(ii) If x e -t cos -1 t, y tan-1(e-t ), find the value
of f when t = O.
11. For the curve y= cos-1 x, determine the (i) gradient
(ii) equation of the chord joining the points on it with
abscissae 0, 1 respectively.
A, B are the points with abscissae
- ,1 13 respectively
H2.
1
on the curve y = tan� x. Determine the (i) gradient
(ii) equation of the chord AB.

Show that ![sin-1 (cos x)]= ![cos-1(sin x)], and hence


d d
that sin-l(cos x) - cos-1(sin x) = C, where C is a con­
stant. Uence, on substituting x = 0, show that C = O.
(Verify also that if x = n/2, C= 0).
Thus show that sin- 1(cos x) = cos-1(sin x).
4
1 . Prove that
(i) ...J!.(sin-1 x + cos-1x)= O, and hence show that
dx
sin-1 x + cos-1x= ;.
1
(ii) ...Q.(tan-1x)= ....!!.(- tan-l - ), and hence show
dx c!x x
tan-lx + tan-1; =
1
f.
*15. Prove that Y = 2 tan-1 tx . -1 (x-1) have the
,-- and Y = sin
l 2
x+l
same derivative, and that Y1 , Y 2 differ by a constant.
Find the value of this constant.
2
6
1 . (1.) tan-J (�
If y t an x,
) show t1at
l .QY. - -- --....-:
dx (4- 3 sinZx)
1

(ii) If y x tan-1 x, prove that (l+x 2 ) ff= 2.


l
dx
(iii) Prove that cos (cos-lx) = x provided lxl i 1.
Hence sketch y= cos (cos-lx) and y = x on separate
diagrams.
7
1 , If y = /sin-1 I + (tan-1x) 2 , find the value of � when
x = 1. [Leave answer in terms of n].

1 A is a point on a vertical pole !Om above the level of
an observer's eye. B is a point 30m above A. If the
observer is x m from the pole, prove that the angle a
subtended by 'AB, at the observer's eye is given by
40 _ - 01 _
a = tan-1 tan 1 · Show that da - -40 +
10
x x dx x2+ 1600 x2+lOO'
and hence find where the observer must stand in order
for this angle a to be greatest?

25
INTEGRATION
�- I �HEGRAT ION
Verify by differentiation that
(i) _g_(s:i..n-1 �) = .....£(·· cos-1 lf.) = �
dx a dx a va--x-

{ii) _g_(tan-1 �) = � z
dx a a +x
.....
Hence ShOI� that I
(i) (al f� = sin-1x + C1 = - c6s-1x + c2

I
1-x
dx = s i n ..: 1 � + C3 = - cos-1 � + C 4

I
a
I
a
( b)
�·

tan-1x + Cs (b)
I tan -1-
dx dx x+
a c6
(ii) (al = -
T+x7 = a x
� a

EXAMPLE I.
( i) f
dx ( I..I , I
Find the fol lowing indefinit� integrals
dx ... ,
( 111 dx
"7-12x2
I
I
T4'='x"l° 7i"TxZ"
(i
v)
f I�=x 2
(v)
l�; l x2
(vi)
dx·
9+ I 6x2 I
wf
Method
1
1 �:2 = f !#�xl = sin- I+ C1 or - cos
4
-1 I+ C2

dx [Note l-4x 2 = 4(\-x 2 ))


(ii) f� f -
•l-4 x - - 4 �- x )
_ 1 _ l . -1(�) + cl
l(�)2-x2 - 2 sin
dx

1
- 74
or - - cos-l

+ C2 <f)
= f 1
sin- ( 2 xJ + C1 or -
1 1
cos- (2x) + C 2
2 2

I /f2
2
(iii) I dx
h-12 x2 = f
dx

/i.2 (....l _ x 2 )
-
x �
---"d";=;
Jch )2-x 2
2 13
-=

12
_ 1 -1 __ x_ -1 x
sin ( ./'f/ /3) +C 1or cos-1(7-ifilj) + C2
- 273 2 213
- _l_ i -1(2 /3 x)+C or .:.!_cos-1( 2 /3x) + C
- 2 13 s n h 1 2 13 h 2

dx dx __l_ tan-1 � + C
-
(312) 2-1-xz
r
312 312
J 18+x f
I
(iv) 2

(v)
l �l x
2

26
EXAMPLES ON INTEGRATION

I
dx 1 [1
dx -l
L3 4 tan
(vi) J =
9+16 x2
+ x 2}
16{
16 /
1:
=�tan
1 -1 (�
4x) + C
12 3
[Note In parts (i), (ii), (iii) -above, the aim is to transform
the integral into the form f ,a� x�
-
whilst in parts (iv), �;
(v), (vi), the aim is to transform the integral into the
form f
a��xz· l

EXAMPLE 2. Evaluate the fol lowing definitP.


/6 in tegra Is. 2
( i)
_
lk
I 7R'x'7 dx
II) I
( ..
-215 4
+
dx
x2

Method

I
%
dx dx
(i)
Jl-9? =
_
-% /
1

t> - sin- (3
6
= 1 rsin- 1 (3 . 1

= 1[:in-1 <f> - sin-lC�)J

= 1[J - c;> j. using sin-i C�>


9
line (P) ,
A lteT'Ylati ve Zy from
6
1�

I
- /6
1
11� x
1
/6
2 = [- � cos-1(3x�f = -1[cos-
1
1

= -1[1- (11- r 6 noting cos-1 (-",)


cos-1

lT - cos-1(\)
{>
<f> - C�>]

lT = lT - j
=· as before. 9
2
[.! �1 .!
( ii)
-213
4+x
-& I=

=
2
tan-1

.12[r�4 _ c;>].
=
2 -2 13 2 [
tan-' (1) - tan-I (-/3)1

using tan-1(-13)=-tan-l(/3)=-;
711
= 24

27
INTEGRATION - SET lC
Find d�{sin-1(e )} an d hence evaluate
.v
x
EXAMPLE 3.
eXdx
f
/i-e
2x

-bt2

EXERCISES SET lC
1. llrite down the following indefinite integrals

(a ) I I J 11�:2;
J 19�:2; 11�zx2; n��16x2

(b) I I I I 1!�2; 1�:x2; 1!�6-;z-; 49!�1x2

( c) I I I 7
�Idx
;
5 dx
25+x2;
dx
1+49x2 ;
14 dx
/l-49x2

(d ) I I�; f �; f ;;-�:
/4��x2;
2

2. Evaluate the following definite int�rals

I I
!, 1 !, -J./ 12

(a) J i��x2; f �1�:2i li��z; l��x2


2 -- 0 -3·I 2 -1
o s 13 -!,
2
k:
2

(b) f
O
11f-x2;
-1
1tx2 ;

f
_12tx2 ; f f ��-�
- 2 _ -3/fi -1/6

(c)
J
k

�·
dx .
o
dx
ll-49x2 ;
13/ lo
I
dx .
ll:=75'x2'
dx
f f
0 -1 -1 I 12
lz110 V2

28
SET lC
3/i. . 5 /21<
dx
2/1!
r,;
;J.dx
f Q
.;dx
(g) f
o
19��6xi; J
-sl2i.
[ �; n
-:. ;;,:;....,_,.
-:- (x+2j2
-2

I
2/"i" o·
_d::.::x�-
6
-5 dx 9 dx dx .
(h) J
2 /5
4+ 25x 2 ;
J
-11/9
1 2 1+81x2 ; f 3 4+(�-1)
2'
f
2
3+(x-3)2

I
!�3
eX dx
3. (i ) Find c!![tan-1 (ex)J and hence evaluate J
l+e 2x
• of lo
(ii) �{sin-1( ; )} and hence evaluate dx

I
Find
x 5
d ll0x-x2
(iii) Show that
dx
(tan-l(cos x) J = - sin2x and hence
.....!!
2- sin x •

I I
find the value of sin x d�
TT

2-sin2x
0
t4. Differentiate (i) (tan-l x ) 2 (1· 1·) (sin · -1 x 2 ) 2with respect

I
to x . llence prove that.
l
J
tan-1 x dx _ �
- and
1/12
x sin-1 x 2 _
.-2!.:
l+x 32 J dx -
11-x4 1 44
2
0 0

I
(i) d
5. Find [ x sin-I x + �) and hence evaluate
dX k
f 2 sin -1x dx

I
0
(ii) Obtain d: [x tan-l x - 21 log(x 2 +1)) and hence eval-

uate f tan-Ix dx

I
I
Illustrate each of t!1e above definite integrals by a
diagram.
6. Prove, using the result for tan(A+B), that
511 r,; . d[ _, x-1 1
tan 75 °
tan 12 = 2+>'3;,fhow tnat tan ( ))
2+ dx x +l = i+;?
and hence evaluate: f � Verify the value of the
1 2
definite integral by
another method.
= cos 15° , and obtain a similar
Show that sin 75 ° =
'3+1
·':7. 212
result for sin 15° = cos 75 ° .
li
+
lG
Hence evaluate f ll��x2• by tw0 different methods.
16-li
d
*8. Show ti1at { x log(l+x2 )- 2x+2 tan-1x}=log(l+x 2 ) ,
(i)
d
and hence ,;., the value of ( log(l+x 2 ) d,

?O
----·---- .. .. ·-- AREAS, VOLUMES
tan-1x tan-lx
Prove that d {log x - 1 log(l+x2 ) --
,

dx 2 x-}= --;T
*(ii)
tan-1x
and hence determine the value of l � dx
fi

1
9. Show that �[�!+! - x2! ]
3
r
(xz!l)(x2+3)'
=
(i)

and hence prove that dx .:!!. 2/1)


= (1
Jo (xz+l)(xz+3) .'3 - -9-
1 _l_ _ x(x+2)
(ii) Sh ow th at x2+l + Zx-l - (xZ+l)(Zx-l)' and hence
x(x+Z) dx
find the value of f 2
13

(x + ;I.)(2x -1)
1

APPLICATIONS OF INTEGRATION. AREAS, VOLUMES ETC.

EXAMPLE I. Find the area bounded by the curve


2
y = ( 16-x )- , the x
2
axis and the ordinates at
x = -2 and x = 2.
Method.
On examination of y = �·
1

notice that the domain of x is


-4 < x < 4 and the range of y is
y -> 7n;'
l. l
i.e. 4. Asymp to tes occur
when 16-x = 0 i.e. x = ±4, A
2 -l,
2

sketch of y = ( 16-x ) is shown.


The required area _,.,.H x:

= .J 2
(l6-x )-� dx
2
=
2
2 I 1�
6-x
-x. ..__._-1:;:,.:,,...:::....:::,i...:::...:,..::,,.,::L-
,-4
2 by symmetry from the figure,
;i

2[sin-l f1: 2[sin- 1 (t)-sin- 1(0)) 1 sq. units.


-2 0
= = 2c2J-1 =

EXAMPLE 2. The area bounded by the curve y /1+4x2 '


the x axis, and the ordinates at x -t and x = f3
= 2'
is
rotated about the x axis. Find the volume of solid of
revolution so formed.
Method. y
Now il+4x2 � 1, since 4x 2 � O.
Hence� 1
i 1, but since
il+4x 2
is positive by conven-
tion, 1
,'. O < ��l.
Also as x � ±�, y � O.
-V
APPLICATIONS OF INTEGRATION

y =7m'xl" is shown; the


1
required volume is ind icated.
Hence the volume

=[ TT(1 !4 2)2 dx
./'J/2

1 x
-172
fih
=
* f �t·
-172
fi/2
=2!. [tan-12x] [tan- 1 (/J) -tan-·1 (-1))
2 -1/2
TT -7TT2
=fr{ - <- !> 1 2 rl!!.1
12 24 cu. units

EXAMPLE 3. The displacement X of a


particle at the end of time t is 'o lj

given by dx = l9-x 2 . Find an expression for x in


dt
terms of t, it when t 0, X = -3.
dt 1 dt 1
Method. Now since = =
.
hence
I
dx
dx
t = � = sin
dx/dt'
-1 x
• 3
dx �
+ C • , , •• , •,•••,,,,,,,,. · • (L)
;
When t =O, x =-3, • • 0 =sin- 1 (-1) + C i.e. 0 =-2 + C
Hence from (L), t =sin-I(!.)+ 2!.
i.e. sin -I t -
3 2
<f> f
and thus { =sin t
( - f>
TT TT
sin t cos - cost sin 2,
2
using sin(A-Il) result
cost, since cos 1 =O, sin�=1
=- 3 t
X

dx
A Zter>nativeZy from (L) • t = �=
9-x
COS

-
- cos 1 -+
X 1C1', •• (M)
When t =o, =-3, 0 = - cos- 1(- 1) + C1
3
i..e. 0 =-[TT-cos-1(1)] + C1
X

i.e. 0 =-(TT-0) + C1
Now from (Mj , 1: = - cos-1({) + TT, i.e. cos-l(j) =TT -t
Thus !. =cos(TT-t),
3 =- cost {using cos(TT-0)
cos e,
or use cos(A-B) result}
i.e, X a - 3 COS t
The use of the inverse co.sine is usuall y simpler to
hand le inthesetypesthan the inverse sine.)
{Note:

31
EXERCISES
EXERCISES SET 10
1. The gradient of the tangent at any point (x, y) on a
curve is ,
-
(a) (l-x 2 ) '2 (b) (l+x 2 )-1
Find the equation of the curve in each case, if it is
known that the point (l,11) lies on each of (a), (b).
2. Find y when x = 2 if
E.Y - 1
(i) · given that y = ..!!. when X = 1.
dx - � 2
E.Y 1
(ii) = given that the curve crosses the
dx 4+x2 '
y axis where y = 5.
3. For a particle travelling in the straight line x'Ox, the
displacement x from O after a time t·, is given by
(i) :� = /4-xz , and when t = O, x = 2. Find x in terms
of t, {Use both methods illustrated in the example
above.}
dx 1
(ii) = 11-9xz ' and when t - O ' X = - j• Find x in
dt
terms oft, and hence determine the value of x
11
when t =
.
6
(iii) :; = x +16, and when t
2
o, x = 4. Find. x in terms
=
11
of t, and hence calculate x when t = .
4
- 1
4. If y - �· find E.Y
dx"
--"'
(a) Prove that a stationary point occurs on y = (1-x2 ) 2
at (O,l) and by considering the sign of E.Y
d , show
- E.Y x+
that when x = 0 , < 0 and when X = 0 , E.Y .
dx dx > o
Hence prove (O,l) is a minimum turning point.
(b) Show that (i) the domain of x is -1 < x < 1
(ii) vertical asymptotes occur at x ±1
(iii) 0 s ll-x2 s 1, and thus that Y. � 1.
1
(c) Sketch the curve y = �· and find the area en-
closed by the curve, the x axis and the ordinates at
= -� and X = -.
X
13
2
1 d
5. For y = + Z• find �. and show that the point (0,1)
l x
is a maximum turning point.
(a) Prove further that (i) l+x2 � 1 and hence that O<y�l
(ii) there are no vertical asymptotes, and a hori­
zontal asymptote exists at y = O.
(b) Sketch the curve y = (l+x2 )-l, and find the area en­
closed by the curve, the x axis and the ordinates at
X '" t/3.
32
SET l D
6. Select the correct answer in each of the following
(i) The area in sq.units under the curve y = (4-x 2) -�2
between x = 0 and x = 1, and the x axis is
(A) 13 - l (B) A - 1 (C) i (D) 1 (E) 1T
(F) None of these.
(ii) The area in sq. units bounded by the curve
y = (4+x 2)-1, the x axis and the ordinates x = -2
and x = 213 is
7Ti
( ) (C) (F) None of
(A) (D) J...:rr. (E) .1!.
12 B 1; 2Z 24 2 these.
(iii) The volume in cu. units obtained by rotating the
-�
curve y = (1+9x 2 ) 2 from X = -
1 1
/j to X = :t /j ,
about the x axis is
2 2 2 (F) None of
(A) _lT_ (B) .1L. (C) -1!. (D) .1!. (E) .1!.
2
9 9 9 9 3 these.
t7. (i) The part of the curve y 2 (x 2 +4) = 9 for values of x
from O to 2 is rotated about the x axis. Find the
volume of the solid so formed.
(ii) Find the area enclosed between the curve
(9-x 2 )y 2 = 81, the x axis and the ordinates x = 0,
X = 3/2.
*(iii) Sketch the curve y = a 2 (a 2 +x 2 )-l, where a> O.
Show that the area enclosed between this curve and
the x axis is 1Ta sq. units. [Hint - what is the
lim - 1(2-)?J.
value of tan
X � co a
8. The sketch shows the semi circle y la 2 -x 2 . Assuming
that the length of the arc RS is given

2
X

by f / 1+(-�) 2 • dx, where x


x1 y
x = xz are the abscissae of R, S
respectively. Prove that the
measure of the length of the
(i) arc PLQ is 1Ta units
(ii) arc J)cl is 1[: units.

1
9. Sketd1 the curve y for O s x s 1. By dividing tile
l+x 2
=

interval [O, 1) into 4 equal parts, find approximftions


to the area represented by the definite integral J �
1 2
by using O
(a) the mid-ordinate rule (b) Simpson's Rule,
[Use reciprocal tables to aid calculations.]
Check these results by integration (Take 1T = 3··1416).
33
SET lD
10. Sketch the curve y: sin-1 x ; state the domain of x and
the range of y.
d d
Asswning that d (sin x ) s cos x , find d (sin_1 x ).
x x
(i) Hence determine the gra dient of the tangent at the
point A with abscissa "fz'
,on the curve y =sin-1 x .
Thus find the equation of this tangent, an� calcu­
late the coordinates of the point where the
tangent cuts the line x = 12·

I
(ii) Write down _g_ [ sin-1 .!. J
o
d
x a ' d
x
and hence evaluate rP.....,..,
125-9x 2
-5/6
11. Sketch the curve x =cos y for -2rr Sy S 2rr. Define the
range chosen by convention to determine y =cos-l x .
Hence sketch the curve y = cos-l x .
Find �(cos-lx), �sswning that �(cos x) =- sin x.
(i) Use this result to calculate the equation of the
tangent and of the normal at the point B on· the
curve y =cos-l x with ordinate rr/3. If these
lines meet the y a x is at L, M respectively, find
in irrational form *ML.
(ii� For certain curve y = .f(x), :; = /144-x 2.
a If
the curve passes through the point (0,0), find x
in terms of y.
12. Draw a neat sketch of the curve y =tan x for the inter­
val lxl S 2rr. Hence ·sketch x =tan y over a
corresponding interval, Define an approprfate range and
sketch the function y =tan-l x ,

Assuming that !(tan x ) = selx, find !(tan-1x).


d d
(i) P, Qare the points on the curve y =tan-• x . with
-1
abscissae 2, �respectively. The tangents at
P, Q to thl curve are inclined at angles a, 8 re­
spectively to the x axis. Defermine the magnitude
of *(8-a).
(ii) Write down !(tan-1
d
!>, and hence determine the
area between the curve y =(25+9x 2 )-1, the x axis
-5
and the ordinates at x. =�and x =73•
5

34
CHAPTER 2

ANALYTICAL GEOMETRY IN THREE DIMENSIONS

SECTION I. REVIEW OF GEOMETRIC RESULTS


In the following course, any geometrical results needed
in the work are usually stated where they occur.
However, some of the more important ideas are now
summarised.
1. Terms, such as points, lines, planes, surfaces are not
generally defined. Our basic concepts on these are usu­
ally adequate.
Thus, we understand what is �eant by ·the surface of a
sphere, or the surfaces of a prism, etc. For a plane,
we mean a surface, such that if any two points on it are
joined by a straigpt line, then that line will lie
wholly on. the surface. [Thus, a plane extends as far
as we please in all directions.]
2. A plane is determined by
(i) 3 non-collinear points,
(ii) a straight line and a point not on the line,
(iii) two intersecting straight lines,
(iv) two parallel straight·lines.

D ///
(i) · (ii) (iii) (iv)

/X/ [fl
These results above mean that one and only one plane can
be drawn through
(i) 3 non-collinear points.
(ii) a straight line and a point not on it
(iii) two intersecting straight lines
(iv) two parallel straight lines.

35
POINTS IN SPACE

,______,/ 7
3. Two distinct planes (a) (b)
are either
(a) parallel

t 7
or (b) intersecting
\./hen two planes in­
tersect, a line of
intersection is de­
termined.
4. If a line Lis perpendicular to a plane,
then every line lying in the plane, and
passing. thro ugh the point of inter- ffi
section is perpendicular to the given

Lt!7
line L. lhis line is called a nonnal to
the plane.
Lines which are normal to the same plane
are parallel to each other.

5. The angle between the two B


s l.
.
intersecting planes
Sz is defined to be the
angle 6· (AB,AD are lines
---- s, D
in the planes S1 • S2 at
right angles to the line
of intersection (AC) of
the two planes.]

6. The angle between the line


TI! and the plane S l is de­
fined to be the angle w;
(Rll is the projection of
TM in the plane S1,l
s

SECTION 2. POINTS IN SPACE


INTRODUCTION. BASIC IDEAS
y
In Two-Dimensional Analytiaal
Geometry, the plane is divided
into four quadrants by means of y
two axes - the x and y axes which
are taken at right angles to each
o ther. -x O >< A X

To position points in the


plane it is necessary to use co-
ordinates of t!le form (x,y). Thus, -Y
to reach the point P(x,y), we pro-
ceed x units from Oto A, then y units parallel to the y axis
from A to P.

36
POINTS IN SPACE
::aI.;.,;;n-"T;.;h.=.r.=.e.;;.e_-.;;.D.;;.i...;.;;. y_ti
.;. .;.o_n-:Ca Z A_n_a_Z.._
me...;n.;;.si _ c_a_Z G_eo_ m _ t_ _ry
_ e .._,
space is di vi -
ded into 8 regions, called .,. __, octants,..,. -:-by means of three axes -
the x, y, z axes which are taken so as to be mutually at
right angles to each other.

X
z'
The 3 planes (the plane xCy or xy- plane; the plane
xOz or xz- plane; the plane yOz or yz- plane) are called the
coordinate planes.
Let P(x,y,z) be any point in space, and through P be
drawn planes perpendicular to the three coordinate axes. (In
fig (ii), the planes FPDA, PFBE, PDCE are perpendicular to
the x, y, z axes respectively. That is, the plane FPDA is
parallel to the yz- plane, the plane PFBE is parallel to the
xz- plane and the plane PDCE i� parallel to the xy- plane).
The displacements OA, OB, OC are x,y,z respectively (x, y, z
may of course be positive or negative).
[To reach the point P from the origin O, we proceed
x units along the x axis from O to A, then y units parallel
to the y axis from A to F, and then z units parallel to the
z axis from F to P. This order is always preserved, i.e. x, y
then z.]
Thus, to designate a point in space, it is necessary to
use coordinates of the form (x,y,z). The positive directions
of the x,y,z axes are indicated by the arrows. Check that the
coordinates of A are (x,0,0), B(O,y,O), C(O,O,z), D(x;O,z),
E(O,y,z), F(x,y,O).
NOTES.
-1-.�-Probably the most difficult feature about Analytical
Geometry in Three Dimensions is to draw a figure to il­
lustrate the exercise. Much practice is necessary to
accomplish this successfully.
Although it is very awkward in many cases to obtain a
figure, you should try to visualise the figure. (If
possible, portray this by a reasonable sketch.) Useful
practical aids. in this work are pencils (to represent
lines); sheets of paper (to represent planes); the
room in which you are working (to represent

37
DISTANCE BETWEEM 2 POINTS
a rectangular prism); and a comer of the room (to re­
present the 3 mutually perpendicular axes).
2. In drawing figures, it is usual to use equal units on
the y, z axes but to use a shorter unit on the x axis.
Also the x axis is taken at an angle of 135 ° to the
y axis. These features give a sense of perspeative to
3-dimensional drawings.
{ In some texts, the x, y axes are interchanged. }
3. For convenience, the complete rectangular prism is not
usually drawn; merely the 3 edges OA, AF, FP. Thus the
points P1(2,3,4), Pz(-1,2,-3), P3(-l,-2,4) would be re­
presented as shown.
z. P3 z
/-X
I (-1,-H)
I 4 /

IV z -.. F I
4 ----

y 'r
I
I
Lz. �(-1,2,-.3) X

�· DISTANCE BETl'/EEN mo POINTS


Case 1. Distanae bet1Jeen O and P (x 1 ,y, ,z,)

In the figure, since P1F is


perpendicular to the xy- plane, P, (x., \1,.2,)
then P1F is a norr.,al to this
plane.
*P1FO ,.
°
90 . B
In fl OAB, right-angled at y
A, l*oFI = r,
I
• I
• • r 2 = x1 2 +y 1 2 by Pythag­
oras' Theorem . . •••. ••• •• •. • (l) F
In fl OFP1 , right-angled at
F, I *OP1 I = d

• • d 2 • r 2 + z 1 2 , by Pythagoras' Theorem.
Hence d 2 '" (x1 2 +y1 2 )+z1 2 , using (1)
Thus d = lx12+y, 2+z12 = l*OP1I

EXAMPLE The distance d from the origin


0 to the point P1(6,-2,5) is given by
y
d Ix l 2+y l 2+z l2
/62+(-2)2+52 '" 165 units.

38
THE MIDPOINT OF AN INTERVAL
Case 2. Distanae between the points

0�
P1(x1,Y1,z1) and P 2 (x 2 ,Y 2 ,z 2 )

(i ) (ii) z (iii) 2.
z (:
d, Pi
P. d,
z, Z2 F B
••�J.)
y
I ( ,,!j,, 5,) (x,,!ja 1&)

Fig (i) shows the points P 1(x1,Y1,z1), P 2 (x 2 ,Y2,z 2 ),


Fig (ii) shows P1P 2 as the diagonal of a rectangular
prism. Verify the ·coordinates of A, B. {As an exercise,
verify the coordinates of Care (x1,Y 2 ,z 2 ), D(x 2 ,Y1,z 2 ),
E(x2,Y1,z1), F(x1,Y1,z 2 ) ·}
Fig (iii) shows the essentials required for the proof.
In fig (iii), l* P1AJ ;: (y2 -Y1), l*ABJ = (xz-x1) and
l*BP2 J = (z 2 -z1), assuming x 2 >x1, Y2 > Yi, z 2 > z1
Now in 6 P1AB, where J*P1BJ=r, then r 2 =(x 2 -x1)2 +(yz-y1) 2
Also, in 6 P1BP 2 , which is right-angled at B, then
d = r 2 + (z 2 -z1) 2 , whence d 2 = (x2-x1) 2 +(yz-y1) 2 +(z 2 -z1) 2
2

:. d = l*P1P 2 J = l(x 2-x1)2+(yz-y1)2+(z 2 -z1)2 .


{Note the obvious similarit y with this result in 3D and
the corresponding result in 2D.}

EXAMPLE. Find the distance between the points Pi(-3,4,-6)


and P2 (-1,-2,4).

The distance from P,,P, is given by


2
d l(x 2 -x1)2+(y 2 -y1)2+(z 2 -z1)
l[-1-(-3) ] +[-2-(4) ] +[4-(-6) J 2
2 2

� 1(2)2+(-6)2+(10)2
• ll4o 2135 units

C. COORDINATES OF THE MIDPOINT OF THE INTERVAL P1P2

In the figure, Qis the midpoint of the interval LM [L,M


are the feet of the normals from P 1, P 2 to the xy- plane.]
Thus the coordinates of Qare [\(x1+x 2 ), �(y1+Y 2 ), OJ
Now P is the midpoint of the interval P1P 2 , and thus has

39
SET 2A
z

r-11

}i
0 11
}, }a..
I y
I
I
X
the same x, y coordinates as Q. The z coordinate is given by
(z-z1) = (z2-z ) , from the congruent . triangles PP1·N, PP2R.
Hence, the coords of the midpoint of the join P1P2 are
{'z(x 1+x2), l,( y1+Y2), 1iz1+z2)}.
[Note the obvious similarity with the corresponding
result in two dimensional analyti·c al geometry. J

EXPJ.f'LE The coordinates of the midpoint o( the join


P1(2,3,-4) �nd Pz(-3,4,-2) are
<H.2+<-3>], ! [3+4], H-4+<-2�} i.e. <-t. 112, ·-3>
z
..o- ------0
EXERCISES SET 2A E

1. In the figure_, the coordinates of F


G, (one of the vertices of the
rectangular prism shown), are
(3,4,6).
(i) State the coordinates of
A
0. 'f
the other vertices.
(ii) Find the lengths of the in-
tervals (a) OG (b). oD· (c) EB (d) /ID,
(iii) Stat·e the coordinates of the midpoints of the
intervals (a) F C (l>) EB (c) OG (d) AD.
2. Plot on the same diagram the points 11(2,2,0), B(2,o,· 2)
C(0,2,2), D(2,2,2). Draw the horizontal and vertical
line segments from A,B,C to the coordinate axes. Join
DA, DB, UC.
(i) What has be:m formed?
(ii) If E, F, G. are the midpoints of the intervals BD,
DA, DC; calculate the lengths of uE, OF, ·FG.
3. A(-4,-1,-1), B(S,5,-3), C(-3,1,-2) are the vertices of a
triangle. Find the perimeter of' l!. 1'.BC. [Leave resu'lt in
irrational form.]
4. The vertices of a triangle PQR are P(-6,5,3), Q(0,7,-7),
R(2,-3,1).
(i) State the coordinates of the midpoint S of��. and
determine the length .;,f the ,11edian PS.
DIRECTION ANGLES
(ii) C alculate the lengths of the other medians of �PQR
5. Show that the triangle with vertices
(i) A(4,2,4), B(l0,2,-2), C(2,0,-4) is equilate·ral
(ii) P(-4,-2,5), Q(4,- l,8), R(6,4,l) is right-angled
(iii) L(4,2,6), M(-2,0,2), N(l0,-2,4) is isosceles, and
fir.d its area.
6. Show that the points
(i) (-1,1,3), (0,5,6), (3,2,2) lie on a sphere with
centre (1,3,4).
(ii) (1,-6,10), (-1,- 3,4), (5,-1,1), (7,-4,7) form a
rhombus. [You may assume the 4 points are co-
planar.]
7. Show that the point S(�l,4,-3) is the circumcentre of
the triangle formed by the points A(3,2,-5), B(-3,8,-5),
C(-3,2,1). [Hint:� is equidistant from A,B,C.]
8. A, B have vert.ices (2,7,-1), (5,-6,-3) respectively.
Find the coordinates of the points P,Q,R which divide
'Ai into 4 equal parts.
9. A point P(x,y,z) is distant 8 units from the point
A(2,4,2). Show that (x-2) 2+(y-4) 2+(z -2) 2 = 64. If P is
on the x axis, find the numerical coordinates of P.
10. A, B have coordinates (2,0,0), (0,2,0) respectively. A
point P is such that l*POj = l*PBj = l*PAj = l*BAj,
where O is the origin. Find the coordinates of P.
[Let P be (x�y,z)J. 2
11. In the sketch,P1 has coordinates
(x1,Y1,z1 ), Find the distance
from P1 to O,A,B ,C. Prove that
the angles P1AO, P1 BO, P1CO are
right angles.
[Note: these results 1,)1,l l be
used in the next sec:tion.]

SECT ION 3. DIRECTION ANGLESI DIRECT I ON COS I l�ES,


DIRECTION· RATIOS.
�. In our work on Analytical Geometry in 2 Dimensions, we
constantly employed the idea of the direction of a line. We
did this by using the angle between a line and the positive
direction of the x axis, and we referred to the tangent of
this angle as the gradient of the line.
(i) Y (ii) L 1. (iii)

0 -Y X 0 -Y X
41
DIRECTION ANGLES
(ii) Calculate the lengths of the other medians of 6PQR
5. Show that the triangle with vertices
(i) A(4,2,4), B(l0,2,-2), C(2,0,-4) is equilate·ral
(ii) P(-4,-2,5), Q(4,-1,8), R(6,4,1) is right-angled
(iii) L(4,2,6), M(-2,0,2), N(l0,-2,4) is isosceles, and
find its area.
6. Show that the points
(i) (-1,1,3), (0,5,6), (3,2,2) lie on a sphere with
centre (1,3,4).
(ii) (1,-6,10), (-1,-3,4), (5,-1,1), (7,-4,7) form a
rhombus. [You may assume the 4 poin::s are co-
planar.]
7. Show that the point S(�l,4,-3) is the circumcentre of
the triangle formed by the points A(3,2,-5), B(-3,8,-5),
C(-3,2,1). [Hint:� is equidistant from A,B,C.]
8. A, B have ver�ices (2,7,-1), (5,-6,-3 ) respectively.
Find the coordinates of the points P,Q,R which divide
'Ali' into 4 equal parts.
9. A pointP(x,y,z) is distant 8 units from the point
A(2,4,2). Show that (x-2) 2+(y-4) 2+(z-2) 2 = 64. IfP is
on the x axis, find the numerical coordinates ofP.
10. A, B have coordinates (2,0,0), (0,2,0) respectively. A
pointP is such that /*POj = /*PBj = /*PAj = /* BAj,
where O is the origin. Find the coordinates of P.
[LetP be (x,y,z)]. z.
11. In the sketch,P1 has coordinates
(x1,Y1,z1), Find the distance
fromP1 to O,A,B,C. Prove that
the angles P1AO,P1BO, P1CO are
right angles.
[Note: these results i,nll be
used in the next section. ]

SECT ION 3. DIRECT ION ANGLESI DIRECT I ON COS I l�ES,


DIRECTION RATIOS.
f:l. In our work on Analytical Geometry in 2 Dimensions, we
constantly employed the idea of the direction of a line. We
did this by using the angle between a line and the positive
direction of the x axis, and we referred to the tangent of
this angle as the gradient of the Zine.
li) Y (ii) L t. (iii)

o -Y X 0 -Y X
41
DIRECTION COSINES, DIRECTION RATIOS
cosy'• - cosy.
For an undireated line in space, we·may take the direc­
tion angles (and the corresponding direction cosines) as
either of the sets above. [However, if a line P1P2 is �iven,
it is usual to imply that the direction required is P1P2 .J It
is usual to denote the direation aosines, aosa = l, aosB �-"!.>
aosy = n.
As with the 2-dimensional case, the angles a,B,y are not
independent of each other. However, there is?'.!£_ direation
relationship between the aatual angles themselves, but there
is an extremely important relation between their direation
aosines, namely aos 2 a + aos 2 B + aos 2 y = 1, i.e. l2+m 2 +n 2 = 1
In the figure, P1 is any
point (x1,Y1,z1) on the line C
''
L 2 , r.nd distant d units from '- (>c.,�, , �.) , ,,. L:,
o.
' P. ./"
Now in Set 2A, exeraise
No. 11, we proved that the \
angles P1AO, P1BO, P1CO are \
right-angles. B
Henae in 6 P1AO, y
x,
cosa = d'
Y1
in 6 P1BO, cosB and
=d.
zl
cosy = d
cos 2 a + cos 2 B
since d2 = x1 2 +y 2 +z1 2
[This is obviously similar to the result cos2 a+cos2 B = 1,
proven in section A , for 2-dimensions. J
Henae, if a,B are given for a line in space, then cosa,
cosB are known. Using the result cos 2 a+cos2 B+cos2 y = 1, we
can now derive cos 2 y. If the direction (upwards or d9wnward�
is specified, then cos y is single valued, and hence y is
single-valued (since O i a,B,y < rr). Thus, if we are g{yen
two direation angles (or direation aosines), the third direa­
pion angle (or direation aosine) may be deduaed.

EXAMPLE I. What are the


(i) direction angles Ci il direction cosines, of the
(al positive x axis (bl positive y axis
(cl positive z axis (dl. ray y = �. x in the first quad­
rant of the xy- plane?
State a set of direction ratios in each case.

43
DIRECTION COSINES, DIRECTION RATIOS
cosy' • - cosy.
For an undireated line in space, we may take the direc­
tion angles (and the corresponding direction cosines) as
either of the sets above. [However, if a line P1P2 is .,siven,
it is usual to imply that the direction required is P1P2 .J It
is usual to denote the direation aosines, aosa = l, aos8 "=_ �
_
aoay = n.
As with the 2-dimensi-0nal case, the angles a,8,y are not
independent of each other. H01,Jever, there is!!£_ direation
relationship between the aatual angles themselves, but there
is an extremely important relation between their direation
aosines, namely aos 2 a + aos 2 8 + aos 2 y = 1, i.e. l 2+m 2 +n 2 1
In the figure, P1 is any
point (x1,Y1,z1) on the line
C
''
L 2, r.nd distant d units from '- (>C.,�,, },) ., / L:.,
o.
' P. .,/
Now in Set 2A, exeraise
No. 11, we proved that the '\
angles P1AO, P1BO, P1CO are \
right-angles. 8
Henae in ti. P1AO, y
x,
cosa =d •
Y1
in ti. P1BO, cos8 =T• and
z}
in ti. P1CO, cosy =d x1 2+y1 2 +z1 2
Thus, cos a
2 + cos 2
8 + cos 2
y = -- .......--- = 1,
d
since d2 = x1 2 +y 2 +z1 2

[This is obviously similar to the result cos2 a+cos 8 = 1,
proven in section A , for 2-dimensions, J
Henae, if a,8 are given for a line in s�ace, then cosa,
cos8 are known. Using the result cos2a+cos 8+cos2 y = 1, we
can now derive cos 2 y. If the direction (upwards or d9wnward�
is specified, then cos y is single valued, and hence y is
single-valued (since O � a,8,y < rr). Thus, if we are gfpen
two direation angles (or direation aosines), the third direa­
tion angle (or direction cosine) may be deduaed.

EXAMPLE I. What are the


Ci) direction angles (ii) direction cosines, of the
Cal positive x axis (bl positive y axis
(cl positive z axis (dl. ray y = � x in the first quad­
rant of the xy- plane?
State a set of direction ratios in each case.

43
EXAMPLES ON DIRECTION ANGLES ETC.

'Z.

0 0

P 1 (1;6,3) with direction ratios -2:-5:4. [The fig­


ures illustrate the fact that direction ratios
-2:-5:4 and 2:5:-4 are equivalent. The lines are the
same, only the direction is reversed.)

EXAMPLE 2. The direction numbers for a I ine L1 are


8,-1,4. Find the direction cosines and direction
angles for L1,
z.
Method. In the figure; "Li, L2 , L3
are parallel lines and all have
direction ratios 8:-1:4. Since
cos 2 a+cos 2 S+cos 2 y = 1; we divide
each of the numbers 8,L---1, 4 by
/a2+(-1)2+42 , i.e. by 181 = 9. y
Hence the direction cosines
of L1, (and thus L2 , L3) are
8 1 4
The corresponding
9' .. 9' 9· _,
direction angles from the cosine X
tables, .are approximately 27 16', (180 -83 ° 37'), 63 ° 35';
° °

i.e. a = 27 °16', a= 96 ° 23', y = 63 ° 37'. (See figure!).

EXAMPLE 3. For a Iine L1, a 211 ' S = 311 ' find y.


3 "'ii°
Sketch the I ine.

f1ethod. Now cos


2;
• �os 120 ° � - f, cos 3� = cosl35 • - -;7f
°

Since cos 2 a + cos 2 S + cos 2 y • 1, .•.·a+!+ cos2 y = 1


i,e, Cos 2 y = 1-(ij) Caand hence COSY•%!
Th · • 211
...Ji§, since O <- y < 11, • • y=
TT
or .
3 3 211 )TT TT
.For L1, the direction. cosines are cos 3, cos "t; • cos
3
or cos 211 i.e •. - 1, - 72 1 % .!
,
3 2 2•

45
DIRECTION COSINES FOR THE JOIN OF TWO POHHS
A corresponding set of direction ratios is -1: -/2: tl.
L, z z

y
y

C. Direction cosines for the join of


P1(xi,y1, z1J and P2fa2,Y2 , z2J

P.

In the figures, new axes parallel to O x,Oy,Oz are taken


through P1( x1,Y1,z1). The direction angles for the join P1P2
are AP1P2 = a, BP1P2 = 8, CP1P2 = y.
Now the triangles P1AP2, P1BP2, P1CP2 are right angled
at A,B,C respectively .
Further I * P1Al=(xr x1), I *P 1Bl=(YrY1), I *P1Cl= (z 2-z1).
x2 x1 z
� , costl =�.cosy= 2� 1,
z
Thus cosa = where

d • l*P1P2I = l(x2-x1)2+(y2-Y1)Z+( z2-z1)2.


Hence, for a line directed from
x2-x1 Y Y1 z2- z l
( i) P1 to P2, the direction cosines are -d-, 2-d • d
x1-x2
(ii) P2 to P1, the direction cosines are ,
Y1-Y2
, .:.C.z.
-d - -d- d
Corres�onding sets of direction ratios
(i) for P1P2 are (x2-x1):(y2-Y1):(z 2-z1)
(ii) for P2P1 are (x1- x2):(y1-Y2): (z1- z2)

EXAt-f>LE. Find the direction cosines and a set of


direction numbers for the fol lowing joins.
Ci) 0 to P (3,2,-ll Ci i) AC-7,-8,I) to BC-3,4,-2).

46
ANGLE BETWEEN 2 LINES
Method
(i) The direction cosines for the
join OP1 are
3-0 2-0 -1-0
d' -d-. -d-
where d = lx12+y12+z12
"' /32+22+(-1)2 • 114
3
m·m·
2 -1
Thus, the d.cosines for OP1 are 7i4•
and an
appropriate set of direction numbers are 3,2,-1.

(ii) the d.cosines for the join AB


-3-(-7)_ 4-(-8) -2-(1)
are
d • d • d •
where d = /[-3-(-7)) 2+[4-(-8)] 2+[-2-1]2
= 1(4)2+(12)2+(-3)2 "' /i69A
i.!..
= 13 (�7,-M
Thus the d. cosines for AB are
4 12 -3
TI• 13• 13·
In the figure, L2 is the line through
4 12 -3
Its d.cosines are also [The direction
TI• TI• TI·
angles a,B,y are indicated on the sketch.] An appro­
priate set of direction ratios is 4:12:-3.

Angle between two lines. z


In the figure, L1,L2 are two
non-parallel lines in space. We
wish to. find the angle between
them. [Note that the lines may not
be coplanar, i.e. they may be skew
lines (i.e. non-intersecting lines,
as opposed to parallel lines). For
example, one may be the diagonal
of the floor of a room and the
other, an edge of the ceiling. ]
First, draw rays L11 , L2 1
through the origin O parallel to
L1 L2. Then, e, the angle between
L1 l, L2 1 is defined to be the angle
between the lines Li, L2.
Take any points P1 (x1,Y1,z1), P2(x2,Y2,z2) on L1 , L 1 l
1

espectively so that P1,P2 are distant d1, d2 from O.


�� � th� �sine rule in 6 P 1 0P2,
d1 +d2 -r
cose .. where T = I *P1P2 I •
2d1d2 '
47
ANGLE BETWEEN 2 LINES
2 2
(x12+y12 +z12 )+(x22 +y2 2 +z2 2 )-[(x2-x1) +(y 2 -y1) +(z2-z1) ]
2

2 d1 d2
2x1x2 + 2Y1Y2 + 2z1z2
on collecting terms
2 d1 d2
xi
d1
x2 + Yl
d2 d1
. Y2 + z1
d2 d1
z2
d2
........................ (1)

Now, the �irection cosines for the ray OP1 are _,_,_ x1
Yl z1
x2 Y2 z2 d1 d1 d1
and those for the ray OPz are ' a";"• a";"· These direction co-
dz
sines will be denoted by l1, m1, nl and l2,m2 ,n2 respective -
ly.

Now, if the lines 11, 1 2 are


(a) parallel, then their direction
cosines are equal (if they �re
in the same direction) or
equal in magnitude and op­
posite in sign (if they are in
opposite directions).
i.e. l1 l2, m1 = m2 , n1 = n2 0 y
or l1 = -lz, m1 = -m 2 , n1 = -n 2 . X
However, since direction ratios are proportional to
the direction cosines, if d.ratios for L1,L2 are proportion­
al, then the lines £1, £2 are parallel. Thus if a1,b1,c1 and
a2 ,b2,c2 are sets of direction numbers for 11, 1 2, then if

"I
a1 : a 2 =b1 : b2 = c1': c 2 , the lines are parallel.
(b) perpendicular, then a.= and thus Z
cos9 = O.
From (2), this leads to
l1l2+m1m2+n·1n2 = 0 • •' ••• •., '. (3)
F'u:rther,since the direction y
numbers a1,b1,c1 are proportional
to the direction cosines l1,m1,n1
mi nl
then ll k1(say), where k1 is a non-zero con-
ai' - �"c"j" =
Hence l1 .. a1k1, m1 = b1k1, n1 = c 1k1.
stant.
Similarly for l 2 , mz, nz.
Thus (3) becomes (a1k1)(a2 k 2)+(b1k1)(b 2 k 2 )+(c1k1)(c 2k z)=O
i.e. a1a 2 + b1b 2 + c1c2 = O, since k1k2 1 O.

EXAMPLE I. Find the cosine of the acute sngle between the


line AB where A is <3,-4,2)·, Bis (5,-1,1) and the line
CD where C·is (-6,2,7), D is (-7,-2,9).

48
EXAMPLES ON ANGLES BETWEEN 2 LINES
Method The d.cosines for the z.
---line AB are given by
x2-x1 y2-y1 z2-z1
·
_d_1_·��
5-3 -1-(-4) 1-2
i.e. y
di' d1 d1 L'2
2 3 -1
i.e. ,
� �·�
where d1 = l*ABI = 1(5-3)2+(-1+4)2+(1-2)2 = 14+9+1 = /I4.
2 3 1
Hence .el = 7fz:• m1 = 7vt, n1 = - "'7f7:• where l1 ,m1,n 1 are
the d.cosines of AB.
Similarly, for the line CD, the d.cosin es ,are
(-7)-(-6) -2-(2) 9-7 -1 -4 2
d2
,
d2
, --;i; i.e. "a:;• "a:;• d2
' where

d2 = l*cDI 2 2
= 1(-7+6)2+<-2-2> +<9-7) = 11+16+4 = m.
Thus, .e.2 = m·
-1 -4
m2 = 7rf• n2 =
2
7ii where .e.2 , m2, n2 are
the d.cosines of CD.

Now cose l1l2+m1m2+n1n 2

I_- 77t -
2 -1 3 -4 -1
I <-m · -m> + <m · m> + <7i4 - 7h') I, \acute

1
angle re­
-2-12-2 16 _ 816
lt. lt.11.1! 21
quired)

{Note the actual angle may be obtained usin g tables, if re­


quired.}

EXAMPLE 2.
(i) Prove that the triangle PQR with vertices
P(-4,-2,5), Q(4,-l,8l, R(6,4,ll is r�ght-angled-.
(ii) Find k if the join AB where A is (3,-1,4),
8 is (-5,2,-1) is para I lel to the join CD where C
is ( 12,k,-2), D is (4,2,-n.

Method
(i) From the sketch, it would seem
P(-4.-2,5)
that the angle P�R is the
right-angljl, It is simpler
bere to use direc.tion ratios,
rather t!KP!, __g._irectiorz cczsine_?,·
For QP, a ·set of direction
ratios is
y
- (y2-Y1):(z2- z1)
(x2-x1):
i.e. [-4-(4) J: [-2-(-1)]: [5-8]
i.e. -8: -1:-3 R(6.4-.1l
whereas, a set of direction

49
SUl+1ARY
ratios for QR would be. [6-(4) J: [4-(-1)):[1-8)
i.e. 2:5:-7.
F:or PQR to be a right angle, then a1a 2 +b1b 2 +c1c2 = O,
where a1,b1,c1 and a2,b2,c2 are sets of direction
numbe1s for QP, QR
Here a,a 2 + b,bz + c,c 2 (-8)(2) + (�1)(5) + (-3).(-7)
-16 - 5 + 21 = 0
i.e. *PQR = 90 °
2
(ii) Here, a set of direction ratios for
AB is [-5-(3)):[2-(-1)): [-1-4)
-8:3:-5
CD is [4-12]:[2-k]:[-7+2] w -8:2-k: -5
If AB is parallel to CD, then 0 )(

al b1 C}
-8 3 -5
-;;- "' i;; "' � i. e • -8 "' 2-k "' -5

Solving, • •• 1 = 2�k i.e. k = -1.



==-======�-=== (ti,i,-1)

SUM'-lARY
].
I. The direction of a ray in space is
determined by 1he use of direction
angles a, B,Y where O S. a, ii·, y < 11. P,
Instead of angles, direction co­
sines cosa, cosB, cosy are easier
to use, but in actual practice, we y
resort to direction ratios (i.e.
sets of numbers proportional to
the direction cosines).
2. cos2 a + cos 2 B + cos2 y = I.
3. For the join of P1(x1,Y1,z1> to
P 2 <x 2 ,Y 2 ,z2l the
< i l direction cosines are 0
x2-x1 Y2-Y1 zrz1 y
- -, - -, -d - where X
d d
d= I *P 1P2 I=/(xz-x1 l2+( Yz-y1l2+< zrz 1l2 L'

( ii} direction ratios usua I Iy tak­ z.


en are (xz-.x1>:<y 2 -Yi>:<z 2-z1l,
4. The angle e t-.,::tween two I ines L1,L 2
is given by cose = l1l 2 + m1m 2 +n1n2 ,
where l1,m1,ri1 and l2,m2 ,n2 are the
direction c. ,ines o·f L1,L2,
5. If the Iints L1,L 2 are
( i) pCU'a2Zel, then l1:l 2 = m1:m 2 = n1:n2
or a1:a 2 = b1 :bz = c1:c 2 ,
where a1,b 1,c1 and a2,b2,c2 are direction numbers
for L1,L2,

50
SET 28
( i j) pe rpendiaular>, then l1l 2 +m1m2+n1n 2 . 0
or a1a2 +b1b2+ c1c2 0

EXERCISES SET 28
1. What are the direction angles a,8,y for the
(a) positive x axis (b) ray y = 13 x in the positive
quadrant of the xy- plane
(c) ray z = x in the positive quadrant of the zx-plane.
Verify in each case that cos 2 a+cos2 6+cos 2 y = 1.
2. (i) The direction cosines for a line L are given by
cos a = 3/ 4, cos B = 1/4. Find the values of cosy.
(ii) If a = 60 ° , B = 45 °, find the poss.ible values of y,
3. For a certain ray OP , it is given that
(i) cos a = J, and cos B = cos y , where these are bot:1
positive. Find cos B,
(ii) the ray makes equal angles with the positive parts
of the coordinate axes; find these angles.

4. The direction ratios of a ray L1 are 3:2:4. By means of


a sketch draw the lines L2,L3 parallel to 11 through the
(i) origin (ii) point (5,-1,6). 1-lhat are the direc­
tion cosines of L1,L 2 ,L3?
5. Determine the direction cosines and direction angles of
a 1:lne directed upwards from the origin O if its direc-,.
tion numbers are 2,-3,6. What are the corresponding
results if the line were directed downwards from O?
Sketch the figure in each case, marking the angles a,6,y
clearly.
6. On the same diagram, plot the points A(-8,1,4),B(3,4,0).
Calculate (i) the distance from O to A, and from O to B.
(ii) the direction cosines of the rays OA, OB,
and corresponding sets of direction ratios
(iii). the direction angles of OA, OB.
(iv) the cosine of the acute angle e between
the rays OA, OB.
7. Find the direction cosines, and direction ratios, for
the join P1P2 where Pi,P2 have the following values
(a) P1(4,5,-3); P 2 (5,3,-l) (b) Pi(0,12,0); P2(-4,0,3)
(c) Pi(-6,5,-4); P2(-5,-2,-4)
(d) P1(2,-1,-5); P2(5,-1,-5),

8. The direction ratios for two lines L1, L 2 are respect­


ively 1:2:1 and 2:-1:1. Determine the direction cosines
for L1,L2, and hence, show if 0 is the acute angle be­
tween 11,12 , then cos e = 1/6. Use tables to determine 6
as accurately as you can.

51
SET 28
9. Determine the cosine of the acute angle between the
lines with direction numbers
(i) 2,1,-2 and -3,6,-2 (ii) 3:4:5 and 1:1:-4
10. The points A,B,C,D in space have coordinates (3,1,-2),
(4,0,-4), (4,-3,3), (6,-2,2) respectively. Qetermine
(i) the direction cosines for the lines AB, CD.
(ii) cos8, where 8 is the acute angle between these
lines
(iii) the magnitude of 8 in radians.
11. Calculate the cosine of the acute angle between the
lines joining the following pairs of points, namely AB
and CD.
(i) A(8,2,0), B(4,6,-7); C(-3,1,2), D(-9,-2,4)
(ii) A(2,3,5)_, B(6,-2,2); C(-2,-2,8), D(4,1,6)

12. A triangle ABC has vertices A(l,-1,0), B(2,1,-1),


C(-1,1,2). (Sketch the figure).
Find (i) the lengths of the intervals AB, AC
(ii) the direction cosines of the lines AB, AC
(iii) cos BAC, and hence show that AB, AC are at
right angles
(iv) the area of the triangle ABC, in simplest ir-
rational form.
13. A(l,-1,2), B(2,0,-1), C(0,2,1) are three points in space.
Calculate (i) the direction cosines of the lines AB,AC
and hence the value of cos BAC
(ii) sin BAC (in surd form), using the ident­
ity sin 2 e+cos2 e = 1
(iii) the area of 6 ABC, using the result:
Area = \ be sinA.
14. A(3,1,-2), B(5,2,2), C(l,-3,9), D(-3,-5,1) are points in
space. Find a set of direction ratios of the lines AB,
CD and hence prove that these lines are parallel.
15. Assuming that the points P(4,7,8), Q(2,3,4), R(-1,-2,1),
S(l,2,5), are coplanar, prove that PQRS is a parallelo­
gram.
16. (i) Determine whether the join of A(2,1,3) and
B(-6,3,7) is parallel to the line with direction
ratios 4:-1:-2.
(ii) For what value of k will the line through (2,1,2)
and (4,1,k) be parallel to the line joining
(-1,4,1) and (2,4,5).
17. (i) The direction numbers for two lines Li, L2 are
2,-1,4 and 3,-2,-2. Show that the lines are at
right angles.
(ii) The direction ratios for two lines are 3:-2:k and
-2:k:4' , Find k if the lines are perpendicular to
each other,
18. A,B,C have coordinates (4,-2,3), (6,1,7), (5,4,-2). Find
a set of direction ratios for the lines AB and AC, and
52
SET 28
hence prove that the triangle ABC is right-angled at A.
19. Show that the join of (5,1,-2) and (-4,-5,13) is the
right bisector of the join of (-5,2,0) and (9,�4,6).
20. Assuming that the points A(2,-l,O), B(0,-1,-1), C(l.,1,-3),
D(3,l,-2) are coplanar, prove that the figure ABCD is a
rectangle.
21. A(4,l,2), B(5,k,O), C(2,1,l), D(l,3,-1) are such that
(1) AB is parallel to CD
(ii) AB is perpendicular to CD.
Find the value of kin each case.

22. Prove that the points A(l,-1,3), B(2,-4,5), C(5,-13,ll)


are collinear, by using (i) direc.tion ratios
(ii) distances. [Note in (ii), JIR • 3/IT.]
23. TI!e figure shows a cube of 2
side 1 unit. Determine the
cosine of the angle be­
tween the diagonal OE and
(i) OC (ii) OB (iii) GB.
24. A line L with direction
numbers a,b,c is perpen­
dicular to eaah of two
lines 1 1, 1 1 whose direc­
tion numbers are 1,2,-4 L
[I
and 2,-1,3. Explain why
a+2b-4c = 0 and 2a-b+3c=O.
Solve these equations sim­
ultaneously for a, b in
terms of c [i.e. by elim­
inating b, express a in
terms of c, etc.).
2 11
Hence show that a:b:c= - c : c :c c -2:11:5. {Note that
5 5
since only direction ratios are given, the direations of
11, L2 are fixed, not their positions. TI!us, if they are
non-intersecting and non-parallel, we simply move the
position of one of them, until the lines intersect. Then
the required line L is a line perpendicular to two
intersecting lines, i.e. L is a normal to the plane
containing these intersecting lines.}
25. Determine the direction ratios of a line L which is per­
pendicular to the lines AB and CD, where A is (2,3,-4),
B is (-3,3,-2), C is (-1,4,2), D is (3,5,1). [Let the
required direction ratios be a:b:c, and proceed as in
exercise 24. J

53
EQUATIONS OF LINES THROUGH 0
SECTION 4, EQUATIONS OF LINES
f;,, LINE THROUGH THE ORIGIN WITH GIVEN DIRECTION

In fig (i), L is a line through O with given direction


cosines 2- cosa, m = cosa, n = cosy. A point P 1(x1,Y1,z 1) is
taken on L, distant d1 from O.
1. Xl m = .,...
Thus • -
y1 zI
n .. -
d1' d1' d1'
In fig (ii), a variable point P(x,y,z ) is taken on L, so
that *OP• t.*O�, where t is a variable reaZ n1471ber. Hence
*OP .. td1,
The triangles OP1 C, OPD are similar
*DP•� *OD
= � C i.e. *DP = t z1
zl dl RO '
Also, the triangles OAC, OBD are similar
, -*OB = -
*BD = --
*OD = - td1
• from above.
' x1 Yl *OC ' d1
Thus *OB tx1 , *BD = ty 1

Hence x = tx 1 , y = ty 1, z = tz 1 .............. (I)

{These results (i), are the 3 parametric equations of


the iine OP . As t varies from -m to +<x>, the point P(x,y,z )
will traverse the infinite line L through O and P 1 ,}
Now from (1), we obtain

:1 = t = �l = f .............................. ( 2)
Divide the denominator of each ratio in (2) by d1 (iO),
• ....1L....... ., ..L_ = _z_ = _t_
• x1/d1 y 1/d1 z1/d1 l/d1

i.e. � .. .i'.. .. .! n X l etc.


since�=
l m n' -;r-l • • • • • • • • • • • • (3)

{The form (3) gives the equations of a line through the


origin O with given direction cosines .f.,m,n.}

54
EQUATIONS OF LINES THROUGH 0

Note that the line is determined by two equations


in x, y, z (see eqn (3)) or by three parametric
equations (see eqn (I>).

EXAl,,'PLE I. Determine the coordinates of the point


P2 on the line joining O to P1 (1,3,4), if
*OP2:*0P1 = 5:2.

Method The coords of P2 are best z


---obtained by using the paramet­
ric equations
x• tx1, y • ty1, z .. tz1,
where t = 5/2.
Th""
..., X "' 2 1 = 2 4
2 2'
5 15 y
3 .. 2'
y = 2 3
5
z = 2 4 = 10

Henae, the coords of P2.are (52, 10).

{Note: This parametric method is ext_•emeLy useful in deter­


mining where a line pieraes a plan�. Ttls will be treated in
a later section,}

EXA�LE 2. Find the equations of the I ine through 0


and (i) the point P (-1,3,-7)
(ii) with direction angles 135 °, 120 ° , 60 ° .

Method
--
(i) The equations of the line OP1
are given by 'I
1L. ,,. L.. ,. .Li e 1L. ., Y ., 2
x1 Yl z1 · • -1 3 -7
(ii) The equations of the line
L are
_x_ • J__ = _z_ since
cosa cos6 cosy'
.t = cosa,etc,
i.e. -�/./2 = :W- = ii 2 )(

i.e. -fix =- -2y .. 2z, i.e. x '"' /i'y •·.fiz

55
CHANGE OF ORIGIN
8. CHANGE OF ORIGIN.WITHOUT CHANGE OF DIRECTION OF AXES
In the diagram,
0 1 has coordinates
(p,q,r) relative to
the origin O of the
x,y,z system. Let
new axes O'X,O'Y,O'Z

-
be taken through d
parallel to Ox, Oy,
Oz respectively.
Let P be a
point in space, with
coordinates (x,y,z)
relative to the ori­
gin O, and with co­ !:;I
ordinates (X,Y,Z) relative to O' • . From the figure, it can be
seen that
*DP *DE+ *EP, i.e. z = r + Z
Similarly *BD *AC+ *FE, i.e. y q + Y, and also x = p+X,
. .
Thus, the equ ations for change of or1g1n to (p,q,r)
witho ut change of direction of the axes are given
by
x = X + p, y = Y + q, z = Z + r

{These results are o bviously similar to the correspond­


ing results in 2 dimensions.}

EXAMPLE The equation of a certain surface.in space is


x2+y2+z 2+2x-12 y-10z+46 = 0. Wh�t does this equation
become if the axes are translated to the po int
(-1,6,5)?

Method. The equations for the translation of.the axes become


x = X+p i.e. x = X-1; y = Y+q i.e •. Y Y+6;
z = Z+r i.e. z = Z+S.
Substituti.ng in the given equation, we have
(X-1) 2 +(Y+6) 2 +(z+s) 2+2(X-1)-12(Y+6)-10(Z+5)+46 o.
This reduces to x2 + Y2 + z 2 = 16.
[The given equation is that of a sphere, centre (-1,6,S),
radius 4 units.]

56
THE RA TIO - PARAMETRIC FO"'I
£;,, OFTHETHE
RATIO- PARAMETRIC FORM FOR THE EQUATION
LINE JOINI NG TWO POINTS. 2.
P1 (x1,Y1,z.1) and P2(x2,Y2, z2) <ic.�.�\P Pz!.xi,1.1&•}1)
are two points in space. The point ,,�t}
P (x,y,z) lies on P1P2 such that I
*P1P : .*PP2 = t : (1-t)
i.e. *P1P : *P1P2 = t : 1, I
where t is a variable real number �I i I �2
(usually rational). lly use of
parallels in the figure, we D I
obtain
*P1P:*P1P2 *GH:*GI
*AB :*AC
*DE:*DF
x-x Y
Thus � = 2 , = -Y1
1 x-x1 Y2-Y1
ll.
i.e. t(x-2 x1) = x
-x, and x = x1 +t(x-2 x,)
Similarly, we may obtain y = Y+1 t(y 2-Y1); and by drawing
a line through P parallel to the xy- plane to cut fGP1 and'IP2
we can also prove z = z1+t(z2-z1),

Thus, theparametric coordinates of h t e point


P(x,y,z) divii d ng the Join P1 x ( 1, Y1,z1l to
P2<x2,Y2,z2l in ht er atio t : ( 1- t) are,
{x1+<t x2-x1l, t y 2-Y1l, z1+t<z-2 z1l}
Y1+ C

Notes.
1. As t takes on all values from - 00 to +oo, the point· Pwill
traverse the infinite line through P1, P2,
If O < t < 1, then Plies in the interval P1P2; if
t = O, Pis at P1; if t = 1, Pis at P;2 if t > 1, Pis
on the ray P1P2 but not between P1,P2; if t < O, Pis
on the ray P2P1 but not between P1, P,2
2. In a plane, the coordinates of the point dividing the

l
join of A(x1,Y1) and B(x,2 Y2)
px2 +qx1 PY2+qYl
p+q ' p +q
}. There ex
in the ratio p : q are

ists a corresponding result


for·3 dimensions. Thus the coordinates of the point div­
iding the join A(x1,Y-1,z1) and B(x2,Y2,z)2 in the ratio
p : q are
px2 +q x1 pz2 +qz1
f 1
l p+q • p+q

EXAMPLE P1 is (3 2
, 4
, ), P2 is (6,-3,5). What are
the coordinates of ht epoint dividing P1P2 in the
( ) 5 : 2 Ci
ratio i ii -5: 2.

57
EQUATIONS OF LINE THROUGH A POINT
Method. Note that the com- z
ponents of the ratio
t:(1-t) add to unity.
� (3, i,4-)

t:f
(i) For the ratio 5: 2, we
write (note 7=5+2); Pa (6,-3, 5") O y
5
and thus t = .
7 X
Hence the coords of the point Pare given by
x = x1+t(xrx1) 3 + 1(6-3) = 3 + ll 1.§.
7
7 7
( -3-2 ) 25 -ll
{Y = Y1+t(Y2-Y1) 2 + 1 =2 -
7 7 7
5 (5- 5 11.
z z1+t(z2-z1) + ) = =
4 7 4 4 + 7 7
-5 2
(ii) For the ratio -5 2, we write _ _ (note -3 = -5+2) ;
3 3
5
and thus t = 3.
The coords of the required point are given by
= 3 + 1(6-3) 8
-19 3 5 17
= 3' !J'lld z =
4 + 3(5-4) = 3•

D. THE EQUATIONS OF THE LINE THROUGH Pi,


HAVING GIVEN DIRECTION.
In the sketch, the line L
passes through the point L
P1(x1,Y1,z1) with given direc­
tion cosines l,n,n. The axes
are translated to P 1 • The
equations for this translation
are (by Section B above),
x a X+x l , y "' Y+y l , z = Z+z l . , . ( 1)
Now, from Section A above
the equations of the line L
are given by
X Y Z
I = ; i• which becomes, using (1),
-- = - - • - -
l m n

Hence, the equations of the I ine through P1<x1,Y1,z1l


with given direction cosines l, m, n are
x-x1 y-y1 z-z1
m n
This form is often referred to as the syTT171etric form
for a straight line.

58
EQUATIONS OF LINES THROUGH A POINT

Note. If a set of direction ratios for the line are


a, b, c then a : b : c·= l: m : n, and hence the equa­
tions of the Iine above may be also written in the forms
x-x1 y-yl z-21
a b C

EXAMPLE I. Find the equations of the I ine through


P1 (2,-1,5) and with given
Ci) direction angles 45 ° ,60 ° ,120 °
Cii) direction ratios -3: 4 : 6.

Method.
(i) The equations of the line are tz
given by I
x-x1 y-y1 z-z1
- -= - - = - -
l m n U!.o
0

i.e. _]cl_ - ....1.±L_ - z-5


cos45 ° - cos60 ° - cosl20 ° y
i.e.
x-2 y+l z-5
I
T77i '2 -�
i.e.
x-1
72
y+l
1
z-5
=�
/Ix
(ii) The equations of the required
line are
x-x1 y- y1 z-z1 -t-z 6
- -= - - = - -
a b c I
x-2 _ .x±1 _ z-5 I
i.e. -3 - 4 - 6 4
(2,-1,5)1
{i,ote. The equations a!'e left - --'>Y
in this fo!'TTI, 'because from it /
we can state, at sight ,
point on the line, and the
a k-x
direction ratios of the line.} 0
X

EXAtv'PLE 2. Determine the direction cosines of the


Iine whose equations are
1-x - 2v-5 - z+2
3 - -I - 4 ' and find the coordinates of 3
points on it,

Method. The given equation may be rewritten as


x-1 y - 5/2 z+2
-3 -'2 ' whence the line can be seen to
pass throuth Pi(l ,5/2,-2), with direction ratios -3:-'2:4
i.e. -6:-1:8.

59
EQUATIONS OF LINES THROUGH A POINT
-6 -1 8
The direction cosines are thus 7fm'•. /IM' 7fiIT"

To obtain points on the line, .we express it in parumet-


x-1 - 5/2 z+2
ria form thus: Let�= Y. = = >. • where>. is real.
z-
5 -i. 4
X = 1-3)., y = I, ' z = -2+4>..
>.
Now substitute arbitrary values for >..
Thus if >. 1·' = -2, y = 2, z = 2
X
3
>. 2·• X = -5, y = 2' z = 6, and so on.
Henae the points (-2,2,2), <-5,23,6) also lie on the line,

{Notes. 1. This parametric method is e�tremely important; it


is the method used to find where a line pierces a
plane. (See Set 2H No. 7.)
l, If l, ru, or n are zero, then care must be exer­
cised with the equations. Thus, the equations of
the line through the point (1,-2,3) with direc­
tion numbers 4,0,5 may be written by convention
x-1 z-5
as � - = � 0
= � -. In this form, no division
4 3
by zero is implied; the O simply indicates the
direction ratio. This equation is also written as
x l z 5
� = ; , y+ 2 = O.

3. Seemingly different equations of lines may be ob­


tained by using different points on the line.
Thus, consider the line with equations
x-3 y_+l z-5
= = '''''
-4- 2 ---:-j' ••••• '' ' ·'''''''• • • • • · · • (A)
The point Pz(ll,3,-1) lies on the line since
11-3 3+1 -1-5
-= = Using the point Pz, the equa-
�-4 �2- ---:}"""·
tion of this line could be written as
x-11 y_-' 3 z+l
= = '''''' ''.''' ''.''. '''''' ''.' (B)
4 2 -3
which seems entirely different to (A).
However, from (A), we obtain
�2x-6 = 4y+4 i.e. x - 2y 5
l-3y-3 = 2z-10 i.e. 3y+2z 7
whilst from (B), we obtain
\2x-22 = 4y-12 i,e, X - 2y 5
L-3y+9 = 2z+2 i.e. 3y+2z 7
i.e. Equations (A), (B) represent the same line.}

60
EQUATIONS OF LINES THROUGH 2 POINTS
�- THE EQUATIONS OF THE LINE THROUGH TWO GIVEN -POINTS
2
In the sketch P1 is (x1,Y1,z1),
P2 is (x2, Y2 ,z2). Let P(x,y,z) be
any point on P1P2 such that
*P1P: *PP2 = t : (1-t).
Now the coordinates of P, (by
Section C above) are given by
x = x1+t(x2-x1) (1) (x,.�.. �.)
{Y = Y 1+t(y2-Y1) (2)
z = z1+t(z2-z1) (3) 0
y
From (1), we obtain X
x-x1
t(x2-x1), i.e. __ t. Thus from (1), (2), (3), we

x-x1 = y-y1 = z-z1 t.


obtain

Henae, the equations of the I ine through P1<x1,Y1,z1>


and P2<x2,Y2,z 2> are

EXAMPLE Determine the equation·s of the Iine _through


the points A(-2,4,-5) and 8(1,-3,-2). Find another
point on the line.

Method. The equations of the line 2.


are given by
x-x1 y-y1 z-z1
x 2-x1 Yi-Yl z2-z1 y
x-(-2) _ :t.:J._ _ z-(-5)
i.e.
1-(-2) - -3-4 - -2-(-5)
i.e.
x+2 rl _ z+S
. A
3 -7 - 3
To find another point on
the line, let x+2
- - = ti z+S- = ><A,whence x = -2+3A, y
_ = - = 4-7A,
3 7 3
z = -5+3A and then take any arbitrary value(s) for A.

F. EQUATIONS OF PARALLEL LINES

The equations of the Iine through (x2,y 2,z2) paral­


lel .to the Iine with equations
x-x1 y -y 1 z-21 x-x2 y-y2 z-22
, are since
-a-= -b- = -c- -a-= -b-= c '
for the two paral lei Iines, the direction ratios
wi11 be proport iona I.

61
SUMMARY
Thus the equations of the line through (-1,2,-4) par·al-
x-3 y-1 z+6 x+l y-2 z+4
lel to the line given by - • • - are - • • --,
5 -3 -7 5 -3 -7

SUMMARY
I. The equations of the line through
0 and P1<x1,Y1,z1l 'Z.
(i) in parcunet:ric form are
x = tx1, y=ty1, z = tz 1
where *OP = t.*OP1,
(ii) in carteeian form are
� :'i.. :-f y
Xl y1 z1
X
2. The equations of the line through i
0 with given direction are tz
2} = 'i.. = .f, I
.c. m n
3. The equations tor translation of
axes to O 1 ( p,q,r) are 0
ok(f',i,� -Y
x = X+p, y = Y+q, z = Z+r. �
X IX
4. The parametric equations of the z.
line through P1(x1,y1,z1l and·
P2Cx2,y2,z2l are x = x1+t(x2-x1l,
y = Y1+t(yi-Y1), z = z1+t(z2-z1l,
where *P1P:*PP2 = t:(1-tl 0,,,____P,......,...........,""""''------
(><,,Y,,
i.e. *P1P:.*P1P2 = t: I. x y
5. The equations of the line through
P1<x1,Y1,z1l with given direction
are x-x1 - y -y 1 z-z1
--:r = - -- m = n
or - x-x1 = y-y1 = z-z1
0
a - -b- -c- X b
y
where l,m,n are direction cosines and a
direction ratios.
6. The equations of the Iine through
P1Cx1�Y1,z1l and P2Cx2,Y2,z2l are
x-x1 y-y1 z-21
-- =-- = --
x2-x1 y2-Y1 z 2-z1
X
7. The I ine through <x2,Y2,z2l
paral lei to the I ine
x-x1 y-y1 z-21
-a-= b- = -c- has equations
-

- - = -b- = -c -
a

62
SET 2C
EXERCISES SET 2C
1. A line L passes through the origin 0 and the point
P1 (5,2,3). �rite down the equations of the line OP,, and
determine the coordinates of the point Pa on the line
such that *OP2 = 2.*0Pi,
2. The point Q lies on the line OA where A has coordinates
(-4,1,-5). If *OQ:*OA = 2:3, determine the coordinates
of Q.
3. Find the equations .of the line through O with
(i) direction angles 60 ° , 120 ° , 45 °
(ii) direction ratios -1:3:4
2 -3
(iii) direction cosines 7ii;'• 7ii;'• 1
m· TTC lie
4. A line through the origin makes of 6,
angles with
3
the positive directions of the x, y axes. Find what
angle this line makes with the positive direction of the
z axis, and write down the equations of the line •.

5. The origin is changed from 0(0,0,0) to O'(p,q,r). A


point P in space has coordinates (x,y,z) relative to the
axes through O, and coordinates (X,Y,Z) relative to
parallel axes through O'. What are the relations between
x,p,X; y,q,Y; z,r,Z?
(i) What do the coordinates of A(2,-3,7) become when
the origin is changed to the point (-1,6,-4)?
(ii) If the point B(7,-l,-4) becomes (-2,3,-6) when the
axes are changed to O'(p,q,r), find the values of
p,q,r.
(iii) The point C on changing the axes to (3,0,-1) be­
comes (-2,1,-3). What were the original coordin­
ates of C?
6. Find the equations of the following surfaces, when
the origin is changed as indicated:
(i) 2x+3y-4z = l· (2,-5 -1)
(ii) (x+1)2 +(y+3)�+(z-6)� = 25; (-1,-3,6)
(iii) x 2 +y2 +z 2 -6x+8y-10z+25 c O; (3,-4,5)
(iv) z = x 2 -4x+y 2 +6y+l3; (2,-3,0).

7. Write down the parame·tric equations for the point


P(x,y,z) on the line through P1(x1,Y1,z1) and
P2 (x 2 ,Y2 ,z2 ) where *P1P:*PP2 = t:(1-t). Hence determine
the coordinates of the point dividin$ the join P1P2 in
the ratio stated.
(i) P1(3,2,-4); P2 (6,-l,2); 1 : 2 [Here t = 1/3]
2
(ii) P1(0,0,0); P2 (2,3,4); 2 : -3 [Here t
.. 2+(-3)
-2
63
= 11
SET 2C
(iii) P1(4,1,5); P2(2 ,-l,4); 3: 2
(iv) P1(2 ,5,4); P2(-6,3,8); -3 :5
8. Show that the points A(-1,-1,-5), B(3,-3,-4),C( ll,-7,- 2 )
are collinear. Hence determine the ratio *AB:*BC.
[Hint let *AB:*BC = t:(1-t).]

9. Find the equations of the line through P1 (2,5,4) with


(a) direction ratios (i) 3:1:� (ii) 3:-1:0 (iii) 3:0:0
2 -2 -1 . 6 3 -2
(b) direction cosines (i) , (ii) , ,
3 3, 3 7 7 7
° ° °
(c) direction angles (i) 135 , 1 2 0 , 60
(ii) a = 60 ° , S=l50 ° [First find y]
x l = v 2 = z= •
10. For the line � ; � state the direction ratios
and direction cosines. Sketch the line.
(i) Ry expressing the given equations in parametric
form, find the coordinates of three points on the
line.
(ii) Show that the line represented by the equations
x+7 , y+8 _ z-9
- i s identica l to the given line.
4 = 3 -2
11. For each of the fo llowing lines, specify the direction
numbers and find the coordinates of three points on the
line.
(i) -x+l = rl = z-3 (ii)
x+ l = rl
z =3
=s 3 -4- -5 3'
3x-4 _ y+3 _ 2 z+9
(iii) (iv) x=l+5t, y= -4+2 t, z =2-3t
7 - -1 - 2
12 . Determine the equations of the line passing through the
points P1, P2 as indicated below
(i) P1(1,- 2 ,5); P2(3,4,2)
(ii) Pi(-2,-4,3); P2(- l,-3,-2)
(iii) P1(3,2,5); P2(2 ,l,5) (iv) P1(0,2,0);P2(3, 0,- 2 )
13. (i) Find the equations of the line through A(5,- 2 ,7)
and B(l,4,3). Does the point C(3,1,5) lie on this
line?
(ii) Obtain the coordinates of the point on the line
· x-4 v+2 - z-l
· h equations
wit � - = where x = 5 .
3 4 - 2 ,

14. Obtain the equatio_ns of the lines through the point


P1 (- 2 ,-1 ,_3) parallel to
x-4 = y+3 = z-5
(i) the line
7 -2 1
1-x = 2y-6 = z+3
(ii) the line
-2 5 -4
x+4 rl
(iii) the line - ' z. = 6 (iv) the x axis
3 5
15. Determine the equations of the line through the point

64
SET 2C
C(4,-2,-5) parallel to the line through tbe points
A(2,l,-6) and B(-3,-2 ,1).

16. What are the di'rection ratios for. each of the lines re-
x-3 - y+5 _ z-4
presented by the equations - - _ - ,
3 4 6
x+-5 y+2 z-1
= = Hence show that the lines are parallel.
-6- 8 12.
17. Prove that the lines with equations ri _- z+5
x-7 z-1 3 -1'
v+B
- - = �= - - are perpendicular.
5 1
18. Find the direction cosines for each of the lines
�= tl= z+2 and x-4 = ill= z-5 and hence determine
3 6 -2 6 -3 6 '
the cosine of the acute angle 8 between the lines. Use
tables to calculate 8 in degrees and minutes.
19. Obtain the cosine of the acute angle between the.lines
1 x 2
/ =} = -} and � = �, z = o.

20. Find the value(s) of k for which the lines


x-4= �-= z-6 x 5
and - = .!::Y = � are
1 -k 3 k 4 6
(i) parallel (ii) perpendicular.

21. Show that the point A(2,0,3) is the point of inter­


section of the lines which join B(7,-6,l) to
C(l?,-18,-3),and D(l,4,-5) to E(3,-4,ll). [Hint: find
the equations of BC, DE and show A lies on both lines.)
Prove that the lines x-l -- rl - z-1 x+l .Y=1 - z-1
22. - - 6
1 -3 - -10' 1 1
intersect, and find the coordinates of this point of
intersection. [hint: Expressing the equations. in para­
metric form, we obtain x = l+A, y = 2-3A, z = 1-lOA and
x = -1+µ, y= 3+µ, z = 1+6µ. The lines intersect if
there exist values of A, µ for which the values of x,y,z
are respectively equal. Prove that A= -3/4, µ = 5/4.J
2 3. Determine the equations of the line Lthrough the origin
0 and wholly in the xy� plane, such that Lis perpendic­
ular to the line with equations x/7·= y/24 = z/60.
[Hint: let the line have equations x/l= y/m= z/n, why?
then show n= O and 7l+24m = O.]
-K24. Oatain the equations of the line through the point
(-4,5,3) and perpendicular to each of the lines
x-1 Y=1 z+6 x-4 _J_ z-3
-3- = = -3-· [Hint: let the equa-
2 ; s' _ _6_ : -4
4 5 z 3
tions be -;;-= Y::2.= -- and then ·show 3l+2 m+5n = O,
,{. rn n '
6l-4m+3n = o. J

65
THE SPHERE
SECTION 5. SURFACES AND PLANES
A. THE SPHERE

A sphere is the s urface generated by a point moving


in space, so as to a I ways remain a constant
distance from a fixed point.

In the figure, the poit;1t P(x,y,z)


moves so as to be always "a" units
from O. Thus P traces out a sphere,
centre O and radius a.
�ow l*GPI = /x2+y2+z2 = a.
Squaring, we have x 2 +y 2 +z 2
as tile equation of the locus of P, ><

Thus, the equation of the sphere, centre 0, and


radius a is x 2 +y 2 +z 2 = a 2 .

For example, the equation of the sphere, centre O, and


radius 3 units is x 2 +y2 +z 2 = 9, whilst in reverse, the equa­
tion x 2 +y 2 +z 2 = 16 represents a sphere, centre O, and radius
4 units.

{Note
If P moves in space so as to
be "a" units from
P1(x1,YI ,z 1) then the locus
of P is given by
l*P1PI = l(x-x1)2+(y-y1)2+(z-z1)2=a,
whence we obtain
(x-x1) 2 +(y-y1) 2 +(z-z1 ) 2 = a 2 , 0
y
X

Thus, the equation of the sphere, centre (x�,Y1,z1l


and radius a is (x-x1) 2 +(y-y1)2+(z-z1) 2 = a,

For example, the equation of the sphere, centre (-1,2,5)


and radius 7 units is (x+l) 2 +(y-2) 2 +(z-5) 2 = 49.
The equation x 2 +y 2 +z 2 -6x+2y-4z-ll =, O, which can be re­
written as (x 2 -6x+9)+(y 2 +2y+l)+(z 2 -4z+4) = 11+9+1+4
i.e. as (x-3) 2 +(y+l) 2 +(z-2) 2 = 25,
represents a sphere centre (3,-1,2) and radius 5 units.

66
PARABOLOID OF REVOLUTION
B. THE FORM z = f(x,y)
_In Analytiaal Geometry in 2 rH.mensions, we found that,
in general, the locus of an equation ·ofthe· form y = f(x) was
a curve.
Thus y = l, (x "# 0) is a ;1yperbola � '"-
; y = x 2-Sx+6
is a parabol: \I ; y = 2x+3 is a straight line
1
;1
and so on. �

Sometimes, the equation is written in the form F(x,y)=O.
Thus x 2 +y 2-9 = O is a circle, centre the origin, and radius

3 units� ; y 2-4x = 0 (i.e. y 2=4x) is a parabola �,


etc.�
0
R
Simi L�_J!!___ l',_n(!lytjf!a�. Geometry in 3 flimensions, we
find, in general, that the loaus of an eouation of the form
z = f(x� {or of the form F(x,y,z) = O} is a surfaae.
21:us, the equation z = 6-3x-2y
(i.e. 3x+2y+z = 6, which is an
equation of the first degree in
x,y,z) represents a plane. [It is.
sketched in the figure. Planes will 3
be treated in more detail in a y
later. section.]
--------
Also, the equation z=IY-x2-y2
represents a hemi-sphere above the
xy- plane (since z > O), centre 0
and radius 3 units. {Note that the y
sphere x 2+y 2 +z 2 = 9 can be re­
written as z 2 = 9-x i -y 2 i.e. as z = ±
Further, the equation z = x 2 +y 2
represents a paraboloid of revolu­
tion. (The figure is sketched.)
!'!!.c_�s exwrrple is extremely im­
portant.
To determine its shape, we
note the following points. y
X
(i) Since x 2 � 0 and y 2 � 0,
••• z l O, i.e. the surface
lies wholly above the
xy- plane, and touches it
only at the or·igin.
(ii) The surface is symmetric
about the ·z axis, and about
the zx- and zy- planes.
(iii) Now cross-sections of this y
surface by planes parallel

67
CYLINDERS
to the xy- plane are 1ti-rcl�t;.. [We can see this, by
noting that as z varies, the equation z x 2 +y 2 becomes
of the form x 2+y 2 = k 2 , and represents a circle in the
plane z = k 2 .J
(iv) Cross-sections of the surface J. /
/
by planes parallel to the
zx- plane are parabolas. /
/
[i.e. if we take a sertes of
planes parallel to the zx­
plane, then the sections are
all parabolas, but of differ­ y
ent sizes. Thus if we take
)(
the zx- plane itself, its
equation is y O and the equation z = x 2+y2 becomes
z = x 2 , i.e. a parabola in the zx- plane. If we- take
the plane y = 1 (which is parallel to the zx plane),
then the equation z = x 2 +y2 becomes z = x 2+1, i.e. a
parabola in the plane z = 1.J z
(v) Similarly, cross-sections of
·the surf ace of the parabolo_id
by planes parallel to the yz­
plane are parabolas. {The
actual surface itself has a
cup-shaped appearance,_but of y
course is not flat at the
bottom.} X

C. CYLINDERS

Any equa.tion of the form f(x,y) = 0, i.e. where the


variab·le z is missing from the equation, represents
a family of straight I ines para I lei to the z .axis.
The s urface thus generated is ca I I ed a oy Zinder.

For exCU71P- le, consider the equa.tion x2 +1/ � which


may be written as x 2 +y2 -25 = O. z
Here f(x,y) = x 2+y 2 -25.
In the fig�re, the circle
x 2+y 2 = 25 lies in the xy-
plane. Lines parallel to the
z axis are drawn; each line
intersecting the circle
x 2+y 2 = 25 in the xy- plane.
The point P10,4,0) lies
on the surface, since its co­
ordinates (3,4,0) satisfies the
equation x 2+y 2 = 25. X
A point P2(3,4,z) is taken on the line through P1(3,4,0)
parallel to the z axis. Now P 2 lies on the surface
68
CYLINDERS
represented by the equation x2 +y2 = 25, since its coordinates
(3,4,z) satisfy this equation. {This will be true for any
point on the line P1P2 , since z does not enter into the given
equation.}
Similaroly, if P3 lies on the circle x2 +y2 = 25, then any
point on the line through P3 parallel to the z axis, also
lie.s on the surface. 2
Thus, the equation x2 +y2 = 25, represents
in space, a rig.ht airculari aylinder. [The
·equation x2 +y2 � 25 only ·represents a circle,
when a cross-section thr,Jugh the cylinder
parallel to the xy� plane is taken.] 5 y
Other examples of cylinders are repre-
sented by the equations 2 z
2 2
(i) (x-2) +<y-3) = 1
I
This represents a I
right airaular cylinder
centred at C(2,3,0),
y
and with radius 1 unit.
{The equation of C(2,,.o)
the surface may also be
written as x2 +y1 -4x-6y+l2 o}.

(ii) (a) y � x2 (b) �Y- 4 -x 2


_:__.-----.- -: = :::::::.,....__,t.

'Y y
X
These are parabolic cylinders.

(iv) y = sin x
z

)(

This is a plane. This surface resembles


that of "corrugated-iron".

69
SET 20; FURTHER PLANES
EXERCISES SET 20
1. Write down the equation of the sphere
(i) centre O and radius 6 units
(ii) centre (3,-2,4) and radius 7 units
(iii) centre O and tangent �o the plane z "' 3.
(iv) with diameter the join of A(6,4,-5) and B (-4,o,-7).
2. State the coordinates of the centre and the length of
the radius of the spheres
(i) x2 +y2 +z2 = 100 (ii) (x-3) 2 +(y+1) 2 +(z-2) 2 = 49
(iii) x 2 +y2 +z2 -6x-10y+8z+14 a 0
3. Show that the spheres x2 +y2 +z 2 = 9, x2 +y2 +z2 -8x+4y+8z= 45
touch each other.

4. Make neat sketches of each of the following surfaces.


(a) x2 +y2 +z 2 = 1 (b) z = /1-x2-y2
(c) z = -11-x2-y2 (d) 4z = x2 +y2
(e) ?' 2 +y2 = 1 (f) (x-1) 2 +(y-4) 2 9
(g) x +y +2y-3
2 2
0 (h) 4y = x 2

(i) y x2 -1 (j) y x2 +1 (k) y = X


(l) y 2-x (m) y 2x (n} y log x
e
(o) y COS X

SECTION 6. FURTHER PLANES


�· EQUATIONS OF PLANES
(i) The Coordinate Planes
The equation of the xy- plane
is z = O (since the z coordinate of
every point on it is O, for varying
x, y.)
Similarly, the equation of the yz plane is x = O, and of
the zx- plane is y = O.
(ii) Planes Parallel to the Coordinate Planes
The equation of the plane z.
through the point (O,O,k) parallel
to the xy- plane is z = k. Similar­
ly, the equation of the plane
through A(-1,0,0) parallel to the
yz- plane is x = -1.

70
EQUATIONS OF PLANES
(iii) The Plane t/ = a:i:+b
We have already seen in the previous z
section that a linear equation of the form
ya ax+b represents a plane, which is
parallel to the z axis, and
to the xy- plane,

The plane with


equation y = 2x-3 is -
shown in the sketch� -a

(iv)

{This is called the no"f'lrlaZ form for the equation of a


plane,} 2
Proof. In the sketch, OP1 is
the normal from O to the plane
S1, Take any point P(x1y1 z) on
the plane and let I *OPi = d units.
The angle a between the
lines O�l• OP is given by
cose = .t'..l'+mm'+nn', where l,m,n
and l',m',n' are the direction
cosines of OP1 and OP respect­
ively. X
x
Now since i.' = · ' m' = Y n' = z
d d' d
the result for cose = l. + m {
lx+mv+nz
d (1)
�o_!,)ever, since the triangle OP1P is right-angled at P1,
cose = .E. • • . • • • • • • .
• •. • •.
• • • • •.
• • •.
• • • . . • • (2)
d
lx+mv+nz .E. i. . e. -l-ox+my+nz = p
Thus, from (1), (2), d = '
d
Henae, the locus of P has equation lx+my+nz = p, and
since P traces out the plane S1, the equation of_ the
plane is .tx+m_y+n�= p. [Note: by convention p > .
o ]

EXAMPLE I. The perpendicular from the or1g1n Oto a


plane 51 (i) has direction angles 120 ° , 45 ° , 60 ° ,
and is of length 5 units.
(ii) meets .it in the point P, (5,-3,6).
Find the equation of the plane 51 in each case.

71
EXAMPLE ON EQUATIONS OF PLANES
Method.
(i) We use the "normal" form lx+my+nz = p,
where .t = cos 120 ° "' -\, i.
° 1 °
m = cos 45 = 72, n = cos 60 = \,
and p = 5.
The required equation of S1 is
- � + 7iJ y + \z 5,
X
i. 1:. x - /iy - z -10, on multiplying by -2.
z
(ii) Here the direction ratios for
the line OP1 are
(5-0):(-3-0) :(6-0)
i.e. 5:-3:6.
Thus, the corresponding di­
rection cosines are
5 -3 6
p' p' p
no.
X
where p = j *OP,I = 15 2+(-3) 2 +62 =
Hence in the equation i.x+my+nz = p, we have
1-- -�
v70' m - � n - �
- v70' - v70' p -- ffoo •

-k k k
Thus the plane S1 has equation
X - Y + Z = n0 i.e. 5x-3y+6z ]0.

EXAMPLE 2. A plane S2 has equation 6x+2y-3z+8 = O.


Determine the perpendicular distance of S2 from 0,
and the direction cosines of the normal to S2.

Method. We arraqge the equation 6x+2y-3z+8 = 0 in the form


i.x+my+nz p, noting that p > 0 by convention, and that
£2 +m 2 +n 2 1.
Thus 6x+2y-3z+8 = 0 can be written 6x + 2y - 3z = -8
and thus --6x-2y+3z = 8 • • • . • • • • • • • • • . • • . • . • • • • • • • • • • • • . (1)

Now divide both sides by l(-6) 2+(-2) 2+(3JZ, i.e. by 7.


The equation (1) becomes
6 2 3 8
- X - y + ] z = ]"
] ]
llence, on comparison with the·
form fx+my+nz = p, we see that
f = - .§.
7' ·
m = -17' n = l7 y
[note £ 2 +m 2 +n 2 = l] and p

72
GENERAL EQUATION OF A PLANE
8
The plane S 2 is units from O;. the direction cosines of
7-6 -2 3
the normal to S 2 are , , .
7 7 7
(v) General Equation of the First Deg�e Ax+By+Cz = D

The general equation of the first degree, namely


Ax+By+Cz = .D, represents a plane.

Thi8 can be easily seen, since if D > O, the equation


Ax+By+Cz = D can be written as
A B C D
x + y + -;::;: :::::::;==� z
IA 2 +n 2 +c 2 IA 2 +B 2 +c 2 IA 2 +n 2 +c 2
On comparison with the 'normal' form of the equation of
a plane, namely lx+my+nz = p, then Ax+By+Cz = D represents a
D
plane, distant units from the origin O, and the
IA 2 + n 2 +c 2 A
==
direction cosines of the normal to the plane are
-l;:: :;+=n::;2 +c:;2 ,
A 2
B C
;:::; ::;==:;
/A 2 +n 2 +c 2 ' - ::::
l. A 2 +B 2 +c 2

Note. The normal to the plane Ax+By+Cz = D has direction


ratios A:B:C, since the numbers· A,B,C are proportional to the
A B C
direction cosines , , ---;:::::::::== of the
IA 2 +n 2 +c 2 IA 2 +n 2 +c 2 IA 2 +n 2 +c 2
normal to the plane.
Thus, for the plane 5x-3y-7z = 10, the dire·ction ratios of
the normal to it are 5 : -3 : -7.

SKETCHING OF PLANES
To quickly sketch a plane of the form Ax+By+Cz = D, we
may (i) convert it to the 'normal ·' form lx+my+nz = p_
or (ii) find the equations of the lines in which the given
plane cuts the 3 coordinate planes. These are called
the "traqes" of the ·given plane on the other planes.
The first method has already been illustrated. The sec-
ond method will now be indicated. �
For ex<J!!.lPJ:!!..., consider the plane
S1 whose equation is 2x+3y+4z = 24.
The plane S1 cuts the xy- plane,
where z = O, in the line 2x+3y = 24,
(which cuts off intercepts 12, 8 on 8
the x, y axes respectively.)
Also, S1 cuts the yz- plane,
where x = O, in the line 3y+4z 24
(which cuts off intercepts 8, 6 on
the y, z axes).
73
PLANE THROUGH A GIVEN POINT
Similarly, the line of intersection of the plane S1 and
the zx- plane has equation 2x+4z = 24.
z.

-----
The plane 3x-2y-5z = 30 is
also sketched.
{Note that these figures -Y 0
do not mean that a plane is -,s y
triangular in shape. They
merely indicate the shape of
the plane as it crosses the co­ -6
ordinate planes.}
1-z.
(vi) The Equation of the Plane S through the
point P1 (X1..1L1....&.1) where the normal to S has given
direction nwnbers A, B. C.
Now the general equation of the plane is Ax+By+Cz= D... (1)
If this plane S is to pass through P1, then the coordinates
(x1,Y1,z1) must satisfy S. Hence Ax1+By1+Cz1 = D.
Thus , substituting for D in (1), we have
Ax+By+Cz = Ax1+By1+Cz1
i.e. A(x-x1)+B(y-y1)+C(z-z1) = O.

EXAMPLE I. The equation of the plane through


(-5, 1,-6) where the normal to the plane has direc­
tion numbers 3,-2,4 is 3(x+5) - 2(y-l) + 4(z+6) = 0
and this reduces to 3x-2y+4z+41 = O.

EXAMPLE 2. Find the equation of the p I ane v1h ich bi-


sects the join of P1(6,-l,4) and P2(2,3,5) at right
angles.

Method. The coordinates of the


midpoint V. of the interval P1P2
are {l,(6+2), \(-1+3), \(4+5)}
i.e. (4,1,9/2).
The direction ratios for
P1P2 are 2-6 : 3-(-1) 5-4
i.e. -4 : 4 : 1.
Henoe the equation of the
plane is -4(x-4)+4(y-l)+l(z - 9/2)

SUMMARY
I. The general equation of tile first degree in x, y, z,
namely Ax+By+Cz = D represents a plane. The direction
ratios of the normal to the plane are A, 8, C .
74
SUMMARY; SET 2E
2. The speci a I cases of Ax+8y+Cz = D,
(a) z=O (the xy- plane); when A=O,
C= I, D=O
y=O (the xz- planei, and
x=O, the yz- plane. x---=--:-.-===
(b) Equations of the {orm z = 3 (which Isa�
plane para I lei to the xy- plane)
through (0,0,3); and so on.

(c) y = A 1 x+B 1 represents a plane


parallel to the z axis, and
perpend ic u I ar to the xy-
plane.

2.
(d) The 'norma I ' form lx+my+nz = p
where the direction cosines
of the normal are l,m,n and
the plane is distant p from
0.
(e) The equ_ation
A(x-x1)+B(y-y1)+C(z-z1> 0
which represents a plane through (x1,Y1,z1>; the
direction ratios of the normal being A,B,C.

EXlRCISES SET 2E
1. Make neat sketches of the following planes
(a) z = O (b) z • 2 (c) y = -3 (d) y "'-X
1
(e) y = 3-2x (f) 3X -3Y+3z ., 4
2 2

(g) 3x+4y+2z • 12 (h) x -2y-4z "' 6


2. Find the equation of �he
(a) yz- plane
(b) plane parallel to the xz- plane and 5 units to the
right
(c) plane perpendicular to the z axis at the point
(0,0,6) only.
(d) horizontal plane through the point (2,3, -7).
(e) plane parallel to the z axis , and cutting off inter­
cepts 2,-3 on the x,y axes respectively.
(f) plane, distant 2 units from O, the direction angles
of the normal to it being 120 °, 60 ° , 135 ° .
(g) plane, whose perpendicular distance from O is 5
units, and cosa • 1/2, cosa"' 1/3, y is acute,
[First find cosy,]
(h) plane t.hrough P1(2,3,1), where P1 is the foot of the
normal to it through 0.

3. Express the equation 4�7y-4z z 27 in the normal form


.lx+my+nz • p, and hence state the values of l,m,n.p.
Verify that !2.t-m2+n 2 • 1, and sketch the plane.
75
SET 2E; PARALLEL AND PERrENDICULAR PLANES
4, Calculate the length of the perpendicular from O to the
plane S1 whose equation is 3x-6y+2z+24 = O. Also deter­
mine the di_rection cosines of the normal to ·s 1,
5 ,· For the plane x-y-2z+6 = O, determine
(i) the direction ratios and direction cosines of the
normal
(ii) the intercepts on the x,y,z axes
(iii) the equations of the lines in which the plane
meets each of the· coordinate planes.
(iv) length of the normal from O to the plane.
6. Repeat question 5 with the planes
(a) x+2y+3z = 0 (b) 3x+4y·= 60 (c) X - 3 0

7. N is the point (- 6,3,6). Determine the equation of the


plane through N perpendicular to ON.
8. Find the equation of the plane
(i) cutting off intercepts 4,5,-2 on the x, y, z axes
respectively
*(ii) through the points (2,1,3), (-1,-2,4), (4,2,1)
[Hint let the plane be Ax+By+Cz = D. Thus in (i), the
points (4,0,0), (0,5,0), (0,0,-2) lie on the plane.
Then find A,B,C in terms of D.)
9. Find the equation of the plane through (-2,5,4) where
the normal to the plane has
(i) direction numbers 1,-3,-7
. . . 9 -2 6
(ii) d1rect1on cosines , ,
11 11 11
10. Find the equation of the plane bisecting at right angles
the join of the points
(i) A(-3,7,-4), B(-1,3,2) (ii) P(S,1,-6), Q(-6,-3,2)

B. PARALLEL AND PERPENDICULAR PLANES


We now investigate the conditions for the planes
A1x+B1y+C1z = D1 and S2 : A2x+B2y+C2z = D2 to be
(i)· parallel (ii) perpendicular.
(i) Para llel Planes F d.�
A,:8,:C,
In the figur�, S1,S2 are parallel
planes, FP1, GP2 are normals to S1,S2,
It is obvio�s ( '.:at if the planes are s
parallel, so too are their nonnals.
Now the normals to the planes
A1x+B 1y+C1z = D1, A2 x+B2y+C2z = Dz
have direction ratios A1:B1:C1 and
Az:Bz:Cz respectively.
The con,lition for the normals A1 B1 C1
(and thus the. planes) to be parallel is that - = - = -.
A2 Bz Cz
76
POINTS IN SPACE
In Three-Dimensional
"" .;;..;. .;;.. Analytical Geometry, space is divi­
ded in_t_o;;;....;8 r.;;..e_g_io_n_s_,..;.__c_a_l_l_e'""d-o-c"'t-a-n-t-s-,-b-y-
r.1_e_a_n...
s of th'I'ee axes -
the x, y, z axes which are taken so as to be mutually at
right angles to each other.

x'

-
/

Y' y
y

X
z'
The 3 planes (the plane xCy or xy- plane; the plane
xOz or xz- plane; the plane yOz or yz- plane) are called the
coordinate planes.
Let P(x,y,z) be any point in space, and through P be
drawn planes perpendicular to the three coordinate axes. (In
fig (ii), the planes FPDA, PFBE, PDCE are perpendicular to
the x, y, z axes respectively. That is, the plane FPDA is
parallel to the yz- plane, the plane PFBE is parallel to the
xz- plane and the plane PDCE i� parallel to the xy- plane).
The displacements OA, OB, OC are x,y,z respectively (x, y, z
may of course be positive or negative).
[To reach the point P from the origin O, we proceed
x units along the x axis from O to A, then y units parallel
to the y axis from A to F, and then z units parallel to the
z axis from F to P. This order is always preserved, i.e. x, y
then z.)
Thus, to designate a point in space, it is necessary to
use coordinates of the form (x,y,z). The positive directions
of the x,y,z axes are indicated by the arrows. Check that the
coordinates of A are (x,0,0), B(O,y,O), C(O,O,z), D(x;O,z),
E(O,y,z), F(x,y,O).
NOTES.
�Probably the most difficult feature about Analytical
Geometry in Three Dimensions is to draw a figure to il­
lustrate the exercise. Much practice is necessary to
accomplish t his successfully.
Although it is very awkward in many cases to obtain a
figure, you should try to visualise the figure. (If
possible, portray this by a reasonable sketch.) Useful
practical aids. in this work are pencils (to represent
lines); sheets of paper (to represent planes); the
room in which you are working (to represent

37
DISTANCE BETWEEN 2 POINTS
a rectangular prism); and a corner of the room (to re­
present the 3 mutually perpendicular axes).
2. In drawing figures, it is usual to use equal units on
the y, z axes but to use a shorter unit on the x axis.
Also the x axis is taken at an angle of 135 ° to the
y axis. These features give
' a sense of perspective to
3-dimensional drawings.
f In some texts, the x, y axes are interchanged. }
3. For convenience, the complete rectangular prism is not
usually drawn; merely the 3 eclges OA, AF, FP. Thus the
points P1(2,3,4), P2 (-l,2,-3), P3(-l,-2,4) would be re­
presented as shown.
z
P,(i,H) I
/-X
I I
A/ z I
4 ---"---,. F

'{
I

�(-t,2,-3) X

�· DI STANCE BETl'/EEN mo PO I NTS


Case 1. Distanae bet1Jeen O and P (x 1 , y1 , z1 )

In the figure, since P1F is


perpendicular to the xy- plane, P, (x., \I., 2,)
then P1F is a norr.1al to this
plane.
*P1FO .. 90 ° . B
In 6 OAli, right-angled at y
A, l*OFI = r,
I
• I
• • r 2 = x1 2 +y12 by Pythag­
oras' Theorem •.••••••.....• (1) F
In 6 OF�, right-anglecl at
F, j *OP1 j = d

• • d 2 = r 2 + z1 2 , by Pythagoras' Theorem.
Hence d 2 = (x1 2 +y12 )+z12 , using (1)
Thus d = lx/+y12+z12 = l*OP1I

EXAMPLE The distance d from the origin


0 to the point P1(6,-2,5) is given by
y
d Ix l2+y I2+z l2
/62+(-2)2+52 = 165 units.

38
PARALLEL AND PERPENDICULAR PLANES
Henae, the planes A1x+B1y+C1z D1, A2x+B2y+C2z D2
A1 B1 C1
= =

are parallel if-=


A2 B2 C2

Thus the equations 2x-3y+4z 13, 6x-9y+l2z = 7


represent parallel planes (since -
6 = - the direction
a

-9 12 '
2
-3 4
= -)·
ratios of the normal to each plane being 2: -3: 4.

EQUATION OF A PLANE PARALLEL TO A GIVEN PLANE

The equation of a plane parallel to the plane


Ax+By+Cz = D is of the form Ax+By+Cz = K, where k
is a ·constant.

EXAMPLE I. Find the equation of the plane through


the point P, :a,6,-7) para I lei to the planel
S1 x+2y+5z = 20

The required plane has equa­


---tion x+2y+5z = k. ·Since this plane.
Method ?.

passes through the point


4

P1 (B,6,-7) then 8+2(6)+5(-7) = k,


whence k = -15.
Thus, the require? plane has
equation x+2y+5z = -15.

(ii) Peryendiaular Planes


In the figure, the planes
S1,S2 are perpendicular to
each other.
FP1, GP2 are normals to
the planes. It is P.asily seen
/

that these normals meet -at


/

right angles. [Remember that


/

the direction of these normals B


is fixed, not their positions.] A,.x+ ��D� �
Henae the lines FP1 (with
direction ratios A1,B1,C1) and GP2 (with direction ratios A2,
- ;I.

B2, Cz) are perpendicular to one another.


The condition for this is t hat A1A2+B1B2+C1C2 • 0 (See
Section 3).
Thus the planes A1x+B1y+C1z = D1, A2x+Bzy+C2z = D2
are pe_rpendicular to one another when A1A2+B1B2+C1C2= 0

77
ANGLE BETWEEN TWO PLANES
EXAMPLE. Show that the planes 3x-4y+7z = I,
2x-9y-6z 8 are perpendicular to each other.

Method. Here (3,2)+(-4,-9)+(7,-6) = 6+36-42 • O.


��-Thus, since the condition A1A2+B1B2+C1C2 • O is satis­
fied, the given planes are perpendicular to each other,
{The direction ratios of their normals are respectively
3:-4:7 and 2:-9:-6.}

C. ANGLE BETWEEN TWO PLANES


In the sketch, the planes S1,S2
intersect along the line EF. The
angle between the planes S1, S2 is
defined as the angle e [or (n-0)],
If Pis any point in space, and
PR, PT are normals to the planes,
then the angle between these normals
is also e. [This can be seen from
the quadrilateral PRGT,)

Henae, the angle between two planes is the angle


between the normals to these planes.

Now if S1, S2 have equations A1x + B1Y + C1z � D1,


A2x+B2y+C2z 2 D2 respectively, then their normals have
direction ratios A1:B1:C1 and A2:B2:C2, The angle 0 between
these normals, (and thus that between the planes) is given by
cose = !1!2+m1m2+n1n2, where !1,m1,n1 and !2,m2,n2 are the
direction cosines of the normals. [See Section 3,]
A1 B1
Here !1 = i 2 2+c1 m1 = i 2 2+c12 etc.
A1 +B1 2• A1 +B1

Thus, the angle, e , between the planes

{Note Lhat if the planes are at right angles, e = 90 ° and


cose = 0, The result above then gives A1A2+B1B2+C1C2 = O,
which is the condition previously determined for the planes
A1x+B1y+C1z = D1, A2x+B2y+C2z = D2 to be perpendicular to
each other.}

EXAMPLE Find the acute angle between the planes


6x+3y-2z = 4, 4x-5y-3z = 9.

78
ANGLE BETWEEN A LINE AND A PLANE
Method. A1A2+ B1B2+C1C2
Now cos6 - I z
A1 +B1 Z+c 12 .I A2Z+B2Z+c22
(6.4)+ (3.-5)+(-2.-3)
.. /6Z+3z+(-2)2,/4Z+(-5 )Z+(-3)2
15 3 3v'2
=� .. Vz-14
From cosine tables, 6 + 72 ° 22 1 ,

D. THE ANGLE BETWEEN A LINE AND A PLANE

The angle made by a I ine RT


in space with a plane S1, is
defined as the angle between
RT and its projection R'T on
the plane, i.e. ljl,
11_ -
T
In the sketch, the angle iJ,
between the line RT and the plane
S1 is the complement of the angle TT
6 between the line RT and the normal RR', i.e. iJ, =
2 - 6,
No�J, let the equation of the line be given in the form
x-x1 y-y1 z-z1
-- - -- - -- (where the direction ratios of the line
a1 bi ci
are a1:b1:c1), and the equation of the plane be
A1x + B1Y + C1z Di (where the direction ratios of the
normal are A1:B1:C1),
Henae cos e = l i2+ i2+ 2,;A
a b c1 12+81z+c12
From this result, we can obtain the acute angle e, and
TT
thence, since ljJ = 0, we can determine ljJ
2 -
[Alternatively, since iJ, =
a1A1+b1B1+c 1C1
e, I- sin iJ, = cose

Le. sinljl = lai2+bi2+ci2,/Ai2+Bi2+ci2' whence we can calculate


iJ, directly.]

EXAMPLE. Find the inclination to the plane


6x+3y-2z 4 of the I ine joining the points
P1(3,-l,4), P2(7,-6,I).

Method.The direction ratios for the line are


--(7-3) : (-6+1) : (1-4) i.e. 4 : "'.'5 : -3.
The direction ratios for the normal to the plane
6x+3y-2z = 4 are 6 : 3 : -2.
79
SUMMARY; SET 2F
4 . 6=)+ ( -5= . 3)+ ( -3 · -2)
Henae cose sinljJ° = -;::;( :; ::;; ;:=:;::::;;::;­
/42+
;::
( 5)
2+(;::
:;:- ;::;;:
-3) 2:--,-; 2+
./6:;; 32+ ( -2)2
;=::;;
,---+-+---,
=
15
T":fl'i = m 3
=
. 3./z
14·

Thus, from sine tables, w � 17 38 ° 1



/
/

/Pll (1, -E., 1)

SUMMARY
I. The planes A1x+B1y+C1z = 01, A2x+B 2y+C2 z = 0 2
< i J are para I lel if A1:A 2 = B1:B2 = C1:C2
(ii) are perpendicular if A1A2+B1B2+C1C2 = 0
2. The plane Ax+By+Cz = K is para I lei to the plane
Ax+By+Cz = 0 for varying va I ues of K.
3. The angle between the
(i) planes A1x+B1y+C1z = 01, A 2x+B2y+C2z = 0 2
is given by cose = A1A2+B1B 2+C1C2
IA1 z+s1z+c1 z ./A 22+B22+c22
(ii) line with direction ratios a1:b1:c1 and the plane
A1x+B1y+C1z = O is given by
a1A1+b1B1+c1C1
s inw =
./ai 2+b12+c1 2 ,/A12+B12+c1 2

EXERCISES SET 2F
1. Prove that the planes
(i) 4x-7y+3z = 2, 8x:-14y+6z 3 are parallel
(ii) 3x+4y+2z = 7, 6x-5y+z = 8 are perpendicular.
2. Show that the plane x+2y-3z = 9 is perpendicular to each
of the planes 4x+7y+6z = 1 and 2x+5y+4z+3 = 0.
3. Prove that the line
x-) ''"'°l z-5
( i) whose equations are�=�= ---=i is parallel to
the plane 8x+12y-4z+31 = 0.
(ii) passing through the points P1(3,5,-2),P 2 (0,0,2) is
parallel to the plane 8x-4y+z = 9.
[Hint in (i), (ii); a line is parallel to a plane, if
the line is perpendicular to the normal to the plane.]

4. Prove that the acute angle e between the planes


2x-y+z = 7, x+y+2z 11, is given by cose =�.and hence
find e in radians.
5. Find the. cosine of the acute angle between the planes
3x+2y-5z = 2
(i) \l2x-3y+5z �i) 53x+2y-z = 6 (iii) �·3x+4 y =16
= 7 ( lx+4y-z = -1 L 4y- z= 5
6. Find the value of A for which the planes
80
REGIONS
2x+3y-4z = 8, 5x+4Ay-10z = 1 are
(i) parallel (ii) perpendicular.
x-4 = .Y=1. = z+3
7. What angle does the line
7 4 4
make with the
plane x-2y-2z = 8?
8. Find the angle between the line x=2t, y=2+t, z= -3+2t and
the plane 2x+y+2z = 3.
9. Determine the angle of inclination of the line joining 0
to the point (2,2,1),and the plane 2x-3y+6z = 11.

10. Find the equation of the plane through the point


(i) 0 and parallel to the plane 5x-7y-8z = 3
(ii) P1(3,-5,6) and parallel to the plane 4x+2y-7z 9.

SECTION 7. REGIONS
In two dimensions, a region is defined as the set•of
points in a plane, which sat.isfy an inequality.
·Thus (i) _lx_i__:_1_ isthe set of points -·x�_,,,. --..--0 ----+
3 0 3 X
for which -3 < x < 3.


-Y
(iii) x 2 +y 2 < 1 defines the interior
of the circle.
)(

(iv) x 2
� y ��defines the I

region shaded, on and


between the semi-circle
and the parabola.

0 I X
Similarly, in three dimensions. a region is defined as
the set of points in spaae, which satisfy an inequality.
z.
Thus (i) lzl � 3 is the set of
points in space for
which -3 � z � 3,
i.e. the region be-
tween the planes
z = -3, z = 3 and in-
eluding these planes.
81
REGIONS; SET 2G
z
(ii) lxl < 1, iyl < 3, lzl < 2
defines the interior of
the rectangular prism
shown. Y

(iii) y .5 2 - x defines the


plane shown, and the
\\
origin side of the plane.i}._ y

(iv) x 2 +y 2 +z2 .'.: 4 defines the


interior and surface of
the sphere, centre 0 and
y
radius 2 units.

(v) x 2 +y 2 .5 1, O .S z S 4 de­
fines the interior and
surf ace of the cylinder
shown. y
(vi) y > x , 0 < y < 3,
2

defines the interior of


the parabolic cylinder.

y
(vii) z � x 2 +y 2 , 0 .5 z .S 3
defines the interior and
surface of the paraboloid
of revolution (i.e. a
cup-shaped region).
y

EXE RCISES SET 2 G


Sketch the regions defined by the following inequalities
(a) y < 2 (b) IYI .5 2 (c) lxl < 2' IYI < 2, I z I < 2
(d) !xi .5 3, I YI .s 4, I zl .s 5 (e) y S 3-x (f) y > X
2 +
(g) x y +z .'.: 25
2 2 (h) 0 < z < 12S-x -y
2 2 2 2 .5
(i) x +y 4, Q.5z.S3
(j) (x-1) 2 +(y-4) 2 .'.: 9, -1 .5 z .5 2 (k) x 2 < 4y, 0 < y < 2
( i) .S 4z, 0 .s z S 2
x 2 +y 2 (m) (x-2) 2 +(y+3) 2 +(z-1) 2 < 16
(n) x y +z 2 > 9 • I zJ > 2
2 + 2 (o) y .5 4 - x 2 , IYI .s 4

82
SET 2H
(p) 2x-Sy+3z .5 30 (q) x 2+y 2+z 2 .5 9, x 2 +y 2 .?. 4
2 + 2+ 2 < 16
(r) 9 < x y z (s) 9 .$ x 2+y 2 .$ 16, 0 .$ z s 2
(t) z � x 2+y 2 , 1 < z < 3 (u) x 2+y 2+z 2 .5 1, x+y+z 1 1
(v) 2x+3y+4z· .$ 12, X � o, y � o, z � o.

FURTHER EXERCISES ON LINES AND PLANES


SET 2H

A sketch 11ill be found useful in many of the


fol lowing exercises. This· sketch need not be drawn
exactly in space. The main ide� of the sketch is to
help you to understand the problem and to assist
in the solution.

1. (a) Find the equations of the line through P1(1,-3,4)


and perpendicular to the plane x-Sy +2z 8.
[If.int: the required line will be normal to the
plane.)
(b) Similarly, determine the equations of the line
through P2 (5,-2,-4) meeting the plane 3x-7y +oz = 9
at right angles.
2. Find the equation of the plane through A(2,-3,l),
(i) the direction ratios of the normal.to the plane
being 5 : 1 : -4.
x-3 v-4 z+l
(ii) normal to tlie line - - =
1
= �·
5
(iii) parallel to the plane 7x-4y-10z = 8
(iv) perpendicular to the join of P(-4,-2,7) and
Q(3,-1,2).
3. Determine the equation of the plane which is the perpen­
dicular bisector of the segmen .t joining A(2,3,l) and
B(6,-3,�).
4. Find the equation of the plane
(i) parallel to the plane 3x-6y-2z-8 = O and distant
3 units from the origin O.
(ii) if the foot P1 of the normal to it from O has co­
ordinates (2,-1,-3).
(iii) parallel to the plane 2x-6y+8z = 7, for which the
sum of the intercepts on the coordinate axes is
1.
5. Find the equation of the plane of the family
3x-ky+2kz = S, which
(i) passes through the point (1,-1,2)
(ii) is perpendicular to the plane 2x-3y+4z 9.
83
SET 2H
6, Show that the planes 2x+y-2z+4 = O, 2x+y-2z-6 = 0 are
parallel, and find the distance from O to·each. Hence
determine the perpendicular distance between them.
[Hint: Is the origin on the same or opposite sides of
the planes?]

7. (i) Determine the coordinates of the point where the


line joining the points (3,8,10), (4,6,6) pierces
the plane 2x-3y+Sz = 8.
(3.�. io)
[lff11._t: Show that the equations
of the line are
x-3 = ..1.:.§. zTlO
= t (say)
-1- _2 = �
and thus that x = 3+t,
y = 8-2t, z = l0-4t•••••••(1) ( 4,6,6)
Now at the point A, where the line pierces the
plane, these coordinates satisfy the equation of
the plane. Hence prove that t 2, and that A has
coords (S,4,2). J
(ii) Similarly find the point where the line
x-2 � z+4
pierces the plane 2x-3y+Sz = 8.
-2- = 3 = ---="s
8. Determine the coordinates of the point where the line
joining the points A(l,-3,6), B(3,-2,S) pierces the
(a) plane x = 0 (b) zx- plane (c) plane x+2y+3z = 8.
9. Find the point where the line x = -1+3t, y = 2-t, z = 4+3t
pierces the plane Sx-3y+z = 14.

10. P, Q are the points (5,-6,S), (2,0,8) and the line PQ


meets the plane 4x-3y-2z = 4 in R. Determine
(i) the equations of PQ in. parametric form
(ii) the coordinates of the point R
(iii) the direction ratios of OR and PQ, and hence
prove that these lines are at right angles.
x-2 = 2Y-2 = z-S the
11. (a) Show that the line lies in
10 11 7
plane 3x-8y+2z = 8.
[�t'!_t: find two points on the line, and prove these
lie in the plane. Then note that if a plane con­
tains 2 points of a line, then it contains the
line.]
(b) Similarly, prove that the line 1<x-2) = -(y+2) = \(z-3)
lies in the plane 2x+2y-z+3=0.

12. (a) Find the coordinates of the point of intersection


A of the planes 2x+y-z = l; 3x-y-z = -2; 4x-2y+z = 3.
[Hint: the point A must satisfy these equations
simultaneously.]
(b) Similarly, find the point where the planes x+y+z = 1,
2x-3y-Sz = -11, 3x+2y-4z = 6 meet.
84
SET 2H
13. A, B ar e the points (-2,-1,2), (7,5,5) respectively.
Determine the coords. of the point P on 'Ai such that
*AP:*PB = 2:1. Find the equation of the plane through P
perpendicular to the line!�.
14. Show that the following planes
(i) 2x-3y+.4z = 5, x+2y-z = 3, 4x-6y+8z = 2, 3x+6y-3z = 1
enclose a parallelepiped.
(ii) 7�4y-4z = 9, 24x-34y+8z 1, 14x+8y-8z -15,
12x-17y+4z = 3 enclose a rectangular prism.

15. (a) Find the equation of the plane through the line of
intersection of the planes 3x-4y+2z =: 6-, 2x+4y-2z = -7
and through the point (1,2,3).
[Hint: let the required plane have equation
(3x-4y+2z-6) + A(2x+4y-2z+7) = O.]
(b) Similarly, find the equation of the plane through
the line of intersection of the planes x-3y = 4,
2y+5z = 2, and througli the point (0,1,3).
16. The two planes x+4y-2z 8, 2x-y+z = 6 meet in a line L.
Determine the
(i) point on L where z = 1
(ii) point where the line pierces the yz- plane.
[Hint: x = 0 then,]
(iii) direction numbers of L [Hint: use the points found
in (i), (ii).]

17. Find the equation of the plane through


(i) the points (-3,1,-2), (1,-2,2) and perpendicular
to the plane 2x+y-z+6 = O. [Hint: let the plane be
Ax+By+Cz = D, and show that -3A+B-2C D,
A-2B+2C = D, 2A+B-C = 0. Then express A, B, D in
terms of C.]
(ii) the point (l,0,-2) and perpendicular to the planes
2x+y-z = 2, x-y-z = 3.
[Use a similar method to (i).]
(iii) the point (-2,-2,3), and containing the line
joining the points (1,-1,2), (1,1,1).
(iv) the points (l,0,-1), (3,2,2) and parallel to the
x-1 v-1 z-2
line - - = � = --.
1 3
(v) the point (1,-2,3) and parallel to the lines
x-3 _ .Y.±!. _ z-4 x+l _ Y::1_ _ z+4
- -
2 - 1 - -1' 3 6 -2
18. Find the equations of the line through the point
(-2,3,4) parallel to the planes 2x+3y+4z = 6, 3�4y+Sz= ll.
[Hint: let the equations be x+2 -_ .1=1 -_ z-4, and show
a b c
that 2a+3b+4c = O, 3a+4b+Sc = O,]

85
SET 2H
19. A point P(x,y,z) moves according to the following con-
ditions. Find the locus of Pin each case. Interpret
the.Zoaus geometriaaZZy.
(i) P is equidistant from the points A (-3,-2,-1),
B{7,0,5).
[Hint: show (x+3) 2 +(y+2) 2 +(z+1)2= (x-7) 2+y 2 +(z-5) 2,
Note that this is an alternative method of deter­
mining the equation of the plane bisecting the
interval AB at right angles.]
(ii) *OP2 +*AP 2 = 16, where O is the origin and A the
point (0,2,0).
(iii) The square of the distance of P from the z axis is
always four times the distance of P from the
xy plane.
(iv) J *PA J : J *PB J = 2: 3 , where A, B have coordinates
(-2,2,-2), (3,-3,3) respectively.

86
CHAPTER 3

�PPLICATIONS OF CALCULUS__T_g_ !�E __PHYSICAL WORLD

MOTION IN A STRAfGHT LINE

SECTION I, DISPLACEMENT, AVERAGE VELOCITY


A particle moves in a straight line X'OX, so that its
displacement x '" , from the fixed point o, is given by
the formula x = x(t), where t is the time measured in se­
conds, and x(t) means that x is a fur.ction of t.

0.�X X
X = x(t)
Notes (i) \-le ccr:sider x to be positive, if the displacement is
from left to right i.e. -- ,
and consider x negative, if the dis­
placement is from right to left i.e. �
(ii) The term displacen1ent refers to signed distances;
the term distcmce is considered signless,
i.e. distance = lcisplacementl.
(iii) Til.'1e is considered from zero time, i.e. '.,Jhen t � O.
[Negative values of t are sometimes taken to indi-
cate time before zero time. For example, let us
consider the running of a race; at some stage
during the race, we press our stopwatch to indicate
zero time t = O. A negative value of t, say t = -3
may here indicate what occurred 3 seconds tefore

X
the stopwatch 11as pressed. He>wever, in our work,
only values of t� 0 are valid values of t,
i.e. values of t� 0 are considered meaningless.]
(iv) The positive and negative directions for horizontal
motion are a matter of convention; unless

I
specified otherwise, these will �e adopt-
ed ,,ere. For vertical motion, the usual
0
convention is that positive displacements
are measured from O in an upwards (t) di-
rection. X
87
DISPLACEMENT
EXAMPLE The particle P moves horizontally and
2
obeys the law x = x(t) � 7+3t - 2t , where x,t are
measured in_�_ seconds res ectivel
(i) To obtain the displacement of the���-+-������
0 X
particle P from O at a certain X'
time te1. (say), we substitute t = to( in x = x(t), and
obtain the displacement x o1. (say) as (7+3t,,c:-21;} )
Thus, when t o, Xo 7; when t = !,, x = 7+3(!,)-2(!,/ 8
!,
when t <i, x� 1+3(G)-2<i!{ 65/8;
when t = 1, x, 7+3(1)-2(lf 8
when t 2, X,1. 7+3(2)-2(2/ 5;
when t 3, x! 7+3(3)-2(31 -2
when t 4, x '+- = 7+3(4)-2(4)� -13 and so on.
Hence, initially (i.e. when t = 0), the particle Pis 7 u1to
the right of O, then proceeds to the right ( �) for a time
(actually G sec, when X = 8� m ;,the significance of this par­
ticular value and how it was derived will be apparent later).
It then reverses direction and proceeds to the left(�) pass­
ing through Oat some time between 2 and 3 seconds. The
displacement of Pfrom 0, x, x+ox .IIl , after times t,
t+ot sec. respectively are given by
x = (7+3t-2t1') and x+&'x = [7+3(t+d't)-2(t+d'd m
(ii) The progress of Pmay be shown as follows
t=4- t-3
- t•a t•I
: t•O:ia•i

-1
(; �12 �8
< �4 : :a b 1 1a 13 14- 5 16 70,
(iii) Another method of representing the � xa7•3t-2i"
progress of P is by a sketch using 7 :
x - t axes (instead of y - x axes), 1
noting t � O. This graph indicates + /
how far from O the particle is at t
different times; it does not mean O f 1 I
that the particle f ]lows a parabo- 1
lie ;::>c.1th (in actual fact, P moves -4 I
in a straight line). I
(iv) To obtain the time when Pis a cer- -
tain distance fmm O.
For excurrr, le -1i !
<�) if P is 2 _mto the left of O, i.e. x -2, then for some
t (real or unreal), 7+3t-2t
2
= -2, i.e. 2t?- -3t-9 = O,
-3
i.e. (2t+3)(t-3) = O, whence t = 3(t f
by convention).
2, since t � 0
Thus after 3 sec., the particle is 2 mto the left of O.
(f,) if Pis 2 mfrom O, then x ='12)
On solving 7+3t-2t2 = ! 2, we find t = l for (x "' +2),
2
and t = 3 (for x = -2).
88
AVERAGE VELOCITY
Thus the particle is distant 2 m from O after 2� sec and
also after 3 sec,
If Pis 10 mto the right of o. then x= +10,
i
On solving 7+3t-2t� = 10, we have 2t -3t+3 "'O, whence
2
the roots are unreal (b -4ac "' (-3l-4(2) (3) � O).
i.e. P never reaches a point 10 m t0 the right of O.
(v) Av�rage Velocity of P
(o<) In the above example, where x "' 7+3t-2tl , the par­
ticle P covers a distance of (8�-7) i.e. ii m during
the time interval t "'0 to t= J, i.e. in,� sec,
Thus P covers l} m in � sec. i.e. an average of
-4 "'
4
1} m per. second of time. !-!e say then that ?
has an average velocity over this time interval t "' 0 to
t "' fi, of l! m per second, (often written as 1! m /s)
(f!,) If a train covers a journey of 15 k,m in 20 minutes
(i.e. 1/3 hour), then its average velocity for this
15

Jour,1L!Y ·
1s = 45 k m per I 10ur. Th.1s c! oes not mean
11 3
that the train is moving steadily at 45 km/h for every
instant of the journey, but rather that in 20 minutes,
it would cover 15 j(m It would be certainly true to
say that the train must attain a velocity of 45 km/h
at some time during the journey.
(¥) The velocity of P at so�e instant in time (sometimes
referred to as the instantaneous velocity of P is de­
fined as the rate of change of the displacement,
(this displacement being measured from O of course).
The units of velocity are cm /sec, k m/h etc. Since
displacement can be both positive and negative, then
the velocity can be ':ooth ?Ositive and negative. A
positive velocity indicates motion in �direction for
horizontal motion and in t direction for vertical
motion. [A zero velocity indicates that the body is
momentarily at rest, not necessarily at O.) The term
"speed" denotes the absolute value o; the velocity,
i.e. speed= lvelocityl.
l
(o) For x "' 7+3t-2t , when t= 1, x= 8 and when t = 4,
x= -13. :'.ence the averGf}e veloci t!J of P during the
time interval t "' 1 to t= 4 is
the displacement of P for this time interval
interval of time
_1=11)�
- rr./sec i.e. -7 m/s
4_1
[Tr.e negative sign indicates that p is t ravelling in
4-- direction during the time t ; 1 to t ; 4.]

( (.) The average s!Jeed of p during the 4th second.

equal to <I-13) <-22


� I
i.e. during the time interval t ; 3 to t
= 11 rr1sec.
4 is
[Note when t ; 3,

89
EXERCISES SET 3A
x = -2 and when t = 4, x= -13],
The distance covered by Pduring t!1e first 3 seconds,
i.e. during the time interval t = O to t = 3 is equal to
l<Bi-7)J + l-2-81 j = llJ m. [Note: check these figures
from the sketch above.] Hence the average speed during
11J.
the first 3 seconds = �= 3a m/sec.

(71) Tlie displacement in m of P during the time interval


t = t to t = t+d't [assuming that there is no cl".ange
in tte direction of P during the time interval) is
equal to .l
i
[7+3(t+cft)-2(t+Sd J-[7+3t-2t l = J(at)-4t(lt)-2(dt)
en simplification.
Hence, the average velocity during this time interval
3(<ft)-4t(<lt)-2(cl't)� • l _ _ 1.r, '1 m /sec,
(t+ot)-t l3 4t 2 ,ot).J

DIRECTED EXERCISES SET 3A


A particle Pmoves horizontally in a strai ght line so
that its displacement x cm from a fixed point O after t sec.
is given by the following equations.
1. X= c1 +Jt
(a) Find x when (i) t = 0 (ii) t = 1 (iii) t 2
(iv) t= 3 (v) t = T+dT
(b) Find when Pis (i) lOcmto the right of 0
(ii) Jcmto the right of O,(as a surd)
(iii) lOcmto the left of O (iv) 4cmfrom 0
(c) Sketch the progress of P for the first 5 seconds,
by the two methods illustrated above.
(d) Determine the displacement of P for the
(i) time interval t = 2 to t= 5 (ii) third second
(iii) first four seconds (iv) time t= T to t= T+dT
(e) Calculate the average velocity of Pfor the
(i) time interval t 2 to t = 5
(ii) third second (iii) first four seconds
(iv) time t = T to t T+fT.
2. X = 2t' -Bt-7
(a) Find the displacement of Pfrom 0
(i) initially (ii) after 3 seconds
(b) Determine the distance of Pfrom O at the end of
(i) 2 sec. (ii) 5 sec.
(c) Calculate at what time Pis
(i) 17 crn to the right of O (ii) 10 1h cm to the
left of O (iii) 17 cm on the negative side
of 0,
(d) Show that Pis at tt.e origin after (z+./7-s) sec.
(e) Assuming the :.iody changes direction after 2 sec,
sketch the progress of Pon (i) a number llne
(ii) graph of x against t [for O � t � S]
(f) Find the displacement of Pfor (i) the first

90
EX£RCISES
second (ii) the time interval t = 3 to t= 5
(iii) time interval t=t to t = t+h, where t, t+h..:2
(g) Calculate the average velocity for the time inter­
vals specified in (f).
(h) Determine the distance covered by Pduring
(i) the fourth second (ii) first 3 seconds
(i) Find the average speed of Pfor the times stated in
(h).
3. X = sin t, 0 :!: t .S 2rr
(a) Find x when (i) t= 0 (ii) t = f(/6 (iii) t= ff;4
(iv) t = n/2 (v) t =J.11{!> -

'f
(b) At what times is P (i) J� cm to ri_ght of 0
(ii) c'll from O (iii) at O?
(c) Calculate the average velocity of Pfor
(i) first
1T
j f
seconds (ii) the time interval t= to
t=
4 [Assume that the particle changes direc -
.
tion l an d t= JTT; 1eave answers in
w l1en t =1T
2
terms of irrationals.)
(d) Sketch the progress cf Pusing 2 different methods.
Ver.ify that Pcomes back to the origin twice in the
time interval given.
(e) Determine the average speed of Pfor (i) first TI sec
( ii) the time interval t = to t= 5 f
1T'
6
T4. x=tan t/2, O�t"TT
(a) Find the displacement oi P
2ff
(i) initially (ii) at the end of� sec.
(b) When is P 1 cm to tLe right of O?
(c) Determine the distance traversed by P during the
first tr/2 sec.
(d) Find the average velocity of P for the tiMe inter­
val t = Tf/3 to t = 2TT/3.
(e) What happens to the particle as t _,, rr?
(f) Sketch the progress of P (i) on a number line
(ii) on a x - t graph.
5. x= logtt, t > 0. [Xatural logarithm and ex , e-x tables
are needed in this question.)
(a) Find the displacement of Pfrom O after (i) 1 sec
(ii) e sec (iii) 1/e sec (iv) 10 sec
(v) 0·0001 sec. Explain what happens to Pas t __. O.
(b) Determine when Pis (i) 3 cm to the right of 0
(ii) 2 cm to the left of O (iii) 1 cm from O
(iv) at O.
(c) Calculate the distance covered by Pduring the time
interval (i) t = 2 to t = 6
(ii) t = e-3 to t = e-2 (iii) t=t to t=t+J't
91
EXERCISES
(d) Find the average speed for each of the intervals
specified in (c).
(e) Sketch the x-t graph of P, and verify that P is
initially an infinite distance to the left of O, is
at O after only 1 sec, and proceeds to an infinite
distance to the right of O with slackening speed.

6. (Tables of ex , natural logarithms are required)

(a) Calculate x when (i) t = O (ii) t = 1 (iii) t = 3


(iv) t = !,.

(b) Determine t when (i) X 1 (ii) X 3 (iii) x=-


3
(iv) x = e1 .

(c) Explain why the particle is never at the origin 0,

(d) Find the average velocity of P for the time interval


(i) t = 1 to t = 3 (ii) t = t to t = t+h.

(e) Sketch the x - t graph of the progress of P; verify


that P is initially 1cmto the· right of O, and trav­
els away from O with ever-increasing velocity.

1
7. (a) X
= t' t > 0 (b) X = / 25-t" ,

For each of the above equations, find


(i) the displacement of P from O after (e() 3 sec
(A) S sec <,)�sec
(ii)- at what time the particle is (�) 3 cm to right of O
(ft) 1 cm from 0
(iii) ·the average velocity of 1' during the fourth second.
Sketch the progress of P on separate x - t graphs,

92
VELOCITY AS A DERIVATIVE
SECTION 2. VELOCITY AS A DERIVATIVE
JL>X.(�)
><' X
o A I
Xs x(t +Gt) t,
Consider a particle P moving in a straight line X'OX so
that its displacement in cm. from O after t sec (namely *OA)
is given by x = x(t) {i.e. x is a function of t}.
After (t+6t) sec, its displacement (i.e. *OB) is given
by x+6x = x(t+6t). {i.e. (x+6x) is a function of (t+6t)}
Thus the displacement (*AB= ox cm) covered by P for the
time interval t1 = t to t2 = t+ot is given by
6x = x(t+ot) - x (t); assuming that the particle does not
change. direction in the time interval.
Hence the average velocity of P during the time inter­
val t1 = t to t2 = t�t
(x+ox )- x ox x(t+ot)-x(t)
(t+6t)-t 6t = at en. per sec.

As ot decreases, the average velocity of P does not necessar­


ily decrease in magnitude, but this average velocity is over
a smaZZer interval of time, (i.e. the at sec at the end of
t sec. ) [To illustrate this idea, consider a body travelling
at a constant velocity of say 30·UVsec. Then in � sec, the
body covers 15 m :md in 1/10 sec it covers 3m and so on.
Nevertheless, although the time interval and distance covered
both decrease, the velocity of the body remains constant
. . 15 3
i.e. the ratios T]2 • l/lO are unchanged • 1
l.'enae, as 6t approaarres zero, the averagf:! velocity of P
becomes an average velocity over an infinitesimaZZy smaZZ in­
tervaZ of time(namely the at seconds at the· end of t seconds.)
Thus as at- O, this average velocity becomes the instantan­
eous velocity v of P at time t, or more simply the
velocity of P at time t, i.e. at the end of t sec.
lim 6x dx
Hence v = ( ) [This is also often written as x .
at-+-o 6:t dt'
Similarly y• -- .QY
dt')

DEFINITION The velocity v, of a particle moving in


a straight I ine, at the end of time t is given by
I im dx I im x(t+otl-x(tl} = ·
V (�) { X
6t -+-0 6t dt at-+-Q 6t

EXAMPLE I. The displacement (xm) of a particle P


from the origin O at the end of t sec is given by
5
x = ' Find from first pri.nciples the velocity
(Zt+I)
v mls of P at the end of t sec.

93
VELOCITY

lim
Method.
ox lim lx(t+ot)-x(t)J
liy definition, v = ( ) where

l
ot +o ot ot+o.._ ot
x(t) = - - 5
2t+l
lim r 2(t�c5t)+l - 2�+1
i.e. V
ot+oL ot

lim [ -lO(cSt) ]
cSt + 0 eft[2t+2(c5t)+l] [2t+l]
lim [ -10 J -10
ot +O [2t+2(ot)+ll [2t+ll (2t+l)2
Thus the velocity of Pafter 5 sec is -10/(2t+l) 2 m Is

EXAMPLE 2. For the equation x = 7+3t-2t2 , defining


the motion of a particle P,
X

Show that the velocity vm/sec of P at the end of t sec


is given by
(i)

lim [0+3(t+c5t)-2(t+6t) 2 }-{7+3t-2t 2 }1 = (3-4t)


6t + 0 l 6t
The initial velocity (i..-. 11hen t=O) is (3-4xO) i.e. 3m/sec;
after 5 sec, its velocity is (3-4K5) i.e. -17m/sec.
(ii)

Hence initially Pis moving to the right (i.e. --) at


3m/sec, but after 5 sec it is travelling at 17m/sec to­
wards the left (i.e. -<---- ).
(iii) Pis at rest (for an instant), when v = O, i.e. when
3-4t = O, i.e. when t = 3/4. Thus Pis momentarily sta­
tionary after 3/4 sec. (Its displacement then from O,
given by substituting t = 3/4 in x = 7+3t-2t2 , is
7+3(1)- 2(1) 2 = .22. m )
4 4 8
that -3-, v > O, and when
v = 3-4t; when t =
3 + 4
(iloting
t = , v < O. Hence for the instant before t = 1, P
4 4
is moving in -- direction, and an instant after t = f,
Pis moving in-+-- direction. Thus, P ahanges direation
when it aomes momentarily to rest.}
(iv) The particle is moving with velocity
(a) lm/sec in -,., when v = +l, i.e. when 3-4t = 1,
i.e. after 1 sec.
(S) 9m/sec in-+-, when v = -9, i.e. when 3-4t = -9,
i.e. after 3 sec,
(v) The velocity of P at the end of 5, 11 sec is (3-4x5),
(3-4xll) i.e. -17, -41 m/sec respectively. The change
in the ve_locity of P over the time interval t = 5 to
t = 11 is -41-(-17) = -24rn/sec, i.e. there is an in-

94
EXERCISES SET 38
crease of 24rr/sec in+-- direction in the velocity of P
over a time interval of (11-5) = 6 sec. This consti­
tutes an avc.�age increase in the veZoaity of Pof 24/6
i.e. 4 m I sec. We say then that Phas an average aaae Z­
eration (in+--) over the time t= 5 to t= 11 of
4�/sec per second of time (often written as 4rr/sec 2 ),
(vi) The average acceleration (in +--) over the time t1 = t
to t 2 = t+ot is given by
J {3-4(t+ot) }-{3-4t}J + ot 4m/sec 2 .
(vii) The graph of v :gainst t is shown.

EXERCISES SET 2B
Obtain , from first principles, the velocity of a point P
whose displacement x m from the fixed point O after t sec is
given by
1. �= 3t+7
H�nce show that the velocity of P is a constant
i.e. independent of the time.
2. �2t 2 -8t-7
Then find
(i) the velocity of P (a) initially (b) after 2 sec
(c) after 5 sec.
(ii) when P comes to rest and how far Pis then from O.
(iii) at what time is Ptravelling at (a) 6m/sec in-
(b) 6r.i/sec in -<- (c) 4m/sec,
(iv) the difference in velocity of P over the time
interval (a) t= 1 to t= 7 (b) t =t to t=. t+ot.
(v) the average acceleration of Pover the times spec-
ified in (iv). Sketch the graph of v against t .
3. X = 3/t, t > 0
(i) Hence find the velocity of Pafter
(a) 2 sec (b) � sec.
(ii) Show that Pnever comes to rest.
(iii) When is the velocity of P (a) 3tr./sec in<-­
(b) 12Q/sec in ---4 ?
(iv) Determine the average acceleration of P over the
time interval (a) t= 1/3 to t = 1
(b) t= t to t= t+h
(v) Sketch the v-t graph for P, and verify that Pis
always travelling away from O t owards-<- with
diminishing veloci•ty.
t lim e 0t -1
4. lL.'.:....!_, given that
cS t -+O
(
� "'
) 1

95
ACCELERATION AS A DERIVATIVE
Find (i) (a) the initial velo city of P
(b) the velo city after 2•5 se c .
(ii) when the velocity is (a) e m/sec in ---'>
(b) 2m/se c in --
(iii) the displacement of P from O in each case of
(ii).
(iv ) the change in velocity and the average ac cel-
eration of P for the (a) first second
(b) time t = 2 to t = 4.
Sketch the v-t graph for P, showing that the.
particle moves away from O with ever-
increasing speed in the positive direction.
5. x = sin t, 0 � t � 2n, noting that
. S+T . S-T
sinS - sinT = 2 cos � - sin and that
2 2
lim sin(6t/2)
1
6t 7 0 (6t/2)
n
Find (i) the velocity of P (a) initially (b) after )
4
sec (c) after lln/6 sec.
(ii) when the velocity is (a) \� .m /sec in ---+

(b)
13 m /se c in
(iii) when P is at rest, and its displa cement then
from 0,
(iv) the average a cceleration of P for
(a) the first n/2 se c (b) the time interval
t = 2n/3 to t = n,
Sketch the v-t graph for P, showing that P changes di­
rection twice during the time interval stated.

SECTION 3. ACCELERATION AS A DERIVATIVE


�l) 0
�+-����-+--'-��+-7����---+�
0 A v+lv � v(t +/t)
Consider a particle P moving in a straight line so that
after t se c , its velo city,v cm/se c is given by
v = v(t) {i.e. v is a function of t}.
After (t+6t) sec, its velo city (v+6v) cm /sec is given by
v+6v = v(t+6t),
Hence the in c rease in veloc ity of P for the time inter­
val t1 = t to t2 = t+6t is {(v+6v)-v} = {v(t+6t)-v(t)} c m/sec.
Thus the particle accelerates by 6v cm/sec for this time
interval 6t sec; i.e. the average acceleration of P for this
t+6��-v t cm
ct sec at the end of t sec is �: { v( ( )} /se c
per second (i.e. cm /sec 2 ),
6v
As 6t dec reases, is taken over smaller and smaller
6t
inter vals of time (but of course, the average ac celeration
need ,ot dec rease). Hence when 6t-* O, this average acceler-
FORMS FOR THE ACCELERATION
ation becomes the instantaneous acceleration of Pat the in­
stant when the time is ·t sec. This instantaneous acceleration
i
i.e (��) is often referred to as the acceleration of P
. 0! :o
after time t. [The units of acceleration her.e are cm /sec 2 ;
other units used are km per ·hour per hou r, i..e.km/h2 • The
acceleration f of a body Pmay be positive or negative or
zero. If f > o, P is accelerating in-> direction; if' f < 0,
=
Pis accelerating in +--tlirection; if f O, Pis moving with
uniform speed, since Pis then neither accelerating or decel­
erating. The terms "deceleration" and "retardation" are used
to denote the fact that the acceleration is against the mo­
tion of P, i.e. that the body is being retarded or is decel­
erating (i.e. slowing down)].

DEFINITION The acceleration of a particle, moving


in a straight Iine, at time t is given by
= lim � _g_y lim {v(t+ot)-v(t)}
f ( ) V
ot-+O ot dt ot-+O ot

OTHER FORMS OF THE ACCELERATION. (To be used later)


dv d d dx d x [ + d2x .
Now �(v) = ( ) ; dt2' • N1 o. e dt2' 1 s o ften wr ·tten
dt dt dt
I

as x. Similarly � = i/]
dv dv dx dv • V = V dv
Also =
dt dx dt dx dx
dv dv - {
Further � = v = f(2v 9.Yi = 1 .Q_ (v 2 )}
dx dx 'dx
9.Y d2x ..
Thus f = - = = x = v -9.Y= _sl,(l v 2)
dt dt2' dx dx , ·

EXAMPLE. A particle'P .is travel I ing with a velocity


�m-/s·ec at the end of t sec; where v = tan t,
0 � t < rr/2. Derive from first principles (i.e. by
the definition) the acceleration fw/sec 2 of P at
time t.
Solution. By definition,
dv = lim {v(t+ot)-v(t)}

�\j
f
dt ot -+O ot sin(t+ot)
-
lim {tan(t+ot)-tan t} = lim cos(t+ot) cos t
i.e. f {
ot-+o ot ot-+O ot
lim f sin(t+ot)cost - cos(t+ot) sint l
6 t -+0 l 6 t.cost.cos (t+ot) 1
lim � sin[(t+ot)-tj 1 using the result for
ot -+O lot. cost.cos(t+ot) J• sin(A-B)
lim {sinot _l_ 1
.
6 t-+ 0 6t cost cos(t+ot) }
1 1 . lim --sine
1 • not ing - 1 ) =sec 2t.
· cost · cost ( e -+ o 8
Thus the acceleration of P
· after t sec is sec 2 t m/sec 2 .

97
EXERCISES SET 3C
POINTS:
�e acceleration of P (i) initially is sec 2 o = 11n/sec 2
in-+ direction (ii) after f
sec is sec 2 ; = (73)
4
0
2 2
2 in--.
-·m./sec
1T 3
Since sec 2 t > 0 for O � t < - then the acceleration is
2'
always in-----> direction.
(b) The acceleration is 4m/sec 2 in. -- when f = 4 i.e. when
sec 2 t .'1, i.e. when cos 2 t "' ':; which yields cos t = ±'-;;,
1T
and thus t -- � [Since O $ t < J. Thus Pis acceler-
2
3
atin� at qm/sec 2 in -----> after i 3·1416
(� -- - = 1"047 2)sec
3
(c) Pis at rest when v = 0, i.e. when tan t = 0, i.e. when
t = 0 (for O � t < n/2). The acceleration of Pthen is
sec 2 o 1�/sec 2 .
[Note that although v = 0 when t = C here, f f. 0 then.
Likewise, if f = 0 for a certain particle, then\ I
it does not necessarily mean that v = 0 at the I
same time; J al -•t
I
I
I
(d) Graph of f against t is shown. I
I I
I I
0 • ,y• w/, iri'/.1. 11
EXERCISES SET 3f.
Derive from first �rinciples (i.e. by the definition),
the accelerationfcm/sec of a particle P moving with velocity
v ::.u:,Jsec at the end of t sec, where
1. V = 5+7 t
Hence show that the acceleration is ccnstant, namely
7cfr'isec 2 . This means that P increases its velocity by
7cw/3ec for every second of motion. Sketch the graph of
(�J f against t, (b) v against t; each for O $ t $ 4
2. V = 7 + 6t-2t 2
Then find
(i) the acceleration (a) initially (b) after 3 sec
(ii) when the body is neither accelerating nor deceler-
ating, and the velocity of the body then.
(iii) at what time the acceleration is (a) Scm/sec 2 in -­
(b) Scn/s€.c 2 in +- .
(iv) the velocity of P when the body is being retarded
at 6crrfsec l . Sketch the graph of f against t, for
0 $ t $ 6.
3. V = {t+22-
1
Hence determine
(i) f when (a) t 0 (b) t = 1 (c) t 98
(ii) t when (a) f -1/25 (b) f = -·01
(iii) (a) v when f -1/16 (b) f when v 1/9
98
DIRECTED EXERCISES SET 3D
Sketch the (i) f-t graph (ii) ·v-t graph of P for O!t!6,
Verify that Pis always travelling in the positive di­
rection, and is decelerating throug�out the motion.
1
r4. v =- -+l!
2t+l 9
Hence show that Pis
(i) decelerating at 16/9 cm .lsec 2 initially, but after
4 sec, it is accelerating at 16/81 crn/sec 2 . What
are the corresponding velocities of P?
(ii) moving with uniform velocity (i.e. f = O) after 1
.sec, and that this velocity is 5/9:cm/sec.
(iii) travelling with a velocity of 29/45 cm /sec after
2 sec, and find its acceleracion then.
5. v = cot t, 0 < t < rr
Then find
(i) the velocity and acceleration of Pafter
(a) rr/6 sec. (h) Jrr/4 sec.
(ii) the velocity when the acceleration is -2•cm/sec 2 .
(iii) the acceleration when the velocity is
1 1
(a) TJ cm/sec in - (h) TJ cm/sec in +-
(iv) the acceleration when Pis momentarily at rest.
Sketch on separate diagrams the graph of (a) v against t
(b) f against t.

SU11/��APY. It a particle moves in a straight line


X'OX so that its displacement x ft trori, 0 - after
t sec is given by x = x(tl, then the
(il velocity (v ft/sec) a+ time t is given by
·dx
V = dt = X

(ii) acceleration (t tt/sec 2 ) �t time t is given by


_ dv _ d 2 x _ dv _ _Q_ , 2
t v dx - ( v l = x
- dt - cit2' - dx ,

:,[CT Ioi.J 4. vr· LOCI TY, .ACC[ LrRATIQI·� AS rmIVAT IVES


DIRECTED EXERCISES SET 30
;JGTE: The following directed exercises should be wor�ed
through by you; they illustrate i7':f! ortant features of the
topic. The usual conventions regarding x, t, v, t are
intended; x, t are �easured in m , sec unless otherwise
s ecitied.
EXAMPLE I. A particle p moves accordirg to the law
X = t1 -6t+8.
dx = 2t 6,
Noting that V = dt - and t = dv = 2, answer the
cit
followinc. (Each resuIt s:iould be in terore ted t, OU,)

�. From x = t 2 -6t+8
(i) Show that P is ir,itially 8,m on the positive side
of O, and after 3 sec it is lmon the negative side
99
DIRECTED EXERCISES
of 0,
(ii) Find its displacement from O after
(a) l sec (b) 2 sec (c) 5 sec.
(iii) Show that the distance covered by P during t'.,e
time interval t = 4 to t = 6 is la - ol m·, and
hence the average speed of P for this time is
8/(6-4) = 4uifsec.
(iv) Determine the displacement of P during the first
second, and the average velocity for this time.
(v) Calculate wten Pis (a) 3m to right of 0
(i.e. solve t2 -6t+8 = 3) (b) 4m to right of O
(c) lm to left ·of O.
(vi) Show that Pis never 3111 on the negative side of 0
(i.e. show the equation t2 -6t+8 = -3 has unreal
roots.)
(vii) Find when P
(a) passes the origin (i.e. solve t 2 -6t+8 = O)
(b) returns to its starting point (when t=O, x= 8;
solve t 2 -6t+8 = 8 ).

3. From v = 2t-6
(i) Show that Pis moving initially at Em/sec in
� direction, and that after 7 sec, it is trav­
elling at am/sec in� direction.
(ii) Find its velocity after (a) 2 sec (b) 4 sec.
(iii) Show P comes to rest (for an instant) after 3 sec
(i.e. solve v = O), and find how far it is then
from 0. Noting'v = 2(t-3), show that v changes
sign as t passes through the value 3. Hence P
changes direction after 3 sec, i.e. when v = O.
(iv) Find at what time the velocity of P is
(a) 1 m :;sec in ---+ (i.e. solve 2t-6 = 1)
(b) 1 m /sec in +--- (i.e. solve 2t-6 = -1).
(v) At what times is the speed of P 3m/sec?
(i.e. v = i3). What distance is Pthen from O?
(vi) Prove that the velocity of P can never reach 7m/sec
in +-- direction.
(vii) Show that P increases its velocity from 2 m/sec
to 12 m Vsec over the time interval t = 4 to t = 9,
and hence the average increase in velocity of P
. . 12-2
per> seovnd over th.is interva 1 is _ i.e. 2 m I sec.
9 4 .
That is, the average acceleration of P over this
time interval is 2 m Vsec per second, i.e. 2m/sec�
[Actually, since f = 2, the acceleration of P is
constant at 2m/sec 2 J.
100
DIRECTED EXERCISES
(viii) Repeat (vii) for the
1
(a) fifth second (b) time interval t to t = 1
2 2'
(ix) Find the displacemel'.t of Pfrom O when the velocity is
am/sec in the positive direction.
(x) Calculate the velocity of Pwhen Pis 24mto right of 0,
(xi) Determine the displacement and the average velocity of P
for the time interval t = 3 to t = S.

_g_. From the above information, verify the sketch below (on
a nwnber line) of the progress of F.
t •66
I
V'• a.
) �t.• +
•� tr·•
4
) e (�t.·!I'
2.
)e
�V'•
f • 2.
)

-I O I 12 3 14- 5 16 17 8 'q ""x


r1;1e effect of f = 2 can be seen from the sl·.etch. Initially p
is Sm to right of O and moving at 6m/sec in+ direction, but
since f = +2, i.e. f acts in -+ direction, then for time t=O
to t=3, P slows down, coming momentarily to rest 1 m to
left of 0. It then reverses direction, now proceeding--+ ,
and since f is in - direction, Pspeeds up, passing its
starting point at 6m/sec. Afterwards it continues to in-­
crease its velocity at the rate of 2m/sec per second.]

Q. The x-t graph X


B
(i) The path of }:' is a
straight line; this 6
sketch does not in­
dicate that P moves 4
on a parabolic curve.
It records the dis­ 2
placement of P from 0
for values of t ,!e O. 0
(ii) The gradient of the -t
tangent at any point
(t, x) on the curved
x = x(t) is given by d: i.e. v = 2t-6. [Compare this
with the tangent at (x,y) on the curve y = f(x) where
the gra d.1ent .1s given
. by � ]. Here the gra d'1ent v has
dx
no dimensions.
dx
(iii) Thus,since when t = 1, V = 2(1) - 6 = -4; the
dt =
gradient of the tangent at the point R(l, 3) on the
curve X = t2 -6t+8 is -4. Hence the equation of the
tangent to the cu rve at R is given by x-3 = -4(t-1)
i.e. 4t+x = 7.
(iv) At L, the tangent to the curve is parallel to the
• dx
taxis, .1.e. = v O. Thus a stationary point
dt
101
DIRECTED EXERCISES
(where v = 0) on the x - t curve represents a position
of zero velocity for P, i.e. where P is momentarily at
rest.
(v) Find the equation of the tangent (a) where t=4 (b) at L
to tile curve.
d2x =
(vi) Now f represents the curvature of the curve
dt 2
X = t 2 -6t+8 at any point (t, X) on it.
d 2x
Since = 2 > o, the curve is always concave up-
dt2
wards (as can be seen from the sketch.)

f.. THE v - t GRAPH V'


6
(i) This sketch records the veloc­
ity of P for t � O_; it does
not indicate the path of P.

(ii) Since the velocity graph is


the straight line V 2t-6, 0 ) 4 5 6
then the gradient of the line
at any point (t, v) on it is (t,") civ- K f •2
c,.-i..•cit
dv
given by dt = f = ? [Compare
the equation v = 2t-6 with the -i)
equation y = 2x-6.J 7 7
(iii) The shaded area = f v dt J<zt-6)dt [t2 -6t)3
3 16 sq. units.
dt :I
s·i.nce dx = v, then x V dt J<zt-6) clt = t2 -6t+C
IIence the displacement during the time interval t=3 to
t=7 is (72 -6.7+C) - (3 2 -6.)+C) 16 m
Thus the shaded area represents the displacement of P
during the time interval t = 3 to t = 7. (This idea
will be discussed further in the next section.)

F. THE f - t GRAPH
This sketch records the acceleration
of P for t� O; since f = 2, �1e graph of
f against t is a straight line parallel
to the t axis. o 3 6 t

EXAMPLE 2. A body P moves according to the law


x = (t-l)(t-lO)(t-25).
(i) Show that x = t 3 -36t 2 +285t-250, and thus
v = 3t2 -72t+285 = 3(t-S)(t-19), f = 6t-72 6(t-12)
(ii) Find the initial displacement, velocity and accelera­
tion of P.
102
DIRECTED EXERCISES
(iii) Calculate the displacement, velocity and acceleration
of P at the end of (a) 2 sec (b) 11 sec.
[Substitute in the factorised forms of x, v, f.J
(iv) Determine the change in displacement of P during the
first second, and hence obtain the average velocity of
P for this period.
(v) Find on how many occasions, P (a) passes the origin
(i.e. x = 0) (b) changes direction (i.e. v = O)
(c) has uniform velocity, i.e. is not accelerating or
decelerating (i.e. f = 0).
(vi) From (v) above, calculate the
(a) acceleration of P when it is last at the origin
(�) displacement of P when it first changes direction
(c) velocity of P when it is moving with uniform' veloc-
ity.
(vii) Determine the change in velocity of P over the time in­
terval t = 18 to t = 20, and hence find the average ac­
celeration for this interval.
(viii) Show that on two occasions the velocity of P is 4Sm/ sec
in-direction, and calculate the displacement of P on
each occasion.
(ix) Show that only once does P have an acceleration of
3Cn�sec� and find the velocity then,
(x) Verify that the progress of P on a horizontal straight
line is as indicated t, 25

t•I
X•O
V•2i6
• -n f. -66
(xi) Study the sketch above, and follow through
(a) the progression of time from t = 0, and the corres­
ponding displacement of P from 0,
(b) the change in velocity of P fr<;>m its initial pos­
ition (taking into account the sign of v, and thus
the direction of motion).
(c) the acceleration or retardation of P and its effect
on. the velocity.
(xii) Verify the following sketches of x against t, v against

,
t, f against t. [These graphs do not indicate the path
of P.)
(a) 41

x-t
graph·" X (t)
(t-l)(t-lO)(t-25)

103
DIRECTED EXERCISES
1-1'
<;''
(':)
v-t
graph v=v(t)= 3(t-S)(t-19)
'
I
,10
I
•l.
:u t,
I
·141 I lo' I
I
I
I

ti
I
(c) I
I
I f= f(t)= 6(t-12)
f-t I
I
graph I

'
0 I
I
I I
I I
I I
cw
•1
�1 e• I

(xj.i i) Verify the following facts from the sketches above,


(a) For arcs ABC, FGII the curve x x(t) is rising,
iLs gradie.nt, n,u;,c•ly dt
dx
= v > 0. This can be seen
from the corresponding arcs A'R'C', F'G i ll' on the v=v(t)
being above the t axis .
Similarly consider the arc CDEF and the corresponding
arc C'D'E'F'.
(b) At C, F the curve x = x(t) is stationary, i.e. :: = v = O.
This can be seen from the corresponding points C', F' on
the v = v(t) curve being on the t axis.
(c) For the arc ACDE, the curve x = x(t) is concave down, and
d2x
= f < 0. This can be seen from the f = f(t) graph
W
where the corresponding section A"C"D"E" lies below the
t axis. Sin1ilarly consider the arc E�'Gli an<! the corres­
ponding section E"F ..C"H".
(d) At E, a point of inflexion on the X = X ( t) graph, the
d2 x
curve changes curvature. Here d"t2 f = 0 and f changes
sign through E. Check this by considering E" on the
f = f(t) graph.
(e) The equation of the inflexional tangent at E on x = x(t)
is given by x+286 = -147(t-12). Note at E, t=l2, x= -286,
v= -147. The equation of the tangents at C, F are x = 400
and x = -972.
( f) Show from the sketches that (i) x<O, when O!:t<l or-when
10<t<25. (ii) v<O, when 5<t<l9 . (iii) f<O, when 05t<l2
State the corresponding values of t for which
(iv) x > O (v) v > 0 (vi) f > O.
(g) ( i) The stationary values of x = x(t) occur when ! = 0,
104
DIRECTED EXERCISES
d2x
i.e. when t 5 or 19 (see C', F'). When t = 5, <lt2'" < 6
=
d2 x
(see C"), and when t= 19, <lt2'" > 0 (see F"). Hence the
theoretical relative maximum value of x is when t= 5
(where x = 400), and the theoretical relative minimum
value of Xis when t= 19 (where X = -972).
(ii) However, the greatest displacement of the body P
for OS. t S. 25, is at F, and this is 972 m (on the
negative side of O); the greatest positive displacement
for this period of time is at C, i.e. 400 m
For o S. t S. 30, the greatest displacement of P from O
is when t= 30, in which case x= 2900, i.e. P is 2900 m
to right of O.
[This exercise illustrates the fact that the calculus
cannot be b Zindly applied to probZems on maximwn or.
minimwn displacement, veZoaity, acceleration; reference
to appropriate sketches is safer.]
dv
(h) (i) The stationary values of v = v(t) occur when =0
dt
i.e. when t= 12 (See E")
v d
Since d =
dt7
2
(6t-72) 6 > O, there is a theoret-
dt
ical minimum value of V when t 12, where
v = -147 (See E').
(ii) However, from the v = v(t) curve, for Osts 25,
the greatest veZoaity(positive or negative) occurs
when t=25, in which case v = 360•m /sec in-direc­
tion (see G'). The value v= -147 can only be
interpreted for O S. t � is as the greatest nega­
tive velocity of P, i.e. the greatest velocity in
+-- direction. The actual minimum veZoaity of P
is z�ro (i.e. wher. it is momentarily at rest),
and this occurs when t= 5 or 19 (See C', F')
(iii) For Ost S. 19, the greatest velocity is 285
m /sec in---+ direction; when t O (See A'). For
t � O, the greatest velocity of P cannot be deter-
minerl (Study H').
�_c..����������

(i) From study of f= f(t) graph, for OS. t � 25, the great­
est aaceleration is at G" (when t= 25, f = 78 m /sec 2 )
and the least acaeleration is at E" (when t= 12, f = 0)
For O S. t S. 19, the greatest acceleration is at A",
(when t= O, f= -72m/sec2 ), but for t� O, the greatest
acceleration of P does not exist (See H").
(j) Sho� that the shaded area (on t�e v - t graph)
� L 3(t-5)(t-19) dt and evaluate this. Hence verify that
this result gives the distance covered by P for the time
interval t= l to t= 5.

105
FtlRTflER EXAMPLES
EXAMPLE 3.. A stone is thrown up1,iard from the edge
of a cl ift 384 m high, so that its height h �
above ground level after t sec is given by
2•
h = 80t-16t �-'--��������������������--'
h
{�ote here we take the positive h axis
as vertical in t, and the origin as
the top of the cliff. �e assume that
the stone travels in a straight line,
and that it will fall into the sea}.
(i) Si1ow that the velocity v m /sec _.;,4L.,4L.,,c..,t.'-,£-,,�+-.,._ -:-::-::-;-
-=-•�L.,•I
and the acceleration f m /sec2
of the stone is given by v = 80-32t, and f = -32.
(ii) Find the displacement, velocity and acceleration of the
stone (a) initially (b) after 2 sec (c) after 6 sec.
(iii) 1-fl1en is the stone 64 �l above ground level? Interpret
these results.
(iv) Determine the highest point reached hy the stone
above 0.
(v) Find the time taken for the stone (a) l,) hit sea
(b) return to its starting point.
(vi) How fast is the stone travelling in each part of (v)?
(vii) Calculate the distance covered by the stone during the
(a) first s�cond (b) time interval t = 1 to t = 2�.
(viii)Determine the (a) average velocity for the time inter­
val t = 1 to t = 2\ (b) instantaneous velocity at the
end of the third second (c) maximum and minimum velo­
city of the stone (d) mean of the velocities at the
end of 1 and 3 seconds.
(ix) \fhen is the velocity of the stone equal to half its
initial velocity and (a) the same sign (b) opposite in
sign?
(x) Sketch on separate diagrams the graph of (a} h against
t (b) v against t (c) f against t.
(xi) For what values of t is (a) v < 0 (b) h > 0 (c) f < O?
(xii) (a) Find the change in velocity of the stone during the
(a) first second (b) time interval t = 1 to t = l\
(S) Detennine the average acceleration of the stone for
each period specified ahove.
(xiii) (a) Calculate the displacement from O when the velocity
is aoni/sec in +.
(b) Find the velocity and acceleration of the stone
when it is 96m below O.
(xiv) Determine the equation of the tangent to the curve
h = 80t-16t 2 , where (a) t = ), (b) v = O.

106
FURTHER EXAMPLES
EXAMPLE 4. The displacement x cm of a particle P
from O after t sec is given by x = t-log(3t+I).
(i) Show that the velocity V Cl'l/Sec and the acceleration
f cm/sec' is given by
= 3t-2
V
3t+l' f
= (3t+lt
(ii) Find the displacement, velocity and acceleration of P
(a) initially (b) at the end of 1
(e-1) sec.
3
(iii) For what value(s) of t is (a) v = 0 (b) v>O lC) v�o?
(iv) Find for the third second (a) the distance covered by
P (b) the change in velocity of P.
(v) Determine the (a) average velocity for the third second
(b) the average acceleration for the third second
(c) the average of the velocities at the end of the se­
cond and third seconds (d) instantaneous velocity
at the mid-instant of the third second.
28 9
(vi) Calculate (a) x when v = (b) v when f =
31 256
(c) f when x = 8 - 2 log 5.
(vii) Find the equation of the tangent to each of the curves
below where t = 0
3t-2 9
(a) x = t-log(3t+l) (b) V = (c) f =
)t+l (3t+l)2
*(viii) Verify the sketches shown for O � t � 5, of
(a) x against t (b) v against t (c) f against t
X .,..I
-r.'',j...-+--4<>-------+�+---

- 0

EXAMPLE 5. A particle P is projected with a speed


of 3 cm /sec aI ong the x axis towards the origin 0
from an initial position at a distance I cm from 0
on the positive side of 0. After time t sec its
displacement x cm from O is given by
x = Ae-t+se-2t, where A, Bare constants.

x.' 0
(i) Verify that v =
-Ae-t - 2Be-2t and
(ii) To find the constants A, B we have, by data, when t•O,
107
FURTHER EXAMPLES
x = +1, v = -3, Thus by substi tution in x,v,f, show
t ha t 1 = Ae
0
+ Be 0 i.e. A+ B � 1, and tha t A+2B 3.
Hence show A= -1, B = 2, and t hus x = -e- t +2e-2t,
V = e-t - 4e-2t, f = -e- t + 8e- 2 t,
(iii) Find (a) the time at which the partic le reaches O and
(�) Jts ve l oci t y and accelera t ion t hen.
- t + 2 e-2 t = 0
\ ( a) Here x= O, and ,'. -e
Le. e t(-1+2e- ) = O, and hence 2 e- t = 1, i.e. 2= e
- t t

and t hus by taking na t ural logarithms, show t = loge2.


1
[No t e e
-t
f O, since ;t" f OJ.
log 2 21og 2 l og 2-1
.n.
l,uen t= 1 og 2, v = e- e - 4e- e e e
(S) e loo?­ 2
-4e Q/:"

2 l og x
= 2-1-4. ( 2- ) = -� [NB e e x]
3
Show that the corresponding value of f is 2.
Thus particle reaches af ter log
O ·693 sec and is
e
2

trave l ling then a t \ cm/sec in+-- direction, but is dece ler­


"
a ting t hencm/sec 2 ,J
at
2
(iv) Show t hat the partic l e travels beyond O, and that t he
t ime taken to travel from O to the fur t hest posi tion
reached beyond O is log 2 sec.
e
{Explain that at O, v = -1,, i.e. v f O and hence t he partic le
continues in+-- direction until t he deceleration causes it
to come momentarily to rest. Show that this occurs when
e-L4e-2t = O, i.e. t = 2 log 2, i.e. it takes 2 log 2 sec
e e
to come to rest from i t s initiai position, and hence it
takes log 2 sec to reach this position from 0. Prove that
e
1
P continues cm past O.}
8
(v) Show t ha t P ceases to accelerate or decelera te 2 log 2
e
sec after passing 0. (First solve f 0.)

* (vi) Verify the sketches of x against t, and of v against t,


X �

Note the use of the fo l lowing features, a lready derived


above.
(a) When t = O, x = +1, v = -3 (b) When t = log 2, x = O,
e
V = -\, =
3
1
(c) When t=21og 2, x = - 8, v = 0. f z
e
108
FURTHER EXAMPLES
(d) When t = 3 log 2, f = O (e) As ·t-+oo, x-+0 and v-+O
e
Remember the geometrical significance of dx
(= and
dt)
d x
V

,f
2
f (= ) f or th e x-t graph .
dt7 210g 2
2t
(vii) Also prove by evaluating (e -l - 4e- ) dtl that the
0
distance covered by Pfrom its initial position until it
comes momentarily to rest is 1� cm , and verify this by
considering the information derived previously. What is
the magnitude of the shaded area indicated on the v - t
graph?

EXAMPL�. The displacement of a particle P is


x cm from O after t sec, where
x = 2t sin 2t + cos 2t OS t S 2rr.
(i) Show that the velocity v cm/sec and acceleration
f cm/sec 2 are given by v = 4t cos 2t and
f = 4(cos 2t - 2t sin 2t).
(ii) Find the initial displacement, velocity and accelera­
tion of P.
rr
(iii) When t= - find the value of x, v, f.
2'
rr 3rr 5rr
(iv) Show that the equation cos 2t 0 when 2t
- 2' 2' 2'
1
; and hence find t. (Note the units for t are seconds
but in our work in Calculus, where sin t is given, tis
measured in radians. Thus if, for example, at a certain
time, sin t = 1, then a value of t= rr/2 f 1•57 sec.
(not 90 ° sec which, of course is meaningless). Similar­
ly, if tan t= �. 4 value of t I 26 ° 34' but rather
t � 0·46 sec, from radian tables; whilst if x=2t+cost,
then the value of x when t = rr/3 is given by
2
x = ; + f� l · 59 cm.]
Thus show that Pis at rest five times during the first
2rr f 6·28 sec.
(v) Determine the displacement of Pfrom O on each occasion
when v = O.
*(vi) Hence sketch the graph of x against t for O � t S 2rr,
(a) along a number line (b) on normal rectangular axes
*{vii) Find from the sketch in (vi b) how many (a) times x = 0
(b) points of inflexion there are on the x - t curve.
Hence determine on how many occasions P (a) passed the
origin (B) moved with wtiform velocity.
(viii) Calculate the *(a) total distance covered by P
(b) the greatest distance of P from o, for the first
2rr sec.
(ix) Find the equation of the tangent to the curve where tarr

109
FURTHER EXAMPLES
EXAMPLE 7. !
A car trave Is from 8 to A A. 8 x
and then returns to B. Its displacement x km
from A after t minutes is given by x = �(t-l) (4-t)
i

(i) Show that V i(t-1)(3-t) and f = �(2-t)


(ii) Prove that I *AB! = 1c k.m also show that it takes
twice as long to go from A to B as to travel from B to
A.
(iii) Determine the velocity Ci.n km /min) and acceleration (in
km /rnin 2 ) a;; it (a) leaves I; (b) returns to B
(iv) Find the average velocity (in ll'ileshiin) of the c;ir fur
the (a) journey from B to A (13) journey from A to B
(y) whole journey.
(v) Find the velocity of the car at the (a) mid-instant of
the whole journey (S) mid-interval of the whole journey
(vi) Calculate the distance from A and the velocity of the
car when its acceleration is zero.
* (vii) Show that t L6t 2 +9t-2 = (t-2) (t 2 -4t+l), and hence show
the car is !., km from A after both (2-13) and
2 minutes. Calculate the acceleration and velocity of
the car when the c;ir is 14 km from A.
(viii) Sketch graphs of (a) x against t (b) v against t
(c) f against t.
(ix) Determine from the sketches (a) the greatest negative
velocity of the car (b) its greatest positive velocity
(c) the absolute maximum velocity (d) its least
velocity. .
9 3 5
(x) Show that v = when t = or . Hence prove that the
32 2 2
car is travelling at 16il<rn/h away from A after both ll,
and 2!2 minutes. Explain these answers.
(xi) Find the distance of the car from A (and its velocity
then) when it is accelerating at 3 km /min 2
(a) towards B (13) towards A.
(xii) From the sketches, determine the domain of t if
(a) x Z O (b) v Z O (c) f � O.

(xiii) Find the equation of the inflexional tangent to


X = �(t-1) 2 (4-t).
(xiv) On the v - t sketch, shade the areas representing the
distance equal to (a) *BA (b) *AB

Verify that
l
/JJ
(t-1)(3-t) dt' =
3
if
(t-1)(3-t)dt
0 I

110
FURTHER EXAMPLES
EXAMPLE 8. The velocity v cm/sec of a �moving
in a straight line is given by v = l25-x2 , when
x cm is the displacement of the body from.O.
Note here, the veloaity is a funation of x.
(i) Using the result (a) f = a:(!,v 2 ), show that
f = ..J!u,<2s-x
dx
2 )} -x

(b) f = V :!;, show that f = /25-xz .


d
:{/z5-x 2 }= -x

(ii) Find the velocity and acceleration of the body when it


is (a) 3 cm to the right of O
(b) 4 cm to the left of 0.
(iii) Determine the value of
(a) x and v when f = 0 (b) x and f when v = 0
(iv) Calculate the position and velocity of the body when
the body has (a) acceleration of 2 cm /sec 2
(b) retardai:ion of 2 cm/sec 2 .
(v) What is the domain an<l the range of v = l25-x2 ; hence
find the (a) maximum velocity (b) minimum velocity,of
the body. Where do these velocities occur?
(vi) Verify the graphs given below of (a) v against x
(b) f against x.
f
V' 5
(a) (b)
5 f. -x
5
-)( -s )(

0 5
tEXERCISES SET 3E
1. A ball rolls up an incline so that its
distance x cm from O (the foot of
the incline) at time t sec is
given by .x = 2t(3-t).
(i) How far up the incline will
the ball roll, and what
is its acceleration
then? 0

(ii) With what speed does the ball reach the bottom of the
incline on its way back?
(iii) What is the velocity of the ball when it is 4 cm up the
plane?
(iv) Find the (a) average velocity of the ball for the first
second (b) average of its velocities at the·beginning
and end of the first second.
111
EXERCISES
(v) Where was the ball when it was travelling at 4 cm /sec
(a) up the incline (b) down the incline?
(vi) Sketch separately the graphs of (a) x against t
(b) v against t (c) f against t.
(vii) Find the equation of the tangent to the x - t curve
where t = 1.
(viii) Determine the greatest and lE>.ast velocity of the ball.
(ix) Shade on the v - t graph the area representing the dis­
tance covered by the ball in rolling.up the plane, and
verify this by (a) evaluating a definite integral
(b) calculating the area without calculus.
2. At time t seconds, the position coordinate x cm of a
point moving in the straight line X'OX is given by
L= at 2 +ht, where a, b are constants. If it passes
through O with velocity 24 cm/sec in the positive direc­
tion at time t = O, and after 8 seconds, it is again at
0, find the values of a and b.
Hence determine the (i) magnitude and direction of the
acceleration at any time (ii) the position and time
when the velocity i� zero.
3. A point P moves in a straight line so that its displace­
ment x cm, measured from the origin is given by
x = 3(1 - cosnt), where t is the time in seconds.
(i) Show that initially P is at rest at O, but its acceler­
ation then is not zero. lvhat is the effect of this
acceleration?
(ii) Prove that when sinnt = O, then nt = O,n,2n,3�, ....•. ,
and hence that P comes to rest at the end of 1 second
intervals, but that it never comes finally to rest.
(iii) Find the furthest point A reached by the moving point P
and show that P runs a "shuttle service" from O to A
and return.
(iv) Verify that the magnitudes of the accelerations at 0
and A are the same.
(v) Determine the speed of P when it is distant� from O,
and show that P is neither accelerating or decelerating
then.
(vi) Use the results above to sketch the graph of
(a) x against t (b) v against t.
(vii) What is the maximum velocity of P, and where does it
occur?
(viii) Show that although the velocity is variable, the aver-
age velocity for the first, second, third, seconds
is constant.
(ix) Determine, by integration, the distance covered by P
112
EXERCISES
during the first 2 sec.
4. The displacement x cm of a body from the origin O after
t sec is given by x=t - log t, t > O.
(a) Determine the
(i) displacement, velocity and acceleration after
2 sec.
(ii) displacement and acceleration when the body
comes to rest.
(iii) displacement and velocity when the accelera-
tion is 4 cm /sec 2 •
(b) Sketch the graph of x against t, using the results
above and hence
(i) show that the body is never less than 1 cm
from O, and that both as t + 0 and t + oo, the
body moves further and further from 0.
(ii) explain why the body never ceases to acceler­
ate, but that its rate of acceleration dimin­
ishes with time.

5. In the usual notation, the.motion x · --+ -------x


�,
of a particle moving in a <� lo
straight line is governed by the equation log x -4t,
(t in seconds, x in cm ) • e
-�----
-4t
(i) Show that x = e , v = -4x, f = 16x.
(ii) Calculate the initial displacement, velocity and
acceleration of the particle.
(iii) Show that the particle (a) is always moving to-
wards the origin but never reaches it
(b) never comes to rest, but is always slowing da.m
(c) never has· zero acceleration. Also determine
the 'limiting' velocity and
'limiting' acceleration of the particle, (i.e. as
t + oo),
(iv) Calculate, correct to 2 significant figures, the
velocity and acceleration of the particle after 1
sec.
(v) Determine the average (a) velocity (b) accelera­
tion, during the time interval t = 0 to t=�.
(vi) Sketch on separate diagrams the graph of x against
t, v against t, f against t.
(vii) (a) Show that the particle is
1 !
cm from O after
log 2 sec, and find with what speed it is then
e
travelling.
(b) How long does it take to traverse the distance
from x = 1/16 to x=1/256?
6. The velocity v cm .'sec of a particle P at time t sec is
given by the equation
(i) v =ct+kt2 , where c, k are constants. If v=3 when

113
EXERCISES
t = 1 and v 4 when t = 2, find the values of c,
k and hence find the acceleration of P after time
112 sec.
(ii) '! = 4t(8-t) (a) Determine the acceleration of P
when it comes to rest.
(b) Sketch the graph of v against
t, ar.d shade in the area repre­
senting the distance travelled by P for the first
4 seconds, and calculate this distance.
(iii) v = 4t % + 2(1+/t) 3; Find the acceleration when
= O
7. ( i) A mass is set oscillati:g a� ��e end of
a spring. Its velocity v ,-111.fsec when
the spring is compressed x cm is givenI
by y2 = 4QQ-900x 2 • X

(a) Show that its acceleration f ___A(l2v 2 ) = -900x ' and


dx
and find the numerical value of f when the mass is
instantaneously at rest.
(b) Noting that the maximum value of v occurs when.v 2
is a maximum, find the maximum velocity of the
mass. What is the minimum velocity of the mass?
(c) Find the maximum accelc•ration of the particle.
fllint: for what valut!s of x does v 2 exist?)
( ii) The velocity v of a particle in the position x is
given by y 2 = a+bx+cx 2 where a,b,c are constants.
(a) Find the value of c if the acceleration is con-
stant.
(b) If a = 9\, b = 2, c· 2 find the values of x,v for
which the acceleration is zero.
8. (i) Tile velocity v and the position coordinate x of
a point moving in a straight line are connected by
the· relation .Y = a+bx, where a, b are constants.
Determine in terms of a, b the acceleration of the
point (a) at the position x = 1 (8) when it is at
the origin.
(ii) A meteorite entering the earth's atmosphere has
¥€lo.pit11 inversely proportional to
the square root of� when at a
distance x from the centre of the
earth. Show that its acceleration
is inversely proportional to x 2 •
9. A particle P moving along the straight line X'OX is
such that after t sec its displacement x cm from O is
given by x__= 6+Jt-2t 3 . After a furthe·r .St sec its dis­
placement is (x+ox) cm from O .

114
EXERCISES
(i) (a) Determine the distance travelled by P in the
&t sec, and the average velocity of P for this
period.
(b) Hence, find the instantaneous velocity after t
sec.
(ii) (a) Determine the change in velocity ov of P for
the ot sec, and the average acceleration of P
for this period.
(b) Hence find the instantaneous acceleration after
t sec,
(iii) If t = 2, ot =0,01, find the value of
ox dx ) ov - dv
(a) < (b) ( )
6t - �-; ot dt
10. The mass shown in the diagram is��---�����
attached to an elastic string "-fi"d poiot
and is released from 0, Its
depth x cm below O after time t
sec is given by x = 1-cos(/gt),
where g is constant,
0

(a) Find its acceleration in terms X


of X,
(b) Determine when and where it has its first (a) maxi­
mum velocity (8) maximum acceleration,

SECTION 5. DETERMINATION OF VELOCITY AND POSITION, GIVEN THE


ACCELERATION, AND ALLIED TYPES
The methods are best illustrated by working through the
following examples.
CASE I. ACCELERATION A CONSTANT
EXAMPLE I. Derivation of the equations of uniformly
accelerated motion. A particle moves in a straight
line so that the acceleration has constant value
'a'. Initially, the particle is at O and travel Iing
w ith a veIoc i ty I
u1 •
�:� x
"
dv dv
For the acceleration I we may use dt' v­
dx
(i) Let��=a, .·. v = fa
dt =at+ Ci,
where Ci is constant (L)
dx dx
Further, since v = dt =at+ Ci
dt'
Hence x = f(at+ Ci)dt =l,,,at+cit
2 +C 2 ,
where C 2 is constant • . • • . .•.••.•• (ti)
Now by data, when t = O, x = O, v = u (u may be positive
or negative)

115
ACCELERATION A CONSTANT
from (L), u =Ci and from (M), O =C.a
Hence (L), (M) become v = u + at and x =ut + �2§l t2 •
(ii) Let �(�v2 ) =a, i.e. �v2 Ja dx
.........
=

Hence �v 2
ax + C3, where C3 is constant (N)
Using x =O, v u, .·• from (N), �u 2 = C3
Thus (N) becomes, �2 =ax + �u 2 , i.e. _v_
2 ����
u 2 +2ax
(iii) Prove direatly by using v =u+at and x ut +fat 2 , that
v 2 = (u+at) 2 = u 2 + 2ax.
dv dv a
(iv) Further, let v �=a, and thus � =-
dx dX V
dx 1 dx =v and thus
Now since = show ;
dv dv/dx' dv
v2
Hence x = Za + C4, where C4 is constant
u2
(P) .........
Since, when x 0, v u, . ·. from (P), 0 = Z + C4
a
=
v2 u2
and .·. (P) becomes x =� - Z ' l.e. _v_2�-u� 2_+�·2 ax
� �
a

Note The equations v =u+at, x =ut+�at2 , v2 u 2 +2ax are


the equations of uniformly aaaelerated motion. These
are extensively used in Applied Mathematics, Physics,
etc. where the acceleration is constant (1!,g, a body
falling freely under gravity, assuming no air resist­
ance), However, in our work, these results will not be
used; calculus methods will be employed.

EXAMPLE 2. A bal I is dropped from rest


from a point 21 m. above the ground; X t•O
• .(,.,Lt I K • + 21
si mu I taneous I y a second ba I I is' thrown v•o
upward with an initial velocity of I
l4m /sec from a point on the ground I
immediately� the first bal I.
Assumin{l the aaeeleration due to grav­
ity is 9•8 m/sea2, determine whether or
not the balls meet while they are still {t20
v-:+t�
in the air. ,.'f.. •O
If they do meet, find the speed, direction and height of
each ball at the moment of impact, Which ball hits the
round first?
Method Take the axes as shown in the diagram.
[Note the positive direction of x, v, f is measured int di­
rection and relative to O.J
(i) Forball·l, noting .!!Yi =-·8,
9 prove v1 = ..9·5t+C1 and
dt
hence that x1 =-tr9t 2 +c1t+C2 , Show that'C1 =O, C2 =2l
dv2
(ii) Forball 2, noting�=-9•8 prove v2 =·Q•Rt+C3 and
hence that x 2 =-lr9t 2 +C3t+C4. Show that
116
ACCELERATION A FUNCTION OF TIME
c 3 = 14, c 4 = O.
(iii) If the balls meet, explain why x1 = x 1 and hence show
that t = f
3
(iv) Verify that when t =
2, v1 = -14•7, v 2 = -.o�. x1 = x2 =9 .975

Hence the balls m eet after 1� sec at a height of 9.9 75 m


above the ground. The balls are t ravelling at 14•7 m /sec )
0'1 'Ill/sec respectively in the downwards direction
1.e. both balls are aoming do�>n when they meet.
(v) If the result above is surprising to you,-
show that ball 2 reverses direction after
lO
sec, and is then 10.m above the �
7 �,
ground. (i.e. after 1\ sec, it has re- boUt
versed direction and is then 9.9nmabove T ball:i.
the ground. ) \·\is ...
0

(vi) When each ball hits the ground x1 = x 2 = O. Show that


f?iri 20
the corresponding times are!f, seconds,
7 and hence
that ball 1 meets the ground first.

CASE 2. ACCELERATION AS A FUNCTION OF THE TIME


EXAMPLE 3. The equation of motion of a particle moving in
a straight Iine is given by f = 12-kt, where k is a con­
stant. The particle starts from rest at a point P dis­
tant 5 cm from a point O in the I ine, and moves unti I
after 6 sec, it reaches the point Oat wh_ich its velocity
is zero.
(i) Calculate the,(a) distance PO (bl acceleration at 0
(c) maximum velocity of the particle.
(iil Sketch the (a) acceleration-time (bl velocity-time
(c) displacement-time graph using the information
derived previously.
ft:{,
{�:�
;,c,;5 lv-=o "
o ,, P GI

Method. kt2
d
Show that : = 12-kt, and thus that v = 12t - � - + C 1,
d 2
Explain why, when t = O, v = 0 and when t 6, V = 0. Hence
show k = 4, C1 = 0, and thus v = 12t-2t 2
x
Also since : = 12t-2t 2 , prove x = 6t 2 - 1 t 3 + c 2 , where
t 3
C2 = 5.
Hence show f 12-4t • • . (X) v = 12t-2t 2 , ..• (Y)
X = 6t 2 -· l_ t 3 +. 5 ••••• • •• . • • ( Z)
3
(i) (a) Prove *OQ = 77 c m, by substituting t = 6 in (Z)
Hence noting *PQ = *OQ - *OP, show *PQ = 72 cm
(b) From (X), prove that the acceleration at Q is

117
ACCELERATION - TIME
12 <'m /sec towards o.
i

(c) The maximum velocity is best obtained graphically.


(ii) the sketches
V'

18

/t•6
-12. V ,.-,2 o 3 6
From the v - t graph, the maximum velocity occurs when
t = 3 and is 18 cm /sec. By Calculus, stationary values
of v occur when dv/dt = O, i.e. when 12 - 4t = 0
i.e. when t = 3, and then v = 2.3.(6-3) = 18.
d1v
Since = -4, this stationary value is a relative
dt 2
maximwn. . [Graphically, we see also that this is an
absolute maximwn. J
(iii) Show that the area bounded by the curve v "' 2t(6-t) and·
6
the taxis is J 2t(6-t) dt = 72 sq units. Verify that
the distance o covered by the particle is 7 2 cm

d .
EXAMPLE 4. og cos t)] = -tan t.
Prove that
dt ll e(
A particle moves so that its acceleration f after t sec
is given by f = -sec 2 t-9cos3t (0 � t < n/2). Given that
when t = O; x = 2 and when t = n/3, x = 4n/3 - log 2,
find v, x when t = n/4.

Method.
(i)
d
: [log (cos t)J
e
--- 1
cos t
- sin t = - tan t;.

(ii) Show that v = -tan t - 3 sin 3t+C1


and x = log (cos t)+cos 3t+Cit+C2, where C1,C2 are
e
constants.
(iii) Using the data, verify that
4n n
2 = log(cos 0)+cos O+ c2 and� - log 2 = log(cos )
3
+cosn +C1 , {+C2
whence C2 = 1 and C1 s 4.
n 3
(iv) When t , show that v = ,-,
(2-i2)
4 2
m

1
and X \ log 2 - 72+ n+ 1

118
ACCELERATION - TIME
EXAMPL� 5. The equation of motion of a particlP is given
by dv I
dt = t-7• t > O. After I
sec, the displacement of the
particle is I I cm to right of O and its velocity is
2 cm/sec away from 0. Calculate the (i) acceleration of
the particle when it is at rest
(ii) distance covered in the time interval t= �to t= e
and the average velocity during this time.
{>(•II
t• I
\/'•2.
0 A X
Method.
(i) f
Show that v = - + C1, x = - log t + Ci t + C 2 and find
e
the values of t he constants C1, C2,
(ii) Verify that when v= O, t= f and f "' 9.
(iii) TI!e distance covered= fi v dt • fe (C1
e 1
- -) dt and
I I · t
hence show t his is (3e-4) ft. Now determine the average
veloc.i ty during this time. "' -,,1--------,
(iv) Tiie sketch of v against t is shown.
(a) What is the equation of the curve,--,,+-�....._____�
and what are the coordinates of B? t

(b) Shade in the area representing the distance covered for


t = 1 to t = e.
(c) From the sketch, indicate what is happening to the par-
ticle (a) as t � 0 (6) at B (y) as t � �
(d) Explain why the accelera tion of the particle is always
positive and in what direction is it ac t ing? S t a te the
significance of this in connection with the slope of
the v - t curve.
EXAMPLE 6. A particle P travelling in a straight line
dt e-2t
X'OX obeys the law dv 2 where v is the velocity
in cm /sec at the end of t sec. After �. I s·ec respective­
ly , Pis �(e+5), �Ce 2 +2) cm to right of O. Determine the
initial velocity and displacement of P, and the acceler­
ation and displacement when the velocity is I cm/sec to­
wards 0.
Method.
--- dv dt dv
= 1 I d' show tha t dt "'
2t
(i) Noting t hat 2e and hence
dt V 2t
that v = e + C1, x = � +c1t+c2,
2t

(ii) Given when t= �. x = �(e+5) and when t•l, x = \(e 2+2);


show that C1+2C 2"' 5, C1+C2 = 1 and thus Ci a -3, C2•4.
(iii) Determine the value of v and x when t = O.

119
VELOCITY - TIME
(iv) Show that when v = -1, then t = �log 2, Hence prove
that the acceleration then is 4 cm /�ec2 , any find the
ogep
corresponding displacement. [Hint: note e = p,
Prove this by taking natural logarithms of both sides.]
(v) Sketch the acceleration-time graph; shade in the area
representing the velocity change in P for the first 2
seconds, and determine the average acceleration for
this time.

EXAMPLE 7. The veIoci ty v 'cm /sec ofa body moving


in a straight line at the end oft sec is given by
V = 3t2 -15t+l8.
Method.
�erify the sketch of v against t 11
shown. From it, find the time
interval f�r which the velocit�
is (a)positive (b) negative vs }(t-l)(t-3)
(c) zero, !foat is the signifi­
cance of these results?
-Zf-"-""'���-c--�� t
(ii) Determine for O� t � 4 the {
(a) maximum and minimum velocity
of the body
(b) greatest negative velocity.
When do these occur?
(iii) Show that the acceleration f is
given by f = 6t-15, and find the
acceleration at the end of 2 se�
•IS
(iv) Verify the sketch of f against t·shown, and find for
0 � t � 4 the greatest (a) acceleration (b) retardation
of the body,
(v) Show that the total distance covered during the time
interval (a)t Oto t = 2 is f (3t 2 -1St+18) dt
2

(b)t= 0 to t= 3 is o
2 3
f (3t2 -1st+1a)dt +if (3t 2 -1st+1s) dtl
2
(c)t�=Oto t=4 is J:v dt +lfz3vdti+fv dt
Evaluate each integral.
What is the area between the graph v = 3t2 -1St+18, the
t axis and the ordinates at t = O, t = 4?
tXAMPLE 8 PRACTICE INTEGRATION TYPES
Ci) Ifv = (3t-1) 7 and when t = O, x = O; find x when
t=I.
Show that x = ili=ll
. + C1; hence find c1, -and then x when t=l
B 3

(ii) Ifdx = sin(2t-l) + cos nt, and when


dt
t = �.
x 3 find x in Terms oft.
2;
l?O
VELOCITY - TIME
-cos( t-l) si t
Show that x .. ; + :' + C..t; hence find C2 and then
x in terms of t ,

*(ii I) If x = sin 3t + tan t, and when t = n, x = O,


find x when t = 0.
(a) Note that cos 2 0 = 1 - 2 sin 2 0 = 2 cos 2 0-l,
then sin 2 0 "' ),z(l-cos 2 0) and cos 2 0 = ),z(l+cos 2 0).
Also sec2 0 = l+tan 2 0, • •• tan 2 0 = sec 2 0 - 1,
dx 1
(13) Hence show that dt = 2(1 - cos 6t) + (sec 2 t - 1)
, sin- 6t
and thus x = >:i[t - ---) + [tan t - t] + C3 ,
n 6 n
Now prove that C3 = , and when t = O, x =
2 2
dv_�
( Iv) If , and when t 0, X 15, when
dt - 415t+4
t = I x = 32: find x when t 9
J,z
(a) Now!:= 7!(5t+4)- , and thus V =
li. (5t+4)'li + c ..
4 5.J,z
i.e. v = 15 l<
+ c4 •
2 (5t+-4)
2

Further prove x .. 15 (5t+4)


, 3/z + C4t + Cs,
2 5 31 2
i.e. x (5t+4) t2 + C4t + Cs
(S) Using the data, show Cs 7 and C4 -2, and find x when
t = 9.

(v) Prove that


d Ct log t - t) = I og t.
cit If t . ��� = 2+ 3t3,
and when t I, v = 7, x = 8!; find x when t=2
d 1 + log t
t • t 1 - 1 = log t.
(a) Show dt(t log t - t)
3
<a> Now � = dv = 2+3t = 1 + 3t2
dtdx dt t t
i.e. = v = 2 loget + t 3 + C�, and prove c6 = 6.
Also
C7 = 4.
dt
show x = 2[t loget - t) + f + 6t + C7, and prove

(y ) Show that when t = 2, x 4(4 + loge2)


.. t I 2
(vi) If X Ce + ,=-> , and when t = 0, V = O,
e
X = 0, express v, X in terms of t.
.. = -d·x
Here x � = e2 t + 2 + -- 1 = e2 t
+ 2 + e
- 2t
dt e2t
2t 2t 2t 2t
Show dx = �2'! + 2 t - J,ze- and x = �e + t2 + �e- -�
dt
121
EXERCISES SET 3F
*EXAMPLE q A particle moves in a straight I ine
X'OX so that the rate of change of the velocity at
any instant is proportional to the velocity at
that instant. Initially the particle is at O and
moving with a velocity of v0 .Find the equation re­
latin (i) v and t (ii) x and t (iii) x and v.
dv
Method. (ex) Here we are given dt = kv, where k is constant.
[The question is thus similar> to questions on the law of
r>adioactive decay, the iCll,) of population inar>ease, eta, J
�ow since
dt
1 / dv
' dt
= =
_!
dv
dt' · · dv kv'
1 1
and thus t = k log v + C, where C - k log v•
e e
1 V
i, e, t = - log (-) and hence that log (.Y...) = kt,
k e v0 e v0
kt
That is, v = v0e is the relation between v and t.
vo _vo
dx kt kt
(8) Using
dt
= voe show x = k e + c 2 where C2 = k
Vo kt
Thus x = k (e - 1) is the relation between x and t.
_
dv
(y) To relate x and v, we use the result v
dx
= kv and hence
:: = k, i.e. v = kx + c3, where c3 = v0•
1
Thus show x = k (v-vo) is the relation between x and v,

EXERCISES SET 3F x.
1. A ball is projected vertically upward
from O with velocity u under cons t ant
gravity. �
d d
Noting that f = � = -(�v2 ) -g,
dt dx o
prove by integration, that (irovnd
(i) v = u-gt (ii) h = ut-� t 2 (iii) v 2 = u2 - 2gh
where g is the acceleration due to gravity, h is t he
height and v is the veloci�y at t ained after time t,
2. A point, ini t ially at A, moves in a straight line AX
wi t h ini tial velocity Sm/sec; and is subject to a con­
stant retardation of lm/sec 2 (i.e. f = -1), Find when
the point is 19,5 m from A on the sicie of A opposite to
x.
3. A point moves with uniform acceleration k in a straight
line X'OX. When at O its velocity is 1sartsec in the
positive direction, and after 8 seconds it is again at
0, Find
(a) the magnitude and direct ion 1
of the acceleration )( 0 X
(b) the position and time at which the velocity is zero.
(c) the time taken to reach a point 75/2 cm from O in
the direction OX', and the velocity at that point,

122
EXERCISES
4. A package slides down a chute 60m long with
constant acceleration of Sm/min 2 . Find
(i) the initial velocity of the
package if it takes
4 sec to traverse
the chute.
(ii) the velocity of the package when it is one-third
of the way down the chute.
(iii) the time for the package to get halfway down the
chute (in irrational form).
(iv) the distance the package travels during the first
half of t:1e time of descent.
5. (a) A ball is thrown upward from ground

I
level with an initial velocity of
28.m/sec. Assuming that the acceler­

1
ation due to gravity is 9 · 8� ec 2 ,
i.e. f = - 9 • 8 find Ground
(i) when the ball reaches its highest point, and
what height it will go?
\
(ii) when and with what velocity it will strike

(b)
the ground.
Answer the questions (i), (ii) above
if the ball were projected from a I
I
point 280m above grouncl level with 0
an initial velocity of 21 m/sec.
ze.o"'
G,-ovn

6. A particle moves in a straight line so that its velocity


is directly proportional to the cube of the time for \
which it has been moving, i.e. v = kt l . After one sec­
ond, the particle has an acceleration of 12m/sec�, Find
k and how far it will move in the next second.
7, A particle starts from rest and moves along a straight
line with a velocity which after t sec has the value
(18t - 3t 2 ) cm /sec. Calculate the velocity when the ac­
celeration vanishes, and the distance covered during the
third second. After how ma·ny seconds has the particle
returned to its starting point, and what is then the
total distance covered? [Hint: For distance, first de­
termine where particle comes to rest.]
t8. A point P moves in a straight line. At time t sec its
velocity is v cm per sec, and its displacement from a
fixed point A in the line is x cm.
If v = pt 3 + qt, where p, q are constants, when t = 1,
v = zf and when t = 3, v = 15, calculate the values of
p, q. Find the acceleration of the point P when t = 5,
and the distance it travels in the first second.

123
EXERCISES
9. The velocity v cm per sec of a par ticle which travels
from rest to rest in a s t raight line is given at time t
sec after the s tart by v = 6t-3t2 , Find
(1) the maximwn speed at tained
(ii) the total distance t ravelled
(iii) the greatest acceleration
(iv) the average velocity for the journey.
dx
10. (1) Given + t = sin t and x = O when t o, find
dt
x, f when t = .
2
(ii) If V = 2 cos 2t and X 1 when t = 11 , find x when
11 4
t =
2· 2 1
(iii) If x t2 + 5
- ?"• and when t 1, X = J' find X
when t = 2.
(iv) If v = 1
( 2t+l)3• and when t 1, X 3, find x when
t = o.
Given dx 1
(v)
dt. =
2t+l'
and when t = 1, x = 3 find x when
t = o.

(vi) If x sec2 (2t+l), and x = 1 when t


= \(f - 1), find
I 311
x when t = �(� - 1).
4
2t-l
11. (1) Given v = e and x "'0 when t = o, find x when
t = �.
(ii) Noting that cos 2t = �(l+cos 2t), and if dx = cos 2 t,
dt
and x = _2 when t = n, find x when t =
11
4'

(iii) If t dx =
dt l+t, and x = 3 when t 1, find x when
t = e,
(iv) If e
t V = 2t
.
e + 1, and x 2 when t o, find X
when t = 1.

12 . (i) The velocity � of a par ticle a t time t is given


dx lit
by sin . Find the
dt = 6
(a) average velocity of the particle over the in­
terval O it i 2.
(b) acceleration after 1 sec.
(c) instantaneous velocity at the end of 2 sec.
(ii) If v = t+I t- 2 1, sketch the graph of v against t
for O i t i 6, and hence find the dis tance
covered during this time interval.
13. The velocity-time equation of a body moving in a
straight line is given by v = 7t-lO-t2 m/sec
(a) Show that the body is momentarily at rest at each
of two poin ts A, B and find *AB.

124
EXERCISES
(b) Sketch the graph of v against t, and hence, for the
t ime interval t= 0 to t = 6, find the
(i) greatest velocity (ii) leas t velocity
(iii) grea t est positive velocity.
(c) Determine the dis t ance covered by the body during
the time interval (i) t= 0 to t= 2
(ii) t = 2 to t = 5 (iii) t = 5 t o t= 6
(iv) t = 0 to t = 6.
1
!�ence state the area bounded by t he curve v= 7t-lo- t ,
the t axis and the ordinates t= O, t = 6.
(d) Find the accelera tion of the body when it is a t rest.
14. The velocity v CTll/sec of a particle P moving in a
straight line X'OX is given by v = 1 - 2 sin 2t , and
when t = 0, x = 1.
(a) Show that in the time in t erval defined by O � t � 2rr
rr Srr 13rr 17rr
P stops after t =
12, 12, 12, 12 sec. Find the
corresponding displacements from O, and the ;3.ccel-
erations in each case.
(b) Use these results to sketch the graph of x against t
for OS t S 2rr and find the average velocity of P
rr
for t =
2 to t= rr.
15. (i) If � = 2t _ ___ 3_an
_ _ d_wh _ _ -- - - v-- - - ---
_ _en t O, 7, x 4 find
x, v, f when t= 1 .
(ii) If f = /l+t and when t= O, v= O, find
(a) v when t= 8
(b) the distance from the start ing point a t the
end of 3 seconds.
dv = t -t
(iii) If 2e - 3e , find v when x = 0 given t ha t
dt
v = 5 and x=-6 whe� t= 0.
t
[Hint: Show x = 2e -3e- -5,
and wnen x = O, (2et+l) (e L3) = O.]
rr
(iv) If f= -12 sin 2t, fine x when t= given that
4'
x = 5 when t= O, and x 8 when t= rr.
[First show x = 3 sin 2t + lrr � + 5]
d2x
16. (i) If dt2°= -27 cos 3t - 36 sin 3t and if X = 0 when
rr
t and x= 2 when t = 1.:! calculate x when
= 3' 3'
t =
2 (!;int show x 31cos 3 t +4 sin 3t +C1 t + C2 ,
=

and hence t hat C1 ;, c2= 7.]

125
EXERCISES
(ii) If :: • It - ft, and when t • O, v • 2, x • ·S,
find (a) v (b) x, when t"' 1,
(iii) Given W• 7
d 2 x 2t-1
and when t • 1, x • 4, v "' 3 find
x when t • 2. 1
(iv) If x• e
1
-2t- + _1...... - --J.; + 1 2 - (1-St)9,
t+l (t+l) - cos t and
when t • O, x• x
1, express in terms of t.
17. A particle starting f�om rest at O moves along a
straight line OX so that its acceleration after t sec
is 12t(2-t) cm /sec2 .
(i) Find when it again returns to O and its velocity
then.
(ii) Find its maximum displacement from O during this
interval.
(iii) What is its maximum positive velocity and its
greatest speed during this interval?
18 .. The acceleration f of a car is given by df = -k, where k
dt
is constant. If the car starts from rest with an accel­
eration of 4m/sec2 initially and lm/sec2 after 10 sec,
-3
show that f = t + 4. Hence find the
10
(i) distance covered and the speed after 10 sec.
(ii) elapsed time, the distance covered and the speed
when the acceleration reaches zero.
19. A particle moves along a straight line so that if fOJ/sec2
. df
is the acceleration at time t sec, then dt = 12. At the
end of 1 sec, the displa.cement from the origin is 4m,
the velocity is 4 m/sec and the acceleration is 4m/sec 2 •
Find the distance of the particle from the origin, its
velocity and its acceleration when t = 2. Show also
that, if vrn/sec be its velocity at time t sec, then
24v = f 2 +80 for all values of t.
20. A body moves in a straight line with initial velocity
9 c m /sec, Its acceleration t sec after motion begi.'1.s is
2(4-t) cw/sec2 • Find how far the body moves before be­
ginning· to retrace its path, and prove that the. time
that elapses from the beginnin� of the motion before the
particle returns to its starting point is 3(2+/7) sec,
21, The acceleration f cm/sec2 at time t sec of a particle
moving in a straight line is given by f = -4 cos 2t, If
the particle is at a distance x cm from the origin at
time t :md if it starts from rest when x = 1, show that
f = -4x, Prove also that the particle reaches the origin
f sec after starting and that the magnitude of its velo­
-----===-===-=========�
city is then 2 cm /sec,

126
VELOCITY AND ACCELERATION IN TERMS OFx
SECTION 6. VELOCITY AND ACCELERATION IN TERMS OF x
0
EXAMPLE I. A particle attached to the end of an
elastic string is al lowed to fal I from rest and
has an acceleration of (32-2x) cm/sec 2 after it
has fallen x cm.
Show that v 2 = 64x-2x 2 = 2x(32-x). Find its
greatest velocity and the distance it falIs be­
fore first coming to rest.
"'9
f.fethod (i) The acceleration f ia given by
--- dv d2x .. dv d 2
f = dt = tH7 = X = V cfi = d:,/lV ), Since f is stated in
terms of :.c, the only result which is applicable here is
d(' 2
dx �2V ) ,
!\ow �(l-2v 2 ) = 32-2x, :. � 2 = J (32-2x)dx=32x-x 2 +c

By data, vhen x = O, v = O. C=O, and tlius lzv2 a32x-x 2

(ii)
The greatest ooiue of v occurs when v 2 is
greatest. This is best determined " ·
from a sketch of v 2 against x, 1�
From this sketch, the greatest
value of v2 is 2 x (16) 2 and thus
the greatest value of v is
12.(16)2 = 1612" cm/sec. X

(iii) When -th e_ _p_a_r_ti


_ cle--c-o_m_e_s----to rest, v = 0
.,.. .,..""',..

i.e. 2x(32-x) = 0 and thus x = 0 or 32, The applicable


answer is 32 cm .

EXAMPLE 2. A particle starts at the origin �


and moves a Iong the x-axis in such a way O (x,o)
that its velocity at the point (x,0) is given by the
dx 2
formula cit= cos nx. How long wi 11 it take to traverse
the distance from the or1g1n to the point x = k? Wi II it
ever reach the point x = i? Why?

dx dt
(i) Since dt = cos2 11x, dx'"
sec 2 nx
dt
= 11dx)
tJethod
(noting
tan 11 x + C dx dt
Hence t =
ny data,
f t
scc2 nx dx =
o, X O•• show C = 0 and thus tannx = Tit.
(ii) When x �. show that nt = 1 and .·. t = l/11.

(iii) If it is possible for x to equal\, then the


equation tan 11/2 wt should have a real solution fort.
However, since tan w/2 is undefined, • ·• t cannot be de­
termined; i.e. the particle never reaches the point x,.!2,
127
v, f IN TERMS OF x
EXAMPLE 3. ( i) If v = kx and when t 0, x 2; whi 1st
when t = 10, x = 4, find x when t = 5.
dt l l
I' thod ()
,1e a Show that x and'nence that t = k 1og x + C
dx � k
(B) Prove that O = k ol g 2 + C and 10 =flog 4 + C,
and hence by subtraction, p rove
1 4
10 = k log() i.e. k = (log 2)/10, and C -10.
2 3
(y) Now when t = 5, show that 2 log 2 log x and thus
X = 2fi.

( ii > If = 8x 3 and when t = 0, x = I, v = 2, find x


and v when t I. [Assume v > 0 for this period of time.]

Method (a) Since f is given in terms of x, we use _!!_(�


2 2)=8x 3 dx
Thus on integration, . · . !,,v2 fsx 3.dx 2x 4 +c. and show
C = O.
Taking the square root,
(B) V = x 2 2 • (V > 0, by
dt 1 -1
data), whence = Show t = + C 1 , and prove
dx 2 2.
x x2
c1 • +', ..
(y) When t = 1, prove x = -1 and v .2

(iii) If f = e and when t = 0, x = 0, v = i, find


8x

v, t when x =!and express x in terms of v and t.


[Assume v > 0 for the duration of the motion.]

Method
---C
(a)
=( , 0,
r:ow /<'2v 2)
X
e
8x
, and thus '2v 2 = ! e
8x
+ C, where
x
4 dx
BJ Hence prove v �e
1
• and using v dt' prove
-4
x 4- x
t = 2Je dx. Show that t '2(l-e ).
(y) When x = J..., from (a) show v = ',e � l •359
-1
and from (B) prove t = '2(1-e ) � •316
(o) Prove from (a), that e = v4 2 and thence on taking
8x

natura l logarithms, that x = l-.. log (2v).


,e
-4x
Also from (B), show e l-2t, and thus that
l g (l-2t).
x = - i... o
e

EXAMPLE 4. ( i) The equation 91v1ng the velocity v �m /sec


of a�ticle at a displacement of x cm from the origin 0
is ./4-x2 /v = I. Initially it is I cm to the left of O;
ex press x in terms of the time t, and find x when
5n � �.������������--,..
t =
12, and t when x = �3. � 0 �
128
EXAMPLES GIVEN v, f IN TERMS OF x
dt _J__
Method (a) Show that 14_x2, and thus t = f ...J1x....
= ;4_x2
dx
d { -1 X ___l_ -1 X
Verify that sin ( )}-µ;::;;i, and hence t=sin ( )+c.
dx 2 X 2
-1 11
By data, when t O, x = -1, .•. O=sin (-\)+C, i.e. C=
6
-1 -1 11
[Note sin (-\) = - sin (�) = - )
6
-1 X 11 -1 X 11
(S) Hence prove t = sin ( ) + 6, i.e. sin ( ) = t -6
2 2
f
and thus = sin(t - !>,
i.e. x = 2 sin(t - ).
11
6
(y) When t = 22!., pro ve x = 2 sin = r2
11 �
12 4
and when x
� -1 /j 1 1 11
r3, show t = sin ( ) = i.e. after
2 + 6 2;

i; (i l• 31) sec, the particle is fi ('l' 1•41) cm from 0,


and it is distant /j (i 1 ° 73) cm from O after f (H•57)
sec.

(ii)The acceleration f cm/sec2 of a body Pis given


by f = 18x(2x +1), where x �m is the displacement of Pat
time t sec. Initially P starts from the origin with
velocity 3 cm/sec; show that v =- 3(x 2 +1)and x = tan3t.
[Assume v > OJ. Determine its velocity and displacement
after 11/12 sec.
d h4
Method (a) Using (�v2 ) = 18x(x 2 +1), show \v 2 = + 9x 2 + C,
dx
and prove C = f. 2
Hence prove v2 = 9(x 2 +1) 2 , and there­
fore = 3(x +1).
v
2

. dt 1 1 -1
(S) Using = J 2 show = tan x + C1, and
dx (x +l)' � 3
-
prove C1 = O. Hence show tan x = 3t, i.e. x = tan 3t.
11
(y) When t = , from x = tan 3t, show x 1, and from
12
v = 3(x +1), prove v = 6. Interpret these answers.
2

EXAMPLE 5. The velocity v of a particle along a straight


I ine Ox is increasing with respect to its distance x from
Oat a rate which is proportional to / x v. When the par­
ticle is ·at Oits velocity is 10 cm/sec; after it has
travelled 10 ft from O, its velocity is 20 cm/sec. What
fU:t"ther distance wil I the particle have travelled by
the time it attains the velocity of 30 cm/sec?
. why�dv kx
Method Explain
(a) =�, where k is constant, and
d dx v
d kx2
prove v � = kx ' i.e. �(\v ) = kx and hence J,v2 =
2 + C.
dx dx 2
Using the data x = O, v = 10 and x 10, v = 20, find the
values of k, C.
(B) Find x when v = 30, and thus answer the given
question.
129
SET 3G
EXAMPLE 6.
A particle moves in a straight --X.--.
°1 �
I i ne x 'Ox under the inf I uence of �· F+ X
a force F so that its acceleration at any point on the
I ine is directed towards O and varies inversely as the
square of its distance from O. If it starts from rest at
a distance of 3 m on the positive side of O and is then
accelerating at 16 m /sec 2 towards 0, find its velocity
when it is 2 m from O and its distance from.O when the
velocity is Sm/sec.
k
Method (a) Here f = - -2 , where k is a
--- constant.
d X k k
Using ( v 2 ) = - -�' show !,v 2 = - + C where C is con-
dx � x X
Stant.
Noting when x = 3, v = O, f = -16 prove k = 144, C = -48.
(S) Using these facts, complete the question.

EXErtCISES SET 3G
1. With the usual notation,
(i) v = 4/x (ii) v = 4x (iiil_v = -2x 3 (iv) v = 3 sec x
(v) v = 8eX (vi) v = 2!i=x2 (vii) v = l+x 2
(viii) v = cos 2 x (ix) v = cot 2 x
dt 1
In each case, start = -;;, and express t in tenns of
dx
x, given that t = 0 when x = 1.
2. In each of the following, express v 2 in terms of x,
given that v=O when x=O. [Start �(!,v ) = f]
2
dx
(i) f 3x 2 (ii) f �--
2x+l
( iii) f

(iv) f = (1-4x 2 )-� (v) f = (1+4x 2 )-l (vi) f sin 2x

rx
(vii) f = x 2 +3x-l

d
3. Prove that (x log x) = 1 + log x.
dx
The acceleration of a particle moving in a straight line
and starting from rest at unit distance on the positive
side of the origin, is given (in the usual notation) by
the equation f = 1 + log x. Calculate v when x = e 2 •
4. The acceleration of a particle P moving along the x axis
is given by x = -4x; if the particle starts from rest at
x= S, find the velocity of P when it first reaches x = 3.
car
ts. The brakes of a car produce a�
constant deceleration of ......,�o�==::-;,0.003°':\"rl.
�===;:--+-�­
k m /sec 2 • The car initially •o
-JOmJ5
at O and travelling at
130
SET 3G
30 m/s (i.e.108km/h) comes to rest in a distance of
100 m from O. Show that !('2v 2 ) = -k, and hence find
d
(i) the value of k
(ii) how far the car travels from the instant it reaches
.54km/h until it comes to rest.
6. The velocity v of a point P is given by the formula

!(v2 ) = -32x + 64, and v 0 when x = O.


d
(i) At what other displacement from the origin O is
v = 0?
(ii) What is the greatest value of v and where does this
value occur? ����������

7. For a body moving in a straight line, in the usual nota-
tion f = -2 , where x -> 1. Given that V = 0 when x "' 1,
2
X
show that the speed is 2 /1 - 1/x, and thus that it can-
not excee,d 2 .
a. A particle p moves in the s�raight line X'OX
so that its acceleration X when its dis-
placement from 0 is x, is given
1 1
by x = �+ 9 2 ' and hence, if Pis projected from 0
+x
with velocity In, prove that �v 2 = sin-1 tan-1{ + f+t f
Find (in terms of n), the speed of the particle when it
is 1:3 units on the (a) positive (b) negative side, of O.
9. A particle initially at rest at the origin, moves so
d
that its velocity v is determined by ( 2 )=sin2 x -cos3x.
dx !,v
Find an expression for v 2 in terms of x, and hence de­
termine the value of v when x = n/2.
10. The acceleration f m'sec 2 of a body moving in a ·straight
xe 2x+3
line x'Ox is given by f = ��-, where x mis the dis-
X
placement of the body from O. The body is projected
away from the �osition x = 1 with velocity e. Calculate
the value of v when x = 2 .

11. The velocity v of a particle is given by v = kx 3 where k


is a constant, and x is the displacement of the particle
from the point O in the line x'Ox.
1
Show that t = - 2 + C, and hence if x = 1 when t = 0
2 kx
and x = 2 when t = 1, find x when t = 2/3.
12. The velocity of a particle is inversely proportional to
the displacement from O. If x = 0 when t = 1, and x = 1
dx
when t = 4, find x when t = 4. [Start - k
-J .
3 dt X
131
SIMPL� HARMONIC MOTION
cos x 2
13. The velocity x of a body Pis given by x =
1 + cosx sinx
Show that t = I (sec2 x + tan x) dx = tanx - log(cosx)+C.
If p starts from the origin; find C, and the value of t
when X = 4•
11

14. The equation relating the velocity v and the displace­


ment x of a particle travelling in a straight line is
x 2 = (Sx-3+4x 3)v. If x = 1 when t = O, prove that
t = 5 log X + l + 2x 2 - 5.
X
15. Two bodies P, Qare travelling towards�--,t===�==�:L-�---•
the p.oint O on the line x' Ox, so that o! x.
their respective velocities when at a distance x cm from
0 are (100-x2 ) 2, 2S+x 2 cm/sec. Determine the time for
k

each body to reach O from a distance of 5 cm.


16. With the usual notation, we are given v = 2-Jx
(i) Calculate the value of f when x =!,
( ii) If x = 0 when t = O, show that t= 1 [log2-log(2-3x)),
2 -3t
3
and hence that�= (1 - e ), Find x when t = 1,
3
and also the greatest value of x for finite values
of t.

SECTION 7. SIMPLE HARMONIC MOTION


8_. DEFIN I TI ON
A special case of the motion of a particle in a straight
line which occurs very often in nature is known as simple
harmonic motion (or S .H.M.), In this motion, the particle
moves backwards and forwards in such a way that its acceler­
ation at any instant varies as its displacement from a fixe�
point (say O in the line x 'Ox). x' .x.
p
More precisely, if x be
0

the displacement of the particle P at time t from O, then the


d2 x
acceleration of P towards O is given by f=-kx, i.e. w= -kx,
where k is constant. For ease of oaloulations later, we let
k = n 2 , where n is a positive constant; and the result above
may be written as � = -n 2 x.
Thus if a point moves aooording to the UM x = -n x,
then the point is said to move in simpZe harmonio motion.

B. EXAMPLES IN NATURE
0

(i) A pendulum making small osoillations ,e


vert"11l :
about its equilibriWI position or ....,
equ,librivol T
The figure shows a simple pendulum position :
.....
(in which a small particle, called the ..., p
"'
I

l-�----

bob, is attached to one end of a mass­ "':I


less, torsion-less, inextensible string).
132
S.M.H. IN NATURE
The string is fixed at O and the particle F makes small
oscillations in an arc of a vertical circle about the verti-
cal (or equilibrium position), OL.
In the diagram, the particle P is in a position where
the string is displaceq a small angle 6 from the equilibrium
position OL. If the mass of the particle is m, the length of
the string is l, then it can be shown that the equation of
energy of the particle P is t�l 2a2-11ngl(l-cos6) = k, where k
is a constant and g the gravitational constant, [6 = ::, is
called the angular velocity of P).
Verify by differentiating with respect to t, that
(tml2 ) . 28.6 + mgl(sin6.6) = 0 [Note � (0 2 ) 2� (8))
d
and hence by simplification, show
la + g sin6 = 0 i.e. 6 = - f sin 6 , ...... , • .. .. • . • .. • (A)
However, for small 6, sin6 f 6 and, (A) becomes a,;-f e,
which is of the form x = -n x, where
2
n2 = 1·
Li'3ht tlastiL sp,in�
(ii) l!_xtension of light elastic: spring (or st!i_ryJ)
j
The diagram shows a particle P A £. _ ,
1 x...;
(of mass) attached to one end of a Smooth horiiontal table
light elastic spring (or string) of natural length l. The
spring and particle lie on a smooth horizontal table and the
sp ring is attached to the fixed point A on the table, The
spring is stretched (or compressed) and when the particle P
is released, P oscillates on the table.
The energy equation for the particle can be shown to be
l m ;2 + _Q_ x2 =
k, where k is constant, and A (the modulus
2 2l
of elasticity) is a constant depending on the spring (or
string).
Verify by differentiating with respect to t, that
• .. A • A which is of the
mxx + f xx = O, and hence that x
•• - ml x,
A
form x = -n 2 x, where n 2 = '
ml I
I

Osaillations of a Fartiale attaahed


I
(iii) I
to a light elastic: spring (or string) it
I
I
The diagram shows a Particle P of mass m I
I
attached to a light elastric spring (or + B
strtng) of natural length l and modulus A. ,a.
I
!,oil,�•...,
The spring is hanging vertically, and is sus­ T
I
c. -' Po1,t,on
pended from a fixed· point A. In the equili­ rx.
.L
F

brium position, the particle is at C where p


*BC = a. When the particle is displaced ver­ �
tically from its equilibrium position and
133
SOLUTION OF THE EQUATION x = -n 2 x
then released (the spring or string remaining taut), the
A
equation of motion of the particle is x • - . x, where x is
measured from. C. mf

iG
(iv) OsaiZZations of a Uniform Solid in 4'iquid
The figure shows a small uniform cylinder of mass m,
floating in a liquid,
Unifo,,., Cylinder

1
- -- ·--;-- -
d.,ityP
rr Liquid of -
- _- =- ly � --• ,� - --:�
den�i o- _ er
-+--- � - - -- -
_ _ _
- -c.,11...;4.,w ,n ;V'--;i,l,rivm
_
�-.:. l --__-
h1hen the ·cylinder is displaced vertically in the liquid
(without being completely immersed), and then released, it
oscillates about its equilibrium position,
The equation of motion of the centre of gravity of the
..
cylinder is given x = - Qg_
pl x, where u,r are the densities of
t he liquid and cylinder respecti vely, (p < u) and l is the
length of the cylinder.

T
(v} Vibrations of a Tuning Fork � ..... ___ �l!_.J' ...... �
In the figure CP is a tuning fork & A
held in a clamp at C, the equilibrium :
position being CO. The free end P has 1
been plucked aside to A and then left
free to vibrate. If the vibrations are G
small, then the path of P may be re-
garded as approximately a straight line, The elastic force
in the fork tending to restore P to the position O is propor­
tional to the distance OP, i.e. x. Howev er, since this force
..
d2x i.e. x,
is itself proportional to� then x is proportional
to x. Since the force tending to return P to O is negati ve
in direction, then x = -n2 x, where n 2 is constant.

The motion of a point in a viol in string when the


string is plucked sideways is asimilar example of S.H.M.

SOLUTI�� OF THE SIMPLE HARMONIC EQUATION x


(i) ·· -- -- ·· · · ·
Repression of the veZoaity v in
'terms of x
.. ·d· ·x dv dv d ,
p
Now x = dt" = dt = v dx = d x ( v� ) 0------
I x----->-
H ,x
;;
are various forms. for the acceleration,
We use the form x = _Q_(J,v2 )., and thus dx (J,v 2 ) - -n2 x,
dx 134 d
SOLUTION OF EQUATION x = -n 2 x

••••••••• (1)
aonditions (aaZZed
In this case., let
is at rest when it is d istant

V ""
0 a. X
n2 a 2 2 a2
n
Now from (1), • •• 0"' - --2- + C1, i.e. C1 = -2 --

n2x2 2a2
and thus (1) becomes � 2 • - � � + n ,
2 �2�
i.e. v 2 • n 2 (a 2 -x2) ....•..•.......•.••. (L)
This result v 2 • n 2 (a 2 -x2) expresses v in terms of x.
Note. The veZoaity in an�ition x is in�, but the
speed is then j in/a.1-xZ j, i.e. n�.
(ii) Expres�ion of x in terms of the time t
From (L), we have v= infa .!-x2 , where -a� x Sa.
dx dx ,.,-.,
Thus, since v "' ' we obtain dt= inra··-x-
dt
dt dx
and because = 1 I dt' . • dt "' i

J J � ............
dx dx n�
7=i2
For convenience, we usually write this as i �! "' nva2=x2•
and hence that it= �dx= 1 (2)
nva'-x· n ,a--x

Noting tha t d(sin-1 X) 1 1 1


dx a v'l-(x/a)2 · a 7a2?
and that �(cos -1 �) -- - • 1
dx a ll-(x/a)2 a
then the equation (2) above, gives
it= 1 sin-1 � + C 2 ••. (3 ) or it = - 1 cos-1 � + C 3······ (4)
n a n a
where C2, C3 are constants.

To find c2, c3 we asswne oth(!�-��ry_ aonditions. In


the present case, let us assume the x t•O
particle is initially a units from ���====:::::!����-1--+
O, on the positive side of O, i.e. 0 a.
when t= O, x= +a.
Henae from (3), we have
0 = 1 sin -1(�)+C 2 i.e. c 2= - lsin-1 (1)
n a n
1,
-I

From the sl;etch, sin-1(1) =


and .·. c 2 = - 2:. .11'/.:i _ - ------
Thus (3) becomes
it=* sin-1; - : i.e.* sin-1;= ( ; i t)
2 2
135
SOLUTION OF THE EQUATION x � -n 2 x
i.e. sin-1 :!. = (.!!. t nt)
a 2
!lnd • • • :!. = sin(.!!. t nt), i.e. x = a sin(-n t nt).
a 2 2
n
Now sin(- t nt) = sin
2
n
2
cos nt ± cos
n
2
sin nt,
using sin(A±B) results
n n
cos nt, since sin
2 = 1, cos
2= 0
n
Thus x = a sin( ± nt) becomes x = a cos nt •.•..•.•..... (�)
2
Also, from (4), since when t=O, ___ _!f 'j ______ _
x=a, i.e. 0 = - �os-1 1 + C3,
ln
From the sketch, cos-1 1 = 0,
and.·. c 3 = O.
Hence (4) becomes tt = - 1cos-l:!.
n a "
1 -1 0
1 1
i.e. - cos- :!. =. t, i.e. cos- � = ,nt and thus x=a cos(,nt)
n a a
Now cos(-nt) = cos(+nt) = cos nt, since cos(-6) cos e.
Thus x = a cos(,nt) becomes x = a cos nt, which is, of course
identical to (M) above.
Notes.
(i) Since the use of cos-1(�) usually leads to easier calcu­
a
lations, then it is preferred in general to the use of
sin-1�.
a
(ii) In the use of these, note that whilst
sin-1(-b} = - sin-1(b}, cos-1(-p) = n - cos-1(p) ,
where b, p are both less than or equal to 1 in absolute
value.
Thus sin -I(-!,) - sin-I(!,) = - f;
cos-1 (-!,) = n - cos-1 (!,)
2n
3'
SUMMARY. For a particl.� moving in simple harmonic motion,
accordina to the law x = -n 2 x, and when t=O, v=O, x=+a,
then v 2 � n 2 (a 2 -x 2 ) and x = a cos nt.
[NB Also v = x = dd tx = - a n sin nt]

NOTES ON THE ABOVE RESULTS


X -n 2 x v 2 n 2 (a 2 -x 2 ) x = a cos nt
t=O
�· ���-,+-���-+������===�====�p������:.._ __.:X
=O

(·11,0) A' (·x,o) A (a. ,o)


-,-
0
(i) The Path of P.

136
NOTES ON TERMS IN S.H.M.
The particle then starts moving towards O with increas­
in� �pe�d; its velocity in position xi, where O < x1 < a, is
-n a -x1 • It reaches its maximum speed at O, when x = o,
v = -na, f = O. (Note "the J• o v=o
minimum acceleration of Pfrom O x, A
� = -n 2 x occurs at 0.) Pthen continues to travel in the
negative sense with decreasing 4-��A�'���-:t:-���+-���-
speed, its velocity in posi- x' -o. ·X.1. O
tion-x 2 , where -a < -x2 < O, is -n/a2-x 2 2 , The acceleration
then is given by x = +n 2 x 2 , and is directed towards O
(i.e. the acceleration of P is directed towards O for all
positions of the particle in the interval A'A.)
When Preach.es A' • the particle comes again to rest,
since v = 0 when x = -a; also lf<O �=o
x then is + n 2 a and thus P A' 0
changes direction. Pthen returns tbwards 0 with a velocity
v = +n� , and accelera- A'
�4 ����
tion f = n 2 x3, where -a<-x3<0,��_Fo.====�'--4_x-��
1 o
whilst at O its velocity is na and its acceleration is O.
It continues through 0 towards A with velocity
v = +n�, where O < xi+ < a
and acceleration f = -n 2 xt+, 0 .><+
i.e. directed towards O. Pthen returns to A, where x =a and
v = 0 again. After this, the 0

������==����,==�
whole process is repeated, and
the particle Pcontinues to oscillate about the central point
0 between the points x = -a and c:::t •
X = +a,
A' 0
The greatest speed occurs at O, i.e. when x = O; i�s
value is na, whilst the least speed occurs at A, A' i.e. when
X = ±a; its value is zero (i.e. when Pcomes to rest).
The �reatest acceleration occurs at A,A' and its magni­
tude is n a, whilst the least acceleration is at O, and its
magnitude is O.

(ii) Terms
(ex) The maximwn displacement 'a ' of the par>tiale from O is
called the ,amplitude of P. It
is half the length of the path
A (a,o) x
along which the particle is moving.0
(ll) The displacement of Pat time t is given by x = a cos nt
(its velocity v then is given by v = -an sin nt). The same
displacement (and velocity) are obtained by replacing t by
211 211
(t + -) . Then x=a cos n(t +-)=a cos (nt+2n) a cos nt
n n
211
whilst v = -an sin n(t + - ) = -an sin(nt + 211) -an sin nt= •
r1 .i-c t o
,$
Hence the particle re- l 1 ;A
turns to the same position and
A O :
' t·�••'9"E j
is travelling in the same direction after 11!. units of time
n
137
NOTES ON S.H.M.
(say seconds). Thus P.returns to any position on �'A after
211 the time
� sec; i.e. T for one complete oscillation is
211
-sec. This is called the period of the simple harmonic
motion.
211
(y) Now the time for one complete oscillation is� sec,i.e.
there are· n oscillations in each 211 sec� i.e. · ;11 osci Uations
per second. This is called the fr'eLiuenay of the S.H.M., and
1 n
is denoted by v (Note v
= T= )
211 ,
(iii) The oscillatorn nature of S.H.M. can be seen from the

I I
graph of x = a cos nt.
t 0
11 211 3.r .2.
2n 2n 2n 2n

X a 0 -a 0 a
X.
,c •o.CDS l\t

(iv) �lternative Forms for S.H.M.


.Alternative forms for the final expression of x in terms
of t are possible by taking different boundary conditions.
(a) For exconple, from ±t = 1 sin-1 (�) + C2
• i :o
�' 0
n
(3)
a · · · · ' 'i/o'
x �
=
let us assume initially Pis at O, i.e.
when t = O, x = 0 (and Pis moving in �
-,. direction). ------ Ii
Hence from (3),
0 = 1 sin-1(0) + C2, i.e. C� 0
n =
Thus from (3), ±t = 1 sin-1(�),
a
n
i.e. x = a sin(±nt), -�------
and thus x = ±a sin nt (since sin(-0) = -sin0).
However, if the body is moving in-,. direction, then
x = a sin nt.

*(B) The General Case.


If we replace C2 by _g_ in (3), then
n
l sin··l(�) = - �
n
t t' i.e. x = a sin(tnt-a)
n a
138
Slff!ARY
TIius x • a sin(nt-a)••••caee 1 Ol' x • a sin(-nt-a)•.•••case 2
Let£• -a in case 1, i.e. x .. a sin(nt+t)
or let£• n+a in case 2, i.e. x = a sin(-nt+n-E)
• a sin[n-(nt+E)]
.. a sin(nt+E)
dince sin(n�e) • sine
Hence the genePaZ soiution of x•
-n 2 x for x in terms of
t can be expressed as x = a sin(nt+E) •••.•..••.••••••••. (5)
Further if we let E = f + E', then (5) becomes
x • a sin(nt + 2!.+ E 1
) = a sin(2!. + nt+E1)
2 2
= a cos(nt+E'), using the sin(A+B) result,
On expansion of (5), we have
x = a{sin nt cosE + cos nt sinE}•(asinE)cos nt + (acosE)sin nt
• A cos nt + B sin nt, where A• a sin E, B • a cos E,
[The constants A,B,E,E 1 are related by E •
a COSE c B]
f+E 1
, a sinE = A,

Hence the general solution of x m -n 2 x for x in terms of


t may be written as
x = a sin(nt+E) = a cos(nt+E') • A cos nt + B sin nt

SUMMARY. For a particle moving towards the fixed point 0,


and undergoing an acceleration proportional to its dis­
placement from 0, i.e. such that x
= -n 2x, where n is a
positive constant, then
(a) v 2 = n 2 (a 2 -x 2), where v = 0 1,hen x = ta ('a' is the
amplltude of the motion).
(b) Speciai results for X in terms of t are
( i) X =
a cos nt, where t = 0, X +a
{Here v = - an sin nt]
( i i) X a sin nt, where t = O, X = 0 and moving in
- direction. [Here V = an cos nt]
(cl The genePaZ solution of the equation x = -n 2x may be
given as
x = a sin(nt+E) = a cos(nt+E 1) = A cos nt + B sin nt.
2n
(d) The period of the motion T = units of time, and
--;,
the fPequency of the $.H.M. is
I
V = - = : osci I iations/unit time.
T 2
[� AU exel'cises on S.ll.M. should be done by diffel'entia­
tion ol' integl'ation. The above roesults should n ot be
quoted, 1.rith the exception pel'haps fol' the period
n
2n
sec.]

139
EXAMPLES ON S.H.M.
The fol7,awing examples should be worked by you.
EXAMPLE I. A particle P moves according to the law
x = 4 sin 3t, where x cm· is the displacement of P from 0
at time t sec.

0 p 4 X

( i) Express the velocity v cm /sec in terms of


(a) t Cb) x
Method Show v = 12 cos 3t, and hence that
v
2
= 144 cos 3t = 144(1-sin�3t) = 144(1 -
x•
)
16
= 9(16-x )
2

Ci i) Verify by direct differentiation that x = 4 sin 3t


satisfies an equation of the form x n 2x,
Method Show x = -36 sin Jt = -9x, where n 2 9 (and n = 3)

(iii) Find the displacement, velocity and acceleration


after 4TT sec.
9
4TT 4TT
Met1wd When t =
9•
show x 4 sin
3
= 4 X (- /J)
2
-2/3;

V =
12 COS
4TT
3
-6•, =
4 TT r., (or f = -9x = -9 X -2,13)

f = -36 sin
4TT
3 = 18,13
r,;
i.e. after
9 sec, P is 213 cm to the left ot O; is then
travelling at 6 crn/sec in
�---�,--���-+-����--+
- direction, and is J../J o x
undergoing an acceleration of 18/3 cm /sec towards 0.

(iv) Oeiennine when the particle is 2 crn on the positive


side of 0, and its acceleration then.
Method Show when x = 2, 4 sin 3t = 2, i.e. sin 3t = � and

thus 3t = 6'
TT
(TT - 6),
TT
(2TT +
TT
6) ,5TT
(2 TT + 6), (4TT +
TT
6),
5TT
(4TT +
6), . .,
TT 5TT l)TT 17TT 2 5TT 2 9TT
i.e, t =
18' 18' 18' 18, 18' 18'
TT 5TT !)TT
cm
Thus the particle is 2 from O after ,
18 18, 18,,,
sec; (the first occasion being after

18
"' --ii3 ';' ·
TT , 3•142 .
175 sec. )
The acceleration of P when x +2 is given = by
f = -9(2) = -18 cm/sec 2 i.e,18c m/sec 2 towards O.
140
EXAMPLES ON S.H.M.
(v) Calculate when P is at rest I

Method. When P is at the extremities of its path, v = O.


Show that when v = O, 12 cos 3t O,
1r 31T 5,r 71T • 1r 1r 5,r 7,r
i.e. Jt "' 2' 2 ' 2' 2'' ·' and · t = ' ' 6' 6' · '·
6 2
1T 1T 51T
Thus P is at rest after 6, 2,
6, ... sec (the first occa-
1r. 3·142 .
sion being after
6 7 �6� 7 · 524 sec)
(vi) Find the amp I itude and the period of osci I lation
of P.

Method. From x = 4 sin 3t, the maximum value of x (i.e. the


��-amplitude of P) is 4, since -1 � sin 3t � 1.
2
The period of oscillation of P is ; = 2; sec. [This may
11 1T 51T
also be obtained by noting that v=O when t =
6, 2, 6,,,,
sec anu t'ius the period of oscillation is
S,r n 211 n
(6 ·- 6) = sec or 2 (2 - ) sec.]
11
3 6

(Vii) Sketch the graph of x against t.

Verify the sketch shown,


X
4

EXAMPLE 2. A particle osci I I ates back and forth in a


straight I ine with velocity v cm/sec in any position x cm,
where v is given by the equation v 2 = 16x-4x 2 +20. Prove
that the motion is simple harmonic; find (i) the period
of motion and (ii) the length of the path.

Method. Show that J.,v 2 = 8x-2x 2 +10, and using x = f = /<JpJ 2 ),


prove that�= 8-4x = -4(x-2) .......•...•....�.. (1)
' d d dx •
let X = x-2, . · . X = X = (x-2) = dt = x, and
dt dt
Now

d2 x d · d · -
X = W = dt( X ) = dt(x) = x
Hence.. the e1uation (1) becomes X = -4X, which is of the
form x = -n x where n "' 2.
141
EXAMPLES ON S.H.M.
Thus the particle moves in S.H.M.; the centre of the
motion being at X • O,
i.e. where x - 2 • O, -,- 1__,___2.
I______I
�-----1:•,--__. s...._
i.e. at the point C, x• t, Q c. A x
J.: • J

(i) Also the particle changes direction when v = o, i. �. when


16x-4x 2+20 a i.e. when x 2 -4x-5
o, i.e. when x"' 5 c o,
or �1. These give the points A, B in the diagram. Hence
the particle oscillates between the points A (x"' 5) and
B (x"' -1); i.e. the length of the path is 5-(-1) =6 cro.
211 211
(ii) The period of the motion is --; • � • 11 sec, i.e. the
particle takes 11 (� 3•142) sec to travel from say B to A
and then back to B.

EXAMPLE 3, A particle P moves in simple harmonic motion


along the straight I ine x'Ox, so that its acceleration x
is given in terms of its displacement x from 0, by x=-4x.

( i) Initially it is 3 cm to the t. ... o


right of O and is projected ---+-----�x�.=3!1'----+x
CQ,Jay from O wi th a ve Ioci ty O .,. • , ... ""
of Ecnisec. Show that the speed of the particle is
Z�cm/sec, and find the position of P at time t.
Determine also (a) the amp Iitude and period of the
motion (Bl the greatest speed and accelerat:on of P
(y) the distance from O when the speed is 4cm/sec
(6) the position and velocit� of P after 11/6 sec
(£) the first occasion when the particle P is dis-
3 cm to the left of O.
tant 72

Method, Using the form !(!,v 2 ) for x, prove �v 2 = -2x 2 + c 1


d
Dy data, when x = +3, v = +6, and hence show C1 = 36, and
thus v 2 = 4(18-x 2 ),
Hence the veloaity i� position x is t2/18-x2 cm/sec, and
the corresponding speed is I t2/18-x I 2
= 21l8-x2 cm /sec.
To find x in te1'111s of t� first show that
an,\ thus that! t = - -1
t i
=
2Jia-x• ,
cos ( /2) + C J .............. (L)
!�
3
[As noted previously, the use of cos-1 (.!.) instead of
a
sin - (.!.) is preferable in most cases.]
a
1

- 1 11
Noting that when t = O, x +3, show 2 = !�os 1 (� = c 8
Hence from (L), prove tt -
- �cos 1 (37z) + i•
' -J 11
1.,e. cos ( and thus x = 3.,L 2t)..( M)
J7i =
4; 2. t r;::2 cos (
4;
X 11
)

142
EXAMPLES ON S.H.M.
[From both these values, we can verify t_hat when t • O,
X '" +3]
To distinguish between these resu.Us in (MJ, we may pro­
ceed as follows:-
On differentiatin_g_ (M) with respect to t,
••• v = -312 sin(n/4 • 2!) x +2 and hence initial­
ly (i.e. when t O), v = =
(-312 sin n/4) x �2 ±6, =
However, by data, initially, v = +6 and thus the result
for v producing this value,
namely v = -312 sin(f - 2t) x -2= 6/2 sin(f - 2t) is
the correct (and only) possible value of v. The corres­
ponding value of x is x = 312 cos({" - 2t).
[Note the extra piece of information supplied in the
data, namely wl,en t = O, v > O, enables us to determine
which of the two possible forms for x in terms of t, is
applicable here. We could not distinguish between these
two forms, if the sign of v were not given.]

(o) From x = 3/2 cos({" - 2t), the maximum value of x is 312,


since I cos<{ - 2t) I � 1. Thus the amplitude of the
motion is 3/2 cm.
n
The period of the motion is 11!. =Z = Tl sec.
n 2
[Since x -4x, n 2] = =
(B) The greatest speed of P occurs when 2118-xz is greatest,
and this !_akes place when x = O. This greatest speed is
2/ia = 612 cm/sec.
The greatest aaaeler�tion of P occurs when is great, !xi
est, i.e. when l-4xT is a maximum. This takes_ylace when
x = ±3/2, This greatest acceleraHon is 1212 ·cm /sec 2 •
(Y') 1-lhen the speed is 4 cm /sea, , ' , 2 /1s-xz = 4, and thus on
squaring 18-x2 = 4, i.e. x t/i4, Thus the speed is
4 cm/sec when the particle is 114cm from O on either
si.de of it.
(ii) After n/6 sea, the displacement x of P is given by
� Tl Tl � Tl Tl Tl Tl
x = 3,2 cos(
4- 2 •
6) = 3,2[cos cos
4 3+ sin sin J,
4 3
from cos(A-B) result
= 3(1+13)/2 cm.,

The corresponding veZoa ibJ of P is given by


v = 6v'2 sin(.1!. - .1!.) = 6/2[si� cos.1!. - co� sin.1!. J
� 3 4 3 4 3
= 3(1-13) CD)lsec. Since v < O, P is moving in ----+-

143
EXAMPLES ON S.H.M.
3 3 � Tr Tr
(£) When x = -
Q
' ,•.-72=3v2 cos(
4-2t), i.e. -�cos(
4- 2t)

Tl Tl
Noting cos(
4- 2t) = cos(2t - ),
4
2Tl 4Tr 2Tl
••• 2t - 4Tr = 3' 3' (211 + 3>. and the

first occasion occurs after t = �( 2; + f> =


lllr
24 sec,

(ii) If initially the particle in the above qu�stion is


3 cm to the f!!.l} of 0
and is projected (!l-!!!!:lf �o
1--����---
from O with a veloc- ���-x+�---�����I0 �
ity of 6 cm /sec, v-:-(,
determine the corresponding results.

\fethod. As before, v =1 2118-x2, and ±t7-�os-1(372)+c3 ..(N)

llere when t = O, x = - 3 , and ':!

thus O = -Y,cos-1 (- "fz') + C3

i.e.

)(
-I 0 Y.Ji I
- 3
Hence from (N), ±t = -J.,cos 1 ( 72) + ;,
and thus x 3 12 cos <
3
3

Z;
2t) ••......•.. , ... , •...•...(Q)

(When t = 0, both values of x give x = -3)


3
However v = - 3 12 sin( : + 2t) x; 2 = ± 612 sin( :
3
+ 2 t),
from (Q)
3 1
Hence,. initially v = ±612 sin( :) = ± 612, (+ 72) = ± 6,

and thus the


3
OOT'T'eat Value of V is V = -6/z sin( Z + 2t),

since v -6 by data when t = O. The corresponding value

of x is x = 312 cos(1 + 2t) •.....•.•••....•.... , •• (R)

(Note again the additional data that when t = O, v < 0


leads to one and only one value of x in terms of t.J

Verify the following results

(a) The amplitude of tne motion is 3 /2 cm•and its period is


11 sec.
(B) The greatest speed and acceleration are 6/2 cm/sec and
12/2 c m/sec 2 respectively.
(y) When the speed is 4 cm /sec, P is distant t/i4 cm from O.

144
SET 3H
(o) After fsec, P is distant 3(r'3+1)/2 ,C3. to the left of O
and moving with a velocity of 3(v'3-l),cm. /sec in -
direction.
3 JTT 2TT 4TT 2TT ,
(e:) When x = - 7i• prove 2t + ,, 3, (2TT +
3)7TT
=
4 3 14TT TT
and th.e first occasion is after \[� - 3 ] = sec,
4 24

EXERCISES SET 3H
1. Show by direct differentiation that each of the follow­
ing forms represent examples of simple harmonic motion,
i.e. show that x = -n 2 x in each case.
(i) x = a cos nt (ii) x = b sin nt
(iii) x = c sin(nt+e:) (iv) x = d cos(nt+e:)
(v) x = A cos nt + B sin nt
[Note a,b,c,d,A,B,e:,n are constants;]

2. A particle P moves towards a


fixed point O under the in­ 04----X._____ ,.. X
fluence of a force F directed towards o. This force pro­
duces an acceleration which varies as the distance from
O. Initially the particle starts from rest, 4 cm on
the positive side of O and is then accelerating at
64 crn/sec 2 towards 0.

x
( tl) Start x = -kx (or� =-n 2 x, where n is a positive
constant); show that k = 16, since = -64 when
x = +4. Hence show �2 = -8x 2 +c1, where C1 = 128,

x
since v = 0 when x = +4.
Thus prove = -16x and v 2 = 16(16-x 2 ).
( tl) Now sh ow that ± dt - 1
, and thus that
dx - -4�11�6=_=x-2
cos- 1 (X ) + Cz. Prove Cz = �cos- 1 (1) = o.
-1
±t =
4 4
Hence prove x = 4 cos(±4t) = 4 cos 4t, and thus
v = -16 sin 4t.
(y) Find the velocity of the particle
(i) at 0 (ii) 2 cm to the right of 0
(o) Determine the
(i) greatest speed and acceleration of the
particle
(ii) amplitude and period of motion of p
(iii) distance from 0 and velocity of p after
TT/16 sec.
(iv) first time when P is 2 cm to the
(a) right of 0 (b) left of 0
3. The equation of the motion of a particle�---+��-"'...,.=-'--�
P moving along the line x'Ox is given by o' P
x = -9x, where x cm is the displacement of P from 0
145
SET 3H
a
and xcmi/sec its acceleration then. Initially P is
4 cm on the positive side of O and is projected
towards O with a velocity of J.2 cm /sec.
(i) Prove that its speed in position x is given by
3132-x2 cm1sec. 1
(ii) Show that tt = - cos -1(472)+C1, and prove C1� ;
3 1
Hence show x = 4/i cos({+ 3t), and by considera-
tion of the sign of� (by data, x• -12 when t•O),
verify that x • 4/i cos({+ 3t), and thus
V • -12f'i sin({+ 3t).
(iii) Determine the
(a) velocity of P
(a) when Pis 2 cm to the right of 0
(8) after n/6 sec.
(b) the greateet
(a) speed of P (8) acceleration of P
(y) displacement of Pfrom O.
t .. - distance of P from 0
(a) when the speed is 6 cm/sec
(8) after 7n/12 sec.
(d) time of a comr lete oscillation of P
(e) time when Pis 2/6 cm (a) on the positive side
of O (8) on the negative side. of O.

4. 11\e retardation of a particle P moving in the straight


line x'Ox is 4x, where x is the displacement of P fromQ
11\e velocity v 3 when. x = 2. Prove that the velocity v
25
is given by v = 4(-;; - x2 ), and find
2
(i) the velocity at 0
(ii) the greatest positive value of x attained during
the motion (iii) the period of the motion

5. A particle is fastened to the �


midpoint P of an elastic �
string fixed at its ends to A o 8
two points A and B on a horizontal table. Pis pulled a
distance of 2 m at right angles to AB and released. If
its velocity is vm/min when P is displaced x m from O,
the midpoint of ia, then V and X are related by the
d !� 2
equation ( ) + 256x = 0. Find the velocity with
dx
which it passes through its equilibrium position.
A
6. A body is suspended from a point A by a spiral
spring; its position of rest (i.e. equilibrium
position) is B. It is pulled down Bernand then-,;
let go. If its velocity is vCJ'QJsec when it is �
lll;j.x.
d2
xcmfrom B, then it is known that
dt� = - �. �
146
SET 3H
Find its velocity when passing B and the height above B
to which it rises. How far should it have been pulled
down if it is to pass B with a velocity of 4crrf,ec?
7. A particle oscillates according
to the law x = -n2x between the Ac�-a.\ 'o '
\\�,c.
points A, B shown, where •AB= 4 m If its speed when
passing through O, the midpoint of 'AB' is 15m/ min prove
n 2x 2 1� 2 5
that .J,v 2 = - �� + C, where n= , C= ; • Hence
2
find the speed and acceleration of the particle when it
is·} m from 0.
8. A particle moves towards a fixed point O with an
acceleration which varies as its distance from 0. Its
velocity when 4 cm to the positive side of O is 20 cm/soc
and its acceleration is then 20/3 cm /sec 2 . It started
from rest a cm from O; calculate the numerical value
of a. [Hint prove iv 2 = - 5 x2 + �
640
-J
6
I

9. (i) A particle starts with velocity V and moves in a


straight line under a retarding force producing an
acceleration equal to k X(distance described).
Prove that the particle first comes to rest after
traversing a distance V//k.
[Start x = -kx, assume when x = O, v = VJ
•({i) A particle moves in a straight line with S.H.M.
and_yasses through its mean position with speed
1013 cm/sec. Calculate the speed when the particle
is halfway between its mean position and a point
of instantaneous rest. [Begin x = -n 2 x; work in
terms of n, which disap.pears in the final result.]
10. A particle is moving in S.H.M. about the central
position O on the x axis. The equation giving its dis­
placement from x at time t sec is
11t
x = a cos(�+£), where a, £ are constants (O � £ � 11)
When t = 2 sec, it passes through th� origin 0, and when
t = 4 sec, its velocity is 4 cm /sec in the negative
direction. Prove that£= 11/4 and the amplitude 'a' of
the path is 32r'2/11 cm.
�����������
11. The position coordinate x of a point moving in a
straight line is given at tlme t by x a cos(nt+£),
where a,n,£ are constant�. Find
(a) the period of the motion (b) the speed as a function
of (i) t (ii) x [Hint for (ii), square result in (:i) )
(c) the acceleration as a function of x.
tl2. (i) Define simple harmonic motion, and from your defi­
nition, find a formula for the speed in terms of
the displacement from the centre O of the motion,
given that the speed is zero at displacement

147
HOOKE'S LAW
a from O, and find the period.
(ii) A particle moving with S.H.M. in a straight line
has speeds vi, v2 when distant xi, x 2 respectively
from the centre. Pr·ove that the period of the
motion is 2nl(x1 2-x 22)/(vf: -v1 2)
[Hint: eliminate a2 from 2 equations.)
(iii) In a particular case of (i), the velocity of the
particle when it is 3 ..:m , 4 ,:-:n to the right of 0
is 8 cm/sec, 6 cm /sec respectively, and is direct­
ed away from 0,
_2i _ n 2 (a 2 -9)
Prov e that Hence find the ampli-
36 - n2(a2_16)'
tude 'a' and period of the motion of P, and the
velocity of P at 0,
13, Prove that if the displacement x of a particle P is re­
lated to the time t by the formula x = 3 cos 2nt, the
motion is simple harmonic. Find the values of the
(i) initial velocity and acceleration
(ii) greatest displacement, velocity and acceleration of
P.

14, A particle moves backwards and forwards in a straight


line, with velocity v cm /sec in any position x cm from a
fixed point O, where v 2 = -9x 2 +18x+27. Prove that the
motion is simple harmonic, and find the centre of the
motion. Also determine the period of the motion and the
length of the path,
15. The speed v cm /sec of a point moving along the x axis is
given by v 2 = 8-10x-3x2 , where x is in cm. Prove that
the motion is simple harmonic, and find
(a) the period (b) the centre (c) the amplitude
of the motion.

SECTION 8. HOOKE'S LAW

�. INTRODUCTION
An inextensible string or cord is one which would bear
any tension without altering its length. In practice however,
all strings are extensible, (i.e. they can be extended),
although the extensibility is extremely small in many
cases, and practically negligible in others, When the exten­
sibility cannot be neglected, there is a simple experimental
law (discovered by Hooke, 1635 - 1703) connecting the tension
of the string with the amount of extension of the string.
This law, known as Hooke's LaJ,J, can be expressed in the
form "The tension of an elastia strinfj (Ol' aord or sprintJ)
148
HOOKE'S LAW
varies as the extension of the string beyond its natUX'al
length."
B. FORMULA
In the diagram, the unstretched �
(or natural) length of an elastic
string or spring is 'a' units, and T
is the tension when it is stretched to
The extension is now 'x' units,
Hooke's Law states that T varies as [(a+x) - a)
i.e. T = kx where k is a constant. [The constant k depends on
the size and material of the string or·spring; it is often
referred to as the 'string' or 'spring' constant. It is also
called the 'stiffness I of the string or spring.)
Sometimes another. constant, namely A, is introduced,
where A is defined by k = 11/a i.e. A = ka. The quantity A
depends only on the thickness of the string or spring and on
the material of which it is made. A is called the modulus of
eZasticity of the string or spring. Thus Hooke's Law may be
written as T kx� x.

ra.-r-- .... �
=
a
[A is equal to the tension which would stretch a string, if
placed on a smooth horizontal table,
to twice its natural length. When
A
x = a, then T = -; , a = A) �'

C. POINTS TO NOTE
1, All possible complications due to the mass and weight of
the string or spring are neglected; i.e. we consider
only light strings or springs.
2. Hooke's law holds only up to a certain point in an
elastic string or s_pring, since there is a limit to the
extension of the string or spring. For a certain exten­
sion, the tens ion varies as the extension. (When the
load is removed, the string or spring returns to its
original length,) Beyond this extension, permanent
change in the string or spring will occur and Hooke's
law will not apply.
3. Hooke's law holds also for steel, iron and other bars
but the extensions here for which it is true are
extremely small.
4. For two springs (or strings etc) different in length but
otherwise alike, the stiffness k = T/x is not the same
for the springs.
5. Whilst a string (or wire etc) can only be stretched, a
spring can also be compressed. o.. F
(To produce this compression, ·- �:.;l•
149
HOOKE'S LAW AND S.H.M.
we must apply a force F at the end; the "stretch" x then
becomes negative). Hooke's law is still applicable.
6. There is no intention here to proceed on to the theory
of dynamics, Terms such as mass, tension, force are un­
defined here; the instinctive concepts of these are
intended.

(a) In the SI s stem used in this section, the wiit of displacement is


*
tTie metre (m}, t e wiit of mass is the kiZogrcun (kg), and the wiit of
fome (or tension) is the nelJJton (N). The wiits for k are newtons/
metre (N/m) and for>. are newtons (N).

{Other systems which are or have been used, are mentioned below:
In the British s stem (the f.p.s. system), the unit of displacement is
the foot (ft), tfie unit of mass is the pound (lb), and the unit of
force (or tension) is the poundal(pdl). The units fork are poundals/
foot (pdls/ft) and for>. are poundals (pdls).
In the metric s stem, the corresponding units are the centimetre (cm),
the gram (g) ana dyne (dyne). The units fork are dyne/cm and for>.
are dynes.}
(b) In dynamics, the force F acting on a body of mass m
is measured by the product of the mass of the body
and the acceleration f of the body i.e. F = mf.
(The direction of the force is that of the accelera-
tion.)
f -- dv __ d2x dv
Since ---..- v = -9.(Yiv2) ,-
dtL
X
dt dx dx _
dv d 2x .. dv d
then F = m = m � = mx mv - = m -(Yiv2 )
dt dx dx
(c) For further information on the above, consult appro­
priate textbooks on applied mathematics, dynamics,
mechanics and the like.

7. A string or spring obeying Hooke's law is said to be


perfectly elastic.

t
D, HOOKE'S LAW AND SIMPLE.HARMONIC MOTION

attached to the end of a string


. le P � Ek:as4ic. ,+,,,,,, cord, o,. wi..e
The d.1agrams s how a part1c

(or spring) of natural length 'a',


A 1'
"'
o T p

.+ .
-x.-
m.

;::�-?
and the string (or spring) is O,;�,,, or e,.,·,libr;rn PM,t,on
stretched to a length (a+x), then T P
,n
Hooke's Law states that the'""'n ;�
particle P will suffer a force El•s+.o 5pring
towards A (where it is fixed to a wall or tab"le say). The
magnitude of this force (T say), is given, according to
Hooke's law, by T = kx, where k is the stiffness of the
string or spring. [This force exerted by the string or
150
EXAMPLES ON HOOKE'S LAW
spring on the particle P is a force trying to return the
string or spring to its natural length; it is sometil!les re­
ferred to as the linear restoring forae.]
Now since the mass of the particle is m, the force F ex­
erted by the particle (on the string or spring) in the
positive direation, (i.e. (J);)ay from O, i.e. in---,. direction)
is, by definition Section C, 6(b) above,
given by F = mf = mx. I
0
T p F
x.1 I "
Now since F, Tact in opposite directions, then F • -T
Hence, since F = mx (by definition of a force) and Ts kx (by
Hooke's Law),.·. mx = -kx.
..
TIius x = - ;k x, and this is of the form x = -n2 x, where
n2 = 1,
m
i.e. n = IE,
;;
That is, the pax>tiale P moves in S.H.M. about the

Ji
aentral (or equilibrium) position 0, The time for 1 complete
osaiUation is 2� = 211 units of time (say seaonds). 'rhe
frequency of osciUation (i. e, the number of osai llations per
second) is given by v = 2; /f
EXAMPLE I. A perfectly elastic cord st retches to lengths
of 40 m, 45 m under tensions of 3g, 5g newtons
respectively. Find the natural length of the cord, and
the tens ion when it is st retched to 60 m

Method.
According to Hooke's Law, if*�.,..
...
�_--:_-_-_-:_-:__C\.__ -l -_-_-_.,.b�==-x-;+••­
a m is the nat.ural length of the �
cord, x m the extension and T newtons the tension in the cord,
then T = kx where k is the stiffness of the cord.
By data, 3g = k(40-a) ... (1) and Sg = k(4S-a) • , ••• (2)
. 3 0
m
Fro (1) and (2), by division 5 = 4 -a_ and hence a "' 32�2 •
45 a
Thus the natural length of the cord is 32� m.
When the cord is stretched to60 min length, T = k(60-32. \)
where k = 2g/5 N/m from (1). Hence the tension is then
llg newtons
Note Keep units of forae in newtons (N} and of
Zength in metres (m}.
EXAMPLE 2. A particle of mass 2 kg is attached to the end
A of an elastic string of neg I igible mass. The string i�
resting on a smooth horizontal table and is connected to
151
EXAMPLES ON HOOKE'S LAW
a fixed point Bon the table.
IA r
A forae of 1 new.ton is need- �
ed to stretaf1 the string by a B _ •:;__
t:::::
A C
distanae ofliB m. When the string is unstretched, the
particle· is projected with a velocity 6 m/s away from B,
and first comes to rest at a point C. Find the distance
from the equilibrium position to C and the time taken to
reach C.


Method.

ts-
Step 1. From the information in the ..._ i8 r
sentence underlined above, we 1== •""Ne.wfo" A
c an apply Hooke's Law to find the stiffness k of the
string. Here T • kx, i.e. 1 = k • f Thus k = B N/m.

Consider a position in
subsequent motion when the
particle is displaced
distance of x m to the of the equilibrium (or
unstretched) position O. �emember A was originally at OJ
Now the forae T exerted by the string on the particle at
A (to restore A to its equilibrium position O), is, �c­
aording to Hooke's Law, given by T = k:r; where x is the
extensi.on of the string, i.e. T=Bx, since k=8 by above.
Also, the forae F exerted by the particle on the string
in the positive direction (--+), is acaording to the
11athematiaal definition of force, given by F = mx, where
x is the acceleration of the particle in position x,
i.e. F = 2x, since, m = 2 kg by data.
Since F, T act in opposite directions, , ·• F = -T
i.e. 2x = -ax, i.e. x = -4x
Since this is of the form x
= -n 2 x, the particle moves
in simple harmonic mot -on about the equilibrium position
0 as centre. [The period of the oscillation is
2rr//4 = 11 sec, and the nwnber of oscillations per second
is l/11. J
Step 3. Solution of the Equation x =--:¥,
" =
Since x d ( I 2 ), • .1. v2 = - 4x
+ C1 • •••••••• •. • ( M)
dx "aV • • 2 2
By data, when x • 0 (i.e. the string is unstretched),
v .. + 6 m/ s • Hence from (M), 18 = C 1
Thus (M) becomes v 2 = 36-4x 2 = 4 (9-x 2) • • • • • • • • • • • • • (N)
The particle comes to rest when v = O, i.e. when x = •J
The first position of rest is at a point C, 3 m to the
right of O.
152
EXAMPLES ON HOOKE'S LAW
dx
Now from (N), continuing the solution, i 2(9-x2
dt
dt 1
and thus i 2 •••••• . • •. •••••.••.• ••••••• (P)
dx 2 {9-x
Hence, i t � cos-1(}) + C2, (Q)
. •••••••••••••••••••
using the inverse aosine instead of the inverse sine, as has
been noted previ,Jusly in the section on S.H.M.
By data, when t = O, x = O; (initially the particle was pro­
jected from 0) and thus from(Q�
O = - � cos-1 (0) + C2, i.e. C2 = f I•
since cos-10= f.
Thus (Q) becomes ± t = - � cos-1(f) + f .................
(R)
To determine the appropriate sign, we may proceed as follows:
(i) ilhere x = 3, (R) becomes it =-�cos-1 (1) +!, i.e. it={•
However, we have shown the first position of rest to be
where x = 3, and thus t > 0 there.
Hence the appropriate result is +t = -
OR
(ii) On rearranging (R), we obtain� cos-1(}) = { + t,
i.e. cos-1(}) = f + 2t, i.e. x = 3 cos(f + 2t)•••••• (S)

On differentiating, x - 3 sin(f + 2t) x:;- 2,


• 1T -
which leads, w he n t = 0, to x. = - 3 sin( ) x,.. 2.
2
However, by data, when t = o, x = +6
Thus the appropriate result from (S), is x=3cos <f - 2t)
and henae from (R), is t = - � cos-1(}) + {•

The time taken now to reach C from O is given from this


result by letting x = 3, whence t = - � cos-1(1) + f,
i.e. t = 4,
1T
i.e. after 41T sec.
�-
EXAMPLE 3. P�
(i) A uniform elastic spring
AB of natural length a �r:�:T� F
and stiffness k lies on �b� +--x-
a sm ooth horizontal tab! . A mass m is attached to
the end B and the spring is stretched a distance x,
and allowed to osci I late. Show that the equation of
m otion of the mass is of the form x = -n 2 x.
(ii) Initially, it is drawn a distance xo to the right of
0 and is then projected away from Oat a velocity of
� x 0; show that the speed of the particle in
153
SET 31
k
position x is //k
iii and find an expression
in
for x in terms of t.
Method
�Using the approach shown in section D and in example 2
.. k
above, prove that x • - ; x ••••••• (a)
kx2
(ii) From (a), show that Jw 2 ., - + C1, and noting that
2m
ffi 2k X 2
when x • xo, v • ;--; xo, prove C1 • __0_
m
Then prove the speed of the particle is /'!<4x0 2 -x 2 ).
(iii) From (ii), show that t dt '" Im · 14xij2=ic2 •
dx Ik and thus

� cos- 1 (.....!....) + C 2 , whence, (since x=x0 for


that tt • - lk 2xo
t•O), C 2 "' 3 ;i".
Distinguish between the signs, and thence prove that
1
t • - � cos- ( 2: ) + {/1· Rearrange this result to

-;"! t)
0
verify that x = 2x0 c6s(f

EXERCISES SET 31
1. State l!ooke's Law for a perfectly elastic spring.
(i) A uniform elastic string has a length l1 when the
tension is Ti and a length l2 when the ten,;ion is
T2 . Show that T1/T2 = (l1 -a)/Cl2 -a) and hence
that its natural length 'a' is given by
a = (l2T 1 -l1T2)/(T1-T2).
(ii) An elastic spring of modulus 96g N has a length
of 28 m under a tension of 16g N . Find its
natural length.
(iii) An elastic cord is stretched to lengths of 3 and
3!:i m when the tensions on the cord are 4g N
5g N respectively; find the natural length of
the cord.
(iv) ,'_ string of natural length 12m, under a tension Ti
is stretched to a length 13m and under a tension
T 2 is stretched to a length 15111. Find Ti : T2 .

2. A spring of negligible mass and stiffness k rests hori­


zontally on a smooth taule; one end of the spring is
fastened to a point of the table and to the other end a
mass m is attached. Show that if the mass is displaced
x mthen released, it will execute simple harmonic oscil­
l_ ations. What is the time of one complete oscillation?
154
SET 31
If the attached mass is:3/ 4·l<g and the sp.ring requires a
force 192 newtons to stretch it 4 m what is the periodic
time? [Ftrst find k using Hooke's Law.]
3. An elastic string of stiffness , • ", •
4 N/m is fixed to a point A A O B
and held horizontally. A particle of mass 1 kg is
attached to the other end B, and the string is stretched
a distance of 3 JD. The mass is then projected towa1'ds A
with velocity b m/s.
(i) Show that it -4x and hence that v 2 = 4(18-x2 ).
z

(ii) Prove that tt = -�cos-1( �) + C and show that


3
C = TT/8.
(iii) By rearrangement, prove x = 3/f cos({• 2t), and
• TT
thus that x = -3t2
r,:;-
sin( + 2t) x +2. By consider-
4
ing the initial velocity (viz -6 Rt; ), show that
x = 3..'2 cos(f + 2t).
(iv) Find the (a) greatest displacement of the mass
from O (b) the greatest speed attained
(c) the position of the mass after TT/4 sec
(d) the first time for the mass to reach o.
4. A particle of mass 2 kg resting
at a point P of a smooth hori.:.
zontal table is connected by an lA

�gooeoQR::t_..;..J::- -
unstretched sprin2 to a fixed point A on the table.
P
.!:
A
force of 4� newtons is needed to extend the spring by
O. 5 m When the spring is unstretched the particle !§. pro­
jected with velocity 6 m/s in the direction AP, and
first comes to rest at a point Q. Find the time it takes
to reach Q, and the distance PQ .
. [Hint: First prove the stiffness k of the spring is
9 NI m , and·then that x·· s · - 92 x. Continue by showing
9 2 fin 1x
v- = (8-x ) and hence that t "' ( cos- 272).]
3 2 -
?
2
5. A particle of mass 50 kg on a smooth horizontal table
is attached by a light elastic string to a point A on
the table. The particle is initially held with the
string extended by 3 m, the tension being 9600 N
and it is then projected directly away from A with speed
of )2 m/s. Find how far the particle will move before
�irst coming to rest. Find also, correct to- two signif­
icant figures, the time taken to move· throug_h this dis­
tance.
{Ji!!!.!:.: show k 3200, x = -64x, v2 = 64(25-x2 ),
t = ij(cos-1(1) - cos-1(�))}
5 5

155
PROJECTILES
6. A light spring is hanging vertically; a mass of
6 kg is suspended from
(i)

the end and the spring is



�0000� 0qx� 'l..___j elongated lJl.2 m and the
_ system is in equilibrium.
0
Explain why 6g = k . ; and hence find k,
1
taking g, the acceleration due to gravity, as
10 m/s 2•
(ii) This spring is now laid on a smooth horizontal
6.S
table; one end is fastened to a fixed point on the
table and a mass of 24 kg resting on the table
replaces the 6 kg mass, If this mass is dis­
placed by extending the sprin� through�. m. and is
then released, explain why 24x = -72g x.
(iii) (a) Find the time of a complete oscillation
(b) Prove that v 2 = 30( � - x 2 ) and hence find the
1
greatest speed of the mass
(c) Show that�t = cos-1(4x)+C, where C
-1'1fjr O,
and hence express x in terms of t.
(d) What is the displacement of the mass from its
equilibrium position after 11/4v'36sec?
An elastic string, of natural length�
a and modulus >,., is fastened at one jo
n
7.
end to a fixed point O on a smooth--�������;:;;;:=--­
horizontal table. A particle of mass m is attached to
the other end of the string and is projected initially
from a point A on the table at a distance 'a' from 0
with speed Vin the direction OA. Prove that the
particle returns to the point A after a time
11�, and find the greatest extension of the string.
Also express x in terms of t, and find the distance of
the particle from O after 11/2 sec.

SECTION 9. PARABOLIC MOTION UNDER GRAVITY. PROJECTILES.

�: DISCUSSION OF THE PROBLEM


We shall now consider the case of a particle freely pro­
jected wu:ier the action of gravity.in a non-resisting mediwn.
(The mediwn we use in all the following exercises is air, and
air-resistance is negwcted.)
Take axes Ox, Oy as the horizontal and vertical lines
through the point of projection O. Let the particle be of
mass m, and be projected with velocity Vin a direction

156
PROJECTILES
making an angle a with the horizontal. y
[a is called the CJ:!IJJleof_projection, A
and the particle is often referred to
as a projectile_. J V

X
0 C
ASSUMPTIONS ON WHICH THE DISCUSSION 1$ BASED
1. In the present circumstances, we shall asswne that the
motion of t:1e projectile P in any position (x,y), after
time t, can be considered �grately as motion in the
x direction and motion. in _the y direction. [All motion
takes place in the same vertical plane through O;
i.e. the motion is two-dimensional. J
2. In our discussion, we consider motion to be within such
a moderate distance of the earth's surface that the
acceleration due to gravity may be considered to remain
constant. This acceleration g will be taken as
10 m/s 2 usually (and occasionally as 9.8 m/s 2 )
3. Let the forces, acting on the particle
P, in the positive x,y directions be
X,Y respectively. [The diagram illus­
trated shol!IS these forces and is
called a force diagram.)
d x .. d y_ ,
dt7 dt7
2 2
..
Let x = and y = be the acce&-
eration components of t!1e motion in
the x, y directions respectively.
B� the mass�acceleration definition of force, then
mx = X and my= Y, since mx is the theoretical force
acting on the particle P of mass m in the positive
x direction, whilst Xis the actual force acting on
P in this direction similarly for my and Y.
{This separation of forces acting en P in the x, y
directions is called resolving forces; and X, Y are
called the resolved forces in these directions.}
However, by data, there is no force acting on P in the
x Q1,.i.'ecti.on ( air resistance has been neglected),
i.e. X "" 0 and . '. mx = 0. Hence x = o.
[llote: Since the acceleration in the x direction is
zero, . ·. the particle moves with uniform velocity in
this direction. J
The weight of the particle is the only force acting on
it in the y direction. Since the particle is of mass m
and its acceleration due to gravity, towards the centre
of the earth is of magnitude g, •·• the force of attrac­
tion towards the centre of the earth (called its weight)
is equal to mg. Hence Y=-mu, l.e. mfj=-mg rozd thus ii=-g.
157
PROJECTILES
[Note: Since the acceleration in the y direction equals
-g, i.e. a constant, the particle moves with unifo!'171
acceleration in the y direction. J
SUMMARY. If a particle of mass mis pro.Jected under
the action of gravity through the air, where the
air-resistance is neglected, then x = 0 and y = -g.

4. If the initial velocity is of


magnitude V oYld acts in the
direction 6 with the x a.xis,
then we can resolve this
velocity into component
y = EY in
.
dx
velocities x = dt' dt
the x, y directions. 0
The diagram shows a velocity x --+ fi :k
diagram; the intervals on it represent the magnitudes
of the velocities, i.e. *OR= *RS= x,
*OS= V. y,
By trigonometry, cos 6 �/V i.e. x = V cos 6
and sin 6 y /V i.e. y= V sin 6
i.e. initially the horizontal and vertical components of
the velociti are V cos 6, V sin 8 respectively;
i. e. when t = 0, x = V cos 6, y = V sin 6.
[Note that v

which is true.]
2
= � + y 2 2
by Pythagoras' Theorem
(Vcos6) 2 +(Vsin8) 2 =V 2 (cos 2 8+sin 2 6) v2 .
5. Initially, the horizontal velocity of the particle in
-,. direction is V cos 8, oYld since there ar•e no forces
in the x direction, the horizontal velocity throughout
the motion remains equal to V cos e.
In the vertical direction, the initial velocity is VsinB
oY1d the acceleration is -g. Hence the vertical motion is

r
the same as that of a particle projected vertically up­
wards with velocity V sin 8 and moving with acceleration
-g.
The resultant motion of the
particle has been described
as that of a particle pro­
jected with vertical velocity
V sin 8 inside a vertical VS,n& i
tube of small bore whilst the
tube moves in a horizontal
direction with velocity 0
V cos 8,

158
SUMMARY
B. ?9LUTION OF THE EQUATIONS x � �•.. ¥.���
J.
given that when t = 0. x = 0 •. ;t...= Q _/ill..d._
X = V cos e, y = V sin e

tt
HORIZONTAL MOTIOM VERT I CAL MOT ION
dzx
Acee Zeration X = 0 = y = -g
cit7 =
(i.e. particle trav- (i . e. part ic I e trave Is
els with uniform with uniformly accel-
motion in the X erated mot ion in the
direction) y direct ion)

Velocity By integration, By integration,


dx �= y
= X = C1, 1·1here C 1 -gt + C2,
cit dt
is a constant, where C 2 is constant.
�y data, v,hen t = 0, By data, 1�hen t = 0,
X = Vcose y = Vsine
C1 = Vcosa, Vsina = C 2 ,
and thus � = Vcosa and thus
dt �- -gt+ V sin a
dt

Position By integration, By integration,


X = Vcosa t + C3 y = -�:,g t 2 +Vsin6.t+C4,
and since 1�hen t =O, and since when t = 0,
X = 0,
Y. = 0
C3 = 0 C4 = 0
Hence X = Vt co·s a Hence y = Vts ina-!,gt 2

The above results give the acceleration component, velo­


city component and displacement component in the x, y
directions after time t.

SUWMARY. For a particle projected with velocity V in a


direction making an angle a with the horizontal, then for
motion in the
2x dx
x direction, edifT = o, cit= V cos a, x = Vt cos a

d 2y __
y direction,� -g, dt
dv = V sin a - gt, y = Vtslna-!gt 2

These results are often written in the notation


x o, x = v cos e, x = vt cos e ................ <L>
and y = -g, y = V sin 0 - gt, y = Vt sin e - !,gt 2 .. • (M)

159
EQUATION OF THE PATH OF A PROJECTILE
THE E�UATJON OF THE PATH (ALSO CALLED THE TRAJECTORY)
OF TH PROJECTILE.
The p(lX'(1JTlet:ric equations giving
the position (x,y) of the projectile y
after time t, are
x = Vt cos 6 (1)
y = Vt sin 6 - \ g t2 •.•••• , • • • • (2)
The cartesian equation of the
path may be determined by elimina­
ting the parameter t from equations -+"-'-----�--�-�
(1) and (2). 0 X
X
Now from (1), t and on substituting in (2),
Vcosa
=
x x "1-
s c2 e
y = V(--) sin6 - l--,g(---) 2 i.e. y = x tan6- �
Vcose Vcose ' 2V -
This result is also written as y = x tan 6 - g x 2 (1+tan 2 6)/2v2
This is the required cartesian equation of ' the
trajectory of the particle .
The above equation is of the form y = Ax - Bx 2 = x(A-Bx)
where A z tan e, B = g sec 2 e/2v 2 .
'i"aus the path of the projectile is that of a parabola,
(with axis of symmetry paraUel tc the y axis), passing
through the origin O, and concave down ( ('. ) .
Ulote The parametric equations
x = Vt cos 6, y = Vt sin 6 - \ gt 2
give m�re info�ation about the �article· than does the
cartes1,ctn equation y = x tan 6 - gx sec2 e/2v 2 • The parametric
equations tell u>here the particle goes and when it is at a
certain place, whereas the. cartesian equation tells only the
curve along which the particle travels.}

EXA,,:PLES. The fol lowing examples should be worked through


by you; they are intended to i I lustrate the type of
problems encountered in this section. The above results
and summary should not be learnt by heart; they should be
derived from the calculus.

l:.Xf.i�PLE I. A. t.>a I I is thrO'im with y


A
an initial velocity of 25 m/s
at an angle 8 (= tan-1 3/4) to
the horizontal through 0. If air
resistance is neglected, deter­
mine
( i) the greatest height attained

160
EXAMPLES ON PROJECTILES
(ii) the time of flight and the range on the horizontal plane
(iii) the velocity and the direction of motion
(a) after\ sec
(b) when the height of the particle is 10 m
(iv) the cartesian equation of the trajectory of the ball.
{Take g = 10 m/s 2 }

Method.
--
d 2x =x =0 .. (1) and d 2y = ..y
Now by data, cft7 -g -10 •. (2)
df2" =

The initial conditions are when t = 0,


x = O, y = 0 and
dx =x• =25 cos e = 25 x 4 = 20 m/s
dt 5
%f = y =25 sin e = 25 x l = 15 m/s
By integration of (1)
dx =CI and x =CI t + C *- = isG&.1 e
cit 2
However from the initial con­ Since e = tan- 1 (fl
ditions, tan e = 43

i.e. ¥t
C 1 = 20 and C 2 = D
=20 and x = 2Dt (L) i.e. cos e = t
By integration of ( 2), sin e = l
%f = -lOt + C 3 and
y =-5t 2 + C 3 t + C4
Using the initial conditions, C 3 15 and C4 = 0
i.e. %f
=-lOt + 15 and y =-5t 2 + 15t .........•.••..••..•..... (M)
A
(j_) When the ball reaahes its greatest height,
%f =0 (since there is no motion in the � �
vertical direction then). :c
Thus -lDt + 15 =0, i.e. t = 3/2. :B ')'
When t = f, 0
y = -5(f) 2 + 15(f) =11-}, from (M)
Hence; the greatest height of the ball is lli" m and this
occurs after 3/2 sec.
(jj_) At the instant when the ball reaahes the point B (the point on
the same horizontal level as 0), y = 0 and thus
-5t 2 + 15t = 0, i.e. 5t(t - 3) = 0, i.e. t =D or 3.
Sin�e t =D corresponds to the initial position 0,
. ·. the time for the ball to reach B is 3 sec.
_Now when t = 3, x = 20 x 3 =60, from (L)
Hence the range of the ball on the horizontal plane is 60 m,
and the time of flight is 3 sec.

161
EXAMPLES ON PROJECTILES
(iii) (�) After> � aea,¥t = 20 and aityVelo Diag:r>am
� -10(�) + 15 = 10, from (M).
=

If v m/s is the velocity of the �


ball then (the direction of v 10
being tangential to the path of
the ball at that instant); o<
then we obtain v, using ,_,,,�--.��'X'
.... ----����-111
ao
Pythagoras' Theorem in the velocity diagram illustrated.
Now v 2 = (M)2 + (l) 2 = 202 + 102

��7��
i.e. V = /202 + 102 = /102(22 + 12) = 10/5"
Further, tan a = � = = ¥a = f
and thus a? 26 ° 34', from tables or calculator.
Hence, after � sec, the ball is travelling at 1015" m/s in
a direction inclined at 260 34' to the horizontal.

(b) When the height is 10 m


- -5t2 + 15t = 10, from (�)
and solving t = 1 or 2.
For> t=l, ( x = 20 and dx/dt=20
\y= lO and dy/dt=5
For> t=2, f x=40 and dx/dt=20
ly= lO and dy/dt= -5
Thus, in both aaaes, V1 = v2 =/x2 + y2 = /zo 2 + 5 2
= 51ifTTT2" = 5 m
Let S, y be the anglea !JJhiah the rl[J
dir>eation of the motion make '\)', = ,
!JJith the horizontal (see figu:r>e).� 1f5
Now tan S = y/x = 5/20 =·1/4
and tan y = y/'x = -5/20 = -1/4 .;oc:20
Hence at, both R, S (when the height is 10 m} the ball fs
travelling at 5/Ii m/s and the direction of motion is the
same, namely 14° 2' approximately.

(i V) To find the aCU'tesian equation of the t:r>ajeato:r>y of the


ball, !JJe eliminate t f:r>om the pammetr>ia equations
X = 20t, y = -5t 2 + 15t.

This gives y = -5(�) 2 + 15(�)

= �60 - x), as the equation of the path

162
EXAMPLES ON PROJECTILES

Example 2. A bullet is fired A


with a velocity of 300 m/s.
Find the
(a) greatest range on the
horizontal plane
(�) two directions possible
to give a range of
4500 m.
{Take g = 10 m/s 2 } O B
Method.
--iry data, W ft,f
d 2 x = 0, d2 = -g = -10 .......................... (Q)
Ife is the initial angle of projection, then
when t = 0, x = y = 0, � = 300 cose, �= 300 sfne
By integration of (Q), � = 300 cose, x = 300t cose ..... (1)
and�= -lOt + 300 sine
y = -5t2 + 300t sine .....•....•.. (2 )
(�) The bullet reaches the horizontal plane again when y = O,
i.e. when -5t2 + 300t sin0 = 0, i.e. t = 0 or 60 sin0.
Hence, the range R of the bullet is given by
R:300(60 sine)cose, from (1); i.e. R = 300 x 30 sin 2e, ..(3)
noting sin 2e = 2 sine cose.
Since jsin 20j � 1, .the ma.:r:imwn value of the range
is 300 x 30 = 9000 m.
(This occurs when sin·2e = 1, i.e. 20 = ff/2, i.e. 0 = ff/4)
{Note,
� i-fe = ff/6 say, then the range of the bullet would
have been 300 x 30 sin j = 300 x 30 x � � m.
The greatest range exceeds the range whene = ff/6 by
(300 X 30 - 300 X 15/3) = �500(2 - /3) � 12 06 m)
(Q.) When the range is 4500 m
. ·. 300 x 30 sin e
2 = 4500
from (3) range
i•e. sin 20 = l,
Hence 2e = 30° or 150°
i.e.e = 15° or 75°
(i.e. � = ff/12 or 5ff/12 )
Thus the bullet may be fired o
either at an initial angle of ....;;;.�...;;.�i........;��--'---.....1-.
150 or 75° (with a velocity
of 300 m/s) to attain a range of 4500 m.

163
EXAMPLES ON PROJECTILES

Example 3. A cricket ball is thrown


so that its path is given by "Ii
x = 36t, y =
5t(3 - t) where x, y
are measured in metres and tin
seconds. Calculate the
(i) initial velocity and the
initial angle of projection
(ii) ratio of the greatest height
to the range of the ball.

Method. Since x 36t, dtX = 36 and dF X


= 0

and since y 15t - 5t 2 , dv dt d 2y = -10


= 15 - lOt, dfT
Ci.l Initially (i.e. when t • 0), x = 36 and Y = 15.
Hence, tfie initial horizontal and vertical components of the
velocity are 36 m/s, 15 m/s respectively.
The initial velocity V m/s of the ball
is given by
V = 1362 + 152 /32(122 + 52) = ff.132" -,s
= 3.13 39
Further, tan e = QY.
dx = x = � t
36 = J.
°
1<: and e � 22 37'�- e
x�.3
Thus, the ball is thrown initially with a velocity of 39 m/s at
an angle of 22° 37' to the horizontal.
(jj) For the greatest height, y O and thus t = 3/2. Hence y
= 45/4. =
For the range of the ball, y = O and thus t = 3. Hence x = 108.

That is, the greatest height is 45/4 m and the range is 108m.
The required ratio is 45/4:108 = 45:432 = 5:48
Example 4. A stone is projected
from the top of a cliff 25 m
high; the horizontal and
vertical components of the
velocity are initially 20'3 t,,/s
::--,..,._�
2013 m/s, 20 m/s respect- ��
ively. Determine the
parametric equations of the � G� lr+�
path of the stone after t sec.�
{Take g = 10 m/s 2 } ;;:,....:....;=;;....----=S"''-°' -=e. .:.;e;..;l ____.�t,.i.
::;.., v

Method. Take the axes through 0, the top of the cliff


--
d2x d 2y = -g = -10 .................. (R)
By data, W = O and dfT

The initial conditions are x = y = 0, �� = 2013, %f = 20

164
EXAMPLES ON PROJECTILES
Thus on integration, we obtain
�� = 20/3, x = 20/Jt, * = -lOt + 20, y = -5t 2 + 20t
Hence, the parametric equations of the path of the stone after
t sec are x = 20/Jt and y = -5t 2 + 20t.
{SHOW THAT THE STONE
(j_) hits the sea when y = -25 i.e. when -5t 2 + 20t = -25
giving t = 5 and· x = 20i'3 x 5 = 100/3. Thus, after 5 sec, the
stone hits the sea, its horizontal distance being 100/3 m from
the cliff.
At that instant, �� = 2013 and* = - 30. If fl is the angle at
which the stone hits the sea, then
tan fl = 2�S =-7:
3= and fl � 180° - 40° 54' = 1 39 ° 6' . The
velocity then is /(2013)2 + (- 30)2 = 1012T m/s.
(ij_) reaches the point M, in the diagram, when y = O, i.e. when t = 4
and thus x = 80/3. Thus, after 4 sec, the stone is at the same
level as it was initTa!Ty projected and distant 80/3 m from 0.
� that then, x = 20/3, y = -20 and the velocity then is
1(2013) 2 + (-20)2 = 40 m/s.
-20 = - 1 and a = 180o - 30o = 150o
Also tan a = 2073 73
(jjj) attains its greatest height above sea level when y = O, g1v1ng
t = 2 and y = 20. Thus after 2 sec, the stone reaches its
greatest height (25 + 20) = 45 m above sea level.
Example 5 A stone is projected
horizontally, from the top of a
cliff 25 m high, the initial
velocity being 40 m/s. Deter­
mine the parametric equations
of the path of the stone after
t sec.
{Take g = 10 m/s 2 }

Method. Take the axes throuQh 0, the top of the cliff


--
d2x d2v
By data, W = O and (!fl" "' -g = -10 .......................... (S)
The initial conditions are x = y = 0, �� = 40, * = 0 (Why?)
Thus, on integration of (S),
--dxdt = 40, x = 40t; dt dv = -10t, y = -5t2

Hence the parametric equations of the path of the stone after


t sec are x = 40t, y = -5t 2 ,

165
EXAMPLES ON PROJECTILES
{SHOW THAT THE STONE hits the sea when y = -25 giving t = 15 and
x= 4015". Thus, after 15 sec, the stone hits the sea, its horizontal
distance being 4015" m from the cliff,
At that instant, x = 40, y = -1015 and the velocity then is
/402 + (-10/5)2= 10/IT m/s.
Also tan y = -l�� =�and hence y � 180° - 29° 12' = 150° 48'}

Example 6 A projectile leaves a point Oon a horizontal plane, and


after 1 sec just clears a vertical wall whose height above the plane
is 15/2 m and which is 15/4 mdistant from O. The wall is perpendicular
to the plane of motion. Taking g = 10 m/s2 , find
(i) where the particle strikes the horizontal plane
(ii) the velocity and direction of motion at 0
(iii) the direction of motion at the point where the wall is cleared.

Method. Let the initial velocity


ci1'""'tne projectile be V m/s and
the initial angle of projection
bee.
Assuming air resistance is neg­
lected, then, with the usual
notation,
d2y = -g= -10 ..(M)
d 2x= 0 ..(L), iJt2"
dfZ"
where the initial conditions are
that when t = 0, x = y = 0, B
dx -_ V cose, � -_ V sin . a. 0
dt dt
Thus on integration,M= V cos a ....(N), x Vt cose (P)
�= V sine - lOt .....(R) y= Vt sine - 5t2 (S)
1
(j) By data, the projectile is at the point(.!!, �) on its path
after 3/2 sec.

1t
Hence, from (P) and (S), we have
= v} cose, i.e. V cose = ! .... ...... .. .. .. .. ...
(1)
and�= v} sine - 5(!)� i.e. V sine = � .............. (2)
The projeatiZe strikes the horizontal plane through Oat B
when y = O, i.e. when Vt sine - 5t 2 = 0 and this gives
t = 0 or V sine
5
Hence, the time of flight= 25,2=%sec, using V sine = 25/2.

�·t
Since V cose = 5/2 from (1) and t= 5/2, thus the horizontal
distance OB= =�. That is, the projectile strikes the
horizontal plane .(--;r
25 - 15 l = 2'11
--;r
5 beyond the wall.

166
SET 3J
(ii) From (1) and ( 2)1 on s�ua.ri1* and adding
V (cos 2 e+sin 2 e)=(%)2 +(-{-) 2=(f) 2 {1 2+5 2 }
and thus V = �
FUPther, on division of (1) and (2)
Vsine = 25/2 = 5 i e tan e = 5
Vcose 5/2 • ·
Hence, the initial velocity is 512'6"/2 m/s and the initial angle
of projection is tan·l(5) � a
7 o 41'
(iii) At Q. V cos e = 5/2 from (1), V sin e = 25/2 from (2), t = 3/2
i
and hence M = from (N), and i i
= - from (R).
The velocity v at Q is given by v = � = ¥
m/s and the
direction of motion a is given by tan a = y/x = -1, i.e. a = 135°
EXERCISES SET 3J
1. A particle Pis projected fromi
0 at an angle a to the hori­
zon, with a velocity of u m/s.
With the usual notation,
explain why x = 0 and y = -g. 0 4=====-=�=ji::::::=====+�
By integration, show that x = u cos a, y = u sin a - gt;
x = ut cos a, y = ut sin a -\gt 2 . Hence find the
(i) greatest height (H) attained
(ii) time (T) of flight
(iii) range (R) on the horizontal plane through 0
(iv) velocity and direction of motion (a) after t sec *(b) at
a given height h metres
(v) Also write down the greatest range (R max) attained for any
given initial velocity u. What is the corresponding value
of a?
USE THESE RESULTS TO ANSWER THE FOLLOWING:­
(A) Determine H, T, R for the case where
(a) u == 20 m/s, a == 30 0 (Take g = 10 m/s )
2
� (� u 25 m/s, a sin·! (3/5), (Take g = 10 m/s 2 }
(�) A ball is thrown at a velocity of 15 m/s; find the greatest
distance to which it could be projected and the height to which
it would rise., (Take g = 10 m/s 2 )
(f.) A stone is thrown with a speed of 30 m/s to hit a target on a
horizontal plane at a distance of 45 m. Find the two possible
angles of projection. If it is projected at an angle of 16 ° ,
how far beyond the target would it strike the ground? (Take
g = 10 m/s 2 )
(_Q_) If the initial velocity of Pis 1960 cm/s and inclined at 30 ° to
the horizontal; taking g = 980 cm/s 2 find the velocity and
direction of motion of P
(!!,.) after\ sec *(£.) when the height is 367\ cm

167
SET 3J
(E) If a boy can just throw a stone 80 metres, find
(i) the initial velocity and direction;
(Ti) how long it is in the air;
(Tri) to what hei�ht it rises?
(Take g = 9.8 m/s J
(F) A particle is projected at an angle of elevation tan- 1 (4/3)
and its range on the horizontal plane is 3840 m, find the
velocity of projection, and the velocity at the highest
point of its path. (Take g = 10 m/a Z J
(G) Find the angle of projection when
(i) the range on the horizontal plane is 4 times the
- greatest height attained;
(ii) the range is one half the maximum horizontal range.
{Do not substitute for g}
(H) Two particles are projected from the same point O in
directions of 60 0 , 300 to the horizontal; what is the ratio
of their velocities of projection if they
(a) attain the same height;
(b) have the same horizontal range.
{Do not substitute for g}
(I) A shot has a maximum range of 2000 metres. What would be
the
(a) range with an elevation of 30 0 ;
(b) elevations with which an object at a horizontal distance
- of 1500 metres could be hit.
{Do not substitute for g}
(J) The highest point of the trajectory of a projectile is
1000 m and the horizontal range is 16000 m. Find the
initial velocity and the angle of projection.
{Take g = 10 m/s 2 }
(K) A qun can fire shells with s_peed u at elevations between
300 and 60 ° . Find the area commanded by the gun on a
horizontal plane, assuming that the gun can be rotated about
a vertical axis so as to point in any desired direction.
{Leave anSIJer in terms of g}

2. A stone is projected from the top 0


of a cliff of height h. The x, y
axes are taken at O. If the initial o
velocity is u at an angle a to the
DOrizon�al through 0, explain why �
x = 0, y = -g, given· that when t = 0, '-�
x = y = 0 and x = u cos a, y = u sin� C.
By integration, show that after
time-t, M= u cos a, x = ut cos a, ¥t= u sin a - gt,
� = ut sin a - \gt2 .
Hence answer the following, (taking g = 10 m/s2 )
�A projectile is fired from the top of a cliff of
height 65 m; the horizontal and vertical

168
SET 3J
components of its initial velocity being 80, 60 m/s
respectively. Find the
(i) time of flight;
(Ti) distance from the foot of the cliff of the point
� where the projectile strikes the water.
(B) A tower, 70 m high, stands on horizontal ground. From its
top, a stone is thrown, with a speed of 65 m/s at an angle
of a = tan-1(5/12) to the horizontal. Find the
- distance from the foot of the tower at which the
(i)
stone strikes the ground;
(jj) velocity (in magnitude and direction) at that instant.

3. A stone is projected horizontaLLy


with an initial velocity u, from
the top of a tower of height h.
-� X

Explain why x = 0·, y = -g �iven �


�hen t = 0, x=0, y = 0, x = u .� 0�
y=O. ,t,i

By integration, show that after


time t, �� =u, x = ut, � = -gt, u z \ - gt2 ,
Hence answer the following (taki"ng g=10 mi's 2):
A particle is thrown horizontally from a tower of height
(A) 31\ m with an initial velocity of 25 m/s. Find where it
will strike the level ground through the foot of the tower,
and its velocity then.
(B) 245 m, and reaches the ground at a horizontal distance of
350 m from the foot of the tower. Determine the initial
velocity, and the velocity (direction and magnitude) on
striking the ground.

The following exercises should be worked from first


principles, using the methods of the examples above,

4. A body is projected from a point 0. After t seconds, its


horizontal and vertical distances from O are given by x = 30t,
y = 40t - 5t 2 . Find, (units are in metres)
(i)
- the horizontal and vertical components of the initial
velocity, and its initial speed;
(ii) the speed at which it is moving after\ sec;
(Tri) when it is travelling horizontally, and the greatest
height attained;
(iv) the direction in which it is moving when it returns to
the horizontal level through O;
(y) the cartesian equation of the trajectory of the body.
5. A bullet is projected, with a velocity of 200 m/s, at an
angle of 30 0 to the horizontal. Taking g � 10 m/s 2

169
SET 3J
find (i) the greatest height attained
(Ti) the range on a horizontal plane and the time of
flight
(i!:!_) the velocity and direction of motion of the bullet
when it is at a height of 180 metres.
6. A cricket ball is thrown with a velocity of 3012" m/s; find
the greatest range on the horizontal plane, and the two
directions in which the ball may be thrown so as to give a
range of 90 m. (Take g = 10 m/s 2 J
*7. A particle is projected with a speed of 20 m/s and passes
through a point P whose horizontal distance from the point

a!
of projection is 30 m and whose vertical height above the
point of projection is m. Take g = ·20 m/s2 . {Hint: Prove
3 = 2t aos e, ? = 16t sine - 4t 2, then eLiminate t.}
Find the angle of elevation e and the time taken for the
particle to reach P.
8. {_f_) From the top of a cliff 40 m high, a stone is thrown
horizontally with a speed of 21 m/s. Show that it hits
the sea at a horizontal distance of 60 m from the point
of projection. (Take g = 9.8 m/s2 )
(1..1_) At time t, the component velocities of a particle,
parallel to the coordinate axes are given by the equations
J• �
�� = 30 cos J-
= 30 sin lOt. If the particle is at
the origin when t = 0, find the values of x and y, at
time t = 2.
9. At time t, the component accelerations of a point, parallel to
the coordinate axes, are given by the equations
� = 0, � = -10. Find the coordinates of the point at
time t = l'l given that, when t = 0, x = 0, y = 0,
dx = 10 cos 11, Ql. = 10 sin . 11 .
dt 4 at 4
10. A stone is projected from the origin O with initial horizontal
and vertical velocity components v 1 , v2 respectively. Write
down the equations of motion of the stone, taking the gravity
as g.
Prove that the time T for the stone to reach the horizontal
ground through O is independent of v 1 •

170
CHAPTER 4

FURTHER TRIGONOMETRY AND CALCULUS

!· SUM'4ARY OF (a.± B) RESULTS


These results have been proven and used in Chapter 12,
"Higher School Certificate Course in Mathematics Form 5,
3 unit course" by Coroneos. In the present section, it
is intended to revise this work, and extend tl�se results to
formulae involving 26, 36.

sin(a+B)=sinacosS+cosasinB sin(a-B)=sinacos3-cosasin8
.••••.•• (I) •••••••• (4)

cos(a+B)=cosacosB-sinasinB cos(a-B)=cosacosB+sinasinB
•••••••• (2) •••••••• (5)

tana+tanB tana-tanB
tan(a+8)= 1-tanatanB ••• <3> tan(a-B)= l+tanatanB .•..(6)

EXAMPLES
(a) sin(3x+4y) sin 3x cos 4y + cos 3x sin 4y
(b) cos(36+26) cos 56 = cos 36 cos 26 - sin 36 sin 26
tan(45 ° + 300) tan 45 ° + tan 30 °
(c) tan 75°
1 - tan 45 ° tan 30 °
1 + 1//3
1 _ 1 , l//3
_ Ii+ 1
- 7)-=---i _ n
- 2+ 3, on simplifying

(d) sin(40 ° -10 ° ) sin 30 ° = \= sin40 ° cosl0 ° - cos40 ° sinl0 °


(e) cosfx+h - x}
(f) can 6 = tan(56-46) = �� �:u-
cos h = cos(x+h)cos x + sin(x+h) sin x
tan 46
56 tan 46

B. EX TENS I O'J TO 2 6 FORMULAE


In results (1), (2), {J) aoove, let a=B= 6
From (1),
sin(6+6) = sin6cos6+cos6sin6 i.e. sin26 = 2 sin6cos6 .•••.(7)
171
29, 39 RESULTS
Thus sin 68 2 sin 3 8 cos 38; sin 18° 2 sin 9 cos
°
9•

sin 7ljl 2 sin� cos�; sin a = 2 sin cos £


Cl
2 2

From (2), cos(9+9) cos9cos9-sin9sin9


i.e. cos 29 cos 2 9-sin 2 9 ••••••••·•• -·�·-······· (8)

Also cos 29 (l-sin 2 9)-sin 2 9, using cos 2 9+sin 2 9=1


1 - 2 sin 2 9 . . .. • . . •• .• . •••••••. • • . (9)

Further cos 29 1 - 2(1-cos 2 9), from (9)


2 cos 2 9 - 1 •••·••••••••••••••••• (10)

Hence, from results (8), ( 9), (10),


cos 29 = cos 2 9-sin 2 9 1 - 2 sin 2 9= 2 cos 2 9 - 1
Thus cos lOa= cos 2 S�-sin2Sa l-2sinZ5a = 2 cos 2 Sa - 1
cos45 ° cos 2 22�0 -sin 2 22\ 0= l-2sin 2 22\ 0 = 2 cos 2 22\ 0 - 1
g 98
cos 91! cos 2 8 - sin 2 l-2sin 291l 2 cos 2� - 1
2 2 2 2
cos 0 cos 2 .!!_ - sin 2 .!!_ l-2sin� 2 cos 2 f - 1
2 2 2 2
Note From ( 9), 2sin 29 = l-cos29, i.e. sin 2 9=\(l-cos29)...( 11)
and from (10), 2cos 2 9 = l+cos29, i. e. cos29=\( l+cos29�..( 12)
Thus sin 2 30 ° \(l-cos60 ° ); sin 2 SA l-;(1-coslOA)
sin 2 £ \(1-cosa); sin 2 2x \(1-cos 4x)
2
cos 2 7ljJ \(l+cosl4ljl); cos 2 25° \(l+cos50 ° )
9
cos 2 !,;(l+cos9) cos 2 3x '2(1+cos6x)
2
tan9+tan9 2tan9
tan(a+9) = 1.· e tan29 . .• (13)
l-tan9tan9 • · =::.;.:.:-z
1-tan _
From ( 3),
9
2tan39 o = 2tanl5
°
Thus tan69
l-tan239; tan3o l-tan215°
2 tan a/2 2 tan 78 /2
tana ; tan78
1 - tan2 a/2 1 - tan2 78/2

£· EXTENSIO'J TO 39 FORMULAE
In results (1), (2), (3) let a= 29, 8 = 9
Frc,n (1), sin(29+9) sin29cos9+cos29sin9
i.e. sin39 (2sin9cos9)cos9+(1-2sin 2 9)sin9,
using results (7), (9)
2sin9cos 2 9+sin9-2sin 3 9, on expanding
2sin9(1-sin 29)+sin9-2sin 3 9,
using cos 2 9+sin 2 9 = 1
i.e. sin39 3sin9-4sin 3 9, on collecting terms."(14)
Thus sin30° 3sinl0 ° -4sin 310° ; sin68� 3sin28 - 4sin 3 2 8
sin a 3sin £ - 4sin 3 £. sin29= 3 sin·lQ. - 4 sin 3 �
3 3' 3 3

172
THE It' RESULTS
From (2), cos( 29+9) cos29cos9-sin29sin9
i.e. cos39 ( 2cos 2 9-l)cos9-(2sin8cos9)sin9,
usin� results (10) an d (7)
2cos 9-cos9- 2sin 9cos9
3

2 cos 9-cos9-2(1-cos 2 e)cos9,


1

using sin2 9=1-cos2 e


i.e. cos39 4cos 39-3cos9, on collecting tenns..(15)
Thus cos 45 ° 4cos 315° -3cosl5 ° ; cosl88=4cos 36B-3cos6B
cos e 4cos 3 i - 3cos i. cos7B =4cos 3 1§_ - 3cos
3 3' 3 3
tan2e+tane
From (3), tan(29+9)
1-tan29tan9
2tan9
1-tanze + tane
i.e. tan39 -=-c. .cc.'-'- '- - -----, using result (13)
2 tan9
1 - . tane
1-tan2e
2 tan9+tan9(1-tan e)
2
on simplifying
1(1-tanZe)- 2 tan9.tan9'
3tan0-tan e 3
i.e. tan39 ��"---,
1-3tan 2.-=-�9 on collecting terms .. (16)

3tan4 ° -tan34 ° 3tan 2y-tan 32y


Thus tanl 2° ; tan6 Y
1-3tan24° 1-3tan22y
2
3 3tan 8 - tan� 8
2
_ n_ �a�/�3_-_ta
�3_ta /
n a� �3,,
tan a
1 - 3 tan a/3 2
_ __ tan 28 = 3 3
1 - 3 tan 2 l.@_ ·
3
D. THE tan 9/2 RESULTS: (THE 't' RESULTS)
Each of the trigonometric (or circular) functions can be
expressed in a simple form, free from surds, in tenns of the
tangent of half the angle. [Thus, for example ,
cot 9 = /1-sin2e/sin9 is the expression for cote in terms of
sine, and this result involves surds. By using tan 9/2 , we
can express cote, sine, cose etc. in terms of tan 0/ 2, with­
out surds,]
Method
e e
(!)Now sin0 2 sin
.
2 cos
2,
using an application of
result (7)
2 sin 0/2 cos 9/2
noting cos2 i + sin 2 2
a = 1
cos29/2 + sinZe/2 ; 2

2 sin eJ1.
cos e/2 dividing nUlllerator and
sin2e/2 denominator by cos 2 6/2.
1 +
cos29/2
2 tan 0 2

1 + tan 9/2
2t
i.e. sine • 2, where t = tan 9/2 ................. (17)
l+t

173
SUMMARY
2 tan 22� 0
2t
sin 4
5 = i+tan222�o; sin 2a = l+t�' where t � tan a.
°
7'hus
(ii) FUY'ther, cos 9 cos 2 B/2 sin 2 B/2, using an appli­
cation of (8)
cos 2 6/2 - sin 2 6/2
noting
cos 26/2 + sin2BJ2'
cos·2e/ 2+sin 2 6/2=1
1 - tan 2 B/2
dividing numerator and
1 + tan2B/2'
denominator by cos 2 e/2
1-t where t
2
i.e. cos 8 =- tan 6/2 ..•..••.•. (18)
l+t2 •
1-� 0
1-t
cos 30 ° cos 213 = l 2 • wi,ere t = tan 13
l+tan215o;
Thus
+t
�an 9/2 Zt
(iii) Also tans= = using an application
l-tan2e;2 l-t2 •
of (13)
l+t 2 l+t 2
In addition, cosec� 1/sine
Zt
secB = 1/cose = i="t2"•
1-t 2
and cote 1/tane = 2t

SlJ\1MARY
sin(a+B1=sinacos13+cosasini3 sin2 8=!;( l-cos261....... ( 11)

I ....(II
cos(a+i31=cosacosS-sinasinl3
cos 2 B=\< I +cos26I ....... I 12)
2 tans

tan(a+l31 =
tana+tanB
.... (21
... (3)
tan26 =
l-tan2e
...... ( 13)
I
1-tanatanB 1 sin36=3s i n8-4sin 3 e.....( 141
c�s36=4cos 3 8-3cos8.....(15)
sin(a-Bl=sinacos8-cosasini3 3tan6-tan 3 6
tan38= ..... ( 161
....(41 I - 3tan 2e
cos(a-8)=cosacosG+sinasinl3
....(51 If t = tanB/2, then
tana-tanl3 2t
tan(a-,31 = ... (61 sine = ...........(17)
l+tanatanl3 i+-i7
l- t2
cos8 = ...........(181
sin26=2sin8cos6..........(71
cos28=cos 2 8-sin2 e ........ (81 2t
=1-2sin 2 e........... 191
tanB =
1-17 ..........(I 3al
'-'2cos 2 B-I .........(101

E. NU\1ER I CAL EXERCISES

EXAMPLE I. Express sin42 ° , cos42 ° , tan42 °


in terms of the ra_tios of (ii 21 ° /iii 14 °

f.1ethod.
(a) sin 42 ° 2 sin 21 ° cos 21° , using result (7)
2 tan 21 °
using result (17)
l+tan'"!il°'
3 sin 14 - 4 sin 314 ° , using result (14)
°

174
EXAMPLES 28 ETC.
°
(b) cos 42 cos 2 21 ° - sin 221 °
1 - 2 sin 221 °
= 2 cos 221 ° - 1, using re-
2 sults (8), (9), (10)
l-tan 21o .
, using result (18)
l+tan221o
4 cos 3 !4 ° - 3 cos 14 °, using result (1 5 )
°
2 tan 21 .
(c) tan 42 ° using result (13) or (13a)
l-tanz21o,
°
3 tan 14 - tan !4 °3
.
, using result (16)
1 _ 3 tan214o

-3
fXAMPLE 2. If cos8 =�and O < 8 < n, determine,
without tables, the value of (i) sine (ii) sin28
(iii) cos28 (iv) cos38 (v) sinS/2 (vi) tanS/2

Method. Now 8 lies in the second


quadrant.

l
By Pythagoras' Theorem, *LP = 4 units
(i) sin8 = from the diagram .,t

(ii) sin28 2sin8cos8, from (7),


4 -3 24
2
·s·s
2
= - zs -�
(iii) cos28 2cos 8-l, from (10), {This result for cos28 is
preferred since cos8 is given}
2C ) 2 -1 = ..!J! - 1 = ::1.
3
5 25 25
(iv) cos38 4 cos 3 8 - 3 cos 8, from (15 )
3 -108+ .225 117
4c.:1) 3 _ 3C ) = -108 + 2. =
5 5 1 25 5 125 125
(v) sin8/2 is determined from cos8, by using
sin 2 8/2 = Y,(l-cos8), an application of (11)
2 1
Thus sin 8/2 = [1 - ( ) J
-3 8
Y,( ) =
4

I I
5 5 5
sin 8/2 = "fs',
since O < 8 < n, , ', 0 < <
and sin 8/2 > O.
(vi) tan 8/2 may be determined by the approaches outlined
below:-
Approaah 1. Now cos2 8/2 l,(l+cos8), an application of (12)

!,.{!+(�)} =
1
t
and cos 8/2 =
7s·
since O < .!
2
< .!!.
2
1 1
Hence, knowing sin 8/2 = 7s and cos 8/2 = Ts•
= sin8/2 ,. 1
tan 612
cos8/2 1
l-t2
Approaah 2. From (18), cos e = l+t Z• where t • tan 9/2

175
EXAMPLES 20, 30 ETC.
-3 l-t 2
� = l+tz• i.e. -3-3t = s-st
2 2

i.e. 2t 2 = 8 i.�. t 2 = 4 i.e. t = ±2


0 11
Thus tan 0/2 2 (since O < < , tan 0/2 > O)
2 2
*ilpproach 3. From (17), sin 0 = !�2, where t = tan 0/2
i.e.�=
1 2 !� 1
, i.e. 2t 2 -5t+2 = 0 on simplifying.
Hence (2t-l)(t-2) = 0 i.e. t = J, or 2
(In this case, the appropriate value of tan 0/2 may not be
decided from sign alone, since both values oft are positive.
However, here 0 is obtuse, and from sin0 =; ·8, = it can be

1�
° ° °
seen that 0 � (180 - 50 ) = 130 noting that
l· 3
sin 60 ° = � = ·87. Thus 0/2 � 65 ° and tan 0/2>1,
°
since tan 45 = 1. �ence tan 0/2 = 2, notl,}.

EXAMPLE 3. 2
If tana = and cosa < 0, tanB =�_7 and sinB > 0,
5
calculate, without tables, the value of

+� +�
(i) a+B (ii) tan28 (iii) tan3a (iv) tan; (v) sin2a

Method.
tana+tanB
�Now tan(a+B) from (3)
1-tanatanB'
2/5 + (-7/3) _ 6-35 _ on simpli-
- -l,
1 - (2/5)(-7/3) 15+14 - fving
Since tana>O and cosa<O, . ·• 180 ° <a<270 °
and since tan8<0 and sinB>O, -x " " -, , ,
90 ° <B<l80 ° . . �·
Thus 270 ° <a+B<450 °, on addition. y
However, since tan(a+B)<O, • '. 270 ° <a+B<360 ° � -�
i.e. (a+B) = 360 °-45 °, noting tan45 ° = 1 -• �
� ·
Hence a+B- = 315 ° or lJ!.. radians .s
4 .,..
-�
_ 2 tan B 2 -7 3
(ii) Now tan2B - frooi (13), = < ! ) 2
l-tan2B 1-(-7/3)
14
- -42 21
3
= = = on simplification.
49 9_49 20
1 -
9
3tana-tan.3a 1.{Y5)-(2/5�3
(iii) Further tan3a from (16),
1 - 3 tan2a 1 - 3(2/5)
6 8
s - ill 150 - 8 =
142
1 - .ll 125 - 60 65
25

176
EXAMPLES t RESULTS ETC.
2 tan a/2 . 2 2t
(iv) Frcm (13a), tana = , 1.e. =
l-tan2a;2 5 l-t2
Hence, 2-2t2 = lOt i.e. t2 +5t-l = 0
5 -5±129
By the formula, t = - ±125+4 = �
2 � 2�
However, since 180 ° <a<270 ° , 90 ° < .S! < 135 °
2
a
and thus tan < 0
2
a -5-129 -5+129
i.e. tan = (the other value �- �- > O)
2 --2- 2

(v) To determine sin2a, it is simpler to use an applica­


tion of result (17).
2 tan a 2t
In this case, sin2a = = where t = tan a.
l+tan2a l+tz
. _ 2 (2/5) _ ....!!D__ _ 20
Thus s1n2a - l+(2/5)2 - !+4125 - 29

EXAMPLE 4. (The purpose of this example is to em­


phasise the many methods available to us to perform
numerical evaluations; in this case to calculate,
as an irrational quantity, the value of tan 15 ° .)
(i) Prove that tan 6/2 �. (6 is acute),
sine l-cos6
l+cos6 = �
and hence from each part, evaluate tan 15 ° .
U i) Show other methods of determining this value.

(i) Method.
� sin26/2 applying results (11), (12)
I�
= /� tan 612 '
cos2e/2 above, and simplifying
sin6 2 sin 6/2 cos 6/2 applying results
= tan 6/2,
l+cos6 2 cos2 6/2 (7), (12)
l-cos6 �-2�s_i_n_
2 /2
e� � �� applying results
tan 612•
2 sin 6/2 cos 6/2 =
=
sinO (11), (7)
l-cos30° 1 /J/2 n-7J /<2-11)
2
Now tanl5 ° /l+cos30 ° = / -
l+l'J/2 = /2+73' = (2+/3") (2-/j)
2 - 13, since tan 15 ° > 0
sin 30 ° l. 1 = 2 - /j
01' tanl5 ° =
l+cos30 ° 1 + l'J/2 2 + 1J
°
l-cos30 1 - l'J/_2
or tanl5 ° 2 - /j a s before
sin 30 ° \
(ii) Alternative methods include:
(a) Use of result (6) for tan(a-B),
°
where a = 45 ° , B = 30 ° (or a = 60 ° , B = 45 )
° °
•i.e tan tan45 - tan30
. 15o = Complete the working.
1 + tan45o tan3oo·
177
SET 4A
l-t 2
(b) Use of result (18), for cos e= 2'
l+t
where cos e cos 30 °
/j/2, and t = tan 15 ° .
Show that t 2 (2+/j) 2-/j, i.e. t 2 = (2-/j) 2 •..••••• (P)
whence t = tan 15 ° 2-/j, since tan 15 ° > O.
{The other root of equation (P), namely t = -(2-,13) is the
value of tan 165 ° , since /j/2 cos(360 ° -30 ° ) = cos 330 °, as
well as cos 30 ° .}
2t
(c) Use of result (13a), for tane
l-t2 '
where tane= tan 30 ° 1//3, and t = tan 15 °
Here show that t 2 +213t-l = O ........................ (Q)
-213 ! 4
whence t = = -,13 ± 2,
2

Since tan 15 > O, • • tan 15 ° = -/j + 2
°
2 - 13.
{The other root of equation (Q), namely t = -13 - 2 is the
value of tan 105 ° , since 1/,13 = tan(l80 ° +30 ° ) = tan 210 ° , as
well as tan 30 ° .}
2t
(d) Use of the result (17), for sine= 2'
l+t
where sine sin 30 ° =� and t = tan 15 °
Show that t 2 -4t+l O .•••.•••.•.• (R) i.e. t = 2±,13,
whence tan 15 ° = 2 - 13 (since O < tan 15 ° < 1)
{The other root of equation (R), namely t = 2 + 13 is the
value of tan 75 ° , since�= sin(l80 ° -30 ° ) = sin 150 ° , as well
as sin 30 ° .}

(e) Use of the result (16), for tan 36,


where e= 15•, and .·. tan 36 = 1
3t-t 3
Here show that 1 = _ 2, where t tan e
1 3t
i.e. prove t L Jt -3t+l = O .••• (S) i.e. (t+l) (t 2 -4t+l)=O
2

whence t = -1 or 2 ± 13.
The only possible value of t to represent tan 15 ° = 2-13.
{The other roots of equation (S), namely t = -1 and t = 2+/3
represent tan 135 ° and tan 75 ° respectively;
since 1 = tan 45 ° (i.e. tan 3.15 ° ) or tan 225 ° (i.e. tan3. 75°)
or tan 405 ° (i.e. tan 3.135 ° ) etc.}

EXERCISES SET 4A
1. Simplify the following
(a) sin7x cos 6x - cos7x sin6x (b) 2sin36 cos3e
(c) cos 2 a/2 - sin 2 a/2 (d) cos5A cos3A + sin5A sin3A
(e) 2cos 2 58 - 1 (f) 3sin40 - 4sin 3 4e
(g) 4cos 3 a/3 - 3cos a/3 (h) �(l - cos6A)
tan4C - tan3D
(i) 1 - 2sin 7B
2 (j)
1 + tan4C tan3D
178
SET 4A
2tan3X 3tanA/3 - tan 3 A/3
(k) (l)
1-tan23X 1 - 3tan2A/3
2 tan 86 2 tan 86
(m ) (n)
1 - tan286 1 + tan286
1 - tan 2 (l/2
(o) (p) cos 2 (A+B)-sin 2 (A+B)
1 + tan28/2
(q) sin 8/2 cos 8/2 (r) 1 + cos6
2. Evaluate the following, (without tables)
(a) 2sin22\ 0 cos22Y, 0 (b) sin58 ° cos2 ° + cos58 ° sin2 °
. l ....2!. tan73 ° + tan62 °
( c) 1 - 2 sin (d)
12 1 - tan73 ° tan62 °
2
1 - tan 15 °
(f) 3tan50 - tan ° 50
° 3 °
(e)
1 + tan 2 15 ° 1 - 3tan' 50
2 tan 1li l2 ° 2tan67�0 2sin15 ° cos15 °
(g) � (h) l+tan267\ 0 (i)
1 - tan 112
2 0
cos215 ° -sin2 15 °
11 11 11
(j) 4cos 20 - 3cos20
3 ° °
(k) cos cos 15 - sin Io5in 11
10 15
1-2sin 2 80 °
(£.) 2 cos 105 - 1 (m)
2 °
2cos210 ° -1
2 511/12 - sin 2 511/12
cos 1 + tan 2 11/8
(n) (o)
sin 311/8 cos 311/3 1 - tan2 11/8
(p) 3sinll0 ° -4sin 3110 ° (q) sin 2 67\ 0 cos 2 67\ 0
(r) sec 22� cosec 22!2
0 °

3. (a) If x = sin 2 75 ° , find without tables, the value of


X - x2
cos15 ° +sinl5 ° cos15 ° -sin15 ° without
(b) Evaluate
cosl5 -sin15
° °
cos15 ° +sin15 ° tables.
. sinl2 ° cos12 °
(c) Sim;-ilify, without tables, -
sin 4o cos 4o
(d) If sin 36 = a sin 6 + b sin 36 for all 6, find the
values of the constants a, b.
4 12 2
4. If A,n,c are acute and sin A=
5, cos B =
13
, tan C =
3,
determine (without tables) the value
(i) cosA (ii) sinB (iii) sin(A-B) (iv) cos(A+B)
(v) tan(A+B) (vi) sin2A (vii). cos2B (viii) tan2C
(ix) sin3A (x) tan3C (xi) cos B/2 (xii) sin A/2
(xiii) tan C/2
5. If x is acute and cos x = l find (without tables) the
3'
value of
(i) cos2x (ii) cos3x (iii) cos4x (iv) cos
(vi) tan x/2 (a) using the results (iv), (v)
(v) sinf I
(b) independently of these results.
5
6. If tan26= ,
12
(a) Calculate the possible value of tan6
(i) If O < 29 < 'i· (ii) If 11 < 26 < 3;
(b) Determine the value of tan46 without tables,
179
ELIMINATIONS
7. (i) Prove that l+cos2e = cote, and hence find the
sin2e
value of cot67�0 'in surd form.
(ii) Show that cos2A = l-tan�A , and hence
l+tan A prove that
tan22�o = lz-1.
Explain the $ignificance of the other root of the
equation.
(iii) Prove that cotS = /l+cos2S
l-cos2S' and deduce the value
of cotl5 ° in surd form. Verify this result by
finding tanl5 ° using the (A-B) result, and hence
the value of cotl5 .
°

8. In I:, ABC, b:c = 4:3 and (f) *B = 2.*C (ii) *B = 3.*C


By the Sine Rule, show
(i) that cosC = 2/3 and hence find *C from the tables
(ii) that sine= 15
z7j

�. ELIMINATIONS - DETERMINATION OF CARTESIAN EQUATIONS

EXAMPLE I. The point P (x,y) moves so that its co­


ordinates are given by x = 5sine+3, y = 7cos2e-4.
Find the cartesi an e.q uation of the I ocus of P.

Method.
He make use here of the identity cos2e = l-2sin 2 e ........ (X)
1
Since y 7cos29-4, cos29 = (y+4)
7
and since x 5sin9+3, sine
= f<x-3)
Henae substituting in (X), we obtain t<y+4)=1-2.z}(x-3) 2 •• (Y)
which is the cartesian equation of the locus of P.
{Note: this result (Y) may be

rewritten, on simplification,
in many ways; for example, as
25(y+4) = 175-14(x-3) 2 or as
2 25
(x-3) = -4. (y-3) etc.
56
In this case, the locus of P
is a parabola with vertex
(3,3) and focal length 25/56.} ·l

EXAMPLE 2. Eliminate e in each of the tollol'ling


cases.
( i l f x = tane-3 (ii) Jx = asine(iii) x = l-4cosl (iv)fx = 6sin9
\. y=tan2e ly=bsin39 {y=2+5tan tv= 5sin2e
2
Method.
(i) Here we use the identity tan2 e =
tan
-tan2
i
� ........•.. (A)
Since x = tan9-3, . ·. tane = x+3, and noting tan29 = y,
180
.,n 48
2(x+3) _ -2(x+3)
hence from (A), y = - on simplifying.
l-(x+3)2 (x+Z)(x+4)

(ii) Here the identity sin30 3sin0-4sin 3 e .• ,.,, ..,,, .• (B)


is used.
By data, sine = � and sin30 = Y
a b
Thus from (B) Y = 3(�) - 4(�) 3
' b a a
3x
i.e. Y = - � Yhich becomes a 3y = 3a 2 bx-40x 3 .
b a a
This result may be written as a 3y = bx(3a 2 -4x 2 )
l-t e
(iii) Th e i"dentity
. cos 8 = � ...•..,, (C) where t tan
t 2
is used here.
Since x = 1-4cos8, cos8 = \(1-x),
and since y = 2+5tan7, , ', tant y 2)
f< -
=

1 - ...l:.(y-2)2
25
Thus from (C), 1 (1-x) which simplifies to
4 i + zs<y-2> 2
1 '
1-x 25-(y-2) 2 1-x 21+4y-y2
give i.e. =
-4- 25+(y-2)2 ' -4- 29-4y+y2
=

(iv) Here there is no direct method of eliminating 8. Two


possible approaches are indicated below.
1. Noting y = 5sin28, i.e. y = l0sin8cose,
·
Approach
on squaring, y 2 =100 sin 2 e cos 2 e
i.e. y 2= 100sin 2 8(1-sin 2 e) .....•..••..... ,, (D)
Hence since sin8 2S. , from (D), y 2 = 100(�) [1 - (�)2 ]
6 6 6
= 2

x 2 36-x 2
i.e. y 2 = 100. (�) i.e. 324y2 = 2Sx 2 (36-x2)
36
Approach 2. �loting y = 5sin28 = lOsine cose ..•......... (E),
and from x 6sin8 , . '. sine = x/6 ... , .. , ......, ..(F)
Thus from (E), y 10.( )cos8
6 3 f .......... , .. ,
X
=
i.e. 3y = 5x cos8 i.e. cose = (G)
From (F) and (G), using the identity sin 2 e + cos 2 e = 1,
x2
(�) 2 + (lY.)2 = 1 . .e + -2:f:_ = 1
6 Sx J. , 36 25x '
which becomes 25x 4+324y 2 = 900x 2 , i.e. 324y2 25x 2 (36-x 2 )

EXERCISES SET 48
uestions 2 (xi ii - xviii) and 3 to be left for revisiQn,
1, The point P (x,y) moves so that its coordinates satisfy
the equations
(i) x=sin8 (ii) x=(sine+cos8)2 (iii) x= tan 8/2
y= cos28+1 y= sin28 y= sin8
Find the cartesian equation of the locus of P in each
case, and sketch this locus.
2. Eliminate e in each of the following cases. {Do not
over-simplify answers.}
181
PROOFS INVOLVING 2e, 3e ETC.
(i) x=cos t (ii) x=cos20 (iii) x =tane
y= �os2t y=4sin0 y=tan20
(iv) x=sin0 (v) x= cos30 (vi) x=tane
y= sin30 y= 2cos0 y= 5tan30
(vii) x= sin0 (viii) x= 2cos0 (ix) x=acos20
y= sin20 y= sin20 y=bcos 2 e
(x) x=asin20 (xi) x= acos20 (xii) x=asine
y= bsin2e y= bsin 2 e y= btan e/2
t (xiii) x= acos0 t (xiv) x =acos e t (xv) x=asin30
y= b tan 0/2 y= b tan 2 0/2 y=bsin0
t (xvi) x=acos e t (xvii) x=2sec0 t(xviii) x=2cot0
y= bcos30 y= cos20 y=tan20
t3 . Find the equation relating x, y and not involving 0 in
each of the following cases.
(i) x= cos20+3 (ii) x= cos20+3 (iii) x=tan20
y= l-cose y= l-sine y=2tan0+1
(iv) x= 2cos30+1 (v) x= l-5cos0 (vi) x=3sin0+1
y= rcose y=3 tan 0/2 y=4sin20
4. (i) If x=acos 2 0+bsin 2 e, y = (a-b)sin0 cosa, express
x, y in terms of 20, and hence eliminate e.
(ii) If x = cos0+sin0, y = cos20, show that
y = x(cos0-sin0) and thence that cos6-sin0 = Y
X
Hence prove 2cos0 = x + Y and 2sin0 =
Y. thus X -
X x'
eliminate 0 and find the relation between x,y.

�· PROOFS INVOLVING 20, 30, 't' RESULTS


{Note Study these examples carefully; they are designed
to illustrate various methods of attaak that are possible in
this work.}

EXAMPLE I. Prove the following


(.1 )cot0/2+tan0/2 sece tan5A+tan3A
tan5A-tan3A = 4cos2A cos4A
('i 'i )
cot0/2-tan0/2
(iii) (cos2a+cos2Bl 2 +(sin2a+sin2Bl 2 = 4cos 2(a-Bl
cosA+s!nA-1
(i vl
cosA-s 1 nA+I = tan A/Z

(vl tan(n/4 + x/2) = sec x + tan x =


/: +- sin x
Sin X
r-.os6/2 sfn6/2
Method. cote/2+tan0/2 +
sin0/2 cos0/2 cos L o/2+sin20/2
(i)LHS cos20/2-sirf0/2
cot0/2-tan0/2 cos0/2 _ sin0/2
sin0/2 cose/2 on simpli­
fication
- -1- noting fcos0= cos 2 0/2-sin 2 0/2
- cosS'
l l= cos 2 0/2+sin 2 0/2
= seca
182
EXAMPLES INVOLVING PROOFS
sinSA sin3A
tan5A+tan3A cos SA + cos3A
(ii) LHS
tan5A-tan3A sinSA sin3A
cosSA - cos3A
sin5Acos3A + sin3Acos5A sin(SA+3A)
sin5Acos3A - sin3Acos5A sin(SA-3A)'
noting results for sin(A±B)
2sin4A cos4A
= sin BA = using result
sin2A sin2A
sin 26 = 2 sin e cos e
2(2sin2A�os2A)cos4A a�ain using \ = 4cosZA
= ( cos4A
s1n2A 's1n26 resultj

(iii) LHS = (cos2a+cos2S)2 +(sin2a+sin2S)2 = (cos 22a +


2cos2a cos2S+cos 2 2S)+(sin2 2a+2sin2a sin2S+sin2 2S),
on expanding
(cos 2 2a+sin2 2a)+(cos2 2S+sin 2 2S)
+2(cos2a cos2S+sin2a sin2S), on regrouping
1 + 1 + 2 cos(2a-2S) Check these results!
2(1 + cos2(a-S)) on factorising
2{2cos 2 (a-S)}, using result l+cos26 = 2cos 2 6;
4 cos 2 (a-S)

cosA+sinA-1 (1-2sin�A/2)+(2sinA/2 cosA/2)-1


(iv) LHS cosA-sinA+l (2cos 1 A/2-1)-(2sinA/2 cosA/2)+1'
noting results for sinA, cosA in terms of A/2
[Check why the substitution 1-2sin 2 A/2 for cosA is used in
the numerator, -but 2cos 2 A/2-1 is used for cosA in the denom-
inator
! J 2sinA/2 cosA/2 - 2sin 2 A/2
Thus L HS on simplifying
2cos 2A/2 - 2sinA/2 cosA/2'
2sinA/2 (cosA/2 - sinA/2)
on factorising
2cosA/2 (cosA/2 - sinA/2)'
tanA/2, on cancelling and simplifying
2t
[Repeat_ the question, using the substitutions sinA = Z
(l+t )
(1-t2) • where t = tanA/2,)
and cosA = Cl+t2)

tan /4 + tanx/2 using tan(A+B)


(v) UIS tan(ll/4 + x/Z) = 1 -11tan11/4 tanx/2' result.
� + tanx>;, since tan11/4 = 1
- tanx
l+sinx (cos x/2 + sin x/2)2 Check this!
Also secx + tanx = � =
cosZx/Z _ sinZx/Z
cos x/2 + sin x/2 on cancelling
cos x/2 - sin x/2
1 + tan x/2
on dividing by cos x/2
1 - tan x/2'
c1+sinx)2 (l+sinx)
and I !1+sinx
1-sinx
= ;
(1-sinx)(l+sinx) cosx
secx + tanx.

183
SET 4C
Alternatively, using the t = tan x/2 results, show that
= l+t 2 2t _ (l+t>2 1 + tan x/2
secx + tanx +
� � - (1-t)(l+t) 1 - tan x/2

and
/1 + sin x _
-
/1 + 2t/(l+t2) _ /Jl+t)� • 1 + tan x/2
-
1 - sin x 1 - 2t/(l+t2) (1-t) 1 - tan x/2

EXAMPLE 2.
Show that sin4 9 = !< l-2cos29+cos 229) and hence find
constants a,b,c such that sin4e = a+bcos29+ccos49
for al I values of e.

Method.
Now sin2 9 \(l-cos28), and thus sin 48 ={\(l-cos28)} 2
That is, sin 4 9 \(1-2cos20+cos 2 20)
\{l-2cos20+\(l+cos48)}, noting result
cos 2 A=\(l+cos2A)
3
\{ - 2cos26 + �os46 }
2
= 3 - \ cos28 +
1
cos48, and this is of the
8 8
form a+bcos28+ccos48, where a = 3 , b = -\, =
1
8 C 8

EXERCISES SET 4C
Prove the following results
l-tan 2e sin8x
1, (i) l+tan2e = cos28 (ii)
l+cos8x
tan4x
' cos2A
(]. ii') = cos A-sin
. A
cosA+sinA
2. (i) cote sin2e = l+cos26 (ii) cotA+tanA = 2cosec2A
(iii) cotA-tanA = 2cot2A
cosA sinA = 2cos(A+B)
3. (i) sinB
-
cosB sin2B
(ii) 1 -- l�-­
tan29 cosece
cos6-sin6 + cose+sine
sin2A _ cos2A l.'i' 2sin0/2 - sine 8
4. (i)
sinA = secA ( ) 2sin2
cosA 1 - cos 8/2 =
sin 3 x+cos 3 x 2cosA
s. (i)
sinx + cosx
= l-\sin2x (ii)
cosecA-2sinA
= tan2A

. 1-cose
6. If e is acute, ; = tan 012 = 1-cose = sine
l+cos9 sine l+cose

7. (i) cosec 2 A _ cos39


- sec ZA (ii) = 2cos28-l
cot 2 A-l cose
sin)e
(iii) = 2cos2e+l
sine

8. (i) cosec2B-cot2B=tanB (ii) cosecB+cotB = cot B/2


9. (i) sin2A-cos2AtanA=tanA (ii) tan48-tan26=tan26sec4
184
TRIGONOMETRIC EQUATIONS

-�--������-
sin 36 cos36 = 2sin59 sin59 _ co s59 =
10. (i) + (ii) 4cosZ.6
sin29 cos29 sin4 6 sin9 cos6
11. (i) If tan(TI/4+ 6)-tan(TI/4-9) = k tan 26 , find the val-
ue of k.
(ii) Show that cos 4 9= �(1+2cos29 + cos2 ze), and hence
find constants a,b, c such that
cos 4 e = a+b cos26+ccos46 for all value s of e.
(iii) If s= sinx, express sin 22 x in terms of s .
s�n)x cos)x
12. (i) Prove that - is independent of
SU\X COSX
What i s its value?
(ii) Show that log sin 3A= log sinA +log(/3+ 2sinA)
+log( /3- 2sinA\
(iii) If 2cos6= x + 1/x, show that 2cos36 = x 3 + :-!·
X
13. (i) If 6 is acute, and cos6 = 1-x, where x is so small
that x 2 is negligible compared with unity, prove
that cos26 ala l-4x, and that cos36 i 1-bx, where b
is a constant. Find the value of b.
(ii) Prove that sina sin(TI/ 3-a) sin(1T/ 3+a)= k sin 3a,
where k is a con stant, and find the value of k.
14, If t = tan6/2, express the following in terms of t
(i) l+sin 6 (ii) l-cos6 (iii) 5 cos6 + 12sin6 + 13
1- sin6
(iv) cot6 .cot6/2 (v) /
l+sin6
_l+_
c_o_
s_
6 -=2:.=c::::o..::cs..:.
6..:.
+.::.
3 l- sin6
(vi) / (vii) (viii)
I
1-cose l-4sin6 co se

(i11) sec
2
'. 2sin 6 - 3cos6+1) (x) l(l+sinBX3sin6 +4c ose+.5)
15. Use the t results to
(i) prove that if tan6/2 b/a, then aco s6 +bsin6 = a.
1-t
(ii) show that if secA-tanA = x, then x = l+t, where
t = tanA/2. Thu s pr ove t = 1-x •
l+x
l-t2
16. If t = tanA/2, pr ove that c osA = . Also if tan B=-�t,
l+tz
5cosA+3
and cosC= where A,B , C are acute angle s, prove
)cosA+5
that *c = 2 .*B.=========�===========
H. TRIGONOMETRIC EQUATIONS

SLMMARY Most of these results have been treated


in Chapters 3, 12 of
Course in Mathematics -
';Higher School Certificate
Form 5, 3 unit Course "by
Coroneos.
I. PROPERTIES OF THE TRIGONOMETRIC FUNC TIONS.
FUNC TION RANGE GRAPH

�.
sin x -l�sinxil Note
Domain: .r .e. I s .r nx I<
1- I sin n1T = 0
-1
�x

0 )'(
Al I real X (n +ve in- _,
teger) '-1-Y
185
SUMMARY; GRAPHS
FUNCT I <l.J RANGE GRAPH
COS X --1 Scosx� I
i. e. I co sx I s I �I
Domain:
Ai"Treai X -Y
Note cos(2n+l)n/2 = 0
tan x -oo<tanx<oo
Domain:
Ai"Treal x,
x;tl(2n+ I )n/2 -x
I
I -Y
Note tan nn 0

fI
cosec x co sec x � I

,\JI
:n•�
Domain: or cosecx!:-1
Ai"Treal x,
i.e.
x ;ti nn lcosecxl�I -X
1f
0
\
y

sec x sec x � I or
s ec x s -I
Domain: i • e. I sec x I � I
AJlreal X,
xi ( 2n+ I )n/2

1/'\ I
-l( -3.,. _.,.

1/ \1
cot x -oo<cot x< 00, I
Domain:
Ai"Treal x,
x ,Inn -x )(

Note
.£• USE OF TRIGONOMETRIC IDENTITIES
- (b) sece 1
Ca) cosec e = -�sin e cose
<c> cote = - 1 - = c�se Cdl tan e = in e
s
tan e sin e cose
(el cos2 e+ s in 2e = I Cf) sec2e = i+tan2e
(g) cosec 2e = l+cot2e (hl sin(a*B>=sinacosB*cosasinB
t tanB
(il cos(a*B)=co sacosB,sina sinB (J') tan(a*Bl = tana
1,tanatanB
186
SUt,ttARY; TRIG. IDENTITIES
(k) sin26 = 2sin6cos6 (l) cos26=cos 2 6-sin 2 6=1-2sin 2e
=2cos 2 6-I
(ml sin 2 e=\(l-cos26); cos 2 6=.\ii(l+ c os26)

I
2tan6 (o) sin3e 3sin6-4sin 3e
(n) tan26 l-tan2e
3e
(p) cos36 4c os 6-3cos6 (q) tan36 3tane-tan
3
l-3tan2e

I
If t = tan 6/2, then sine = 2t cos6 = l-t
2
(r)
TttT· l+-i7


2· SPECIAL ANGLES; Ratios of 0 ° ,30 ° 145 ° 160 ° 190 ° , 180 ° ,

I
270 °,360 °

.J\
m
° ° °
(a) The ratios of 30 ,45 ,60
can be expressed in surd

I
form from the figures
indicated.
(bl The ratios of 0 ° ,90 ° ,180 ° ,270 ° ,360 ° are best deter­
mined by reference to sketches of the trigonometric

I
functions.
Cc) The use of radians instead of degrees in giving solu­
tions is very common, and 1,,i 11 be constantly employed
here. Note the result n c = 180 ° can be used to
transform degrees into radians and vice versa, or if
required, reference may be made to radian-degree con­
version table.
4. FURTHER RELATIONS BETWEEN THE TRIGONOMETRIC RATIOS
The diagram indi cates
(a) the ASTC rule, giving the
sign of the trigonometric
ratios in each quadrant. -x#"-���---,1,:-�--1�-.�
(These may be verified
from the sketches of the
trigonometric functions.)
Of' •8
Cb) the fa c t that the trigon- ° ·�
ometric ratios of a, (180 -e>, (180 ° +6), (360 ° -6),
(-6) do not lead to a change in ratio.
Thus sin( 180 ° -6) sine; -sin(180 ° +6) = sinC360 ° -6)
= sin(-6) = -sine
° °
c os(-6) = case; cos( 180 -e)=cos( 180 +0>
°
c os(360 -6)
=-case
tan C180 ° +6l tans; tan( 180 ° -6) tan(360 ° -6)
tan(-6) = -tans
etc.
Cc) Trigonometric ratios such as (90 ° 16), (270 ° 16)
do change ratio.
Thus sinC90 ° -6l=cos6 tan(90 ° -el=cota sec(90 ° -el=cosec0
°
°
cos C90 -6l=sine cot C90 -6l=tans cosec< 90 ° -6l=sec0
TRIG. EQUATIONS
Case 1. REVISION TYPES: [These do not include a±B, 29, 39, t
Results. ]­
These types have already beer:·treated in Chapters 3, 12 of
"Higher S chool Certificate Course in Mathemati cs - Form S,
3 unit Cours..e'' by Coroneos .
EXERCISES SET 4D
1, For rev1s1on purposes, the methods of approach are
out I ined in the following represen tative selection
of types. Each exercise should be copied and then
worked through by you. To save time, take your
workinq onlu as far as you feel is really necessary,
The angles 9,a,S et c. are required in (i) degrees and
minutes in the domain 0 ° to 360 °, both in clusive.
(ii) radians in the domain Oto 211, ooth in clusive; (leave
answers in terms of 11 where possiule; otherwise, use radian
tables).
�. No Change of Ratios Necessary.
(a) I
sin9 = f3/2 :-ioti cing that the basi c angle is 60 ° or
11
- C, show that
3 lJ·c 2 C
(i) 9=60 ° or (180-60) 0 = 120 ° (ii) 9 = - (11 ·- 2!.) C = ...2!.
3 or 3 3
(b) I
cosa = -1//2 l
The basi c angle is 45 or ° 11 C
4
. Show ti1at
(i) a = (180-45) 0 =135 ° or (18o+45) 0 =225 °
11 c 311c 11 c Snc
(ii) a=(n - ) = or (11 + ) =
4 4 4 4

( c) I 3tan2x = I I, i.e. tanx = ±. 1/ Ii. The basi c angle is 30 °


TI C
or 6. �how that
+1
(i) for tanx = X = 30
°
or 210 °
73•
and for tanx -1 X = 150 ° or 330 °
= 73•
TI C 711 C
(ii) for tanx = +1 X = - 6 or 6
73·
Sn c l111 C
and for tanx = -1, X = 6 or -- 6
73
(d) sec2B=I i.e.
From the sketch,
I, cos 2 B=l, i.e.
"\":' t� �
(i) if cosll=+l, 8=0 ° or 360 ° ,
and if cosB=-1., 6=180 ° l(

(ii) if cosB=+l, ll=Oc or 211c ,


and if cosB=-1, B=n c

(e) I S={e.:tan9=1//3};T= {9:sin9 = -la} State Sn T a.nd S UT


we 7nc • 711 c 1111c
Now Ss{J0�,210 °}or 6 ,6 and T- {210 ° ,330 }or {6 ,�}
711 C 11 C 711 C lllf!
SnT = {210 °} or and SUT={30 ° ,210 ° ,330 °}or{ ,- -,- -}
6 6 6 6
188
TRIG. EQUATIONS
(f) I
i.e. (3sinx+l)(sinx-2) • O
3sin x-5sinx-2=0
2

i.e. sinx = -1/3 or 2


Now since -1 :': sin x :': 1 0 • · , s·in x j 2, and the only
valid solution is sin x = -1/3 i -·3333,(The basi c angle
is approximately 19 ° 28' or •3398C,)
Thus (i) x= (l80 ° +19 ° 28') = 199 ° 28' or (360 ° -19 ° 28') = 340 ° 32'
(ii) x= (3•1416+·3398)C = 3·4814 c
or (6•2832-•3398) c = 5•9434� [from the radian tables]

(g) i i i.e. se cll( v'3se cll-2)=O


13'sec2a-2secll=O
Now sin c e I se c ll I �l, secll 1 0 and Y I
the only valid solution is I
se c ll = 2//3 i.e. c osll = i'3/2
Thus (1) ll 30 ° or 330 °
-,
0

(ii) ll =
c
f or !
l nc
,�1
,--,=------=,--:======:;;==;---t-Y 1' \1
(h) [ v2sinetane-/6sine-tane+/3=o
Groupina in pairs, Y2sin6(tar.·6-v'3)-l(tane-/3) = O
i.e. (12sin6-1)(tan6-0) = 0 i.e. sin8=1/Y2 or tan8=13.
n C 3n C nc 4n C
Thus (i) 8 =45 o ,135 o or 60 o ,240 o (ii) 8 = , or ,
4 4 3 3

1'1i.E!!......§. Use of Identities Required to Change Ratios


(i) i 3sin8=5cos8 i,
i.e. 3sin99 = 5, on dividing by cose. (1'0)
cos
i.e. tan8 = 5/3 i 1·6667
Thus (i) 8 = 59 ° 2', 239 ° 2' (ii) e = 1·0303c , 4,1729 c

I I,
(j) . 3sec8 = 4cose . . i.e. _1___ = 4 cos6 i.e. 4c os 2 6=3 and
cos9
cose = t./i/2 n.c lln c
Thus (i) e = 30 ° ,150 ° ,210 ° ,330 ° (ii) e = :I!. 22!. 7
c c
6' 6' 6' 6

(k) J -Q:tan8-4+T3'cote=O
· i, i.e . .fitan8-4 + tane = o .fi

and • '. v'3tan 6-4tane+,'3,;, 0 i.e. (v'3tan8-1)(tan8-i'3)


2 0
n c n c 7n c 4n c
Thus (1) 8 = 30 ° ,60 ° ,210 ° ,240 ° (ii) 8 = , 3, ,
6 6 3

(l) i 3cosec2x+2cot2x = 8 I i.e. 3(1+ c otl x)+2 c ot2 x = 8,


and . •• cot2x=l i.e. tanx = ±1, Henc e find x,

(m) [}s:I:m<+2sin2x+ll = 13cos2x i


i.e. 7sinx+2sin2x+ll = 13(1-sin 2 x)
Hen ce show that 15sin 2.x+7sinx-2 = C
i.e. (5sinx-l)(3sinx+2) = O, and ti.us find x,

(n) i 10sin2a+cosa= 7 i i.e. 10(1-cos 2 a)+ c osa= 7, whenc e


(5 cosa-3)(2cosa+l) = O, and henc e find a.

189
TRIG. EQUJ\.TIONS
{o) I 4sec2w=3taniv:+5 !
i.e. 4(l+tan2 �)=3tanlj,+5, whence
(4tanlj,+l)(tan�-1) • O, and thus find �.

{p) I 2tan2S-5sec/3+4=0 I
i.e. 2(sec2 8-1)-5secf3+4aO, whence
(2sec(3-l)(secf3-2) = O, and thus find (3,

(q) I sec4e-6tan2e+2=0 !
Thus i.e. sec 4 e-6se c2 e+a = o
sec 4 e-6(se c2 e-1)+2�0
i.e. (sec2 e-4)(se c 2 e-2)=0 i.e. cose=±� or 11/./2.
Alternatively� noting sec e= (l+tan2 e) 2 , the equation becomes
4

(1+2tan2 e+tan 4 e)-6tan2 e+2=0 i.e. tan 4 e-4tan�e+3=0,


Find 6 by ea c h method.
��������
(r) I cos6=cot6 I cose
i.e. cose
sine i.e. cosesin6
= = cose.

[Note, we cannot divide by cos6, otherwise we lJse solution


c ose = 0,) i.e. cos6{sin6-l) = 0 and hen ce find 6,

(s) I cos6=tan6 I sine


i,e, cos2 6 = sin6
i.e. cose =
cos9
2
Hence (1-sin 6) = sine i.e. sin2 6+sine-1 = 0,
{Since there are no rational factors of sin2 e+sin6-1, we use
the quadrati c formula to solve the equation sin2 6+sine-1=0,}
.. .
nere sine = -lt/I+'4 -11/s
= �2
2 �
Using /s oio 2•236, find the values of e.

'!iJ.E£.._£. Homogeneous Equations in sinx, aosx


{t) I sin2x-3sinxcosx+2cos2x= O I
Divide both sides by cos2 x(cos x 1 0),
and ,', tan�x-3tanx+2 = 0, Eence find x,

(u) J 10sin2y+IOsinycosy-cos2y =2 I
[First repla ce 2 by 2(cos2y+sin2 y)J
.·, l0sin 2 y+10siny cosy-c os2 y = 2cos2 y+2sin2 y,
i.e. 8sin 2 y+l0sinycosy-3cos2 y = 0, and dividing by c os2 y,
.·. 8tan 2 y+1Utany-3 = O. Hence find y,

2. Solve the following equations for the variable in the


domain 0 ° to 3f0 ° , or o c to 211.::, (both in c lusive in ea c h
case). Give answers alternately in degrees and radians,
beginning with (a) in degrees, (b) in radians, etc.
(Work eaah question only as far as you feel is neaessary]
(a) 2 cosa= l3 (b) sec2 S=2 ( c) (tanx+l)(2tanx-l)=O
{d) (2sinx+l)(cotx-13) 0 = (e) siniµ(cos�-1)= 0
(f) cosecf3(cosecf3+1)=0 (g) 3tan2 e-4tan0+1=0
(h) 6 cos x+cosx-1=
2 0 {i) sin2 e-sine-6=0
(j) 4cos 4 8-7cos2 8+3=0 (k) 8sin 4 a-2sin2a-l=O
(l) sine = c ose (m) Scose+3sine=O (n) tane=3cote
(o) �cosec�sin� (p) 2cosx= v'3cotx (q) 3sin2 e=4sine cose
{r) 5cot2 x= llcosec2 x-13 (s) 7sin2 x+2sinx=cos2 x
(t) Ssin2 6+2cose=2 {u) 3se c2 a= 8tana+6
190
TRIG. EQUATIDriS
(v) Stan2 8+8se cS+8=0 (w) 2 cosA -3secA=l
(x) tan4 x+7=4se c2 x (y) 4sinxcosx-2sinx-2/Jcosx+IJ=O
(z) sin2 B-2sinB cosB=3 cos2 B (A) 2sin2 e=sin8 cos8+ cos2 8+1
(B) 9sin8 cos8=4+sin2 8 (C) sin2 a-2sina-l=O
(D) sin8=3 cot8 (E) 2se c x=tanx+ cotx
(F) tanx+secx= cosx (G) (2tanx-1) 2 =3(se c2 x-2)
[Question 3 is to be done later for revision]
RR 3, In ea ch of the following, 0 ° 5- 8 ::: 360 °
A={8:�in28=1}; 11={8: (12cos8-l)(sec8-2)=0};
C={8: cos8 cot8=cos8} D={8:sin8=3 cos8};
E={8:2�os2 8+ cos8=1}; F=f8:8cos 4 8=6 cos2 8-l}
G={8: cot8-3tan8=2}; �={8:2sin2 8-cos8-l=O};
I={8:6 cos2 8+5sin0=7}; J={8:3se c2 8-4tan8=3};
K= {8: 2se c8·-3tan 8=2};
2 L={ e: 2cosec2 8=3 cot8+ 1};
M={0:2sin2 8=3sin8 cos8-cos2 8}; N={8:3 cos2 8+8 cos8sin8+sin2 8+2=0h
P={8:3se c8tan2 8-2tan2 8+15se c8-10=0};
Q={8:2sin2 8+3 cos2 8=2}
Determine t:he elements of the sets
(i) p. (ii) E (iii) C (iv) D (v) E (vi) F
(vii) G (viii) H (ix) I (x) J (xi) I� (xii) L
(xiii) M (xiv) N (xv) p (xvi) () (xvii) B n N
(xviii) c n 1 n :1 (xix) A U C U K (xx) (L n M) i.; I
(xxi) (C U Q) n (A U D)
Show that (a) E = H (b) p is an empty set ( c) EC F.

Case 2. EQUATIONS USING a ± 8, 28, 38, t Results


rro-rnustrate th� methods involved here, you should copy
each of the exer cises and work t:1em through carefully,}
Take the working of each question only as far as you need.
EXERCISE SET 4E
The domain of values of the variable is 0 ° to 360 ° or
oc to 2Ti c, (both·inclusive in each case).
1, (a) sin8= 13/2 Here (i) 8 = 60 ° or 120 °
l.· · Ti c c
or ( 1. ) 8 = or l!!.
3 3

(b) j sin28=/3/2 j Eere (i) 28=6() 0 ,l20 ° ,(36o+60) 0 ,


(36o+l20) 0 {Appropriate angles for 28 are required up to
2 revolutions; whenc e the values of 8 up to one revolu­
tion may be determined,} i.e. 8 =30 ° ,60 ° ,210 °,240 ° .
TI C TI C 7Ti c 4Ti c
or (ii) 8 = 6, , ,
3 � �

(c) sin38 = /3/2 !


P.ere (i) 38 = 60 ° ,120 ° ,420 ° ,480 ° ,(72o+60) 0 ,(72o+l20) 0
{The domain of 38 is 0 ° S 30 S 3x36C 0 }
i.e. 20 ° ,40 ° ,140 ° ,160 ° ,260 ° ,280 °
a=
TIC 2TIC 7 TI C 8TI C 13TI C 14nC
or (ii) a = , , , , - - 9
9 9 9 9 9

191
TRIG. EQUATIONS
<d> I s i n40=?tZTJ
Here (i) 40z60 ° ,120 °,420 ° ,480 ° ,780 ° ,840 ° ,ll40 °,1200 °
{0 ° .:!:40�4· 360 ° }
i.e. 0=15 ,30 ,105 ,120 ,195 ,210 ,285 ,300 °
° ° ° ° ° ° °

n c nc 7n c 2nC 13n c 7n c 19nC 5nC


Ol' (ii) 0 = 12 . 6 • 12 • 3 • 12 . 6 • 12 . 3

(e) I sin0/2=73/2 I
Here (i) 0/2 = 60 ° ,120 ° {O" .:s 0/2 .:S �.360 ° }
2n C 4n c
i.e. 0 = 120 ,240
° °
(ii) 0
01'
"'3• 3
(f) [sin 0/5 = ,/J/2
Here (i) 0/5 = 60 °
I {o• s 1 .:s t-360 ° },
5nC
S
i.e. 0 = 300 ° 01' (ii) 0 = -
3

2.
(a) i
cos2a = - I/./'[ I
Heral (i) 2a= (l80-45) 0 ,(l80+45) 0 ;(36o+135) 0 ,(36o+225) 0
i.e. 2a = 135 ° ,225 ° ,495 ° ,585 ° {0 ° s 2a � 2,360 ° }
i.e. a= 67� 0 , 112\ 0 , 247� 0 , 292\ 0 or (ii) a = 3;\ 5
l�n c 1 c
. t
t,
(b) ! tan23ti= I i i.e. tan3tl = ±1
Here (i) 3tl=(45 ° ,225 ° ,405 ° ,585 ° ,765 ° ,945 ° ) for tan3tl +l
and (135 ° ,315 ° ,495 ° ,675 ° ,855 ° ,1035 °) for tan3tl = -1
i.e. tl= l5 ° ,45 ° ,75 ° ,l05 ° ,135 ° ,165 ° ,195 ° ,225 ° ,255 ° ,285 ° ,
315 ° ,345 ° (arranging values in as cending order of magnitude.)
n c n c sn c 7n c 3n c lln c 13n C Sn c 17n C 19nC
or (ii) tl - 12 '4 '12 '12 ' 4 ' 12 ' 12 ' 4 ' 12 ' 12 '
7n c 23nc
4'12
Answers t o questions·-3:...11 given in degrees and minutes.
Take the workin as f ar as ou need.
3.
(a) j 2cos22x+3s i n2x-3=0 i Noting that cos2 2x=l-sin2 2x, using
result cos20= 1-sin20, then the equation be comes
2(1-sin2 2x)+3sin2x-3=0, i.e. 2sin2 2x-3sin2x+l= O. Henc e
find X,
t(b) I 3s ec23a-tan3a = 5 Here use sec 2 3a = l+tan 2 3a.
The equation be comes 3tan2 3a-tan3a-2= 0. Henc e find a,

��) I If 3sin(x + ·5> = 2cos(x - �), find tanx I


TI 2n TI
Now 3(sin x cos + cos
x sin )=2(c os x c os 3
3 3 2n
+ sin x sin 3),
using sin(a+B), cos(a-tl) results.
fi.) = 2 cos x(-1) + 2 sin x( Ii
Thus 3 sin x(1) + 3 cos x("""z """z �
2
whence sin x(3-2/3) = cos x(-2-3v'3) on grouping,
192
TRIG. EQUATIONS
-(2+313) 3+2;') _ -(24+1313) �- 24+1313
i.e. tanX =
3-2lf
X
3
3+2/3" - -3

(b) J If tan(8+60 ° )=-tan8, find a in domain 0 ° !: a 3}6'c!]


tan8+tan60 °
Here -tans, using tan(a+S) result.
l-tanStan60 o =
i.e. (13tan8+1)(tane-13) = o. Thus find e.
s.
(a) I 3sin28+2sifl 8=0 J Here 3(2sin8cos8)+2sin8 = O, using
result for sin28=2sin8cos8 i.e. 2sin8(3cos8+1)= O, on
factorising. Hence find 8.

(b) I sin8-cos 8/2=0 Here 2sin8/2 cos8/2 - cosS/2 o,


using an application of sin2a = 2sina cosa,
i.e. cos8/2(2sin8/2-l)=O, whence 8 may be found,

(c) I sin68+2cos38-sin38-l=O I
Noting sin68=2sin38 cos38,
the equation becomes 2cos38(sin38+1)-(sin38+1) = 0
i.e. (sin38+1)(2cos38-l)=O i.e. sin38 =-l or cos38=Y,.
Hence find a.

(d) I 16sin2e cos28 = I J Noting that sin28 = 2sin8 cosa,


' sin22a = 4sin2a cos 2 a,
the equating becomes 4sin 2 28 1, Renee find a.
6.
(a) 13cos2x-5sin2x· = cos2x J
Here we use the result cos2x=cos 2 x-sin 2 x. The equation
then becomes 13cos 2 x-Ssin 2 x = cos2 x-sin 2 x
i.e. 12cos 2x=4sin 2 x. Divide by cos 2 x(!O), 4tan 2 x=l2
;;ence find the values of x.

(b) j2cos2A-7cosA=OI Here using cos2A 2coi A-1, the


equation becomes 2(2cosl A-l)-7cosA=O
i.e. 4cos�A-7cosA-2= 0, Hence find A,

(c) J Y,-3cos4a = cos22a I lfow cos4a = 2cos 2 2a-l, using an appli-


cation of cos28 = 2cos 2 a�1. Thus the equation becomes
\-3(2cos 2 2a-l)=cos 2 2a; hence find a,

(d) i 3sin8-2cos28+1=0 I Here, we use cos28=1-2sin 2 8. The


equation becomes 3sin9-2(1-2sin 2 8)+l=O, Hence find a .

(e) J 3cos8+7sin8/2=0 ! Start cos8= 1-2sin 2 8/2. Show that


(3sin8/2+1).(2sin8/2-3)=0, and hence find 8,
7.
. 2tana
(a) I tan28=3tan8 I Noting that tan28 = 2 , the equation
1-tan 8
2tan8 2 8) = 0.
becomes = 3tan8; whence tan8(1-3tan
1-tan28
Thus find a.

193
TRIG. EQUATIONS
(b) l tan66=3tan30 I Using tan66 • 2tan30
- 2 , prove that
l tan 36
tan30=0 or 11/7:3. Hence find 6 ,

*(c) l 8tan6-5cot26-4=0 !(Answers in degrees and minutes only)


1 1-tan 2 e l-t2
Start cot26
• tan Ze = Ztane =� · where t = tan6,
Show that 21t -Bt-5 O i.e. t = 5/7 or -1/3,
2

Hence find 6.
8.
(a) i sin36-2sin6=0 i Using sin36 = 3sin6-4sin 3e, show that
sin6(1-4sin26)=0, and thus find 6,

(b) i cos36+cose=O i Usin� cos36 = 4cos 36-3cos6, show that


cose = O or cose = 11/12. Henc e find a.

(c) ( i) sin66-2sin26=0 Start sin66=3sin26-4sin 32a.


(ii) cose+cosa/3 =O Start cose =4cos 3 6/3-3cos6/3
(iii) sin66-2sin36=0 Star t sin66=2sin36 cos36
(iv) cos6+cos6/2 =O Start cosa =2cos 2 6/2 - 1
Solve for 6 in each case.

*(d) 2sin3x+3sin2x= O ! Using sin3x = 3sinx-4sin 3x and


sin2x=2sinxcosx, show that 2sinx(3-4sin 2x+3cosx) = O,
i.e. 2sinx(�cos 2 x+3cosx-1) = O. Hence find x,

(e) ! tan3x+tanx= O i Start tan3x 3tanx-tan 3 x 3t-t 3 w here


=
1_3tan2x = 1_3tz
2 -l)=O. Hence find x,
t = tanx, Show that 4t(t
9.
(a) i sin56cos6+cos5esina=cos36 i
Noting sinAcosB+cosAsinR=sin(A+B), show that sin66=cos36
and hence that 2sin36cos36 = cos36. Thus find a.

(b) isin2x/2 = cos2x ! Start sin 2 x/2 = �(1-cosx), using an


application of the result sin 2 e = \(l-cos26),
Henc e find x.

(c) J sin6 = cote/2 2t2• where t = tan6/2.


Start sine = l+t
2t
The equation then becomes ii?" = 1/t, and hence find t,

i
Start cos6 = l-t2 w here t=tan6/2.
2
(d) ! cose+tane/2-1 = 0
l+t
Show that the equation becomes t(t-1)2 =0, and thence
find 6,

(e) I 10sin2e-5sin2e= 4 i On using sin2e = 2sine cose, the


equation becomes l0sin 2 6-10sin6 cos6 = 4. In this type,
we r eplace 4 by 4(cos 2e+sin 2 e), and then divide both
sides by cos 2 e, Show that 3tan 2 6-5tan6-2 = O, and hence
find e.
194
TRIG. EQUATIONS
(f) cos2x-3sin2x+3= 0 i Substituting cos2x = cos 2 x -sin2 x,
sin2x = 2sinx cosx, 3 = 3(cos 2 x+sin2 x) , and then divid­
ing by cos 2 x as in (e) above, show that tan2 x-3tanx+2= 0,
and hence find x.
{Alter-natively, let cos2x = l-t 2t
2

l+tZ• sin2x = l+tz


where t=tanx}

1 0. Speaial Examples
(a) ! sin3x= sin2x J [Note
3x= 2x i.e. x= O, is only one possible
solution.}
Here 3sinx-4sin 3x= 2sinxcosx, i.e. sinx= O
or 3-4sin 2 x = 2cosx. Show that sinx= O or cosx = �- l±v'S
4-.
Hence prove that x= 0 °,180 ° ,360 ° or 36 ° ,1 08 ° ,252 ° ,324 ° .
{Note that when x = 36 ° , then sin(3x36 ° ) sin(2x36 ° )
since sin108 °= sin72 ° .
Thus x = 36 ° is a solution of the equation sin3x = sin2x.
l+v'S. •
Hence � � is the exaat value of cos36 ° , since cos36 ° > 0.
4
Likewise, sin(3xl08 ° ) = sin(2xl08 ° ) since stn324 ° = sin2 1 6 ° ,
i.e. x = 108 ° is a solution of the equation sin3x = sin2x.
lience l-{5 is the exaat value of cosl08 ° , since cosl08 ° < O,
but since cos 108 ° =-cos72 ° , , ·. cos72 ° = -( 1 -{5) v'S l.}

Graphical Solution
Further, these equations may be solved graphically
(although the results are only approximate).
1

-�
(b) J
sin3x = cos2x
Here 3sinx-4sin 3x 1 -2sin 2 x , i.e. 4s 3 -2s 2 -3s+l = O, where
s = sinx.
.
Show that (s-1) (4s 2 +2s-l) = O i.e. -l±v'S
s= l or s = �- �
4
Hence, prove that x= 90 or 18 ,162 ,234 ,306 °
° ° ° °

{Note that when x = 18 ° , then sin(3x18 ° ) = cos(2xl8 °), since


sin54 ° = cos36 ° .
Thus x = 1 8 ° is a solution of the equation sin 3x = cos 2x.
-1+/s
Hence �- �
4
is the exaat value of sin1 8 ° , since sin18 ° > o.
1 95
x 3 �ax+b = 0 TYPES
-1-rs
Li ke1,Jise, - - is the
4 exact value of sin234 ° ,
!+Is
°
but sin234 = -sin54 ° , is the exact value ·of sin54° .}
4
G.roqphiaal Solution
'j

11. Solution of Equation�_of the Type x 3 +ax+b 0


(a) (il Prove That sin38 = 3sin8-4sin3e
(ii l Letting x = 4 sine, verify that the equation
x 3 -12x+8/3 = 0 can be expressed in the form
sin38 = k, where k is some constant.
Hence, solve the cubic equation x L 12x+8/5 = O.
Method. (i) Begin· with sin(A+B) where A=2El, B=B-.
(ii) On substituting x = 4sin8, show that the equation
x 3 -12x+8h = o becomes 64sin 3 El-48sin8+8/3 = 0, 13
= 13
8
i.e. -l6(3sin8-4sin 38)+8>'3 = 0 i.e. sin3El
16 2
Henae show 38 = 60 ° ,120 ° ,420 °,480 ° ,780 ° ,840 °
and .·. a= 20 ° ,40 ° ,140 ° ,160 ° ,260 ° ,280 °
Now since x=4sjnEJ,
· x= 4sin20",4sin40� 4sinl40�4sinl60 ° ,4sin260 ° ,4sin280 °
i.e. x= 4sin20 ° ,4sin40 ° , 4siu40 ° , 4sin20 °, -4sin80 ° ,-4sin60 °
i.e. x= 4sin20 ° ,4sin40 ° ,�4sin80 °
[The others are repetitions of these.)
i.e. x= l·3680,2·5712,-3·9392
[These are the roots of the equation x 3 -12x+8>'3 = o.]
Gra:phiaally, these roots y
give the approximate 8J
values of the abscissae
of the points of inter··
section of the curve -x
+2---�--+- --+'111:--+--,j�
y = x 3 -12x+8>'3 and the �-+i�-_+�-_ 3 , o
x-axis. -Y

(b) (il Prove that cos38 = 4cos J8-3cose.


(ii) By letting x = 7cose, verify that the equation
x L 3x- I
�------ J roduces to cos3El = !,.
Show that, for 0 ° <8<360 ° , then 8=20 ° ,10() 0 ,140 ° ,220 ° ,260 ° ,340 °
and hence that x=2cos20 ° ,-2cos80 ° ,-2cos40 ° . (The other 3
196
a cose + b sine = c
values are repetitions of these:- note 2cos220 ° m -2cos4o 0 ,
2cos260 ° = -2cos80 ° , 2cos340 ° = 2cos20 ° ,)
Thus find the 3 roots of the equation x 3-3x-l�O, correct
to 2 dec imal plac es.
EXERCISES SET 4F
1. Solve the following equations, for t'he variable in the
domain 0 ° to 360 ° , or oc to 2nc (both inc lusive).
Express the roots alternately in degrees and in radians.
[Take the working only as far as you need. Question I
parts A� I; question 2 (ii); question 3(iii); should be
left for revision. 1
(a) c os0 =� (b) c os20 =\ ( c) c os30 =\
(d) c os6/3=� (e) tana = -1 (f) tan2a=-l
(g) tan3a =-1 (h) tana/2=-1 (1) sin 2 B =\
(J) 2
sin 2B �
= (k) sin2 3B=� (l) sin2 B/2=�
(m) sin26- cos0= 0 (n) sinx1sinL�=O (o) sin4a-sin2a =O
(p) c os26-cos6 = 0 (q) cos2 6+2cos26=� (r) c os26-1= 2sin0
(s) 3cos26+sin2 6+2sin6= 0 (t) cos2x= 4c os2 x-2sin2 x
(u) 4tan26+tan0 = 0 (v) tan20-c ot0 = 0 (w) cos36 = cos8
(x) sin3x+sinx=O (y) sin(x+n/3)= c os(x-n/3)
(z) sin(6+30 ° )=sin(0-30 ° )� (A) cos26 cos6-sin26sin6 =sin36
(B) sin78 cos6+cos76sin6=sin46 (C) 5sin2 a-3sin2a+7cos 2 a = 3
(D) c os2x-�in2x=sin x
2
(E) 2sin6 c os26-2c os26-sin6+1= 0
2 2
(F) cos46se c 26+se c 26 2c os46+2 (G) cos x/2= cos x
=
(H) c os4x=sin2x (I) sin 2 9 cos2 9=\

2. (i) Solve the trigonometric al equation sin2x= cos3x for


O�x�2n. Sket ch on the same diagram, graphs of
y=sin2x and y=cos3x for x in the domain o c to 2n c
both inclusive. Hence solve sin2x = c os3x graph­
ic ally.
RR(ii) Solve the equation cos2x =c os3x for 0 ° � x � 360 ° .
By sketc hing the curves y = cos2x, y = c os3x for
0 ° � x � 360 ° , solve graphically c os2x = c os3x.

3. (i) Show that the equation x 3-3x+l = 0 reduces to the


form sin36 =\, on putting x = 2sin6, Use the trig­
onometric tables to obtain the 3 roots of the
algebraic equation.
(ii) If x=1+2cos6, express x 3 -3x 2+1 in terms of cos36
and henc e solve the equation x 3-3x 2+1 = O .
RR (iii) Prove that sin3A= 3sinA-4sin 3 A.
Using this result find from your tables, c orre c t
to 3 de cimal places, the smallest positive value
of x which satisfies the equation 12x 3 -9x+2 = O.

Case 3, Equa.tions of form aaos6+bsine = ·a


EXAMPLE I. Express 3 c os x - sin x in the forms
(i) Acos(x+a) (ii) Bcos(x-B) (iii ) Csin ( x+y)
< iv) Osin(x-6) where A, B, C, Dare positive, and
oc � a,B,y,6 � 2n c.

Method (i) lfc osx-sinx = /( 13) 2+(-1) 2 [2cosx-!sinx]


Che c k this step!
a cose + b sine = c
13 . �----
=2[ cosx-fsi.nx]•••• (P), noting /("3) 2 +(-1) 2 =2
2
= [cosa cosx-sinasinx], where cosa = /)/2, sina =l,
2
=2cos(x+a),
sina l, 1
where tana � = 7f7z = 73, and a is a cute, _since both
11C
cosa, sina are positive. 'I'hus a = 6
Hence /J cosx-sinx= 2cos(x + f> [Note A 2, a
c
f l

(ii) From lin�(P),


/Jcosx-sinx= 2[ cosx-l,sinx]
2
2
= [ cos8 cosx+sin8sinx], where cos8
13
, sinS = -!,.
=
2
-1
=2cos(x-8), where tanB = and since cos8 > O,
73,
sin8 < 0, S lies in the fourth quadrant,
• a 2!. c 1111 c
1..e. " (211 - ) =
6 6
1111 1111 c
Hence /Jcosx-sinx 2 cos(x - --) (Note B 2, 8
6 -6- l
(iii) Also, from line (P),
/Jcosx-sinx=2["4:osx-l,s.inxJ
2
= [siny c osx+ cosysinx], where siny cosy -�
2'
=2sin(x+y), where tany -13, an d sin ce siny > O,
cosy < o, 1T C 2JTC
Y lies in the second quadrant, i.e. y =(11 - ) �.
3
2
Hen ce IJcosx-sinx = 2sin(x + ;)

(iv) Further, from line (P),


/Jc osx-sinx=2(�osx-\sinx]

=2 (-sinocosx+cososinx], where sino = -;3, c oso = \- ,,


= sin(x-o), where tano= IJ, and since sino < O,
2
c oso < O, 11 < o <
3 ; c
; i.e. o = (11 + .!!. ) !±:!!..
- 411
3 3
Hen ce /Jcosx - sinx = 2sin(x - �)

EXAMPLE 2. Find the (a) maximum ( b) min i mum va I ue


of /3"cosx-sinx, and state the first positive value
of x for which this occurs.

From example 1 above, /Jc osx-sinxcan be expressed in various


forms as shown.
Using (i}, l3cosx-sinx= 2cos(x + 2!.6)
(a) the maximum value of l3cosx-sinxis 2
[sinc e -1 S cos(x + f> S l ,]
"
f>
�ff X

and this o cc urs when cos(x +


i.e. when x + f
= 0,211,411, ,,,,,
2!.
_, -�
= 1111
The smallest positive value of xis (211 - )
---------··---- 6 6
198
a cos e + b sine= c
(b) the mi� value of /Jcosx-sinx i.e. of 2cos(x + f>, is
-2, and this occurs when
cos(x+71/6)=-l, i.e. when (x+71/6) = 71,371,571,,,,,
5
The sma11 est posit ive va1ue of x is (71 - !.) _ 71
6 6
{Note, the same values are obtained by using results (ii),
(iii) or (iv) of example 1 above.
4
Thus from (iv), {Jcosx-sinx =2sin(x - ;).
(a) The maximUJ7! value of hcosx-sinx is 2, :1/v � = ,4,<,n x
4
[since -1 � sin(x - ;) � l], ;:,.
, 471
4
and this occurs when sin(x - ;) = 1,� � >
-371 71 571 971
/,j��
i.e. wnen x - = , , , ,
3 -2- 2 2 2
471 .!L 1171
The smallest positive value of x is + =
3 2 6
4
(b) The minimum value of 13cosx-sinx i.e. of 2sih(x - ;), is
4
-2, and this occurs when sin(x - ;) = -1,
471 = -71 371 771
i.e. when x - , ,
3 2, 2 2
471 71
The sma11 est posit · ive va1ue of x is - = S:rr
3 2 6
Verify that the other results lead to the same answers.}

EXAMPLE 3. Solve the equation cosx-sinx = I for


0 s x s 271 (usin the results of exam le I.)
Method. ?ow by example 1,
1171
13'cosx-sinx =2cos(x+71/6) = 2cos(x - ) 2sin(x + �)
6 3
. 471
= 2sin(x - )
3
f:'�Y select any one of _tf!:.ese.
(a) For example, taking {Jcosx - sinx = 2cos(x - 1171 -), the
6
1171 71
equation becomes 2cos(x - ) = 1, i.e. cos(x - 1 1 ) :!2
6 6
1171 71 -71 r. . 71

and x = ( t-
Eence x - � = .•• ,-(271 - ), , ,(271 - ), ...
1 5 !
3 3
;), < 71 -1).
1
3 3
{The other solutions do
not lie in the given domain} i .e. X =716' l.!!.
2
2
(b) For example, taking hcosx - sinx =2sin(x + ;),
2 2
equation becomes 2sin(x + ;) = 1, i.e. sin(x + ;)=\.
.
hence (x + 2'1) 71
71 71
= ,,,, 3-(71 - 716)'6'(71 - 6),(271 + 6),,,
-271 571 -271 1371
and . · , x = (-- + ), (- - + - -) • Tnese are t',Je only
{,
3 6 3 6
71 371
values for x in the given domain}. i.e. x = ,
6 2
t,hat the use of tlle other two results yields the same
solutions.

199
a cos a+ b sin a = c
EXAMPLE 4. Solve the equation 3cos6+4sin 0 = 2, for
0° Sa S 360 ° , by introducing an extra angle (cal led
a subsidiaPy angle> in the work. Use the results of
this question to solve the equations
(a) 3cos20+4sin20 = 2 (b) 3cos0/2 + 4sin0/2 = 2
Method. Step 1. Divide both sides by � = 5.
3 2
The equation 3cos0+4sin8 = 2 becomes cos8+ 4 in8 = S ... {Q)
5 s5
IntI'Oduae an aaute angle a such that cosa = land sina = !
5

LJ
5
sina _ ill _ !
i.e. tana _-
cosa - 3/5 - 3'
i.e. a � 53 ° 8', from the tables
Thus equation {Q) becomes cosacosa+sinasin • =

f 4
5

i.e. cos{8-a) = -
5 3
Now from tables, 8-a = 66 25' or (360 ° -66 ° 25').
0(
°

i.e. 8 (53 8'+66 25') or (53 8'+293 35')


° ° ° °

i.e. 8 119 d33• or 346 ° 43'

Notes (i) the 'subsidiary angle' a introdUtJed in this method


need net be aaute, but the working is simpler if sina, cosa
are both taken positive, i.e. if a is taken to be acute.
(ii) instead of taking cosa = 3/5, sina = 4/5 from
equation (Q) and proceeding as indicated above, we could take
cos8 = 4/5, sin8 = 3/5 whence equation (Q) would become
sinBcos8+cos8sin8 = 2/5 i.e. sin{8+8) 2/5.
3
Here t an..,O = , 1.
• e. O" = 36052 I
4 2
From sin(8+8) = 5, (8+8) = 23 ° 35',(180 ° -23 ° 35'),
(360 ° +23 ° 35'), ••.
Thus 8 = (23 ° 35'-36 ° 52'), (156 ° 25'-36 °52'), (383 ° 35'-36 ° 52').
The only valid solutions are 8 = 119 ° 33 1 or 346 ° 43',and these
agree with those found _a_b_o_v_e _. _____
(iii) the values of 8 are more easily determined, if the
compound angle result be used with 8 before a. That is, the
result (cosacosa+sinasin6) would Le written as cos(6-a)
not cos(a-6), although these are identical,
Similarly (sinacos6-cosasin6) would not be given as sin(a-8)
but would be rewritten as -(sin8cosa-cos8sina) = -sin(8-a).
The roots of 3cos8+4sin8 = 2 for 0 ° � 6 S 360 °
are 8 = 119 ° 33', 346 °43'
(a) The roots of 3cos28+4sin28=2 for 0 ° S 28 � 720 ° are thus
given by 28=119 ° 33',346 °43',(360 ° +119 ° 33'),{360 ° +346 ° 43 1 )
i.e. 0 = 59 ° 49\', 173 ° 21�', 239 ° 21\', 353 ° 21�'
(b) The roots of 3 cos i + 4 sin J1. = 2 for 0 ° S J1. S 180 ° are
2 2 2
thus given by 9/2 = 119 ° 33' i.e.-6-=_2_3_9�0-6-,-

200
a cos e + b sine = c
EXAMPLE 5. Solve by using the t results, the
e uation 3cos0+4sine = 4.
e
, then cos e = 1-t2, sin e 2t
2
� Now if t = tan
2 l+t l+t2
Substitution in the given equation yields
3(1-t 2) 2t _
+ 4. - 4, i.e. 3(1-t2 ) + 8t = 4(1+t 2 )
l+t 2 l+t2
1
i.e. 7t 2 -8t+l = O i.e. (t-1)(7t-1) = O. Thus tan i = 1 or 7
Since 0 �
°
1� 180 , •.
°
1 e
° ° 1
2
= 45 or 8 8 (The other values
of e are outside the domain of ). i.e. e = 90 ° or 16 ° 16 1
2

NOTES ON THE "SUBSIDIARY ANGLE" AND THE "tan i= t'' METHOD OF


2
SOLVING EQUATIONS OF THE TYPE acose·+ bsin0 = c
The most generally appZiaable method is the Subsidiary
Angle l.fethod, sinae the equation aaose + bsinB = a may be thus
solved for all values of e.
(a) It is definitely to be preferred over the t method in.
the cases when (i) the aoeffiaients a..b.a are irrational.
(Thus l3cos0-sin0 1 may be solved easily by the 's�b;ic!iary
angle' angle; but the 't' method leads to awkward manipu­
lation,)
!al,
(ii) the aoeffiaients jbj are the smallest
pa:f:t.!!f _a__"Ejj_JhClg_orean triad. (Thus 3sin0-4cos0 = c for- any
real c, may be easily solved; since we first divide by
132+(-4)2 = 5.
Similarly for equations such as 12cosB+Ssin0=c, 7cos0--24sin8=c)
(b) Howeve�, the subsidiary angle method needs more aareful
thought in its application, In this respect, the 't' method
is easier to follow; {in most cases, especially when the
quadratic equation in t may be solved by the use of faators.)
(c) One further problem arises in the use of the 't' method,
and that occurs when one of the solutions of the equation
acos·e+bsinB = c is 8 = 180 ° .
Thus, aonsider the equation 2sinB-3aos8 = 3�
A .solution of this equation is e = 180 ° ,
since 2sinl80 ° -3cos180 ° = 0-3(-1) = 3
2t 1-t
1
However, by the 't' results, since sinB = cose =
l+t2• l+t2•
8
where t � tan , the equation becomes
2
2(2t) 3<1 t 2 )_ .
- - 3, i.e. 4t-3(1-t 2 ) 3(1+t 2 ), and
1+t2 l+t:z
2 e 3
the t term cancels out, giving 4t = 6 i.e. tan
2
= 2 only.
This gives I= 56 19' °
i.e. e = 112 38'
°

Note here that the 't method' fails to yield the solution
e = 180 ° . If the 't' method is used, and the t 2 term
cancels out, then the value e = 180 ° should be tested by
substitution in the original equation.
(d) Both methods are subject to slight inaccuracies because
of the (i) double use of tables in the 'subsidiary angle'
method, 201
SET 4G
(ii) use of tables to find I in the 't' method and
hence the magnification of the error. if any. when
a is derived.

EXERCISES SET 4G
!Questions 3,4,9 are intended to be left for rev1s1on later.!
L (i) (a) Express cosa +sina in the form Acos(e-a). where
A>O and O c <a<2nc.
(b) Hence find the (i) maximum (ii) minimlUII value
of cosa+sina. and give the first positive
angle a which produces these values.
(c) Use the result of (a) to soive the equation
cos8+sin8= 1 for 0�5 8� 2n C.
(ii) Solve the equation cosa+sina = 1. using the t
method.
2. (i) (a) Express 7cosa + sine in the form Bsin( a+B) •
where B > 0 and 0 ° < B < 360 ° .
(b) Hence determine the (i) maximlUII (ii) minimum
value of 7cosa+sine. and thus find the first
positive angle a givino �hese values.
(c) Use the result of (a) to solve the equation
7cos8+sin0 = 5 for 0 ° �a� 360 ° .
(ii) Verify the results of (c), by using the t results.
(i) (a) Express sin8-2cos9 in the form r sin(8-y).
where r > o. 0 ° < y < 360 ° .
(b) Hence, find the (i) maximum (ii) minimum value
of sin8-2cosa. and a possible value of a prod­
ucing these results.
(c) Use (a) to solve the equation sin8-2cos8 = -1.
(ii) Use the t results to solve the equation
sin8-2cosa= -1.
RR4. (i) (a) Express 4cos0-3sin8 in the form tcos(a+o)'
where t > O, 0 ° < 6 < 360 ° .
(b) Hence, determine the (i) maximlUII (ii) minimum
value of 4cos8-3sin8, and calculate the small­
est positive a producing these results.
(c) Use (a) to solve the equation 4cos8-3sin0 = 3.
(ii) Solve the equation 4cos8-3sin8= 3 using the taIT'z
e
results.
s. e
By using the tan results, show that the equations
(a)
2
(i) Ssin8-12cosa = 13 (ii) Ssin8-12cosa = 12
both lead to one value of tana /2. Further, prove
that a= 180 ° is a root of one equation but not of
the other. Give the roots of each equation in the
domain 0 ° �as 360 ° .
(b) Express Ssin8-12cosa in the form Asin(e-a) where A>O
and 0 ° �as 360 ° , and hence solve each equation of
(a).
202
SET 4G
6. (i) Transform cos6 -r'3sin6 in the form
(a) Acos(6+a) (b) Bcos(6-13) (c) Csin(6-y)
(d) Dsin(6+6) where A,B,C,D > 0 and O�a,13,y,6�271.
(ii) Hence determine the greatest and least values of
cose-1:f sinB, and state the first positive angle e
for which these occur.
(iii) Solve the equation cos9-/3sin9 = 1, for O .S 9 � 211,
without using tables.
(iv) Use the result of (iii) to find solutions, for
0� 9�211, of the equations
(a) cos29-/3sin26 = 1 (b) cos !.
2
- 13 sin !.
2
= 1

7. Use the subsidiary angle method to solve each of the

1
equations given below, for 0 ° �9�360 ° .
(i) sin9+2cos6 = (ii) sinB - l:fcose = 12
(iii) 4cos9-3sin6 = -2 (iv) 5sin6+12cos6 = 6·5
(v) 8cos9-6sin9 = 7
8. Use the tan
e
2 results to solve each of the following
equations for 0 °� 9 � 360 ° .
(i) 2sin6-cos6 = 2 (ii) 3cos6+sin9 = 1
(iii) 4cos9-3sin6 = 5 (iv) 4sine+3cos9 = -3
*(v) 3cos9+sin9 = 2
Solve each of the following, for o � e � 211, by
a
(a) the subsidiary angle method (b) the tan method
2
(i) sinB-cosB = -1 (H) 2cos6-sin9 = 1
(iii) cos9+7sin9 = 5 (iv) 7sinx+24cosx = 25
(v) 7sinx+24cosx = -24

*10. (i) Express the fwiction 6cos 2 9+8sin9cos6 in terms of


cos29 and sin29. Hence deduce an expression for
the function in the form A+5cos(29-a) where A,a
are constants, and 0 ° � a i 360 ° .
Thus write down the greatest and least values of
the function, and find, correct to the nearest
minute, one value of 9 corresponding to each.
(ii) Use the result of (i) to solve
6cos 2B+8sin6cos9 = 4 for 0 ° �a�360 ° .
tu. (a) Show that sece+tane = l+sine and hence solve the
cosB '
equation secB+tanB = 2, for 0 °� a i 360 ° .
(b) Noting that sec 2 a-tan 2 9 = 1, and given
that sece+tanB = 2, prove that sec9-tan9 =�. Find
e , by solving simultaneously sec9+tan6 = 2,
· secB-tane ,. l5,
tl2. Solve for x·in the domain 0 ° ix� 360 °
(i) sec x - tan x = 2 (ii) 3tanx+secx = 1
(iii) 7-24cotx = lOcosecx.
203
DERIVATIVES OF cosec x, sec x, cot x
I. DERIVATIVES OF cosec x, sec x, cot x
The derivatives of the circular functions sinx, cosx, tame
(where x is in radians) have already been treated in Form 5,
These results l.e. !(sinx) = cosx; !(cosx) = -sinx;
d d
�(tanx) = sec x} are now extended to the functions
2

(i) cosec x (ii) sec x (iii) cot x,


1
(i) Let y = cosec x = - � = ( sinx)-1
s inx
d d
Thus�= -l(sinx)- 2 . (sinx), as a function of a func-
dx
d tion.
-l( sinx)-2 .co sx, since (sinx) = co s x
dx
-cosx
sin2x
c?sx
Thi s result may be rewritten a s .::.l__
sinx s1nx =-cosecxcotx
Hence, dxd(co s ec x) = -co s ecx cotx
1
(ii) Let y = sec x = - -- = (cos x)-1
COS X

Thus 1; = -l(co s x)-2.�(cosx) a s a function of a function


d
-(co sx)-2.- s inx, since -(cosx) = ·- sinx
dx
s inx 1 sinx
= = sec x tan x.
co s2x cosx· cosx
i.e. ;(sec x)= s ec x tan x
�-,-�������
1
(iii) Let y = cotx = - - = (tanx)-l
tanx d
.Q.Y. -- -l(tanx)- 2. (tanx)
dx dx
-l(tanx)- 2 . s ec 2 x, since !(tanx) = sec 2 x
d
s ec 2 x 1 x cos 2 x _ 1
- = =
tan2x - cos2x s in2x sin2x
- cos ec 2 x
i.e. a!<cotx) cos ec 2 x

ASSOCIATED
FUNCTION & SKETQ-1 DERIVATIVE INTEGRAL
y · �(sinxl= cosx
d fcosxdx=s inx+C
�Csinax)= acosax Jcosaxdx
d
=; J_ s i nax+C
a

COS X y•
d
i(cosx) = -sinx sinxdx=-cosx+C
d
(cosaxl Js i na dx
dx 7
_."'-...,.....__...----1-��----.1o1-.� = - a s i n ax
=-- cosax+C
a

204
SIJ,1MARY
ASSOCIATED
FUNCTION & SKETOi DERIVATIVE INTEGRAL
tan x; fsec x dx
-,
-2.c 2x
dx tanx ) =sec
I
=tanx+C
2

x-l(2n+ I >2 I
r
'II

�(tan ax) Jsec 2 dx


I I

·rr = asec 2 ax
��,___,._��-........-..J.-�.:..

= -'-1-a nax+C
X

a
•ff

I
I

se� : a
d�<: �e� x ta n x f e: :: �
e ) x

,.._..."-+-n_..._ +---'w -l-:--+e---,1----�


(se c ax)
d� = aseca x ta n ax J
sec a x ta nax dx
= -;;s1 ec ax + C

[�otes 1, A useful mnemonic to remember is that the deriv­


atives of airaular funations beginning with a (namely cos x,
cosec x, cot x) give a - sign in the result.
d d
Thus (co x) = -sin x; (cosec x) = -cosec x cot x;
dx : dx
2
d
/cot x) =_-c_ os_ _e_c_ x_ ._ ______
£.• In all the derivatives (and integrals) to follow,
we ass1.111e that the function is defined for the domain of x
(under consideration or implied under consideration. Thus,
d
for example, (cot x) = -cosec x, is true -for all values of
2
clx
x except at x = I111; see sketch above.)]
205
EXAMPLES ON DERIVATIVES
EXAMPLE I Find f in each of the fol lowing X
( i) y sin7x (ii) y cosec 7x (iii) y cos
2
(iv) y seCz
X
(v) y tan x 0 (vi) y = cot x 0
d
Method (1) dxd(sin 7x) "' cos7x,dx(7x) = 7 cos 7x
d d
(11) dx(cosec7x)=(-cosec7x cot 7x). (7x)
dx
=-7cosec 7x cot 7)". .•
{Altel'Ylatively, to differentiate cosec 7x ,-,e may proceed as
follows.
Let y = cosec7x 1 and put u = 7x,
f = -cosec uducot u;
• •• y = cosec u, whence
whilst d = 7, from u = 7x.
Using esul t f = fu f = (-co:ec u cot u) . 7
i.e.
� !�
(cosec 7x) = (-cosec7x.cot7x).7, re-substituting for
dx
u = 7x.}
(iii) d/co5z) -sinX2 . dX
dx( 2) ··>asin2'.
d X t X
=
d X X
(se� tanX) d X X X
(iv) dx(se�) 2 . d/2) = �e� tan2
lAltel'Ylatively, let u = f, . ·.
if y = se�, • •. y = sec u.

du .
dv dv du
Thu� = dx = (secu tanu).\ !,se�x tru1z.}
x
%z

(v) If y = tanx 0 , then before differentiating, we must


transform x 0 into radians, using the result 180 ° = Ti c.
Here x 0 = xc , and thus tanx 0 = tan( ;�)
1;0 1
No
' w ...Ji( tanx ") -- ...Ji{ tan�} -- "oec � • ...J!(....2!2!.)
dx dx 180 180 dx 180
2 • _TI_ ' noting
• sec �- 2x•
_::_:_:__...'.__li!Q 180- sec
(vi) ...Ji
TIX _ _ 2...2!2i _4 TIX
(cotx O) -_ _4 dx cotl80} - cosec 180. dx(l80'
{ )
dx
180cosec x
= .=1!..... 2 0

EXAMPLE 2. Find f 1 (xl if


Ci) f(x)=sec(x 7 ) Cii) f(x) = cosec(l)
X
(iii) f(x) = cotCl3x+ll (iv) f(xl=x 2 sec2x (v) f(x)= �
X

.\Jethod. d
�f'(x)={sec(x 7)tan(x 7 )},dx (x 7) = 7x 6 .sec(x 7) tan(x 7)
3 d 3 +3 cosec(-)cot(-)
3 3
� (-)
(ii) f'(x)={- cosec(-)cot(-} = �,
X dx X X :l. X
3 d
(iii) f' (x)={-cosec2 (13x+l)} dx(l3x·H) = �osec 2
(/3x+l)
2
206
EXAMPLES ON DERIVATIVES
(iv') f I (X) = �( sec2x ) + sec2x. !(x 2 ), using the product
d
x

2
= x .(sec2x tan2x).2 + sec2. x 2x
r ule.
= 2x sec2x (x tan2x+l) on factorising
Ll(cotx ) - cotx
dx ....!!cx 3 )
dx , using the q uotient
x

(v) f I (X) 2 rule.


3
(x )
3 • (-co se c2 x
x
i -
X
-x 2{x cosec 2x + 3 cot x} -,x cosec2x+3cotx)
=
x6 x4

EXAMPLE 3. This ex ample introduce s othe r functions


pr eviously studie d; th e logarithmic, ex ponential
and inv erse trigonom e tric functions.
F"d.21.if
in
dx
3secx 3x
( i) y= e (ii) y = sec(e ) (iii) y = log(cot3x)
(iv) y=cot( log3�) (v) y = log3x .cot3 x
(vi) y = log(
ex c
%
ec 3
�) (vii) y = tan -1(cot f>
cosec
(viii) y = cos-l( e -
x
)

Method.
3secx 3 ecx
(i) c!<e )={e s }. !(3secx) • noting d�(e )
u u
e •
d
3se cx
=3secx tanx . e
.. d { 3x x 3x d 3x
(u.) dx sec(e ) }= { sec(e )tan(e )},d(e ),
3
X •
us ing
d tanu
du(sec u) secu
=
3x 3x 3x
= 3e se c(e )tan(e )
d
(iii) dd{log(cot3x)}={� 1 } . d (cot3x), noting
d
(logu) = l
X COt3X X dU u
1 2
-(cosec 3x). 3
=
cot3x.
(This res ult may be rewritten as
-3 sin3x -3 -3 6
sin23x · cos3 x = sin3x co s3x = !;;sin6x = ·-6 cosec X·)
d d
(iv) {cot(log3 x) }={-cosec2(log3x) }. dX (log 3x),
dX d
noting � (cotu)=-cosec2u
du
1
= -cosec2 (1og3x)."j;.3 = -cosec2 (1og3x)/x
d d d as a
(v)
dx {log3x.cot3x}=log3x.dx(cot3x )+cot3x.dx (log3x), produ ct
1
=log3x.-3cosec23x+cot3x."j"""" .3
·cot3x x
23x.log3x
= -- - 3cosec
X

207
EXAMPLES ON DERIVATIVES
2
(vi} N
' ow --rx--} .. 1
1og ( e
X cosec 3x x2
og e +'...og cosec 3 x-log./;,
using 1 og
laws
=x log e + 3 log cosec x - \ log x
2

1 1
. -1
Thus .£1. d = 2x + 3 - -- (-cosec x cot x) - -
dx cosecx 2 x
[Note loge= l]
2x _ 3 cot x _ -1.
2x

l+(cotx/2) 2 , dx(cotx/2),
1 d
using _!! 1
(tan-lu)
du = l+u2
1 1
-cosec 2 x/2 •
l+cot2x/2 2
-\, since cosec 2 x/2 = l+cot 2 x/2
-cosecx }= 1 _-4(e-cosecx)
(viii) -4{ cos-l (e )
dx /i -cosecx ' dx '
-(e f
l
noting _i(cos-1u) = - 2 = - �(sin-1u)
du 11-u au
1 - cosecx
e • cosecx.cotx
= 1 -2cosecx
- ,l-e

EXAMPLE 4. Find .<1Y. if


dx
2x-l-)
(i l y=sec5 c- ( i i ) Y -- I x = a cot 2 e
(iii)
3 /4+cosec3x y=b cosec3 e

Method. Zx-l s
(i) l'.ere y = [sec(- - )]
3
_s!Y. 2x-1 4 d 2x- 1
dx = 5 [sec(- -)-] . {sec(- -)}
3 dx 3
2x-1 , { 2x-l 2x-1 } -4 2x-1
= �sec4( ) sec ( )tan( ) , ( )
3 3 3 dx 3
10 2x-1 2x-l
= - sec 5 (--) tan (--)
3 3 3
-
(ii) liere y (4+cosec 3x) !,
.s!Y. -- - t(4+cosec3x)
- 3h. ,
dx (4+cosec3x)
d
dx 3cosec3x cot3x
-
- \(4+cosec3x ) 3k, (-cosec3x cot3x),3 %
2(4+cosec3x)
(iii) If x = a cot 2 e=a(cote)2, • ·. : = 2a(cote) l, (-cosec 2 e)
If y b cosec 3 e = b(cosece) 3 ,
• _s!Y._ 3b(cosed))2(
. -cosece cote)
. , de -
.l!Y _ � _ -3b cosec e 3
cots = 3b cosece
Thus - d
dx x/de - -2a cosec2e cote 2a

EXAMPLE 5.
(a) If f(x)=3tanx-2cotx, find Ci) f'<f> c i i > f" c .!. l
4
What is the geometrical significance of these results?
(bl Determine the equation of the tangent toy= f(x) at
208
SET 4H
11
x = , and find where it crosses they axis.
4
Method.
(a) Here f'(x)=3sec2 x+2cosec x
2

and f"(x)=6( sec x) 1• (sec x tan x)


+4(cosec x) 1 .(- c osec x cot x)

=6sec2 x tan x - 4c ose c2 x cot x.

Thus f'(.!!.) = 3sec2 .!!. + 2 c osec2 .!!. = 3(/z) 2+2(/z) 2 = 10; Geo-
4 4 4
metrically, the tangent at x = { to y = f(x) has gradient 10.

Further, since f'({) > O, • • the curve is rising at x = {,
11
Also f"(.!!.) = 6sec2· .!!. tan .!!. - 4 c o sec2 .!!.' cot
4 4 4 4
i
6(/2) 2 . 1 - 4(12) 2 .1 = 4-,

Geometri cally, since f"({) > O, • ', the curve is ooncave up


at X = 11/4,
11 11 11 11
(b) Nov f(
4) = 3 tan
4- 2 tan 1, and f' (
4) 10
=
4
Thu s the equation of ti1e tangent at <f, 1) on y= 3tanx-2cotx is

y-1 = lO (x - {>, This tangent c ros s· es the y axis, where x = 0

i.e. when y = 1 + 1 0(
-11
4
) = 1 - z·
511

EXERCISES SET 4H
1, Find 1; in ea h c of the.following

(a) y = sin9x; y = c os9x; y = tan9x; y = sec9x; y = cosec Sx

y = se c
0
(b) Y = s ec(x/9); y = cos ec (x/5); y = c ot(x /2);
x

(c) Y = s e c(x );
3
y = c os e c (l/x); y = cot(,'x); y = sec(hx+S)
(d) y = x c osec x; y = (c ot xVx; y = /c ose c x)/x2; y-= x 7 sec x
sec x
(e) y = e
x
; y =
log(se c3x); y = se c(e ); y = sec (log x)
-cosec X, -x
(f) y e ' y = log( co sec 2x); y = c os ec(e )
c ot3x
(g) y e ; y = log(cot3x); y = c ot(log 3x); y =c ot(c ot3x)
c o s ec x x x
(h) y sec x log x; y y = e c otx; y= log(e . se c 2 x)
log x'
c x
(i) y
-1
log( rx >; y log(log secx); y = tan ( c ot2x)

(j) y sec x;
3
· y = {c:ose c 2x; y = ----=-
1
--. · y se c 2x-tan2x
(cotx_t3) 2 '
t (I�) y =c os e c 7 3x; y=(2-Scot2x) 6 ; y= y=lsecx+tanx
(3+4cosec 7x '

2, Find 1; in the following


(i) fx= a sec0 (ii) ix=a s e c lj! (iii) pc = 2cot3 a(iv)f x= Ssin3 e
[y= b. tan0 ly= b cos e c lj! ty= 3c ose c 2 a LY=4cot 2 e
209
SET 4H
*(v) x = 0/sece, y sece/e.
3. (i) If y = x sec 2x, find� when
(ii) If f(x) log(cosec x), find
=

(iii) Given that y = cos-1(cotx), determine the value of


f when x =
se x

(iv) If g(x) = :x • calculate the value of g'(O).
4. For the function y = tan x + cot x, show that
..QY. = sec 2 x - cosec 2 x and�!= 2(sec 2 x tanx+cosec 2 xcotx)
dX d d2 TI
Calculate the value of�. dx!when x = . Explain the
4
geometrical significance of the result. Find the equa­
tion of the (i) tan gent (ii) nonnal to y = tanx + cotx,
where x = TI/4.
5. A particle moves along a straight line so that its dis­
placement x from a fixed point O at time t is given b y
x = 2sec\t. Find the (i) velocity (ii) acceleration of
4TI
the particle when t = .
3
6. If y= sec x, show that
d2
(a) J = y(2y2 -1)
d
(b) 6_
Tx"2 = tanx.
d
f
+ y3 .

7. (i) Prove that !{tan-1(sec x + tan x)) = �.


d
(ii) Show that 1 {tan- 1 (cotx)} = a!{sin- 1 (cosx)}, and
x
hence prove that tan- 1 (cotx) - sin-1 (cosx) = C,
where C is a constant. Calculate the value of C.
8, [This exercise is extre111ely imp ortant; the results wi U
be used in the later work on integration. J
(i) Prove that !(log sec x) = !<- log cos x) = tan x
d d
d
( l.. 1.·) Show tnat ' (- 1 og cosec x) = d(log sin x)= cot x
dx dx
d
(iii) Prove that � {log(cosec x - cot x)}
dX
d
= {-lo�(cosec x+cot x)) = d(log tanf)= cosec x
dx . dx
l+sin x
*(iv) Show that sec x + tan x = cos x
_ (cos x/2+sin x/2) 2 _ l+tan x/2 x TI
= tan( + )
- cos2x/2 - sin2x/2 - 1-tan x/2 2 4
hen ce prove ti1at
ci d X TI
{log(sec x+tanx)} = log tan( + )} = sec x
dx dx! 2 4
9. If y = tanx-cotx-3x, prove that�= (tanx-cotx)2 +1, and
hen ce that y increases as x increases in the domain
0 < X < TI/2,
d n-1 n+l
10. Show that dx(sec x tann x) = sec x{ntan x+(n+l)tan x}
Use this result to show that if y = sec x, then
210
APPLICATIONS OF DERIVATIVES
d�
ft=
dx
sec x(1+2tan2 x), �= s ec x(Stanx + 6tan 3x),

sec x(5+28tan2 x+24tan 4 x)


�=
dx
J. APPLICATIONS OF THESE DERIVATIVES
EXAMPLE. Determine the stationary points on the
curve y = sec x + 313 cosec x, where O < x < 211.
Distin uish between these.
Nethod.
--- Since y = sec x + 313 co sec x,
.QY. -- s ec x tan x - 3/3 cosec x cot�
• • dx
d2y
Al so�= { sec x , sec2 x + tan x . sec x tan x}
x
-3/3{ co sec x , -cosec2 x + cot x , (-cosec x cot x)}
= sec x(sec2 x+tan2 x)+3/i co sec x(cosec2 x+cot2 x)
For stationary points, .QY. = O, i.e. s ecx tanx = 3,/J cosecx cotx
• dX
c s
Hence� 3/3 � � , i.e. sin 3 x = 3,/J cos 3 x
cos x s in x
On dividing by cos 3x(;IO), .•• tan 3 x = 31), whence tanx
Thu s for O < x < 211, x = 3, 3.
11 411

Now for x = }• y = s ec
} + 3/3 co sec = 2 + 31) x i
= 8 i
411 411 411
3, y= sec 3 + 3>'3 co sec 3 = -2 + 31) X � -8
x = r.:
TT
i.e. the stationary points are (3, 8), ( 4;. -8)

3, W 3,
11 4_:y TT 11
FW'ther, when x = > 0 since s ec co sec
3 are both
po sitive. Hence <f, 8) i s a relative minimum turning point
and when x = 3,
411 fi . 411
, cosec 411 are both
W4 < O, since sec
3 3
negative. Hence ( ;, -8) is a relative maximum turning
point.

EXERCISES SET 41
'Questions 7,8,9 are intended to be done later for revision,!
1. (i) Solve the equation cosec2 x = 2 for O � x � 211,
(ii) Find the gradient of the tangent to the curve
x-1
y = 3 log x - Se at x = 1,
(iii) Use the results of (i) to find the coordinates of
the point s on y = cot x at which the tangent is
parallel to the tangent stated in (ii).
2. (i) Show that the equation 3 sec x tan x = 2 leads to
the equation 3 sin x = 2 co s2 x, and hence find x
in the domain O � x � 211.
(ii) Use the results of Ci) to determine the coordinates
of the point s on y = 3 sec x at which the tangent
is inclined at an angle tan-12 to the x axis .
211
SET 4!
(iii) Find the equation of.the tangents a t the se points
to the curve and show tha t the length in tercepted
4
on ·the y axis by these tangents is (41:f+ ;) units.

+64cosec 90 prove tha t .!!Y z7s!n 9 6�c�s 9

*
3. If = 27 sec 9
y = -
dS cos 9 s in 9
d 4
S-how that !EL = 0 when tan 9= -.
d&
d3
acute, find y (when = O) • without the use of
!!b�e!�
4, If y = -co tx-4x+11, where O<x<11 •
show t hat ·
.!!Y. = (cosecx-2)( c osecx+2 .
dx � j
(a) (i) Hence find the s tationary I
--r-I

points of t he curve. 'I


(ii) From the sketch of I
I
I

y = cosec x shown, state 'I


I

the dom ain of v alues of x I


I
for whi c h' co sec x-2 > o. I
I
Thus state the values of I

x 0 in the in terval O<x<11, 0 b , X ff

for which the function y -co tx-4x+1r i s


(a) rising (B) falling.
(iii) Deduce the na ture of the stationary poin ts of (i).
d2y
(b) (i) Fur ther, prove t hat p = -2cose c 2 x cotx, and show
X
d2 �
that � = 0 for only one
dx
11
value of x· (namely ) in
2
0 < X < 11.
(ii) For what values of x i s
(a) cot x > 0 (B) co t x<O�-o+-�-"'<--�-+�_.
l(

{Use sketch shown}. ltence


state the values of x for
which y = -co tx-4x+11 i s
concave up or concave
down. -�
(iii) Is the point where x =fan inflexion on
y = -cotx-4x+11?
*(c) _Sketch the curve y = -co tx-4x+11 for O < x < 11.
tanx .!!Y_ t anx
5. If y = e , show = ethat .sec2 x,
dx
B= .sec2 x(l+tanx) 2•
tanx
e
dx
(i) Prove that there are no stationary poin ts on the
curve.
(ii) Prove�=fi 0 when x 3n, bf.. 1. f O !: X <- 2 11.
4 4
However, show that neither value of x gives a point
of inflexi0n on t he curve. y = e tan x.
6. For f(x) = t anx+3cotx, find f' (x), f"{x). Show that
212
SET 41
stationary points occur when tan 2 x = 3, and hence find x
in the domain O < x < 11. Determine the nature of these
stationary points.
RR7. (i) For the interval O � x � 211, determine the nature
of the stationary po ints on the curve y=2secx-tan�
(ii) For what ·values of x, where O�x�211, is the curve
represented 0y y = tanx-2x *(a) stationary
(b) rising (c) concave up.
If y = log (sec x )· - x2 +3x+e g , -'11 < x < 11 , f' nd iY., �
lo 2
2 2 1. dx dx2 ·
Determine the absciss2e of the points on the curve where
= O, and find whether these are points of inf lexion.
dx2
RR9. Ti1e curvature l, of a curve y = f(x) is given by the
d2y/dx2
fonnula K = Ca lculate the curvatui;e of the
{1 + <f) l3/1
l

curve y = log(cosec x) at the point where x lL



y p
10. A variable interval P() passes through
the point (27,8) and has its ends
P,Q o n Oy, Ox respective l y.
Find, without tables, an ex-
pression fo r the minimum length o f
rpQ'.
{Hint: Proceed as fol lows. Let *PQO=S
(a variab le) and show that
*RP = 27secS, *QR = 8cosecS,
and thus *QP (.e. say)= 8co secS+27secS. O+--z7-+
. dl
Find d8 and w·
d2 .f.
Sho� that
dl
dS
= 0 when tans= 1, and
3
since S is acute, then �sf;;:,,, O. Without tab les, prove
secs=� and find a similar expressio n for cosec s.
Eence show that the minimum length of rpq' is 13 % units.}
*11. The prob lem in question 10 is rewo rded in many ways; one
of the most common of these being : "Find the length of
the shortest ladder which can lean Y
a gainst a vertical wa ll, rest on
the horizontal ground and just
pass over a wall b high and situ­ X
ated a from the wa ll ."
Use the method of question 10 to show that for the
shortest ladder , tans (�)�3• and thence that its
1
length is (a 3+b %)3/z
2

�- FURTHER TRIGONOMETRIC INTEGRALS

Integration of trigonometric functions is a rather


complex matter. Some integrals can be determined directly
213
SUMMARY OF TRIG. INTEGRALS
from derivatives; others can be found by using the
functions so far treated in this course (namely, the log­
arithmic and exponential functions, and the trig9nornetric
and inverse trigonometric functions); others employ the
results for sin2x, cos2x, sin3x, cos3x; whereas many
further integrals cannot be found by the means so far at
our command.
The purpose of this section is to systematise the
most important results, and indicate the type of exercise
which can be treated at this stage.

'!1:i.E!!__l.
Integrals Determined from the Derivatives
W-d�(sinx) =cosx, and Jcosx dx = sinx+C
d
(2) (cosx) =-sinx, and fsinx dx = -cosx+C
dx
d
(tanx) =sec 2 x, and fsec 2 x dx = tanx+c·
(3) dx

(4)
d
dx
(cosecx)=-cosecx cotx, and.·. fcosecx =cotx-cosec
dx
x + C
(S)
d
d/
secx) =secx tanx, and , fsecx tanx dx secx +C

!(cotx) =-cosec 2 x, and fcosec 2 x dx = -cotx + C


(6)
d

Hr:Mever. al though we can differentiate each of the trig­


onometric func tions with ease, the integrals of these trigon­
ometric functions are not so obvious.
'DiE!!.....!· Inte rals o the Six Tri onometric Functions
�--1.be integrals Jsinx dx, cosx dx may be written down
easily.
(1) fsinx dx -cosx+C since
d
!(-cosx) = sinx

(2) [cosx dx = sinx+C, since -9.(sinx) = cosx


dx
The remaining integrals Jtanx dx, f cosecx dx, fsecx dx,
Jcotx dx cannot be determined using simply trigonometric
func tions. These particular integrals require the use of the
logarithmic function. They have already been derived in
Set 4ii no. 8 • and are summarised here.

(Note
I
*(3) tanx dx =
I
secx tanx
sec x dx = log(secx)+C or -log(cosx) + C
(a) sec x tan x is the derivative of the denominator
i.e. of sec x.
(B) The results log(sec x), -log(cos x) are identical,
214
SUMMARY OF TRIG. INTEGRALS
1
s ince log(sccx)=log(� �) = log 1 - log co sx=-log cosx
.cos ec x(��=ec x - cot x) dx
4) Jcos ec dx = J
cosec x - cot x
*( x

-co s ec x cot x + cosec2 x


J dx
cos ec x - cot x
log(cos ecx-cot x)+C or -log(co secx+cotx)+C
or log(tan x/2) +c
{Note (a) (-cos ecx cotx + co sec2 x) is the derivative of
(cosec x - cot x).
(B) The results log(cosecx-cotx), -log(cosecx +cotx),
log(tars-) are identical for
X

(cosecx+cotx )(cosecx-cotx)
cosecx-cotx
(cos ecx + cotx )
1 �
cosecx + cotx
=
(cos ec x + cot x )

and also cosecx-cotx


sinx
=�
1-cosx = -"-"-'=--=�-
2sin x/2
2
=----���- - tan x/2
2 sin x/2 cos x/2 -
Pence log(co secx-cotx)= log(cosecx+cotx)-l = -log(co secx +cotx)
= log(tan x /2)}

s ecx(s ecx+tanx) tanx + sec2 x


*(5) Jsecx dx = / dx = fsecx dx
secx + tanx secx + tanx
= log(s ec x + tan x )+C or log tan( + ) + C
X 1T
2 4
{/'.lote (a) (s ecx tanx+sec2x) is the derivative of (s ecx+tanx)
l+sinx x
(B) secx+tanx = tan(- + - ); See Set 4H no 8}
11
co sx 2 4

J cotx dx c sx
fs ? dx = log(sinx)+C or -log(cosecx)+C
inx
*(6) =

{iiote (a) co s x is the derivative of sin x.


1
(B) log(sin x) = log( ) = -log(co sec x)}.
co s ec x

]'Yee 3. Integrals of tiie Squares of t;ie


Trigonometric Funct�ons

The integral s Jsec2x dx, f co sec2x dx may be written down


easily, since �(tanx) =!(cotx) = -cosec2 x,
s ec2x,
d
However, the others: Jsin2x dx,Jcos2x dx,Jtan2 x dx, fcot2 x dx
are not s o obvious .
(i ) To integrate sin2x and cos2 x, we must employ the cos 2x
res ults ; namely that co s 2x = l-2sin2 x and cos2x=2cos2 x-1.
From the s e, it follows that sin2 x = \(l-co s 2x) and that
cos2x = \(l+cos2x).
_ sin2x + C
T;1us I sin2 x dx = \I(l-cos2x)dx = \[x 2 1
215
SlJ,1MARY OF TRIG. INTEGRALS
and fcos = �I(l+co s2x)dx = �[ x +
si

Zx
) + c

f
2
x
3(sinxf,co s x
{Note that si n� x dx; �x , since < � )=
3 3

d!
si si x

)
This mistake is a very ao11111on error, = si n x
2
cos x.
and should be guarded against. }

(i i) To i ntegrate tan 2 x and cot x , we must employ the i dent­ 2

i tie s sec x = l+tan x and cos ec x= l+cot x :re spectively.


2 2 2 2

Thus f tan 2
x dx = J< s ec 2
x -l)dx = tan x - x + C

and tot 2
x dx = f(cosec 2x -l)dx = -cot x - x + C

�of the More Important Trigonometric Integrals


I. J sin x dx = -cos X + C
2. Jcos dx = sin x + C
s ecx tanx
x

*3. Jtanx dx = dx = log(sec x) + C


f sec x or -log(cos x) + C
'*4. f cosecx dx co sec x Ccosecx-cotx)
dx
f cosec x - cot x
=

= log(co sec x -cotx)+C


or -log(cosecx+cotx)+C or log(tan2X)+C
*5. f sec X dx = secx( s ecx+tan x)
dx
f sec x+tan x
= log( sec x+tanx )+C or log tan(i + + C !>
c
*6. f cot x dx = s�inx dx = log(s in x) + C
f or -log(cosec x ) + C
sx

7. J sin 2 x d x = f\( 1-cos2x )dx = '2Cx _ sin2x) + C


2
8. Jcos 2 x dx = J.i,,cl+co�2x)dx = l,(x + sin2
2 ) + C
x

9. Itan 2 x dx = J(sec 2 x - I )dx = tan x - x + C


10. Icosec 2
x dx = -cot x + C
11. I sec 2 x dx = tan x + C
12. Jcot 2
x dx = fcco ec x-lldx
s 2 = -cot x -x + C

13. f.sec x tan x dx = sec x + C


14. Jcosec x cot X dx = -cosec x + C

216
SUMMARY OF TRIG. INTEGRALS

J�
Other Important Integrals
l
15. fd
-
x
= log
. x+C·, a lso
ax+b
= _ log( ax+b)+C
X a

16. Jex d x = e +C;


x
also fe
ax
d x =} e
ax
+ c

17. (f/l��2 ,;,-lx+C oc -ccs-1 x+C

I
a

dx sin -i�+ C or - cos-1 � + C


� a
=

I� J� -
a

18 • - tan-lx+C·' also l tan-1 � + C


I +x"' - a "' +>< "' - a a

Note The results of a I I these need not be I e arnt by


he art; however, the method of determining al I of
the non-b asic integr als should be thoroughly under­
stood.

EXAMPLE I. Write down the indefinite integrals of


(i) sec5 x tan5x (ii) cosec x 0 cot x0 (i i i) sec l x/2
Hence ev alu ate

J J
45
fs 3
(a) sec5x t an5x d x (B) f cosecx0cotx 0dx (y) se�f dx
0 30 -11/3

:'.1et7'.od. (i)
5 secSx + C
secSx tanSx dx = 1
Jcosecx cotx dx = f cosec �� cot !TX dx,
0 0
(ii) O 180
since x0 = ; xc
1 0
-180
-- cosec + C
ltX
180
[Check this!],
TT
_180 +
and this may be written as -TT- cosec x 0 C
tan x/Z + x+
(iii) J sec2 �
2 dx � C or 2 tan 2 C

I
TT
1 2!..
Thus (a) -[sec Sx]l5
secSx tanSx dx
is 5 0
0 1 [se TT - secO
5 �
1
] = 5[2-1] =
1
s
45
-180 45
(�) J cosecx0 cotx0 d� --[cosec x0 ]
TT 30
-180(
30 -- cosec45 • - cosec30° ]
-180(
fi-2]
TT
217
EXAMPLES ON TRIG. INTEGRALS
11

f11 x3 • 11 -11 ]
3 Ti

sec 2 !. dx = 2[tan ] 2[tan - tan(� )


( y) 2-11 6
/3
2
-3 2[";i + 7*1 = 7! since tan(-e)�-tane

EXAMPLE 2. Write down the primitive function in


each of the fol lowing
(a) fsin 2 3x dx (b) fcot2 2x dx *Cc) f cot� dx
2 11 *Cd) fcosec2x dx
Hence evaluate (il J-g-sin 3x dx 2

11 11
l 0 �
(ii) f cot4 2x dx *(iii)f 2 cot { dx *(iv) f "Ii" cosec2x dx
11
6
311
" 11
6
Method.
wT,in2 3x dx = l\( 1 -cos6x)dx• using sin2 u = \(l-cos2u)
2[ x
_ sin6x] + C
6

(b) fcot" 2x dx = -fcot2x


ccosec 2x-l)dx, using cosec u = l+cot u
_
2

-cot2x
2 2

.. x + C , since ..J!c ) cosec2 1x


2 dx 2

f 3 dx = fsin
*(c) cot cos x/3
x/3
dx = log(sin
d
1/3
x/3)
+ c,
1
using (log sinu) • cosu = cotu
du "'sinu
3 log(sin 3) + C

f
*( d) cosec2x dx
_
fcosec2x(cosec2x-cot2x) dx
cosec2x-cot2x
• log(cosec2x-cot2x) + C
2
{Note: it is vital to remember the methods of tackling
(a)
these integrals, but not the finaZ results.
(8) there are alternative results possible;
for example,�log(cosec2x-cot2x) = \log( 1;��;:x)
, log( 2sin2 x
.. >, log(tamq}
2sinx COSX)='2
2 11

(i) f:rsin 2 3x dx • \[x

• 811+3/3
� - �, since sin 411 =
-
13
72 3 2
21 8
EXAMPLES ON DERIVATIVES
11/4
(ii) I 4cot2 2x
11
d x = [�cot2x··x]
11
11/6
l 11
11
• {-�cot-::- -
2
11
4
noting
11
3
11
-}-{-�c o t":" - - }
6
6 { 0 -4}-{- �,73-6}, 11 11 l
cot .. o, cot =- n
2 3
213-11 on simplifying.
E -----iz-

311
2
*(iii) J cot { d x = 3[log(sin 3)/3111 4 =l-3[log(sin
})-log(sir{>]
111 2

311 1
-,. = 3[l og(l )-lo g(72)]= 3[0-{log l - �l og 2})
= log 2.
2
3

*(iv) f 411 cose c2x


xd = �[l og( cose c2x - c ot2x)]
11/4

11 11/6
11 11 11) }
�[log( ose - c t 2 )-log(coseCj-COt:"j
2
11
c c

2
o

l
6

�[l og(l-O)-l og( /3 - 73)],on substitution


\l og3
{A simpler meth o d is to use �log(tanx)
instead of \log( cose c2x - co t2x)}

,
EXAMPLE 3. (il Find �ecosec�x and hence evaluate
f 2 ecosec2x cosec2x cot x dx
11

(ii) 4 Show that - d r1sin-l <-


cotx ] -cosec2x _
11

dXL -l
2 = IS-cosec2x

and hence evaluate I 11.!.


4 cosec x
v'5-cosec
2 x
2 dx

*<iii l Find the derivative of x tan x and hence


6

find the value off! x sec2 x dx

d
cose c 2 x co se c x
)=e , (2 cose c x.- cose c x c o t x)
Method.
(') �( e
---

x
2

cose c 2 x
"'-2e • c ose c2 x co t x
1

11 11 1 2
·
Hen ce
2 efcose c2 x
cose c x cot x
2 dx ,. - 2 Le
_I, COSe
r c2 xJ
n/ 4
.!. = - �[e-e 2 J • \(e 2 -e)
4
l 2x
d[ -1 co tx ] I co tx)2. -cose c2 x = -/4cosec
(ii) Now sin ( ) = { 2x
dx 2 l-(
2
2 -cot
219
SET 4J
-cosec 2 x
= 2 x· l+cot 2 x
it5-cosec2x'. using cosec
cosec x x 111 4
Thus f 1+.'-cosec
5 xd
2
2 -[sin-1(\cotx)J
=-[sin-l(�.l)-sin-l(!a./3)]
Tl Tl 1 6

-[ 11- J11 = 11 ,noting sin-(\)


1 = 11 etc
6 3 6 6
"* (iii) Now d: (x tan x) x sec 2 x + tan x
Thus x sec 2 x · tan x
!<x tan x) -
d
z

i.e. Jx sec 2 x dx fd!<x tan x)dx- ftan x dx


x tan x - log sec x + C
Hence Jfx sec 2 x dx [4Tl tan 4Tl - log sec 4JTl-[Otan0-logse�O l
0
[f. 1- log./z]-(0-0] = f- !alog 2
EXERCISES SET 4J
1. Find the indefinite integral in each of the following

(i) Jsin7x dx; Jcos f dx; f sec9x'tan9x dx; Jcose� cot} dx;

Jsec 2 10 x dx; Jcosec 2 t dx.


(ii) f sin 2 5x dx; fcos 2
} dx; J tan2 3x dx; J. cot 2 4x dx

(
* iii) Jtan} dx; Jcot3x dx; f cosec8x dx; Jsec f dx
(iv) fsec x tan x dx; f sec 3x dx; Jsin (ax+b)dx;
0 0 2 0 2

f (sin x+cos x) dx
2 2

(v)
fc:: x; fs��zx; fs!�zx; fco�c x; L�2x
z z

2
t(vi) Jsec(1-x)dx;
2 Jcos2 � dx; Jcot 2 (3x+4 )dx;
x+l x+l- dx
cose.c - - cot - 2
I 2

2. Evaluate the following definite integrals.(Leave answers


in terms of irrationals, where necessary.)

f4sinx dx; f 2cosx dx; f 3secx tanx dx; f4cosecx cotx


Tl

(i) dx
Tl Tl Tl

-11/4 2!, 0 2!,


11 6 11 11 °
2cosec2 x dx; l2sec3x tan3x dx;
(ii) f6sec 2 x dx; f f
-11/6 f O r:os �- dx
220 0
SET 4J
.TT TT TT TT
(iii) f sin2e de; f \in2 z x dx ;
2
fcos2x dx ; f2cos2 � dx;
TT /4 2
-TT/ 3
TT TT
0 0
TT
(iv) f4tan dx; ftan2 2x dx ;
dx;
f2cot�x dx;
ff
2
co t 3x
x 2

11/3
TT
- /8 11/ 12
0

11
(v) f (sinx +sin2x) dx; Jfc2 cos2 a+3sin 3 2d
3a)da J (1-aosx) x
2

11/6

f f f4
0
TT
0
· 11 11 11
*(vi) frse ce de; 2cosec x dx; 3 cot x dx; tan X dx

0 11 I 3 11/6 0

3-sin x
. ow th at
sin2 x
3. (i) Sh

Hence evaluate

3+Zsinx
(ii) Pro ve that = 3 se c2 x + 2 sec x tan x;
cos x
i.

1-;en ce evalua te
(k
3+2sinx
cos x
0
2 dx

4. Noting that sin x cos may be writ ten as


(sin x) 1cos x 01' ( cos 1sin x 01' �sin 2x ,
x
· hence
(i)
x)

show that f sinx cosx dx = �(sin x) +c1 2

01' -�( cos x)2+c2 01' -\ co s2x+c3, By expressing each


of the primitive functions in terms of sLn x ,
2

derive a relation aetween c2 and Cp and C3 and Cf,


(ii) Using (i), evaluate
TT
(a) p·(sin x + -cos x)2 dx (h) [ sin x(3 cosx+4) dx
TT/ 6

Find !{log(tanx+l)} and hen ce obtain the value of


d
5, (i)

f se c x dx
f
2

tan x + 1
0 TT
Find �(sec 4 x), and hence evaluate J1'sec"x tanx dx
(ii)
d

0
( i) Differentiate e and then ce find the va1ue of
cotJx
6.
11
6 cotJx
J e co se c 3x dx
2

11/i2

(ii) Using the result for


differential the coefficient
11
of co s- I (tan x ), evaluate
(if sec x
J ll-tan-2 x dx
2

221
APPLICATIONS OF INTEGRATION
7, (a) \·/rite down the formula for sin 3A in terms of sin A,
Hence show that sin 3A=l,i [3sinA-sin3A),
and use this to evaluate 2 sin 3 x dx

(b) Show that :(-cosx + t


cos 3x) = sin 3 x,
f.!.
d 11 O
Use this result to verify the value 2
J sin 3x dx,
obtained in (a) for
0
Show that :(tan x) = 3sec x-3sec x,
d
8. 3 4 2
(i)
Use this result to evaluate

Prove that
sin 2ecos 2 e=�sin 2 2e =�(1-cos4e),
J8sin 2 ecos 2 e de
(ii) 311

and hence determine the value of

Show ti1at
11/8
9.
2cos�c 4 x dx
(i)
_i(-cotx _ l cot 3x)=cosec 4x, and find f
11

dx 3

.!.Prove that d[xtanx+logcosx]=xsec 2 x; thus evaluate


11/4
(ii) dx
f 4 x sec 2 x dx

1 1 l cosx
0

Prove that -- -- = - -- x - = cosec 2 x -cosecx cotx


10. l+cosx l+cosx 1-cosx
1
= l,sec 2 x/2·
=
2cos2x/2
dx
t;ence show that I --- = cosec x - cot x + C
l+cos� or tan x/2 + c, and using
each result, evaluate 2 (l+cosx)- dx,
f-I
I

sine
11. (a) Show that _i (sece-tane+e) =
11 3

de l+sine•
and hence evaluate i+ sine d e
11

f l+sine
* (b) Prove that
x
lir.: -1 2
(i) �:{�
0

X -+- 1X
00
J cos t dt}=J..; and (ii) r sin 2 bt, dt} =l2,

where b is a constant.
0 0

JI.PPL I CAT I 0NS 0F INTEGRALS OF TR IGONOMETR IC FUNCTIONS

�XAMPLE I. The gradient of the tangent at the point


(x,y) on the curve y = f(x) is given by
.sti.
_
e
x x
an +
I + e -2x + _ I_
dx - s cs t 3 9+x2 1+7x'
If this curve

222
APPLICATIONS OF INTEGRATION
passes through the point (0,1), find the equation of
the curve.
Method.
x x 1 -2x 1
Here y ( (se� tars- + g+x2 + e + l ? ) dx
+ x
i.e. y
J
3 sec1" + t tan-It - �:,e
-2x
+ {1og(l+7x) + C
Since (0,1) lieson the curve,
1 = 3sec0 + t tan-I(c) \e 0 + t log(l) + C,
l
and 1 = 3-1e+C, i.e •. C
2
Hence, the equation of the curve is
-2 1
Y = 3 sec � + .l tan- I � - J..e :x. + ] 1og (1+ 7X ) 2
3 3 3 l.
J
-

EXAMPLE 2. S�etch the curve y = 3-sinx for the in­


terval O 5 x � TI. Determine the magnitude of the
area bounded by the curve y = 3-sinx, the x axis,
and the ordinates at x = 0, x = TI, This area is
rotated about the x axis; find the volume of the
sol id so obtained,
Nath.ad. The sketch of y 3-sinx is shown.
TI
Area required= f (3-sinx) dx= [3x+�osx]
TI

0 1
0
[3TI+cos TI] - [o+cosO]
(3TI-2) sq.units,
since cosJT=-1, "
Volume of solid required y
lT

TI TI
f f
3
TI(3-sinx) 2 dx= TI (9-6sinx+sin 2 x) dx
0. rr 0
rr J (9-6sinx+\(l�cos2x)}dx, 0
using sin 2 ::c=\(l -cos2x) ·1
IT
[ 0
9',X
1 sin2x TI
+ 6 COS X - J, • -- -j Q
2
·2.
-l
-'i
T
TI{[ � I + 6 cos TI - \sin2TI]-[o+6cos0-0] TI( - 12)
l l'.lTI
2
cu.units
EXAMPLE 3. The acceleration of a partic-le is given
by dv cot 2 v. If it is at rest after 3 sec. find
dt
TI
at what time the particle is trave I I ing at ttt /sec
in-. direction
· dt
"'·..'etn' o-�1• S 1.nce = 1 = ...1....,,.....
cot £ v tan· 2 v '
dv dv/dt
• •• t f tan 2 v dv = J<sea 2 v-l)dv=tanv-v+C (L)

�:ow when t = 3, v ·J, • •. 3 = o-o+c, i.e. c = 3.


223
SET 4K
Hence from (L), t = tan v - v + 3
When v = - \ t= 1 - 2!. + 3 (4 - 2!.)
4 4 4,
i.e. after (4 - {)sec, the particle is travelling at { ft/sec
in ->

EXERCISES SET 4K
11
1. (i)If = cosec x cot x - cosec2 x, and when x= y
.2Y
dx 11
2, 2.
find y when x= .
4
(ii)If .2Y = sec 2 x + sec x tan x, and y 1 when x= O, find
dx
y when x
3'
t2.(i)If f'(x) sin 2 x+3cos2 x+ sinx cosx, express f(x) in
tenns of x, and given f(O) = 1, find f(11/4).
11
(ii)It is known that Dy= tan 2 x+cot2 x, and find y,when x = -
3
if y = 2 when x= 11/4.

3. The gradient at the point (x,y) on the curve y= f(x) is


given by
(i)
.2Y= 1 + 1 . If the curve passes through
dx cos2x sin2x
the point (11/4,0), find the equation of the curve.
_ cos2 x+cosx
*(ii) .QY_
d.x
- sin2x
If tne
. curve cuts the x axis
where x= 11/2, find where it cuts the lirie x =11/6.
(iii) f'(x) = � + �2 + sin(x-1). 3
1 If f(l)= �,
find f(x).
4. The velocity in ft/sec of a particle travelling in a
straight line x'Ox, and whose displacement is x ft from
the fixed point O after t sec. is given by
1 1 11
({� ) v= t + - + 77 + cos t. If its displacement is
t t
2 ft after 1 second, find
2
-:;;- its displacement after
2 seconds.
3 2 1
({i) v - _ _ + - + sec2t tan2 t, Initially,
� - 1+2t 4+t2 �
it is at the origin O; find x in terms of t.
(iii) __J_(',v2) = secx tanx, and v= 4 when x = O. Find v
dx 11
when x = .
dv
3 ev
(iv) =
dt l+eVtan2v·
v
Show that t = J<e- +tan 2 v)'dv= -e- +tanv-v+c.
v

Initially the particle is at rest; find C.

5. The sketch shows part of the curves y = sinx, y = cos x.


224
SET 4K
(i) Find the coordinates -0f P.
(ii) Determine tl1e magnitude of
the area bounded by ·the.
(a) x axis and ti1e arcsOP,
PQ.
(b} y axis and the arcs·OP,
RP.
(iii) Calculate the volume of the Q X
solid cib tained when the lfll.
areas of (ii) are rotated about the x axis.
(iv) Find the angle between ·the .tangents to the .curves
at P.
6. Sketch th� curve y = l�sinx for O � x � 2n. Calculate
the area bounded by y = l+sinx, the x axis and the ord­
.ina tes at x = 0, x = 2rr. This area is rotated about the
x axis; find the volume of the solid so generated.
3
7. The curve y = .sinx+cosx for O S x S � cuts the axes at
P, Q. If O is the origin, determine the magnitude of the
(i) area bounded by O r P, , roq' and the arc PQ.
(ii) volt.one generated when this area is rotated about
the x axis.
8. Sketch the c urve y = a sin 2 x for -rr !:: x � rr and prove
that the area of the region bounded by the cuive and the
x axis is ·na sq.units.
-TT TT
9. Sk.etch the curve y = sec x for
4 !, x !,
4. Find the
magnitude of the area bounded by y sec x, the x axis

4, 4.
-TT TT
and the ordinates· at x x. = [P.int: Show that
d
{log(sec x + tan x)} sec x). This ar.ea is rotated
dx
about the x axis; find the volume of the solid thus
generated.
d
10. (i) Show that [log(sec2x)) = 2 tan 2x,
dx
(ii) Sketch the curve y = tan2x for X in- the domain

S'
t•
1f
0 .$ x s; Find the area bounded by ·y = tan2x, the
x axis. and the ordinates at x =O, x = using the
result · of (i) at some stage in the working.
(iii) The area of (ii) is ro.tated about the x axis,
through 2TT �adians. What is the volume of th�
solid io generated,

225
CHAPTER 5

FURTHER POLYNOMIALS

=A. PRELIMINARY l�OTES


l• The basic idea of a polynomial has been met already in
Fonn 5, in relation to the discussion of the quadr>atic
polynomial ax 2 +bx+c, wherie a F o.
(i) In the discussion then, the coefficients a, b, c werie
riestricted to be rieal nwnbers, but in most cases, were
in fact, integers.
"t
Thus, nx 2 - x + (5/:3 - 1) is a quadratic polynomial
with real coefficients n, - f, (5/:3-1), but these types
of polynomials did not enter into our discussions. We
concentrated our attention on quadratic polynomials
such as 2x 2 -3x+4, where the coefficients 2, -3, 4 are
integral.
(ii) Further, for the polynomial ax 2 + bx + c, the x was
referred to as the indeterminate. (The word indetermin­
ate. means "not settled or fixed in detail, Le. in­
definite"). This sounds rather a strange term to apply,
and in fact is, without knowing something further about
polynomials.
[To try to understand this, we must accept an idea that
is basic to more advanced discussions of polynomials.
This basic idea
defines the polynomial ax 2 +bx+c as the ordered triad
(a,b,c) with certain rules involving addition and
multiplication to be obeyed. Note that the order
(a,b,c) is vital - this triad represents the polynomial
ax 2 +bx+c whereas the triad (b,a,c) represents the poly­
nomial bx 2 +ax+c.
Now in this definition, the "x" is brought in as a
place-marker. In fact we use two place-markers:
"+x 2 ", "+x". As a notation, we write
(a,b,c) � (+x 2 )a(+x)b+c
ax 2 +bx+c.
226
DEFINITION OF TERMS
The place-markers are used simply to separate the co­
efficients and to keep these in the correct order.
Thus, in this approach, no ctatement is made (Jhout the
"x"; it is left widefined. Hence, the reason for the
statement that in the quadratic polynomial ax1 +bx+c,
the x is the indeterminate.]
(iii) The degree of the polynomial ax2 +bx+c is the numerical
value of the greatest index, or exponent. In this case,
it is 2.

2. Associated wi.th the quadratic polynomial ax 2 +bx+c Cl1'8


the
(i) quadratic fwiction y = ax 2+bx+c {also written as
P (x) = ax i +bx+c}
and the
(ii) quadratic equation ax 2 +bx+c = O.

(i) In the quadratic polynomial function (or more simply,


the quadratic function) y = ax2 +bx+c, we treat the x as
a variable, with a domain to be specified. In our work,
we specify this domain to be the domain of real numbers;
i.e. we may select any real number for x.
It is obvious then, that y = ax 2 +bx+c represents a
fwiction, since, corresponding to each real number we
care to select for x, then there is one and only one
corresponding real number. y. 1�·a.x
1+h+c
Graphically, the function
y = ax 2 +bx+c is repre­
sented by a parabola,
with axis parallel to the
y axis.
Note that the graph of <i<o
y = ax 2 +bx+c is continuous everywhere.
(ii) In the quadratic polynomial equation (or more simply,
the quadratic equation), ax2 +bx+c = O, we are primarily
interested in the solution set, i.e. witr. the roots.
Here we regard the values of x a1 fixed values to be
determined.
We know, of course, from our previous work that al­
though the coefficients a, b, c may be real, the roots
of the quadratic equation a:r: 2 +bx+c = 0 are not necess-
arily real.
Thus, for the quadratic equation x2 -4x+3 = O, the roots
are real (namely x = 1, 3) but for the quadratic equa­
tion x2 -4x+5 z O, the roots are unreal (or complex).
[l:ote: for x2 -4x+5 .. O, b 2 -4ac < O.]

227
FUNCTIONS AND EQUATIONS
1, Further, there are direct relationships between the
polynomial a:xl +bx+c, the pol'tfnomial. function
y = a:x:2 +bx+c, -and the polynomial equaticrn ax 2+bx+c = 0.
These are:
(i) If the quadr_an(.· equation ax 2 +bx+c = 0 has r'Oots x = a
and x = a, then the corresponding quadratic polynomial
ax 2 +bx+c has ,zeros a and a; and vice versa ..
J.'hus, the quadratic equation x 2 -Sx+6 = 0 has roots x = 2
and x = 3. The quadratic polynomial x 2 -Sx+6 {s said
then to have zeros 2 and 3, · [�ote we do not say" that
the polynomial x 2 ,-Sx+6 has zeros x = 2 and x =·
.3. This
would be incorrectly" stated, because in the polynomial
x 2 -Sx+6 the x· is· indeterminate, i.e. undefined.]

An alternative method of stating this is as follows:


If ?(x) = ax 2 +bx+c, where the domain of x is the set of
real numbers, then the equation a·x2 +bx+c = 0 may be
written in the form P(x) = 0.
Now those values of x such that P(x) = 0 are the roots
of the equation P(x) = 0, and are the zeros of the
polynomial ax 2 +bx+c. Thus, if a real number a- is such
that P(a) = O, then x = a is a root of the equation
ax 2 +bx+c = 0, and a is a zer-o of the polynomial
ax 2 +bx+c.
For example, 4(3) 2 -7(3)-15 = O, and thus x = 3 is a root
of the equation 4x 2 -7x-15 = o, and hence 3 is a zero of
the polynomial 4x2 -7x-15.

( ii) The quadratic equation ax 2 +bx+c = O, i.e. P(x) = 0 may


have 2 real and distinct roots (when b 2 -4ac > O) or
2 real and equ.al roots (when b 2 -4ac = 0), or may not
have real roots (when b 2 -4ac < O).
These roots, where they exist, are the abscissae of the
points of intersection of the parabola y = ax 2 +bx+c
{i.e. y = P(x)) and the line y = 0 {i.e. the x axis}.

y I
!j=Qx +.8,,c c,o.>o
a

)(

2 real and distinct 2 real and equal


roots, roots, no rea-Z roots
X = a X = f3 X = Y, X = y

228
DEF! NI TI ONS
y y y

!Jc ax\.t-lx-+ C, 0. <'.O


2 real and distinct 2 real and equal
roots, roots,
x = a, x = B X = Y, X = y no real roots

!!.• The work ahead on polynomials in general, follows


the pattern set out above for the quadratic polynomial.

B. DEFltllTIONS
A polyn omial of degree n is an expression of the
form Poxn + P1Xn-l + P2Xn-2 + · • · · · + Pn, where Po F o.
Notes.
(i) T'he coefficients Po, p 1, p 2, • • • , Pn are restricted to
the set of real numbers, but, as with the quadratic
polynomial ax1 +bx+c, these coefficients will usually be
integers.
Thus Sx 7 - Bl3x 4 + nx 3 - * x - 9 is a polynomial with
real coefficients (5, -813, n, -
1
!,
-9). However, we
shall concentrate our attention on polynomials such as
2x 3 -7x 2+9x-l where the coefficients (2,-7,9,-1) are in­
tegral.

(ii) Each exponent n,n-1,n-2,•.• m"Ust be a non-negative in­


teger, i.e. n,n-l,n-2,••. are either positive integers
or zero. Further, the degree of the polynomial
p 00 + p 12'1-l + ••• + Pn, where Po F o, is n. {Note
that "n" is the largest of the exponentsJ
Thus, the polynomial Sx 3 -7x-2 is of degree 3, whereas
the polynomial 9-llx 2 -8x 4+3x l 7 is of degree 17. Poly­
nomials such as 7 (i.e. 7x 0 ), -4 (i.e. -4x 0 ) etc, have
� degree. In such cases, the polynomial is often re­
ferred to as a constant polynomial.
If all the coefficients Po,P1,P2,•••,Pn are zero, then

229
FURTHER DEFINITION OF TERMS
the poly n omial reduces to O. I n this case, the polyn o­
mial is called the zero or null polynomial and has no
degree.
Ho,.,ever, expression s such as Sx 3 - 7x% - 2 and
9 - llx2 - ..§.4 + 3x 17 are not polyn omials, sin ce the ex­
x
pon ents are not all n on-n egative integers.

( iii) As with the quadratic poly nomial ax2 + bx +.�,. also


with the ge neral polyn omial p0xn+p1xn-l+p 2 xn - +,,, .+p n,
the x is referred to as the indeterm{��te.

Associated with the polyn omial p0xn+p1xn-1 +p 2 xn -2+ .•+pr


are the
( i) pol!;nomial__[u_nctio_n y p 0xn+pixn-1+p 2 xn-2+•••+pn, more
often written as P(x) P0Xn+p1x n-1+•••+pn ,
and the
( ii) 0
n n -2
(i) I n the polynomial function y=p0x +p1xn-1+p2x +.. •+pn '
the domain of x is specified 'to be t:ze set of real
nwnbers. Hen ce, for any real n umber we care to select
for x, we obtain one and only one correspo ndin g real
n umber for y.
. cer t ain
Thus y = p 0xn+p1xn -1+ ..•+p is · ly
n
a function.
The graphical representation of the polynomial fun ction
y = P(x) depends o n the degree of the polyn omial, but
in all cases, the curve representing y = P(x) is con­
tinuous everyzi/wre. {These ideas will be discussed
later.}

( ii) For �he polyn omial equation


n-2
PoXn + P1Xn-l + P2X + .•. + Pn = O,
i.e. P(x) = O, · our primary interest is in the roots;
their derivation (if possible), approximations to them
(where possible), .a n d relations betwee n them.
As :Jith the quadratic polynomial, the general poly­
nomial p 0 x11 + p 1 :xfl-1 + p2x11-2 + . . . + Pn need not have
real mots, although the coefficients Po,Pi,P2,, •• , ,Pn
are themr,elves real.

Again, there are direct relationships between the poly-


. l p xn+p xn-1+p xn-2 +.•.+p ,
nomia the polynomia I' •
· l ,unction
0 1 2 n
P(x) = p0 xn+p1xn-1+•.•+p ' and the polynomial equation
n
n
PoX +p1 x·n -1+ •••+pn = O, i.e. P(x) =O.
(a) The roots_of the equation P(x) =O are those values of
x for which P(x) = 0. These values of x are the zer>os
230
SET 5A
n n-1
of t�e polynomial PoX + , •• + P ·
+ P1X
n
For example, 2(-3) 3 +5(-3) 2
-7(-3)-12 = 0. Thus x = -3
is a root of the polynomial equation 2x 3+sx 2 -7x-12 = O,
and hence -3 is a zero of the polynomial 2x 3 +sx 2 -7x-12 •
.J1 n-1 +p xn-2+ ... +p = O has at most
(3) The equation p 0x·+p 1x 2 n
n real and distinot roots. However, i f n is even, there
may be no real roots, whilst if n is odd, there is at
least one real root. {The reasons for these stateme nts
will be made clear i n a later section.}
The roots of the equation P(x) = O, where they exist,
are the abscissae of the poin ts of intersection of the
graph representin g the polynomial function y = P(x) an d
the x axis (i.e. y = 0).
Possible shape Possible sha;:;e
of of
y=ax 3 +bx 2 +cx+d, y=ax 4 +bx 3+cx 2 +dx
a > O +e, a > 0

-x
3 real roots, x= a,B,y for 2 real roots, x = a,B for
ax 3 +bx2 +cx+d = 0 ax 4 +bx 3 +cx2 +dx+e = 0

EXERCISES SET 5A
1. For the polynomial p0 xn+p1xn-l+pzxn -2+.•• +p n, the con­
stant term is p , the leading term is p0xn, the degree
is n. The polynomial is called !!J.Q1:il.!:. if Po = 1.
Thus, consider the polynomial
x 3 (8x+1)+7x-11-(2x 2 -1)(4x 2 -3).
On expansion, this becomes 8x 4 +x 3 +7x-ll-(8x 4 -1ox2 +3)
= x 3 +lox 2 +7x-14, arranging the terms in the usual
order.
Hen ce, the polynomial is a manic polynomial (sin ce the
coefficient of x 3 is 1) of degree 3; the leading term is
x 3 and the constant term is -14.

For the following polynomials, state (i) the degree


(ii) the leadin g term (iii) the constant term
(iv) whether the polynomial is mon ic or not.
(a) Sx 2 -7x-l (b) 2-3x (c) X 7
(d) 4 (e) 0 (f) x 4 (x 3 -1)
(g) 4x 5 -7x 2 -4x(x 4 -2) (h) (x+l)2:.(x-l) 2

231
SET 5A
(i) 6x 4 -7x 3 -(2-3x 2 )(5-2x2 ) (j) x(x 3 -3x+4)-x(x2 +3x-4)
2. Select the polynomials from the following
l/
(a) 2x2 -sx-1+7 (b) ix 2 -Sx+7 (c) X 3 - 4x 3
x +x
2 2
(d) x-4x 3 (e) (f) 2
2 x +x
2 1
(g) -;2 + -; (h) 1 x 2-sl3x+nx 0 (i)
7
rx (j) x 3 +3
X

x+3 x+3
(k) log1 0 x (l) 2+e (m) 2x+e (n) (x-1) 4

3. If P(x) = 2x-l, Q(x) = x 2 -3, R(x) = x 2 -x+l, obtain the


polynomials
(i) P(x)+Q(x) (ii) Q(x)-R(x) (iii) P(x).Q(x)
(iv) P(x). [Q(x)+R(x)J.
t4 . Use the polynomials of question 3 to p rove the following
results
(i) P(x)+Q(x) = Q(x)+P(x) (ii) P(x).Q(x) � Q(x).P(x)
(iii) P(x)+{Q(x)+R(x)} {P(x)+Q(x)} + R(x)
(iv) P(x).{Q(x).R(x)} = {P(x).Q(x)}. R(x)
(v) P(x).{Q(x)+R(x)} = P(x).Q(x)+P(x).R(x)

S. If the quadratic polynomials ax2 +bx+c, dx2 +ex+f are


equal (i.e. identically equal), then a = d, b = e, C"'f.
(i) If 4x2 -7x-3 = a0x 2 +a1x+a2 , find the values of
ao,a1,a 2 .
(ii) The polynomial functions P(x) = (a+2b)x 2 -(a-b)x+7,
Q(x) = 3x2 -6x+7 are equal for all values of x.
Find the values of a and b.
(iii) The polynomials x 2 and ax(x-l)+bx+c represent the
same poly.nomial; find the values of a,b,c.
(iv) It is given that the p olynomial functions
y = a(x-1) 2 +b(x-2) 2 +c(x-3) 2 and y = Bx-18 are
identical functions; determine the values of a, b
and c.
t6. (i) If P(x) = A(x)Q(x)+R(x), and P(x) = x 2 +2x-7,
A(x) "' x-1, Q(x) = x+3, find R(x).
(ii) If P(x) = A(x) Q(x), find Q(x) if A(x) c x-1,
P(x) = x 2 -Sx+4.

7L For the polynomial (a-3)x 9 -(2b+S)x 4 +(8-4c), find the


values of a, b,c if the polynomial is (:l)monic of degree 9
(ii) of degree 4 (iii) of zero degree
(iv) a zero polynomial.
-4
8. Consider the exp ression Sxp -3x 2 -7. Find the value(s)
of p if the exp ression is (i) a polynomial of degree 7
(ii) a polynomial of degree 2 (iii) to be a polynomiaL

232
9. If A(x). = x2 -3x+2, B(x) • x2 -3x+4, C(x) = x-2 are poly­
nomial functions, then (i) Miu.. (ii) M& (iii) 1!.hl
C(x) B(x) C(x)
are called rational functions.
Assuming that x + 2, which of the three; (i),(ii),(iii);
represents a rational polynomial function?
10. Complete the following tables
DEGREE DEGREE DEGREE
P(x) 2x+5 1 x +4
5
7x 6 +2
Q(x) 3x-l 4-x 5 6x 4-l
P(x)+Q(x)
P(x).Q(x)

Can you make any general statement about the degree of


{P(x)+Q(x)} and of {P(x).Q(x)} in relation to the de­
grees of P(x) and Q(x)?

Note. The results of question 4 indicate a definite similar­


ity between the behaviour of polynomials and of integers, {In
this question, we used particular polynomials P(x),Q(x),R(x)�
but the results are true for all polynomials.} The table
below indicates the fact that the poli·nomia.Zs obey exaatly
the same laws as the integers.
THE NUMBER LA�IS THE POLYNOMIALS
If a,b,c, ... -are integers and If A(x), B(x), C(x), ... are
S is the set of al I integers, poiynomials and S(x) is the
then �et of al I polynomials, then
I. a + b £ S I. A(x) + B(x) £ S(x)
2, a X b £ $ 2. P(x) x B(x) £ S(xl
(The closure law for
mu Iti p I ication)
�. a + b = b + a 3. A(x)+B(x) B(x)+A(x)
(The commutative law
for addition)
4. a x b = b x a 4. A(x)xB(x) B(x)xA(x)
<The commutative law
for multiplication)
5. a+(b+cl = (a+bl+c 5. A(x)+{B(x)+C(i)}
(The associative law = {A(x)+B(x) }+C(xl
for addition)
6. ax(bxc) = (axt,)xc 6. A(xl.{B(x).C(x)}
(The associative law = L",(x) .B(x)}.C(xl
for mult.ipl icationl
7. ax(b+ct = axb + axe 7. A(x).{BC xl+C C xl}
(The distributive law of = A(xl.B(xl + A(xl.C(xl
multiplication over
addition)
233
GRAPHS OF SIMPLE POLYNOMIALS
THE NUMBER LAWS THE POLYMOMIALS
8. There exists zero (0) such 8. The zero polyn omial is 0,
that where
a+O = O+a = a A(x)+O = O+A(x) = ACx)
9. There exists un ity (I) 9. The unit polynomial is I
such that (i . e. IxO ) where
a x I = I x a = a A(x)xI = IxA(x) = A(x)
10.There exists an addi 'tive 10.For the polynomia\
n -2
i nyerse (-a) for eachl a, A(x) = a0xn+a xn - +a2x
1
such that + ..• +an ,
the additive i n verse is
-A(x)
a+(-al = (-a)+a = fr i.e.

where
A(x)+[-A(x)J=[-A(x)J+A(x)=O

For integers, there is no multiplicative inverse;


i.e. no integer a-1 exists so that for each a, axa-l=a-lxa•l.
Similarly, from question 9, we can see that in general
no polynomial {A{x)J-1 exists such that
A(x) x [A(x)J-1 = [A(x)J-1 x A(x) = 1.

GRAPHS OF SIMPLE POLYNOMIALS y


C
Case 1. y = c (where c is a constant polynomial)
Graphically, y • c represents a
straight line parallel to the 0 X

x axis.

Case 2. y = bx+c (bx + c is called a


linear polynomial)

...
Graphically, y .. bx+c represents
a straight line LJith gradient b.
The equation bx+c = 0 has one
�_;;,��...,..����
real root, given by x = -c/b.

1,
Case 3. y = ax 2+b:r:+c (ax2 +bx+c is a quadratic polynomial)
Gry:phicall y • ax 2 +bx+c represents a parabola, with
axis paraZ el to the y axis.

234
GRAPHS OF POLYNOMIALS
In the above figures, the parallel lines repr.�.s.e��
possible positions of tl:e x axis.. According to the actual
position of the axis relative to the parabola, the x axis
will cut the parabola in two points, 1 point or not at all.
Henoe, the equation ax2 +bx+c z O will then have 2, 1 or 0
real roots.

Case 4. y = a;c3+bx 2 +ox+d; (ax 3 +bx2 +cx+d is called a 01.bia


--- p oZy norrnat.) Gr>apni£._aZZy , y • ax 3 +bx2 +cx+d represents a
continuous curve whose general shape is indicated below.

The parallel lines, in the above figures, representing


possible positions of the x axis, cu·t the curve in 3,2,1
points. Thus, the equation ax 3 +bx2 +cx+d = 0 has at most
3 real and distinct roots, and at least one real root.

Case 5. y = ax 4 +bx 3+ax 2 +d:x:+e (ax 4 +bx 3 +cx2 +dx+e is called a


--- 4 3 2
qua�tia polynomial.) . Gro:phiaaZZy , y � ax +bx +cx +dx+e
represents a continuous curve whose general shape is in­
dicated below.

235
SKETCHING POLYNOMIALS
The parallel lines, representing various positions of
the x axis, cut the curve in 4,3,2,1,0 points. Thus, the
equation ax 4 +bx 3 +cx2 +dx+e = 0 has at most 4 real and
distinct roots, but may have no real ·roots.

Note that the graphs of polynomials are aontinuous


everywhere.

EXAMPLES ON SKETCHING POLYNOMIALS

EXAMPLE I. Sketch, wit�out the aid of the calculus, the


following polynomial functjons Ci) y = x3-3x2-4x
(ii) y = (2+xl(l-x)(2-xl(3-x) (iii) y = x5-2
Henae find the set of values of x for which
Ca) x3-3x2-4x > 0 Cb) (2+x)(l-xl(2-xl(3-x) .:!: 0
(c) x5 i! 2

(i) Now y = x 3 -3x2 -4x = x(x 2 -3x-4) = x(x-4}(x+l).


Thus the curve y = x 3 -3x 2 -4x cuts the x axis at the
?Oints where x = �1,0,4.
fig ( i) y fig (ii) y

-x
X

-'I
?oss·ible sketches are indicated. However>, to determine
the correct shape, take another value of x; say x = 5, whence
y = (+)(+)(+) > O and thus only the sketch in fig (i) can be
correct.
[Note that the presence of a non-repeated factor such as
(x+l) in the expression produces a point where the curve auts
across the x axis, since if x < -1 and x > -1, t/:e sign of
(x+l) and henae of y 1Jill be differ>ent. Thus, non-r>epeated
faator>s of P(x) lead to r>eal and distinat roots of the equa-
tion P(x) = O.]
Now from the sketch, we can deduce that the
(a) equation x 3 -3x2 -4x = 0 has 3 real and distinct roots
X '" -1,0,4,
(B) polynomial x 3 -3x2 -4x has 3 real
and distinct zeros -1,0,4.
(y) x 3 -3x2 -4x > O for the set of
values -1 < x < 0 or x > 4.

(ii) The curve y (2+x)(l-x)(2-x)(3-x) = Q(x) au.ts the


x axis at x -2,1,2,3 (there are no repeated factors).
Taking another value of x, say x=O, y = (2)(1)(2)(3)•12
236
SKETCHING POLYNOMIALS
Thus, fr,:-m the sketch, \.'.e can see that the
(a) equation (2+x)(l-x)(2-x)(3-x)=O y
has 4 real and distinct
roots, x = -2, 1, 2, 3.
(8) polynomial
(2+x)(l-x)(2-x)(3-x)
has 4 real and distinct
zeros, -2, 1, 2, 3.
(y) (2+x)(l-x)(2-x)(3-x) SO for the XS -2
or 1 S x � 2 or x � 3.

(iii) To sketchy= x s -2, it is


simplest to sketch y= x s
and then lower the curve
2 units.
The curve cuts the x axis
where x s s 2, i.e. where
X= S fi. )(

Now
(a) the equation x S -2 = 0 has
only 1 real root (x = S fi).
(8) the polynomial x s - 2 has
only 1 real zero, s rz.
(y) XS � 2 i.e. XS - 2 � 0 for X � S fi.

EXAMPLE 2. Sketch, without the calculus, the fol lowing


polynomial functions
(i-) y = (x+ll2Cx-3) (ii) y=(x+l)3 (x-3) (iii) y=x4 -9x2
Hence determine the set of values of x for which
(a) (x+l)2(x-3) SO Cb) (x+l)3(x-3) > O Cc) x 4-9x2 � 0
(i) The curve y= (x+l) 2 (x-3) auts the x axis at x = 3 {the
factor (x-3) i.s not repeated}, and touches the x axis
at x = -1. {Note when x < -1 and when x > -1, the
factor (x+l) 2 and thus y does not change sign. Thus
the curve touches but does not aut aaross the x axis at
X '" -1}. y
Taking another value of
X, Say X 2
0,
y = (+1)2 (-3)= -3 en­
ables us to sketch the
curve.
Now,
(a) the ejuation
(x+l) (x-3)=-0 has
a double root x • -1 and single root x • 3.
(8) the polynomial (x+l)i(x-3) has zeros 3,-1,-1 (-1 is
a double zero)
(y) (x+1)2 (x-3) � 0 for x � 3.

237
SET 58
(ii) The curve y = (x+l) 3(x-3) auts the x axis at x = 3, {the
factor (x-3) is not repeated}, and auts the x axis at
x = -1.{The factor (x+1) 3 , and thus y, changes sign for
x < -1 and x > -1. The significance of the degree 3, of
this repeated factor (x+l) 3 , is in the fact that the
curve has a point of inflexion at x = -1, This can be
2
verified by determining B·} Y
Taking ·x = 0 (say), then
y = (+1) 3 (-3) = -3, and
the sketch of the curve
is obtained. �ow
(a) the e�uation
(x+l) (x-3) = 0 has
roots x = -1,-1,-1,3;
(i.e. X = -1 is a
treble root).
( 8) the polynomial
(x+l) 3 (x-3) has zeros -1,-1,-1,3 (i.e. -1 is a
treble zero).
(y) (x+l) 3 (x-3) > 0 for x < -1 or x > 3.

(iii) Now y = x 4 -9x 2 = x 2 (x 2 -9) x 2 (x-3) (x+3).


The curve y = x 2 (x-3)(x+3) auts the x axis at x= -3, x= 3
and touahes the x axis
at X = 0,
Taking another value
of x, say x = 4, then
y = (+)(+)(+) > 0 Jnd
hence the sketch shown
is obtained. Now
(a) the equation
x 4 -9x 2 = 0 has
roots x = -3,0,0,3
(i.e. x = 0 is a
double root of the -Y
equation),
( 8) the polynomial x 4 -9x 2 has zeros -3, 0, 0, 3 (O is
a double zero of the polynomial).
(y) x 4 -9x 2 ? 0 for x � -3, x = J or x? 3,

EXERCISES SET 5B
1, Without the aia of the calculus, sketch the polynomial
function y = P(x) in each of the following cases. State
the (i) real roots of the equation P(x) = O, (ii) real
zeros of the polynomial P(x), and (iii) the set of
values of x for which P(x) � J.
(a) P(x) 2x-3 (b) P(x) = (x-l)(x+2) (c) P(x)=x 2 +3
(d) P(x) (x-1)(x-2)(x-4) (e) P (x)=(x+l)(x-2)(x-3)(x-4)
(f) P(x) x 3-x (g) P(x) = x 3-x 2 (h) P(x) = x 4 -x 2
(i) P(x) x4 -x 3 (j) P(x) = x 4 -x (k) P(x)=(x-1) 2 (x-4)

238
SET 5B
(l) P(x) (x-1) (x-4)
3
(m) P(x) (x-1)2 (x-4) 2
(n) P(x) (x-1) 3 (o) P(x) (x-1) I+

2. Verify that 6x-S-x 2 (x-1)(S-x) •. Eenoe, if Q(x)=6x-S-x2


find the (i) roots of Q(x) = O (ii) zeros of Q(x)
(iii) set of values of x for which Q(x) So.
3. Show that the polynomial equation x t+ +l = o
· h.:is no real
roots, and hence that the polynomial x i+ + 1 has no real
zeros. Sketch the graph of y = x 4 +1.
4. Prove by direct multiplication that
A(x) = x 3 +3x2 -x-3 = (x-l)(x+l)(x+3) and hence sketch the
graph of y = A(x). For what set of values is A(x) > O?
5. Verify that x 4 -31 = .(x-3)(x+3)(x2 +9). Hence find
(i) the real zeros of the polynomial x 4 -81
(ii) the real roots of the eQuation x 4 -81 � 0.
For what values of x is x 4 � 81?
6. ( i) Show that
x 4 -14x2 +45 (x 2 -9) (x 2 -S) = (x-3)(x+3) (x-/s)(x+/5)
Hence prove that the polynomial x 4 -14x 2 +45 has 4
real zeros.
(ii) Simila�ly , prove that
(a) x 4 -Sx2 +4 has 4 integral zeros
(b) x 4 +x 2 -6 has only 2 real ze,os
(c) x 4 +Sx 2 +4 has no real zeros.

7. By letting v = x 2 -3x, prove that the factors of the


polynomial P(x) = (x 2 -3x) 2 -2(x 2 -3x)-8 are
(x-4)(x+l)(x-1)(x-2). Hence find th e (i) real roots of
the equation P(x) = 0 (ii) real zeros of the polynomial
P(x). Sketch the graph of y = P(x), and thus obtain the
set of values f�r which P(x) � O.
e. What are the conditions for the quadratic polynomial
ax2 +bx+c to have (i) no real zeros (ii) 2 real zeros?
How many real zeros have the polynomials
(a) x 2 -6x+S (b) x2 -6x+9 (c) x 2 -6x+l0 (d) x 2 -6x-10?
Where they exist, find these zeros·.
t9. If B(x) = (x2 +x-3) 2 -(x2 +x-3)-6, find the rational
factors of B(x), and hence determine the real zeros of
the polynomial B(x).

tlO. Find the stationary points (and determine their nature)


of the following functions. (i) y 2x 3 -6x2+7
(ii) y = x 3 +3x+2 (iii) y = x 4 - 1 x 3 + 2
3
Henoe, sketch each of the curves, and find the number of
real zeros of the polynomials
(a) 2x 3 -6x2 +7 (S) x 3 +3x+2 (y) x4 - 1 x3 + 2
3

239
THE DIVISION TRANSFORMATION
D. THE REMAINDER AND FACTOR THEOREMS
SECTION I. RATIONAL FUNCTIONS
We have already seen that if A(x), B(x) are polynomials,
then ��:� is ,called a rational function, but is not
necessarily a polynomial. Thus
(i) if A(x) 2x 2 -sx-8 and B(x) = x 2 -3x+6, then
Af& 2x2 -sx-8
and this is obviously not a poly-
B(x) x2-3x+6
nomial.
(ii) If /',(x) x 2 -Sx+6 and B(x) = x-3, then
Af& _ x 2 -Sx+6 x-2)(x-3 ) =
= ( x-2, which is a poly-
B(x) - x-3 x-3
nomial (provided x f 3).
H<wever, if we consider the functions . y = Af& (x-2)(x-3)
n(x) = x-3
and y = x-2, then these are not identical, since
y = ��:� is undefined at x = 3.

y y
In other words,
graph.1.ca11y, y - Af&
B(x)'
y = x-2 represent the
same curve, except at
� = 3, when y _ _M& 0 )< 0 i 31 x
- B(x)
is undefined. I
- (x-�Xx-� I
H- X-3

SECTION 2. THE DIVISION TRANSFORMATION


(i) In Arithmetic, when an integer such as 47 is divided by
a non-greater integer such as 5, the operation may be
written as follows.
!tl 9 + , where the quotient is 9 and the remainder is 2.
2
5 5
(Of course, the remainder must be less than the divisor.)
Quotient
For convenience, this operation
9.,/
may be expressed in the form
5 -Divi-
47 "' 5 X 9 + 2 /1 dend
/ f "-. ""-.. Divisor �
dividend= divisor x quotient+ �mainder
2
This result is called the '
· der
Remai.n
Division Transformation for integers.
{Note that (a) the divisor is less than or equal to the
240
DIVISION OF POLYNOMIALS
dividend..
(b) tl!e remainde:r is lees than (but not equal to)
the divisor.
(c) for (Xl'ly such
particular division, the quo­
tient and remainder are UYfique. }
(ii) As we have already seen, polynomials obey a11· the law.s-
of· the integers, Thus, as we would expect, we hav.e · a
similar .c\ivision transformation· for polynomials.

Thus, if the polynpmial P(x) = x 2 -Sx+l7 is divided by


the poly,rromi.ai A(x) = x-3, then the quotient polynomial
Q(x) = x.-2 and the remainaer p_olynomial R(x) = 11.

This result may be written by the Division Transform­


ation in the form P(x) = A(x)Q(x)+R(x)

i.e. x 2 -sx+17 (x-3)tx-2) + 11


/ " 1
Dividend Divisor x Quotient+ Hemainder

{tlote that (a) the degree of the divisor must he not greater
than the:degree of the dividend, i.e. the de­
gree of A(x) ·is less than or equal to the
degree of P(x) .
_
(b) the degree of the remainder must be less than
the degree of the divisor, i.e. the degree of
R(x) must be less than the degree of A(x).

(c) the quotient and


remainder are unique, i.e.
there can be one and only one possible quo-;
tient, and correspondingly one and only one
possible re�ainder.}

DEFINITION. If P(xl, A(x) are polynomials such that the


degree of A(xl is less than or equal to the degree of
P(x), then
P(x) = A(x).Q(xl+R(x)
where Q(x) .is the quotient and R(x) is the remainder.
Further; (i) the degree. of R(x) is less than the degree
of A(x),
Ciil·Q<xl and R(xl are unique.

SECTION 3·. DIVISION OF ONE POLYMOMIAL BY ANOTHER.

In order to apply the d-hrision transformation to poly­


nomials, we must obviously· be ab le to actually carry out the
mechanics of the division, i.e. go through the algebra 9f de­
termining the quotient and ·the remainder.

For those students, who have not learnt the procedure or


who have forgotten, the method of long division will now be
qlustrated,
241
DIVISION OF POLYNOMIALS
EXAMPLE I. Divide 2x 2 - IIx + 13 by x - 3

2x-5
x-3 2x -llx+l3 Divide x into 2x2 , and write 2x above.
2x2 -6x Multiply 2x by x-3; then subtract.
-5x Divide x into -5x; write -5 on top line.
-5x +15 Multiply -5 by x-3; then subtract.
- 2

Hence, by the division transformation,


2x2 -llx+l3 = (x-3)(2x-5)+(-2)
Note that since the divisor (x-3) is a polynomial
of degree 1, then the remainder (-2) is a polynomial of lower
degree, i.e. O, and thus is a constant.

EXAMPLE 2. Find the quotient and remainder when the poly­


nomial 5x 3 -17x 2 -x+I I is divided by the polynomial x-2.
Express the result as an identity.

x-2 5x -17x -x+ll Divide x into 5x 3 ; write result above


5x 3 -lox 2 Multiply 5x 2 by (x-2); subtract
- 7x 2 Divide x into -7x2 ; write - 7x on top
- 7x2 +14x Multiply -7x by (x-2); subtract [line
-15x Divide x into -15x; write -15 above
-15x+30 Multiply -15 by (x-2); subtract.
-19
Hence, the quotient is Sx 2 -7x-15 and the remainder is
-19. The identity corresponding to this division is
Sx 3 -17x2 -x+ll = (x-2)(Sx 2 -7x-15)+(-19)

SYNTHETIC DIVISION
When a polynomial is divided by the polynomial (x-a),
the working above can be written more simply as follows
employing the coefficients only.
(a) The division above omit- (8) Same division as in (a),
ting x. where circled entries of
5 -7 -15
(a) are omitted. and the

1
-2 S -17 - 1 +11 arrowed entries of (a) are
raised.

i
-7 -15
(2)+14 -2 5 ·-17 - 1 +11
2-::.!Q -15 i':-10 +14 +30
@+30 IC- 7 -15 -19
-19

242
SYNTHETIC DIVISION
Same division as in (B), with -2 5 -17
the top line of (B) omitted,
30
(y}

and the circled entry of (B)


lowered to the bottom line, -1f'-19
The bottom line gives the co­ remat:ier
efficients in the quotient (which is Sx2 -7x-15) and the
remainder is -19.
[We proceed in this array by copying down S, i.e. the co­
efficient of Sx 3 ; writing down -2.(5) i.e. -10; subtract to
get -7; then write down -2.(-7) i.e. 14; subtract to obtain
-15; write down -2.(-15) i.e. 30 and subtract to get -19.J

since we multiply each 2 5 -17 -1 +11


time by -2 and subtract, why
(o) But,
10 -14 -30
not multip ly by 2 and add.
- ' 15 -19
Simply copy down 5; then 2.(5) gives 10;
/

-17+10 = -7; then 2,(-7) = -14, and remainfer


-1+(-14) = -15; then 2.(-15) = -30, and
11+(-30) = -19.
Henae, on dividing x-2 into sx 3 -17x 2 -x+ll, the quotient
is Sx 2 -7x-15 and the remainder is -19.

\
EXA�PLE I. Use synthetic division to perform the follow­
ing divisions
(i) 2x 4-sx 3-6x2 -4x+50 by x-3 (ii) 3x s -7x2 -2x-3 by x+2

I
2 -s -6 The first line gives the co­
-4 +so
efficients of the dividend
(i)
, .6 3 -9 -39 (in descending powers of x),
"2' 1 -3 -13@ First copy down the 2,
I

3X(2) = 6; -5+6 = l; 3X(l)=3;


The quotient is 2x 3+x 2 -3.x-13
and the remainder is +11. -6+3 = -3; 3X(-3) = -9;
-4+(-9) = -13; 3X(-13) = -39;
So+(-39) = 11.
Thus, by the division transformation,
2x 4 -Sx3-6x 2 -4x+SO = (x-3)(2x 3 +x 2 -3x-13)+11

(ii) -t 3 0 0 -7 -2 -3 Since the coefficients of


\1 -6 12 -24 62 _120 x 4 , x 3 are zero; we in-
sert O, 0 in the top
\...31 -6 12 -31 60 E§) line.
The quotient is 3x 4-6x 3 +12x 2 -3lx+60, and the remainder is
-123. Thus, by the division transformation,
3x 5-7x 2 -2x-3 = (x+2}(3x 4 -6x 3+12x 2 -3lx+60)+(-123)

243
SET SC
EXAMPLE 2. Use synthetic division to divide 6x -7x -9x+I
b 2x-3.
We write 2x-3 2(x - 3), and carry out the synthetic
= 2
division as if we were dividing by (x - 3 ).
2
Hence, from the division, we see that 3 2 6 -7 -9 +1
6x 3 -7x2 -9x+l = (x - 3/2)(6x2 +2x-6)+(-8)
2x-3 9 3 -9
(�2�).2(3x 2 +x-3)+(-8) 6 2 -6@
= (2x-3)(3x2 +x-3)+(-8)
Case 2. Division by a Qua<ll'atia Polynomial
There is no short-cut fo:r division by a _quadratic poly­
nomial (unless it can be factorised into real factors).

I EXAMPLE Divide 5x4 - 7x3 - 28x2- - 3 by x2 - 3x - 2.


sx2 +8x+6
x 2 -3x-2 Sx - 7x -28x - 3 Divide x 2 into Sx 4
Sx 4 -1Sx 3 -lox2 Multiply Sx2 by (x 2 -3x-2); 1
+ 8x 3-18x2 Divide x 2 into 8x 3 [subtract
+ 8x 3 -24x 2 -16x Multiply +8x by (x2 -3x-2);J
+ 6x2+16x Divide x 2 into 6x 2 [subtract
+ 6x 2 -18x-12 Multiply +6 by (x 2 -3x-2); t
[subtract

----,. Remainder.
Thus Sx 4 -7x 3-28x2 -3 = (x 2 -3x-2)(5x 2 +8x+6) + (34x+9).

EXERCISES SET SC
1. Use long (or synthetic) division, to perform each of the
following divisions, Express each result in the form
P(x) = A(x).Q(x)+R(x).
(i) 2x 2 -3x-l by x-1 (ii) Sx 2 -7x+3 by x+l
(iii) x 3 -sx2 -7x-3 by x-1 (iv) 2x 3+3x 2 -8x+4 by x+l
(v) x 3 -7x-3 by x-2 (vi) 3x 4 -sx 3 -7x-9 by x+2
(vii) 7x 4 -3x2 -182x-4 by x-3 (viii) 2x 5 -3x 3 +9x-l by x+3
(ix) 6x 3-sx 2 -8x-l by 2x+l (x) 9x 4 +2x 2 -7x-5 by 3x-l
2. Use long division to carry out the following divisions;
express each result as an identity.
(i) sx 3-7x 2 -3x+2 by x 2+1 (ii) 2x 4 -3x 3 -7x-l by x2 -3
(iii) x 4 +3x2 -Sx-l by x 2 -x+2 (iv) 3x 5 -7x 4 +2x-7 by
x2+2x-l

3. Show that x 2 +Sx+6 divides exactly into p(x) =x 3 +4x 2 +x-6,


and hence express p(x) as the product of 3 linear poly­
nomials. Find the (i) real roots of the equation p(x)=O
(ii) real zeros of the polynomial p(x). Sketch the graph
of y p(x), and state the set of values of x for which
p(x) .:! O.
244
SET 50
4. Prove that x-1 divides into x 3 +5x-6 without remainder.
Hence show that the equation x 3+5x-6 = 0 has only one
real root.
ts. Find polynomials Q(x), R(x) such that P(x)=A(x)Q(x)+R(x)
where (i) P(x) x 3 -3x2 +5x-l; A(x) = x-2
(ii) P(x) = x 4 -3x 3 -4; A(x) = x 2 +2.

FURTHER EXERCISES ON THE DIVISION TRANSFORMATION

EXAMPLE Find the quotient and remainder when


2x4 - 5x3 - 7x2 + 2x - 3 is divided by x2 + 3.
Hence, find the values of the constants a and b, if the
polynomial (2x 4 -5x 3 -7x2 +2) + (ax+b) is exactly divisible
by x 2 + 3.

Solution From the divi-


sion the quotient is 2x 2 -Sx-13, x 2 +3 2x -Sx - 7x +2x -3
and the remainder is 17x + 36. 2x 4 + 6x 2
-sxL13x 2
Thus, the division can be -sx 3
written as -15x
-13x2+17x
2x -sx -7x +2x-3 = (x +3)(2x -sx-13)
4 3 2 2 2
+(17x+36) -13x 2 -39
17x+36
On rewY'itiYl{J the division transformation, we have
(2x 4 -sx 3 -7x 2 +2x-3) - (17x+36) (x 2 +3)(2x 2 -sx-13)
i.e. (2x 4 -sx 3 -7x 2 ) + (-!Sx-39) (x 2 +3)(2x 2 -Sx-13),
collecting terms.
i.e. (2x 4 -sx 3 -7x2 +2)+(-15x-41) (x 2 +3)(2x 2 -sx-13).
Check this line!
!Jenee the polynomial (2x 4 -sx 3 -7x 2 +2) + (-lSx-41) is exactly
divisible by x 2 +3. Thus, on comparison with the polynomial
(2x 4 -sx 3-7x 2 +2) + (ax+b), we see that a = -15, b = -41.

EXERCISES SET 50
1. Find the quotient and remainder when the polynomial
2x 4+sx 3 -x 2 +3 is divided by x2 +2. Determine the values
of a b such that 2x 4 +Sx 3 -x 2 +ax+b is exactly divisible
by x� +2; _ and for these values of a, b,find the factors
of this yolynomial.
2. Find the quotient and remainder when x 4-6x 3 +x 2 -4x-4 is
divided by x 2 +x+l. Write down the identity obtained from
this division. Calculate the values of a, b such that
(x 4 -6x 3 +x 2 -4x-4)+ax+b is exactly divisible by x 2 +x+l,
3. (i) The polynomial (x 4 +x 3 -3x2 +2) - (ax+b) is exactly
divisible by x 2 +1. What are the values of a and b?
(ii) The polynomial x 4 +Sx 3+4x 2 +ax+b is exactly divis­
ible by x 2 +3x-l; find the values of a, b.
245
THE REMAINDER THEOREM
4, For what value of a does (x 2 +x+l) divide exactly into
the polynomial x 3 +5x-9-(x-a)(x+7)? Determine the other
factor,

SECTION 4. THE REMAINDER THEOREM


The division transformation for the polynomials P(x),
A(x) is usually written: P(x) = A(x)Q(x)+R(x) ........., (1)
where Q(x) is the quotient and R(x) is the- remainder. {The
degree of R(x) is less than the degree of A(x)}.
�ow. if A(x) is a linear poZynomiaZ of the form (x-a),
then R(x) must be a constant r, say (either zero or non-zero).
Th is is so since the degree of R(x) is less than the degree
of (x-a).
Hence, (1) may be written P(x) = (x-a) Q(x)+r•••••.• (2)
{Before proceeding, it should be remembered that in the poly­
nomials P(x), Q(x) etc. the x is an indeterminate, i,e, it is
undefined. However, to every polynomial P(x) corresponds the
poly1:!f1mial function P(x), provided we specify the domain of
x, i.e. the set of real numbers in our work. In· a poZynomiaZ
function we may replace the x by any reaZ nwnber, and.hence
obtain·a real number. We can thus consider equation (2) as an
identity involving polynomial functions.}
Now from (2), P(x) (x-a)Q(x)+r
Subst. x-a, .'.P(a) a (a-a) Q(a)+r
i.e. P(a) = r ••••••••••••••••••••••••••••• (3)
This result is known as the Remainder Theorem.

If the polynomial P(x) is divided by (x-a), then


the remainder on division by (x-a) is P(a).

EXAMPLE. Find, without division, the remainder when


the polynomial 2x 4-5x 3+6x 2 -7 is divided by < ii x-1
(ii) x+2
Method Let P(x) = 2x�-5xl +6xa -7
--- The remainder when P(x) is divided by
(i) (x-1) is P(l) = 2(1}"-5(1f +6(ll -7
= 2-5+6-7 = ,:i
(ii) (x+2) i.e. [x-(-2)]
� 3 2
is P(-2) = 2(-2) -5(-2) +6(-2) -7
= 2.(f6)-5.(-8)+6.(4)-7
= 32+4o+-24-7 = fil.

246
THE FACTOR THEOREM
SECTION 5. THE FACTOR THEOREM
Now if P(a) • o, then from (3), r • O
Thus (x-a) divides into P(x) and leaves a zero remainder,
i.e. (x-a) is a factor of P(x).
[This result is of course true in reverse, i.e. if (x-a) is a
factor of P(x), then P(a) • O].
The result above is known as the Factor Theorem.

If P(x) is divided by (x-a), and the remainder


P(a) = 0 then (x-a) is a factor of P(x).

EXAMPLE. Show that (x + 3) is a factor of


zx3 + 7x2 - 4x - 21 and hence find the factors of
this polynomial.

Method. Let P(x) 2x 3+7x2 -4x-21


Now P(-3) • 2(-3) 3+7(-3) 2-4(-3)-21
= -54 + 63 + 12 - 21
.. 0
Thus {x-(-3)} i.e. (x+3) is a factor of P(x).
To determine the other factor, we use long division, synthet­
ic division or intelligent inspection,
From the synthetic division process,
-3 2 7 -4 -21
we have P(x) = (x+3)(2x +x-7)
2
-6 -3 21
{Note 1, 2x 2 +x-7 cannot be readily 2 1 -7 0
factorised; in fact there are no /
Rema1 , nder
rational factors since
b 2 -4ac • (1) 2 -4x2(-7) = 57 which is not a perfect square.
We may if desired, find the irrational factors, by noting
that the equation 2x 2+x-7 = O may be solved by the quadratic
' -b±�
formula i.e. x •
2a
F-
-1 ± ./l-"'-4 - ()-- 2(7-- -)- -1 ± ill
= =
4 4
-1+/IT -1-/IT
Thus 2x2 +x-7 _: [x-(-4--) [x-(-4--)], where the underlined
2 must be inserted to ensure the coefficient of x 2 is 2.

Note 2. Such quadratic polynomials as 2x 2 +x+7 cannot be


factorised using even irrational factors, since
b2-4ac • (1) 2 -4(2)(7) = -55, which is less than zero. The
quadratic formula for the equation 2x 2+x+7 = O leads to
-l±i-55
x • Level 1 students here introduce a set of non-
4
real numbers, namely complex numbers, to state the factors,}

247
REMAINDER AND FACTOR THEOREMS
FURTHER EXAMPLES ON THE REiMINOER ANO FACTOR THEOP.rns

EXAMPLE I. The polynomial 3x3+ax2+x+b has x-1 as a


factor and leaves a remainder of -60 when divided
by x + 2. Find the va Iues of a, b.

Method.
Let P(x) = 3x3+ax2 +x+b
No1.J P(l) = 3(1) 3 +a(l) 2 +(l)+b = a+b+4,
but (x-1) is a factor of P(x), • '. P(l) = O
i.e. a+b+4 = O, i.e. a+b = -4 . ................... (1)
Also P(-2) = 3(-2) 3 +a(-2) 2 +(-2)+b = 4a+b-26,
b�t the remainder on division of P(x) by (x+2) is -60,
• • P(-2) = -60 i.e. 4a+b-26 = -60, i.e. 4a+b = -34.•(2)
Hence, on solving (1), (2), 1-1e obtain a = -10, b = 6.

EXAMPLE 2. Use the factor theorem to factorise


p(x) = 2x3-3x2 -11x+6.
f!ence ( i ) find the rea I
(a) roots of the equation p(x) = 0
(bl zeros of p(x)
(i ii sketch the graph of y = p(xl, and deter­
mine the set of values for which p(x)50.

{Note 1. The principal use of tpe factor theorem is to


factorise cubic polynomials (and sometimes quartics).
However, there are a great many �ubic (and quartic) poly­
nomials which cannot be factorised by this method.
Note 2. Since all the coefficients of p(x) are integers,
then for (x-a) to be a factor of p(x) = 2x 3 -3x 2 -llx+6, then
it can easily be seen that "a" must divide exactly into +6.
In our course (2F level), we assume that "a" is integral,
and this leads us to the conclusion that "a" is t 1, ± 2, ± 3,± 6
only. (In the level 1 course, we assume that "a" is rational
and thus "a" may be t 1/2, ! 3/2, i.e. of the form ± c/d,
where "c" divides into 6 and "d" divides into 2.)
Obviously "a" cannot be say ±4, ±5, ±7•.• since these
integers do not divide into �6 �xactly.
Hence, to factorise p(x), we try in turn (x,1), (x:;2), (x.:3),
(x;6), if necessary. In practice, we are not usually given
exercises requiring more than 3 or 4 trials.
Once a factor of p(x) has been obtained, we obtain the other
factor by long division, synthetic division or any other
method we may know.}

Method. Here p(x) = 2x 3-3x 2 -llx+6


Now p(l) = 2(1)3-3(1) 2 -11(1)+6 = 2-3-11+6 = -6 1 o,
and thus (x-1) is not a factor of p(x).
Also p(-1)= 2(-1)3-3(-1) 2 -11(-1)+6 = -2-3+11+6 1 0
248
SET 5E
p12) • 2(2) -3(2) 2 -11(2)+6 = 16-12-22+6 - 0
3

p(-2)= 2(-2f -3(-2f-11(-2)+6 • -16-12+22+6 • o,


and thus (x+2) is a factor of p(x),

Hence p(x) .. (x+2)Q(x) where Q(x) is -2 2 -3 -11 6


obtained by division, -4 14 -6
i.e. p(x) = (x+2)(2x 2 -7x+3), 2· -7 3 O
= (x+2)(2x-l)(x-3),
on factorising 2x 2 -7x+3.

(i) (a) The real roots of the equation p(x) = O


i.e. of (x+2)(2x-l)(x-3) = 0 are x = -2,�,3.
'i
(b) The real zeros of the
polynomial p(x) are
-2,\,,3.
(ii) From the sketch, p(x) � 0
for x � -2 or � � x � 3

EXEP.CIS[S SET 5E
1. Find, (without division), the remainder when the first
stated polynomial is divided by the second polynomial,
until this remainder is independent of x.
{a) 3x 2 -7x+3· x-1 (b) x 4 +2x 3+4• x+l
(c) x 5-sx+a;'x-2 (d) 2x 3-4x 2 +3; x+2
(e) x 3 -4x 2 -3x+5; x-3 (f) 4x 3+3x 2 -1; x+3
(g) 4x 9 -7x 3 -8; x-1 {h) Sx 6 -9x-2; x+l
2. Find whether each of the following polynomials has as
factor the polynomial indicated.
(a) x 3-sx 2 +4; x-1 (b) 2x 3+3x 2 -2x-1; x+l
(c) x 4 -7x 2 -18; x-3,

3. (i) Show that (x-1) is a factor of the polynomial


P(x) � x 3-6x 2 +11x-6. Hence, by long division,
synthetic division or otherwise, find the other
factor. Thenae express P(x) as the product of 3
linear polynomials.
(ii) Using the results of (1), find the
(a) real roots of the polynomial equation P(x) 0
(b) real zeros of the polynomial P(x)
(c) Set of values of x for which P(x) ?. o.
[Hint: sketch y= P(x).J
4. Prove that p(x) = x 3+2x 2 +4x+21 is exactly divisible by
(x+3). Thence find q(x) such that p{x) = (x+3).q(x),
and show that p(x) has only one real zero.
5. If (x-k), where k is integral, is a factor of the poly-
nomial P(x)= 2x 3-3x 2 -12x-7, what are the possiblP
249
SET 5E
values of k? Use the factor theorem to find k, and
thence factorise P(x) completely. Show that P(x) has a
double zero, and indicate on a sketch of y s P(x), the
roots of the equation P(x) = O.
6. Use the factor theorem to find one factor of each of the
following polynomials. Thence, find the other factor(s),
What are the rational zeros of each polynomial?
(i) x 3 +3x2 -x-3 (ii) x 3-7x-6 (iii) x 3 -x 2 +x-6
(iv) x 3 -x 2 -16x-20 (v) 3x +10 x 2 -9x-4
3

(vi) 5x 3 -16x 2 +9
t7. Find the real roots of each of the following equations
(i) x 3 +6x 2 +3x-10 0 (ii) x 3+2x 2 +5x+l0 = 0
(iii) x 3�x 2 -7x-5 = 0
8. Show that x =�is a root of the equation 2x 3 -9x 2 -8x+6=0.
Hence determine the other two roots in simplest ir­
rational form.�������������
9. (i) Find k if (x-1) is a factor of the polynomial
p(x) = x 5 -2kx 2 +2.
(ii) If 7+(x-l)(x+p) is divisible by (x+l), find p.
(iii) When- the polynomial 2x 3 -3mx 2 -7x-2_ is divided by
(x-2), the remainder is 12; find m,
(iv) The remainder when x 4 +13x+k is divided by x+3 is
20 ; find the value of k.
10 . Find the remainder when the polynomial P(x)=x 3 -3x 2 +bx+l2
is divided by x+2.
If b is chosen so that (x+2) is a factor of P(x), find
b and the other factors.
11. Determine the value of p for which x-p is a factor of
A(x) = x 3 +px2 -(2p 2 +12)x+(7p+lO), and for this value of
p, factorise the polynomial A(x).
(i) What are the (a) zeros of A(x) (b) roots of A(x)=O
(ii) Sketch the graph of y = A(x).
12, For what value of p is the polynomial 4x 3 -x+p divisible
by (2x+3)? For this value of p, find the other factor.
13, (i) When the quadratic polynomial 3x2 +ax+b is divided
by (x-2), the remainder is 4. If a+b = -3, find
the values of a, b.
(ii) The polynomial 2x 3 +ax 2 +bx+6 is exaccly divisible
by (x-1), and leaves a remainder of -12 when
divided by (x+2); find the values of a, b.
14, The quartic polynomial x 4 +ax 2 +b has factors (x+l) and
(x-2). Find the values of a and b.
15. When the quadratic polynomial ax 2 +bx+c is divided by
(x-1), by (x-2) and by (x-3), the respective remainders
are -1,4, and 11. Determine the values of a, b, c.
16. When the polynomial ax 4 +bx 3 +3x2 -2x+3 is divided by
250
DEDUCTIONS FROM THE FACTOR THEOREM
x -3x+2, the remainder is (x+l); find the values of a
1

and b. [Hint:· start ax '+ +bx 3+3x 2-2x+3=(x-J,)(x-2)Q(x)+(x+l),


where Q(x) is the quotient, and noting that the factors
of x 2 -3x+2 are (x-1)(x-2). Now substitute x = 1, x = 2 in
succession in this identity.]
17. When x 2 -4 is divided into x 5+4x 2+ax+b, the remainder is
3x+2; find a, b.

SECTION 6. DEDUCTIONS FROM THE FACTOR THEOREM

RESULT I. If the poly n omial P(x) = p0x n +p1x n-l+p2x n -2+...


.• +pn has k disti n ct zeros a1,a2,a3,•••,ak (where k < n ),
then (x-a 1)(x-a2)(x--a3)...(x-ak) is a factor of P(x).

The reasoning behind this is as follows:


Now a1 is a zero of P(�) by definition if P(a1) = O; but by

the factor theorem, if P(a1) = O, • • (x-a1) is a factor of
P(x).
Simila:l'ly, if a 2 is a zero of P(x), then P(a 2 ) = O and hence
(x-a 2 ) is a factor of P(x),
This process continues for each zero a1,a2 ,a 3 ,••,ak_and hence
(x-a1),(x-a 2 ),(x-a 3),••• ,(x-ak) are all factors of-"P(x).
Since a1,a2 ,a3 ,••• ,ak are distinct zeros, then the factors
(x-a1),(x-a 2 ),(x-a 3 ),•••,(x-ak) are all distinct. Thus the
product of these factors, namely (x-a1)(x-a 2 )(x-a 3)•• ,(x-ak),
is also a factor of P(x). [This is somewhat the same as
having distinct integers 2,3,5,7,11,••• factors of a number
X say; then 2.3.5.7.11••• is also a factor of X.]

EXAMPLE. Show that· I and -2 are both zeros of


P(x) = x '++x 3+3x 2+5x-lO, and hen ce factorise P(x).
Method. (1) '++(1)3+3(1) 2+5(1)-10 • 0
N01,, P(l)
=
(-2) '++(-2) 3+3(-2) 2+5(-2)-10
and P(-2)
=
16-8+12-10-10 = 0
Thus both 1 and -2 are zeros of P(x),
Hence since 1,-2 are distinct, ••• (x-1),(�+2) and thus
the product (x-l)(x+2) are factors of P(x).
i.e. P(x) = (x--1)(x+2)Q(x), where Q(x) may be obtained
by various techniques (synthetic division, long division
etc.)
x 2+5
+x....,..+3- x- """+-5-x-- l- -O
......--'
1 1 1 3 5 -10 -2 1 2 5 .10 x 2+x-2 x.,,........
'+
1 2 5 10 -2 0 -10 x +x 3�2x 2
1 2 5 10 0 1 0 5 O 5x2+5x-10
5x 2 +5x-10
Thus in both cases, Q(x) = x 2 +5
1
and .·. P(x) z (x-l)(x+2)(x +5)
251
DEDUCTIONS FROM THE FACTOR THEOREM
RESULT 2. If P(x) = p0xn+p1xn-1 +p2 xn -2+...+pn has ·n dis-
tinct zeros a1 ,a2 .,a3 ,•.• ,an , then (x-a1 ><x-a2><x-a3 ).•.
.. (x-an> is a factor of P(x), by result I. Since P(x)
is of degree n, then P(x) = p0 (x-a1><x-a 2 ><x-a3 ).. (x-an),
where the £.,Q is necessary to�ive the correct term in xn ,
namely p0 xn.

EXAMPLE If A(x) is a polynomial of degree 4 an d has zeros


I -I I I
, -7, 5 then A(x) = K(x - 2><x + )(x+7)(x-5), where
2, 3 3
k is a co nstan t.

llote that P (x) is not unique! If t!le po lynomial is to


(i) be monia, then the coefficient of x"" is unity, i.e. k = 1
(ii) aontain only integral aoeffiaients, then A(x) may be
2x -l -
written as A(x) = k( )( )(x+7)(x 5)
3x+l
3
= 6k(2x-1)( 3 x+l)(x+7)(x -5),
2

and by choosing k a multiple of 6, then A(x) contains


only integral coefficients.

I RESULT 3. A polynomial of degree n aannot have more than


n zeros.

This result states that a polynomial of degree n such as


P(x ) = Poxn--+p1 xn- 1+p2x·n-2 +•••+pn may have n zeros or less
than n zeros; it aannot have more than n zeros,
To illustrate this, let P(x) have n distinct zeros
a1,a 2 ,a3,•••,a ' Then by result (2),
n
P(x) = p0(x-a1)(x -a 2)(x-a3)•.••(x -a ).
n
Obviously since Po is a constant, and there are n fac­
tors of the form (x-a1),(x -a 2),•••,{x -8n) then there are n
distinct zeros at most.

That P(x) may have less than n real zeros is obvious if


we consider polynomials such as
(i) P(x) � 3(x +l)(x - 2)(x 2 +7), where P(x) is of degree 4, and
there are only 2 real zeros,
(ii) P(x) c (x 2+1)(x 2 +4)(x 2 +9), where P(x ) is of degree 6,
and there are no real zeros.
{Note P(x) = 4(x-1) 2 (x+2) 3 is a polynomial of degree 5, with
real zeros 1,1,-2,-2,-2.}

252
DEDUCTIONS FROM THE FACTOR THEOREM
RESULT 4. If P(x) vanishes tor more than n values of x,
then it vanishes tor al I x, i.e. Po= p1 = p2 = .. = Pn·O

Reasoning. n, xn-1+p xn-2+.••+pn, and assume


Let P(x)= p0x-�p1 2
that P(x) vanishes for the distinct values of x, namely
a1,a2,a3 ,.••,� and b. {There are (n+l) distinct values here.}
Now by above, P(x) = p0(x-a1)(x-a2)(x-a3).••(x-an)•
However, P(b) = 0, i . e. p 0(b-a1)(b-a2) (b-a3).•.(b-�)= 0 and
thus either Po = 0 or b= a1, b = a 2, b = a3,..•,b. = an.
By data, b is distinct from a1,a 2,a3,.•• ,an• and thus Po = O.
n-1 n-2
Hence, P(x) = P1X +p2x +...+p
n
and since a 1,a2 ,a3,••• ,an are distinct zeros of P(x),
-l

. • P(x) = p1(x-a1)(x-a2)(x-a3 )... (x-a n-1>
But again by data, P(b) = 0
. ·. p1(b-a1)(b-a2)(b-a3)..•(b-an_1 ) = O,
and thus p1 = 0 (since b 'f a1, b 'f a2, •.. , b 'f a _ .
n 1)
Proceeding in this manner, we can shm, that
Po P1 = P2 = ... = pn = o.
=

cmd thus P(x) vanishes for aZZ values of x. {P(x) is actually


n n-1 n-2
O.x +o.x +O.x + .•. + O, and this obviously is zero for
aZZ x}.

RESULT 5. A direct consequence of result 4 is as follows:

If A(x), B(xl are polynomials, each of degree n,


a�d are equal tor more than n distinct values of x,
then the coefficients of Iike powers of x are
equal; i.e. the polynomials are equal for aZZ
va Iues of x.

Reasoning.
n n-1 n-2
Let I A(x) a0x +a1 x +a2x + + a
n n-1 n-2
n
LB(x) b0x +b1x +b2x + + b
n
be two polynomials which are equal for more than n dis-
tinct values of x;
n
Now {A(x)-B(x)} = (a0-b0)x +(a1-b1)xn-1+(a2-b2)xn-2 (a -b )
+• .+ n n
i.e. {A(x)-B(x)} is a polynomial of degree n with more than n
distinct zeros. [Thus if a is a value of x for which both
A(x), B(x) are zero, then a is a zero of {A(x) - B(x)}J.
253
IDENTITY OF POLYNOMIALS
Hence, by result 4,
(a0-b0 ) • o , (a1-b1) • o, , , , , (a -b ) • O; and thus ao • b0 ,
n n
a1 • b1, a 2 • b 2 , .. • ' an • bn.

Hence, A(x),B(x) have equal coefficients for correspond­


ing powers of x, and thus A(x),B(x) are equal for all values
Of X,
{Actually, A(x) • a0xn+a1xn-1+a2 xn-2+ + an
n-1 n-2
b0xn+b1x +b 2x +
.. B(x)
i.e. A(x), B(x) are equal for all values of x,}

EXAMPLE. If the polynomials


x3; ax(x-ll(x-2l+bx(x-l)+cx+d are equal for more
than 3 values of x, find the values of the con­
stants a,b,c,d.
{Note: This question could have been worded
(i) If x 3 ; ax(x-l)(x-2)+bx(x-l)+cx+d specify the same poly­
nomial
(ii) Express x 3 in the form ax(x-l)(x-2)+bx(x-l)+cx+d,
Obviously if the two polynomials are identical, then
they are equal for more than 3 values of x.}

Method 1, The polynomial ax(x-1) (x-2)+bx(x-l)+cx+d may be


rewritten as ax 3 -3ax 2 +2ax+bx 2 -bx+cx+d
i.e. as ax 3+x 2 (b-3a)+x(2a-b+c)+d.
Since this is equal to the polynomial x 3 for more than 3
values of x, then a = 1, b-3a = O, 2a-b+c = O, d = O.
Solving these equations gives a= l, b=3, c=l, d= O,

Method 2. Since x 3 ; ax(x-l)(x-2)+bx(x-l)+cx+d are equal for


more than 3 values of x, then
x3 ax(x-l)(x-2)+bx(x-l)+cx+d for all values of .x.
This is tr>ue u.>hen :r: = O, i.e. o 3 = d, i.e. d .. o .. • • .. • (i)
and when :r: = 1, i.e. 1 3 = c. l+d , i.e. ca 1 ( ii)
and when :r: = 2, i.e. 2 3 b.2.l+c.2+d,
i.e, b = 3 ( iii)
{Can you see why we choose x = O, x = 1, x = 2?}
To determine a we may take x = 3 say,
whence 3� a.3.2.l+b.3.2+c,3+d, and then we o btain a= L
=
o� alter'Ylatively, equate coefficients of x 3 from which we
have 1 = a immediately.
254
SET 5F
EXERCISES SET 5F
1. Prove that P(x) • x 4+6x 3 +3x 2 -26x-24 has 2, -3 as dis­
tinct zeros. Hence find
(i) all the factors of P(x) (ii) the roots of P(x) ,s O
2. Find the value of k for which the polynomial
9x 4 -25x 2 +10kx-k 2 is divisible by both (x-1) and (x+2).
With this value of k, find the roots of the equation
9x 4 -25x 2 +10kx-k 2 • o.
3. Find a cubic polynomial with the zeros indicated
(i) 1,2,3 (ii) 1,-1,-3 (iii) -1,-1,7
4. (a) Find the monic polynomic of degree 4 with zeros
-2,2,1,1.
(b) Obtain a polynomial of degree 3 with integral co­
efficients having zeros 1/2, ·-1/3, 2,

5. (i) The polynomial (12-a)x 3 +(b+2)x 2 +(5c-6)x+(2-3d) has


4 zeros;- what are the values of a,b,c,d?
(ii) The polynomials 2x 2 +x+3; ax(x-l)+bx+c are equal
for 3 values of x; find a,b,c.
6. The.polynomials 4x 3 -x 2 +3, ax(x+l)(x+2)+bx(x+l)+cx+d are
equal for all values of x; determine the vaiues of
a,b,c,d.
7, If 3x 2 -16x+5 = a(x-2) (x+3)+b(x+3)(x-l)+c(x-1) (x-2) for
all values of x; find the values of the constants a,b,
c.
8. Calculate the values of p,q,r,s in order that
2x 3 +4x 2 +5x+6 and p(x-1) 3 +q(x-1) 2 +r(x-l)+s specify the
same polynomial.
t9. Express x4 +2x 3 -3x 2 -4x+3 in the form a(x 2+x) 2 +b(x 2 +x)+c,
and hence solve the equation x 4 +2x3 -3x 2 -4x+3 .s o, giving
roots in irrational form.
t 10. If the polynomials x 4 +40x-96, (x 2 -ax+l2)(x 2+2x-b) are
equal,find the values of a, b.

t· RELATIONS BETWEEN THE ROOTS AND COEFFICIENTS


OF POLYNOMIAL EQUATIONS
Case 1. Quadriatia Equations.

If a, Bare the roots of the quadratic equation

- a' a'
b C
ax 2 +bx+c = 0, a# 0, ·te
h n a+B aB =

Proof. If a, B are the roots of the equation ax 2 +bx+c = O,


then a, B are zeros of the quadratic polynomial ax 2 +bx+c,
Thus (x-a), (x-B) are factors of ax 2 +bx+c,.
i.e. ax 2 +bx+c = a(x-a)(x-B)
255
ROOTS AND COEFFICIENTS
i.e. x 2 + .£ "' x 2 -(ex+B)x+ex8, since a f O.
x + ..£
a a
Hence, since these polynomials are equal for all values
of x, b
'. ex+B = - - and exB = ,-
a a
[Note. (i) These results were derived in Form 5, by consider-
ing - �
ex =
b+ -b-�
Za , B = Za and then evaluating
ex+B, exB,
(ii) The results above and to follow, assume the roots
ex, B are real, but are still true if the roots are
complex.]

Case 2. Cubic Equations

It ex,B y are the roots of the cubic equation


ax 3 +bx�+cx+d = 0, a# O, then
� d
ex+B+y exB+By+yex = �. exBy =
a' a a

Proof. If ex,B,y are roots of the equation ax 3+bx 2 +cx+d=O,


then ex,8,y are zeros of the polynomial ax 3 +bx 2 +cx+d.
Hence ax 3+bx 2 +cx+d = a(x-a)(x-B)(x-y)
i.e. x 3 + .£
a
x 2 + .£
a
x + .£ = [x 2 -(a+B)x+aB] [x-y]
a
= x 3-(ex+B+y)x 2 +(aB+By+ya)x - aBy
aaµ +oµy+ya = .£ µ0 = - ;d
b
Thus a+B+y
- ;, a' CX y
Case 3. Quartic Equations

It a,8,y,6 are the roots of the quartic equation


ax 4 +bx 3+cx 2 +dx+e = 0, then a+B+y+o = - -,
a
d
aB+By+ya+ex6+B6+y6
a' a8y+aB6+ay6+By6 = - i
e
a8y6 =
a
[These results are often written in Sigma notation, thus
b e•
Ea = - ;, EaB = -;;-C, EexBy = - ;d , Ea8y6 = ;

Here EaB means the sum of all terms of the form aB,
etc.]

256
ROOTS AND COEFFICIENTS
Caee 4. General Reeult

If a ,a ,a ., ... ,a n are the roots of the polynbmial


1 2 3
equatio n P(x) = 0, where P(x)=p0x n+p1 x n -l +p2 x n-2+••
,,+ Pn- 1x+pn' Po #·O, then
Ea = -p1/p0, Ea a = p2/p0, Ea a a -p3/p0, ••••
i i j i j k
n
,.,,a1a2a3.. .. a n = (-l) Pn lPo·

[Note (i) Ea.a.ak denotes the sum of the products


'!, J
a1a2 a3,a1a2a10 a1.a2a5,•••,a2 a3a4,,, .where i,j,k are
always unequal and a particular result does not
occur again in a different order. TI!us, also
Ea a = a1a2+a1a3+•••+a1�+ 2 a a3+ .•• + �- l�l
i j
(ii) TI!e results above may be obtained as were the
corresponding results for the quadratic, cubic
and quartic equations.]

EXAMPLE I. If a, B, y are the roots of the equation


2x 3-5x2-3x+I = O, fi n d the values of
(i) 4a+4B +4y-7aBy
(ii) (a-l)(B-l)+(B-1)(y-l)+(a-l)(y-1)
'.iii) a -1+9-l+y-l ( iv) a2 +92+y2

Method.
Now a+B+y -- .±.22. -- 1. a B+By+ya - +c-3) -- -3 '
-
2 2' 2 2
- -(+1) - -1
a By - 2 - 2·

(i) 4a+4B+4y-7aBy 4(a+ B+y)-7a8y


4<2> 1Ct> = ll..
2 - 2 2
(ii) (a-1) (B-l)+(B-1)(y-l)+(a-1)(y-1)
(aB+By+ya)-2(a+B+y)+3, on expanding out
= ( -3 5
2> - 2 <2>+ 7
3 = - 2·

a-l+a-l +y- 1 + .l + .l =
By+ay+aB
(iii) = 1.
a B y aBy
-3/2
= -1/2 = Jc.
(a+B+y)2-2(a B+By+ya). Cheak th i s li ne!
c1>2
2
_ 2 cl>
2
lZ.
4

257
ROOTS AND COEFFICIENTS
EXAMPLE 2. If a1,a2,a 3,a4 are the roots of the equation
x 4-3x2-2x-4 O, find the values of
(i) r.a (ii) r.a a (iii) r.a a a
i i j i j k
(v) Ea .2 (vi) E(a .a .)-1
'!. '!. J

Method
( i)

( ii)

(v) Ea . 2 a12+al+a/+a4 2
'!.
(a1+a2+a 3+a4)2-2(a1a2+a1a 3+a1a4+a2a 3+a2a4+a 3a4)
(0)2 - 2(-3) = .2.·
[Cheak the identity here for a12+a22+a 32+a42 !]
1 1 1 1 1
(vi) E(a .a .)-1 = _ _ + _ _ + _ _ + _1_ + _ _ + _ _
-z. J a1a2 a1a 3 a1a4 a2a 3 aza4 a 3a4
a3a4+a2a4+a2a 3+a1a4+a1a 3+a1a2
a1a2a 3a4
-3
= -4 3
4'

EXAMPLE 3. If x = -2 is a root of the equation


3x 3+5x2+bx-10 = 0, find band hence determine the sum and
p reduct of i"he other foo roots of this equation.

Method. Since x = -2 is a root of 3x 3+5x2+bx-10 = o,


3(-2) 3+5(-2) 2+b(-2)-10 = O, which gives b = -7.
Thus the equation is 3x 3+5x2-7x-10 = Q,
Appr>oaah 1 Let the roots of this equation be -2,a,S,
Thus (-2)+a+S = -5/3 and (-2).a,S = 10/3
whence a+B = 1/3 and aS = -5/3.
i.e. the sum and product of the other two roots is 1/3
arid -5/3 respectively.
Appl'Oaah 2. One of the factors of 3x 3+5x2-7x-10 is (x+2).
Thenae 3x 3+5x2-7x-10 = (x+2)(3x2-x-5) -2 3 5 -7 -10
Thus if 3x 3+5x2-7x-10 = o, then x+2=0 -6 2 10
or 3x2-x-5 = O. The other> roots a, S 3 -1 -5 0
of the aubia equation are the roots
of quadr>atia equation 3x2-x-5 = O, whence a+S = 1/3 and
aS = -5/3.
258
SET 5G
EXAMPLE 4. Find the equation of the tangent at the point
(2,5) on the curve y = x3-2x+I, and determine the coord­
inates of the point where this tangent meets the curve
again.

Method Now y = x 3-2x+l, • •• � = 3x 2 -2


At the point P(2,5), the grad of
the tangent is 10. Hence the
equation of the tangent is
y-5 = lO(x-2) i.e. lOx-y = 15.
The point Q, the intersection of
the tangent and the curve, is
given by solving the equations
( y= lOx-15
tY = x 3-2x+l simultaneously.
Thus, on eliminating Yt we have G.
x 3 -2x+l= lOx-15 i.e. x j -12x+16= O.
Now this equation gives the abscissae.of tie points of
intersection of the tangent and the curve.
However, the abscissa of the point P must satisfy this
equation, but since the line touches the curve at P, then x=2
is a double root of the equation x 3-12x+16= O.
Thus, the roots of x 3-12x+16 = O ar.e 2,2,a (say) and
hence 2+2+a = O (since the coefficient of x 2 is 0), :i.e. a =-4.
{Alternatively 2.2.a= -16/1, whence a= -4, as before;
or since x= 2 is a double root of x 3-12x+16= O, , ', (x-2) 2
is a factor of x 3 -12x+16. Completing the factorising,
x 3-12x+16 = (x-2) 2 (x+4), etc.}
Hence, when x = -4, y= -55 (from the equation of the
tangent or the curve), i.e. the coordinates of Q are (-4,-55).

EXERCISES SET 5G
.1. If a,B are the roots· of the quadratic equation
2x 2 -3x-4= O, find the values of
(i) (ii) aB (iii) (a-2)(B-2) (iv) a-l+s-1
(v) (vi)*+� (vii� a- 2 + s- 2 (viii)�+ �a
B
1
(ix) a 2 B 3 + B 2 a 3 (x) (a + -)(B + -)
B a
(xi) (3a+4B)(3B+4a) (xii) (a-B) 2 *(xiii) a 3 +B 3
*(xiv) a 2 s-l+a-1B 2
2. If a,B,y are the roots of the cubic equation
x 3 -3x 2 +4x+2= O, find the values of
(i) a+B+y (ii) aB+By+ytt (iii) a By
( iv) (2a+1)+(2B+1)+(2y+ 1)
(v) (a-2)(B-2)+(B-2)(y-2)+(a-2)(y-2)
259
SET 5G
1 1 1
(vi) - +- +- (vii) a-1+a-1+y-l
aB By ya
(viii) (a-2)(8-2)(y-2) (ix) a2+a2 +y 2
t 3. Repeat question 2, for the equation 2x 3-sx-3 • o.
4. If a,81y,o are the roots of the quartic equation
2x 4-5x j -7x2 -1 = o, find the values of
(i) Ea (ii) i::aa (iii) Ea8y (iv) a8yo (v) i::a-1
(vi) i::a2 -
(vii) I: (aa) l (viii) E(a8y)-1

5. If X = 3 is a root of the equation x 3-19x+p o, find p.


Also find the sum and product of the other two roots of
this equation.
6. Show that 2 is a zero of the polynomial x4 -x 3 -3x-2.
If a,8,y are the other 3 zeros, find the value of
(i) a+B+y (ii) aBy
7. Two of the roots of the equation 2x 3 +3x 2 +.•• =O are
X � -3, X = 4, Find the third root and complete the
equation.
8. If x = 1, x 2 are the roots of the equation
ax 3 +bx 2 +1lx-6 O, find the values of a, b; and hence
determine the third root of this equation.
9. If (x+l) is a factor of the polynomial P(x)=x 3 +ax 2 +bx-S,
and on division by (x-1), the remainder is -14, find the
values of a, b. Hence determine the sum and product of
the zeros (not including -1) of the polynomial.

10. Determine the equation of the tangent to the curve y = x 3


at the point (1,1) on it. Hence obtain the coordinates
of the point where this tangent meets-the curve again.
11. Sketch the curve y = 12 , x IO. Find the equation of the
X
tangent to the curve at the point P (1,1) on it. Hence
calculate the coordinates of the point Q where this tan­
gent meets the curve again. In what ratio is the
interval PQ divided by the y axis?
t12. Find the equation of the tangent and of the normal to
the curve y = � at the point (2,1) on it. Calculate
x +4
the coordinates of the point at which the tangent meets
the curve again and show that the normal does not meet
the curve again.

13. Fi�d the equation of the normal at the point (2at,at 2 )


on the parabola x2 = 4ay. If this normal is to pass
through the point T (9a, Sa) prove that t 3 -6t-9 =O. Use
the factor theorem to help solve this equation, and
prove that only one real normal to the parabola passes
through the point T. What is the equation of this
260
APPROXIMATIONS TO THE ROOTS OF p(x) = 0
normal, and find where this normal meets the curve.
14. Derive the equation of the normal at the point (2t,t 2 )
on the parabola x 2 = 4y. Hence determine the equations
of the 3 normals to x 2 = 4y which pass through the point
(-12,15). If t1, t 2 , t3 are the parameters of the
points of contact of these 3 normals and the curve, find
the value of t1+t 2 +t3.
).5. P is the point (1,2) on the curve y = x 3-4x 2 +2x+3. From
P a line is drawn to touch the curve at Q. Find the co­
ordinates of Q. [Hint: let the line have equation
y-2 = m(x-1), and hence show that if the abscissa of Q
is a, then a,a,l are roots of the equation
x 3-4x 2 +(2-m)x+(l+m) = O.]
16. Find the stationary points of the curve y = x 3-3x 2 -9x+l5,
distinguishing between them. Give a rough sketch of the
curve. If P, Q are the stationary points and R is the
midpoint of 'i>q, prove that R is the point of inflex�on
of the curve. Any line through R cuts the curve again
in H, N; prove that *MR = *RN. [Hint: let the line be
y-4 = m(x-1) and the abscissae of M, N be x1, x 2 ,J

F. METHODS OF APPROXIMATING THE ROOTS


OF THE EQIJAT ION PCxl = 0
Historical Note
-b±�
The quadratic formula x = expresses the roots
2a
of any quadratic equation ax�+bx+c = 0 in terms of the co­
efficients a,b,c; and thereby guarantees, provided b 2 � 4ac,
that the roots can be always evaluated for a quadratic.
Results for polynomial equations of degree 3 (cubic) and
degree 4 {quartic) were found about 1540. From then on until
1824, mathematicians searched for formulae for equations of
degree 5,6,.... However, in that year, the famous Norwegian
mathematician, Niels Abel, (at the age of 22) proved that it
was not possible to write the roots of an equation higher
than 4 as an expression in terms of the coefficients.
In our course, we do not learn these formulae for
equations of degree 3, 4. Numerical methods are ava·ilable for
finding the roots of equations (not necessarily polynomials).
High-speed computing machines make these methods very
efficient but of course, since we are relying on our own cal­
culations, these methods can become very arduous.

Important Note. Before being able to approximate to the


root(s) of the equation P(x) = O, it is first necessary to
locate the interval(s) within which the root(s) lie. To do
this, we make use of the fundamental idea stated below:

261
LOCATING REAL ROOTS
If y = P<xl is a polynomial function such that P(a),
P{b) have different signs, then since P(x) is continuous,
there is at Zeast one root of the polynomial equation
P(x) = 0 in the interval (a,b), i.e. there is at least
one zero of P(x) in the interva I a < x < b.

P(t) P(a)

From the sketches, it can be seen that at Ieast one


root of the equation P(xl = 0 occurs between x = a, x = b
since the curve y = P(x) cuts the x axis at least once in
this interval.

EXAMPLE I. If P(x) = 2x 3 +5x-4, show that there is a real


root of the equation 2x 3+5x-4 = 0 in the interval
Q S X S I.

Method Now P(O) = -4, P(l) = 2+5-4 = 3


Thus the curve y = 2x3+5x-4 cuts )
the x axis between x 0 and = �='l.X. +5x.-4
X = 1.

Hence the equation P(x) = O,


0
i.e. 2x 3+sx-4 = O has a real
root in the interval OS xS 1,

EXAMPLE 2. Given that the equation x 3-3x 2 -9x+4 = 0 has 3


real roots, find in what intervals these roots I ie.·

MethodProbably the best method, apart from using the Calcu­


���lus to sketch the curve, is to tabulate y x3-3x 2 -9x+4
for integral values of x,

I;I I I I I I I I I
[Note
-3
-23
t
-2
2
-1
9
t
that a curve such as y
0
4
1
-7 -1�
t
-2�
x 3-3x 2-9x+4 for large
4
-16
5
9

negative values will be negative, whereas for large


positive values the expression will be positive.]

262
SET SH; HALVING THE INTERVAL
Henae the curve
y • x 3 -3x2 -9x+4 cuts
the x axis in the
intervals -3<x<-2,
0 < X < 1, 4 < X < 5
respectively.

EXERCISES SET SH
1. Show that the equation
(i) 3x 4+5x-2 = 0 has a root in the interval O < x < 1.
(ii) x 5+2x 3 -5x-l = 0 has at least one root in the
interval (1,2),
2. Prove that the equation x 3-6x+l = 0 has roots in the in­
tervals -3 < x < -2, 0 < x < 1, 2 < x < 3.
3. Find intervals within which the real roots of the
following equations lie.
(i) x 3 +5x-30 = 0 (there is 1 real root)
(ii) x 3 -4x 2 -x+8 = 0 (there are 3 real roots)
(iii) x 5-4x 2 +2 � O (there are 3 real roots)

APPROXIMATION METHOD I. HALVING THE INTERVAL

the sketches above, since P(z1),P(z2 ) are of opposite


signs, there is a root of.the polynomial equation P(x) = 0 in
the interval z1 .< x < z2 ,
We now compute the midpoint z 3 = \(z1+z2) and determine
the value of P(z 3),
�ither P(z3) = 0 or P(z3) I 0
If P(z3) = O, then x = z3 is a root of the equation
P(x)., 0,
If P(z3) IO, then there is a root of P(x) = O in the
interval (z 3 ,z2) if P(z 2 ), P(z 3) have opposite signs [See
fig (i) above], or there is a root in the interval (z1,z 3) if
P(z1), P(z 3) have opposite signs [See fig (ii) above].
263
HALVING THE INTERVAL
Assuming x = z3 is not a root, we repeat the procedure.
First compute the midpoint z4 = \(z3+z2) in fig (i), and
z4 = �(z1+z 3 ) in fig (ii), and determine the value of P(z4),
If P{z4) = O, then x = z4 is a root of P(x) = O.
lf P(z4) IO, then there is a root in either the interval
(z 3 ,z4) in fig (i) or in the interval (z1,z4) in fig (ii).
This process is repeated un.til either the root is deter­
mined or an approximation to the required degree of accuracy
is obtained.
(Note: On a computer using the binary number system,
this process can be perfotmed rapidly. In actual fact, there
is a gain of exactly one binary digit of accuracy at each
step.
However, this method is often clumsy without a.computer
because of the heavy mechanical work involved, and the number
of times the procedure must be repeated to obtain a reason­
ahly good approximation.]

EXAMPLE Show that there is a root of the equation


x3-6x+I O between 2 and 3, and find an approximation to
it by "halving the interval" three timE.s.

Method Let P(x) = x 3 -6x+l,


whence P(2) = -3 and P(3)=10.
Thus, since P(2) < 0, P(3)>0,
there is a root of the equa­
tion P(x) = 0 in the interval
(2,3).
Our first approximation is
X = \(2+3) = 2•5,
Now P(2•5)=(2·5) 3-6(2·5)+1
=15•625 - 15 + 1
= 1·625
[(2·5)3 was obtained from cube tables]
Hence x = 2•5 is not a root, but since P{2) < O, P(2•5) > o,
there is a root of P(x} = 0 in the interval (2, 2·5).

Ou� second approximation is x = \(2+2·5) = 2·25


Now P(2'25) = (2•25)3-6(2•25)+1 11•40 - 13·5o+l -l · 10
[(2·25) 3 = 11•40 was obtained using common logaritluns. Al-
729
ternatively, (2·25) 3 = (2�) 3 = c.2.) 3 = = 11•390•.•]
4 64
l/ence, x = 2·25 is not a root, but since P(2'25) < O,
P(2•5) > O, there is a root in the incerval (2·25, 2•5).

� third approximation is x = \(2·25+2·5) = 2·375


Now P(2•375) = (2,375)3-6(2•375)+1 = 13•39 - 14•25o+l 0'14
[(2·375)3 was obtained using common logarithms]

264
SET SI
Thus x • 2•375 is nQt a root, but since P(2•375) is
small (0•14), then x • 2•375 is an approximation to the root
of x 3-6x+l • O between x • 2, x = 3.

{Better approximations will occur on repeating the pro­


cedure, but naturally the mechanical work becomes heavier.}

EXERCISES SET SI
1. (i) Show that if P(x) = x 2+x-3, then P(l) < O, P(2) > 0
and that there is a root of the equation P(x) = O
in the interval (1,2). Prove that the first ap­
proximation by the "halving the interval" method
namely x = 1\ is too large, that the second ap-
proximation, namely x = \(1+1•5) = 1•25 ·is too
small, that the third approximation, namely
x = 1•375, is too large. {If we repeated the pro­
cedure, the next successive approximations would
be 1•3125, 1·28125, 1•296875,••• }
(ii) By using the quadr�tic formula on the equation
x2 +x-3 = O, prove that this root is
-1+/IT +
,;. 1·303
2
{The purpose of this exercise is to illustrate the
fact that generally speaking, the "Halving the
Interval" Method does not give good approxima­
tions without many repeated applications,}

2. Show that the real root of the equation x 3 -2 = 0 lies


between x = 1 and x z 2. Starting with the first approx­
imation z1 = �(1+2), find the approximate real root.
[Use four approximations.] {The actual root is 1•259921
to 6 decimal places; in fact, the second approximation
is closer than the third or fourth approximations.}
3, In each of the following, show P(a), P(b) have opposite
signs and use the "Halving the Interval" �!ethod (at most
3 times) to estimate the root of the equation P(x) = 0
which lies between a, b.
(i) P(x) = x 3+x-1; a=O, b=l
(ii) P(x) = x 3 -5x+3; a=l, b= 2
(iii) P(x) x 4-2; a=l, b=2 (iv) P(x)=x 5 -x-1; a= l, b = 2
t(v) P(x) = x 4 -12x+7; a= O, b=l
t(vi) P(x) x 3+x 2 -lOO; a= 4, b=5.

APPROXIMATION METHOD 2 - NEWTON'S METHOD


[Note: The following method of determining approximations to
the real root(s) of the equation P(x) = 0 is due to Sir
Isaac Newton, one of the founders of the Calculus.
Newton's method has several advantages over other ap­
proximation methods:
265
NEWTON'S METHOD
U) It applies to polynomial equations of any degree, (and
most important of all) it applies to non-polynomial
equations such as ex = 2x+l, 2 sin x - x = O,
loge x + x 2 - 4x = 0 etc.
(.!!) It gives the answer in numerical form. The method is
repeated until the required degree of accuracy is ob­
tained. (The method can be programmed into a computer.)
(iii) It gives a more accurate answer with usually fewer
steps. {However, like all approximation methods, the
arithmetical calculations become involved (unless a
calculating machine is used).}
[Newton's Method aannot be applied blindly; and will
sometimes fail if suffiaient aare is not exeraised in its
use. This will be discussed later.]

The Method.
We wish to find the real root(s) or a particular root of
the equation P(x) = O.
First, we usually sketch y= P(x) and estimate the root
x = r (say) where the curve y= P(x) crosses the x axis.
[Alternatively we may isolate the root in the interval (a,b)
by noting that P(a), P(b) have different signs.]
This estimate is our first y
approximation; call it z1.
Our seaond approximation,
z2, is the value of x where the
tangent to the curve y = P(x)
at the point Z 1 [z1,P(z1)l cuts
the x axis.
To determine the value of
z2, we note that the equation 0 }, :,.
of tangent at z 1 , is
y-P(z1) = P'(z1)[x-z 1 ), F,rit
Appro�.
This tangent cuts the x axis
where y = o, i.e. where O-P(z1) P'(z 1 )[ x-z1]
( z
p l, P(z1)
i.e. - = x-z 1 , z; z1 -
P'(z 1 ) p �j X = P'(z1)
Henae z2 = z 1 - �) •,,••, •• · · •• · • • ••••••••• · •• (1)

Our third approximation, z3, is �:Pexl


the value of x where the tangent at
the point Z 2 [z2,P(z 2 )l cuts the
x axis.

y - P(z 2 ) = P'(z2)[ x-z2l and hence z3 = z 2 - � .•••• (2)


This result (2) is the result (1), where z1, z 2 are
266
NEWTON'S METHOD
replaced by z2, z3 respectively.
{The· process can be repeated, so that the (n+l)th approxima­
tion, z + ' may be determined from the nth approximation, z n ,
. n l P(zn)
by the result z + = z - ,,,••,,,.,,,,,••,••,,•• ,, (3)
n l n �)
Such a process is called an iterutive process ("Iterative"
means "characterised by repetition", i.e. repetitious.) A
tomputing machine can be programmed to keep Tepeating the
process of result (3) automatically.

Previous P(z) New


approximation --- zn+ = zn - P'(z) --- approximation
I n
(zn) '------ ----'-'_. n I)
(z+

POINTS TO NOTE ON NEWTON'S METHOD


1. The key factor in applying this method is to obtain a
good first approximation. If this first approximation is
a good one, then it will be certainly true that the next
approximation will be a better one, and so on.
In actual fact-, the nwnber of deaimal plaaes of aaauraay
doubles with eaah suaaessive appliaation of Newton's
Method. Thus, if one approximation is good to 1 decimal
place, then the next approximation is good to 2 decimal
places, and the following· one will be good to 4 decimal
places, and so on,
2. Various factors, such as the first approximation not
being a.good one, the gradient of the curve and the
curvature of the curve, may together or individually re­
sult in the method failing. To see this, consider the
following sketches of y • P(x).
P(z1) > 0 (ii) P(z1) < 0
(P'(z1)> O lP'(z1)> O
(i)

P11 (z1)> 0 P11 (z1)> 0

(iv) (v) P(z1) > 0 (vi)


(P'(z1)< 0
P11 (z1)> 0

267
NEWTON'S APPROXIMATION
(vii) (viii) P(z1) < O
{P'(z1) < 0
P"(z1) < 0

The favour>able cases where Newton's Method will certain­


ly give a better second approximation than the first, are
where Zais between the root x = r and the first approxima­
tion x = z1. This occurs in parts (i,iv,v,viii above); and
in eaah aase note that the sign of P(z1J and P"(z1) are the
same.
In the other parts, (ii,iii,vi,vii) , the second approxi­
mation z2 may or may not be nearer to r than is the first ap­
proximation z1. In some cases, it may actually be worse.
However, in all aases, if the first approximation is
near to the true value of the root, then the seaond approxi­
mation will be nearer, and so on.
(a) (b)

Case (a) illustrates a typiaally favour>able aase, where re­


peated applications of Newton's Method leads to better
and better approximations.
Case (b) illustrates a typiaally unfavourable aase, where
because of the slope and curvature of the curve, and the
fact that z1 was not near enough to r., Here z2 is actu­
ally worse than z1; z3 will be worse than z2, and so
on.]
].. If any approximation z is really a root, then success-
ive approximations z + n' z + ' z + ,•.• all give z •
n l n Z n 3 n
P(z )
This can'be seen from z + = z - n since if z is
n l n �)· n
n
a root of P(x) = O, then P(z ) = O, and thus z + = z .
n n l n

268
EXAMPLES ON NEWTON'S APPROXIMATION
EXAMPLE I. Use Newton's Method to approximate to the root
of the equation x 3 -6x+I = 0, which I ies between 2 and 3.
Method Let P(x) x 3 -6x+l.
Now P(2) = -3 and P(3) = 10, and
thus there is a real root of
P(x) = 0 in the interval (2,3).
{Note that P'(x) = 3x 2 -6,
P"(x) = 6x.
Stationary points occur on
y = ·p(x) where 3x 2-6 = O, i.e. at -,--�f;-1--=�--l-
x = t/f, However, neither value h
lies within the interval (2,3);
in this interval,3x 2-6 = 3(x 2-2) > O, and hence the
curve y = P(x) is rising. There is a point of in­
flexion at x = O, but in the interval (2,3), P"(x) > O,
and hence the curve is concave up. The&e features are
illustrated in the sketch.}

Because of the shape of the curve, and the fact that


P(2) = -3, and P(3) = 10, it would seem that the· root is
nearer to 2 than 3.
We may take our first approximation z1 = 2 (and feel
reasonably certain that the second approximation, z2 wil'i be
nearer to the root than is z1, {Alternatively, we may take
our first approximation as any number in the interval (2,3).}
p (z1)
Newton I s formula for- z2 = z1 - --
P'(z1)
Thus z 2 = 2 - � = 2•5; i.e. the ZJ 2
second approximation is x = 2·5. P(z1) 2°-6.2+1 = -3
P'(z1) 3.2"-6 = 6

The third approximation z3 z2 2·5


is given by
P(z2) (2·5) 0-6(2·5)+1
Z3 = z 2 - �
= 15•625-15+1 = 1·625
P' (z2)
P' (z 2 ) 3(2·5} 2 -6=18·75-6=12•75
1·625
2•5
- 12•75
)
2'5 - ·1275 via common logarithms
2•3725
He would be safe in taking as our second approximation
z2 2·37 (i.e. adding one deoimaZ pZaoe to the approximation
with each appZioation of !:ei,;ton 's Yethod. This does not imply
of course, that we expect the answer to be correct to the
number of decimal places taken.)

269
EXAMPLES ON NEWTON'S METHOD
The fourth approximation z4 2•37
is given by Z3

P(z 3) (2•37) 3-6(2•37)+1


z� • zJ - �
P'(z 3) •13•30-14•22+1 • •09
, •09
" 2'37 - 10•851 P' (z3 ) 3(2•37)2-6
) •16.•851-6 • 10•851
� 2•37 - •008292 via common
logarithms
+ 2•361708
Henae our fourth approximation would be 2•362.

EXAM"LE 2. The equation zx 3 +x-8 = O has one real root.


Isolate it, and determine its value to 2 decimal places.
Let P(x) • 2x 3 +x-8, ,', P'(x) • 6x2 +1
The real root must be positive since P(x) < O for x � o.
Method

Fl'Om the tabZe. this root lies in the 0 1 2


interval 1 < x < 2, Ix I
_P(x).-8.-5.10.
1 1 1
Since both P(l}, P(2) are not near zero, the real root
is not near either x • 1 or x = 2. In such a case, it is
better to start our first approxi-
mation z1 • �(1+2) = 1•5,
[Although P(l) < P(2), the fact
that P" (x) > O for this interval
suggests that the root is near
x = 1•5; perhaps a little less
that> X = 1,5,]

P(1•5)
Thus z2 • 1·5 - P' (l•5) z1 1•5
= 1'5 - 0•25
14•5 P(z1) 2(1•5) 3+(1•5)-8
=2(3•375)+1·5-8=·25
= 1•5 - •01724 P' (z1) 6(i•5)£+1
1·48276 =6(2, 25)+1 = 14·5
Thus z2 "' 1·483.
Certainly the real root of 2x 3+x-8 = O will be x = 1•48
(correct to 2 decimal places).
{Note P(z1) = P(l•5) = 0•25. Since this is nearly zero, the
approximation z1 = 1,5 is an excellent one with which to
begin Newton's Method, Check that P(l•4) = -1•112, and hence
the real root is near x • 1•5},

270
SET SJ
EXERCISES SET SJ
1, For the quadratic equation x2+x-3 • o, show that there
are roots in the intervals (1,2) and (-3,-2) respective­
ly.

!•
(i) Starting with z l • 1, use Newton's }1ethod to prove
that z2 m and hence that z 3 = j� +
(+ 1,303 ).
7
(ii) Starting with z1 = -2, show that z2 • - and hence
76 3
• -
that Z 3"' - 33 (� -2•303 ),
(iii) Use the Quadratic Formula to show that the roots
-1±/IT
of the equation x 2+x-3 = 0 are x·= and
this gives x + 1•103 or -2•303.
2

{Tiie purpose of this exercise is to demonstrate the ac­


curacy of Newton's Method. Since the equation x 2+x-3 = O
is a quadratic, we would, of course, use the quadratic
formula to determine approximations to the roots.}

2. Use Newton's !1ethod to find the cu!:>e root of 2, i.e, to


find x·such that x = 312, i.e. x 3 2, i.e. x 3-2 = 0,
{Here P(x) = x 3-2, and since P(l) = -1, P(2) = 6, then
1 < X < 2,
4 -
Taking z1 = 1, show that z2 = and z 3 = 9l (� 1·264 ),
+ 3 72 -
[Now z2 = 1·33 is correct to 1 decimal place, z 3=1•264
is correct to 2 decimal places.)
Taking z 3 = 1•26, compute z� to 4 decimal places.
{A table of cube roots gives 3 /2 = 1•259921 to 6 decimal
places.}

3. Tiie equation x 3-3x+l = 0 has 3 real roots. Determine


within which intervals these roots lie,
(i) For the middle root, starting with z1 = O, prove
that z2 sf and that z 3 = �� + •347,
(ii) For the larger root, show that we cannot start
with z1 = 1, since P' (z1) = O. Start with z1 = 3/2,
and find z2,
4. Sketch the graph of y = P(x) for a� x � b, in each of
the following. Show that P(a), P(b) have opposite signs
and use Newton's method to estimate the root of the
equation P(x) = 0 which lies between a and b, correct to
2 decimal places. [Take z1 = '::(a+b) J ,
(i) P(x) = x 3+x-l; a=O, b=l (ii) P(x)-x 3-Sx+3; aa l,b=2
(iii) P(x) = x q-2; a=l, b=2 (iv) P(x)=x 5-x-l; a=l, b=2
t(v) P(x) = x q-12x+7; a=O, b=l
t(vi) P(x) = x 3+x2-lOO; a = 4, b = S.

271
SET SJ
rne following exercises 5,6,7 show the necessity of hav­
ing a good first approximation.
5. Show that x = 5 is an approximate root of the equation
P(x) = O, where P(x) = x "l-5x 2 -24x+l18. Prove that if
z 1 = 5, then z 2 = 7, which is ·not a closer approximation
to the root. {The actual root is 5•393 ••• }.
Now, repeating the procedure, take z1 = 5• 4 and hence
prove z 2 = 5·393. y
6. Consider the continuous function
defined by P(x) = lx-1, x � 1
= -/i=x, x< 1.
The sketch of this function is
shown.
Take z1 = 2, and prove that
1
z2 = 2 - - = o.

Taking z 2 = O, show that z 3 = 0 - ¥':?
= 2.
(In this exercise, no matter how often the method is re­
peated, we do not obtain any closer approximations to
the real root of P(x) = O, namely x = 1.)
7. Repeat exercise 6, where P(x) �rx=r, X � 1
-3 /i=x, X < 1
{In this exercise, if z1 = 2, then z2 is actually worse
than z1, z 3 is worse than z2 and so on.}

8. For the polynomial P(x) = 8x 3-16x 2 -2S, prove that P(2),


P(3) have different signs.
Taking z1 = \(2+3), show that z1,z 2 ,z 3,... are all the
same.
9. A polynomial function y = P(x) for which P(a) < O,
P(b) > 0 satisfies the following conditions in the in-
terval a ix i b:
whenever x2 > x1, then P(x2) > P(x1) and P'(x2) > P'(x1).
Sketch a graph which represents such a function.
Show that the polynomial x 3-18x+ll satisfies these con­
ditions in the domain 3 � x � 4, and find a better ap­
proximation than x = 3·9 to the root of the equation
x 3 -18x+ll = 0 in this interval.

272
CHAPTER 6

THE BINOMIAL THEOREM

A. INTRODUCTORY
A 'binomial' is an expression containing two terms. For
2
example', (x-7), (3+2x ), (Sa + t> c-l:'e binomials,
The bin omial theorem is a method of expanding positive
integral powers of binomials. Thus we can, by means of the
binomial theorem, as it is called, expand such expressions as
(l+x) 7, (3x + 7) 5 , (2x2 -3x-1) etc,
6
without very much
trouble. {The result can be extended to trinomial expansions
such as [l-2x+3x2 ) 4 , by grouping, as here [l-x(2-3x)J 4 , and
then treating the trinomial expi!Ilsion as [l - Y) 4 , where
Y = x(2-3x).}
Before proceeding to the binomial theorem proper, we
shall discuss the work and results in this connection,by a
famous French mathematician, called Blaise Pascal (1623-1662).
His name is associated with a renowned triangle, called
Pascal's tr>iangle, which enables us to fin d the coefficients
in the expansion of positive in tegral powers of (l+x) i.e.· of
(l+x) n . {Pascal was the first mathematician to systematise
these results, although according to Lancelot liogben, the
pascal triangle set was known as early as 1300 A.O. Here it
occurred in books called "The Precious Mirror of the Four
Elements" written by a Chinese mathematician, Cl1U Shi Kei.
However, Pascal did not give the general formula for the bi­
nomial coefficients in the expansion -of (l+x)n, where n is a
positive integer; this was done by the great English
mathematician, Sir Isaac Newton (1642 - 1727),}

�. DEVELOPMENT OF PASCAL'S TRIANGLE


To develop the set of coefficients in the expansion of
(l+x)n for positive integral values of n, we begin by expand­
ing (l+x)n for n = 1,2,3, ••• {Note: To make the triangle more

273
PASCAL'S TRIANGLE
complete, we shall also include the case when n � O.
VALUE NLMBER
OF i, ( l+x) n EXPANSION
OF TERMS
n= O ( l+xl 0 l I
n= I ( l+x) 1 l+lx 2 I
n=2 ( l+x) 2 1+2x+lx 2 3
n= 3 ( l+x) 3 ( 1+2x+x 2 )( l+x)
=1+3x+3x 2 +lx 3 4
n= 4 ( l+x)4 ( 1+3x+3x 2 +x 3 )(1+x)
= 1+4x+6x 2 +4x 3 +lx 4 5
n= 5 < l+xl 5 < l+xl 4 < l+x)
= 1+5x+lOx 2 +10x 3 +5x '+ +lx s 6
n= 6 ( l+x) 6 ( I+x) ( I +x)
5
=1+6x+l5x 2 +20x 3+15x '++6x 5+lx 6 7
n= 7 ( l+x) 7 6
( l+x) ( l+x)
= 1+7x+2lx2 +35x 3 +35x 4 +2lx 5 +7x 6 +lx 7 8
{Note. (l+x) n is a polyn omial of degree n in x; the indices
increase by unity as we proceed term by term from left to
right. There are (n+l) terms.}
From the above table, the coeffiaients in the various
expansions for n = 0,1,2, •.• ,7 may be written down as follows
and these arrangemen ts are called Pascal's Triangle.
Power Coefficients or as
0

1
2 2 2
3 3 3 3 3
4 4 6 4 4 6 4
5 5 10 10 5 5 10 10 5
6 l 6 15 20 15 6 6 15 20 15 6
7 l 7 21 35 35 21 7 l 7 21 35 35 21 7
Non;;s ON PASCAL'S TRIANGLE
1. Apart from the 1 at each end, any number in a row is the
sum o f the two numbers
(i) immediately above it and to the left in the first
arrangement.
(ii) immediately above it on either side in the second
arrangement.
Thus in both cases, for example, 4=1+3, 6=3+3, 4= 3+1,
6= 1+5, 15= 5+10, 20=lo+l0, 15=1o+5, 6=5+1
and 7=1+6, 21=6+15, 35=15+20, 35=2o+15, 21=15+6, 7=6+1
etc.
274
EXPANSION OF (l+x) n
2. The coefficients which are equidistant from the ends of
a row are equal in size.
3. Pascal's triangle does not give the general formula for
the coefficients of the expansion. of (l+x)n .
4. The main disadvantage of the triangle is the necessity
to complete each row before being able to proceed to
further rows. Thus to expand (l+x)9 by the Pascal tri­
angle coefficients, we must complete the rows for n = 1
to 9 inclusive. {In practice, however, some of the early
rows are usually learnt by heart, say 1 4 6 4 1, and the
triangle completed from there.}
5. For values of n from n = 1 to n = 10 (say), Pascal's
method is fairly rapid and efficient, but for larger
values of n, Newton's metho.d of writing the coefficients
is much easier. (This will be discussed later.)

USE IN EXERCISES INVOLVING EXPANSIONS OF TYPE (l+x)n

EXAMPLE I. Give the expansion of


(a) (l+x) 8 (b) (l+x)lO
by means of the Pascal triangle coefficients.

Method. To expand (l+x) n where n = 8,10 the triangle above


must be continued further, Thus for n = 8, 9, 10 the rows
become

n = 8 1 8 28 56 70 56 28 8 1
n = 9 1 - 9 36 84 126 126 84 36 9 1
n = 10 1 10 45 120 210 252 210 120 45 10 1

Hence we may write down at sight now


(l+x)1 =. 1+8x+28x 2 +56x 3 +7Dx 4 +56x 5 +28x 6 +8x 7 +x 8 , remembering
that (l+x) 8 is a polynomial of degree 8; that the indices of
x increase from term to term, by unity in proceeding from
left to right; and that t here are (8+1) = 9 terms in the ex­
pansion.
Also (l+x)l O = 1+10x+45x 2 +120x 3 +210x 4 +252x 5 +210x 6 +120x 7 +45x 8
+10x9+x l O.

EXAMPLE 2. Give the expansion of


2x 7
(bl ( I + -c;:> 2x 7
l-x) 5 (cl (I -
(a) (
3>
and state the coefficient of x 5 in each case.

!-iethod.
�t Y -x,= expansion (1-x)5 may be written
[1+(-x)J5 = (1.+ Y) S
Using the Pascal triangle coefficients,
• •. (1 + Y) 5 = 1+5Y + lOY2 + 1oy 3 + 5Y 4 + y S
275
EXAMPLES USING PASCAL'S TRIANGLE
i.e. [1+(-x)] 5 = 1+5(-x)+l0(-x)2 +10(-x) 3 +5(-x) 4 +(-x) 5
i.e. (1-x) 5 l-5x+lOx 2 -10x 3 +5x 4-x 5
Note the expansion contains alte!'Ylate positive and negative
terms, and this is the case for all binomial expansions where
the signs of the two terms are opposite.
1
(b) Let Y = �; the expansion becomes (l+Y) , and using the
2x

Pascal triangle coefficients above, we have 7


3
(l+Y)1 = 1+7Y+2 li'+35Y +35Y''+2 1Y 5 +7Y"+Y
i.e. (1 + �) 7 = 1+7( �)+2 1( ;)2 +35(2�)3 +35(2;)4 +21( �) 5
2 2 2 2

+7( )6 +( ) 7
2x 2x
2x
1+7( )+2 l( 4x 2 8x 3 3 3
)+35( )
3 9 27
16x 4 2l(3 2x 5 64x6 128 x 7
+35( )+ )+7(729 ) + 218 7
81 2 43
14x 28 x 2 28 0x3 22 l1x s
+ 560x +
4
1 + + +
3 3 27 81 81
448x 6 128x 7
+ 72 9 +
2 18 7 '
on expanding,out.

(c) Using the result noted in (a),


280x 3 56ox4
(l _ 2x) = 1 _ 14x + 28 x -27 + -- _ + 448x
22 4x
7 2 5 6

3 3 3 8 8172 9
128x 7
- 2187
22 4 -224
Coeffiaient of x in (a) is -1 (b) is
5
(c) is 8 1
81

EXAMPLE 3. Expand (1+2x-1) 6 and hence determine the


(i) coefficient of x-3 (ii) term independent of x,
in the expansion (l-3x-4x2 )( 1+2x-1) 6
Method.
(1+2x- 1) 6 = 1+6(2x-1)+15(2x-1)2 +20(2x-1) 3 +15(2x- 1) 4+6(2 x- 1) 5
+< 2 x-l )6
= 1+6 (2 x- 1)+15(4x-2 )+20(8x- 3)+15(16x-4 )
+6(32x- 5)+(64x -6 )
=1+12x- 1 +60x-2 +160x-3 +240x-4+192x- 5 +64x-6
Now (l-3x-4x2 )(1+2x-l) 6
= (l-3x-4x 2){1+12x- 1 +60x-2 +160x- 3 +240x-4+192x- 5 +64x-6}
(i) The terms involving x-3 are l.l60x- 3-3x.2 4bx-4-4x2 .192x-5
i.e. 160x- 3-720x-3 -768x- 3 i.e. -1328 x-3
Thus, the coefficient of x-3 is -1328.
(ii) The terms independent of x are l.l-3x.12x-l-4x 2 .60x-2
i.e. 1-36-240 = -275

EXAMPLE 4. Find the value of kif the coefficients


x
of x3 and x4 in the expansion of (I - Zk )9are equal.
3

2 76
APPROXIMATIONS TO (l+x) n
Method.
2 x 2 x
�x )9 l-9( � )+36( � ) 2 -84( �x)3+12 6(2�x)4-...... ,
2 2
Now (1 -
using the Pascal triangle coefficients for (l+x)9,
2k 2k
llere the coefficients of x 3 , x 4 are -84( ) 3, 1 2 6( )4 and by
data, these are equal. 3 3
2 k ,.. 2k
-84(2k)3 = 1 2 6( ) .. , 1.. e. -84 = 12 6( ) on cancelling,
3 3 -3.84 3
k /. 0, whence k = = -1.
2.12 6

EXAMPLE 5. If (1+2/3) 5 a+b/3, find th.e values of


the constants a, b.

Method.
Now(H2j3) 5 1+5(2 /3)+10( 2 /3) 2 +10(2 13) 3+5( 2 13) 4+(2 ,'3) 5
1+5 (2 >'3)+10(4. 3)+10(8.3 >'3)+5 (16. 9)+(32 . 9>'3)
841 + 538,'3, on collecting terms.
Thus a = 841, b 538.

EXAMPLE 6. Use the expansion of (1-x)lO to find


499 10
the value of correct to 8 decimal places.
c500)

t:ethod.
HeY'e (l-x) 10 l-10x+45x 2 -120x 3 + 2 10x 4 -25 2 x 5 +210x 6 - +x!O
499 2
N01,' l - __l__ = 1 - - - = 1 - •002 { = · 998}
500 500 1000
499 IO
Thus ( ) (l-·002 ) 10
500
�-10(·002 )+45(•00 2 ) 2 -1 2 0(,002)3+210(•002)4- ••,
using the result for (1-x) 10, where x = •002 .
1-0·02 + 45(,000,004) - 1 2 0(·000,000,008)
+ 2 10(•000,000,000,016)-.•••
l-·02+·000,180-·000,000,96o+,000,000,003,360-..

·000,180 -·02 l•000,180,003,360


•000,000,003,360 ·000,000,960 - • 0 2 0,000,960
l·000,180,003,360 ·02 0,000,960 ·980,179,043,360
499 IO
Hence ( ) , ·980 179 043 360
500 ·980 179 04 (correct to 8 decimal
places)
[Note. Too often in approximations, the calculations are not
taken to enough decimal places to ensure the required accur­
acy. In the example being considered, if the result is
correct to 8 decimal places, then the next term in the ex­
pansion, namely - 2 5 2 (·002 ) 5 must not affect the ninth
decimal place.
Now 252(·002 ) 5 = 2 5 2 x •000 000 000 000 032
,000 000 000 008 064,
and this obviously does not affect the result above.)
2 77
SET 6A
EXAMPLE 7. Calculate the first four terms in
ascending powers of x of ( l-2x+3x 2 J 4 , and state
the coefficient of x 3 .
Method.
Now l-2x+3x 2 l-x(2-3x) = 1-Y where Y = x(2-3x)
Thus (1-2x+3x 2 ) 4 = (1-Y) 4 = l-4Y+6Y 2 -4y 3+ y 4
l-4x(2-3x)+6x 2(2-3x) 2 -4x 3 (2-3x) 3+.•••.•.,
the other term is unnecessary here, since the result required
is of the form a+bx+cx2 +dx 3.
= l-4x(2-3x)+6x2(4-12x+.••)-4x 3(8.••)
Only terms to and including x 3 are needed.
= l-8x+l2x 2 +24x 2 -72x 3 -32x 3 •••.
= l-8x+36x 2 -104x 3 ...•
The coefficient of x 3 is -104.

EXERCISgs SET 6A
1. Make up your own Pascal's triangle set, for values of n
from n = 0 to n = 12. Keep this in a prominent place for
use in the following exercises.
2. Using the Pascal Triangle binomial coefficients, give
the expansion of the following
(a) (l+x) 4 (b) (1-x) 4 Cc) (l+y) 6 (d) (1-y) 7
(e) ( l+b) 5 (g) (l-2x) 5 (h) (1+3x) 6
8 2x 4
2 7
t(k) (1 - -)
(i) ( 1 - li)
2 tO.) (1 + )
X y
t(m) (1+3x-l) 6
3.

3 5
For the expansion of (1 - ) , determine the
2x
(i) third term (u 3 ) (ii) fifth term (u 5 ) (iii) us : u 3.
What are these values when x = 1?

5. (i) Show that the coefficient of x 3 in the expansion


of {(l+x) 3+(1+x) 4 +(1+x) 5 +(1+x)b} is equal to the
coefficient of x 4 in the expansion of (l+x)7.
(ii) Verify by direct expansion that
(l+x) 6 -(l+x) 5 = x(l+x) 5
6. (i) If (l+x) 3 = l+a1x+a 2x 2 +a 3 x\....
(l+x) 5 = l+b1x+b2xl +b 3 x 3 + •••
find the value of l+a1b1+a2 b2 +a3b3
(ii) For the expansions (l+x)7, (l+x)6 show that
(a) the sum of the coefficients of the first
expansion is double the SIJll of the coeffic­
ients of the second expansion.
278
SET 6A
(b) the ratio of the products of these coeffic­
ients is 7 6 : 6! (where 6! = 6.5.4.3.2,1)
7. (i) In the expansion of (l+x) 8 where x =�. show that
u 4 , us, u 6 are in the ratio 8: 5: 2,
(ii) Without expanding out, show that
1+7(x-1)+2l(x-1)2 +35(x-1) 3 +35(x-1) 4 +21 (x-1) S+Xx-1) G+(x-1) 7=x 7

8. Derive the expansions of


(i) (l+x)6 and (l+x- 2 )4 ; hence determine the value of
the term independent of x in the expansion
(l+x)6 ( l+x- 2 ) 4
(ii) (l+x) 3 and (1-x) 3; hence calculate the coefficient
of x 4 in the product (l+x) 3 (1-x) 3 , { Check this re­
sult, by finding this coefficient directly from
[(l+x)(l-x)J3 = (1-x 2 ) 3 },
9. If x is so small that third and higher powers may be
neglected, then (3+2x)(l+x)4 = a+bx+cx2 where a,b,c are
certain constants. Find a,b,c.
tlO. Find the coefficient of
(i) x 4 in (2-3x)(l-x) 4 (ii) x2 in (l+x)2 (1+2x)5
(iii) x in (1+2x+3x2 )(1 - l.) s (iv) x 3 in (1+2x)6 (1-x) 4
X
X 4 2 3
(v) x 0 in (1 - -) (1 + -:-z-)
-- 2 __ X -----
11. (i) If (1+/i) 5 = a+b/z, find the values of the integers
a, b.
(ii) If (1-2/3) 4 = x+y/3, where x, y are certain in­
tegers, find the values of x and y.
(iii) Without tables, find the value of
(a) (1+/z) 3+(1-fi) 3 (b) (1+,12) 3-(1-fi) 3
12. In the e?(pansion of (l+x) 7, (i) the second, third and
fourth terms form an Arithmetic Sequence; find the
possible values of x. (ii) the second, third and
fifth terms form a Geometric Sequence; find x,
13. (i) For what value of k will the coefficients of x 3
and x 4 in the expansion of (l+kx) 8 be (a) equal
(b) in the ratio 2: 3.
(ii) Show that the value of (1-x) 6 +(1+x) 6 where x = /3,
is rational and determine its value.

14. (i) Expand (l+x) 5 as far as the term in x 3. Hence de­


termine the value of (1'002) 5 correct to 6 decimal
places. {Check that the term in x 3 does not affect
the answer given.}
(ii) Expand (1-2x)6 in ascending powers of x. Hence
calculate {0·98) 6 correct to 5 places of decimals.
t 15, (i) Expand (l+x) 7-(1-x)7 in ascending powers of x,
giving the answer in its simplest form. Hence
calculate (1·1) 7-(0,9) 7 correct to 5 decimal
places.

279
EXPANSION OF (a+x)
n

(ii) Determine the expansion of (l+x) 5 + (1-x) 5 in


ascending powers of x, and hence evaluate
(1•002) 5 +(0•998) 5 correct to 6 places of decimals.

16. Write down as deoimal,s terms in the expansion of


all
(1 + � ) . Hence find correct to the nearest cent, t;1e
5
1 0
compound interest ($I) on $1000 at 2% p.a. for 5 years.
{Hint: The amount $Ato which a principal of $P grows
at r% CI over n years, interest paid annually, is given
by A= P(l' + 1�0)n }. Compare this result with the best
answer you can obtain from logarithm tables.

17. Show that (l+x) 7 = 1+7x+2lx2 +35x 3 +35x 4 +2lx 5 +7� 6 +x 7 ... (P)
(i) By substituting successively x = 1, x = -1 in (P)
prove that (a) 1+7+21+35+35+21+7+1 = 2 7
(b) l-7+21-35+35-21+7-l = 0
(ii) By differentiating both sides of (P) with respect
to x, show that
7(1+x) 6 = 7+2.21x+3.35x 2 +4.35x 3 +5.21x 4 +6.7x 5 +7x 6
and hence that 7+42+105+14o+l05+42+7 = 7.2 6 .
(iii) On integrating both sides of (P), prove that
(l+x) 8 7x 2 21x 3 35x 4 3-Sx 5 21x 6
+ C = X + + + + +
8 2 -3- -4- -5- -6-
-1 7x 7 x8
Show that C = , and hence that + +
8 -:; 8
2 8-1 =
l + l + ll + ..li + ..li + ll + l + l
-8- 2 3 4 5 6 7 8
18. (i) Expand (l-x+2x 2 ) 5 in ascending powers of x as far
as the term in x4 •
(Hint: start {1-x(l-2x)} 5 =(1-Y) 5 where Y=x(l-2x)).
(ii) In the expansion of (l+x+x 2 ) 4 , show that the first
four terms are 1+4x+1Dx 2 +16x 3, and find the co­
efficient of x 4 •
D. EXPANSIONS OF TYPE (aTx) n
In the previous section, we saw how the use of Pascal's·
Triangle enables us to give expansions of the type (l+x)n
where n is a positive integer (and n = 1 to n = 10). Here we
proceed to use the Pascal triangle sets to expand types such
as (a+x)n where n is a positive integer (n = 1 to n = 10).

EXAMPLE I. Expand Ci)· (a+x) 7 (ii) (a-x) 7


in ascending powers of x.

Metnod.
��low (a+x)7 = [a(l + �)]7 = a 7 (1 + �) 7 .
a 2 3a 4 5 6
Hence (a+x) 7 = a 7 [1 + 7(�)+21(�) +35(�) +35(�) +21(�) +7(�)
a a a a a a 7
+(�) l
a
a +7a x+2la x +35a x +35a x +2la x +7ax +x 7 ,
7 6 5 2 4 3 3 4 2 5 6

on multiplying by a 7
280
SET 6B
X 7 = a7 X 7
(ii) Similarly (a-x) = [a(l - -))
7
a (1 - -)
a
a 7-7a 6 x+21a 5 x2 -3Sa 4 x 3 +35a 3x 4-21a2 x 5
+7ax 6 -x 7
{Note The expansion of (a+x) 7 is a polynomial of degree 7 in
x, It t·ontains (7+1) = 8 terms; the powers of x increase by
unity from term to term as we proceed from left to right
whereas the powers of .a decrease similarly by unity; h()l,)­
ever, the sum of the indices in any term-in.the expansion is
7.}
EXAMPLE 2. Expand out the following
5
(i) (a+b)4 (j i) (2x - y_) (iii) (x2 +2x -1)6
3
Method.
�(a+b) 4 = a 4 +4a 3 b+6a2 b2 +4ab 3 +b 4 ,
{Note that (a) the sum of the powers in any term is 4, and
that from term to term the powers of 'a' decrease by unity
whilst the powers of b increase by 1,
(B) the coefficients are determined from Pascal's
triangle.}
5
(ii) (2x - 3)

(iii) (x 2 +2x- l)6 = (x2 ) 6 +6(x2 )5(2x- 1)+15(x2 ) 4(2x-1)2


+20(x2 ) 3 (2x-'1) 3 +15(x2 )2 (2x-1) 4+6(x2 )1(2x-1 )5+(2x- 1) 6
-2
· = x l2 +6(x lD ,2x- )+lS(x8,4x 2 )+20(x5,8x-3 ) 6
l 6

+15(x 4,16x- 4 )+6(x )(32x- )+(64x - )


x l2 +12x 9 +zOx 6 +160x 3 +240x0+192x- 3+64x- 6 -

EXERCISES SET 6B
Questions I U'.-p), 8, 9(b), 11, 14 are to be left for
revision.
1. Expand out each of the following, using the Pascal Tri­
angle to give the appropriate binomial coefficients.
(a) (x+y) 3 (b) (x-y) 3 (c) (p+q) 6 (d) (p-q)S (e) (a+b)8
(f) (a-b) 9 (g) (x + �)5 ' (h) (a - f> 4
(i) {x-sx-2) 3
(j) (2x-b) 7 (k) (3x+4y)3 t(l)(!. - 2) t(m) (x2 + .Y.)6
6
1/ 2 2 3
Hn) (x y• + 2y 3) 4 t(o) (�-1 + x )5 t(p) (a-/3) 6
3
2. (a) Without expanding out, show that
(i) x 3 +3x2 (x-5y)+3x(x-5y)2+(x-5y) 3 = (2x-5y) 3
(ii) (a+b) 4- 4(a+b) 3 .a+6(a+b)2 ,a 2 -4(a+b).a 3+a 4 = b 4
281
SET 6B
(6) Write simpler results, without multiplying out, for
the following
(i) a 6+6a 5(2b-a)+l5a 4(2b-a) 2 +20a 3 (2b-a)3+15a 2 (2b-a)4
+6a(2b-a) 5 +(2b-a) 6
4
(ii) ( 13+/2) _5( 13+12) ( .fi)+ 10( 13+.fi) 3 ( .fi) 2
5

-10( /J+/2) 2 ( .fi) 3 + 5 ( 13+12) ( .fi) 4_( .fi) 5

3. (i) In the expansion of (2x+b) 6 , the coefficients of x


and x2 are equal; find b.
(ii) If (3+12) 5 = a+b.fi, find the value of the integers
a, b.
4. (i) If u1, u 2, u 3 , •.• are the first, second, third, •••
terms in the expansion of (a + x) 6 in ascending
15
powers of x, and a = lOx, show that u 3 = u� and
2
find u4 :u 5 ,
1 10
(ii) Show that (x + 1) 6 -(x - .1) 6 = 12 [x 4 + ;?i" + J.
X X· 3
5. (i) Expand (2+x) 6 as far as the term in x 4 , and hence
find the coefficient of x4 in the expansion of
( 1-x) 2 ( 2+x) 6 •
(ii) By direct expansion, find the
(a) coefficient of x 5 in the expansion of (x-�) 8 •
(b) term independent of x in (x J -x-2 ) 5 ,
6. Find the coefficient of
(i) x 4 in (x + .l) 2 (x+1) 6 (ii) x 6 in (x+l),x-1) 5 +(x+lf(x-1) 3
(iii) x 0 in (2+3x�4x 2)(2 - 1) 3 (iv) x 0 in (l-2x 3) 2 (x-x- 2 ) 6
X

7. (i) Show that (2+.-'3) 4 +(2-.-'3) 4 = 194


and <2+13> 4 -<2-13> 4 = 11213.
(ii) Prove that (v'6+.fi) L ( 16-.fi) 3 = a .fi, and find the
value of a.
ta. Ci) Show that p = {(3-15) 5+(3+/s)S}is rational, and
determine the magnitude of p.
(/5+2) 6 -(/s-2)6
(ii) Without tables, find the value of
815
9. (a) If f(x) = x 5, evaluate
f(x+h)-f(x) lim I f(x+h)-f(x)J
(i) (ii)
h h-+- 0 l h
t(b) Repeat (a), where f(x) = 3x 4 -5x 3
10. If x is so small that x 3 and higher powers may be
3x 5
neglected, show that (1 - ) (2+3x) 6 is of the form
2
a+bx+cx2 , where a,b,c are certain integral constants.
Find a,b,c.
t11. If x is small, prove that the value of
3 4 3 2 2
(2 + --1i) (2 + �) - (2+x) 6 is of the form ax 3 approx-
2 8
imately. Determine the magnitude of a.
282
GENERAL EXPANSION OF (l+x)", (a+x) n
12. Show that (x + 1) 3 = (x 3 + .!..3) + 3(x + l) and hence if
X x X '
l 3 + -r1
x + - = 1, prove that x = -2. Fux,ther, show that
Xl 1 X 1 1
(x + -) 5 = (x 5 + - 5 ) + S(x 3 + -r) + lO(x + -),and thence
X X X x
l 1
that x 5 + 5 = 1. Also, pro·,e that x 7 + - = 1,
X X7

13. (i) Taking x = 0·001, use the expansion of (2+K)S to


evalua·te (2 · 001) 5 co rrect to 5 decimal places.
(ii) Show that the first 3 terms of the expansion of
(2-x) 6 where x = 0•002 are suffi�ient to calculate
the value of (1·998)6 correct to 7· significant
figures. Determine this value.
(i) Expand (a+3b)6 and use it to evaluate (1'03)6 cor­
rect to 5 places of decimals.
(ii) Evaluate (2 •02)6 correct to 5 significant figures.

*15. Write out in full the expansion of (x+y)B, Express


x 8 + x-8 in the form Au 8 + Bu 6 -+ Cu 4 + Du 2 + E where
u = x+x-1 and A,B,C,D,t are numerical coefficients,
16. Usin g the fact that a+b+c = a+(b+c), obtain the expan­
sions of (i) (a+b+c)2 (ii) (a+b+c) 3

E. THE GENERAL EXPANSICN OF (l+x)n, (a+x) n


(a) DEFINITIONS The expansions of (J+x) n, (a+x) n for
positive integral values of n , have been demonstrated to
be polyn omials of degree n in X,
When n is a given positive integer (say 7), then the
coefficients in the expansion of (l+x)',(a+x)1 can be derived
directly from the appropriate row in Pascal's Triangle.
Ho1,,ever, Pascal's triangle does n ot give the ao­
effiaients in the expansions of (l+x)n, (a+x)n for the case
when n = n.
For this ge neral case, say of (l+x)n , we know that the
expansion will corrunence with 1, end -with x n, and have (n+l)
terms. Thus (l+x)n = 1 + a1x + a 2 x 2 + a3 x 3 + .• .. + xn , where
a1, a 2 , a 3 , •• ,. , an are certain undefined constants. For
uniformity, we take the subscript to correspond to the index
denotin g the power of x; for example a3 is the coefficient of
x 3 and so on , {Usually, two further constants are introduced;
a 0 = 1, the coefficients of x 0 and an = 1, the coefficient of
xn.} Hen ce (l+x)n may be writ-ten as
(l+K)n = a 0 x 0 + a1x + a 2 x 2 + a3x 3 + .•. + arxr +•••+ 3nxn ,
1Note 1 Sr Sn ).
li01,JeVer, it is usual to introduce a new n otation in this
connection to replace these constants a 0 ,a 1 ,a 2 , ••. ,an , This
notation has a double significance; it refe rs to both the
wo rd aoeifiaient and to the word combination. {The
283
PASCAL TRIANGLE RELATIONS
aombination idea will be discussed later .
n
The new notation, namely er o r c r or ( �), rep laaes
the aoefficnent ar, of xr in the expansion (l+x) n, asswiing
1 � r � n. rhus, the expansion for (l+x) n would be written as
(l+ x)n = ne.oxO + e1x 1 + ne 2 x 2 + e3 x3+•.•+n e x r +•••.+ e xn
n n n
r n
or (l+x) n = cox 0 +cix 1 +c 2 x2 +c3x3 +., ••+c rxr+ ..••+c n xn
or (l+x) n = (no)x O +( n 1)x l +t 2 )x 2 +(n3)x3+••.+t r)x r+ ... +(nn)xn
In our wor k, we shall ge nerally use the notation ne
(and occasionally c r). r
Similarly, the expansion of (a+x) would gen erally be
0

written n
(a+x)n � e 0 a + e1a -lx+ e 2 an-2x 2+ ne3 a -3x 3+•••+nera n-r r
n n n n n
lf+·n:
•.+ e x ,
n
n+l
[Fur ther , the expansion of (l+x) would be
o n+l
(l+x) +l -_ +leox + e 1x l+n+le2 x 2 + +le3x 3 +•••+n+le xr+•••• ..
n n n
r
n+l
and of (a+b) 8 would be •.•+n+le x
n+l
(a+b) 8 8 eoa 8+ 8e a 7 b l + 8 e a 6 b 2 + 8e a 5 b3+8e a 4 b4+ 8e a3 b 5
1 2 3 4 5
'
+- 8e6a 2 b 6+ B e 7a lb 7+ 8eab s]
n n n
= l•
{Note These aoefficnents e 0 , e1, ne 2 ,••, e are still
n
undefined in value; we have mer ely used them to denote the
aoefficnents of x0 ,x 1 ,x2 ,..•• � in the expan sions of (l+x)0,
(a+x)n .
2. The n ext steps will be to der ive a certain relation
(called-the Pascal triangle relation) between a pair of con ­
secutive coefficients, and then to obtain a gen eral formula
fo r ne r, which will be pr oven by induation.}

(8) THE PASCAL's TRIANGLE RELATION


Ci) THE OUTER COEFFICIENTS nCo, nC n
n n
These are obviously both un ; • y, since eo • e co rr es­
n
pon d to ao, an �espectively. However these r esults n eed not
be asslllled, but will n ow be proven .}
2
Fr om (a+x) n = ne0 an+ne1 an-lx+ne:zan -2x +•••+ne _ a, xn -l+ncnxn
0 1
put fi1'8tly a=l, x•O, (1) 0 n
e o(l) n i.e. neo = 1

and seaondly a= O, x=l, e (l)n i.e. ne � 1


n
n �n""-�-
n
(ii) THE PASCAL TRIANGLE RESULT C + C
n = +I
n C
r-1 r r
This relation is derived fr om de�yrmining .the co­
efficien t of xr in the expansion of (l+x)n ,by two different
284
FORMULA FOR nC r
methods.
• ( l+x) ntl= n
No1v (l+x)(l+x) n n n
(l+x) { nCoxo+ CJxJ + C2x 2+...+ c r -1
r -1 x
+ nc x + ....+ c x }
r n n
r- n
r n r n r -1
The terms in this expansion are J. C x + x. C _ x
in x
r n n
r r 1
and,hen ce .the coefficie nt of x is { C + c .
r r _1}
-�• J!lJ def_ini_tion,
(l+x ) +l =
n+l
C0x o + +lCJxJ+ +lC2x·2 +;••+n+lc xr + +n+lc n+l
n n n
x
r · · n+l
r n+l
an d the coefficient of x is C •
r
r
Thus, equating these results for the coefficien t of x ,
n n n+l
we have c + c c �her e 1 Sr S n.
r-1 r r'

!!£l!J.. . result C _ + C = n+lC does not give the value


n n
This
r 1 r r
n n n
of the coefficients Co , CJ , C2 ,···• but gives a r elation
betwee n them. Thus, for example
n n n +l
7c i; + 7 cs = Bes, J J ca+J Jc9 = J2 c9; c1+ c.,. ci.., n .:: 3 etc.

(¥) TllE FORMULA FOR nC1'

This section shows a method of developin g a formula for


n
c , which will be pr oven in the next section . by. induc·tion.
r
n n
-N-ow- (l+x) = co x 0 +cJxJ+c2 x2 +c 3 x 3+.c 4x 4 +c5x 5 + •.. +cnx .... (P)
n n n
where co = Co, CJ = CJ, c2 = C2 for convenie nce her e.
n
I n (P), when x = 0, (l) = cox , i.e. co = 1 •••• (1)
0

Differe n�iatin g both sides of (P) with r espect to x ,


• n n -l
• • (l+x) = cJ+2c2x J +3c 3 x2 +4c4x 3 +sc5x 4 +••. +nt:n xn -1 ..•(Q)
I n (Q), when x = 0, . •• n(l)n-l = CJ i.e. c1=n .•. (2)
Differentiatin g both sides of (Q) with respect to x ,
n -2 n -2
• ·• n (n-l)(l+x) = 2.lc 2+3.2c 3 x J +4.3c 4x2 +•.•+n(n-l)c x
n
.•.••. (R)
n-2
I n (R), when x = O, • . n(n-1)(1) = 2.lc2 ,
.
i.e. c 2 -- .!U.!l:.ll ••••••.•••••• (3)
2_1
Differentiating both sides of (R) with r espect to x ,
n-3
n (n -l)(n -2)(1+x) 3.2.l.c 3 +4.3.2.c4 x+, .•. • n-3
+n(n-l)(n-2)c x .•••(S)
n
n -3
In (S), when x 0, n (n -l)( n- 2)(1) = 3.2.lc3,
n(n-1) (n -2)
i.e. C 3 = • • • •••••(4)
3.2.1

285
PROOF OF "er RESULT
Henae, from (1), (2), (3), (4)
nc = l nc ,. n nc = n(n-1) nc 3 = n(n-1)(n-2)
o ' 1 ' 2 2 .1 ' 3. 2 .1
Thus, we can guess that C4 = n n(n-l)(n-2)(n-3) , and henae
4•3•2.1
that in general,
nc = n(n-l)(n-2)(n-3) ••.••• [n-(r-1)]
r r(r-l)(r-2)(r-3) •..•.• 3.2.1 ' where 1 s r � n
or nc _ n(n-l)(n-2)(n-3) .•...•(n-k+l) wh ere 1 ! ks n
k - k(k-1)(k-2)(k-3) ...... 3.2.1 '
*(o) PROOF OF THE nc FORMULA BY INDUCTION
r
n
�. Assi,ne the result for cr is tI'Ue for two aonseautive
positive integers r = k-1,k and for a positive integer n,
where 1 s k-1 s n
i.e. assume nc _- n(n-l)(n- 2)(n-3)••.[n-(k°=T-l)j
k-1 (k-l)(k-2)(k-3) .•• 3.2.1
n(n-l)(n-2)••• (n-k+2)
(k-l)(k-2)... 3.2.1
n� _ n(n-l)(n- 2).•• [n-(k-l)j
and that - k(k-l)(k-2)... 3.2.1
_- n(n-l)(n-2)..• (n-k+2)(n-k+l)
k(k-l)(k-2)... 3.2.1
Now by the Pascal Triangle relation in (tl) ,n+lc = n�_ + nc
k 1 k
n+ l = n(n-l)(n-2)..•(n-k+2)
i.e. c
k (k-l)(k-2)(k-3)...3.2.1
n(n-l)(n-2) •••(n-k+Z){n-k+l)
+ k(k-1) (k-2) ... 3.2.1
= n(n-1)(n-2)..•(n-k+2) [l + n-k+l], on
(k-l)(k�Z)(k-3)...3.2.l k factorising
n(n-l)(n-2)..•(n-k+Z) n+l on simplifying
(k-l)(k-Z)(k-3)... 3.2.1 [ k ]
= (n+l)n(n-l)(n-2)... (n-k+2) Check this step!
k(k-l)(k-2)(k-3)••• 3.2.1
= (n+l)(n)(n-1) ... [n-FT-(k-1)] · Note rearrangement
k(k-l)(k-2) .•. 3.2.1 of last factor.
n
and this is the formula for - c r where n, r are repZaaed by
(n+l), k respeatively.
Henae, for 1 � k-1 � n, if the result for nc (where n
is a positive integer) is true for r= k-1 and r =rk, then it
foZZows that it is triue for the integer (n+l) and r = k.
n = n n(n-1)
�. Now c1 n and C2 = 2•1 by (i)
These results satisfy the formula for
n C = n(n-l)(n-2).•• (n-r+l), 1 � r � sinae when ral, nC =
r r(r-l)(r-2)•.. 3.2.1 n 1 !!.l
286
GENERAL RESULT FOR (l+x)"
{the other terms (n -1), (n -2), ... , r, (r-1)... have n o mean ­
ing here when r = ,1}.
n n-1
and when r = 2, C 2 = (2•1 ) {the other terms (n -2), (n-3)...
n

.., (r-2), (r-3).•. are meani ngless here when r = 2}.


n
Thus, the result C
r
(for positive integral n) is true
for r 1, r = 2 an d henae by Step 1, it follows that the re­
=
sult is true for r = 2 and integer (n+l), i.e. the fact ihat
the formula holds for n c1, n c2ensures that it holds for n 1c�

�. By repeated appliaation of steps 1, 2 we can deduce


tfiatt'he formula ncr holds for all integral values of n , r.
Thus the formula holds for n C1 , nC 2 and hence for n +lC2.
Similarly, the formula holds for n C2 , nC3 an d hence for n +l c3.
·
Likewise, the formula ho1ds for nC3, nc4 and hence for n+1c .
4
Thus the result can be exten ded to include all coefficients
of·the form n + lc .
r
n +l
Now, the formula can be shown to be true for C1 an d
n +l.
since it is true by above for C2 , then it follows that it
n +2 n +l n +l
·:.olds for c2 • Similarly from C2 , C3 th e resu1t can
n +2
be exten ded to C3, and so on to include all coefficients
n +2
of the form Cr• Then the result can be exten ded to the
n +3
form Cr etc, and finally to all coefficien ts of the form
N
C , where R, N are positive integers and 1 � R � N.
R
(£) RE-STATEMENT OF RESULTS FOR (l+x) n, (a+x) n
llY definition
(l+x)n = 11coxO+nc1x1 +nc2x2 +..• +n crtr+ ....+ncn xn
where n c0, n c1, n c 2 , .•., ncn were undefined.
, n _ n (n-l)(n-2)••(n-r+l)
However, we have n ow proven that C
r - r.(r-l)(r-2) ••3.2.1 •
for 1 � r � n , and Co = en= 1.
n n
n
Thus (l+x) may be rewritten as
n n 1) 2 + n (n-l)(n-2) x3 +
(l+ X ) = 1 + nx + ( -
n
2.1 X 3.2.1
n(n-l)(n-2)•••(n -r+l) r
+ x + ... + xn ,
r.(r-l)(r-2).•.2.l
or using the faatoriaZ n otation (e.g. 31=3,2,l, 5!=5.4.3.2.1)
then (l+x) = l + n x + n (��l) x 2 + n(n-;�(n-2) x3 + ...
n

n n l n 2
... ' ( - )( - ),.:( -r+l) Xr + "' +xn
n
r!

287
THE BINOMIAL THEOREM
Similarly, n-2 2 n n-3 3 n n -r r
x +,, .x n
-(�-h:yn-;,, an+n c1an - 1x+ c2a x + C3a x +•. ,+ e a
n
r
n n-1 n(n-1) n-2 2 n(n--1) (n -2) n -3 3
= a + na x + a x + a x +....
2•1 3•2.1
n(n-l)(n-2)....(o-r+l) n -r r n
+ + x
r(r -l)(r-2) .... 3.2.1 a x +

This resu lt is kn ow n as the Binomial Theorem for a


ositive i n te ral n.
Notes 1. The use of factorials enables us to give an altern­
ative form of ncr.
n
To understand it, consider say _ C3.
n n(n-1) (n-2) {there are 3 factors in both numerator
Now C3
3.2.1 and denominator,}
n(n-l)(n-2){(n-3)(n-4)....3.2.l}.
Check this li ne!
"3.2.1 {(n-3)(n-4) ....3.2.l}
n! where n ! • n(n -l)(n-2) 3.2.1
3! (n -3) ! and (n-3) ! • (n -3)(n -4) 3.2.1
s· ·z z
um ar Y,
nc n(n-l)(n-2)....(n-r+l)
r r � r -1)(r-2).. ,. 3.2.1 {the r e are r factors
in both numerator and denomin ator}
n(n-l)(n-2)...(n -r+l){(n-r )(n -r-1)..•3.2.1}
r(r-l)(r-2)...3.2.1 {(n-r)(n-r-1).••3.2.1}
n!
r!(n-r) !
n
2. The (r+l)th term, denoted by u ' of the expansion (a+x)
r+l
0 n-r r
1·s gi ven by u = c a x , note 1 Sr Sn.
r+l r
n (n -l)(n-2)...(n -r+l) n-r r
a x
r(r-l)(r-2)...3.2.1
n! n -r r
a x
r !(n-r) !

r, EXAMPLES ON THE GENERAL EXPANSION C l+x) n, (a+x) n


n
EXAMPLE I. Evaluate by the formula for Cr,
l:j
(al c5 (bl llc4 (cl 12C12 (d) 7C3/lOc6

n(n-l)(n-2).••( n-r+l)
n
Method, Now, by above,
r
• C
r(r -1)( r-2)••• 3.2 .1 •
{Note ther e a re r factors in both n umerator and den ominator }
8(8-1)(8-2)..• (8-5+1) putting n•8, r•5 in
(a) Thus 8Cs
5(5-1)(5-2).•. 1 n
Cr r esult.
{There are 5 factors in both numerator and denominator}
8 8·7·6•5·4 56 on cancelling.
Hence c5
5.4.3.2.1
A Z temative.,1 y, 8C5 -- 8! ._fil_.8.7.6 . 56
5!(8-5)! 5!3! 3.2.1 n!
noting n c = --,----,-­
r!(n-r)!
r

288
EXAMPLES ON THE BINOMIAL THEOREM
11.10.9.8 (there are 4 factors in both numerator
Cb) 11c4 = and denominator)
4. 3.2.1
330 on cancelling.
11 Ci+ = _ill = 11•10•9•8
Alte:r>natively, 4! 7 ! 4.3.2, l
330

12.11.10.9••••2.1 (NotP 12 factors in numera­


(c)
12.11. 10•• :• 2.1 tor and denominator)
• �= 1 l 2 12 ! 12! 1
A 7,te:r>nat1,Ve1,y, C1 2 =
-1-2!c..(1"""2--1
- -2_)_! = 12!0! = O!

and since this result is unity, • • we define O! as 1

7! 4!6!
=--x--
(d) using factorials for
3!4! 10!,
simpler working
6.5.4 1
= 10,9.8
on cance11'1 ng, = 6

Note 1. The expansion of (l+x) 7 by


=ca) Pasaal 's Triangle, is 1+7x+2.1x-2 +35x 3 +35x 4 +2lx 5 +7x6 +x 7
(B) the Binomial Theorem is
1+ 7 C1x+ 7 C2x 2 + 7 C 3x 3 + 7 C 4 xt+ + 7 Csx 5 + 7 c 6 x6 +x7
Verify from the formula, that the Pascal triangle set
1 7 21 35 35 21 7 1 is respectively

This idea applies for each row of Pascal's triangle.


Note 2.
n
c
n
c
n
since c _n:.::.!;;.__
r n-r' r r!(n-r)!
n n! n!
and c
n-r (n-r) ! [n-(n-r)]! (n-r) !r !

EXAMPLE 2.
Solve the equation nc3 - 2. n+lc2 = nc1 for n
nc
Method. The result for 1 � r in, is given by
r'
n n{n-l}{n-2} .•• {n-r+l}
c =
r r(r-1) (r-2) •••• 2.1
n _ n{n-l}{n-2} n+lc _ {ntl}n n
Thus c 3 - 2 C1 = n
3.2.1 - 2.1 '
}
n{n-l}{n-2 - 2. {n+l}n
Hence n
3.2.1 2.1
{n-12 {n-2} -
i.e. (n+l) 1, on dividing by n;IO
6
Solving this equation, n 2 -9n-10 = 0 i.e. n = -1 or 10
n
H(Jl,)eVer, since n is a positive integer (and n � 3 for C3 to
exist), n 10.

289
EXAMPLES ON THE BINOMIAL THEOREM
EXAMPLE 3. i
F nd, n
i indexform, the ht irteenth term
of the expansion(5y - 2/b)l 9 ·

[The binomial theorem for (a+x)n, where n is a positive


integer, aan be cipplied to determine a speaifia term of an
expansion. The Pascal Triangle method of expansion is far too
long in this aase.J
. n of (a+x)n
Method. For the expan sio ne an-r xr
, u
r+ l
=
r
Henae, for the expansion (Sy - t> l9 ,
u1 3 � 19 c12(Sy)19-1 2(-�)12, wher e a= Sy , x = -�.
n= l9, r= 12.

? -2
__12.L (S y) ( )12 = 19.18 .17.16 .15.14.13(Sy) 7 2)12
=
12!7! b 7.6.5.4.3.2.1 Cb
= 19.17.13.5 7 .3.2 14 y 7 /b 12 on simplification.

EXAMPLE 4. Find the


( x-l ,12
( i I coefficient of x 3 in 2
( i) coefficient of x-7 n
i i (3x - �)8
2-x 3 15
(iii) term independentof- X in £_
( + x -2
)
3

Method.
�In the expansion of (2x-1) 12 , we can w r ite down at
sight the term in x 3 , pr ovided that we first write the
expansion in the form (-1+2x)12 .
Thus the requir ed ter m is the four th term, since
1 .11.10
12c3(-1) 12 -3(2x)3 = 2 (-1)3(2x)3
3.2.1
-1760x on simplification
3

(ii) The coefficien t of x-7 in (3x - �) 1 8


cannot be calcu-
lated without some effort. Probably the best method is
to use the gener al term, i.e. u .
r+ 1
_ s 8 -r .:.L r
Now u + - C/3x) ( 2) .......... •..•........ •. (L)
r l 2x
On examination of this term, we see that it is of the
form Ax 8 -r(x -2)r = Axs -3
r
wher e A is the nwneriaaZ co­
efficien t of u l" Sin ce the ter m in x- 7 is r equired,
r+
. ·. we must have 8 -3r = -7, whence r = 5.
Thus the r equired
term from (L) is
_ 8 8 -5 -1
u + - C5(3x) ( 2)5
5 1 2x
8 .7.6 - -189 x-7 '
i.e. UG = 3 x • .::1._
3 3
3.2.1 25xl0 - 4
an d the coefficient of x-7 is -189/4.

2 90
EXAMPLES ON THE BINOMIAL THEOREM
2x3 15
(iii) The ( r +l)th term of (� + x-2) is given by
= lSC/ � )',-t (x-2) r ',., •,,",, .. " .. " (M)
2 3
u
r+l
This term.is of the form B(x3)1S-r(x -1) r = Bx 4s-s r ,
whe re B is a n umerical constant,
For the term to be independent of x, the power of x
must be zero, i.e. 45-5r = O, whence r 9.
Thus f rom (M), u + 1S 2x 3 15- 9 -i .9
9 l C 9( ) (x )
3 6
15.14.13.1 2 .11,10 2x 3
( ) (x-i)q
6.5.4.3.2.1 3
13.11.7.5.2 6
i.e. u 1 0 on simplification, and this is
3
the r equi r ed te·rm

EXAMPLE 5. The first three terms in the expansion


of (l+axln in ascending powers of xis
3 2 2
- 16x+ ; x , Find the values of
a n.
Method, The expan sion of (l+ax)0 in ascendin g powers of x is
(l+ax) = 1 + C1(ax ) + Cz(ax)2 + C3 (ax)3 + ,,,
n n n n

_ + n(n-1) a2x2 + n(n-1)(n-2) a3x 3 +


- 1 + nax 2'1 3'2'1

... h-:; •' : ��:


Sin ce the first 3 terms are 1 - 16x + 352
- - x 2,
3
••••••(1)
on equatin g coefficients
• • • • • • (2) of x, x 2 •
n(n-1 ) 256 352
From (1), a = -1§. and thus (2) becomes , �
n 3
-1
2
2
i.e. 128(n- 1) = 3� whence n = 12, and , ·, a =
n

E'XAMPLE 6. In the expansion of (5+4x) 17, de.termine


(il the ratio of coefficients of the consecutive
terms u and u
r r+I
(i il which pair of consecutive terms have equal co­
efficients.

Method.
(i) Now u = 17c (5) 17-r(4x)r
r+l r
and the coefficien t is 17c 5 17-t 4 r
r
Thus u = 17c (5)17-( r-1)(4x)r-1 •
r r-1 r l
and the coefficient is 17c 510-r 4 - ,
r-1
Henae the ratio of the coefficients of is given by

291
SET 6C

--�1=7_!___ x r! (17-r)! Check this


(r-1)!(18-r)! 17! step!
r 5
18-r ' 4
{Note r I r(r-l)(r-2)•••3.2.1 = r{(r-1)! }
and (18-r)! (18-r)(17 -r)(l6-r)•••2.l
(18-r){(17-r)!}
Sr
4(18-r)

(ii) Let the two consecutive terms with equal coefficients


be u ' u Thus from (1) above, since the ratio of_
r r+l'
the coefficients is unity, • • Sr 1 i.e. r = 8
4(18-r) on solving,
Hence the 8th, 9th terms of (5+4x) l 7 , {. in ascending pow­
ers of x} have equal coefficients.

EXERCISES SET 6C
[The following exercises are generally best done by using the
general form of the expansion of (a+x)n].
n
1. Write down the result for c where 1 '!!' r � n. Hence
r'
evaluate the following
(a) sc3 (b) 7cs (c) 9c6 (d) 6C 3 (e) l Oc4 (f) 13c4
(g) i1c7 (h) i2c2 (i) lOC10 (j) 1 7 c3 (k) sc4 /1ocs
Gc 1++ 7 c 2
(l) 7c?/ C 2
9 (m) s c3/12c 1+ (n)
11c
3
t2. Build up Pascal's Triangle for n 1 to n = 10. Verify
that the row 1,8,28,56,70,56,28,8,l corresponds to the
set 8c0, 8 c1·, 8 c :: , 8c 3, 8c4, 8c 5, 8c 6, 8c7 , 8 c 8 respectively.
Take the row for n = lO, and verify a similar result.
n n n+lC and the
3. The Pascal Triangle relation C + C
n r-1 r r
n
result c = c hold for 1 tr � n. Verify these
r n-r
results for the cases (i) n = 10, r = 3
(ii) n = 7, r = 5 (iii) n = 25, r = 2
4. (a) Solve each of the following equations
n
(i) C1- 6C3 = 10c2 n
(ii) C2 + 7 C3 = 8C3
(iii) 2.nc2 +2.. n+lc1 t (iv)
t(b) Show that there is no valid value of n for which
k2n c + nc ,. 1. n
C3.
I l n
5. Use the results stated in question 3, to solve for n
n n
(i) 8 C3+8 C 1+ "' C1+ (ii) C1 +n C2 = 12 c2

292
SET 6C
n n n
(iii) C4 =
Cs [Start C 4 =
n n k k
(iv) C 7 = C13 (v) C7+ Cs
{Note n, k are integers.)
�-------�
6. In each of the following expansions, find the value of
the specified tenn. (Leave in index fonn where necess-
ary:)
(i) (3 - !--,x?
7 [u ]
5
x 9
(ii) (y 3 - ) [u 6 ]
3
t (iii) (x -x-'2)12 ; [u 9]
7. Find the
(a) middle term of the expansions
l0
(i) (a -2 b) 8 t (ii) (x % + X -�)
(b) middle terms of the expansions
11
(i) (�
3 - y)7 t (ii) (1. + x 2)
X
8. (i) Show. that the coefficients of x 6 and x 7 in the ex­
pansion of (x+ 2 ) 20 are equal.
[Hint: start ( 2 +x) 2 0J
(ii) In the expansion of (l+x)14, show that u1 o= l6.u6•
if X = 2.
9. (i) In the expansion of (l+x) 20 in ascending powers of
x, find the ratio of the tenth term to the eighth
term.
(ii) For the expansion of (2+y 3) 3 5in ascending powers
of y, find u 1 3 :u12 •
10. Determine the coefficient of x�in the expansion of
(i) (3-x)7 (ii) ( 2 -x 3 ) 5(iii) (a+bx) 6 t(iv) (kx+l) l 5
11. Show that the (r+l)th tenn in the expansion of
12 -3r
(x + 1.2)12 can be written as 1 c x
2
, and hence
X r
find the (i) coefficient of x 3 (ii) coefficient of
x-9 (iii) term independent of x, in the expansion.
12 . Find the value of the coefficient of the power of x
specified in each of the following
(i) (x - -1/x 2) 6 ; x o (ii) (x 2-x-l)10; x 2
X 2 7
(iii) (x _ b,12; XG (iv) (- - -) ; X
.<.X 2 3x
10
t(v) (3x 2 +sx- 3) XO t(vi) (2x 2 _ _ l) 11,' x 7
4x
1 1 7
t(vii) -(3x - -) ; x 2 t(viii) (x 2-3x-5)12; x-4
X 2x
[Leave answer to (v) in simplest index form]
13. Find the value of the term independent of x in the ex-
pansion
2 9 lj,
(i) (_l - k..) (ii) (x + 2x- 3)8 t (iii) (yx 6
3x 2
293
SET .6C
J.4. Determine the r equired coefficient in each of the
following 2 9
(i) (2-tit3)(1-x)l3; x 4 (ii) (2-x+3x 2 )(1 - -) ; x O
l X 1 6
(iii) (x-1) (x+l) (x+2) 1 2 ; xl o *(iv) (x - -)lO(x + -)
X X
_; x 6

15. (i) Find the value of x so that the numerical values


of the twelfth an d thirteenth ·terms of the expan ­
sion (3+2x)l7 in ascending powers of x are equal,
(ii) The exp ansion of ( 1+ax+bx 2 )(1+x) 1 O in ascending
powers of x begins with the t·el111S 1 + x + x2 ,
Determine the values of a an d b.
16. In the expansion of (l+x)n in ascending powers of�
(i) �he third term is 28x 2 , find n an d the first
3 te r ms.
(ii) the tenth and eleven th terms are equal in value
when x • �. find n.
17. The first three terms in the expansion of ( l+cx)n wher e
n is a positive intege r .ar e
(i) 1+6x+l6x 2 , Find c and n .
20 2
Hii) 1 - 4x + x , Find c, n and the four th term of
3
the expans ion ,
18. (i) In the expansion of (2+3x)n in ascendin g powers of
x, the ratio of the thi r d and four th terms is
5 : 14 when x = 2/S, fin d n.
(ii) If (1+2x)n is expanded in ascending powers of x,
find the value of n fo r the r atio of the coeffic­
ients of x 4 and x 6 to be 5: 8.
19. Noting that (3x+4) 2 7can .be w r itten as (4 + 3x) 2 7, show
that the r atio of the coefficients of the r th an d
( r +l)th terms of the expansion in ascendin g powe r s of x
is 4r : 3(28-r).
Hence fin d which two con secutive terms of this expansion
have equal coefficients.
20. iir ite down the term involving x r in the expansion of
and p rove that
(ax+b)n, where n is a. positive inte e r,
the ratio of the coefficient of x r+ 1
to that of x r is
(n-r)a I ( r +l)b,
Deduce in the expansion of (3x + 2) ll1that the coeffic­
ien ts of x 9 and x 8 are equal,

21. The fifth, sixth and seven th terms in the expansion of


(l+x) 7 in asce nding powers of x form an a rithmetic se­
quence. Show that 2.2lx 5 = 3Sx 4 +7x 6, and hence find x.
22. In the expansion of (1 + x)n in ascending powers of x,
C4,nC5,nC5
n
show that the coefficien ts of x 4 ,x 5 ,x 6 are
respectively. If these coefficients form an arithmetic

294
APPLICATIONS OF BINOMIAL THEOREM
sequence, prove that
n(n-1)•••(n-4) .. n(n-1) •••(n-3) + n(n-1) ••• (n-5) and
2 • 5! 4! 6!
thence that 12(n -4) = 30 + (n-4)(n-5). Hence show that
two values of n are possible and find them.

23. The first term of a binomial expansion is unity, the


second term is positive and the fifth and seventh terms
have equal coefficients and themselves in the ratio
25: 4. Prove that the expansion is of the form (l+x)n
where .x = 2/5, n = 10.

24. (a) The sum of the coefficients of an expansion may be


determined as follows, without evaluating each in­
dividual coefficient.
For example, consider (5-3x) 7 .
Now (5-3x) 7 = a0+a1x-+-a 2 x 2 +a3x 3+... +a 7x 7,
where ao,a1,a2 , •.• ,a 7 are the coefficients of
x 0 ,x l ,x 2 , •••,x 7 .
Putting x = 1, .•. (5-3) 7 = a 0 +a1+a2+••+a 7 , i.e. the sum
of the coefficients is (2) 7 = 128.
(b) Similarly, find the sum of the coefficients in the
expansions
(i) (4-5x) l3 (ii) (x-\)9 (iii) (3x-2y) 7 a., - -;i
' • ) (-
.{iv 3b.
2 4
G. APPLICATION OF THE BINOMIAL THEOREM TO OTHER FIELDS
{The exercises below indicate some applications of the
binomial theorem to other aspects of mathematics; it will
be used considerably in the next chapter, in the section on
bin omial probabilities and the binomial disttibution.}
EXERCISES SET 60
1, l'he diagram shows a sketch of y
y = x 7 . P,Q have abscissae x,
x+h respectively. Show that
tht:< gradient of the secant PQ
is equal to
f(x+h)-f(x) (x+h) 7 -x 7
h h
=
Use the binomial theorem or
Pascal's Triangle to expand
(x+h)7, and hence find the
gradient of PQ in simplest
0

form. Deduce the gradient of the tangent at P to y�x 7 ,


2. The points R, S on the curve y = 2x 6 have abscissae 1,
l+h respectively. Determine the gradient of (i) RS in
simp.lest form, using the binomial theorem to aid in the
calculations (ii) the tangent at R to the curve.
If f(x) .. x , where n is a positive integer, find f'(x)
n
*3,
from the definition f'(x) = �� lf(x+h) �f(x) 1, using the
binomial theorem to aid in the work.

295
SET 60

·r
4. The sketch is that of y = x(l-x) 3.
The shaded area is represented by
the definite integral X (1-x) 3dx •
X
0
Evaluate t!tis area; (use Pascal's triangle to help in
the calculation).
5. Find the stationary points on the curve y = x 2 (1 - x) 4 ,
distinguishing between them, Hence, sketch the curve.
-Calculate the area bounded by the x axis and the curve
y = x 2 (1-x)4, between x = 0 and x_ = l.· �
6. Show that critical points occur at
1
x = O, �· 1 on the curve
I.
y = x 2 (1-x) 2 . Verify the sketch.
Calculate (i) the shaded area I
"
(ii) the volume of the solid gene�1ted when the
shaded area is rotated about the x axis; (use Pascal's
triangle to !1elp in the calculations).
l 1 _ (/J-/2°)5+(/J+/z)S
7, (l.') Show that (/3+72)5 +� 5 - (/J+/z)S,(/
) 3-Tz)S
and hence simplify this with the aid of the
binomial theorem,
l l l
t(ii) Prove tI tat +
( fJ-l)4 ( {)+l) 4 2
* 8. If ( 3+/s) 4 = a+b /s, find the values of a and h. Further,
express (/s-1) 6 in a similai: form,
ence 1 16
simplify
(3-/5)4 - (/s+1)6·
,11

9. Use the binomial theorem to find the value of


99 9
.(i) ( ) correct to 4 decimal places
100
t (ii) (1·01)11 correct to 5 significant figures.

10. A bag con ta ins 30 marbles, 10 of which are coloured


white and the remainder are Vari-coloured (but are not
white). I close my eyes and draw a ball from the bag.
The chance (or probability) that it will be white is
lO l
an d t he c I 1ance that 1· t wi. 11 not be wh'1 te is
.
30 3
20 2
I replace the ball after ob�erving the colour
30 3.
and repeat the drawing seven times, (replacing the ball
each time). According to the theory of probability, to
be discussed in Chapter 7, the probabilities of drawing
a white ball out of the bag on exactly 0,1,2,3, ..•... ,7
occasions are the successive terms of the binomial ex­
pansion (-2/3 + 1/3) 7•

296
C0'1BINATORIAL INTERPRETATION OF r
nc
Thus the probability of drawing a white ball on exactly
0,1,2,3, ••• ,7 occasions would be the numerical values of
2 7 7 2 6 1 1 7 2 5 1 2 7 2 4 1 3 1 7
(3) ' Ci (3) (3) ' Cz (3) (3) ' C3 (3) (3) '. • .. •' (3) '
(a) Calculate these respective probabilities (as frac­
tions), and verify that their sum is unity,
(b) Use these results to determine the probability of
drawing a white ball from the bag (i) on 2 or 3
occasions (ii) at least 5 occasions (iii) less than
2 occasions.
tll, In tossing an unbiased coin, the probabilities of exact­
ly 0,1,2,3,.•. ,10 heads in 10 tosses of the coin, are
the successive terms of the binomial expansion (��)10,
(a) C alculate these respective probabilities (as frac-
tions), and verify that the sum of these ·probabil­
ities is unity.
(b) What is the probability of (i) 8 or more heads
(ii) at most 4 heads.

H. COMBINATORIAL INTERPRETATION OF nC r
The binomial aoeffiaients of the form
nc where r, n are
r
positive integers such that 1 .:: r !: n, ean be intewreted not
only as aoeffiaients but also a.� aombinations. To understand
this, let us consider the expausion c>f
(a+x) S = a 5+ 5 C1a4x+ 5 Cza 3x 2 + S C 3 a 2 x 3 + 5 C 4 ax 4+x 5
Now (a+x) 5 may be writ ten as (a+x) (a+x) (a+x) (a+x) (a+x),
The resultant terms in the expanded product of these five
factors are all of degree 5, since each such term has as its
components one and only one of the letters from each paren­
thesis. Thus there are terms involving a 5 ,a 4 x,a 3x 2 ,a 2 x 3 ,ax 4 ,
x S , The terms of the form a4 x are obtained thus
(� or (j����+x)
or (a�'Tra� =+x) or (..-.
a�+in"a� -+x)
.. - -- -
= = = = .- ... ..
or (a�Th�+x>Tu-+x>fa+
� x)
Each of the indicated products is of the form a 4x, and
there are .5 i.e. 5 c1 such products. Hence the term in a 4 x is
S c1a 4 x,

To determine the term in a 4 x we select out of the five


groups or "boxes" represented by the parentheses
(a+x), (a+x),•• ., one "x" and four "a's". Thus the ao­
effiaient 5 c 1 is equal .to the nU11ber of ways in whiah we aan
seleat one "x" and fow> "a 's" out of the five boxes (or
groups), each box containing
both an 'a' and an "x". ! a+x i , ia+xi , i a+xi , i a+x i • I a+x !
Similarly, the coefficient of a3x2 is Sc and this is 2,
the number of ways in which we can select exaatly two "x ' s"
297
COMBINATIONS AND nC
and three "a's" Ol't of the same five boxes. These 5c2 = 10
combinations are shown below

�--��-
.h..1. (aQ,Qa+�J(a+':x) hl (a�""-+x)?'b�)
�---- .:.:...:,;.� �:..:::..!.>I"-........:._...

..____,,,��
.2..,__£_ (a"+x}ra�x�'"'ln,+x) hl. (a+xTia�a�IA.+x)

2..lQ_ (a+����+x)
Hence, 5c2 is equal to the number of combinations or
selections from the product (a+x)(a+x)(a+x)(a+x)(a+x), where
each combination contains exactly two "x's" and·exactli three
"a's". [There are five factors (or groups or boxes), each
containing an "a" and an "x"].

C is equal to the number of ways in which


In general,
n
r
we can choose exactly r "x's" and exactly (n-r) "a's" out of
.n boxes, each box containing both an "x" and an "a". That is,
nc is equal to the number of combinations which are possible
r
to produce terms of the form an -r x r out of the expansion of
(a+x) n = (a+x)(a+x)(a+x)(a+x) , . , , (a+x), where there are n
facto rs each being (a+x).
n
Note The notation Cr can be extended from the interpretation
as a set of binomial coefficients for varying r, n (provided
r, n are positive integers and 1 � r 5 n), and the combina­
to rial interpretation as the number of . ways of selecting r
objects "·x" and (n-r) objects "a" out of n boxes, each of
which contains both an object "x" and an object "a".
For examp�. consider the case of 6 students Arthur (A),
Belinda (B), Connie (C), Don (D), Ethel (E), Fred (F). Two of
these students are to be selected to represent the school at
a social function , The number of possible sets are shown
below:-
AB BC CD DE EF
AC BD CE DF
AD BE CF
AE BF
AF
[Note, order is not considered in each of the above sets.] On
counting, the number of such pos�igle groups of two students
from the six students, is 6c 2 = �2· = 15.
•1
Similarly, the ntUD.ber of selections of grouis of three
cards from a set of eight unlike cards is 8c3 = 3'7'� = 56 •
• 2.
{The group of 3 cards is selected as a whole, and it is con­
sidered only as an unordered subset of the original set of 8
cards.}
This extension of n Cr to selections of groups of r ob­
jects from a set of n unlike objects, (where each group of r
objects is selected as a whole unordered subset of the set of
n objects), will be considered further·in Chapter 7.

298
CHAPTER 7

THEORY OF PROBABILITY

A. HISTORICAL INTRODUCTION
According to texts on the History of Mathematics, the
theory of probability originated in the seventeenth century.
In 1654 a French gambler named Chevalier de Mere became con­
cerned with the problem of how to share the "pot" (or divide
the stakes) when a gambling game of chance broke up before
the game was completed, He thought that some formula could
be devised to share the money equitably, on the basis of the
chance of each player to win ·the game (at the stage it broke
up, of course). He proposed the problem to Blaise Pasc•l
(who was the foremost French mathematician and philosopher of
the day). Pascal corresponded on the matter with another
eminent French mathematician called Pierre Fermat. It is
amusing to note what Pascal said, of de Mere, to Fermat: "Tl
a tres-bon esprit, mais il n'est pas geometre: c 1 est C0/7U71e
vous savez, wi grand defaut. 11 [Translated roughly, this says
"He is very intelligent, but he is not a geometer (i.e .. a
mathematician): this, as you know, is a great defect."]
Although much of our present-day theory on probability
had its beginnings in the research of Pascal and Fermat (and
a little later Jacques Bernouilli and D'Alembert), de Mere
was greatly disappointed with their findings in reference to
his problem, (Needless to say he eventually became bank­
rupt I)
From these beginnings, it soon became apparent that the
theory of probability was useful in many fields in which in­
telligent forecasting is important. However it was not until
the early nineteenth century that it was stated in rigorous
form. That the theory of probability has been a most contro­
versial topic is evident today in the writings of modern
mathematicians, university scholars and the like. The very
basic definition of the probability of an event happening
varies from text to text, .from authority to authority. In
this syllabus, we steer a conservative course through the sea
299
BASIC IDEAS
of probability. (For those students who wish further inform­
ation on these different approaches, there is a wealth of
material readily available on the theory of probability,)
The application of the theory of pro�ability to areas of
knowledge other than mathematics seems to have developed
rapidly only during the twentieth centl'ry. A working W1der­
standing of probability theory is now essential in many
phases of such diverse fields as governmept, sociology, econ­
omics, psychology, business administration, statistics, bi­
ology, physics, chemistry, engineering, astronomy, medicine,
etc.

B. BASIC DEFINITIONS AND IDEAS


(i) The basic idea in the theory of probability is that of a
random experiment. This term is not usually defined in math­
ematics but is used.to refer to any experiment /or situation,
or event) where non-identical data result from what seem to
be identical processes.
For example, let us assume we have a perfectly symmet­
rical 6-sided die (the word "die" is the l;iingular of "dice").
If this die is �ut into a container l the container then
shaken a number of times, and the die then thrown onto a
table, on the uppermost surface will appear one of the

D D. D·. D·. . D·. . · D·. . .·


following:

i.e. 1-spot, 2-spots, 3-spots, 4-spots, 5-spots, 6-spots.


These are usually referred to as a one (1), two (2),
three (3), four (4), five (5) or six (6).
This experiment is repeated; the die being put in the
same container in the same manner, the container then shaken
just as vigorously as previously and for the same number of
times, the die then thrown onto the same table in exactly the
same fashion. Again on the uppermost-surface will appear a
1, 2, 3, 4, 5 or 6.
If this experiment is repeated a number of times, then
the appearance of numbers on the uppermost surface of the die
will riot be in any regular pattern, but the outcomes i.e.
1, 2, 3, 4, 5 or 6 will be entirely at random. That is, al­
though we try to keep the process of shaking and throwing the
die identical in each case, the possible results or outcomes
will not be identical.
Similarly, if we return to the beginning of this chapter
and consider the first letter of every word in the first
paragraph, then although the process of finding the letter
is identical, the results will certainly not be identical.
Such an operation could be classified as a randan eXEc>eri­
ment.

300
NOTES
NOTES. 1. Unless othen.,ise stated, all dice in our course
are perfectly balanced, although this is virtually impossible
in practice, Similarly, all coins are identical, except in so
far as there is a "head" on one side and a "tail" on the
other side. FUI'thermore, all balls, marbles etc, are in­
distinguishable from one another, except in as far as a
characteristic such as colour, is concerned,
2. A coin may only land with the head (H) or the
tail (T) uppermost; there is no landing and remaining on
the edge. Similar comments apply to dice, and in other ob­
vious circumstances.
3. All experiments involved in this course will be
considered to be random experiments; i.e. we asswne a
randomising process, such as that described for tossing a
die. Simila:rly, coins are tossed in an identical way:
balls thoroughly mixed before being taken out of bags; cards
thoroughly shuffled before being dealt, etc. All these vari­
ous experiments are random experiments, i.e. the process is
duplicated on each occasion, but the results are not. This
randomising process is the means whereby we ensure that the
experiment is statistically regular, i.e. that we have made
no attempt at all to influence the final result of the exper­
iment. Moreover, we assume that all data collected by us in
these experiments, and all infoIIDation taken from source
books etc. have been similarly obtained, i.e .• with statistic­
al regularity.
4. In this course, the random experiments discussed
involve only a finite ni,nber of possible outcomes. Thus, in
tossing a single die once and once only, the number of out­
comes is finite, namely six; these are 1, 2, 3, 4, 5 or 6.
5. For the non-card player:· in a regular pack,
there are 52 cards; each with three characteristics (on one
face only - the backs of each of. the 52 cards are identical),
(i) Colour (Red - 26 cards, or black - 26 cards)
(ii) Suit (Hearts Cv - red, 13 cards; Diamonds O - red, 13
cards; Spades+ - black, 31 cards;
Clubs• - black, 13 cards)
(iii) Order within the suit, called the Face Value
Eac'r; suit has 13 cards called respectively the
ace (A), two (2), three (3), four (4), five (5),
six (6), seven (7), eight (8), nine(9), ten(lO),
jack (J), queen (Q), king (K). The latter three
cards, the jack, queen, king of each suit, are
called the picture cards since they have im-
m prin ts of faces on the m ' whereas the cards from
the
wn ace
b to the ten inclusive have the same
n er 0f 1arger mar ik ngs a s the digit'on it.
Thus, the three of spades would appear as sketched,
Note the ace is often considered the highest card of
each suit, but we will take the ace as lowest card.
301
SIMPLE AND COMPOSITE EVENTS
(ii) Terms suah as "equally likely", "on an averaee of 1 in
ten", " in 3% of aases" etc. slip into the terminology of
probability with great facility.
(a) I f the outcomes of an experiment are "equally likely"

I
(or "equi-probable", in. some texts), theri we infer that there
is no reason to believe that any outcome (or result) is more
likely to occur than any other. Thus, in a bag

I
3W
there are 3 white (W) balls, 5 red (R) balls,
SR
4 black (B) balls, and we reach into the bag
(with our eyes closed) and select a ball at ran- �
dom, then -·of the (3+5+4) = 12 possible outcomes, there is no
reason to believe that any one ball is more likely to be
chosen than any other.
(b) "On an average of 1 in ten" means literally that of
every ten cases studied one and only one has the characteris­
tic in question. However, this does not mean that if we per­
form 40 such experiments in pra.ctice there will always be 4
(a.nd only 4) with the required characteristic; theoretically
we expect 4 such cases in every 40, and so on.
Similarly, if we say that 3% ofT.V. components on an
assembly line are defective, then we take the theoretical
view that of every 100 components produced, then 3 (and only
3) will be defective.
(c) Many errors are made in the work on probability by
asswning, without sufficient thought, that the outcomes of an
experiment are equally likely.
Thus (a) if I played chess with you, there are only 3
possible results - win, loss or draw. However, we
cannot assume these outcomes. are equally likely, be­
cause of lack of information on your and my chess
ability.
(a) if two coins are tossed simultaneously, what array
would be most likely: 2 heads, a head and a tail,
2 tails? Do you say "All equally likely"? So did
D'Alembert in 1754, when the proposition was given
to him. However, he was incorrect since if the
possible outco�es are listed: Coin 1 H HTT
Coin 2 HT HT ,
there are four (not three) equally likely cases, and
thus a "head and a tail" (on either coin) would
theoretically be twice as likely to appear as
2 heads or as 2 tails.

(iii) SIMPLE AND COMPOSITE EVENTS; SAMPLE SP�


(a) Consider the random experiment of tossing a die and
noting the .number of spots on the uppermost face.
We know that the possible outcomes show 1· spot, 2 spots,
3 spots, ••• , or 6 spots. These six results are usually re­
presented by the integers 1,2,3,4,5,6 respectively, and are
called the simple events of the experiment. These outcomes
are often written in set notation as S = {1,2,3,4,5,6}, where
the collection (SJ of all possible simple events is aalled
302
SAMPLE SPACE
the sample space of the experiment. [Note the order of
appearance of elements in the set S is i1rJ11aterial. Sets are
equal if they contain the same elements, in any order. Thus
{S,4,1,3,6,2} is equal to the set S.]
Any result consisting of more than one simple result or
event, of a random experiment is said to be a composite re­
sult or event. Thus, in the above experiment with the die,
the. composite result (or event) of the number of spots on the
upper face being odd consists of the three simple results (or
events); namely l�ot, 3 spots or 5 spots. In set notation,
the possible outcomes for this composite event (C say) would
be written as C = {1,3,S}. Similarly, the composite event
(E say) of the ntQ'llber of spots on the upperrnost face being a
multiple of 3 would be denoted by E = {3,6}. [Note: events
are often represented by capitals C,E,A, •.• which are not
pronumerals; in this context, .the capitals represent state­
ments. J
An 11event 11, then, is a subset of the sample space. To
say that an event A hap pens, is to say that the result of the
random experiment is an element of A.
Thus, the event of obtaining a number on the die above
greater than 2 is the subset F = {3,4,5,6} of the sample
space S = {1,2,3,4,5,6}, since the elements of this subset F
are exactly those and only those elements of S which -satisfy
the given condition, namely, that the number on the die is
greater than 2. Similarly, the event G of getting an even
number is the subset G,= {2,4,6}, since the elements of G
are the only elements of the sample space S which .obey the
given condition, namely that the number is even.

w
(b) The sample space, of course, depends on the experiment
be.·ing performed; the wording of the experiment must be
studied carefully, as too often mistakes are made due to
hasty or incorrect interpretation of the experiment.
Thus, consider the selection of balls from an urn
containing 1 black (B) ball, 1 red (R) ball, 1 white
(W) ball.
Expei•iment 1. Select 1 ball. Consider the colour of the
ball. The sample space, S1, here is given by S1�{B,R,W}.
ExperimP-nt 2. Select 1 ball, (note the aolour), then select
another ball (note the colour again). Consider the
colours of the 2 balls. The sample space,
S2 = {BR.BW,RB,RW,WB,WR}, since BR RB WB the order of
BW RW WR
the colours is of importance here.
Experiment J. Select 1 ball, (note the colour), replaae the
batl; mi:r: the balls, seleat another ball (note the
aolour). Consider the colours of the 2 balls. The
Sample space, S3 a {BB,BR,BW,RB,RR,RW,WB,WR,WW}, since
303
RANDOM EXPERIMENTS
again t!1e order> of the colours is vital. BB RB WB
BR RR WR
BW RW WW
?xver>iment 4. Select 2 balls simultaneously. Consider> the
colours of the 2 balls. TI1e. sample space, S4={BR,BW,RW},
since the order of colours on the balls is not import-
ant here.
r.:xµer>imont 5. Dclect 3 balls simultaneously; consider the
colourn of the 3 balls. The sample space, s5 = {B R W},
si�ce only one outcome is possible. (Order of colour­
ing of the balls is not vital here.)

(iv) RANDOM VARIAULE

The idea of a rw1.dom variable is essential in any course


on probability.
Consider a c:ertain random experiment() in which there is
a finite number of. possible outcomes. If X is used to repre­
sent the result of a single J:appening (called a trial) of Q,
then X is called the random variable for the experiment. Now
if there are n possible values of X (X may, of course, take
the results x, ,x 2 ,... ,xn at random) the complete set of these
values is called the sam;:,Ze space of the experiment and is
denoted by S = {x1,x2,x3, ••.,xn} where x1,x2,••••xn are the
possible values of X.
Thus if X represents the number of spots on the upper­
most face of the die when it is tossed, then X may take the
values 1,2,3,4,5,6 at random.
S = (1,2,3,4,5,6} is called the smnple space for the random
variable X. By this, we mean that the outcome (or result) of
a single experiment (i.e. tossing tile die) must be one of the
elements of S.
Similarly, let Y represent the order of colours on two
balls selected one at a time, with replacement after the first
selection, from an urn containing 1 black (R) b�ll, 1 red (R)
ball, 1 white (h') ball. Then, from exrer-irnent 3 a!:>ove, B
Y may take the values BB,BR,BW,RB,RR,RW,WB,WR,WW at lR

random. s3 {BB,BR,BW,RB,RR,RW,WB,WR,WW} is the 1W
sample space for the random variable Y.

(v) GE0:1ETRIC REPRESENTATION OF SA:1PLE SPACES

It is often convenient to think of the elements of a


sample space as points (often called sOJT1ple points). We can
thus represent them Jeometrically (i.e. graphically) on a
diagrOJT1, either as points along a number line or as points on
a plane, using rectangular> axes.
Thus, consider the following cases:
(Cl) For the experiment ,Jith the die, where the sample space
S = {1,2,3,4,5,6}, we may simply take the real number

304
SAMPLE SPACE
line, and circle the points (i.e. numbers) belong-
ing to S. -- 4 +-
� �0 �1--�2--�3--�4�5�•6�7 ��-'x
��--1
If we wished to shOJJ geometriaaZZy the event (CJ of ob­
taining an odd number on a throw of the die, we would
colour in, or shade in, or put a cross in (or in some
way, mark) the appropriate circles on the diagram above.
The graph of C
is shown.

(S) Let two diae (referred to as say X, Y) be tossed sunuZ-


taneously, and the experuent aonsist of noting the
pairings on the uppermost faaes. The sample space here
could be written down thus:-
1 1 2 1 3 1 4 1 5 1 6 1
1 2 2 2 3 2 4 2 5 2 6 2
1 3 2 3 3 3 4 3 5 3 6 3
1 4 2 4 3 4 4 4 5 4 6 4
1 5 2 5 3 5 4 5 5 5 6 5
1 6 2 6 3 6 4 6 5 6 6 6
in set notation, (where the ordered coordinates re­
OR,
present the numbers uppermost on the two dice, X, Y
respectively) as
S = {(1,1),(1,2),(1,3), •• ,(1,6),(2,1),.. ,(2,6),...,(f\1), ••,( 6,6)}
OR graphiaaZZy, as shown. dAiY6 (B • 0 0 0 0
The point (3,2) say, marked®,
represents the result of a 3 5 EB O O O 0 e
appearing on die X and a 2 ap- 4 ij) O O O 0 e
pearing on die Y. The ordered
pair (2, 3). marked®. repre- $ O O • 0 e
sents, of course, a different
outcome to that represented by 2 $ 0 ® 0 0 •
the pair (3,2 ).
0 $ $ $ EB (9
If the event A specifies cfu.X
the event t!1at the sum of the O 1 4
numbers on the upper faces is equal to 8, then the elements
of A are {(2 ,6),(3,S),(4,4),(S,3),(6 ,2 )}. These are denoted
graphically thus, e ,
on the diagram.
Similarly if the event B denotes the appearance of a
1 on either die X or die Y, but not both, then the elements
of B are {(1,2),(1,3),(1,4),(l,S),(1,6),( 2 ,1),(3,1),(4,1),
They are represented thusEfjon the sketch. (S,1),(6,1)}

(y) Consider the 4 aards: aae, king, queen, jaak of diamonds.


Let two suaae6sive dr(])J)s be made, without repZaaement
after. the first dra;..J. The experuent aonsists of noting
the faae values of the 2 aards in order.
AK KA QA JA
The sample space mav be obtained thus: AQ KQ QK JK
AJ KJ QJ JQ
305
TREE OIAGRAMS
To represent this sample space graphically, we may de­
note the ace, king, queen, jack by 1, 2, 3, 4 respectively.
Thus the ordered pair (1,4) on ti1e x - y :,lane represents the
outcom,': king on the first draw followed by jack on the
second draw. Secor,d' 0 0 @
Card 4-
Note that here the pairs (1,1),
(2,2), (3,3), (4,4) are meaningless; 3 @ @ @
they do not belong to the sample
space of the experiment. If E de- 2 0 !Z\) 0
note the event that a queen appears
either on the first or second card, 1 @ O
then the graphical representation of
E is shown thus @ Fi-st
O

3
(al It is becausu of this geometric representation of
0 4Cord

the po:;s i b I e outcom8s of random ex perimen ts that the


set of possible outcomes is called a sample spaoe.

A very useful method of determining sample spaces for


various experiments (especially repeated drawings, with or
witl1out replacements, and repe?ted operations, tossing coins,
etc.), is that involvinr, what is known as a tree diag ram.
[Tree diagrams will be used extensively later to determine
the probabilities of various occurrences.)
----------------
(ex) F.xperiment 1. In an urn are ,2 green (G) lla 11 s and
3 yellow (Y) balls. If 3 draws are made, each
2
without rt?placement, and the colours noted on y
the halls, find the sample space S. 1 3 GI
The sample space may be written down by trail fairly
readily, but because of the application later to probabili­
Method.
ties, the tree method is sh01m.
First Dra,.,,

Start
I

�·
y

Step 1 incl.icates In the second step, In step 3, along the


the selection of the various possibil­ top branch only
lG or lY ball, ities are shown. G-. G -.y exists
since there are only
.,,.
306
4,,.,._;.
TPEE DIAGRAMS
The tree may be set out in various ways
(i) OR (ii)----------
GGY Y
GYG
GYY
Start y
YGG
YGY
YYG
yyy

Also, the tree can be put as in (ii) but upside down.


Thus the sample space S = {GGY,GYG,GYY,YGG,YGY,YYG,YYY} fol­
lowing along the various branches.
[Note (i) such a sample space can only be represented geomet­
rically in a 3-dimensional coordinate system;
(ii) if 4 draws had been made in this experiment, com­
plete the "top:; of the tree and verify that the
sample space S1 = {GGYY,GYGY,GYYG,GYYY,YGGY,YGYG,
YGYY,YYGG,YYGY,YYYG}].

(SJ Experiment 2. In a quiz, you are given two multi.pie


choice questions, one with three possible answers
A,B,C and the other with five possible answers
a,b,c,d,e. Assuming you do not know the answers and
decide to write down al I pairings on separate pieces
of paper, what are.al I these pairings?
On a tree diagram, the pairings appear

The sample space (i.e. the


pairings) is
{Aa,Ab,Ac,Ad,Ae,Ba,Bb,Bc,
Bd,Be,Ca,Cb,Cc,Cd,Ce}

307
DEFINITION OF �ROBABILITY
S,, THE PROBABILITY OF AN EVENT

( i) OEF.INITICN Let us assume that we have a random ex-


periment E in which there are a finite number (cl of
possible simple events, i.e. there are "c-" elements
in the sampI e space of E. Of these "c" events,
assume that the number of simple events comprising a
given event A is "a", i.e. there are "a" elements in
A.
The pr>ohal>iZity of the given event A is defined
to be the r>atio ale, and is denoted P(A) = a/c.

For> example, 1uhat is the pr>obahiZity of tossing a nwnber>


gr>eater> than 4 in a single toss of a die?
Here, the simple ev e nts are the rP.s ults 1,2,3,4,5,6 on

rr�. ri-� r
the uppermost face , that is, there are six simple eve nts,
i.e, c = 6. ·For the ev e nt A of toss · ing a numbe r greate r than

;1m��= :;:�!:.
events a e t r i

1 l:J
6
s 5

i.e. a = 2; := r. rr� i� ·• ·· "•
.. D·. . .·
Here the sample
s pace A=
S = {1,2,3,4,5,6} D·
·
and A= {5,6}. Now the probability of A occurring, written in
probability notation as P(A), is by definition, given by the
ratio 2/6 1/3, i.e . R._®__:'__ill, '.This probability
is often d enote d by .the small ·letter p, i.e. here p 1/3]

(ii) Notes In the above d efinition, there were c simple


eve nts for the e xperiment E, i.e. S contains c elements ,
x, ,x1 ,x3, • • • , Xe say.
(ex) If the event A comprises aU of these events, i.e. the
se ts A, S are e qual, then P(A)= c/c = 1. [Here p= 1]
It is obvious that the prohabili.ty of an event cannot be
greate r than unity. In this case, if P(A) = 1, then 1,1e
say that the event A is aer>tain.
Thus, if a coin is tosse d, the probability of the event
A that either a he ad or a tail will appear is 1. Here
the s ample space S= {H,T} whilst A = {H,T} and hence
s ince A, S both contain th e same 2 ele ments, the proba­
bility of A, denoted by P(A) = 2/2= 1, It is obvious
that A is certain to occur .

(8) If the eve nt B comprise s none of the events of S, i.e.


if the sets B, S are disjoint (i.e . have no e lements in
common), the n the probability of B is 0/c= 0, It is ob­
vious that the probability of an ev e nt cannot be less
than ze ro. If P(B) = o, 1Je say the event B if! imposs­
ible.
Thus, if a coin is tossed, th e probability of the e v ent

308
DEFINITION

B that a six appears on the co.in is obviously 0, Here


the sample space S = {P., 1 } whilst B = {6}. Hence since
B, Shave no elements in common, the probability of B,
denoted by P(B) = 0/2 = O.

Thus, from (a), (8) above, the range of the proba­


bility of an event A is defined by O � P(Al � I.

(y) Other non-mathematical examples of the above are


for p 1, probability of (i) death (ii) day following night,
for p = O, probability of (i) swiouning ·.the Pacific Ocean
(ii) flying (without aids)
(6) If the sample space Sfor a random experiment consists
of the n simple events F.1,E2,E3,.,.,En
then S= {E1,E2,E3, ••, , Fn}.
Now it is obvious that
(i) P(Ei) for each value i (where i = 1,2,3,.•• ,n) is
such tha.t O S. P(Ei) S. 1
n
and (ii) E P(Ei) = P(E1) + P(E2) + ... + P(E n) 1,
i=l

(£) For convenience, various notations are introduced in


problems.
(i) Thus, in the case of a single die, we may ask for the
probability that there appears on the upper face an odd
number greater than three. This. could be asked as
P (odd number > 3).
(ii) Similarly, if two dice are thrown simultaneously, w.e
may want the probability that on the first die appears a
number less than or equal to 2 and on the second die an
even number. Here, we may word the problem as follows:
Let X, Y te,)resent the values of the random variable for
die 1, die 2 respectively, (i.e. X may assume the values
1,2,3,4,5,6 at random; similarly for Y). Our question
could be asked as P (XS. 2 and Y is even).

(i;) (a) The word "chance" is often used instead of "prob-


ability". Thus the chance that a coin will fall
heads is !,.
(b) The phrases· "odds. on" or "odds against" are often
used in courses on probability. Thus if the "odds
are 7 to 4 on II an event A happening, this indicates
that the chance or probability that A occurs is
7 = 7, i.e. P(A) = 7. [This means that if the
7+4 11 11
sample space of the experiment consists of (7+4) = 11
simple events, then the event A comprises 7 of these
11 simple events].
Similarly, if the "odds are 7 to 4 against" an
event B happening, this indicates ti1at the

309
EXAMPLES ON DEFINITION
4 �
priobabi Zi ty that B OOOU1'8 U is
7+4

i.P., P(J) .. 4/11.
[This means t;1at the event B com::,rises cnly 4 of the
(7+4) • 11 simple events which constitute the
sample sp ace of the experiment.]
Thus a bookmaker who quotes a horse C at "3 to 2 on"
se.ys in effect that the chance that C' wins is

5;
3 3
a whereas if he quotes a horse D at
3+2
3 to 2 against", he considers the probabilit y of D
winning is
2
3+2
s s·2
EXAMPLES ON THE DEFINITION OF PROBABILITY
r;:ote: EaCi\ of t:1e following examples should be carefully
wor�ed by you, They illustrate the previous theory,
and emphasise the methods of approach in this section]

EXAMPLt I. A card is chosen at random from the 13 cards


comprising the suit of hearts (from a regular pack).
Its face value is noted.

��I Li]� [iJ [J [J


/'l'J
V
fl

C:tO
'"
•.•
• •
•••
••• GTI-
•:•
•••
[Ja
!"!
•••
[J EJo
:::
•••
: .. :
... ,

1!1
f#
ti}·
'tY
f1
�<
I!/
..

(i) Si:ow that t;1e sample S, for the experiment is given !Jy
31 ={l,2,3,4,5,G,7,8,9,10,ll,12,13}, where the face
values 1,11,12,13 are allotted here (for the purposes
of this example) to the cards ace (A), jack (J), queen
(0), king (K),
(ii) The events B,C,D,E,F,G consist of,choosing a card whose
face value is for B: less than 5; for C: a multiple
of 4; for D: �n even number between 2 and 5;
for_L: prime 01' exceeds lC; [or F: prime and exceeds 10
for G: divisible by 17,
Show ti1at the corresponding sets are: B {1,2,3,4);
C =
{4,8,12); D {4}; E =
{2,3,5,7,11 ,12,13}; =
F = {11,13); G = { } or�
rote (a) that the number 1 is not consi,:!ered to be a
p_rine, l.Jecc>.use of its unique property that it
divides exactly into ever"J integer.
(8) the terms such as "x or y" mean •·either x o r
y o r both;i.

(iii) Assuming the simple events of S are equally likely,


i.e. that they have equal measures of probability [it
is natural here to assUl!le P(l)aP(2)=•••=P(13) 1/13], =
explain why the p robability of the event l3 happening is
4/13. Verify that P(C) = 3/13; P(D) = 1/13; P(E) = 7/13;
P(F) = 2/13; P(G) O. =
EXAMPLES

------------··-�-. . .----
(iv) Show that. the geometric representation of event E
can be illustrated thus:

-1 o 1 2 3 � 5 6 1 a 9 10 11 12 13 14
Similarly, represent events ll, F, G.

(v) Let X represent the face value of the card chosen, i.e.
the sample space S 1 al: ove is the sample s;:-7aae for the
random vo.riable X; determine the appropriate sets in
eac:1 of the following, and hence find the required
prob.shilities.
(a) ?(X is odd) (b) P(X • 6) (c) P(X • 2,3 or 11)
(d) P(X is a multiple of 5) (e) P(;t is divisible by 3)
(f) P(X is less than or equal to 8) (g) P(X is even or odd)
(h) P(X is even and odd) (i) P(X is uetween 2 and 11)
(j) P(X is odd or exceeds 6) (k) P(X is odd ar.d exceeds 6)
(!) P(X is prime) (m) P(X is even or 7) (n) P(X is an odd
number divisible ty 2) (o) P(X is e perfect square)
(p) P(X is not 8) (q) P(X is not even and not 3 or 7 )
(r) P(X 2 � 81) (s) P(X 3 < 64) (t) P(X+4=10) (u) P(X+4 ,; 10)
(v) P(X-4 = 10) (w) ?(IX-71 = 3) (x) P(IX-71 < 3)
(y) P(lx-71 ?. 3) (z) P( l2X+31 s 5)

EXAMPLE 2. nly the 5 cards (ace,2,3,4,5) are retained


from the suit of 13 hearts (of
��u][;Jfsl
2 3 4
examrle I above). Tvio cards _ _: ; g; �
are drawn in succession, (the
first card being replaaed after being dravm). This ex­
periment consists of noting the face value of each card
drawn.

(i) Show that the sample space S2 for the experiment con­
sists of the 25 simple events which can 1:,e repres:?nte d
as ordered pairs thus:
S2 = {(1,1),(1,2), ••• ,(1,5),(2,1).•.,(2,5),(3,1), •..
...,(5,5)}
i.e. if X,Ydenote the face values of the cards, then
the ordered pair (X,Y) represents a face value of X on
the first card and a face value of Yon the second card.
'1
0
(ii) Copy and complete the geometric re­ 0

presentation of the sample space for 4


this experiment.
(iii) Shade in, (in different markings),
the events
(a) I:'.: the sum of the face values is 6 o o o
(8) !: each face value exceeds 3 x
(y) L: the first face value is a 2 or 5 o ,. l • �
(6) :-·:: the first face value is odd and the second face value
is even
(iv) tfoat are the corresponding sets for E, I,L,M?

311
EXAMPLES
(v) Detennine the values of P(H); P(I); P(L); P(M)

t (vi) Calculate the appropriate sets in each of the following


(the sample spaae sketahed al:ove is Vel'"J useful here).
Henae ol--tain the required protiabilities
(a) P(X=3 and Y=2) (b) P(X=3) (c) P(Y=4 or S) (d) P(X � 2)
(e) P(Y<3) (f) P(X=3 or Y=2) (g) P(X>3 and Y=2)
('.;) P(X<4 and Y<3) (i) P(X<4 or Y<3) (.;) P(X+Y = S)
(k) P(X+Y 'f 5) (i) P(X+Y>S) (m) P(X+Y � 5) (n) P(X 'f 3)
(o) P(X � Y) (p) P(i=S and Y is even) (q) P(X is even and
Y is odd) (r) P(X and Y are both even)
(s) P(X and Y are bot!1 odd) (t) P(X is prir.:e)
(u) P(X is odd or Y is even) (v) P(X is odd or Y is odd)
(u) P(X � 3 and Y � 2) (x) P(X�3 or Y�2)

EXA�PLE 3. The 5 cards of example 2 are used here. Two


cards are drawn in succession,
without replacement of the
first card. The experiment 1YJ &J �
[Al[2lf3114lj5l
l..!_I L�
consist� of notin the face value of each card drawn.
(i) Shew that the sample space s 3 here consists of the
sample space S2 of exaraple 2 less the events (1,1),
(2,2), (3,3)' (4,4), (S,S).
0
(ii) Copy and complete the graph-
ical reeresentation of the 4 0
sample space for this exper- 3
iment. 2 0 0 0
(iii), (iv), (v) Complete the 0 0
question as given in example
2 above, parts (iii), (iv), (v), 2. 3 4 5 X
(vi).
EXAMPLE 4. In an urn are 4 marbles indistinguish­ I\•/
able except for colour; one is white (WI, another
IB
b Iack (GI, another red IR I, and the other is
IR
ye 11 ow (YI. IY

Emeriment 1 You seleat one marble at randon: from


the urn and note the aolour.
(i) :-;ive the sample space S for this experiment, and state
the value.of (a) P0n (�) P(Y) (c) P(W or Y)
(d) P(R or Y or B) (e) P(R or Y or B or W)
(f) P(a green marble).
[Here: the notation P(W) refers to the probability of
selectitg a white marble.]
(ii) Show S on a diagram, (using the numbers 1,2,3,4 to re­
fer to W,B,R,Y).

312
EXAMPLES
Experiment 2. You select two ma:L'bles in succession
from the urn, with replacement after the first
marble is d:rai,m, and note the colours of the 2
ma:l'bles.
(i) For this experiment, show the sample space Sin (a) set
notation (b) on a graph,
(ii) Hence determine the numerical values of
(a) P(WW) (b) ,P(RW) (c) P(BY or YB) (d) P(YW or YR or YY)
(e) P(one of the marbles is white, but not both)
(f) P(one of the marbles is red and the other is yellow)
(g) P(the first marble is red)
(h) P(the first marble is not white)
(i) P(the first marble is black and the second marble is red)
(j) P(both marbles are the same colour)
(k) 'P(at least one marble is yellow)

Experiment J. Repeat experiment 2 !Jut without


replacement.

Experiment 4. You select two marbles simultane­


ously out of the urn. You are concerned with the
colours of the marbles, but not in the order of
these colours i.e. ,IB, Efv are identical, etc.

(i) For this experiment, show the sample space in set nota­
tion,
(ii) Determine the value of the following
(a) P(one of the marbles is black) (b) P(the marbles are
red and yellow) (c) P(neither marble is white)
(d) P(neither marble is black or red)

EXAMPLE 5. A married couple plan a family of three child­


ren. Assume that the result of a birth is as I ikely to
be a boy (B) as a girl (G).

(i) Obtain (for yourself)


the tree diagram
shown, making certain BGB
you understand each BGG
step. GBB
GBG
(ii) Write out tile sample GGB
space for this "fam­ GGG
ily tree", and
determine the proba-
bilities listed below, assu�ing
°
that each of the
"simple events" (BBB etc) are equally likely to occur,
[Here notation such as P(BBG) means the probability of
having the 3 children in the order specified boy, boy,
girl.)
(a) P(BBB) (b) P(GGB) (c) P(BGB) (d) P(three girls)
(e) P(two boys and a girl, in any order)

313
EXAMPLES
(f) P(two girls and a boy in any order) (g) P(either three
boys or three girls) (h) P(two boys, then a girl)
(i) P(at least one boy) (j) P(at least 2 girls)
(k) P(exactly one girl) (!) P(exactly 2 boys) (m) P(at most
1 girl) (n) P(more boys than girls) (o) P(at least 1 of
each sex),
(iii) Had the couple planned for a family of four children,
oanpZete the new "family tree", and hence determine
(a) P(BBBB) (b) P(BGBB) (c) P(BGGG) (d) P(four girls)
(e) P(three boys and a girl, in any order) (f) P(three boys,
then a girl) (g) P(two boys followed by two girls)
(h) P(two boys and two girls) (i) P(three girls and a boy)
(j) P(three girls, followed by a boy) (k) P(either all boys
or all girls)
(!) P(boys and girls alternate, either first)
What is the most likely family (irrespective of order)
that the couple may have?

EXAMPLE 6.
(i) Not deterred by the fate of Cheva I ier de Mere I
I "challenge" you to a game of dice. We throw
two dice and consider the sum of the numbers
on the two faces. · We each se I ect a number
between 2 and 12, i.e. a possible sum for the
two dice. I decide on 7, you decide on 9?
You "pay" me $ I for each occasion a tota I of 7
is obtained and I "pay" you $1 tor each occa­
sion the total is 9.
TheoretioaZZyi who should win?
(a) List the sample space here in some systematic way.
(b) Show that P(T = 7), {i.e. the probability that the
total is 7} is 6/36 • 1/6 and find P(T = 9).
· Assuming that we eaoh throw 36 times, what should be
the final reckoning (in dollars).
(c) List the totals 2 to 12 with their respective probabil­
ities.

(ii) NOlJ let us toss 3 dioe. If you can determine


correctly the probability of obtaining a
total of 12 with these dice, I shal I return
the wager you "owe" me from part (i).
(a) How many possible outcomes are there? Do not list them,
(b) Canplete the list, (begin at the 651 561 462
right) for the event T • 12, and 642 552
hence find P(T = 12). Are we now
"square"?
(c) Similarly, determine the value
of P(T "' 13).
314
EXERCISES SET 7A
*(iii) Show that the probabi I ities of throwing
a total of six with 2, 3 or 4 dice
are in the ratio 18: 6:

EXERCISES SET 7A
1. (i) A coin is tossed once. What is the sample space in
(a) set notation, using H, T to represent head,
tail respectively
(6) a diagram, using 1, 2 for H, T.
What is (a) P(H) (b) P(H or T)

(ii) The coin is tossed twice. State the sample space


in (a) set notation and (6) graphically, (Verify
that the sample points form the vertices of a
square).
What is (a) P(TH) (b) P(TT) (c) P(two heads)
(d) P(a head and a tail) (e) P(a head followed by
a tail) (f) P(two of a kind) (g) P(at least one
head) (h) P(no head on either toss)

(iii) The coin is tossed three times.


(a) Use a tree diagram to detennine the sample
space. How many simple events are there?
*(6) (Level 2F students only). Show the sample
space on a diagram using x, y, z coordinates
(i.e. in 3 dimensions), and verify that the
sample points are the 8 vertices of a cube, of
unit side.
(Y) Find the probabilities of the following events
(a) head, tail, head in this order
(b) three tails (c) two heads, then a tail
(d) two heads and a tail
(e) 3 heads or 3 tails (f) at least 1 head
(g) at least two tails (h) at most 1 tail
(i) exactly two heads.

(iv) The coin is tossed foU1' times


(a) Complete the tree diagram of part (iii). How
many simple events are there?
(6) Determine the probabilities of the following
events
(a) four tails (b) exactly 3 heads
(c) at least 3 tails (d) at most l tail
(e)·more heads than tails (f) equal numbers
of heads and tails.

2. A number is selected at random from the numbers 1,2,3,4,


S,6,7,8,9,10. Find the probability of choosing
(a) 7 (b) an even number (c) a number gre�ter than 6
(d) a prime number (e) a number divisible by 5
(f) a perfect square (g) a rational nlDitber

315
EXERCISES
(h) an irrational number (i) 4 or 9 (j) the number If
3. From the English Alphabet, a letter is selected at
random. What is the chance of drawing
(a) the letter "p" (b) either the letter "x" or "y" or
"z" (c) a vowel (y is not.considered as a vowel)
(d) one of the letters of the word "party".
(e) the letter e.
4. There are 6 marbles in a bag; 1 red, 1 white, 1 blue,
1 green, 1 pink, 1 yellow marble. Find the probability
that if a marble is drawn at random, then it is
(a) red (b) white or yellow (c) not blue (d) not red
nor white nor blue.
5. From a regular pack of 52 cards, 1 card is drawn at
random. Determine the probabilities listed
(a) P(it is a heart) (b) P(it is a club or a diamond)
(c) P(it is an ace) (d) P(it is red) (e) P(it is red
or black) (f) P(it is red and black) (g) P(it is
the three of spades) (h) P(it is less than 10; ace
considered as 1) (i) P(it is a picture card)
(j) P(it is a black 7 or a red 8).

6. "Random digits" are sets of digits (i.e. 0,1,2, , , , ,9)


compiled at random. These are useful in more advanced
work on probability. (If you were asked to compile a set
of 1000 digits at random, you would also certainly find
that each digit did not occur 1/10 of 1000, i.e. 100
times. This is because, unconsciously, you have "favour­
ite" digits which you keep using.) Electronic devices,
known as random digit generators, are used by statistic­
ians to obtain sets of random digits, (which are also
available in sets of tables in more advanced works on
probability).
A particular generator produces two digits in such a way
that all cases OO,Ol,•.. ,09,10,11,•••,19,20,.•• ,99 are
equally likely. Taking 00,01,..,09 to denote 0,1,2, ..,9
respectively; find .the probability that a certain number
produced (which must lie between O and 99)
(a) is divisible by 12 (b) is not a muitiple of 10
(c) has both digits the same (d) is less than 30
(e) is greater than or equal to 92 (f) is such that its
cube < 200 (g) is the root of the equation 2x+7=20
(h) is a perfect square (i) is prime and less than 30
(j) is odd a�d between 50 and 60 (k) will end in 9
(!) is 91 or 92 (m) is not greater than 70,
7, A red and a yellow die are tossed simultaneously, Let
X, Y be the number of spots on the uppermost faces of
the red and yellow respectively,
(i) Graph the sample space, and indicate on it, the
following events
(a) ! : the sum is 6 (b) � : the sum is even
316
ALGEBRA OF EVENTS
(c) .9.: the numbers on each die are both prime
(d) D: X = 3 or Y 5 2. Hence find P(A); P(B);
P(C); P(D)
t(ii) Determine (using the graph to aid in the work),
(a) P(X=2) (b) P(X=S and Y=2) (c) P(X=S or Y=2)
(d) P(X > 3) (e) P(X > 3 and Y > 4) (f) P(X>3 or Y>4)
(g) P(X 5 3) (h) P(X is odd) (i) P(X and Y are. both
odd) (j) P(X or Y is odd) (k) P(X is odd and Y is
even) (.f.) P(X is odd and Y = 2) (m) P(X = Y)
(n) P(X -, Y) (o) P(X < Y) (p) P(X .� Y) (q) P(X � 2 and
Y.:, 5) (r) P(X+Y>B) (s) P(X exceeds Y by 2)
(t) P( Ixi = 4) (u) P( IYI " 4) (v) P(l xl > 1)
(w) P( lx-41 = 1) (x) P( lx+31 � 2) (y) P(X 2 -3X+2 = O)
(z) P(lx+ll .!: 4 and IY-21 � 1)

t(iii) What is the probability that


(a) the score is not 9 (b) only 2 or 5 or both
appear (c) neither a 2 nor 5 appear
(d) one dice shows at least 2 more than the other.

8. In a bag are 4 black balls and 3 white balls. Two balls


are drawn in succession. Denoting the black balls by B1,
B2,B3 ,B4 and the white balls by H1,!?2,W3; graph the
se.mple space assuming that the balls are drawn
(i) 1,,ithout replacement (ii) with replacement
Hence for both (i) and (ii), determine the probabilities
of the following events (a) One, or both, of
the balls is black (b) Both balls are white
(c) The first ball drawn is black (d) The second ball
is either black or white (e) At least one of the
balls is white (f) Both balls are the same colour
(g) The balls are different in colour

D. FURTHER TERtvlS; THE ALGEBRA OF EVENTS


(i) T'ae application of tr.e definition of probability, as out­
lined in the previous section, has obvious limitations
when the sample space is large or more complex. This
present section is devoted to introducing further terms
and results, which will make the calculation of ;,rob­
abilities less arduous (in the a·ppropriate circumstances
of course).
(ii) As we have already_ seen, the theory of probability is
closely associated with the theory of sets. This assoc­
iation will be continued further here.
(a) Let Ebe a given set, with 1T
elements e1,e3,e4,e7, and
U the universal set, with
elements e1,e2 ,e3,e4,e5,eG,
e7. This may be
317
SETS AND EVENTS
represented graphically e.s shown.
elements not in S, is caZZed. the CQ!!:PZement of the set !:,
Then, tr.e set of

and i8 denoted in val'ioue ways as E, -ri: r: ', U11e shalZ

'.:'or:respondinG to fhe set E, in probability theory, we


use E].
aPGooiate the event D, w!1ose simpZe events are ei,e3;ei+,
e7, To correspond to t:ie universaZ set U, we associate
the sanrpZe space f, comprheci of the simple events
e1,e2,e3,e1+,es,eG,e7,
A diagr�umatic representatior. of s
this •.;ould be identicaZ to the
set diagrarr: above,
Then, the compZement of the

do not beZong to I::. (Tltis complement is denoted again


event E, consists of aU the simple events in ,': !,.,hich

by r., 'E, E'; but here we use E), [E is the event that "E
does not occur"; often referred to c:s the opposite or
contrary event to L]

t:01,.,, by observation, from the diagrams,


(a) the number of elements in set E is sfofly t!1e number of
elements in set U less the number of elements in set E.
the number of simple events in event E is the number of
simple events in S less tiie number of sir.1ple events in
(3)

event E.

B (the certain event) Zess the procability of the event f,


Hence, the probol;iZity of event!' is the probal:ility of

i.e. P(E) = P(5) - P(r:) i.e. P(E) = 1 - P(E)


Thus, if a die is thrcrwn, and is the event of obtaining a
3, t:,en E is the event of not obtaining a 3.
Since P(n = 1/6, • ·�(E) = 1 - 1/6 = 5/6.
Or, in a family of 2 children, if A is ·t:,e event ti,at both
childi•en are i.>oys, t;1en A is the event that ,�oth chiZdren are
no_! boys, i.e. that the children are ooth girls or a boy and
a girl,
=
[�ote, of course, that P(S) = 1 - P(S) = O, arid constitutes
I-�ere t>(A) 1/4, . '. P(I) ·= 1 - 1/4 3/4

the ;:irobability of S, i.e. of the ir.1possil:le event.]

(h) The aseooiation of set, and events (i)


is now carried further.
Let A,� be two distinct sets. Tjen,rel­
I I
O [:)
ative to one another,
either (i) A, B have no elements in
common. {They are then called
(ii) (a)
disjoint sets.}
or (ii) ft, B have some eZement(s) in
common. [If every element of
A is contained in B, or vice �
318
SETS
versa, then A is a subset of n.J (ii) (b)
These cases are illustrated the
diagrams.
Further, the notation AU B, �
(also written often as A+D) means the
set of elements, eaah of whiah belong to set A or to set 2 or
to both A and B.
::>hilst, t:ie notation An ::, (also written ofte.n as A.B
or as A3), means the .;et of elements, eaah o.f whiah belongs
to both A and 3.
AU s·is called the W1ion and An B the_ interseation,
of sets A and R.
Diagrammatically, these would be shown as follows

l••ll• I fl
(i) (ii) (a) (b)

I o o I co I @
The aorrespondenae from the·algebra of sets to the alge­
bra of events,· carries on as did the idea of the complements
of a set carry over to the co�plement of an event.
[ere, corresponding to the sets A, B we have the events
A, B. Relative to one another,
either (i) the events A-and B have no
simple event in cor.unon;
i.e. no simple event be­
longs to both A and n, In
this·case, we call the
events disjoint or more commonly, mutually exalus­
ive. . The diagrorrmatia representation o-? mutuallu
exalusive events is identiaal to that of disjoint
sets. s
or (ii) the events A, !:l have 4
some simple event(s)
in common. In this
case, they can be
represented as in­
dicated in the sketch.

The notations AU� or A+ B, and An B or AB have the


same meanings with events as with sets. Tr.us A U B or A +' B;
the union or "swn" of two event.s A, B; represents the event
whiah aonsists of all the si.n;pte events that be forzg to A or
319
ALGEBRA OF EVENTS
to B or to both A and B. Whilst An B or AF; the intersection
or "product" of two event�; represents the event which
consists of all the simple events that belong to both A
and B.
More simply, A U B means the event "either> A O!' B
occurs". (Note,. if we say either A or Boccurs,
then we mean either event A occurs or event B
occurs, or both events A,Boccur.)
and A I\ B {or, AB) means the event "both A and B
oaaur, II
sketches above), are now shown.
(b)
s

AUB {e1,e2,e3, AUB = {e1,e2,e3,e4,e5} AUB = {e·1,e 2,e3,


e5,e7} e5,e7}

AB = AnB = ¢ AB=AnB={e2,e5}
ILLUSTRATIONS OF THE ABOVE IDEAS
EXAMPLE I. Consider> the tossing of a die. Let A be the

w
event that a 3 appears on the uppermost face; let B be
the event that the number is even; Iet C be the event
that we toss a number less than 5.
Here S = {1,2,3,4,5,6}, A = {3}, B = {2,4,6}, C = {1,2,3,4}
Now, (a) consider> the· events A, B. s
Since there are no simple events
I
CD A 2 4
in common, we say that A, B are 3 6
mutually exclusive events.
5

and AB = 0 and,'. P(AB} = 0


t °t
Her>e AUB = t2,3,4,6} and . '. P(AUB) = % = + = P(A}+P(B),

For> mutually exclusive events, these properties, namely


P(AUB) = P(Al+P(B) and P(AB) = 0, are always true. { In
many texts, mutually exclusive events A, Bare defined as
events which satisfy the result P(AB) o.}
If events A, B have no simple events in common, i.e. if
they are mutually exclusive, then these events cannot both
occur together> (i.e. at the one and the same time.) {In the
above example, we cannot obtain a 3, and obtain an even
320
EXAMPLES
number, at the same time. Aatually, if two (or more) events
are mutually e:x:alusive, then the oaaurrenae of any one of
them exaludes the possibility of the oaaurrenae of any one of
the others. Thus the event of horse P winning a race ex­
cludes the possibility of any other horse winning that race;
(no dead-heats allowed) ,
(�) Consider the events B, C (which are
not mutually exclusive, since the
simple events denoted by 2, 4 be-
long to both),
Here, BUC = tl,2,3,4,6! S
and •.• P(BUC) =
6 7
4
Now P(B) = 3 and P(C) = ,
3 4
6 6
P(B) + P(C) = 6 + 6 = 6.
Note three points here:
-- firstly, P(BUC) � P(B) + P(C)
2
secondly, since Bnc= t2,4l, P(BnC) i.e. P(BC) = 6

i
and thus P(BVC) = �= % + � - = P(B) + P(C) - P(BC_l
thirdly, the numerical value of [P(B) + P(C)] has no
further significance than that it represents the addition of
the probability of event B to the probability of event C.
For aZZ events B, C, the property above, nameI Y
P<BU Cl = P(B) + P(C) - P(BC) is always true. It is
cal led the theorem of total probability. In the ca se of
mutual exclusive events B, C, P(BC) = O, and the result
becomes PCBU C)= P(B) + P(Cl, · which has already been
noted.
()') �le can verify this result by considering
the events A, C r-������--,
s
Here A= l3!, • ' , P(A) 1/6, '
C = \l,2,3,4),. '. P(C) 4/6 (E)
. .
Now A U C = �1,2,3,4! . . i>(A U C) = 4/6;
A (\ C = l3L ... P(AC) = 1/6
Thus P(A U C)= !!.= 1 + !!. - 1= P(A) + P(C) - P(AC)
6 6 6 6
EXAMPLE 2. Consider a set of I 3 discs, each one with a
-number from to 13 inclus.ive marked on it, <The discs
are identical except for the number, and the same number
does not occur twice.) The discs are put in a bag and
one selected at random. What is the probabiI ity that it
bears an odd number or .that the number is reater than 4.
Method. (a) Using the definition of probability method
of tackling this problem, we first list the samp1e space
S = ll,2,3,4,5,6,7,8,9,10,ll,12,13}. Let A be the event
of drawing an odd number, .·.A= tl,3,5,7,9,11,13} and
B be the event of drawing a number greater than 4,

• • B = \5,6,7,8,9,10,11,12,13}.

321
EXAMPLES
Here we re9uire the event that "A or B occurs", i,e, AUB
Now AUB = ll,3,5,6,7,8,9,10,11,12,13},
and.·. P(A U B) 11/13,
(b) Using the Theorem of
Totai Probability,
namely P(AUB) .. P(A) + P(B) - P(AB)
Now P(A) = 7/13, P(B) = 9/13, and
since AnB = {5,7,9,U,13\, • '. P(AB) 5/13,
Thus P(AUB) =
7 9
+ l3 - l3
5 11 as before.
=
13 13'

EXAMPLE 3.From a pack of 52 cards, one card is


drawn at random. What is the probability that the
card is
(i) a ten or a picture card
(ii) a ten or a diamond
(iii) either less than six or� heart.
Method. (i) Let A, B be the respective events of dra�ing . a
ten, and a picture card, Now because A, B cannot both
occur together (i.e. we cannot
select a ten and select a s
picture card at the same time),
.·, the events A, B are mutu-
aUy exatusive.
Here, we w ant A or B to occur,
i.e. we want P(AUB). [We have the
option of listing the sample sp ace
or enough of it to grasp the problem, or proceeding by the
theorem above. J
Now, A = �lOH,lO D,10C,lOS1, i.e. ten of hearts, etc,
and B = \JH,QH,KH,JD,QD,KD,JC,QC,KC,JS,QS,KSl
then P(A) = 4/52 = 1/13, P(B) 12/52 3/13,
Since A , B are mutu ally exclusive, P(AUB) = P(A) + P(B),
and • ', P(AUB) 1/13 + 3/13 4/13

(ii) Let E be the event of dr awing a diamond, Here , we want


A or E to occur, i.e. we s neither tens nor
require P(A U E), diamonds
However,
L.J-..:;;..,-a..�-------­•·�...J.s
since the event of � _ :tw
dr awing a ten does not exclude ofl£�
the possibility of drawing a
diamond, the events A, E are not mutu ally exclusive. The
probability· law P(AUE) = P(A} + P(E) - P(AE) holds here,
Now P(A) = 4/52 = 1/13 from (i), and P(E) = 13/52 � 1/4,
(there are 13 diamonds ·1n a pack),
Here, we have to consider P(AE), Noting that AnE = \.lOD\
i.e, the ten of diamonds is the only event common to both A ,E ;
P(AE) 1/52.
Thus P(AUE) 4/52 + 13/52 - 1/52 4/13

322
FURTHER EXAMPLES
(iii) If the card is to be " either less than six or a
heart", then the card must be less than or equal to·5
and not a diamond, club or spade. Let F, G be the re-
spective events of selecting a car�S; and nota diamond
club or spade, then we requir/ P(Fl.JGJ, noting that the
events F, G are not mutuftlly exclusive.
Here F consists of 4 ,c. 5 = 20 simple events; (count these
without listing the sample space ; ace is counted as 1); and
G comprises 1 x 13 = ll simple events. Considering FAG, we
see that. there are 1 )( 5 = 5 simple events. (Check this I)
Thus using result P(FUG) P(F) + P(G) - P(FG),
we obtain · P(F U G) 'L.0/52 + J 3 /52 - 5 /52 = 28 /52

[Now aheak the results for (i), (ii), (iii) by using the
definition of probability.]

EXAMPLE 4. In a true - false quiz containing 6


questions, I put down true (Tl or false (F) at
random as the answer for each of the 6 questions.
\\Ihat is the probabi Iity that at least one of my
answers is correct?
Method. If I want at least one of my answers
Q, Q.
tobecorrect, then I want to o;,tain either 1
correct answer out of the 6, or 2 correct T? F?
answers, or 3 or 4 or 5 or (if I am "lucky"
enough?) 6 correct answers. To tackle this problem directly
would lead to awkward calculations and som� hard thinking. It
would be much easier to find out the chance of no �orrect
answers (i.e. the chance of all 6 answers being incorrect).
If we denote the event of all 6 answers being wr0:1g as E,
then the aomplement (or opposite) event to E, namel,y E denotes
the event that all 6 answers are not wrong, i. e. that some
answer or answers are aorreat.
Here, S consists of 2 = 64 simple events. (Reason this
b

out!), and the event E comprises one and only one simple
event (i.e. the set of 6 incorrect answers).
Hence P(E) 1/64.
Thus, since P(E) = 1 - P(E), by above,
.·. P(E) = 1 - 1/64 63/64.
Thus, the probability of obtaining at least one correct
answer among the 6 answers presented is 63/64. [Verify this
result by drawing a tree diagram to determine all. the poss­
ible outcomes, and assume, say that the correct set of
answers is F F T FT F.J

EXAMPLE 5. A class of 30 contains 19 persons who


watch cartoons on T.V. for relaxation, and 12
persons who watch westerns. In the class, are
5 persons who watch both cartoons and westerns. If

323
FURTHER EXAMPLES
one. of the c I ass is chosen at random, what is the
probability that he or she watches either aartoons
or· wes te:rns?
Method. (i) If C is the event of being a cartoon watcher and
W the event of being a western watcher, then we require'
P(C U W).
Since these events are not mutually exclusive;
(we have some watchers of both), then the required probabili-
ty is given by P(C U W) P(C) + P(W) - P(CW).
19 12 - ..2. =. 26 13
Verify that P(C U W) = 30 + 30 30 30 15

(ii) Repeat this question by 5


copying and completing the Venn dia­
gram shown.
(iii) Show that P(C) = 11/30,
P(CW) = 5/6, and that the chance that
the person chosen does not watch
either cartoons or westerns is 2/15.

EXAMPLE 6. In a group of 36 girls, 14 had blue'


eyes, 16 had fair hair, 12 were over 5 1 611 in
height, 5 had both blue eyes and fair hair,
6 had blue eyes and were also over 5 1 6", 7 had
fair hair and were over 5 1 611 , whi 1st 2 had -blue
eyes, fair hair and were over 5 1 6" in height. If
the events B, F, H denote the characteristics
blue eyes, fair hair, over 5 1 6" in height respect­
ively, find the probability that if a girl be
chosen at random, then she has either blue eyes or
fair hair or is over 5 1 6".
Method (i) This is best done by using
a Venn diagram. Copy and complete the
one given.
Hence prove that P(B U F U H) = 26/36;
. (ii) Also find P(B n F n H) and
P(B O F O H)
(iii) Is P{BUFUH)=P(B)+P(F)+P(H)?
(Note that events B,F,H are not mutu­
ally exclusive.]
*(iv) Show that
P{BUFUH)=P(B)+P(F)+P(ll)-P(BH)-P(FH)-P(BF)+P(BnFnH)

EXI\MPLE 7. In a bag are 20 bal Is, identical in size


and colour, but numbered from I to 20 inc lusive
(without repetitions). A bal I is selected at
random. • What is the chance that the number on it.
is less than 4, or a multi le of 5 or ends wi-th a 7?
Method The SaJl!ple set S = {1,2,3,4,5,6, 7,8,9,10,11.12,13, 14,
15,16,17,18,19,20J.
324
SUMMARY
If the events A, B, C denote respectively selecting a number
less than 4, a multiple of 5, a number ending with 7; then
A = \.1,2,3\ ' P(A) = 3/20 ·, B "" \ 5,10,15,201 ' P(B) = 4

C = l7,17\ , , P(C) = 2/20. 20;

Note that since A, B, C have no simple events in ccmmon,


•·• A, B, Care mutua?Zy exaZusive events. We require
the probat:ility of event A or of event B or of event C,
i.e. P(A u Bu .c).
Fow A Un U C ,. \1,2,3,5,7,10,15,17,20}. .•. P(AUBUC) = 9/20,
Here, n(A U BU C) • 9 = 3 + 4 + 2 = P(A) + P(B) + P(C)
20 20 20 20
Thus, if A, B, C are mutuaZZy exaZusive events, then
P(A U B U Cl P(AI + P(B) + P(CI
i.e. P(A or B or Cl = P(AI + PCB)+ P(C)
CoroZZary If E, ,E2 ,E3 , • • • ,En are mutuaZZy exaZusive
events, then
P(E, or E2 or E 3 or , .. or En>= PIE, l+P(E� )+PCE3 1 +...+R<Enl

EXAMPLE 8. Let S consist of the seven simple events


e, ,e2 ,e3 , • • • ,e.7 and let
P(e, =
) P<eil=·I, P(e3 = 1 P(e11-1=·15,P(e_,.)=,4, Ple1,l=P(e 1 )=·05
If A= le,,e,,e.1i, B= le,.,es\,
C= le�,e1}, fi�d
(a) P(A) (bl P(B) (cl P(AUC)
(di P(A U 8 U Cl

Method. Since the sample set for S = (e 1 ,e 2 ,e 3,e "'",es,e4,e1 \,


and ·thus e 1 ,e 1 ,e3, •• ,, (�eing simple events) are mutually
exclusive, thus
P(e , or e� or...or e 1 )=P(e,)+P(e 1 )+...+P(e1 )
=•1+·1+·15+•15+·4+·C5+•05 = 1,
as required
Now P(A) P(e 1 ,e l ,e3) = P(e,)+P(e3 )+P(e.s-) = ·1+·1+·15 = ·35
P(B) P(e 4,e�) = P(e 4)+P(es) = ·15 + ·4 = ·55,
.
. . P(B) = 1 - •55 = ·45
P(ftUC) P(A)+P(C) since A, C are mutually exclusive,
(·1+· 1+·15) + (·05+·05) = •45
P(AUBUC) P(A)+P(B)+P(C), since A,B,C are mutually exclusive
1
SLJ.1MARY
The algebra of events aorresponds to the algebra of sets.
"EVENT" PROBABILITY

a
"SET" RESULT SKETCH RESULT
RJC<:111 T
I A, the com- A,the comple- P(Al=I-P(AI
plement ment to set A, PCAI is
(contrary or i.e. the set prob. that
opposite) of elements event A does
event to A. not in A. not occur.

325
SUMMARY, EXERCISES SET 78

2
"EVENT"
RESULT
AI\S or AB
i
I
"SET" RESULT
Af\S, the inter-
SKET� PROSAB I LI TY I
RESULT
P(ASl is the
the inter- section of· probab i Iity
section or sets A and S, that both
"pr0i:Juct" i.e. set of el- events A and
of events ements common 8 occur.
A and S to A and S. sffi


3 AUS or A+S, AUS, the union s AUS PCAUS) Is
the union of sets A and S ,prob. that
or ''sum': i.e. the set of either event
of events elements in A A or event S
A and S. or in S or in (or bothl
both. occu r.
PCAUBl=P(Al+P(Bl-P(AB)
4 Events A,S Sets A, B are If A, 8 are

0G
are mutu- disjoint if s mutua 11 y ex-
ally ex-!they have no elusive,
elusive if elements in th.en P(AB)=O
they have common. and P(AUS)
no simple =P(A)+P(B)
events in
common.
Further, if A, 8, c, ... are mutually exclusive events,
Hien P(A or B or C 9r... ) = P(A)+P(Sl+P(C )+ .•.
EXERCISES SET 78

oo
1. The ci�grams show various events A, B in a sample
space S. The theorems illustrated in the previous
section are to be used here.
(

i i
(il ri ) @
e@
In (i) ._ P(i') = • l! and P(E) = • 35, state the values of
P(A), P(ii), P(AB), P(A�IH), P(ALIB)
In (ii), P(A) = ·4, P(B) = ·3, P(At)=•2.�, find the values of
P(A:J), P(AUB), P(AU;)
In (iii)' P(A) = • l, P(B) = .4, deterr.iir.e the values of
P(B). P(AB), P (AUI:)
2. It is given that for events r.,, i 2
(:! .)P(E, E�) = P(Z,) = 1/4, P(E,) = 1/2,
what is P(E, U E�)?
(ii) P(E , U E,) =· • 7, P(E, ) = • 5, P (F..) = · • 3,
what is P(E,S 2 )?
326
EXERCISES
(iii) P(E,) = 1/3, P(E1 ) = 1/4, P� = 1/5,
what is P(E 1 U Ei)?
(iv) P(E 1 U E 2 ) = 7/10, P(Z,) = 1/5, P(E 1 Ei ) = 0,
what· ls P(E 2 )?
3, Ycu are askec to select one digit from the 4igits 1,2,3,
4,5,6. The events P.., ll, C, D are defined as follows:
A = \1,2,3,4}, B = f3,4,S,61, C = ll,2,3\, D = t4,S,6\,
i;nich of the following reasonings is correct? Correct
t!1e one (s) which are incorrect,
(a) P(AUB) P(A)+P(B) = 4/6 + 4/6 = 4/3
(b) P(CUD) = P(C)+P(D) = 3/6 + 3/6 = 1
(c) P(AVD) = P(A)+P(D)-P(AD) = 4/6 + 3/6 - 1/6 = 1
4. (i) If A, B, Care mutually exclusive events and
P(A) = 1/2, P(B) = 3/10,·P(C) = 1/5, what is
(a) P(AUn) (1->) P(ilUC) (c) P(B) (d) P(C)
(e) P(AUC) (f) P(AB) (g) P(BC) (h) P(AUilUC)
(ii) If events A, B, C are not mutually exclusive, and
P(A) = •2, P(B) = ·3, P(C) = ·4, P(AC) = • 1.
P(BC) = •25, P(!\ll) = ·2, what is
(a) P(AUC) (b) P(BUC) (c) P(AUB)?

5. In t!1e diagrams, e, ,; , ., .,es are simple


events, constituting toe sample space S.
In (i) the simple even ts have equal measures
of probability.
In(ii) p (e, ) = P(e 1 ) P(e3 ) .2,
P(e·..) = P(es-) = •CS,
p (e�) = P(e 1 ) = P(e l! ) ·1 lll)
For each diagram determine the values
(a) P(A) (:-) P(1.) (c) P(B) (d) P(AB)
(e) P(AUB) (f) P(AUB)
6. (i) A,R,C are events_ such that A = \.e, ,e 1,e 3 ,e7 ,ei1,
B = £e 1 ,e 1 ,e i.,e7!, C = t_e,.. ,es,e,3
where e,,e,,.. ,,egare equally liiely simple events

and S = le,,e,,e3 , . . . ,e 11
Find, using the .2robabil1ty laws,
(a) P(A) (b) P(A) (c) P(A) (d) P(C) (e) P(F�)
(f) P(AUB) (g) P(AC) (h) P(k,C) (i) P(BUC)
(j) P(C) (k) P(ZC). '·:-nich of these events A,3,C
are mutually exclusive?
(ii) Repeat (i) if P(e,) = P(e,) = P(e 3 ) = ·1,
P(P.�) = P(e,) = •15, P(e,J � ·3, P(e .., )=P(e i; )=·OS

7, Three boys, X,Y,Z toss a die, A is the event that X


tosses a 2.' B is the event that y tosses a perfect
square, C is the event that Z tosses an even number;
.;hat is (i) P(AUB) (ii) P(AUC) (iii) P(A.'"')
327
EXERCISES

(iv) P(E) (v) P(BUC)


8, (i) If the probabilities that two boys A and B will
top the form in an examination are 2/5 and 1/3
respectively, what is the probability that either
A or B will top the fona?
(ii) Three golfers enter a tournament; the probabili­
ties that each will win are 1/3, 1/4, 1/5 respec­
ively. What is the probability that one of the
three will win?'
(iii) Discs numbered from 1 to 100 are placed in a bowl.
If one disc is drawn at random from the bowl, find
the probability that it will bear the number 29 or
a number less than or equal to 10 or a multiple of
12.

9. (a) In a class bf 35 students, 17 voted Maths their


favourite subject, 15 voted Science their favour­
ite subject, whilst 5 said they liked Haths and
Scfence equally, If M, s· denote events that
Haths, Science respectively are favourite subjects
find (i) P(M VS) (ii) P(Mf' S).
(b) Verify these results by drawing a Venn Diagram and
using the definition of pr.obability.
10. Of the 20 girls at a picnic, 5 were sunburned, 8 were
bitten by mosqui.toes . and 10 got ·home without mishap.
Uhat is the probability that a (i) bitten girl was also
burned (ii) sun-burned girl was ignored by mosquitoes?
[Use a Venn diagrar.i to help .in the question.]
11. T'.1ere are 120 senior students at a girls' school;
44 play tennis, 43 play basketball, 49 play hockey,
10 play basketball and tennis, 8 play basketball and
hockey, 21 play tennis and hockey, whilst 3 play all
three sports.
One of t:1e girls is picked at random. What is the proba­
bility that she plays (i) at least one sport
(ii) only one sport. [,lint: draw a diagram)
-�12. A survey of a class of students showed that each student
had at least one birc. Let R, 3, G be the respective
events of a student :1aving a red, black or greer, biro.
Also the notations n(R),n(B) etc. are used to denote the
number of simple events in R, 3 respectively.
If n(R) = 25, n(3) = 37, n(G) = 10, n(RnB) = 16,
n(B�G) = 5, n(RnG) = 3, n(BnRnG) = 1 find (i) P(RUB)
(ii) P(BUG) (iii) P(RUBUG) (iv) P[(RnG)U(BAR)]

13. (i) In a series of births at a maternity hospital,


what is the probability that at least 1 child born
was a girl if there were
(a) 2 births (b) 3 births (c) 4 births
(d) 5 births (e) 10 births

328
EXERCISES
(ii) A die is tossed, What ts the chance of obtaining
at least one six if the number of tosses is
(a) 2 (b) 3 ( c) 4 l
14. (i) If· one of two events must happen, and the chance
(p) of one happening i8 2/3 the chance (q) of the
other happening, find p.
(ii) There are 3 events A, B, C one of which must, and
only one can, happen. If the odds are 8 to 3
against A, 5 to 2 against B, find the odds against
c.
15. A die is cast. If Xdenotes· the number on the uppermost
face, use the probabili!l....l!ws above where possible, to
find (a) P(X ..3) (b) P(X= 3) (c) P(X= 3 or 5)
(d) P(X= 3 or 5) (e) P(X is even or odd)
(f) P(Xis even or odd) (g) P(X= 3 or X, 5)
(h) P(Xis even or prime) (i) P(X � 3 or X� 5)
(j) P(X � 4 or X� 3) (k) P(Xis a multiple of 3 or Xis
even) (l) P(X+ 4 = 9)
Now check each of the above by direct application of the
definition of probability.
16. A regular pack of 52 cards is taken. One card is select­
ed at random. By using the laws of complements and add­
itions, where possible, fit.d
(a) P(ace) (b) P(six) (c) P(ace or six)
(d) P(ace or six) (e) P(spade) (f) P(ace or spade)
(g) P(ace or spade) (h) P(ace or six or jack)
(i) P(ace of clubs) (j) P(jack of hearts) (k) P(ace of
clubs or jack of hearts) (l) P(red) (m) P(black)
(n) P(red or black) (o) P(red or black) (p) P(picture
card) (q) P(red picture card) (r) �(red or picture
card) (s) P(picture c�rd or a heart)
(t) P(less than 9 or a picture card) (u) P(more than 9
or a picture card)
(v) P(a queen or a 7 or a red jack)
Check these results using the sample space for the ex­
periment:.
H 7. Two dice are tossed. If X, Y denote the number of spots
on the upper face of each die, use the probability laws,
where possible, to calculate
(a) P(X= 3) (b) P(X= 3 n Y = 2) (c) P(X = 3 U Y = 2)
(d) P(X1 3) (e) P(X=3) (f) P(X = 3 ('I Y = 2)
(g) P(X= 3 UY= 2) (h) P(Xis odd) (i) P(Y is even)
(j) P(Xis odd n Y is even) (k) P(Xis odd or Y is even)
(.£) P(X or Y is odd) (m) P(Xor Y is odd) (n) P(X= Y)
(o) P(X= Y) (p) P(X= Y or X= 2) (q) P(X+ Y= 5)
(r) P(X+ Y 1 5) (s) P(X+ Y, 5) (t) P(X+ Y L 5)
(u) P(X+ Y � 7 or X+ Y = 11)
(v) P(X+ Y f 10 or X+ Y is even)

329
PRODUCT RlJLE
1· TWO STAGE EXPERIMENTS. THE PRODUCT RULE. P(AB)
In certain experiments, the occurrence of an event A may
have no effect at aU on the occurrence of an event B, and
vice versa. That is, the fact that one event happens does not
influence the happening of the other event. Such events are
said to be independent events.
These may take ·place in various circumstances; such as
(i) When there are two different random experiments in pro­
gress. For example, in experiment 1, a coin is tossed,
and in experiment 2, a die is cast. If the event A is
the tossing of a head, and the event B the casting of a
number greater than 4, then it is obvious that there is
no connection at ali between events A and B. In other
words, they are independent events.
Now AB (or AnB) denotes the event that both A an� B oc­
cur, i.e. that the coin turns up heads and that the die
shows a number > 4. What is the value of P(AB) here?
The sample space S \Hl,H2,H3,H4,H5,H6,Tl,T2,T3,T4,T5,
T6)
[Hl refers to head on the coin and 1 on the die; of
course, the order of writing the H and 1 is not im-
portant here. J s T, T � T1
Now A= (H1,H2,H3,H4,HS;H6)
and B = lHS,H6,T5,T6}
AB = (H5,H6}, and • P(All) =
2 = 1
ii 6
However, here P(A) 6/12 1/2
and P(B) 4/12 1/3 T•
thus P(All) .1 .1 .1 = P(A) P(B)
6 2 3
For independent events, A, B this property, namely that
P(AB) = P{A) . P(B) is al1Jo:ys true. It is often referred
to as the product rule. [�any texts define independent
events A B as those tor which P(AB) = P(A) , P(B)l.

When there is a repetition of a randan experiment, such

w
(ii)
as tossing a coin, t:1rcwing a die, selecting a card
from a pack, or a ball from a bag (provided that the
card or ball is replaced after each draw), bearing
children.
For example, consider an urn in which there W
are 4 red baUs, 3 white balls and 2 green balls. G
Three balls are draIJn in succession from the um,
the ball being replaced after each draw. What is the proba­
bility of ohtaining a red ball, then a white ball, then a
green ball?
Here the sample set for each draw,
S = \_R.,R 2 ,R 3 ,R ..,W,,W� ,W3 ,G, ,G,1 }, the subscripts used to
distinguish the balls. For the first draw, if A is the event
of drawing a red ball, then A = \.R, ,R;a. ,R3,R,.� and .". P(A) "' f
For the second draw, if Il is the event of drawing a white
ball, then 8 = {W1,W2,W3} and .·. P(B) • 3/9.

330
EXAMPLES
For the third draw, if C is the event of drawing a green
ball, then c .. \_G, ,G �� and • '. P( C) • 2/9. [Shce the
the
happening of A, Bor C in any one draw does �ot affect A, B ,
occurrence of any of these in any ot,1er draw, tne events
C are independent.]
We require the event A, then B , then C i.e. A/\ BAC
(written as ABC).
Now ABC= R 1 1 ,R1WG
{ 1w<; 1 2 , R1W2G1 R , 1W2G2 ,R1W3G1 ,R1W3G2 , R2W1G1 , ..
i.e. 4 x 3 x 2 = 24 simple events. .., R�W3G2}
Also, for the 3 draws, the total sample space S comprises
9 x 9 x 9 729 simple events.
24 4 3 2
Thus P(ABC) = 729 = 9. 9. 9 P(A).P(B ).P( C)

w l1U lJiJ
Hence if A,B,C are independent events, then
P(ABC) P(A) . PCB) . P(C)
l/a,J in the above example, if w ' '
the balls are drawn without �e­ G G G
plaaement, then the probability
of drawing a Draw 1 Draw 2 Draw 3
(i) red ball on the first draw (event A) is 4/9
(ii) white ball on the second draw (event B °
) is 3/8
(iii) green ball on the third draw (event )C is 2/7
Now th_e three events A, B , C have taken into account
the assumption that after the first draw one red ball has
been selected, and after the second draw one white ball
has been selected, This idea has been shown in the sketches.
As they have been performed, the events A, B,- C can be con­
sidered independent events. the probability of event A
follo:;ed by event B, followed by event c,
i.e. P(ABC) = P(A) P(B) ,' P( C) 4/9 . 3/8 , 2/7 1/21

EXAMPLE I In a heap of cards, there are 7 black and


5 red cards. What is the probabi·lity of drawing in
succession (without replacement)
(i) a black, then a red card [Two draws]
(ii) 3 black cards [Three draws)
I (iii) 3 black and then 2 red cards [Five draws]
tr} The chance of selecting (a) a black card on the first
draw (event E) is 7/12 BBBBBBBR RRRR
(b) a red card on the second
draw (event F) is 5/11 BBBBBBRRRR R
(the second draw assumes, of course, that a black card was
selected in the first draw.)
Since these events E, F are independent, and we want to
find P(EF), we use the result P(EF) = P(E) • P(F)
_ ______.. 7/12 • 5/11 • 35/132.
(ii) The probability of selecting (a) a black card on the
first draw (event E) is 7/12 BBBBBB BR RR R R
(b) a black card on the
second draw (event G) is 6/11 BBBBB BR RR RR
(c) a black card on the
third draw (event H) is 5/10 BBBBBRRRR R
331
EXAMPLES
the probability of event E, then event G, tqen event H,
i.e
. P(EGH) • P(E) • P(G) , P(H) , since E,G,H are independent
• 7/12, 6/11 . 5/10 7/44.
(iii) The probability of drawing 3 black, then 2 red cards is
7/12, 6/11 . 5/10 . 5/9. 4/8 (Check this!)= 35/792.

EXAMPLE 2. Three students A, B, C try out for a


footbal I team. The.chances that A, B, C wl I I make
the team are respectively 60%, 66\%, 75%. What
is the chance that (i) al I three make the team
(if) A, B make the team but C does not
(iii) they all miss out
(iv) onl one of the� si successful.
Method . If E, , E�, F3 are the respective events of A, B, C
maki.!!_g t�e t�am, then P(E,) = 3/5, P(E2) = 2/3, P(E3 ) = 3/4.
Now E,, EA , E3 are the respective events that A, B.L C do not
make the team, · with co�responding probabilities P(E,) = 2/5,
P(E2 ) = 1/3, P(El ) = 1/4.
(i) For all three to make t!ie team, we want the "product"
event E1 E1 E3 to .occur, and since E, ,E A ,E3 are independ­
ent events, then
P(E1 E 2 E�) = P(E1 ).P(E�).P(E3 ) = 3/5. 2/3. 3/4 = 3/10.
(ii) If A, B are to make the team and C is to be unsuccess­
ful, w� want the.event E,E,E3 to occur. The events
1 1 ,E 2 ,!3 are also indepenient of one another, and thus
P(E 1 E2 E 3 ) = P(E1 ).P(E 2 ).P(E3) = 3/5. 2/3. 1/4 = 1/10
(iii) He.E_e.1.. l_he even! P(E,!2E3) .!_s required, and as before
P(I: 1 E :l E3) = P(E 1).P(E2) .P(E3) = 2/5 . 1/3. 1/4 = 1/30
(iv)

SUMMARY. If the occurrence of an event A has no effect on


the occurrence of aG event 8, then the events A, B are
indP.�P.ndent events. Here P(AB) = P(A) . P(B)
Similarly, if A, B, C, ..• are independent events then
P (ABC. : •) = P (A). P_(_B_l-'-._PC'--l
_( _-'-. '-------------'
'
-'--

EXERCISES SET lC
1. If A, B, C are independent events, and P(A)=f, P(B) =f,
P(C) = f, what is the value of (i) P(M) (ii) P(BC)

332
EXERCISES SET 7C

(iii} P(AC) (iv) P(ABC).


2. A set of 5 cards contains the ten, jack, queen, king,
ace of hearts whilst another set of 6 cards contains the
nine, ten, jack, queen, king, ace of spades. One· car.d is
dr2wn from each set; what is the probability of drm•ing
2 queens.
3. One bag contains lC cards numbered 1,2,3,...,10, A
second bag contains 26 cards lettered a,b,c, •••,z. One
card is drawn from each bag. '.:hat is the proLability of
the combinations (i) 7 and f (ii) a vowel and 3
(iii) a consonant and an even number (iv) a number
greeter than 2 and a letter of the word "exactly".
4. A coiP and a die are tossed simultaneously. Find the
probability of (i) a head on the coin and a 3 on the die
(ii) a tail on the coin and an odd number on the die.
5. A 2nd B attempt a rroblem; if A has a 60% chance and B a
70% chance of solving the problem, Hhat is the chance
that (i) both solve the problem (ii) A solves the
problem and Bis unsuccessful (iii) A solves the problem
(iv) neither solves the problem (v) at most one is
successful.
6. In bag 1, there are 3 black and 4 white balls, and in
bag 2, there are 5 black and 6 white balls. One ball is
drawn from each bag. What is the probability that they
are (i) both black (ii) �oth white (iii) black from
bag 1 and white from bag 2 (iv) white from bag 1 and
black from bag 2 (v) different colours.

7. A hat contains 6 red and 5 yellow balls; a bag contains


4 discs marked respectively E,A,S,Y; a refrigerator con­
tains 3 bottles of soft drink and 2 bottles of milk. A
ball is selected from the hat, a disc chosen from the
bag and a Dottle taken from the refrigerator (each op­
eration being at random). Find the probability of
obtaining a yellow ball, the letter S and a bottle of
milk.
8. I toss a coin, I toss c1. die and select a card from a
regular pack. Find (i) P(a head, a 4 and an ace)
(H) P(a tail, an- odd number and a cl1;1b)
(iii) P(a head or a tail, a number less than 3 and. a
picture card).
9. Three students A, B, C apply for scholarships at the
H.S.C. Exam. If the probabilities of eaci1 gaining a
scholarship is 2/3, 3/4, 4/5, find the chance that
(i) all three gain a scholarship (ii) A and B gain a
scholarship but C misses out (iii) none of them are
successful (iv) only two of them win a s.cholarship
(v) only one is successful,
333
EXERCISES
lJ. The probability of a new-born child being male is 0•51.
What is the chance that three consecutive births result
in (a) 3 males (b) 3 females (c) 2 males and a female
in this order (d) 2 males and a female. [Use square,
cube tables to help in the calculations.]

11. In a bag are 7 white balls and 3 red balls.


What is the probability of drawing in success­
ion (a) with replacement (b) without replace-
ment
(i) two white balls (ii) two red balls (iii) a white
ball then a red ball (iv) a red ball then a white
ball (v) two balls of the same colour (vi) 2 balls
of different colours.
12. (a) Suppose we throw a die twice, �vhat is the proba­
bility of throwing in order (i) a 5 then a 3
(ii) two fours (iii) a number greater than 3 then
a number less than 5 (iv) two even numbers.
(b) The die is now t:-irown 3 times. Hhat is the prob­
ability of obtaining
(1) the numbers 3,5,1 in order (ii) three twos
(iii) two fives then a six (iv) three even numbers
(v) each number greater than 4 (vi) the first
number divisible by 3; the second one divisible by
5 and the third an even number,
13. From a pack of cards, two cards are drawn at random
(a) wit:1 re;>lacement (b) without replacement
What is the probability of drawing
(i) a spade then a club (ii) two diamonds (iii) a red
card followed by a black card (iv) two black cards
(v) the king of hearts and the queen of diamonds
(vi) a king then a queen (vii) two jacks (viii) the ten
of heart's and then a spade.

14. (i) An urn contains 5 yellow and 3 red balls. Find the
probability of dra11ing out balls alternately yel­
low and red (starting with yellow), if the ball is
rep laced after each drc1.w, and there are
(a) two draws (b) 3 draws (c) 4 draws
(ii) Repeat (i) asswning that the ball is not replaced
after each draw.
15. A beg contains 10 counters marked with the integers 1 to
10. A counter is drawn and replaced. A ccunter is
· again drawn. Find the probability that
(i) the same counter is drawr. a second time (ii) each
counter is an odd number,
16. A bag contains 9 counters markec 1 to 9. If two counters
are drawn at randol!I, find the probability that
(i) both are odd (ii) both are even (iii) one is odd
and one is even (iv) they are numbered 3 and 7,
334
SUMMARY
17, <�> A pencil is in the shape of an octagonaL prism.
There are 3 white faces, 1 red face and 4 black
faces; (the ends are ignored). The pencil is
tossed so that it lands and remains on one of
these faces. If W is the event of a white face,
B the event of a black face, R the event of a red
face, what is
(i) P(W} (ii} P(B} (iii) P(lNB} (iv) P(WUB}
(v) P <ii>
(�} If the pencil is tossed twice, find
(i} P(WW) (ii) P(BB) (iii) P(two faces of the
same colour) (iv) P(a black then a white face}

SLMMARY OF PROBABILITY LAWS


I. If a random experiment consists of "c" simple events,
of which "a" comprise the event E, then the probabi 1-
ity of E occurring is defined as the ratio a/c,
i.e. P(E) a/c
2. P<E> - P(E)
3. P(AUB) P(A)+P(B)-P(AB). Total probabi Iity law of
addition
P(A) + P(B), if A, B are mutually exclus-
ive events.
4. P(AB) P(Al .P(Bl, if A, B are independent events
5. P(AUBUC) = P(A)+P(B)+P(C) if A,B,r, are mutually ex-
elusive
6. P(ABC) P(A) .P(Bl.P(C) if A, B, C are independent

F. EXERCISES INVOLVING TH� TOTAL (ADDITION)


PROBABILITY RULE AND THE PRODUCT RULE
EXAMPLE I. Two bags contain resp ectively
6 red and 4 green bal Is;
green ba11s.
5 red and 3 6R
A ba I I is drawn from each
I I
bag. What is the probability that they �
are (i) of different colours (ii) of the
sam e colour?
Meti:od. (i) There are two possibilities: a red ball from the
first bag following by a green ball from the second bag, and
vice versa. Let A be the event R,G, and B be the event
G,R, The events A, B are mutuaUy exclusive events; i.e. the
events cannot occur together.
For the event A, the selection of a red ball from the first
bag is ind2�endent of the selection of a gr-een tall from the
second bag. -The probability of these separate selections is
6/10 and 3/8; hence the proLability of the draw R,G is the
product of these separate probabilities,
6 3 18
i.e.
lC ' 8 .. -80
335
EXAMPLES
For the event E, the
probability of the draw G, R is
4 5 2r 18 20
= i.e. P(A) = P(R,G) = - and P(B) = P(G,R) =
lO , S 80• 80 BC
We require the ?robabilityof event A or event B, i.e. P(AUB).
Since A, Bare mutually exclusive events •• P(AUB)=P(A)+P(B)

.
i.e. the required probal>11ity =
18 + 20 38 19
80 80 = 80 = 40
(ii) Again t:1ere a.re 2 mutuallyexclusive events to consider:
namely (a) RR and (b) GG.
6 5 30
For (a) P.R, the ;,robahility .is =
10 · 8 80
· . 4. 3 12
(b) GG, t.1e
, ;,robab.l. 1 l.ty is =
.
10 30
•8 .ll 80
42 21
H�nce, the required probability= 80 + 80 = 80 = 40

EXAf,!PLE 2 • Thero are three hea?S of Heap I Heap 2


cards, the first contains 3 �'Iack �RBRR I 8B98RRR
and 2 red cards, t�e second contains
Heap 3
4 black and 3 red cards, the third
contains 2 black and 4 red cards.
BBRRRR
What is the probabi Iity of selecting
3 heao at random and drawinq � black card from it?

(a) t-fetl,od. There are three mutually exclusive cases:-


(i) Event A: Selecting heap 1, then dra11ing a black card
from it.
(ii) Event l: Selecting hea;, 2. then drawing a black card
from it.
(iii) [vent C: Selecting i1eap 3, then drawing a i>lacL card
from it.
For each of these mutually exclusive events, selecting
t:1e heap and drawing the card therefrom are independent oper-
ations. Ti1e respective prob·abilities are:
(i) P(A) P(heap 1).P(black card) 1/3 3/5 3/15 = 1/5
(ii) P(E) = P(heap 2). P(black card) = 1/3 • 4/7 4/21 = 4/21
(iii) P(C) = P(heap 3).P(black card) = 1/3 . 2/6 2/18 = 1/9
1 4 1 158
Thus the required probability= + 5 21 9
+ = 315

(b) TREE METHOD TRI/IL 2 PROBABILITIES


TRIAL 1 (Selectin a card)
r,
!'(teaµ l,bleck end)
(Selecting a heap)
Ys_...::,B

1 �1-R P(heap 1, red card)


�3 .
4 __.. B P(heap 2,black card)
�-�'7
j�},--.R P(heap 2, red card)

)/; 3 Jt,�B P(heap 3,black card)



·tieap 3
-6
4
i�R P(heap 3, red card)

336
EXAMPLES
The required probability could have been easily obtained
also from a tree diagram, where we plaae the appropriate
probabilities along the "branahes" (In this case, we refer
to the diagram as a "probability" tree.)
Notes. The numbers on each branch represent probabilities.
Thus "1/3" on the branch leading from start to heap 2 means
that the chance of selecting heap 2 from the start is 1/3.
Also "4/7" on the branch leading from heap 2 to B, means that
the chance of selecting a black card from heap 2 is 4/7.
Thus, proceeding a long the 2 branches IB
"start to heap 2, then on to B", we can
determine the probability of performing
these 2 independent events, namely sel­
ecting heap 2, and then, once it has been selected, drawing a
black card from it, Using t\e product rule, P(selecting
heap 2 and then from it, drawing a black card) = P(selecting
heap 2). P(drawing a black card from it)= 1/3. 4/7= 4/21.
Similarly, for tile other branches.
Hence, the required probability of drawing a black card
can be determined from the three branches in the diagram
(leading to B).
1 3 1 4 1 2 158
Thus P(black card)= . + . + . = as before.
3 5 3 7 3 6 315

(c) flow, determine, by both methods, the probability of


drawing a Ped card from one of the heaps. Check your
result by using the aomplement event rule on the
P(black card).
EXAMPLE 3. In a census taken of adults in two towns A and
B, it v1as found that in town A, 55% were married, 35%
were single and 10% were vlidowed; In tam B, 60% were
married, 40% were single,
A pub·I ic opinion pol I representative selects one of the
towns at random, and interviews two adults, also chosen
at random. What is the probabi I ity that (to<) they are
both married (�) at least one is single (�) neither is
wi do1ved.
Nethod 1. The various mutually ex- Town A Town B
elusive events are set out below.
(Tl:e letters M, s, 1-i stand respect­
ively for a married, single or
widowed adult.)
(i) Seleat town A, and therefrom,
choose _(a) M, M (b) S, S (c) W, W (d) M, S (e) S, M
(f) H, W (g) W, 11 (h) S, W (i) W, S
(ii) Seleat town B, and therefrom, choose (a) M, M (b) S, S
(c) M, S (d). S, M. Consider the probability of any par­
tiaular event, say, selecting town A and then choosing
from it, a married then a single person. This probabil­
ity is written for convenience here as P(A: M,S). Since
337
EXAMPLES
all three steps, (namely, choosing town A, selecting a
married adult and selecting a single adult) are indepen­
dent,
' by the product rule, P(A:M,S) • P(A),P(M).P(S)
1 55 35
.
= 2 100 • 100
Returning to the <..••iginal question, for
(�). we want P(A:M,M or B:M,M)
Now P(A: M,M) • 1 z, 55 . 55 1 60 60
lOO lOO and P(B:M,M) .. 2. 100·100•
and hence the required probability
1 11 11 1 3 3 265 1
<2 · 20 • 20 + 2 • s • s> "' 800 < = 33s%>
(13), we want P(A:SS or A:11,S or A:S,M or A:S,W or A:W,S or
or B:S,S or B:M,5 or B:S,M)
Show that this probability is equal to
1 35 35 1 55 35 2 1 --12_ 10
[2 . 100 • 100 + 2 • 2 . 100 100 + . 2 . 100 100
1 40 40 40
+ 2 100 ' 100 + 2 l 2
_§Q
100 100 1
The numerical value of this is 487 = 60P.7 %
800
we want P(A:M,M or A:S,S or A:M,S or A:S,M or B:M,M or
B:S,S or B:M,S or B:S,M). Here, it would be simpler to
use 'the aomplement rule for probabiliti_es. If E is the
event of neither being widowed, then E is the event of
not "neither being widowed", i.e. the event that one or
the other or both being widowed.
i.e. P(E) = P(A:H,H or A:M,W or A:W,rl or A:S,W or A:W,S)
= 1 10 10 2 1 55 10 2
2 . 100 • 1:'.'0 + • 2 . ·100 • 100 +
+ 2 l
2
--12.
100
..JQ
100

181
Hence the required probability P(E) 1 - -12.
200
=
200
= 901,2%

Method 2. This type of question lends itself readily to a


tree diagram, and is more simply done by a probability tree
as shown ���� _next..P�-· Cheek it earefuUy for yourself.
The required probability for
121 144 265
�'o< ) is. 800 + 800 = 800 = 331% 0 8

77 +77 49 14 14 +96+96+ 64 = 487 =


(�) is + +
800
+ 60:%
!10(1
121+77 +77 +49 +144+96+96+64 = 724 = 181
('ti1:1) is
. 800 800 200
= 901<2"'•'·

The above results should be aarefully checked from the


probabilities listed next to the probability tree. (�lote that
the sum of the listed probabilities must be 1)

338
TREE DIAGRAM
TRIAL 1 TRIAL 2 TRIAL 3 EVF.NT f..ROB.A;31LITrns
(Select another adult)
(Select an adult)
(Select a town) M A M,M l .!l 11 = 121
2 20 20 800

�1---1
100
� s A M,S l 11 ....2 = ...1.]_
------... 100
2 20 20 800
...!2..
100
w A M,W l .!l --1. = 22
2 20 20 800
M A S,M l 7 .!l = _]]_
55 2 20 20 800
Too
Town A � 35 S s A S,S l _]_ 7 = �

,�
,oo

�r
100 ��
..!.2.
2 20 20 800
°'
100
w A s,w 1 7 1 = 14 M
M
2 20 10 800
M A W,M l ...l 11 = 22
.ll 2 10 20 800
w l ...l
100
35
s A w,s ....2 = 14
ioo
lo 2 10 20 800
ioo l ...l
w A W,W ...l = __i
I 2 10 10 800
M i;o
M B M,M l ....1 -1 = 144
ioo

'��
4-0
2 5 5 800
ioo
B lOO s B M,S l ....1 2 = --2£
!Q. 2 5 5 800
\00 ---- .
s � !-1 B S,M l --1. ....1 = 96
100
2 5 5 800

l

100 2 2 �
s B S,S =
2 5 800
FURTHER EXAMPLES
EXAMPLE 4. At a shooting gallery, fire 4 shots at
a target. Assuming that a shot has the probabi I ity
of I /3 of knocking over the target, I I 3 of hitting
it but not knocking it over, and I /3 of missing i .t
altogether, what chance have I of knocking the tar­
get over, supposing that 2 hits wi I I knock the
tar et over?
Method 1. Tliis type of problem can be
also done by the tree diagram method.
(K, H, M stand for knock over, hit,
miss respectively). Chee\; the diagram
below very carefuUy !

I
3
'J
1

1'
1
.!.
3

The probability of knocking the target over


=31 1 1 1 1 1
+ (3 . 3. 3) + (S . 3. 3. 3) + (? . 3
1 1 1 l) = J.i
3 3 3 81

Method 2. The probability of not knocking the target over


with one of the four shots = P(4 misses or 3 misses and 1 hit)
= P(4 rnisses)+P(3 misses and 1 hit),
since we are considering 2 mutually exclusive events
Now P(4 misses) = P(miss with shot 1) .P(miss with shot 2) .•••
since these are independent events
=..:!:. l 1 1 _l
3 3 3. 3 81
1 1 1 .
3 3 3. 3
Also P(3 misses and 1 hit) 4 • l (since the h'it
mny be scored.on
� any of the 4 shots)
The probability of 81 1 + 4 5
not knocking the target over 81 ii
81
and .·. the probability of
knocking over the target 1- -2. Ii
81 81

340
EXERCISES SET 170
1, Tw0 urns contain respectively 2 black balls and 4 white
balls; S black balls and 3 white balls, A ball is drawn
from each urn. \.'hat is the probability that the balls
are (i) both white (ii) both black (iii) different
colours (iv) the same colour,
2. In two heaps of cards are 8 diamonds and 7 clubs, and S
diamonds and 6 clubs respectively. A card is drawn at
random from each heap; what is the chance that they are
(i) both clubs (ii) different suits.
3, A red die A and a green die B are each tossed. With
my eyes closed, I select one of the dice; What is the
probability the die chosen bears on the uppermost face
(i) the number S (ii) an odd number (iii)an even number
or S. A B

lill �
4. Urn A contains 4 white and 2 red balls,
and urn B contains 6 white and 3 red
balls. An urn is chosen at random and
(i) one ball is drawn; what is the probability that
the r.all is red?
(ii) two balls are drawn in succession without replace­
ment; what is the probability that they are both
white?
*(iii) two balls are drawn in succession with replacement;
what is the probability that they are different
colours?
s. Urn X contains 3 white and 4 black balls; Urn X Urn Y

ULl LJj
urn y contains 4 white and 3 black balls.
You select one of the urns at random, and
B B
draw a ball, Find the chance that it is
white. Criticise the following reasoning and then
correct it:
"The chance of selecting a white ball from urn X is f,
and from urn Y is "1· Thus the chance of drawing a white
l !±.
ball from either urn is + � 1, i.e. a white ball is
7 7
certain to be drawn."
6. (i) A coin is tossed, If it turns up heads, a ball is
drawn from hat 1 which contains 3 black and
2 whi�e balls ; if it turns up tails, a ball is
drawn from hat 2 which contains 1 black and
1 white ball. What is the probability of drawing a
white ball?
(ii) Repeat the experiment of (i) with a die; drawing
from hat 1 if the die shows a nwnber divisible by
3; otherwise, drawing from hat 2.

7. The probabilities that a man X and his wife Y will be


341
EXERCISES
ea c h alive in 20 years time is 3/7 and l;./5 respec tively.
Fin� the probability that, 20 years hen ce,
(i) only the man is alive (ii) both are alive
(iii) at least one of them is alive.

8. If there is a 80% chance that A can hit a target and a


9':% chan c e that :: c an hit the target, what is the chan c e
that
(i) hot:! of them will hit the target (ii) neither of
them will �it the target (iii) A hits the target
but I' misses (iv) at least one of t:1em will hit the
target (v) A hits the targe� (vi) at most one of
them hits the target.

9. Criticise t;1e following statement and oorreot it if you


c an:-

Re c ords show that the probability of a male candidate


passing a c ertain exam is ·6 ,:hilst the pro'.ial.;ility of
a female candidate passing t'.le exam is •4, T:1erefore,
if a male and a fenale c andidate are selected at ran
dorn, t·,;e r,robahility that at least one of them will
pass the exam is 1,

10. The proba�ilities that students �. 8, C c an solve a


problem are 4/5, 2/3 and 3/7 respec tively. Determine the
;irobability that
(i) A, 3, C all solve the problem (ii) A and B solve
the :,roblem but C does not (iii) none of them is
successful (iv) at least one is suc c essful
(v) at least 2 solve the pro0lam.

11. If the c:iances that 5 s:lips uill '.;rea!, the existir,g


speed record are respectively 1/4, 2/7, 1/3, 2/5, 1/2.
\{?at is the chan ce that
(i) a.11 ships l:rea!c. the E cord (ii) nc ship breaks the
record (iii) ti1e rec ord will be broken.

12. A box con ta ins 5 reel flags, 4 w'.1ite flags and 3 blue
flags. T:.ree flags are sele<'.ted at random. What is t'.1e
:-;robability of drawing one red, one white and one blue
flag (in any order), if the flags are drawn
(a) with replacement (b) without replacement.

13. A bag c ontains one 20 c coin and three 10 c c oins; a se·-


c0nd bag co!1tains two 2,,c coins and four 10 c coins and a
third 0ag contains t:1ree 2c c coins and one in c coin. If
a coin is taken from one of the !Jags selected at random,
find t�e probability that it is a 20 c coin.

·:-14, There are three hats; t!1e first contains S red and 3
green discs; the second contains 4 red and 2 green
disc s; the third c ontains 3 red and 7 green disc s�
(a) C'1I1e of the hats is selected at random and a disc
withdrawn. What is t!1e prohability that. it is
(i) red (ii) green?
(b) A dis c is drawn from each of the hats. What is the
342
EXERCISES
probability that the discs (i) are all green
(ii) are not all the same colour.
15. (a) A die is thrown t1.iice. What is the probability of
the following events (i) two 3's (ii) a pair
(iii) either a 3 and a 2 or a pair of sixes.
(b) The die is now thrown three times. What is t:1e
chance of obtaining
(i) the nwnbers 2,3,4 in order (ii) the numbers
2,3,4 in any order (iii) t·;1ree fives
(iv) three of a kind (v) two fours then a six
(vi) two fours and a six (vii) each number is
even or each number is odd.
16. Hag 1 contains 4 red and 2 white balls; �ag 2 contains
1 red and 3 ;,hite balls. A ball is drawn from bag 1 and
then put into bag 2. The balls are now mixed and one
ball is drawn at random from bag 2. Find the probability
that it is white.
tl7. Two successive draws (each of 1 card) is made from a
regular pack of 52 cards. If X and Y are used to denote
these cards, determine the measures of the following
probabilities, considering the drawini;s are made
(i) with replacement (ii) without replacement
(a) F(X is the two of cluos and Y is the ten of diamonds)
(b) P(X is a heart) (c) P(X is a heart anf Y is a spade)
(d) P(X is a heart and Y is the ace of spades)
(e) P(X and Y are both clubs) (f) P(X and Y are differ-
ent colours) (?,) P(neither card is a spade)
(h) P(at least one card is a spade) (i) P(both cards
are jacks) (j) P(ncither card is a ten)
(k) P (X is a three and Y is. a ;king) ( 0 P (bot!1 are
picture cards). (m) P(X is a picture card and Y is
not a picture card)
No.of heads O I 2 3
18. (i) A coin is tossed three times Pro�aDi I it
Ora�, a probability tree to
represent this ex:;erfr1ent, and hence complete the
table sho;m. \fuat is the pro�aLility of obtaining
either 3 heads or 3 tai�s.
(ii) Toss the coin four times. No.of heads
Ry extending the prcLa·­ Probab i I it·
hility tree of (i), com­
plete the attac�ed table.
1". For a fa."'lily of 3 chilc!re.i, construct a probabil::.ty tree
on the assur.iption that (i) a Joy and a girl are equally
likely to he born (ii) 60% of births are boys and
40% are girls, Find the probability that
(a) the 3 children are boys (b) there are 2 girls and a
hoy (c) at least one child is � girl,
20. Fer a certain randnm experiment, t�e probaoility of an
event� succeeding is p and of it failing is q, (q=l-p).
This experiment is repeated tl:ree times. Use a tree

343
EXERCISES
diagram to complete I No.of successes ' 0
the table shown. Probab i I i ty
21, A bag contains 5 good apples and 3 bad apples. Three
apples are taken out of the bag at random (one at a
time). Construct a probability tree to illustrate this
question. Hence determine the probability that
(i) 3 apples were all bad (i�) at least 1 �pple is bad
(iii) none of the apples is bad.
Consider two cases (A) Each apple is replaced after
being selected, (B) Apples are not replaced
22. Three players A, D, C are to play squash. Lots are
drawn to decide which one is to be left out of the first
match. B and C are evenly matched, ea . ch being equally
likely to win against the other; A has a 60% chance of
winning against either B or C, 'Using a probability tree
diagram, find what chance A has of winning this match.
23. In a classroom there are 10 persons on one side of the
room (7 boys and 3 girls) and 10 on the other side (5
boys and 5 girls). T:1e teacher selects one side of the
room at random, and then chooses
(i) one person. What is the probability that the
teacher chooses a boy?
(ii) two persons, one after the other. What is the
chance of getting two boys?
(A probability tree can be used to advantage here, if
desired,]
24. In two large co-educational high schools, A, B, it 11as
reported that the ratio of boys to girls was as follows.
At A 2 : 3 and at ll 1 : 2. A school inspector
selects one of the schools at random, and approaches two
students. :·!hat is the probability t!,at
(i) both students 11ere girls (ii) exactly one student
lias a girl (iii) neither student was a girl.
25. The election results for two centres X and Y showed the
percentage of votes erst for candidates A, !I, C were as
follows:
Centre X Candi�ate A 50%; Candidate B 30%;
Candidate C 20%.
Centre Y Candidate A 60%; Candidate B 40%.
A current affairs interviewer was assigned to select a
centre at random, He was asked to choose at random
(i) one elector, (P say) (ii) two electors (Q,R say)
Find the pro�ability that
(a) P voted for candidate A (�) P voted for candidate B
( c) Q and R �0th supported B (d) neither Q nor R voted
for A (e) at least one of Q or R voted for A.
26. In a referendum on a certain proposal held in Australia,
the results in three towns A, E, C were as follows

344
EXERCISES
Tawn A In favour 6('%, against 30%, had no opinion 10%
Town B In favour 55?, against 45%
T01,.>n C In favour 50%, against 50%
(a) An interviewer selected a town at random, and ap­
proached one voter, What is the probability that
the voter was (i) in favour of the proposal
(ii) against it (iii) not in favour of it.
(b) The interviewer then selected one of the towns B
and C at random,. and approached two voters. What
is the probability that
(i) both voters were against the proposal
(ii) at least one was against it
(iii) neither one nor the other was against it,
27. In an age ·survey in two latge sixth form schools, A, B
the following results were found.
Schoo! A; 20% age 16, 50!o age 17, 30% age 18
School B; 60% age 17, 40% age 18
A visitor enters one of the schools , chosen at ran­
dom, and asks the age of
(i) One student. What is the chance that the student
selected is (a) aged 17 (b) over 17
(c) 17 or under.
(ii) 'll:Jo students. What is the probability that the two
students selected are (a) both 17
(b) at least one is 17 {c) neither is 17,
28. In the two sixth form schools of question 27, the home
study habits per night were as follows:
School A; Less than 2 hours: 10:�; 2 to 4 hours: 60%;
Over 4 hours: 30%.
School B·• Less than 2 hours: 20%; 2 to 4 hours: 80%.
A newspaper reporter enters one of the schools at
random, and selects two students also at random, What is
the chance that
(i) both students do not exert themselves very much
(ii) at least one works from 2 to 4 hours each night
(iii) neither works less than 2 hours per night,

345
ORDERED AND UNORDERED SELECTIONS
SECTION F. COUNTING PROCEDURES IN A FINITE SAMPLE SPACE
- ORDERED AND UNORDERED SELECTIONS.
Note. When the number of possible outcomes in a random ex­
periment is large, then the actual enumeration or tabling of
the sample space can be arduous. In these cases, we adopt
other techniques or procedures. These procedia>es are intro­
duced not as an end in themselves, but merely as an aid in
oia> caiaulations.
(i) Basia counting theorem.
This will be illustrated by an example.
Suppose we wished � �
to go from Sydney to

Adelaide via Melbourne; S d ne /""_r..-1.
0_ �
.e I b our�� Ad e I a1· d e
c.aJt-
let us assume that Y ( � M,;r
-:---;. . ..!
-
there are 5 methods of �_
'� 1 �
travel available to us
from Sydney to Melbourne (namely plane, bus, car, walk, train)
and 3 methods avai1able from :ielbourne to Adelaide (namely
boat, cycle, hitch-hike). In ho,1 many ways can we complete
our journey?
The number of ways can easily be seen, from the diagram,
to be 5 x 3 = 15 (there are 3 ways of completing the journey
from Melbourne to Adelaide for each of the 5 ways of getting
from Sydney to Helbourne).
The sample space of this experiment would consist of the
15 simple even ts {plane-boat, plane-cycle, plane-hitchhike,
bus-boat, bus-cycle, bus-hitchhike, car-boat, car-cycle,
car-hitchhike, walk-boat, walk-cycle, walk-hitchhike, train­
boat, train-cycle, train-hitchhike}.
Obviously, the order of each of the simple events is of
vital importance. (Plane-boat would indicate, of course, an
air journey followed by a sea voyage, and evidently, would be
regarded in this work, as entirely different to boat-plane.)
Thus there are 5 x 3 z 15 ordered possible outcomes for this
experiment.
The b asic counting theorem can be expressed either in
the form

(i) If one event aan happen in m different ways, and


after this has occurred, another event aan happen
in n different ways, then the two events aan
occur in suaaession in mn different ways.
or in set notation, in the form
(ii) If a certain set M contains m different elements,
and if a second set N contains n different ele­
ments, then the set of all ordered pairs (x, y)
where x belongs to set M and y belongs to set N,
contains mn differerit elements.
346
ORDERED SELECTIONS
[This set of ordered pairs (x, y) is often cal led the
aartesian product of M, N since the set can be rep­
resented graphically on a plane using cartesian coord­
inates.)
Extension. It is eas i I y seen that if a f irs t event can
occur in n1 ways, a second event can occur in n2 ways, a
third event in n3 ways and so on, then the number of ways
for these events to occur in succession is n1n2n3....

EXAMPLE I. How many different (i) 2 digit (ii) 3 digit


numbers are possible from the digits 0, 1,2,3,4,5,6,7,8,9,
where repetitions are ·(a) not allowed (b) al lowed.

Method.�
(i) (a� Each of the two circles (or spaces) is to

be occupied by one of the digits specified. When


this has been done, we shall have formed 2 - digit
numbers (i.e. ordered selections of 2 digits from
the given digits). This first space can be occupied
by any one of 9 digits, namely 1,2,3,4,5,6,7,8 or 9
[Note zero cannot begin the number.) The second
space can be occupied by any one of the 8 digits
left after using one for the first space, and also
zero aan be used. Thus there are 9 ways of select­
ing the first digit, and having done this, there
are also 9 ways of selecting the second digit. By
the basic counting principle above, there are
9 x 9 = 81 ways of performing the two operations in
succession, i.e. there are 81 two-digit numbers
possible.

(b) W UQ] If repetitions are aZZowed, there is a


choice of 9 digits, namely 1,2,3,4,5,6,7,8,9 for
the first space, and a choice of 10 digits, namely
any of these 9 and zero for the second space, The
required number of two-digit numbers is 9 x 10 .. 90,

(ii) (a) [f] [2J []] If repetitions are not aZZowed, the
number of choices for the first space is again 9,
for the second space is also 9 (i.e. any one of the
remaining 8 digits and zero), and for the third
space is 8. Hence the required number of 3-digit
numbers is 9 X: 9 X 8 = 648.
(b) [f] CT2J CT2J If repetitions are aZZowed, the

various selections possible in order are 9 for


space 1, 10 for space 2, and 10 for space 3. Hence
there are .9x1ox10 .. 900 three-digit numbers possible.
347
ORDERED SELECTIONS
EXAMPLE2. In how many ways can (a)three coins fall;
(bl you select four cards from a pack of 52 cards, if the
cards are drawn one at a time (i)without replacement
(ii)with replacement?
Method,
{ a) [TI 00 The first coin can fall in any of two ways,

he ads (H) or t ails (T). The second and third coins can
f all similarly. Hence, by the b asic counting principle,
the two coins c an.fall in 2 x 2 x 2 = 2 3 = 8 ways.
This result can be e asily checked, since the sample
space for the experiment is
{HHH, HHT, HTH, HTT, TTT, THH, THT, TTH},
and there are 8 simple events.

(b) (1) � � [ soJ G2J There are 52 different ways of

choosing the first c ard; after the dr aw, if there


is no repl acement of the card, there are only 51
c ards, and thus we have a choice of 51 for the
second c ard. Simil arly, the two remaining cards
c an be chosen in 50 and 49·ways re�pectively.
Hence the 4 cards can be selected in ordered sets
in 52 x 51 x 50 x 49 different w ays. Note the
ordered set, say {3 of spades, j ack of hearts,
king of clubs, five of he arts} is one of these
(52 x 51 x 50 x 49) ordered sets, and the
ordered set, say {3 of spades, king of clubs,
j ack of he arts, five of hearts} is different from
the previous set.
(ii) @] (m � � If replacements are allowed,
the number of ordered sets would be
52 X 52 X 52 X 52 = 52 4 ,

(11) Ordered selections of n different elements from a set of


n suah elements.
G] !n-11 !n-21 ! n-31 .... GJ uJ
S = {al , az, a3 ,.. , ,8,i }
Asswneth at there are n different elements a1,az,a 3,•••
···•8n comprising the set S. We are to choose these elements
one at a time and keep them in the order th at they have been
selected; i.e. eaah ordered selection contains n elements.
The first element can be chosen at random in n·different
w ay s (i.e. there are n ways in which the first_ space can be
occupied). Having done this, there are (n-1) elements from
which to choose the second element (or occupy th� second
space). After these two things have been completed, the third
element can be chosen from (n-2) elements and so on.
348
ORDERED SELECTIONS
Eventually, there will be only one choice for the nth
place. Hence, by the basic counting principle above, the
number of ordered selections (each· one containing n different
elements) is n(n-l)(n-2) ..• 1. This product (as we have seen
in the previous chapter) is written in factorial notation as
n! (Remember, for example,
7! 7(7-1)(7-2).... 1 = 7.6.5.4.3.2.1 = 5040
4! = 4 x 3 x 2 x 1 = 24, and O! = l by definition)

EXAMPLE. In how many ways can 5 boys and 4 g ir Is be


arranged in a I ine (a) without restrictions (b) if the
boys and gir-ls alternate (c) if the boys and girls are
in separate groups (d) if two particular girls want to
remain together,

Method.
(a) 0 @J 0 @J 0 � 0 0 [I There are 9 = (5+4) places
to be filled in the line. Thus, we want ordered selec­
tions (each of 9 persons) from the original group of
9 persons. By the result above, there are 9.8.7.6..1=9!
such ordered selections.

(b) B G B G B G Il G B
(I]© G} G) Ll:j (D [J Q) II] If ti,e boys and girls alter-

nate, it is obvious that a boy must be at the beginning


of the line. (If there was an ·equal number of boys and
girls, then either could occupy the first place.) The
5 boys must then occupy certain positions as shown and
since there are 5 positions, this can be done in
5.4.3.2.1 = 5! = 120 ways. Similarly, the girls can be
selected to occupy their places in 4.3.2.1 = 4! 24
ways. Thus the two events can occur in succes.sion in
5!4! = 120 x 24 = 2880 ways.

(c)
10000 DI (0000 ) or(QQQO) !DODOO!
There are two problems here; first determining how raany
different groups are possible and then determining the
order of the groups in the line.
The number of ordered selections (often called arrange­
ments) for the boys in a group is 5! and separately for
the girls is 4!. Hence the boys and girls can be
arranged in ordered selections (each containing 9
persons) in 5! 4! ways. However, here the group of boys
may precede the group of girls or vice versa. Thus the
required number of ordered sets is 2. (5!4!) = 5·7-60.

349
ORDERED SELECTIONS

<d> �:DD D D D O O
Two particular girls X and Y may be together in 2 ways,
either X before Y or X after Y (i.e. X,Y or Y,X). Now,
consider them as a unit. They, and the rl?maining 7 boys
and girls, constitute for our purposes, 8 people, and
the number of orderied selections in a line is thus
8! = 40,32C. Hence, the required number of ordered sets
is 2 (8!) = 80,640.
[Similarly, if 3 particular girls X,Y,Z wish to remain
together, there are 3! ways this can be done, and
having done so, there are 6 other people plus the gr�up
X,Y,Z (now considered as a unit), to be arranged in
order.
Hence the required number is 3!7! = 6 x 5040 30,240. J

(ii:!) Orc.ered selections of r different elements from a set of


n such elements, where :: .s r .s n

� ��E}J
� I n-(r-1)!
-,- 1
space 1 space 2 space (r-1) space r
S {a1,a2,a3,.,,., an}
Here we are considering the ordered selection of sets of
r different elements from a set S of n such elements, where
O .$ r .5 n.
The first element (to occupy space 1), is selected at
random from the set of n distinct elements; the second ele­
ment is then chosen from the set of (n-1) remaining elements;
the third element from the set of (n-2) remaining elements,
and so on. When we reach the stage of selecting the
rth element (i.e. to occupy the rth place) then we will
have used up (r-1) elements from the original n elements.
Hence, for the rth e:lement, we shall.have to select at random
one of the remaining n-(r-1) elements ,
Thus, the number of ordered selections, from a set of n
distinct elements, each selection comprising r elements is
equal to n(n-l)(n-2) •• , [n-(r-1)] = n(n-l)(n-2),,,(n-r+l),
This result can be written in factorial notation thus:
nn �, .(n-r+lXn-r)6t-r-1) •. 2 .1 n!
n(n-l)(n-2)., (n-r+l) = ( -1Xn-
(n-r) (n-r-1) •.• 2 .1 = (n-r) !

SUMMARY
(i) An ordered set of r distinct elements is cal led a
permutation (or arrangement) of the k elements.
Two permutations are identical, if and if only,
tney contain the same elements in the same order.
(ii) Because of this reference to permutation, the
350
np
r
resu It above for the number of o·vdered sets (each
with r elements) from a set of n distinct elements
n
(0 !: rs n), is denoted by p . (Here the P has no
r
reference at al I to work on probability).
(iii) Thus npr = n(n-l)(n-2) ..• (n-r+I) = :� ' O!:rSn.,
(n )!

Note that npn n(n-l)(n-2) •..• (n-n+l) n(n-l)(n-2) •• l=n!

For example,
8
P = 8(8-1)(8-2)(8-3)•.•(8-5+1) = 8.7.6.5.4
5 8!
= 8! 6720 on evaluation.
(8-5) ! 3!
and

7(J-1)(7-2) •••(7-7+1) = 7.6.5 ••. 1 7 ! = 5040


7! 7!
7 ! since O! 1 by definition.
(7-7)! = ciT
=

EXAMPLE I. From the letters of the word PROBLEMS how many


words consisting of 5 letters are possible if (a) ther-e
are no restrictions (b) the words begin with P and end
v1ith S (c) they include P, do not begin \vith P and the
letter :.i is to bP. excluded .
f,Jethod. [Note the "words" obtained do not necessarily have a
meaning; they can be "nonsense" words. J
(a) l}J[z][f}[}JGJ The word P�.OBI..E;'.S consists of 8 unlike
letters; these are to be arrangen in ordered sets, each
of which has 5 letters. That is we require the number of
permutations of 8 distinct letters taken 5 at a time,
(i.e. 5 letters per ordered set). From the above, the
8 8!
required number is P = 8.7.6.5.4 =
5 31 = 6720.
[Alternatively, using the basic counting result, the
first space can be occupied by any of 8 letters, the
second space by any of 7 letters and so on.]

(b) 00[1] mm If t'.1e words are to begin with p and end


with S, then the first and fifth letters automatically
select themselves. There are left 3 places to be occu­
pied, and there are (8-2) = 6 letters available, i.e.
6 6!
the number of ordered sets is P = 6..5.4 .= 120.
3 31

3 51
EXAMPLES ON np
(c) Since\he letter His excluded,
0 D6ITo
this leaves only ·PrOBL�S, and if P is included, then P
may occupy any of tour positions. Hence ROBLES (i.e. 6
letters) remain, ana since 4 spaces are left to be occu­
pied, ·we have to. determine the munber -of permutations of
6 letters, each ordered set to consist of 4 letters.
6
This number is P = �: = 6.5.4.3 = 360. The required
4
number of words = 4 x 360 = 1440.

n
The formu Ia given above for p shou Id not be
applied blindly; the basic counting technique is often
Note. r

much better to use.

EXAMPLE 2. Six boys enter a


boat in which there are
8 seats, 4 on the bow side
IQ O D
D D D
O
Q
Bo\'/

StrokeI
and 4 on the stroke side. In how many ways can they be
seated (i) if there are no restrictions (ii) if two
particular boys A and 8 want to be on the bow side and
one particular boy X wants to be on the stroke side.

�e boys enter the boat one at a time; the first boy may
Method

select from 8 seats, the second boy from 7 seats and so


on. The number of ways is 8. 7.6.5.4.3 = 20160.
we want the number of permutations of 8
8
distinct things taken 6 at a time= P )
6
(Alternatively,

(ii) Since A,B,X have particular preferences, they are seated


first. A,B may select seats on the bow side in 4x3 = 12
ways; X may select a seat on the stroke side in 4 ways.
There are 3 boys to be seated for which there are 5
available seats; the number of ways of doing this is
5
5.4.3 = P = 60. Hence the required number of
3
arrangements 12 X 4 X 60 2880.

EXAMPLE 3.
(i) How many numbers less than 5COO are possible using
the digits 1,3,5,6,7,8 (no digit being repeated)?
(ii) How many of these numbers are even?

Numbers less than 5000 may contain 4,3,2,1 digits.


(i) (a) 4 digit numbers. uJw[i]u] The first digit must
Method.

be selected from 1 or 3, since the number is to


352
SET 7E
be< 5000), i.e. it may be chosen in 2 ways. The re-
maining digits can be chosen in 5 x 4 x 3 .= 60 ways.
Thus, there are 2 x 60 = 120 .four-digit numbers< 5000.
(b) 3-diqit nwnbers [I]Ll][}] Any 3-digit number< 5000.
Thus the number of 3-digit numbers required= 6x Sx4 = 120.

(c) 2-digit nwnbers �� There are 6x5=30 such numbers

(d) 1-digit nwnbers []J There are 6 such numbers.

In all, there are 12o+-120t-3o+-6 = 27 6 such numbers< 5000.

(ii) (a) 4-digit nwnbers [I]GJ[TIDJ To be less than 5000,


the first digit must be 1 or 3, whilst the fourth digit
must be 6 or 8 (for the number to b� even). The remain­
ing digits can be selected in 4 x 3 = 12 ways. There are
2 x 2 x 12 = 48 four-digit numbers.
(b) 3-digit nwnbers [I][}]� The numbers must be even,
•·. the last digit must be either 6 or 8. There are
( x 4) x 2 = 40 three-digit numbers.
S
(c) 2-digit nwnbers [I][]] The numbers must be even,
i.e. the last digit is �it�er 6 or'S.
There are (5) x 2 = 10 two-digit numbers.
(d) 1-digit nwnbers [}]
There are only 2 such numbers (6 or 8).

In all, there are 48+4o+-lo+-2 = 100 even numbers< 5000.

EXERCISES SET 7E
6 12
1. Evaluate (a) 7P2 (b) 7P (c) SP (d) P (e) P
5 4 .6 l
9P 6 4
SP P SP P
2 2 'f..
(f) _
3 (g) _4 (h) (i) __g (j)
3
7P 9! 6 9P
P SP
4 3 2 5
2. Let S= {a,b,c,d}. How many permutations of the elements
of Sare there, taken (a) two (b) three (c) four at a
time? List of all these for (a).
3. Letters are chosen for the English Alphabet. Without
multiplying out results, find how many "words" (i.• e.
ordered sets of letters) are possible if each word con­
tains (i) 2 letters (ii) 3 letters (iii) 4 letters.
Cons.ider the cases where (a) no repetitions (b) repet­
itions, are allowed.
4. (i) There are 6 runners in a race; in how many ways
353
SET 7E
can the 6 runners finish?
(ii) How many different outcomes are there for a winner
and runner-up?
5. In how many ways can
(i) 2 dice (ii) 3 dice (iii) 4 coins, fall?
6. Five candidates contest an election; in how many ways
can the digits 1,2,3,4,5 be assigned to the candidates?
7. (a) The digits 1,2,3,4,5,6 are considered. How many
numbers are there, each of (i) 1 digit
(ii) 2 digits (iii) 3 di3its (iv) 4 digits
assuming (a) no repetitions are allowed
(b) repetitions are permitted.
(8) Repeat (a) with the digits 1,2,3,4,5,0.
8. (i) In a nearly complete crossword puzzle, there are
three blanks. If there are 4 alternatives for the
first blank, 5 for the second blank and 7 for the
third blank. In how many ways �an the puzzle be
completed?
(ii) A boy possesses 2 suits, 1 hat, 4 shirts, 3 pairs
of shoes, 6 pairs of socks and 5 ties. In how many
different ways can he be attired, assuming he uses
one of each of the articles mentioned?
9. "Words;' are to be made from the letters of REASON. How
many 4-letter words are possible (a) without restriction
(b) if each word begins with R (c) if Sis excluded
(d) if the word contains both letters RN (e) if neither
E nor A is included (f) if R is included but O is ex­
cluded (g) begin with Sand end with N.
10. In how many ways can 7 persons be arranged in a line
(a) without restriction (b) if two particular persons
A, Bare together (c) if a particular person Xis at
one end (d) if Xand another particular person Y are
on neither end (e) if A is third in the line and Xis
last.
11. In how many ways can 8 stories be arranged in order to
form a book? In how many of these will the
(i) longest story be last and the shortest story be
first
�i) longest and shortest stories be next to each other.
12. Five persons enter a railway carriage in which there are
8 empty seats. In how many ways can they take their
places, if '*-
(i) any person can occupy any seat
(ii) two particular persons X , Y sit facing the engine
(iii) Xsits in a corner (iv) X sits with his back to
the engine whilst Y sits with his face to the
-'):- engine.
7;.;
g::�
[Assume carriage is of shape t Engine]
351.
SET 7E
13. A car licence plate consists of 3 letters (A,B,C,•••,Z)
followed by 3 digits (0,1,2, ••• ,9). Find how many
licence plates are possible if there are no restrictions
(i.e. both letters and digits may be repeated). [Leave
answer in index form.]
In how many of these
(i) are there no repetitions of letters or of digits
(ii) are there repetitions of letters but not of digits
(iii) is the first letter D and the last digit 7
(iv) are the first 3 letters DAE?
14. From the digits 1,2,3,4,5,6,7,8,9· (no digit occurring
more than once), how many numbers are there
(a) (i) lying between 3000 and 4000
(ii) less than 3000
(8) containing 5 digits (a) without restrictions (b) if
the first and third figures are even (c) if the
first figure is odd and the last figure is even.
15. How many permutations of the letters of the word
"FRIEND" are there if the letters are taken
(i) all 6 at a time (ii) 4 at a time.
16. In how many ways can 4 boys and 3 girls be arranged in a
line (a) without restrictions (b) if the boys and
girls take alternate positions (c) if the boys and
girls are kept in separate groups (d) if two boys A,B
are to stay together (e) if three particular boys
A,B,C are to remain together.
17. What is the total number of signals that can be sent by
using 4 different flags, one above the other on a flag­
pole, if the flags can be used
(i) 1 (ii) 2 (iii) 3 or (iv) 4 at a time?
•18. (a) From the digits 1,2,3,4,5,6 how many numbers ate
possible (i) less than 400 (ii) greater than 400?
Assume repetition of digits (a) is (L) is not,
allowed.
(a) Repeat (a) with the digits 0,1,2,3,4,5.
19. On my bookshelf are 4 books on Probability (P), 3 books
on Calculus (C) and 2 books on Analytical Geometry (A).
How many ways can these books be arranged if they
(i) may go in any order on the shelf
(ii) the various sets of books are together and in the
order PCA (iii) the various sets are together.

355
LIKE LETTERS
(iv) Ordered Seleotions of n elements;
not all of whioh are unlike.
{The result established below will be used in the work
on the binomial distribution which occurs later.}

EXAMPLE I. Consider the number of ordered selections of


the words (i) DRIES (ii) EERIE, takingal!5ata
time.

Method.
�RIES consists of 5 unlike letters, and the number of
ordered sets is 5! = 120.
(ii) EERIE consists of 5 letters, . three of which are alike.
For our purposes, attach subscripts to the like letters
E, so that we can distinguish between them, i.e. we take
EERIE to be E1E2RIE3. Hence there are 5 different
letters which can be permutated in 5! ways.
However, when the subscripts are removed, some of
these ordered selections will become identical. To in-
vestigate this further, consider one such permutation
say EzR E1 E3I. Assuming that the R, I remain fixed in
position and only the E's rearranged, then there are
3! = 6 such arrangements, namely
E2 E1 E3 (!) ' Ez E3 E1 , E1 E E
@ @ (!) @ i !
E1 E3 Ez E3@ E 1 E2 E3@ E 2 E 1@
@ (!) ' (1) ' (!)
When the subscripts are removed, each of these 3! permu-
tations become identical, namely EREEI.
Now, if we select another permutation from the original
set of 5 ! permutations, say E3 E i IRE 1 , then there are 3 !
arrangements keeping the R,I fixed and rearranging the
E's. When the subscripts are removed, each of these
3! permutations become identical, namely EEIRE.
Hence, each set of 3! (i.e. 6) permutations of the
letters of E, E2 R I E3 become identical when the sub­
scripts are removed. Thus, the number of permutations
5!
of EERIE is
31 .
fiimilarlu, the number of permutations of letters
�; .
of r.tlHTT, taken 5 at a time, is {'I)lis can be
3 ,
seen, for using subscripts, the word is H1H2H3T1T2, which can·
be arranged in ordered sets in 5! ways; the H's may be
arranged among themselves in 3! ways whilst the T's may be
arranged among themselves in 2 ! ways. Thus, e·ach set of
3!2! = 12 permutations of the'letters of H1H2H3T1T2 become
identical when the subscripts are removed}.

356
SET 7F
Generally, if we have n elements, of which x are alike of
one kind, y alike of another kind, z alike of another
kind, ... , then the number of ordered selections is
nI e x+y+z+... = n.
,
X, y, Z, •••
1 1 1 wher

EXAMPLE
( i) How many 8-digit numbers can be formed from the
digits 2,2,2,3,4,5,5,6?
( ii) How many distinct patterns are possible when 4 heads
and 3 tai Is are uppermost on 7 coins, and the coins
are in I ine?

Method.
(i) The number of
such 8-digit numbers, in which there are
!
three 2' s and two 5' s, is
B . 3360 •
3 ! 2!
7!
(ii) The number of distinct patterns = , ,=35, since there
4 3
are 4 heads and 3 tails.

EXERCISES SET 7F
1. How many permutations of the letters, taken all at a
time, of (i) SPEND (ii) SPEED
(iii) EASE (iv) TOT (v) MASSES
(vi) AUSTRALIA (vii) ISOSCELES (viii) COMMITTEE
2. In how many distinct ways may the following balls be
arranged in a line (all balls being used in each
arrf!'ngemen t)
(f) 1 red, 1 white, l·blue, 2 yellow, 3 green
(ii) 4 white and 3 black (iii) 2 black, 2 red, 2 white
(iv) 1 pink, 1 purple arid 8 blue.
3. Five coins are tossed on a table. How many distinct
patterns can be obtained, if the coins form a line, when
there are (a) 5 heads uppermost
(b) 4 heads and 1 tail showing
(c) 3 heads and 2 tails on the upper faces.
Using H, T for head, tail show all the patterns for
each of (a), (b).
4. Eight coins are laid in a line. How many distinct
patterns are possible (all coins being used) if on the
uppermost surfaces, there are (a) 8 tails ·(b) 7 tails
(c) 6 tails (d) 5 tails (e) 4 tails (f) 3 tails
(g) 2 tails (h) 1 tail (i) 0 tatls.
5. How many 7 digit numbers (taking all at a time) can be
formed from the digits (i) 2,2,2,2,3,3,3
357
UNORDERED SELECTIONS
(ii) 2,3,4,4,4,5,5 (iii) 1,2,3,4,5,6,6.
6. In how many ways (taking all at a time) can
(i) the letters (a) HTI-ITHT (b) HHHTTTTT- be arranged?
(ii) the numbers (a) 23343 (b) 1112333 be arranged?

( v) Unordered Se footior!§_
In many exercises in our work on probability, unordered
seleotions are required. That is, we are not interested in a
definite ordering of a set of elements, but merely in the
oontent of the set, In other words, order within the selected
group is irrelevant.
For example, if we were to count the number of ways in
which a committee of 3 students can be selected from a group
of 7, the order of selection would be immaterial; only the
actual make-up of the committee chosen would be of interest.
That is, we are only concerned with the committee as a whole,
i.e.\as an unordered subset of the original set of 7 students.
To cobipute the actual number of such committees· possible, let
us call the 7 students A,B,C,D,E,F,G.
If the 7 students were used to form an ordered selection
of 3 students, the number of such ordered sets would be
7
P = f .6.5 = �;. [Alternatively, the first_ committee member
3
may be �elected from 7 students, the second from the remain­
ing 6 students and the third from the remaining 5 students.
That is, the committee may be selected (order being tak�n
7
into account) in 7.6.5 ( = P ) ways.]
3
Now consider the committees possible with the .students
A,B,C. As ordered selections, there are 3! = 6 such
committees, namely ABC, ACB, 3C,\, BAC, CAB, CBA. However,
since only who are on ·the commit tee is of interest; not in
what order they are selected to go on the committee, then
these 3! permutations become one and the same committee.
This idea applies, of course, to every set of 3 students
selected for the committee; namely that although these
3 students may be ordered among themselves in 3! ways, they
still constitute one and only one committee.
Hence, ·the total number committees of 3 students
of
1
possible from 7 students is equal to 3! of the total number
of ordered selections. Thus, the number of possible
7
P 7! 7.6.5
committees 3 = = 35.
- 4!3! 3!
3!

,'lotes. 7P
!
This number, namely 3!3 = 7 , gives the number'of uz:!OJ�red
3,4,
358
selections of sets of 3 unlike elements from .a set of 7 such
elements. It is called the number of combinations of 7
7
elements taken 3 at a time, and is denoted by c . [Other
3
7
symbols used are c , c
' (�) J.
3 7 3
7
Such numbers as c are not new to us, of course, iince
3
they were encountered in the previous chapter on the binomial
theorem for a positive integral index.

EXTENSION TO THE GENERAL CASE


Consider the case when there are n different elements
constituting Sand we require the number of unordered selec­
tions each consisting of r elements, where OS rs n.
Proceeding as before, the number of ordered selections,
each selection containing r elements, is
np _n_! - = n(n-1) (n-2) , , .• (n-r+l),
r (n-r) !
Now consider the particular r elements a,,a ,a 3 , • • • ,ar'

These r elements would form one and only one unordered selec­
tion, but would form r! ordered selections among themselves,
Since there are r! ordered selections of these particular
np
elements, then r! of the ordered selections must involve
r
the set of elements a, ,a�,···,ar. (The same principle would
hold for any.subset of r elements.)
Thus, since any subset of r elements leads to r! of the
np n.I
ordered selections, there must be
rT
r = r! (n-r) !
such sub-

sets. That is, the number of different unordered selections


(or subsets) each containing r elements, possible from a set
np
of n unlike elements is r n! This number is
r! ro! (n-r) !
called the number of combinations of n things taken r at a
time, and is denoted by ncr.

SUMMARY
( i) An unordered set of r distinct elements is called a
combination of r elements . Two combinations are
identical, if and only if, they contain the same
elements (in any order).
(II) The number of different combinations (i.e. unordered
sets) of r e I ements that can be formed from n
n
distinct elements is denoted by c , wher-e o�rin.
np r
nc _r n(n-l)(n-2) .... (n-r+I) n!
(!ii)
r r! r! r! (n-r) !

359
EXAMPLES ON UNORDERED SELECTIONS
nC
(llote l. - _____!!J_ - .!!..L = 1, since O! = 1 by definition.
n - n!O! - n!
n
2. This result, that c
r -r-,-(-!-- -r)
- !- ' has been proven in
the previous chapter on the binomial theorem. In
that particular chapter, the proof was based on the
method of induct;_i9_!!, Other results established there
n nc 10c 10c
were that cr n-r
e.g.
7 ·3
and n c nc n+ lc lOc + lO c = llc l
+ e.g.
r-1 r r 2 3 3

EXA�.-JPLE I. Six students are to be selected at random fr'om


a group of 10 students. How many such unordered selec­
tions are possible if
( i) there are no restrictions ( ii) if two particular boys
A, 8 are to be included (iii) if A is to be excluded
( iv) if A and 8 are not in the same grouping?

:vfethod
(i) H there .sire. _n9__ l;' ';!_ _E;_t;_ri£ti9fl$., we require the number of
unordered subsets of 6 students, these students being
chosen from 10 students. That is, we want the number of
combinations of 10 students taken 6 at a time. Hence,
. d number is
the. require . lOC 10! 10.9.8. 7 =
210.
6 = 6!4! = 4•3.2_1
(ii) I.f_fl ,. _ Q_ ___�.I.!i.__!:Q_ _b_!i._iriclyd.�<:I in the group of 6 students,
then we include them, This can be done of course in
only one way. After this has been done, we select
(6-2) = 4 further students from the remaining (10-2) = 8
students. The number of such unordered sets is
8c � 7 o.
4 4 ! 4!
(iii) Since A is to be excluded, we exclude him. Having done
so, there are still (10-1) = 9 students remaining from
which unordered sets of 6 students have to be obtained.
9!
There are qc 84 such sets.
6 6!3!
(iv) ;rf ,'. and 13 are not in the same gro�p, then we have the
possibilities listed (a) A in, Bnot in (b) A not in,
Bin (c) A not in, 13 not in. The number of possible
groups is for
8
(a) c ; since if A is included only 5 further students
5
are needed to fonn the group; and if B is excluded,
there remain only (10-2) = 8 from which it is to be
selected (the 2 here are of course A, B).
(b) c5; similar reasoning to (a)
8

360
UNORDERED SELECTIONS
8
(c) c6; since if both A, B are excluded, then
6 students are to be selected from (10 - 2) = 8
students.
Hence, the total number of unordered groupings possible
8 8 8
• c + c + c 56 + 56 + 28 = 140, on evaluation.
5 5 6
Alternatively, the question could be answered by deter-
10
mining the total number of possible groupings [ c from (1)]
6
and subtracting the number of groupings in which both A and B
8
are included [i.e. c from (ii)]. The required number of
4
sets in which A, B are not in the same group is
10
c6 - 8 c4 210 - 10 = 140.

EXAM"LE 2. Committees of 5 are to be obtained from a


group of 7 men and 4 women. How many such committees are
possible if
Ci) there are no restrictions
Ci i) there are 3 men and 2 women
( i i i) it is a I I ma I e
(iv) at least I woman is included
(v) there is a majority of women.

Method.

(i) The nwnber of possible aorrD11ittees, with no restrictions,


11
is c5 = 462, i.e. simply the number of combinations
of 11 distinct elements taken 5 at a time.
(ii) The nwnber of unordered seZeations of 3 men from 7 men
7
is c .. 35, and of unordered sets of 2 women from
3
4
4 women is c2 • 6. Hence, the number of ways of per-
forming these two operations, i.e. selecting 3 men and
7 4
selecting 2 women, is c x c 35 X 6 210.
3 2
That is, 210 committees are possible out of ·the poss-
ible total
( 11c = 462), in which there are 3 men and
5
2 women.
7
(iii) The nwnber of all-male corrmittees is C
= 21,. since
5
the 5 committee members must be selected from the
7 males available.
(iv) If at least 1 woman is inaluded, the various possibH-
ities are as follows: (a) lW, 4M (b) 2W, 3M
(c) 3W, 2M (d) 4W, lM.

361
UNORDERED SELECTIONS
The number of committees possible for each case is
4 4
(b) c x c = 6x35
7 7
(a) c x c = 4 X 35 = 140 210
1 4 2 3
4 7 4 7
(c) c x c = 4 x21 = 84 (d) c x c 1X7 7
3 2 4 1
Hence the required number of committees in which there
is at lea8t 1 woman is 140 + 210 + 84 + 7 = 441.
[AUemativeZ.y, the number of committees without any
7
female representatives is c by (iii), and the total
5
number of committees possible (without restriction� is
11
c from (i). He.nee the number of committees with at
5 11 7
least one woman on it is c - c = 462-21 = 441, as
5 5
before.]
(v) Ii__there is a majority of women, we consider only the
cases 3 W, 2M and 4W, ill. From (iv), (c,d) the number of
such committees is 84+7 = 91.

EXAMPLE 3. A "hand" of 5 cards is dealt out from a regu­


lar pack of 52 cards. \'/hat is the number of possible
"hands" if (i) there are no restrictions (ii) they are
al I black (iii) there are 4 aces (iv) there are 3
diamonds and 2 hearts (v) there are at least 2 kings.

Method. The number of possible hands in each case is


52 --2lL_ 52.51.so.49.48= 2,598,960, on
(i) c = =
5 5!47 ! 5.4. 3.2.1 evaluation.
26 2.§1_
(ii) c = = 65,7 80 (Note there are 26 black cards!)
s 5!21!
4 48
(iii) c x c = Ix48 = 48. (Note if there are 4 aces, then
4 1
1 other card must be selected from the remaining
(52-4) = 48 cards!)
13
(iv) c x13 c 286x7 8 22,308 (Note there are 13 diamonds
3 2

(v) = 6xl7296+4x1128+l x 48
etc.
(Note there are 3 mutually exclusive cases, 2 kings and
3 other cards but not kings, 3 kings and 2 other cards
but not including a king, 4 kings and 1 other card.)

EXAMPLE 4. There are 10 points


(no 3 of which are col I inearl;
. A
,B .ol..
• /3
•£
5 are lettered A,B,C,D,E and
and 5 lettered a,S�y,6,E. 'D c •£
• '6
Using these points, find how • 8-
362
SET 7G
many
Ci) Lines are posslale with
(a) no restrictions
(b) through A
(ii) Triangles are possible with (a) no restrictions
(b) 2 vertices denoted by capital letters and I de­
noted by a Greek letter (cl with o( as a vertex
(d) with rAs' as one side.
(iii) Quadrilaterals are possible with (a) no restrictions
(b) all vertices denoted by capitals.
(iv) Diagonals would a convex decagon have if these
10 oints were the vertices of the deca on,
Method. The required numbers are
10
(i) (a) c = 45, since every selection of 2 points pro-
2
duces one line
(b) 9c = 9 , since if A is fixed, then one other point
l,
is selected from the remaining 9 points.
l
(ii) (a) OC = 120, since every selection of 3 points pro�
3
duces one triangle.
5 5
(b) c2 x cl = 10 x s = so, selecting 2 capitals from 5
and 1 Greek letter from 5.
(c) 9c2 36, since Cl is fixed, then two other points
are to be chosen.
B
(d) C1 = 8, since �' is fixed, one point must be
chosen from 8.
10
(iii) (a) c = 21r,, since every selection of 4 points pro­
4
duces one quadrilateral.
5
(b) c = S, since the 4 vertices are chosen from the
4
5 capitals only.
lOC - 10
(iv) = 45 - 10 = 35, since
2
the number of lines possible is
i) (a)' but of these g
lOC D
2 by (
10
c lines, there are 10 sides
2
of the decagon.
·====================
EXERCISES SET 7G
6 S
Evaluate (a) c (b) 7c 7c (d) 9C (e) C
3 (c)
1.
2 4 8 s
6
c 7c x sc 4 x 3 x 2c
c c
(f) 3 (g) 2 1 (h) 3 2 1
12
c 9c
3 3 6
363
---
SET 7G
2. A committee is to be appointed in a club containing
10 members. How many committees are posi,ible, if each
committee is to have
(i) 2 (ii) 3 (iii) 4 (iv) 5 (v) 6 members?
Verify that there are exactly the same number of com-
mittees possible with 4 members as with 6 members.
3. A group of 5 prefects is to be chosen from 8 prefects,
A,B,C,D,E,F,G,H, to do canteen supervision duty. In how
many ways can they be chosen (i) .with no restrictions
(ii) the captain A is to be included
(iii) the vice-captain Bis to be excluded
(iv) A, B are to be both included
(v) A,Bare to be both excluded
(vi) A is to be included, but Bis not.
(vii) A and Bare not to be in the same group at any
time.
4. The set S= {a,b,c,d,e,f,g,h,i} is considered. How many
unordered subsets of Sare possible, if each subset con-
sists of 4 letters, and there are (i) no restrictions
(ii) 2 vowels and 2 consonants (iii) all consonants
(iv) exactly 1 vowel (v) at least 1 vowel
5. A committee of 4 is to be chosen from 5 boys and 6 girls.
How many committees are possible if there are
(i) no restrictions (ii) all boys
(iii) all.girls (iv) 3 boys and 1 girl
(v) 2 boys (vi) at least 1 girl
(vii) a majority of girls
(viii) a particular boy A is included •
. r,, There are 5 white, 3 red and 2 blue balls in a bag. In
how many ways can 3 balls be withdrawn if
(i) there are no restrictions
(ii) they are all white
(iii) they consist of 21 red and 1 blue ball
(iv) one ball of each colour is chosen
(v) 2 particular balls W1, R1 are selected.
7. In a hand of 12 cards, 3 are diamonds, 2 are- hearts,
6 are clubs, 1 is a spade. Two cards are selected at
random. What is the number of ways this can be done if
(i) both cards are red (ii) both cards are.clubs
(111) one card is red and the other is black
(iv) one card is a heart and the other a spade.
8. Consider the digits O,l,2,3,4,5,6,7,8,9. How many
4 digit numbers are possible if the digits are to be in
(i) ascending o�der e.g. 2 5 6 9
(ii) descending order e.g. 7 5 3 1.
[Hint: This is an unorder>ed selection question, since
once the digits are chosen, then each selection can be
ordered in only 1 way.]

364
APPLICATIONS OF COUNTING TECHNIQUES TO PROBABILITY
9. There are 7 points P,Q,R,S,a,b,c (no 3 of which are
collinear). Using these points, determine how many
(i) lines are possible (a) without restrictions
(8) using R as one point.
(ii) triangles are possible (a) without restrictions
(8) using S as one vertex (y) using rRs' as a side
(6) with vertices all capital letters
(c) with at least one vertex a capital letter.
(iii) quadrilaterals are possible
(a) without restrictions (8) using S and b
(iv) pentagons are possible (a) without restrictions
(8) using exactly 2 capitals. 7
10. How many different hands of 4 cards are possible from a
regular pack of 52 cards if each hand
(i) contains cards of any suit or any denomination
(ii) includes exactly 2 queens and 2 kings
(iii) includes exactly 3 hearts
(iv) includes at least 3 hearts
(v) does not contain any picture cards
(vi) consists only of clubs
(vii) consists only of cards of the same suit
(viii) each card is of a different suit
(ix) must include exactly two queens or exactly 2 kings
11. On an examination paper, there are 5 questions on Calcu­
lus and 6 on other sections of the course. In how many
ways can 8 questions be chosen if (i) exactly 4
(ii) at most 4
(iii) at least 4> are to be Calculus questions?
12. An urn contains 5 black and 6 white balls. If 4 balls are
drawn out simultaneously from the urn, how many drawings
are possible if (i) they may be of any colour
(ii) there are exactly 3 black balls
(iii) they are all white
(iv) they must be of the same colour.
13. How many diagonals can be drawn in a polygon of (i) 6
(ii) 7 (iii) 8 (iv) 12 sides?

SECTION G. APPLICATION OF COUNTING TECHNIQUES TO PROBABILITY

·otes The purpose of this section is to illustrate the


application of the counting procedures discussed in the
� revious sections, in the numerical calculation of proba�
bilities. (This section does not introduce new techniques
of counting or new theorems on probabi I ity.l
• j
These counting processes are par�icularly useful in two
cases:-
Case 1 w1um the samp Ze spaae S is Zarr,e. (Here it becomes
arduous to enumerate systematically, i.e. list fully
the various simple events constituting s.

365
SUMMARY
For exam?le, if 7 digits are to be selected one at a
time from the digits 1,2,3,4, ••••,9 (no repetitions
allowed), then the sample space would consist of
9P : 9.8.7.6.5.4.3 simple events.
7
Case 2. �!hen the exper'iment is simple in concept, but the
diversity of possible outcomes may cau se trouble.
For example, if 2 cards are selected from a pack of
52
cards there are c % 1326 possible outcomes, but
2
these outcomes may consist of 2 black cards, 2 tens,
1 ace and i king, etc.

SUMMARY
I. COUNTING TECHt� I QUES
np
(a) (i) is the number of 01•dered selections of
r
r uni ike elements, where there are n un Ii ke
elements from which to choose (Here 0 frr �n).
n n! n
(ii) P = n( n-1)( n-2)...(n-r+ I) = -(-- -,; p = n!
r n-r ) . n
(iii) The number of ordered selections of n elements,
of which x are alike of one kind, y a I i ke of
. nl
ano ther k.ind, 1s , 1 1•1here x + y + .• = n
x. y • . • •
( b) ( i)
nc is the number of unordered sele�tions of
r un Ii ke elements, where there are n unIike
elements from which to choose <Here O � r ! n).
n np n(n-l)(n-2) ..• <n-r+I). ��n�!��
(ii) cr �
r! r! r! (n-r) !;

2. PROR,'BI LI TY l'.LSULTS
.<al If the sample space S for a random experiment con­
sists of "c" simple events, and if a certain event E
comprises "a" of these simple events, then the proba­
bility that the event E wi II happen is defined to be
a/c. This is denoted: P(E) = a/c.
(b) For events A, B
(i) P(7i) = I - P(A)
(i1) P(AUB) = P(A) + PCB) - P(AB) .•. Total "addition"
law of probabi I ity.
This result is true tor all events A, B.
= P(A) + P(B) ............ True only for
mutua7.l!· exclusive events.
(iii) P(AB) = P(A).P(B) .•. True o;ly for independent
events.

366
(c) Notes
_(_i_)� P(Al is the probability that A does not occur.
(ii) P(AUB) is the probability -that A Ol" B OOOUl"S.
(iii) P(AB) is the probabi Iity that A and B both
OOOUl",
''1utuaUy exolusive events A, 8 have no simple events
,in common, i • e • P(AB l = 0 .
Independent events A, 8 are such that the happening
of one does not influence the happening
of the other.
(-dl Further, if events A1, A2, A3, ... are
( i) mutually exclusive, then
P<A1UA2UA3 ••• l =
P(A1l+P(A2)+P(A3)+•••
(i i) independent, then
P<A1A2A3 •••) = P(A1),P(A2l,P(A3l .••

uu
EXAMPLE I. In a bag are 6 red, 4 white and
3 black bal Is. Three bal Is are simul- w
taneously drawn out. 8
What is the probability that
(i) the balls are all red (ii) they are all white
(i i il they are al I the same colour
(iv) they are of different colours
(v) there are two red and I white bal I
(vi) there are exactly two white balls
(vii) there is at le<1st one white bal I
(viii) a particular ball R is included.

:>fethod'The sample space for this experiment consists ,of the


number of unordered selections of 3 balls from (6+4+3) 13 =
balls. Thus , the number of simple events in the sample space
s is
13
c3 = 286.
(i) If R is the event that the 3 balls selected are all red,
then the number of simple events comprising R is
6
c3 = 20. Hence, by the definition, P(R) 286 20
==
10
143
.

(ii) If Wis' the event that the 3 balls selected are all
white, then the number of simple events comprising Wis
4 4 2
c3 s 4. Hence P(W) = 286 • 143.

(iii) Let B be the event that the 3 balls . selected are all
black. Here we require P(R or W or B) i.e. P(RU:IUJl),
because the balls are to be·of the same colour. Since
these events are mutually exclusive, (for if they are
all red or all white or all black, then they cannot
have any colour in common), then

367
P(R or� or B) = P(R)+P(W)+P(B)

20+4+1 ...11.
-� 286

(iv) If 1 is the event that the 3 halls are of different


colours, then the number of simple events in E is
6 c x 4c x 3 c
1 1 1
72 i.e. P(E) =
2 �!

143

(v) If F is the event that there are 2 red and 1 white ball,
6 4
c x c
2 1
then P(F)
13
c
3

(vi) If there are exactly two white balls, then we have the
possibilities: 2 white and 1 red hall (event G) or
2 white and 1 black ball (event H). Since these are
mutually exclusive events, then
4c2 X 6 c1
P(GUH) = P(G)+P(G) +
13
c
3
6x6 + 6x3
286

(vii) The possibilities when tlwre is at least one white l1all


are one white and 2 red b::lls (event J); one white and
2 black balls (event K); two white and 1 red boll
(event G); two white and 1 black ball (event :;);
three white balls (event W); 1 red hall 1 white ball
and 1 black ball (event E).
P(J or !< or G or ll or WorE):. P(J)+P(K)+P(G)+P(H)+P(W)+1P(E)
4 6 4 3 4 6 4 3 4
c x c + c x c + c x c + c x c + c
1 2 1 2 2 1 2 1 3
13 6 4 3
c +! c X C X c
202 101 3 1 l 1
286 1 43
�iii) If Lis the event th�t a particular ball R1 is included
12
then the number of &imple events in·L is c (R1 hav-
2
ing been included, there remain only 2 balls to select
from 12 balls). 12
c
Thus P{L)
2 ...l.
13 13
c
3
[r:ote In this question, the 3 balls were drawl' out together;
noaccount ,1as taken of the orc.Ier in which the colours

368
appeared. liowever, if the 3 balls were "ithdrawn one at a
tune, without replacement, and a specified order required,
(say red, white then blacl<.) then the method illustrated above
is not applicable; i.e. the probability in (iv) (namely
6 4 3
cl. cl. cl 36
) gives the probability that the balls
143
13c
3
are of different colours, but does not specify the order of
selection of these colours.
To find the probability that the balls drawn are red,.�hite,
then blue, we must use the fact that the drawing of these
colours may be regarded as independent events, once the
previous draw has been made,
i.e. P(red ball. then white ball, then blue ball)
P(red ball) .P(white ball assuming a red ball has been
drawn).P(blue ball assuming both a red and white
ball have been dra�m)
�R
6 4 3 · 6
. . (see diagrams), = 3W
13 12 11 143
3B

EXAMPLE 2. Seven persons are to be arranged in a line.


What is the probabi I ity that three particular people
A, B, C (i) will be together (ii) will occupy·the
first, middle and last positions in that order in the
Iine?

Method. In this experiment, the sample space consists of


the number of ordered selections
of 7 persons taken 7 at a time. [z) [ill[] G] [111}] []

This.number is P 7!
7
(i) If Eis the event that the people A,B,C
are together, then the number of simple events compris­
ing Eis 3!5! (The 3 people A,B,C may be arranged
among themselves in 3! ways. When this has been done,
the group A,B,C is regarded as a unit, and the unit
together with the other 4 persons constitute a group of
5, which may be ordered in 5! ways. Hence, by the
basic counting principle, the number of ways of doing
these operations in succession i1 3!5!).
31 5 1 1
Thus, by definition, P(E ) = � =
7

(ii)
� D D � D D �
The marked positions are to be o<;cupied by A, B and C
in that order. There is only one way in which A,B,C can
be placed in these positions. The remaining four people
may be arranged in the unmarked positions in 4! ways.
369
SET 7H
Hence, if Fis the event that A,B,C occupy respectively
the first, middle and last positions in the line, then
4! 1
P(F)
7! = 210·

EXAMPLE 3. From the digits, 1,3,5,6,S;numbers are formed,


each one being less than 6000 (no repetitions al lowed).
If one of these numbers is selected at random, what is
the probability that it is even?

Method. Numbers less than 6000 may comprise either 4,3,2 or


---1 digit.
( a) 4digit nwnbers [l]G][}][I] The first digit must be
either 1, 3 or 5, i.e. there are 3 selections possible
for the first digit; the other digits may be ordered in
4 x 3 x 2 ways.
Hence, there are 3 x '• x 3 x 2 = 72 such numbers.
(b) J digit numbers W [}] Q]
There are 5 x 4 x 3 = 60 such numbers.
<c> 2 digit nwnbers rn GI
There are 5 x 4 = 20 such numbers.
(d) 1 digit nwnbers I]] There are 5 such numbers.

Thus, the sample space S consists of


(72 + 60 + 20 + 5) = 157'numbers.

Let E be the event that the number selected at random


is even. Then E consists of 4,3,2 or 1 digit numbers.
(a) 4digitnwnbers Ll]uJGJw The first digit is select,-
ed from 1, 3 or 5; and the last digit is selected from
6 or 8. The middle digits may be ordered in 3 X 2 ways.
Hence, there are 3 x 3 x 2 x 2 36 such numbers.
(b) J digit numbers ill uJ W The last digit is selected
from 6 or 8. Hence there are (4x3)x2 = 24 such numbers.
(c), (d) Similarly, there are 4 x 2 = 8 two-digit and 2 one­
digit even numbers.
Thus, the event E consists of (36+24+8+2) = 70 even
numbers < 6000, Hence, by definition, P(E) = 70/157.

EXERCISES SET 7H
1. An urn contains 4 blue, 3 red and 5 yellow balls. In
how many ways can 3 balls be simultaneously selected at
random from the urn? Hence, find the probability of
drawing out (i) 3 blue balls (ii) 3 yellow balls

370
SET 7H
(iii) either 3 blue or 3 yellow balls
(iv) 3 balls of the same colour
(v) 2 red balls and 1 yellow ball
(vi) 1 red and 2 yellow balls
(vii) balls of different colours.
2. A committee of 4 is to be formed from 5 boys and
6 girls. What is the probability that the committee will
contain · (i) all boys
( ii) all girls (iii) 3 boys and 1 girl
(iv) 2 boys and 2 girls (v) 1 boy and 3 girls
(vi) at least 1 girl (vii) at most 1 girl
(viii) a particular girl G.
3. A box contains eight discs marked a,b,c,d,e,f,g,h. Two
discs are drawn out one at a time (i.e. without replace­
ment; order is important). How many different possible
outcomes are there (i.e. how many points are there in
the sample space for this experiment)? Assuming that
each disc is equally likely to be drawn, find the
probability that
(i) discs b, c are drawn in that order
(ii) discs b, c are drawn in any order
(iii) disc b is one of the discs drawn
(iv) either disc b or disc c is drawn (but not both)
(v) both discs are marked with vowels
(vi) both discs are marked with c0nsonants
(vii) a vowel and a consonant are on the discs drawn.
4. The hearts in a pack of· cards are separated and
shuffled. Four of the 13 cards are drawn together; what
is the probability that
(i) the 4 cards are the jack, queen, king and ace
(ii) the king is among the four
( iii) the two and three are not included
(iv) the five is included but the seven is excluded
(v) both. the jack and queen are among the cards drawn.
5. If 5 coins are tossed simultaneously, what is the poss-
ible number of outcomes? What is the probability that
there are ( i) O heads
(ii) 1 head (iii) 2 heads
(iv) 3 heads (v) 4 heads
(vi) 5 heads (vii) at least 2 heads.
6. A bag contains 5 apples and 6 oranges. Two pieces of
fruit are selected one at a time; what is the chance
that ( i) 2 apples are drawn
(ii) an apple and an orange are drawn
(iii) at least 1 apple is drawn.
7. From a regular pack of 52 cards, two cards are selected
simultaneously(without replacement). What is the proba­
bility that (i) they, are both kings
(ii) they are both hearts (iii) they are both

371
SET 7H
picture cards (iv) they are the ace of clubs and the
ace of hearts (v) they are both red (vi) one· is red
and the other is black (vii) one is a heart and the
other is a diamond (viii) one is a ten and the other
is a queen (ix) one is a spade (x) at least one is a
spade.
8. Five girls and 6 boys are arranged in a line so that
each girl stands between 2 boys. How many ordered
selections are there? What is the probability that two
specified boys A and B should stand at the ends of the
line?
9. Six girls A,B,C,D,E,F are to seat themselves in a row.
�lhat is the probability that (i) two friends A, B are
together? (ii) three friends A,B,C are together?
10. (a) A three-digi!: integer N is formed from the digits
1,2,3,4,5 (no repetitions being allowed). Find
(i) P(;,1 is odd) (ii) P(N is even)
(iii) P(!, ends with 3 or 4) (iv) P(!l is divisible
by 5) (v) P(N = 341)
(vi) P(N = 300) (vii) P(N > 300)
(viii) P(N < 300) (ix) P(N begins with
5 and ends with 1)
(x) P(N contains the digit 4).
(b) Repeat (a) if repetitions of digits are allowed.
11. In a launch, there are 8 seats; 4 on either side. Two
persons A, B wish to lie on one particular side and an­
other person C wishes to be on the other side. If the
seats are filled at random, in what proportion of the
total number of '.�ays will the 3 persons A,B,C be satis­
fied?
12. The digits 1,2,3,4,6 are used to form numbers, no rep­
etitions being allowed. How many numbers are possible?
(Note they may have 1,2,3,4 or 5 digits,) If one of
these numbers is selected at random, what is the proba­
bility that it is odd and·over 4000?
13, In a group of 15 men, there are 7 teachers, What is the
probability that if 12 men are selected, then there are
(i) exactly 6 teachers (ii) at least 6 teachers.
14. A group of 6 girls are to be chosen from 10 girls. Find
the probability that
(i) 2 particular girls A and B are included
(ii) A and B are both excluded
(iii) A is included but B is excluded
(iv) neither l'. nor B are in the saUle group together.
15. l-lumbers of 4 digi,ts are to be formed from the digits
0,1,2,3,4,5,6,7,8,9 (no repetitions being allowed).
What is the pro!Jai:>ility that the number formed
(i) is greater than 6000 (ii) is less than 3000
372
SET 7H
(iii) contains zero (iv) is a multiple of 10
(v) is not divisible by 10. A
16. The plane is divided into 3
regions, (area X, area Y,
area Z) by the rays OA,Ol3, <1ifo. Y · tkax
OC. If there are 5 points
in X, 3 points in X and
2 points in Z, (no three of
these points are collinear)
(a) find how many triangles
are possible from these B �Z C
points. If a triangle is selected at random from
tl,ese, what is the probability that it will have
(i) all vertices in area X
(ii) two vertices .in area X and 1 vertex in Y
(iii) one vertex in each of the areas
(iv) at least 2 vertices in Y.
U) find i1ow many quadrilaterals are possible from these
points. If a quadrilateral is selected at random,
what is the probability that it will have
(i) all verti�es in X
(ii) two vertices in X, one in Y and one in Z
(y) find how many intervals ·are possible from these
points. If one of these intervals is selected at
random, what is the probability that it does not
cross any of� the.3
_ �ys OA, OB, OC?
17. The letters of the word INTEGRAL are arranged in a row.
(a) How r,1any different arrangements are there? If one of
these arrangements is selected at random, what is
the probability that
(i) it ends with L and begins with I
(ii) the vowels are in the same positions
(iii) the vowels are together
(iv) the vowels are together and the consonants are
together (v) ti1e 11ord formed is INTEGRAL
(fi) In how many ways can 3 consonants and 2 vowels he
selected from the letters of I!-ffEGF.AL. What is the
probability that the letters T, E are chosen?

18. Five discs have the digits 1,1,2,3,3 on them. If the


discs are placed in ·a line, how many different numbers
may be formed? If one of these is selected at random,
what is the probability that it is (i) even (ii) odd?

373
THE BINOMIAL DISTRIBUTION
SECTION H. BINOMIAL PROBABILITIES AND
THE BINOMIAL DISTRIBUTION
(a) Notes.
_l. In this section, we are concerned with random experi­
ments involving repeated t1•ials, where there are only
tu.,o possible outaomes (which remain fixed throughout
the series of trials). Included in this category
would be such experiments as the following:
(a) repeatedly tossing a aoin, to determine the
probability of the appearance of a certain number
of heads. (Here the only two possible outcomes
are a head (H) or a tail (T), and these remain
constant no matter how often the coin is tosned,
i.e. assuming the coin is perfectly symmetrical,
then P(H) = P(T) = -}. )
(b) repeatedlµ rolling a die, to calculate proba­
bility that a certain number of times a 3 appears.
(Here, the only two possible outcomes are a 3 or
not a 3. Further, (for a perfect die) P(3) = 1/6,
and P(not a 3) = 5/6 remain the same for every
roll of the die.)
(c) �rom a cards,
w1,t,1 re er ever,· ara;.'. t
two outcomes are either a picture
picture card.
12
Further, P(picture card) = = 3 52 13 whilst P(not
. 40 10
a picture card) = = remain constant for
52 13
every draw no matter how many are made.)
(d) aJ,oosin
4 white an

or is
white. The probabilities of these are ·different,
namely P(W) = 4/11, P(w) = 7/11, but remain con­
stant for each draw, since the ball is replaced
eaclt time.)
(e) prediating the sex of unborn oanes (The two
possibilities are male or female. In our work we
nonna.Zly assume each is equally likely, but in
aatual fact, the probabilities of each occurring
are not both exactly 1/2. On the latest figures
available to the author, the probability of an
unborn baby being a boy in Australia is near to
·514 and of being a girl is near to •486.)
(f) !'lantin� a aertain variety of seed and studyi
1J/!;
the pro, abititf,' that a aertain number of see
germinate. (Here, the only possible outcomes ate
that the seed germinates or that it does not
374
THE BINOMIAL DISTRIBUTION
germinate. Obviously the chances of these occur­
ring are not usually equal. The actual probabili­
ties here depend on many factors, such as soil,
water, condition of the seed, etc, These proba­
bilities cannot be de.cided without experiment in
the first instance, but once they have been
decided, we assume that they remain constant for
every one of the p:c1rticular variety of seed being
considered.)
(g) checking the nw11ber of d.efective parts produced
b!' a machine in a factory,. (Ilere the part being
checked is either defective or not defective.
Again the probability of these occurring can only
he approximated to by repeated trial. Obviously,
in normal circumstances in mass-production the
probability that a part is defective is very
small, say ,OS (5%) or less, and the correspond­
ing probability that a part is not defective is
nearly unity, say 0,95 (95%) or more.)

�: In each case stc>.ted above, there 1-ere only t1w possible


outcomes for each trial (namely, the trial resulted in
the property under consideration, or it did not. A trial
resulting in the required property is usually called a
success (S,) whereas a trial not resulting in the re­
quired property is called a failure (F). The probabili­
ties of S and F are either easily deduced as in
(a), (b), (c), (d), assumed as in (e), or else derived
approximately by repeated trial as in (f), (g), Thus,
the probabilities quoted in (a), (b), (c), (d) are
exact mathematical probabilities, assuming perfect coins,
dice, etc, but those stated in (e), (f), (g) are approx­
imations to the required probabilities. In all cases,
however, once the probabilities of Sand Fhave been de­
cided, then these remain constant for each trial under
consideration. Of course, in practice, such proba­
bilities as in (f), (g) are re-assessed continually.)

]. Further, in .each case illustrated above where a trial is


repeated, the trials or events are independent of one
another. (Thus, the fact that one toss of a coin yields
a head has no effect on the result of the next toss or
of any toss. Similarly, the germination of one particu­
lar seed exerts no influence on the germination or
otherwise of any other seed,)

4. Experiments in which
(al the chance of success (and therefore, of failure)
is the same on each trial,
g:n_jj. (bl the chance of success on any particular trial
w i I I not be affected by the success or fai I ure of
375
BINOMIAL DISTRIBUTION
any other t rial
are aaZZed "JJinomiaZ ,x,;eriments", since the
binomia Z theorem formu I a app I i es to
them. In other
words, we can apply the binomial theorem (discussed
in the previous chapter) to experiments involving re­
peated trials given
( i) that P(S) and P(F) are aonstants for each trial and
(i i ) that the tria Is or events are indepenclen t of one
another.

(;l) Consider the following experiment. A student selects


answers at random in a quiz containing 5 true-false
questions. i�hat are the various probabi I ities of ot,-
taining ·),1,2,3,4,5 correct?

.',:ethod. These probabilities mny i:>e obtained by various


methods; the probabiZit-,.; tree met/;od is illustrated
here. The probability of selecting a correct answer in
any question is 1/2, .•. P(C) = P(I) = 1/2, where C is
the event that the student selects the aorreat answer
and I is the event that he selects the inaorrect answer.

This question shows an application of equal probabi Iities.

Q5

Q4
Ouestion 3 (Q3)
Question 2 (Q2)

Question 1 (Ql)

�----- 2

liotes.
1. On every branch, there should appear 1/2.
2. 'fnere are 2 5 = 32 possible simple events, whicll are ob­
tained i>y following the branches through from start to
question 5.
3. The probability of a combination such as CCIIC occurring
1 1 1 1 1 1
is
2 , 2 , 2 . 2 , 2 = 2s, This combination of 3 C's and
2 I's occurs, in ant' order, exactly 10 times. (Other
such combinations are ICICC, CIICC etc.) Hence, the
probability of 3 correct and 2 incorrect answers (in any
1 10
order) is thus 10 . 2 s = 32.

376
BINOMIAL DISTRIBUTION
Use the diagram to complete the following table:-
0 I 2 3 4 5
Combination _orrect �orrect orrect correct correct orrect
i.e. i.e. i.e. i.e. i.e. i.e.
11111 II11C IIICC IICCC ICCCC CCCCC
etc. etc. etc, etc.
How many 5
combinations I

Probab i I i ty I
of each _!_
32 32
combination

Prob ab i I ity 5
of al I such _!_
32 32
combinations

Verify that
(i) the sum of the probabilities on the last line is 1
(ii) the numerators of the fractions in the last line form
the binomial (or Pascal triangle) pattern
5 5 5 5 5 5
1 = c o• 5 = c 1 10 = c2 10 = c 3• 5 = c 4, 1 = Cs·

'iii) If Xis the number of questions correctly selectedat


random by the stud
ent, then
(a) P(X= O), i.e. the probability of O correct answers
(and.'. 5 incorrect ones)
=_l=SC (la)S(J2)0
32 0
(b) P(X= 1), i.e. the probability of 1 correct answer
(and.·. 4 incorrect ones)
= ...2 =SC •,(k2)4 (k2)l
32 1
(c) P(X = 2), ·i.e. the prohability of 2 correct answers
(and • •• 3 incorrect ones)·
= 10 =SC (k2)3 (k2)2
32 2
(d) P(X= 3), i.e. the probability of 3 correct answers
(and.·. 2 incorrect ones)
= 10 =Sc (k2)2 (k2)3
3
32
(e) P(X = 4), i.e. the probability of 4 correct answers
(and.·. 1 incorrect one)
s
= � = c (la) i (la) !+
3 4
(f) P(X= 5), i.e. the probability of 5 correct answers
(and.·. 0 incorrect one)
s
= 3� = c (la) O (la) S
s

377
BINOMIAL DISTRIBUTION
(iv) Remembering, using the binomial theorem, that
S S S 4 1 S 3 2 S 2 3
(q+p) = q + C q p + C q p + C q p
1 2 3
S 1 4 S
+ C q P + P
4
show that the probabilities listed in (iii) are the
suooessive terms on the right-hand side of this expan­
sion, assuming p = P(C) =�and q = P(I) = �.
(v) The variable X which can take only the 6 values
0,1,2,3,4 ,S at random is called a random variohle.·
This term has already been used earlier in this chapter
on probability. 1-iere the probabilities associated with
1 10 10
X = 0,1,2,3,4,S are
32 , 32S , 32, 32, 32S , 321 respective-
ly.

Exe1•oise (ex) Assuming that there are only 4 questions, use


the tree above, to show that the respective probabili­
ties of obtaining 0,1,2,3,4 correct answers are the
successive terms in the expansion of (q+p) 4 where q=p=\2.
(S) With 3 questions, show the respective probabil­
ities of obtaining 0,1,2,3 correct answers are the
successive terms in the expansion of (q+p) 3 where q=p=�.

(y) Consider the experiment in which a bal I is selected


at random from a bag containing 30 black bal Is and
20 white bal Is. The bal I is replaced after each
draw, and the process is carried out 7 times. What
is the probability of obtaining a black bal I on
exactly 4 occasions?

Method. On every draw, since the selected ball


is replaced every time, the probability of J
�O� J

selecting a black ball (event E say) remains



constant at • ..P(P)
l' i.e � - l
- s· Also, the
S
probability of not selecting a black ball, (event F.) re-
2 2
mains constant at ' i.e. P(m
S s·
Note here P(E) f. P(E), i.e. the probab·ilities of success
and failure are unequal.

If the prooess is oarried out 7 times, the possible


outcomes consist of 7 black balls, 6 black and 1 white ball,
S black and 2 white balls, ••• , 7 white balls. Each of these
outcomes is listed below (order is immaterial).
BBBBBBB BllBllBBW BBBBBWW BuBBWWW
BBllWWWW BBW\v'WWW BWWWWWW WWWWWWW
ho1,;eve1°, these possible outco1,1es do not occur with the
same probability; ( t:1us, we would expect, by common sense,
378
THE BINOMIAL DISTRIBUTION
to obtain say, 4 black and 3 white balls more frequently than
say, 7 black balls).
In our question, we requir>e the p1•obabi lity of obtaining
the aombination BBBBrvf,'f./ in � order. Examining this case
more carefully, we see that exactly 4 black balls may occur
in ·many ways. Some of the ordered sets (i.e. distinct
patterns) which are possible are
BBBBWWW BWBWBWil BBWWWBE WWBBilWB
WBWBBBW WBBBBHW BWWBBWB WWWBBBB
(For example, the ordered set, say J3BW1vWBB, means the
first 2 ·draws result in black balls, the next 3 draws in
white balls, then the last 2 draws in black balls.)
There are two problems aonfronting us
(i) to decide on the probability of � such ordered set or
pattern
(ii) to decide on how many such patterns are possible
(i.e. in how many ways 4 black and 3 white balls may
come about in the 7 draws).
(i) The first Droblem is to decide on the probability of
any one such pattern, e.g. consider the ordered set
BWWBBWB.
Here P(BWHBBWB) = P(B).P(H).P(W).P(B).P(B),P(W).P(B)
since they are independent events
3 2 2 3 3 2 3 = 1 4 l 3
5 5 '5 s·s·s·s ( ) ( )
5 5
(The result is the same for any pattern involving
B,B,B,l:S ,W,W,!n.
(ii) Our next problem is to decide exaatly how many such
ordered sets are possible. To do this, we employ a
counting procedure discussed previously, namely the
number df ordered sets (i.e. permutations) possible
from n objects (or elements), of which x are alike of
one kind, y alike of another kind, and so on. The re-
sult is
x!�;...
Hero, we want the number of ordered
sets involving B,B,B,B,W,W,W. by the counting technique
stated, the number of distinat patterns would be
7!
( 3 ). Note that this number is exaatlu the same
4131 = 5
as the nwnber aombinations of 7 differont things taken
four at a time; namely 7c .
4
7
Finally, each of these c4 = 35 patterns, if now called
events, are mutually ex_a_lusive, since the appearance of any
one, such as WBBBWWB precludes the possibility of the oc­
currence of any other such pattern or event. We require the
probability of one of these patterns (i.e. events) to occur;
i.e. we want P( BB BBWWW or BWBWBWB or WWBBBWB or , ...........)
379
THE BINOMIAL DISTRIBUTION
Since these events are mutually exclusive, this result is
equal to l'(BBBBWWW) + P(BWBWBWB) + P(WWBBBWB) + ...
7
) ) 3 + <i)4<f)3 + <i)4<f)3 + "· to c4 (=35) terms
= <i '+cf

7 C4 <l) <t) 3,
I+

'.. hus, the probability of obtaining a black ball on


n

exactly 4 occasions, in 7 successive draws (one ball at a


time, the ball being replaced after each
. draw) from a bag
2
containing 30 black and 20 white balls is 7 c 3 4 (5) 3 •
s)
4<
Now the binomial expansion of (q+p), ; the 7 ap-i:,ears be-
cause cif the 7 draws; is
5 2 4 3 4
(q+p)' = q + 7c q6 p + 7c2 q p + 7c q P 3 . + 7c4 q P
7 l 3
2 5 6
+ 7 q P + 7c6 q P + P 7
cs
In our oase, let p be the probability of drawing a black
2
ball on any draw i.e. p = 1 and let q = 1-p = �. be the
5. .J

probability of not dra1·dng a black ball on any draw.


Thus the binomi.al expansion above becomes
('r + 1) 7 (1) 7 + 7c (.?..)6(1)1 + 7c (l) s (l) 2 + 7c (1)'+(1)3
5 5 5 l 5 5 2 5 5 3 5 5
7c (1)3(1)4 + 7c (l 2(l s 7
+ ) ) + c ( 1)1(1)6 + (1)7
'4 5 5 5 5 5 6 5 5 5
The tenn underlined represents the required probability,
i.e. the probability of selecting a black ball on exactly 4
occasions in the 7 draws!

Note
� The successive terms of this expansion denote the re­
spective probabilities of selecting a black ball on
exactly 0,1,2,3,4,5,6,7 occasions in the 7 draws.
Thus, the probability of selecting a black ball on
exactly
(a) 0 occasions (and a white ball on 7 occasions) is (1 )7
5
(b) 1 occasion (and a Hl,ite ball on 6 occasions)
is 7cl 6 l<i) <l)
(c) 2 occasions (and a white ball on 5 occasions)
is 7c f) s (1) 2
2<
(d) 3 occasions (and a white ball on 4 occasions)
is 7 C (l) I+ (1) 3
3 5 5

(h) 7 occasions (and a white ball on O occasions) is (1) 7


5
3 80
THE GENERAL RESULT
Henae, in gen eral, the probability of C: rawing a black
ball on exactly k occasion s (whe re 0 $ k $ 7), is
7 ,2_ 7-k 1 k
C ) ( )
k ·s s
(ii) If x den otes the num�e r of occasions on which a black
ball is drawn , then the va riable X may take the values
0,1,2,3,4,5,6,7 at ran ciom. The randon. variable X takes
these values with probabilities 7, 7c <i)
( 1,
1 (f) 6 !)
7
c (f) 5 (i) 2 , ... , <i)
7 respectively.
2
(iii) Sin ce one of these events is aertain to occu r, the
sum of these probabilities should be unity. This can
be easily seen from the binomial expan sion above be­
cause the sum of these respective probabilities is
(5 + 53 )7
2
.
i.e. (1)7= 1.

C Ii) The General, Case

Consider a trial in which the probabi Iity that a cer­


tain eve n t E happe n s is p, and the probabi Iity that E
does not happen is q, whe re q = 1-p. Assume that an
expe rime n t co n sists of n t rials, �here P(El = p, P(rl = q
remain constant fo r each trial. The p robability that the
eve n t E may occur o n exactly r occa_sions; and thus fai I
to occur on ( n- r l occasio n s, •11he re O .:: r .:: n, is
n n -r
cr q p
r

This result may be written more aonaise l,y as follows:

If, o n each trial, P(El = ;:,, P(E) = q = 1-::i, the n the


p r obability that the eve n t E ,,ill occur exactly r times
n- r
in n indepe n de n t trials is c q p .
n r
r

* Outline of Proof
The r events p r oducing L may occur du r ing the n in-
· · n = n! ·
dependent tria 1sin c combinations. r:ach of
r r !(n -r)!
thesi:: combinations is an event which is mutually exclusive of
the othe rs, an d each such combination has a p robability given
n-r
by p q , since E occurs exactly r times and thus E occurs
r

exactly ( n- r ) times.
Now the re a r e c mutually exclusive combination s, each
n

n -r
with probability p . q , such that fo r each of these com­
r

binations, L occu r s exactl�· r times. l!ence, the probability


of one o r an other of these combinations is the sum of their
separate probabilities.
381
EXAMPLES ON BINOMIAL DISTRIBUTION
n -r· r
trials)= C· q
n
i.e. P(E occurs exactly r times in n p .
r
[THis expression is the (r+l)th term of the expan sion of
(q+p)n in ascendin g powers of p.]

Notes.
� If X den otes the n umber of times the even t E occurs in
the experiment above, X may take on the values 0,1,
2 , 3, • , , , r, , .. , n at ran dom, X is called the ·random
variable of the binomial distribution, and the proba­
bilities associated with X = 0,1,2,.•• ,r, .•., n are the
successive terms of the bin omial expansion (q+p)rl, where
P(E) = p, P(E) = q, i.e. p+q = 1.
n n n l
Thus P(X 0) q , P(X = 1) = c q - p,
1
P(X 2)
n
c q
2
n -2 2
p '
n-r r
...
' P(X = n) = p
n

anc! in general, P(X r) = ncr q p ' 0 � r � n.

(ii) If a variable X obeys the laws defin ed above; i.e. if


the probabilities associated with X = 0,1,2 ,.••,r••., n
are distributed according to the law
n n -r r
P(X = r) = c
r q P ,
the random variable A · is often referred to as a
binomia Z variab Ze.
n
(iii) The binomial expansion for (q+?) can be expressed
n l n -2 2 n �r r ·n ·
+ C q - p+C q
n n n n n
(q+p) = q p + +C q p + ..+ p
1 2 r •
=P(X=O)+ P(X=l) + P(X=2) + + P(X=r) +... +P(X=n )

EXAMPLES ON THE BINOMIAL DISTRIBUTION

EXAMPLE I. A coin is tossed nine times. What is the


probabi I itv of obtaining
(i) 2 heads, 4 tails, 2 heads, I tail in this order
(ii) 4 heads and 5 tai Is
(iii) exactly 8 heads (iv) nine heads
(v) at least 7 heads
(vi) not less than 2 heads.

:'-.'ethod Here we have a series (namely 9) of trials


(i.e. tossing a coin ) which are in dependent of each
other, an d further the probability of a head, P(H), on
every toss remains the same (1/2). The bin omial distrib-
ution can be thus applied.
Hence, the successive terms of the expan sion
(q+p)n which represen t the probabilities of exactly 0,1,
2,...,n successes in n independent trials, can be adapt�
ed to this problem.

382
EXAMPLES ON BINOMIAL DISTRIBUTION
We have n .. 9 , p = P(H) = t, q = P (TI) = 1 - t = t. Thus
the successive terms of (i + t) 9 here represent the
probabilities of exactly 0,1,2, .•• , 9 heads in 9 tosses
of a coin.

(i) If the coins are req uired in a definite order, the bi­
nomial result cannot be di rectly used. In this case, to
find P(HHTTTTIIHT), we note that
P(HHTTTTiiHT) = P(H) .P(H) .P(T) ,P(T) .P(T) .P(T).P(ll) .P(li) ,P(T)
1 1 1 1 ·1 1 1 1 1 1 1
=2·2·2·2·2·2·2·2·2 = � =
512

If X denotes the number of heads, then X = 0,1,2, .•.• ,9;


r r 9 9 r
and P(X=r) = 9C q9- P
r
r c (i) - (t) = 9c (i)9 .
r r
9 9 4 4
(ii) P(X=4) = c (t) - (t) = 126 .
4
= ��,
2
zi
9
since c = 9! = 126
4 4151
This result means that in 9 tosses of a coin, the proba­
hility that exactly 4 heads wtll appear is 63/256.

Thus the
p robability of obt·aining exactly 8 heads is 9/51 2 .
1
(iv) P(X = 9 ) = (.l.)q
,. - _ _, i - · the p robability that all
- 512 . �
coins will fall heads is 512.

(v) P(X � 7) P(X= 7 o r X=8 or X=9 )


[At least 7 here means 7 or 8 or 9 )
P(X = 7) + P(X = 8) + P(X = 9 ),
since these events are mutually exclusive
2 l
7
(t) (i) + c <t) (t) 8 + (i) 9
9
8
[36 + 9 + 1) _11
2 56'
noting- 9c = 36 '
9
8
c = 9
7
i.e. the probability of at least 7 heads on the 9 tosses
is 23/256.

(vi) P(X � 2) = P(X = 2) + P(X = 3) + P(X = 4) + ... + P(X � 9)

This method requires a great deal of arithmetic. The


calculations are easier if we note that
P(X = 0) + P(X = 1) + P(X = 2 ) + P(X = 3) + ..+P(X=9 ) = 1
9 9 8
Thus P(X � 2) = 1 - {P(X=O) + P(X=l)} = 1 - {(t) + c 1(t) (t)�
1 9 251
= l - {512 + 512} = 256.
i.e. the probability of not less than 2 heads in the 9 tosses
is 2 51/ 256.

383
EXAMPLES ON BINOMIAL DISTRIBUTION
Notes
� Instead of tossing one coin nine times, the same re­
sults bJould be derived if nine coins were tossed simul-
taneously.
(ii) The expansion of <t+t)9 has been used here to find
probabilities involving the number of heads in 9 tosses
of a coin.
The same expansion can be used in various other prob­
lems such as determining probabilities involving the number
of:
(a) questions answered correctly in a true � false quiz of
9 questions bJhere each question is answered at random,
i.e. without thought (e.g. with a pin, etc.). Here
p = t, and q = -l: are the prcbabilities of obtaining the
correct and incorrect answer to any question..
(b) in a family consisting of 9 children. Here p =�and
q = � are the probabilities of a child being a boy and
not a boy (i.e. a girl).
(c) times an odd number turns up when a die is cast
9 times. Here p = t and q = t are the probabilities of
obtaining an odd number (1,3,5) and not an odd number
(i.e. an even number 2,4 or 6),
(d) occasions a red ball is selected frcm an urn containing
20 red, 12 green, 5 yellow and 3 white balls, where the
ball is replaced a fter each drm; and there are 9 draws.
Here p = -l: and q = -l: are the probabilities of obtaining
a red ball and not a red ball.
(e) times a black card may be selected in 9 draws from a
pack of cards (with rep Zacement after each draw). Here
p = t and q =t are the probabilities of selecting a
black card and not a black card (i.e. a red card).

In each case stated above, each of the terms of the ex-


pansion of (t+-l:)9 refers to a certain probability.
9
Thus, for example, the term c (t)5 (t} 4 = �� gives the
4 2
probability of
(a) obtaining exactly 4 questions correct in the true-false
quiz of 9 questions
(b) there being exactly 4 boys in a family of 9 children
(c) tossing an odd number exactly 4 times in 9 tosses of a
die
(d) drawing (with replacement) a red b�ll exactly 4 times in
9 draws from an urn containing 20R, 12G, SY, 3W balls.
(e) selecting (�ith replacement) a black card exactly 4
times in 9 -draws from a pack of cards.

(iii) It is obvious that when n is large or p is an awkward


fraction or decimal, then the actual calculation of
probabilities is very arduous. Tables giving the

384
EXAMPLES ON BINOMIAL DISTRIBUTION
various probabilities for different values of n, and
differing p are available. These make caiculations
far easier. (In the exercises set below, all except
those clearly indicated, are intended to be done with­
out use of any tables except squares, cubes,)
(iv) In actual_p_raatiae_, when n is large, the binomial dis­
tribution of probabilities is usually replaced by other
distributions. F o;: example, if n > 50, the binomial
distribution approximates very closely to another dis­
tribution known as the normal distribution. The various
probabilities are readily derived from the normal dis­
tribution, whereas the binomial distribution leads to
arduous calculations.

EXAMPLE 2. At an election 30% of the voters favoured


party A. If an interviewer selected 5 voters at random,
what is the probabl lity that
(i) exactly 3 of them favoured party A
(i il a majority of those selected favoured party A
(iii) at most 2 favoui·ed party A?

Method The selection of a voter at random can be regarded as


a trial, and the selection of 5 voters at random as
5 independent trials. The probability (p) that each
voter selected favours party A is 3/10 (i.e. 30%), and
the probability (q) that each voter selected does n ot
favour party A is 7/10 (i.e. 70%). Hence, if X rep­
resents the number of voters in the group of 5 favour­
ing party A, then X may take the values 0,1,2,3,4,5 with
probabilities given by the successive terms of the ex­
n
pansion of (q+p) , where n = 5, p = 3/10, q = 7/10.
5 5 3 3 49 27
( i) '.llow P(X=3) C (_]_) - (·..l) lO '
3 10 10 100 ' 1000
13230
= '1323 13· 23��
100., 000
i.e. there is a 13'23% chan ce that exactly 3 of the 5
voters selected favoured party A.
(ii) If a majority of those selected are to favour party A,
then X = 3, 4 or 5.
Now P(X=3 or X=4 or X=5) P(X=3)+P(X=4)+P(X=S)
s s
c (-1.)2(_..l)3+ c (-1.)1(_..l)i++E!-)s
3 10 10 4 10 10 lD
13230 2835 243
+ +
100,000 100,000 100,000
·1323+·02835+•00243
·16308 16·308%
[Alternativefy,P(Y.=3 or 4 or 5) = 1 - P(X=O or 1 or 2) etc.]
Thus the probability that a majority of voters favoured

385
SET 71
party A is ·16308, i.e. the chan ce of this event occurrin g is
16 • 308%.
(iii) If at most 2 favour party A, then X = 2, 1 or O.
Now P(X=O or X=l or X=2) l-P(X=3 or X=4 or x�S)
l-·16308, from (ii) above
• 83692 = 83 · 692%
Thus, there is an 83·692% cha nce that at most 2 of the
group of 5 selected will favour party

EXERCISES SET 71
[To speed up calculations in this set of exercises, use
should be made wherever possible of expansions already worked.
Some exercises have been deliberately framed to en able pre­
vious expansions to be employed.]
1. In a certain trial, the probability that a certain event
E occurs [i.e. a success (S)] is p, whilst the probabil­
ity that E does n ot occur [i.e. a failure (F)J is q,
where q = 1-p. The trial is repeated, each trial being
indepen den t of any other.
- - P(SS) p
- -P(SF) pq X 0 1 2
Start
- -P(FS) qp P (X) q 2qp p2
2
__ p(FF) ql
(i) Verify, from the probability tree shown, that the
probabilities of exactly 0,1,2 successes in 2 in ­
dependen t trials are q2 , 2qp, p 2 respectively.
Hen ce, if X represents the number of successes in
the 2 trials, verify the en tries in the given
table.
(ii) Extend the probability tree to in clude
(a) 3 trials (b) 4 trials
Obtain for each of (a) and (b), a table similar to
the one above.
(iii) Check that the successive en tries in the bottom
line of each of the 3 tables are the successive
terms in the bin omial expan sion (q+p)n where n = 2,
3,4 respectively.
(iv) For the case of 4 trials, write down expressions
for the probability of (a) 4 successes
(b) four failures (c) exactly one success
(d) exactly on e failure (e) exactly 3 successes
(f) at least 2 successes (g) at most 2 successes
2. A contest in volves the selection of a box from 3 identi­
cal boxes. The box is replaced after each selection, and
the result of the selection is n ot kn own by any other

386
SET 71
contestant. If the contestant
selects a particular box, he
i 1
or she is awarded a prize. ���)
Assume that the probabilities / - /)- - /)- -
p and q of winning and not
winning a prize are respectively 1/3, 2/3.
(a) Two contestants are each asked to select a box.
Use the results of question· 1 (i) to determine
the probabilities (as numerical fractions) that
0,1,2 contestants win a prize.
(b) Use the results of question 1 (ii) to calculate
the various probabilities for (a) 3 contestants,
and (e) 4 contestants to win prizes.
(c) Extend the results to Number of
0 1 2 3 4 5
complete the table prize-winners
shmm, if 5 contest­
ants are to select a Probability
box. What is the Slli"
of the probabilities
on the bottom line of the table? Why should this
be so?

3. A coin is tossed 6 times, 'Jse the binomial expansion


(q+p) 6 where p is the probability that a head appears,
to calculate the probability that in the 6 tosses, there
are
(i) exactly 2 heads and 4 tails (ii) no heads
(iii) as many heads as tails
(iv) at least 2 heads (v) at most 2 heads.
4. If the births of boys and girls are assumed to be equal­
ly likely, what is the probability that in a family of
8 children, there are
(i) exactly half boys (ii) all girls
(iii) at least 2 boys
5, A true - false quiz consists of 10 questions. Each of
the questions is answered purely by guess (or by the re­
sult of the toss of a coin - say heads for true, etc.)
Find the probability of o!.>taining
(i) exactly 6 questions correct
(ii) exactly 8 questions incorrect
(iii) at most 2 questions wrong
(iv) a score of 60% or better, assuming all questions
are of equal value.
6. At a referendum, 50% of voters. were in favour of a cer­
tain proposal. A T.V, interviewer approached 7 voters
chosen at random. If Xis the number from this group who
were in favour, find the values (i) P(X= 0)
(ii) P(X = 3) (iii) P(X� S) (iv) P(XS. 1)
387
SET 7I
(v) P(l � X � 4) (vi) P(l � X < 4)
(vii) P(jxj > 3) (viii) P(lx-41 < 1)
7. A card is selected at random from a regular pack, the
card being replaced after selection. What is the proba­
bility that it is a diamond? lf 5 cards are drawn at
random, the probabilities of drawing O,l,2,3,L1,S dia­
monds are the successive terms of a certain binomial ex­
pansion. State what it is, and use it, to determine the
probability of drawing (i) exactly 1 diamond
(ii) at least 1 diamond (iii) at most 1 diamond
(iv) more diamonds than all other suiti;. together,.
A bag contains 8 white balls and 12 black halls. Balls
are withdraim one at a time, the ball being replaced
after each draw. There are 4 drawings. What is the
probability of drawing out (i) all black balls
(ii) at least 3 white :>alls.
Two dice are thrown simultaneously on a table. What is
the probability of obtaining a total of 7? These two
dice are then thrown 4 times. Letermine the probability
that the total of 7 (i) appears exactly once
(ii) at least twice (iii) does not appear
[This question could have been reworded thus:
"If 4 players each roll a pair of dice, determine the
probability that (i) only one of them rolls a 7
(ii) at least 2 of them roll a 7
(iii) none of them roll a 7. "]
Suppose we take a sample (at random) of 5 items from a
large number of items which are known to be 10% defect­
ive. What is the probability that our sample should con-
tain (i) all defectives
(ii) no defectives (iii) more than 2 defectives
\Terify that the chance our sample should contain more
ci,an 2 defectives is< 1%.
11. A die is rolled 3 times; find the following probabili­
ties, where X denotes the appearance of a 4 on the upper
face of the die. (i) P(X = 2)
(ii) P(X 1 2) (iii) P(X � 3) (iv) P(X = 0)
12. It is known that at noon in Newcastle the sun is hidden
by clouds on an average two days out of every three.
Find the chance of the sun shining at noon on
(i) 5 consecutive days
(ii) only the first 4 of 5 consecutive days
(iii) 4 of 5 consecutive days
(iv) at least 4 out of 5 consecutive days
(v) at least 1 but no more than 3 of 5 consecutive
days.
13. ( i) An event has a 10% chance of o.ccurring. Evaluate
to 2 decimal places, the probability of its occur­
ring at least once in 10 independent t>ials.
388
FURTHER TYPES ON THE BINOMIAL DISTRIBUTION
(ii) A biassed coin falls heads 60% of the time. What
is the probability in 15 throws it falls heads
exac.tly 8 times. [Use logarithms to aid in the
calculations.)

H/\RDER TYPES HWOLVHJG THE BINQf.�IAL DlSTRIBUTIOi�


In the following types of exercises, the values of
p, q have to be first determined before the binomial re­
sult can be used.

EXAMPLE I. Two men A and B work on separate machines pro­


ducing thousands of the same item. Their ratei of
production are not the same, but in the ratio 3 : 4.
Because of this, there is a falling off in efficiency be-
tween A and 3; in fact, 1% of A's B
production is defective but 2% of A
B's production is defective.
Vlhat is the prob ab i I ity that an �
m; � m ,. .
item chosen at random from the
factory is defective?
The parts are all mixed together, '.lmQ.ctwnes
then �,rapped up in parcels of 10,
and sent for storage. A parcel i.s chosen at rancom
from the store. 'tlhat is the probabi I ity that the parcel
contains Ii I exactly two defectives (i ii at least
2 defectives (ii i I at most 2 defectives.

,,Jethod.For convenience, let us assume that A and n together


--.-produce (3+4) x 100 i.e. 700 items, of which 300 are
produced by ,, and 400 produced by B. Since l�� of I.' s
production are defective, ,',3 items out of his 300 are
defective.and 2% of B's production are defective,

8 items out of his 400 are defective. �hus of the
7ll0 produced by .:i. and ll, (3+8) = 11 items are defective.
This gives the probability (p) of any one item out
11
of the 700 being defective as and the probability
700
(q) of any one item out of the 700 as not being defect-
. . 11 68'.) 11
ive is l - These probabilities p = ,
700 = 700' 700
6!.'9 are assumed constant for every item produced 0y
q = 700
;._, B. Hence, if 10 items are selected at random and
wrapped together, we simply have 10 independent trials
(a trial being the selection of an item) in which the
probability of any of these items being defective iG
11
700•
That is, if }� is the numl.er of defective items in
a parcel of 10, then the random variable X takes on the
value5 0,1,2, ... ,10 with probabilities given by the
389
FURTHER BINOMIAL DISTRIBUTION
successive terms of the expansion (q+p)
IO
' where
689
p = ..11.
q C 700' 700'
i.e. of (��b + 7��) 10
f (•9P.428 •• + ·01571•. ) 10

)8(..11) 2 � 45( •9843)8 (,01571) 2


689
(i) Now P(Xc2) = lOC (
2 700 700
(ii) If there are at least 2 defectives, X? 2.
Thus P(X � 2) = P(X=2 01' X=3 01• ••• or :� = 10)
i=t-{P(;(=O) + P(X=l)}
6 9 l 6 9
=l-{( 3 )1 o + OC ( 8 )9 (..11) 1}
700 1 700 700
+ 1 - {(•9843) 10 +1C(•9 843)9 (•01571)}

(iii) If there are at most 2 defectives, then X � 2.


Tl:us P(;,;52) = P(X=O or :{=1 or X=2)=P(X=O)+P(X=l)+P(X=2)
= (·9843)1 0 +10(•9843)9(·01571)1
+45(•9843)8(,01571)
2

iiote The actual nwneriaaZ aaZauZations themselves are


arduous, but mwJ be done with Zogaritnm tables, if re-
quired.

EX/'.\1PLE 2. Answer the questions


of example I, assuming that three
men !', B, C produce the sarne i tern;
their rates of production being
3:4: 5 rnspecti vely and the number
of defectives produced being 1%,
2�, 3% of their respective produc­
tions .

.'1ethod. Take a convenient number here as (3+4+5) x 100


i.e. 1200 items; of which A produces 300, R produces 400,
C produces 500. ,Uso the respective num\;er of de­
fectives are 3 (out of 300) for 11., 8 (out of 400) for R
and 15 (out of 5•)0) for C', being 1%, 2%, 3% of their re­
spective productions. Hence, the probability (p) that
any item producecl by A,B,C is c!efective
3+8+15 26 13
= = and the probability (q) that any
12Cli i200 600'
13 587
item �s not defective is 1 - =
600 6�0'

Selecting the 10 items for wrapping consists of 10 inde­


pendent trials, 11here p, q re,:1ain constant for each
trial. Thus, as l;efore, the successive terms of
587 13 ,c IC
( + ) (·9783 .. + ·02166 •.) give the respect-
600 6,,0 ·
ive probabilities of C:,1,2, ..• ,10 defectives in a parcel
390
SET 7J
10 587 8 13 2 8 2
(i) Now P(X=2) = (c2
600
) ( ) + 45(· 9 783) f·02 1 67)
60 0
587 10 10 58 7 9 13 1
(ii) P(X .:! 2) l - ( ) + ( ) ( )
{ 600 Cl 600 6 00 }
1
+ 10 (• 9 783)
10
� 1 - {(• 9 783) (•02 1 67) }
9

(iii) P(X � 2) = 10 9 8
(• 9 783) +1 0(• 9 7 83) (• 02J67)+45(• 9 783) 2
(• 02 167)
[Again, the numerical values of these results may be obtained
using tables, if desired. J

EXERCISES SET 7J
[In the following exercises, the actual numerical calcula­
tions are not done; the answers should be left in similar
forms to those in the examples above. If necessary, logar­
ithm tables may be used to complete the arithmetic,]
1. A, B are machine-operators pro­
A� /s
.:: V JI;
ducing identical parts, which
mix together on the production
line indicated. The number of
parts produced by A, B is in
the ratio 2 : 3. Of the parts
t
produced by A, 21tf are defect­
ive, whilst of the parts
produced by B, 5% are defective.
(i) What is the probability p, that a part selected at
random from tile production line ,is defective?
(ii) Ten parts are selected at random from the produc­
tion line. What is the probability that of the ten
(a) exactly 5 will be defective
(b) at most 2 will be defective
(c) at least 2 are defective.

l1
2. The diagram shov,!s 3 factory oper-
ators A, B, c. Each operator 8
t
produces the same item on differ-
ent machines, and these items are �
mixed together as they progress
"'
by moving belts to the production
line. If the number of parts pro-
}
-0
duced l.;y A, Il, Care in the ratio r8.
4:5:6, and of their respective - C
':I 0
productions, 2•51;, 3· 4;{, 5·5% are "' :::t
defective. :s

(i) What is the probability, p, that an i terr, selected at


random from the production line is defective?
(ii) If regular inspections of the manufacturing process is
made by selecting samples of size 20 off the production
line at regular time intervals, find the probabilities

391
SET 7J
that in the sample, there are
(a) n o defective items (b) just one defective item
(c) at most 2 defective items.

t3. A certain machine in a factory mass-pro duces T.V. comp o­


nents. These T.V. components are made in large bat ches,
wrapped up in parcels o f 5 and st ored. What is the pr ob­
ability that if a parcel is chosen at random from the
st ore, then the number of defective comp onents in it is
(a) zero (b) exactly 3 (c) at least 3 (d) at most 3

Consider rqach of the cases belo�1; first find p, the


probabi I ity that a component drawn at random from the
output of the machine wi I I be defective.

(al The machine is run by one man only, namely A. (\,lh en he


finishes for the day, the machine is idle unt il he
resumes next day.) Of the number of comp o nents produced
by A, 1% are defective.
( p.) The machine is run by two men, ,\. [ the man mentioned in
(a)], and B. The number of components produced by A, B
respectively are in the ratio 4: 5, and of the parts
produced by B , 2% are defective.
(y) The machine is run by three men A, B (mentioned in a , B
abo ve) and C. The number o f components produced by A, B , C
respectively are in the ratio 4: 5 : 6, and of the
parts pr oduced by C , 3% are defective.
(o) The machine is run by f our men A,B, C (menti oned in a,B,y
above), and D. The number of components pr o duced by
A,I:, C ,D respectively are in the rati o 4 : 5 : 6 : 7, and
of the parts produced by D, 4% are defective.

4. One urn co ntains 2 white and 2 red balls,


whilst a second urn contains 3 white and I�w I I �� I
S•red balls. An urn is cho sen at random �

and one ball is drawn from the urn (and replaced after
the draw). Pro ve that the pr obability that the ball is
. 1 2 + 1 3 7
white l.S (
2 . 4 2 . 8) =
16 . The o perati on is carried
out 5 times. Find the pr obability that
(i) exactly 3 whit� balls are drawn
(ii) at least 3 white balls are drawn.

FUPTHER NOTE 01� RANDOM DIG I TS


The idea of a random digit has been mentioned previ o usly.
The aim in co mpiling a table o f rand o m digits is to select
digits o ne at a time fr.o m the digits 0,1,2,3, •• , ,9 but in
such a way as to ensure (as far as is possible in practice)
that each digit has n ot only the same chance (namely l/ 10=• l)
o f being selected, but als o that in ac·tual fact, in a large
192
RANDOM DIGITS
number of cases (say 1,000 or 10,000 etc.), then each of the
digits 0,1,2,••• ,9 occurs (as far as is possible) exactly the
same number of times.
This may be done by putting 10 identical counters
(marked 1 to 10 respectively) in a I.at and draw:/.ng them out
one at a time, but replacing the counter after each draw.
However, this method is too laborious, and nm,; modern elec­
tronic machines are used to generate the digits.
Since the drawing of any one counter (or the generating
of any one digit) is independent of the drawing of any other,
the compiling of random digit tables is a binomial experiraent,
in which p and q, the probabilities that any particular digit
occurs, fails to occur are 1/10 and 9/10, i.e. •l and •9 re­
spectively.
The results of 1000 drawings of a counter, grouped in
sets of 5 for convenience, could appear thus

53381 79401 21438 83035 92350 36693 31238 59649 91754 72772
91962 04739 13092 97662 24822 94730 06496 35090 04822 86774
87637 99016 71060 88824 71013 18735 20286 23153 72924 35165
49323 45021 33132 12544 41035 80780 45393 44812 12515 98931
14422 15059 45799 22716 19792 09983 74353 68668 30429 70735
98275 32388 52390 16815 69298 82732 38480 7��17 32523 41961
78985 05300 22164 24369 54224 35083 19687 11052 91491 60383
82674 66523 44133 00697 35552 35970 19124 63318 29686 03387
53363 44167 64486 64758 75366 76554 31601 12614 33072 60332
27889 47914 02584 37680 20801 72152 39339 34806 08930 85001
74211 63465 17361 62825 39908 05607 91284 68833 25570 38818
10119 69917 15665 52872 73823 73144 88662 88970 7 4492 51805
95452 92648 45454 09552 88815 16553 51125 79375 97596 16296
14267 20979 04508 64535 31355 86064 29472 47689 05974 52468
41744 81959 65642 74240 56302 00033 67107 77510 70625 28725
96783 29400 21840 15035 34537 33310 06116 95240 1 5957 16572
89728 17937 37621 47075 42080 97403 48626 68995 43805 33386
33732 05810 24813 86902 60397 16489 03264 88525 42786 05269
51281 84463 60563 79312 93454 68876 25471 93911 25650 12682
81973 37949 61023 43997 15263 80644 43942 89203 7 1795 99533

The table of random digits may be used in many ways,


some of which are indicated below.
Exverimen ts
1. Check the randomness of the table, by taking any one row
(SO entries) and counting the number of say 3's (or O's
etc.). Make up "at random" your OY1Il set of SO digi'.ts,
and check the number of 3's (or O's etc,).
2. lvaluate, using logarithms, the successive terms of
393
EXPECTATION
(·9 + •1) 1 0, which can represent the respective proba­
bilities of obtaining 0,1,2, ••.,lO three's (say) when 10
digits are taken from the table of random digits. Check
these results by taking some groups of 10 digits from
the tables (start anywhere).
3. Use the table to help you choose a random sample of size
(20 say) from the students in form 6 at a large school
(you may list the names of students and number them).
Thus, if there are 78 students, they could be listed
consecutively from 01,02, ••. ,78. Then start anywhere in
the tables, and take the first 20 entries which lie in
the range 01 to 78. Thus, say we start with the line be­
ginning 98275.... We would discard 98, then take 27,
then 53, then 23, discard 88, etc. After discarding all
repeats, and unuseable sets, we would eventually obtain
20 entries. Those students whose names a:.-e opposite
these 20 entries would then constitute our sample.
4. This random sampling can be extended to many other
avenues; for example, sampling of the pages of a novel,
cf the na�es in the telephone book, of the houses in a
street, of the countries of the world, of the states of
Australia (or of the U.S.A.), of the entrants in a horse
race, and so on.

EXPECTATION. THE EXPECTATION OF A BINOMIAL VARIABLE


A. BASIC IDEAS
1. Suppose that a perfect die
is tossed 48 times. The X 1 2 3 4 5 6
number (X) of spots on the
uppermost face, and the f or f(X) 6 7 10 11 9 5
frequency [f or f(X)) of
occurrence of X, in an actual experiment, are shown in
the given table. The arithmetic mean of X is defined
to be Ef(X).X I Ef(X) {or EfX I Ef}
6.1+7.2+10.3+11.4+9.5+5.6.
i.e. the mean of X
6+7+1o+ll+9+5
169
3• 52
48

In our work on probability, there is a corresponding


process to that of finding the mean in the tossing of the die
problem above. This process associates the values of the
�iZities of t_�°.'!�!:lr�rice__ o_ �� Thus, for a perfect die, we
random variable X 1'Yith the corres ondin mathematical probE_�
assume that
P(X=l)P(X=2)
= P(X=3)
= = •••••• = P(X=6) = 1/6,
i.e. the probabilities that the numbers 1,2,3, •.•,6 appear on
the upper face of the die are
all 1/6. This can be shown in IXX I 1 2 3 4 5 6
P( ) 11611;6 11;61 l/61 1/6I1/6I
the table form given.
394
EXPECTED VALUE
The theoretical counterpart of the arithmetic mean of X

l
t6"
1 +
1
6"
2 +
1
6" 3
+
1
6"
4 +
1
6"
5 +
1
6"
6
j'
is l:P(X).X / EP(X) = EP(X).X since EP(X) = 1. This value here

=
1/ 6 + 1/ 6 + ••• + 1/6
= (1+2+3+4+5+ 6 ) = x 21 = 3·5, noting EP(X) = 1 as stated.
1 1
6 6
This theoretical counterpart is called the ��pectatio�
qr the e:pected_ value ofX, and is denoted by E(X).

DEFINITION The expectation of X, or the expected value of


the random variable Xis defined to be EX.P(X).

SIGN.IFICANCE OF THE EXPECTED VALUE.


In the example above, we have shown E (X) = 3•5. This
states that if a perfect die is tossed, fren the expected
value of X is 3· 5, i.e. that the "expected 1.urnber of spots is
3·5". By the latter statement, we do not mean that we expect
to obtain 3•5 spots each time the die is thrown. (It is, in
fact, obviously impossible to get 3·5 spots on any throw.)
i/hat it does mean is that in the long run, the .:.werage
number of spots per thro1,., of the die 1,JiU be 3, 5.

2. Consider another experiment, that of tossin� 6 coins


simultaneously 64 times. Let X represent the number of
heads uppermost on the 6 coins, and f(X) the frequency
of the occurrence of these values of
X 0 1 2 3 4 5 6 X. In our experiment th� corres­
ponding values of X, f(X) are
f(X) 2 8 13 21 1 6 3 1 sho�m in the given table. Here the

mean of X = 2.o+8.1+13.2+21.3+1n.4+3.5+1. = 182 • 2.84
6
64 64

To determine the expected value of X, i.e. E(X), we


first derive the mathematical probabilities of X=0,1,2, .••, 6.
The tossing of 6 coins simultaneously is equivalent to 6 in­
dependent trials. In each trial, the probability (p) of a
head on any coin is 1/2, and that of q = 1/2, where q is the
probability of a tail on any coin. The p.robabilities of
X = 0,1,2,•.• , 6 are the successive terms in the expansion
('. + \) 6.
Here (\ + \)6 = (\)6+ C (\) 5 (\)1+ C (�)4(�)2+ C (\)3(\)3
6 6
l
6
2 3
+ C (�)2 (�)4+ C (�)1(\) 5+(\)6
6 6
4 5
P(X=O)+P(X=l)+P(X=2)+P(X=3)+P(X=4)+P(X=5)
+P(X= 6 )
i.e. P(X=O) =
6
!, P(X=l) = �, P(X=2) =
6
!�, ....., P(X= 6) = !
6

We now complete the probability table shown. From this


395
EXPECTATION
table we can determine the value X 0 1 2 3 4 5 6
of E(X), i.e. the expected
value of the number of heads on P(X) ...1 ....§. .ll
20 .ll 6 1
the 6 coins. 64 64 64 64 64 64 64

. definition,
By the expected value of X = EX.P(X)
6 15 20 15
i.e. E(X) = 0 .
64
1
+ 1 64+ 2 64+ 3 .
64
+ 4 ' 64
+ 6 ...1
6
+ 5 '
64 64
1 · 1
= [0.l+l.6+2.15+3.2o+4.15+5.6+6.1] = [192] 3
64 64
This means that, if 6 coins are tossed simultaneously,
then the expected number of heads is 3, That is, in the long
run, the mean or average number of heads per throw, each
throw involving 6 coins, is 3,

NOTES.
1., The expectation or expected value of a random variable X
in a probability distribution is the counterpart to the
arithmetic mean of a sample. Just as the arithmetic mean
is not necessarily integral, so then the value of E(X)
need not be integral. For a die rolled once,
E(X) = 3•5 whilst for the tossing of 6 coins, E(X) = 3 ,
1, (i) If the value of E(X) is integral, then this value
oaaurs as one of the possible values of X. Thus
for the tossing of 6 coins, E(X) = 3, which is a
member of the sample space of X. Here X takes the
values 0,1,2,3, ••• ,6.
Hhen this occurs, this value of E(X) and thus of
X, is often referred to as the "value of X most
likely to oaaur" or the "most probable value of
X". Henae, the most probable number of heads when
6 coins are tossed is 3, (The aatual probability
6
that exactly 3 heads occurs is c (�) 3 (�) 3 =
3
!�,)
(ii) If the value of E(X) is not integral, these terms
"most likely to occur", "most probable value" are
usually used to denote the greatest integer less
than the value of E(X). Thus if E(X) = 1•7, then
the value of X most likely to occur is usually
taken to be X = 1.
]_. The observed results and the theoretiaal results of an
experiment do not alway s agree exaatly. Thus, if a die
is tossed once, the observed results should be identical
for each number on the die, since each is equally likely
to occur. That is, in 48 throws, 1,2,3, 4,5,6 should
1
each occur theoretically x 48 = 8 times. Similarly,
6
15, for example,
for the 6 coins, P(X=2) = 64 and thus in
396
THE BINOMIAL VARIABLE
64 tosses, theoretically, there should be 2 heads upper-
. 15
most on x 64 i.e. on 1 5 occasions. 'l'he observed fre-
64
quencies and the theo1•etical frequencies for the experi­
ments involving the die and the 6 coins are shown in the
tables be low.
Exoer i ment - 48 t hrows of die Ex ,eriment - 64 tosses of 6 coins
X 1 2 3 4 5 6 X 0 1 2 3 4 5 6
Observed Observed
6 7 10 11 9 5 2 8 1 3 21 16 3 1
freouencies freouencies
Theo retical Theoretical
8 8 8 8 8 8 1 6 15 20 15 6 1
freQuen cies f requen cies

B. THE Bl�!O"IAL VARIAeLE


The above experiment5 both involve ran dom variahles.
However, the variable X in the experiment involving the coins
is a binoir,ial variable. Note that the expected number of
heads when 6 coins are tossed, i.e. E (X) = 3 • 6 x .1, n p,
where n is the number of trials (here n 6) an·d p the proba­
bility of a success in any trial (here to obtain a head on
any coin).

This rernlt, non;ely that ii:(XJ np, holds true fol' all
i:,ino111ial random var�o.!Jles X.

*?roof. Consider a series of n independent trials, in each


of which an .even t E occurs (i.e. succeeds) and does n ot occur
(i.e. fails) with constan t p r obabilities p and q respectively
(p+q = 1). Let the random variab!e X denote the number of
times i:. occurs (i.e. the number of successes) in the n trials.
here X takes on the values 0, 1 ,2,... ,r, •••,n with probabil­
ities given by the successive terms of the bin omial expansion
(q+p)n.
n n r r.
Thus, in general, for O � r � n, P(X=r) = C q - p
r
Now, by definition, E(X) = [P(X).X
i.e. E(X) = P(X=O).o+P(X=l).l+P(X=2).l+ ...+P(X=r). r+.•.
••+P(X=n).n
n r n n- 1 n n- 2 2 · n n- r r
. C q p l+ C p .2+... + C q .r+..
c; ..+ l . 2q r P n
n ..+p .n
n ' k
c q -� P-. k.
n
E
k
k=O
1-/e are tY"1/ing to prove t;;at �(X) = np.· To do this, con ­
sider the binomial expansion ( l+y)n •.
n · n· n 2 n 3 n r n
Now (l+y) = l+ c y+ c y + C y + ..• + C y +... +y
1 2 3 r
Differentiate bot;, sides 1.-i th respect to :·,
n-1 n n 2 n r-1 n-1
= 1. c +2. c y+3.C y +.••+r. C y
n
n(l+y) + •.• +ny
1 2 3 r
397
SET 7K
Mul.tiply both sides by y,
, n-1 n 2 n 3 n r n
, , ny(l+y) .. 1. c y+2. (; y +3. c y +•••+r. C y +•••+ny
n
1 2 3 r
This identity is true for all values of y and is true for
y = 1!.
••• 1!. .E_ n-1 2. c 1!. 1 + 3 c 1!. +
i 3
(l + )
n n
q ' q 2 q • r 3 q� ...
� n
••• +r. C 1!.r + , , .+ n , 1!.n
n

r q q
n
Multiply both sides by q ,
n-1 n-1 n 2 n -2 n 3 n-3
n p(q+p) = 1, c pq +2. c p q +3. c p q +
n
1 2 3
r n-r
•.• +r. C p q +••• +np
n n
r
Thus since p+q = 1, we have E(X) = np(l)n-1 = n p.

EXAMPLE. Let us assume that in Australia, 60% of people are


in favour of proposition A. If a random sample of 10 people
be taken, then the expeated nwnber of people in this sample
who are in favour of A is np = 10 x 3/5 = 6, noting that
n = 10, p = 60% = 3/5. [Thus, in a sample of 10 people,
the most probable number of persons in favour of A is
6, i.e. the most likely number of people in the group of 10
to be in favour of A is 6.)
T!1e theoretical probability that in a sample of 10
people there are 6 in support of A is given by the term
1
oc
<f <1)
6 >4 G'
i.e. the seventh term of the expansion of
cl + 1) 10 in increasing powers of 3 <s).
5 5

[i'}ote. The result E(X) = n p is true only for the bi­


nomial variable X. It provides a simple formula to determine
E(X) in these cases. The value cf E(X) is used in conjunction
with the variance (cr 2 = npq) of the variable X to test the
validity of theories in more advanced 1:ork in probability.
Generally speaking, for a non -hin omial variable X, the
value of E(X) can on ly be determined by appliaation of the
definition .]

EXERCISES SET 7K
1. A contestant selects a box
one at a time from 5 iden­
tical boxes. I n one of the
boxes is a prize. The random variable X denotes the
n wn:,er of boxes he tries, and P(X) the correspondin g
probability that each box wins the prize. Thus X = 4,
P(X) = 1/5 states the probability that the fourth box
tried is the correct one is 1/5. Use the definition to
fi nd t(X), the expected n umber of boxes which must be
tried Lefore the correct one is chosen.
398
SET 7K
2. T he given table shows th e
various probabilities of
seeing X = 0,1,2,3,4 ac­
cidents while on a long
car trip, Determine E(X), the expected number of acci­
dents to be seen on the trip.
A true-false quiz consist­
ing of 4 questions is PX) 1/16 1/4 3/8 1/4 1/16
;), X O 1 2 3 4

answered at random.
(i) Calculate E(X), the expecte d number of questions
corre ct.
(ii) Verify the result of (i), using the result
E(X) = np, noting that this experiment is a
binomial one; (the probability p of any question
being correctly answered is 1/2; the number, n, of
trials is 4).
In a bag are 4 red and 2 black balls. Balls are witl1-
drawn, one at a time, the ball being replaced after each
4.

� : ! : � = � �; 0
draw. The re are 3 draws. The probability distribution
of X, t he number of red balls
d
! th; �:��i�� / PX� ) 1 1�27 1 2�9 1 4�9 / 8J27 I
he

expansion (1/3 + 2/3) 3 • Find th e expecte d numbe r E(X) of


red balls sele cted (i) from the defir.ition
(ii) noting that X is a binomial variable,

5. A die is tossed 3 times. Complete th e


table shown, whe re X de notes the
numbe r of sixes in the 3 throws.
[Hint, He re p = 1/6, is the probability of a six on any
throw, and use the expansion (S/6 + 1/6)3 J, He nce find
E(X), He nce find E(X). the expe cte d numbe r of sixes
by the de finition, and verify from the formula. If th e
die we re tossed 216 times, what would be the the ore tical
fre quencies of eacl, value of X,
6. Five draws are made from a pack of cards; each draw con­
sists of a single card and th e card is re place d after
e ac h draw. Wh at is the expe cted number of times a spade
appe ars? [Use result E(X) = np].
Verify your result by completing the
table indicated [using th e binomial
5
e xpansion (3/4 + 1/4) ); and thence
find E(X) from the definition. I f this drawing process
were done 2048 times, what would be the theoretical fre-
1uencies of �ach value of X?
t7. pair of dice are rolled on a table. If X denotes the
�1Jm of the number of spots on the uppe rmost face s of the
two dice , determine by actual enwne:t'Cltion th e number of
ways the totals 2,3,4, ... ,12 may be obtained from the 2
dice.
399
SET 7K
Hence com­ X 2 3 4 5 6 7 8 9 10 11 12
plete the P (X) 6/36
table given theoretical
assuming frequencies 12
that ti1e observed
dice arc frequencies 1 3 8 10 11 14 9 6 5 3 2
rolled
72 times. Use the table to determine (i) the o:iseJ:;Ved
1:iean of the sample (ii) the ex._?eatec!. value of X.
8. In each of the following, the formula for E(X) is to be
used.
(a) Twenty voters are selected at random. The election
results showed 70% of electors voted for party B.
How many party B supporters are most likely to be in
the group selected.
(b) � hexagonal pencil is rolled 120 times on the table.
The 6 surfaces (excluding the ends) are numbered
1,2,3,4,S,6. t:hat is the expected number of times
the surface numbered (i) 5 (ii) 3 or 5 will appear?
(c) A bag contains 2 red and 3 blue counters. Seventy
draws are made from the bag, one counter at a time,
( the counter being replaced after each draw). 1-'hat
is the expected numuer of (i) red (ii) blue
counters?
(d) What is the, expected number of girls in a family of
(i) 6 (ii) 5 children? Explain the answer for (ii).
(e) Suppose that 95% of all students h�ve fillings in
their teeth. If a sample of 60 be taken from a
school, wi1at is the most probable number of students
in the sample without any fillings?
(f) T11enty six draws (with replacement) are made frol'l a
standard µack of cards. Hhat is. the expected number
of (i) aces (ii) picture cards (iii) diamonds?

9. What is the most likely number of heads if a coin is


tossed (i) 10 (ii) 100 (iii) 1000 tirr.es?
!·/rite down (but do not evaluate) an expression for the
probahility for the r.iost likely number of heads to occur
in each of ( i) , (ii), (iii) .
10. Ten coins each of wi1ich is equally likely to come dm-m
"i1c,ads or tails" are tossed together 1'124 tir.1es. \:hat is
the probability of a single toss giving .3 heads and 7
tails, and how many times would this coml,ination be ex­
pected to happen in 1024 tosses?

400
APPENDIX

SEQUENCES AND LIMITS OF SEQUENCES

SECTION A. DEFINITION

A sequence is a function of a variable, usually n,


whose domain is the set of positive integers. The
function is usually denoted by f and its value at n is
f(n). The sequence f is the set {(n,f(nl):n=l ,2,3,••• },
i.e. the set of all pairs (n,f(nl) �,here r, is a positive
integer.

4
For example, {(n, -) : n= l,2,3,••• } is a sequence whose
n
value at n is 2-. Since the domain of n is the set of positive
n
integers, it is usual to shorten the notation and write
{f(n)} instead of { ( n, f(n))}.
4
Thus the sequence {(n, ;-) : n= l,2,3,•••} would be ab­
1
breviated simply {2-}. Similarly {-3 + __} would denote the
n zn-4
sequence {(n, -3 + ��)1 : n =l,2,3, ••• • }
2n- 4
{Note: if the set n = 1,2,3,... has a largest term, the
sequence is finite. Otherwise, the sequence is infinite.
We shall be concerned with infinite sequences.}

SOME EXAMPLES OF SEQl)ENCES AND THEIR GRAPH ICAL REPRESENTAT I


{Infinite sequences fall into three categories.
Category 1: the terms of the sequence tend to approach a
given finite number V as n increases without bound.
Such sequences are called convergent.
CategorJ 2: the terms of the sequence increase (or decrease)
without bound as n increases. Such sequences are called
divergent.

401
SEQUENCES
Category 3: the terms of the sequence do not behave as in
either of the other 2 categories. Such sequences are
called oscillating.}
Examples on these ti,ree categories follow: -
Categor>1 1.

( i) {-1-}
2n-l
1 1 1 1
{zo, zl, z2, z3, . . . , _2__
n -l' . . . }
l

(ii)
2n- 1
{3
n+l
} {3 ' H'
2-1
+ 1 6+1
6-1 8-
9+1' 12+1'
. . . ' 2n-1
1
Jn+l''' '·
}

1 1...1
{4, 7' 10'
7
13' 16' 19'
9 11 · ··
2n-1
' 3n+l' ... }

(iii) {1 +�} = { 1 + .i::1l 1 + 1±12. 1 + .i::1l


3 , 1+ fill •·
n 1 ' 2 ' 4
i.:1-.2.n
1+
...'
n ' ..}
3 2 5 4 7 1 9 8
3' 4' 5' 6' 7' 8' 9, .. ,l + n ,.}

{O, 2'
f"'> f < nl

' ·6

•4
ft,,)
..
- - -..... -=.-
I Yi

.,
2!1-1
I f<11): 3nTi

_CategoY".., 2.
(iv) {log1on 2 (i.e. 2 logion)} {log1ol, log1o4, log109, ..
i
. .,log1on ' ..}
i {n,·60,1·21,1·40, ...
: ..,log10n � ...}
(v) {-n} (- 1,-2,-3,-4, ...,-n,... }

402
SET APP. lA
{ (-1:) n
n 2+1} 2 5 -10 Jl -2 ·6 37 -so
(vi) n+2
{- 3' 4' �· '6 7'8' 9'
···• (-l)
n n 2+1
n+2' .. }
.... .......
f<11)

'
. =i--+--+-'-----4-=---+-'-+c..+:<-+7 � j<111 t
,
o '71. I
I -I "­
/ . ',, 3
,. � II I
I � -2 1 1
I fen.)" 2-�,o n -3
'.
J\3!11i1 1
, n2
=� 11. '- 0 i 2. 4 6 I I gTI
�: f('11.)= -
•S /
I lo -.. 'Ii \/ II
n -6 ""' -3
0 2 3 4 5 6
-7
-6
f(1\)•t1}11.1D!
\
\I

'11,+2,.
tl
The terms of the above3sequences increase beyond bound
in (iv), de·crease beyo nd bound in (v), and both increase and
decrease beyond bound in (vi).

Category 3.
(vii) {sin90 = { sin90 ,sin180 ° ,sinZ 70 ° ,si3 60 ° ,•• sin90 n ;,.}
° °
n } n
= { l,O,-l,O,l,O,-l, •... ,sin90 n ,••.••••••••• }
°

(Vl'ii') { (-l) i..!!±12.} - {-2 1' -i 1 -.§_' l 1 (-l) i.!!±!l.


n n
n ' 2 3' 4' 5 6 · · · • n •·· }
( ix) f(n) = !Yi if is odd 3 313131
{z,1 ·2·2·2•3•2•4• ···· · ··· · ·}
n

=l if n is even
f (1') • %;811� ...U
n
fc111 ft'II) f<11)
co/n.jn.u.t«llf,
\ f(11jc5inqh•�
I\ I

-r1--'r-L--
fl
%,
\\ I II I /7\ ,'
I \ I\ /\ fl 111 'Ii \/\ I
0 4 S 6 7 8 I I 2 \3j 4 \5/ f, \7 I I \ ,'
I(,
\ I ..i -:-�--v-•- I�
'ij

)I _
2 J f(11,)�l- 1l
11
¥ -+,,o,......,.
1 -+.-z-!:'3-+.--<:-
4 -t5 -ti:
6 �
The terms of these sequences osaiZZate as n takes on
integral varues.

EXERCISES SET APP TA


1. Write down the first 6 terms of each of the following
sequences, graph these terms and - indicate if the··
sequence appears to converge, diverge or oscillate.

(d) {(-l) n}(e) {cosl80n }


n
(c) {n 2 }
°

" n+l
{ (- )n
L
(f) { 2n-l} (g) {5 - .!!.....}
10 (h) {!} ( i) }(')
J {log n}
e

1 5
(k) { n -1
} (.l) {2 - ; }
3
n
n 1+(:..1)
(o) { (-1) l!!±.ll.}
2n-l (p) { ;l }
403
SET APP. lA
(r) {sinl80n}
°
(s) { l(n odd); 2 (n even)} (t) {sin t�)}
3 n
(w) {2 [ ( ) - 1)}
2

2. ( i) For the sequence {u}, u1 1, u 2 = 2


n
and u = u + u _ , for n 1 3; find the first 8
n n-l n 2
terms of the sequenc·e.
1 n
(ii) I n the sequence {f(n) = (1 + -2 ) },
n
458
show that f(l) -f(3) = 729

2n-l = 2n = -
3. (i) A sequence {urt}is defined by u 1, u 1.
Write down the first 6 terms. What is the sum of
the first (2n-1) terms of the corresponding
series?
(ii) In a sequence {f(n) ; f(l) = 1, f(2) = 2 ahd
f(n+2) = f(n+l) .f(n) for n � 1. Write down the
first 7 terms of the sequence and determine which
is the first term greater than 200.
4. The following are unusual sequences; find the first
6 terms of each.
n
l+(-l) 1 1
(i) {n } (ii) {1 + -(1 - --)}
9 n-1
10

5. Two students A and Bwere asked to write down the nth


term of the sequence { 1,8,2 7,.. • }, and to then give the
fourth term. A gave the nth term, u = n3, and Bgave
the nth term, u 6-lln+6n2 . Show tRat A and Bare
n
both correct, and determine the fourth term in each
case.
6. Verify for the sequence {1,16,81,256, •.. }, u may be
n
given by u = n and also by u
4
= 10 n3 -35n2 +50n-24. What
n n
is the fifth term in each case?

{Note: Exercises 5, 6 are intended to i I lustrate the


fact that being given a finite number of terms of a
sequence does not define a unique nth term.
However, i.t is certainly true to say, in reverse,
that if the nth term is given, then the sequence is
unique.}

*7. The Fibonacci sequence {u} is one in which "'1,


n Ul
u2 = 1, and u = u + u , n � 1. Find the first 8
n+2 n+l n
an-bn
terms. Verify that the formula
Un=
where
-rs-•
404
LIMIT OF A SEQUENCE
a = !,(1+/s), b l,(1-/s) is true for n 1,2,3,4.

SECTION B. THE LIMIT OF A SEOUE�CE.


In th e pre vious se ction, we saw that the te rms of some
sequence s {f(n)} approach ce rtain constants as n incre ase s
without bound (through integral values). These se que nc e s are
called con vergent.
Examples mention�d. we re (�-!} which te nds �o z e ro,
2n-l . 2 2
{
Jn+1
} which appe ars to approach
3, and {l +�}which
n
be comes close to 1.
We sh�ll select two of the se quence s to illustrate what
we really mean by the phrase s such as "close", "approach",
"tend to'·'.
EXAMPLE I.
(-1/ 3 L 5 4 7 b (-1) n
{I+ --n-} = {O, 2' 3' 4' ':f• 6' 7'
, I + --n-, ..... }

It would appear that the se quence is approaching the


limit 1, because the terms of the sequence become close to,
and remain close to, the numbe r 1. [Howe ve r words such as
"close ", "approach" are not mathe matically e xact enough for
us. Thus, I may be "approaching" my de stination afte .r a long
journe y, but still be many miles away. "Close " to an insect
may mean pe rhaps 10-" inches whe re as "close " to an astronome r
may mean a light y e ar, approximately 5•87. x 10 12 mile s.]
(
n
:,'i:en i.:e sa;: . nce {1 + -v
- that the seov.e n
} has a lil"lit
equal to 1, we shall l'?ean that, by going out far moug/;, aZ l
subsequent members of t;,e sequence lie as close to 1 as
:.'e care to specif!:·. ;/e then say that tl:e terms of the
sequence become a nd remain arbitrar>il11 close to the nwnber 1.
For example, if we se le ct a small numb e r (£ is usually
used), £ = O·l, then it should be possible for us to show
that after a certain nwnber of terms (to be deter>rr:in ed), all
su:Jseqv.ent tems lie 1:ithi n this inte;•val £ = O· 1 of the
nur.:�'er 1. U.11
I+, • 1 o_
In the st:etc:1, £ is
s;eci:ie� as 0·1. Con­
side r the te rms of th e
se que nce {u } denot e d by
n
the points C, A•
.'.'ow if the coo1,din­
ates of Care (n, u}
n
then l*ECj = u , whilst
n
l*E�I = 1, and thus
405
LIMIT OF A SEQUENCE
l*CDI = 1 - un' which may also be written as ll - uni. It is
obvious that for the point C, the ordinate, u ' is not within
n
O•l of the number 1. In other words, u does not lie within
n
. 9 11 An alternative way
the interval (10, 10) i.e. (1-·l, 1+·1).
of stating this, is that l*CDI is not less than O·l,
i.e. 11-u I is not less than O·l. Obviously, for the case of
n
the point C, there are probably terms further out than C, for
which the same result is true: namely that 11-u I is not less
n
than O•l, i.e. that u does not lie within O· l of the number
n
1.

[low, however, cons ider t;ie point A, with coordinates


{N, I\;}. Here l*FAI = ' 1� and l*trnl = l;
thus l*ABI = 1-u
N'
or ll - �I. It can be seen that for the point A, the ordin-
ate A is within O•l of the number l; i.e.� does lie within
the interval ( �,
1 i�). Alternatively,
we may say that
l*ABI is less than O·l, i.e. ll -�I is less than O·l.
For the case of our sequence, every term further out
than A has the same property, namely that 11-�I will be less
than O·l. That is, every subsequent term after� will lie
within the interval (1-·l, l+•l), i.e. lie within O•l of the
number 1.

The basic idea of a limit is now carried further:


n
I-le are considering the sequence {1 + -82..:} which we be-
n
lieve has a lin1it 1 as n increases without bound (through
integral values). If this is the case, then for> our specified
£ = O·l, we should be able to find a value of n, (N say) such

that J'
+'o·-·
- ·all values of n > N' then _..!!_ 10' or in other>
10 < un < Q.
words, such that 11-unl < 0·1. Now for> cases where£ is not
Vel":_.- small, this value of N may be determined by evaluating
the terms of this sequence,
8 .!l 10 13
Thus {l + �- } - {O, 111 4 7 .§. .2.
n 2' 3' 4' S' 6' 7' 8' 9' 10' ii• 12'
12
13' ......
10 13 12
It can now be seen that the terms
ii• 12, 13, •••• all lie in
9 11 9 10 13 12 < 11
value between d
10 an 10
, i.e.
10 < ll' 12,
13, ........ 10·
10 13 12 9 11
Thus the terms , , , ... lie in the interval
11 12 13 <io • 10)
.

406
LIMIT OF A SEQUENCE
10
Henoe, since is the eleventh term in the sequence,
11
then we can say that from the eleventh term onwards ea·ch mem-
ber of the sequence { 1 + �} differs from the number 1 by
n
less than O · 1.
In other words, N = 10 and the terms u11, u12, u13, ,,,
all lie in the interval < ;,
1
i�). �lt_ernatively, this may be
stated as: for n > 10, This idea I1 - u I < Q, 1, can be
1_nn n

"
readily seen from the sketch below of {l + �}.
n

I I fl. •
� .p,,.-, --/ !\
-,-,-- "' I ,...---
� --
---1.__1_ L .L I \ I � \ --

/ I�/ \ I \ I -._------r ¥'


-r-- I 1--' I

� ---4f- __.Y_ -- -
I � < 11. Un. = 1+ /
I

0 Z 3 4 5 6 7 S 'l 10 II 12 I� 14- 71
/./hen£ is small, the above procedure of evaluating terms
is laborious, We prefer to proceed as follows:-
Now 11 - u
n
J 11 - (1 ; (:i) l-1�
n
n
j-<.::nffffj = l
n
Hence for 11 u I to be less than O•l, it is necessary
for i to be less than\)·!, i.e.�< O·l,
0
\ (on whence n >
taking reciprocals), and thus n > 10. Thus, provided n > 10
(and the integral values for this are 11J2,13,.•. ), then
J 1 - u
n
I < o, 1.
In other words, given£ = O•l, then all subsequent terms
after the tenth term are such that they lie in the interval
(
1�
, i�), i.e. within 0•1 units of the number 1.

Let us noi..1 repeat the prooediwe, where instead of taking


£ = O·l, �e speoify that£ is to be 10-�. Our problem now
is: "Having specified£ = 10-6, and provided that the limit
of the sequence {1 + �} is 1, we must again be able to
n
find a value of n, (N say) such that for all values of n > N,
then u lies within 10-6 of the number 1,
n
i.e. so that 11 - < 10-6• 11 �J
It is obviously too arduous to write out each term of
the sequence until we finally reach an appropriate one. In­
stead, �e proceed as foll�ws:
Now I 1 - un J = l.
n
Henoe, for j 1 - un I to be less than
10-6, it is necessary for .! to be less than 10-6, i.e • .! < 10-6,
n n
407
LIMIT OF A SEQUENCE
when ce on takin g reciprocals, we have n > 10 6 • Thus, provided
that n > 1,000,000, then 11 - u I< 10-6; i.e. all subsequent
n
terms after the millionth term lie in the interval
(1-10- 6 , 1+10- 6 ), i.e. in the interval (·999999, 1'000001).

/late that no matter how small we decide to make c, (pro­


vided c ,/, 0), then we will always be able to find a counting
n umber t� such that for all values of n > N, u will differ
n
from 1 by less than c.
In other words, for an y pre-assigned (i.e. specified) c,
then we can always find N, so that I 1-u I will be less than £
for all n > i,, (I.e. for all members u of the sequence from
n

n = N onwards).
n

If tne result above is true, then i;e say that the Zimit
n
of the sequence (1 + (-l) } is 1.
n

EXA:-"PLE 2.
Con sider the sequence {
2n-l} which �1e feel has
3n+ I
th e I.,rn,'t .
2
3

Now if 1. is the correct limit of this sequen ce, then the


3
terms of the sequence may be made arbitrarily close to as %,
n increases without bound (through integral values). That is,
2
the terms of the sequence can be made as near to as we
3
wish, by simply taking n large en ough. When this value of n
has been obtained, all subse­
quent terms of tRe sequence
will be even n earer to 2/3.
In the terms of the ex-
plan ation above, the seqw= ,ce %-t:
<;��i} will have a limit of 1,
if for any specified value of
c, we can fin d a counting
n umber N, such that for all
/1 -
O 7l,
2n -l
values of n N, then u i < c i.e. 11. _ / < c.
n 3 3n+l
>

Firstly, let us specify c = O· 1, what then is N?

Now 11 _ u i
3 n
1 _ 2n-11 = 12(3n +l)-3(2n-1)
13 3n+l 3(3n+l) I I I 9n5 +3
...... ( )
9n;3 • • •••• • • • X
2 9n+3 > -1-
Thus 1 - u I < O • 1, when ever < O •1, i.e.· ••hen
w
5
3 n 9n+3 5 O•l
on taking reciprocals.
408
LIMIT OF A SEQUENCE
9n 3 +
i.e. when ; > 10, i.e. when 9n 3 > SO, i.e. when n > 5%.
2
provided that n > Sj, i.e. n = 6,7,8, ..•
then
11 - �
Hence,

O l, [Any value 5 �,7,,,, will do for N, since for
n > 5 (or 6, or ,•••) then [ - u i < O•l, The least value
� 3 n
of n for which 1 - u i < O•l i n = 6 .' In other words, if we
3 n \
substitute n = 6,7,8,.••, then 1 - u i wil be less than O•l
3 n �
but if we substitute n = 5,4,3,2,1 then 1 - u i will be
3 n
greater than (or equal to) O·l, i.e. not less than O•l,

11 -
To check this, let us take n = 5,6.
Now u i = �+ • by line (X) above.
n 9 3
When n 5, 9�+3 = � = 0•1042 > O·l
4
When n 6, 5 + =
5 = 0·08772 < O•l,)
9n 3 57

Secondly, let us specify£ = 0,001, what then is N?


Now 11 -
u i = �+ , and thus
n 9 3 11 -
u i < 0•001
n
5 . 9n+3 1
whenever + < 0 •001, i.e. when - - > , , i.e. when
9n 3 5 0 001
+ . 2
9n 3
- - > 1000, i.e. when n > SSSj,
5
Hence, for all values of n > 555, i.e. n = 556;557;558;.••
then 11 - •
;:�ii will be less than 0 001.
[Here we can take N as 555; 556; ••• but the least value
of n for which
n
11 -
u i < O·O l is n = 556. Again, if we sub­

stitute n 556;557;558; •.• 1 - u i will be less than 0•001,
= 3 n
but if we substitute n = 555;554;553; •••,1 then
not be less than 0·001.J
u i will
n
11 -
Once again, we reach the conclusion that no matter how
small we decide to make£, (provided£; 0), then there will
exist a counting number N such that the deviation of u from
n
2
the number will be less than£ for all values of n > N.
3
In other words, for any specified£, we can determine N
so that
2n-1
If -
u i <£ for all members u
n n
of· the sequence
+ l} from n = N onwards.
{3n 2n-1 . 2
Hence, we say that the limit of the sequence { + } is
3n l 3
The idea of the limit is illustrated in the following sketch.
409
DEFINITION OF THE LIMIT OF A SEQUENCE

%-�1=1x-:��L
• • • • •
• •
• • •


0

*FORMAL DEFINITION OF THE LIMIT OF A SEQUENCE

A sequence of numbers {u1,u2,u3, ••• ,u ,···} is said to


n
have the I imit U, it tor any pre'-assigned £ > 0, there
exists a counting number N such that ju - u I < £ for a.I I
values of n > N. n

Un,
- - -- - - - -- - -- - - - - -
- /g_-)J._J
µ.t-jj.., )(

+. .u,.

0 I a. 3 :1 G ?i N N•I N•t. 11.

In the above sketch, we have a sequen ce {un } which has u


as the limit. From it, we see that no matter how small we
care to specify£; we can fin d a counting number .N such that
I U - un I < £ for all values of n > ��, That is, all terrr.s
after the Nth lie within the specified distance£ of U.

OTHER TYPES OF SEQUENCES.


Zn-l
The sequences discussed above { l + �}, {
1
} have
n 3n+l
limits 1, respectively, Other types of sequen ces which have
no limits have been previously mentioned. In these cases the
above definition does n ot hold.
For example, consider the sequence {n 2 } = {1,4,9,16,••••
•. , n 2 , •.•}, In the sketch of the sequence, we n ote that the
terms increase without bound (as n increases). We state this
property in the following manner:

410
DETERMINING LIMITS OF SEQUENCES
LL,.. "
"Givena positive num- •
ber G, as large as we G
please, then there exists ... "I
a counting number N such GI ...
,\' I
that for all n > N, I "' "' I .I.LN
u > G. II
..
.ll11.•"'' I
n .,.I*
i lU11 I
T'nus,.if we specify G as "I. "i-
10 , then u > G whenever 0 ,,.
6
n N 71,
n2 > 10 6 i.e. when n > /1Qb i.e. n > 10 3 •
Hence, for n = 1001; 1002; 1003; ••• ; u will be greater
n
than 10 6 •

--·· occurs in the sequence


Another type of example
nn <L,,.
{sin - } = {l,0,-1,0,1,0,-1,, •• }.
2 I • •
Here, obviously the sequence ', 1
'
I ,
has no limit. The sequence , 1
,
osc:illates from 1 to O, o 2 � 14- 5 6 7
'
to -1 and so on. \
\
'ti
'
-I �
DETERMINING LIMITS OF SEQUENCES
Up to this stage,,we have been concerned with the formal
definition of a limit in order to understand its precise
meaning. The definition does not determine a Zimit; it merely
verifies a Zimit. The actual Zimit of a sequence {u) ob-
viously depends on the value of un as n increases �ithout
bound (tJ1roUfJh integral values).
[For convenience, the notation "n -- " (read as "n tends
to infinity") is used to stand for "n increases without
bound". By this we mean that n becomes larger and larger,
eventually exceeding any given number N, no matter how large
N may be. We do not mean that n becomes closer and closer to
a number called "infinity". In Mathematics, the symbol "co"
and the word "infinity" have no meaning by themselves; they
do not stand for numbers. We must at all stages avoid any
suggestion whatever that n can be put equal to "co".]

The existence or non-existence of a limit for a sequence


{u } can be deduced from th� form of u as n 4 co. A Zimit for
n n
the sequence {u} exists or does not exist, according as to
n
whether a Zimit for u exists or does not exist, as n 4 co.
n
Thus, the sequence {u } is said to have the Zimit U if
Zim n
= u.
n-- un
411
DETERMINING LIMITS OF SEQUENCES
lim
[The meaning of the phrase n-><x> un ="� is contained en-
tirely in the formal definition above. That is, we say that
lim
u U, if for any given E > O, there exists a counting
n-><x> n
nu mber N such that Ju - un I < E for all n > N.]

To aatuallu determine the limit, we may have to make use


of the following results on limits.
lim ) lim
( i) If u = U and ....., ( v )= V, then
n-+a> ( n n n
lim lim ) t lim )
( a) u t v )=
n-><x> ( n n
u
n-+a> ( n
(v
n-><x> n
u t V

lim lim lim )


( b) u v )= n-><x> (un) v = u.v
n....., ( n • n n-><x> ( I'
u lim ) u provided VI 0
(c) lim (_!!) n-+a> ( n =
n-><x> V
n lim )
u
v•
v
n-><x> ( n
n
(ii) As n -+ oo, terms of the form r
(a) -->Oif - l<r·< 1·' e.g. (!=-)n + 0 as n + 00•
3
3)n +
(b) ++oo, if r > 1·• e.g. ( 2
- + CC\ if n +""
3 n
(c) if r< -1; e.g. (- -) +too ' if n + ao
-+,±oo'
2
(d)=+l if r= 1, and= t 1 if r= -1

Thus, considering the sequences previously treated


1
(i) {-- }. Here nlim (un) = lim 1
)= O, and thus the
2
n- 1 ...., n....., ( 2n_ 1
sequence {_!__ } has limit 0.
2
n- 1

{2
n-1
}. Here �
l� )
u lim 2 n-)i lim 2 - 1/n)
n-><x> ( 3n+l = n-><x> ( 3 + 1/n
(ii)
3n + 1 n ( n
_ lim (2 - 1/n) l
- lim 3 + 1/n)= 3' and thus the
2n- (
sequence { n+1} does have 2 /3 as limit.
3 1
1 lim
(iii) { 1 + �}. Here � ( u ) = (l + �)
n n� n n-><x> n
lim
= (l) + lim �= l+O= 1
n-><x> n-><x> n
Thus the sequence { 1 +�} has limit 1.
n
( iv) {log10n 2 }; {-n}. These sequences obviously have no
limit.
412
SET APP. 1B
(v) {(-l)
n
::;i}. Here �_!:(un) =
�_!: (-l) .�
n
! i�: , a d n

this limit does not exist, since n increases without


bound.

(vi) {sin90n° }, This sequence obviously has no limit, for no


matter how large n may be, the terms of the sequence
oscillate (1,0,-1,0, l, ... ).

EXERCISES SET APP lB


1, Consider each of the following sequences, and classify
them into o ne of three categories: Category A (a limit
exists, an d.find this limit); Category B (the sequence
increases or decreases without bound; i.e. the sequence
is u nbounded); Category C (the sequence oscillates).
(a) {.!} (b) { : ) (c) {n 2 } (d) {(-l)n n} (e) {cos l 80n ° }
n n 1
�l
-l
(f) {2n-l} (g) {S - .!!......} (h) {l} (i) {( ) } (j) {log n}
10 n e
1 3 n n n+ 3
(k) {- -} (.t) {2 - _ s} (m) {(1. ) } (n) {(- ) }(o) {(- l)
n
}
3n-l n n 3 2 2n-l
l n
(p) { (; ) } (q) { (� ) } (r) {sinl 80n ° }
l+ l+ l

- n 2n+l
(s) {l (n odd); 2 (n even)} (t) {sin
n
�} (u) {( l) }
n
+1 3 (x) { 3 1/ n}
2
(v) { n } (w) {2 [ (- )n-l )}
n
n+l
(y). {(l- 3) }
2
y n+l)(2n-1
(z) {(4-3) } ( ) (
n
)
( ;3) {�!12} 4n2

Given that u .!L. a.3


n
2.
n
for n
= + =
0,1,2, ..., in each of
3 n
the followin g cases, fi nd a, b and the limit of u as
n _.. oo.
n

(i) uo = 1, ul = 3 (ii) Uo
= 3, ul = 1
(iii) Uo = 11, ul 17

3. Select the correct answer in each of the fol lowing:


(i) {u } is a convergent sequence if
n
n +1
3

(a) u log n (b) u = 2 ( c) u = cos(n+l)l 80 °


n n n +2 n
n -n
(d) u (-l ) .(2n+ 3) (e) u n.3
n n
- n
(f) ll
n
(1·2) (g) u ( 3 4) .
n n
sin 2n
( ii) The sequence \ }
n
(a) converges to 2 (b) converges to -1
(c) converges to 0 (d) diverges
(e) osci ll ates between 11
413
SET APP. 18

(iii) The limit of the sequence { 2 / } is


! n

(a) l, (b) 2 (c) 0 (d) (e) 1 (f) 11.

4. (a) Which of the following seque nces has a limit?


2
n -1} n l} 5n+ 3 } }
(i) { (ii) {2 - (iii) { (iv) { (1)n
n+2 2n-l 5
n}
(v) { (-l) , 3
n

(b) What is the limit of each of the following sequences


(n+l) tan -l n tan-1 n
(i) !tan-l n ) (ii) { n } (iii) {
n }

s. Consider the sequence {u }


....!!....
n = n+2
( :/.) State the first 8 terms of the seque nce, and find
the limit U of the sequence.
(ii) Show that if lu - u I < 0·1,then n > 18. Explain
n
what this result means.
(iii) Verify that if n 19, then lu - u I < O·l but if
n
n = 17, lu - u I > O•l. What happens here to
n
lu - u I if n = 18?
n
(iv) Find the least value of N such that lu - u I < £
n
for all values of n > N, in the cases
(a) £ 10-3 (b) £ = •000001
2
{v) If lu - u I <£,show that n > < - 2)•
n -;;
3n-l
6. Consider the sequence { u = and find u if
n 4n+7 } '
lim
u = u.
n-><><> n
(i) Show that IU - u i = �� ' and hence if
n 4 n+7 )

lu-u I < 0 01,prove that n > 154�. Hence state
n
beyond which term do all terms of the sequence
{ 3n-l } d"1 ff er from 3 1· n a bso 1ute value by less
4n+7 4
than 0•01.
(ii) What is the least value of N such that
lu-�I < 0•01? Cheak this by actual substitution
a nd evaluation,using n = N,n = N-1, [Logarithms
will help in the calculations.]
(iii) Given £> O, show that lu - u I <£whenever
n

����������-
25
n > !.,( ""t;; - 7),

7, Write out the first 6 terms of the sequence {u} wnere


u = 3 - n-2• What is the limit U of this sequenge? For
n
414
SET APP lB
what values of n is IU-(3-n -2) I < 10-4?
(-l
8. Determine the limit U of the sequence {u 1 + r}.
n 3
. 1
Prove that IU - (1 + �·1 = """'n• a nd hen ce that .L < e:
3n j 3 3n
r
whenever n> tlog1oe:- l I 10�103. Find the least value of
l
N such that lu - u I < 10- for all n> N.
n
-1 n 1
9. 2 + 3 / • w -1 +
In'
1+2. lOn
n

, find the limit as oo of


S+3•10n
n 4

(iv) a (v)
n
b
n

10. Show that the sequence {�����} converges to the limit t·


3 3n + 13
Prove that if < e:, then (2n2-l) < e:, and
12 2n.;_1
- s1 2
3
thus that n> "'1<� + 1). Hence fi nd which terms of the
3n2 +s �
sequence } differ from the limit l by less than
1 {2n2_1 2
100·
*11. Consider the sequence {s } where
2 n -1 _ a(l-rn)
s a+ar+ar +••. +ar - and O < r < 1, a> O.
n

l-r
What is the limit s of this sequence?
n

a a(l-r ) - arn
(i) Prove that Is - sn I = I1-r - - - -
n

1-r I 1-r
(ii) Show that Is - sn I < e: , whenever ( ) > � ,and
e:(l-r )
l n

1
hence when n> llog1o( (�- )) I log10(�)}. [Thus a
r
e: r
suit able value of �;, for which Is - s I < £ for all
n
n> N, is any integer greater than
a 1
log1o[ log10( ).J
e: (l-r)l I 5 5
-;

(iii ) For the particular sequence s 5 + 7 + 72 + .••


+ � l' find N if e: = 10-6•
n

7 -
*12. Consider the sequence {S E r }. Sho� that the se-
n
n - n-1
quence r= l
(i) converges when -1 < r < 1. Find the limit S in
this case.
(ii) oscillates wh·en r -1, and diverges when r = 1 and
when ri > 1.
I
F or the convergent case, prove that
if S differs from S by less th�n 1%, then
n
1
n log1 o <-;) > 2 assuming r > O. If r = J.,z, determine
the least value of n satisfying this condition.

41S
SYSTEMATIC REVISION OF THE COURSE - TOPIC BY TOPIC

TOPIC 1
T HE R EAL NUMB E R SY STE M

A. REVIEW OF THE NUMBER SYSTEM


Copy the plan shown of the real number system; study it
carefully.

ItJTEGERS J RATIONAL REAL

T
NIJMBERS R NUMBERS R*
Positive
Zero Inte ers
Rationals
Neoative Fractions
Irrationals
For convenience, the set of integers has heen denoted by
J, and consists of 3 non-intersecting sub-sets (the positive
integers, also called ti1e natural numbers, zero and the nega­
tive integers). The set of rational numbers has been denoted
by R, and consists of 2 disjoint subsets - .(the integers and
the fractions). The set of real numbers has been denoted by
R* and consists of the non-intersecting subsets, (the
rationals and the irrationals).

1. Into which category would the following numbers be?


(Select the smallest set or subset in each case;
e.g. 7 is a positive integer and thus is also a ra.tion­
al number, and therefore a real number; however the
smallest set or subset of which 7 is an element is the
set of positive integer�.)
(a) -7 (b) l
7
(c) lz (d) •3 ( e) •3 (f) TT (g) 0
(h) 8 (i) it (j) e (!t) log 10 3 (.t) log e (m) ./ 4/9
3 e
(n) ·57 (o) ·57 ( p) 1·1 (q) 1•1010010001•••
(r) 7+3/s (S) rz X rs (t) 7i.1

416
NUMBER SYSTEM
2. Give a careful definition of a rational number in terms
of m, n where m, n are integers. What restrictions (if
any) are there on m and n?
3, (a) Can every rational number be expressed as a termin­
ating or recurring decimal?
If so, express 5, 3/5, 2/3, 2/7 in such form.
($) Is the converse true, namely that every terminating
or recurring decimal can be expres�ed {I� a !a�iona!
numb-er? If so, express 7•0, •41, •7, ·23, ·213, ·23
in rational form.
[Hint: for ·7, let x = ·7 i.e. x = ·777 •.. and mul­
tiply both sides by 19 1 giving lOx = 7·77 ... ; then
subtract x. For ·23, l�t y = ·2323.•., multiply
by 100 giving lOOy = 23•23 ... and subtract y.]

4. If J,R,R*,P,N,I represent the set of integers, rationals,


reals, positive integers, negative integers and ir-
rationals respectively, which of the following are true:
(a) R is a subset of R* (b) J is a subset of R
(c) N is· a subset of R* (d) I is a subset of R
(e) I is a subset of R* (f) R* is a subset of J

5. (a) If a, b are any two elements of a set s and (a+b)


is an element of S, then we say that the set s is
c-C.Q1�4. under..............
However, sometimes (a+b) give elements of s for
some values of a and b, but we can find one or
more exceptions to this, then the set s is not
....... under.............
($) Simila;:>ly,state properties to explain what is
meant by saying the set Sis closed under
(i) subtraction (ii) multiplication
(iii) division (division by zero omitted),
(y) Test the four- closure laws on
(i) the positive integers
(ii) the negative integers (iii) the integers
6. If a,b,c,d are integers such that b IO, d IO, c IO,
complete the following
a C
(a) � + .£. )
b d = ,$ b-d =

( ) �
C
y b � d =
(o) !+{ =
From these, can we say the rational numbers are closed
under the 4 operations?

B. IRRATIONALS; SURDS
1, Are the following statements "true" or "false"?
(i) Every surd is an irrational number. Consider /3,
417
IRRATIONALS
-.Fi, 314
(ii) Every irrational number is a surd. Consider lz, e,
11, log102.

2. Simplifl. (al_ 5/j x -2lz (b) (-s/j2__2


(c) (317-2/s) (3/7+2/s) (d) (2/j-3lz)(s/j+2lz) (e) (3-{2) 2

3. Find x in each of the following


(a) xlz= /is+ 3/a - 2>'32 (b) 154 + v'24=
(c) x-yi3=(2>'6+31z) 2 (d) x+2y+2/x-y= n+ho
rx
4. If 13 i 1•732, find, without tables, the approximate
values of
1 /j+1
(a) /48 (b) T36 (c) 2-h (d) 13-1

5. If a= /j+lz, find the value of a + 1, and hence the


1 1 a 1 2
value of a 2 +-;z. [Use the result a2 +a2 = (a +;) - 2.J
6. Simplify the following by rationalising the denominator.
3 1 1 1
(a) 15"+1 - 2/5-4 (b) /2+1 2 +-(-= '-- -2
( ) /f. -1)

7.
unequal,1- consider the forms (i)
(iii) Ix • /y (iv) h + /y.
rx
(a) If h, /y are surds, where x, y are not necessari!I_
+ ly (ii) h-ly
Are they also surds?
(B) Consider the forms (i) alz + bi2 (ii) a/2-blz
(iii) alz • biz (iv) afi + biz where a, b are
rational. Are surds of the form afi. closed under
+,-,x,.;?
(y) Consider numbers of the form r+slz, p+q/2 where
r,s,p,q are rational. Express each of the fo llowing
in the form a+blz.
(i) (r+sl2)+(p+qlz) (ii) (r+slz)-(p+qlz)
(iii) (r+sfi)(p+qlz)
. r+slz (here, rationalise the denominator
( iv)
p+ql2 first)
Are numbers of the form r+slz closed fot·
+,-, x ,f?
*a. Prove that 1z is irrational. (Start: assume Ii =t, where
a, b are both integers with no common factor.)

C. INEQUALITIES
1. If a,b,c are real numbers and a> b, then a-b> O,
and if a< b, then a-b< O.
For examp.,e,
� - 1- > J....• whe-re � is a
to prove n+3
n+4 1
po�itive integer; we should prove - > O.
Complete the proof. n+3 n+4

2. If a> b, show that


(i) a+c> b+c. [Consider (a+c) - (b+c)]

418
INEQUALITIES
(ii) ac>bc, if c>o [Consider ac-bc]
(iii) ac<bc, if c<O [Consider ac-bc]
3. If a,b,c,d are positive numbers, and a>b, c>d, show that
(i) ac>bd [Consider ac-bd = (a-b)c+(c-d)b]
(ii) a2 > b 2 . [Consider a 2 - b 2 ]
(1·11·) 1 < b
1 [C 1 l
onsidn -; - b]
-;
4. Answer true or false in each of the following
(a) Now 5 > 2; is (i) 5+9>2+9 (ii) 5-9>2-9
(iii) 5x9>2x9 (iv) 5 2 > 22
(v) - 1 1
>- (vi) 5x-9>2x-9
5 2
(b) Now 5 > -8; is (i) 5+9>-8+9 (ii) 5-9>-8-9
(iii) 5X9>-8X9 (iv) 5 2 >(-8) 2
1 1
(v) ->5
-- (vi) 5x-9>-8x-9 ?
8
(c) Now -5>-8; is (i) -5+9>-8+9 (ii) -5-9>-8-9
(iii) -5x9>-8x9 (iv) (-5) 2 >(-9) 2
1 1
(v) --> -- (vi) -SX-9>-8X-9 ?
5 8
*5. If x,y,z,w are real and x>y, z>w, what restrictions must
be put on these, if the following results are always
true:
(i) x+z>y+w (ii) xz > yz (iii) xz > yw
1
(iv) -<- 1
(v) x 2 > y 2 (vi) xz < yz
y
1 -
(vii)-> 1
?
X y

6. Solve the following inequalities, and sketch the solu­


tion on a number line
(i) 2x+3 .S 17 (ii) 3-2x > 14 (iii) x+3 .S 2x
(iv) 8 .S 3x+2 < 14
7. If P = {x:-3.Sx.S2}, Q = {x: -6.Sx.S-2}, find pn Q, Pu Q.
Sketch these on a number line.
*8. Find the solution of the following inequalities, and
sketch these results on a number line.
(i) x 2 < 16 (ii) x 2 .?. 25
(iii) 1 < 3 (Consider x > O, X < 0 separately.)
X

(iv)�� 4. (Consider x > 2, x < 2 separately.)


x-
9. Find the least positive integral value of n such-that
(i) 1 < ·07
n C ·>
:u -2 5 < •03
n- (iii) -1
zn
< ·001

D. ABSOLUTE VALUES
1. (i) Now 171 = 7 and j-,.7j = -(-:7) "' 7, and we have the

419
ABSOLUTE VALUES, FUNCTIONS
definition fxl = x .if x � 0 and lx l = -x if x � O.
AZternativeZy, Ix! = F-7. Thus j-71 = F7)1 =M=7
(ii) Use the values x = 7, y = -10 to determine which
of the following results are true or false

1� \
(a) lxyl = Jxl. Jy l (b) lx+yJ = Jxl + Jyl
1
(c) Jx-yl = lxl - IYI (d)
Y
=
IY I '
.1Ll
Y f 0
(e) lx+yl � Jxl + IYI (f) xy � lxyl
(iii) Evaluate l-312 -4 . I -SJ+ l-21 • l-61 + 3 . Joi
2. Solve the following, showing the solution an a number
line.
(a) lxl = 2 (b) J 3x-2 I = 4 (c) J2x-sl = I 1-xJ
1 > 1
(d) J2x+1J � 7 (e) Jx -21 > 4 (f)
J3x-41
1
(g)
r;.:zT � 3 (h) {x: [xi� J}(){x: lx+21 .$ 4}

TOPIC 2
THE CONCEPT OF A FUNCTION, AND ITS
REPRESENTATION IN ANALY.TICAL GEOMETRY

A. FUNCTIONS
L If y is a function of x, i.e. y = f(x), then
(i) x is called the •..••••.••••••••• of�he function
(ii) y is called the •.••.•.•••••••... of the function
(iii) t�ere is .•.•..••• value of y for each value of x
within the •••.••••• of the function.
[(iv) x, y are also referred to as the •••••••••••• and
•••••••••.••••.••. variables respectively.)
2. Study the following sketches
(i) (ii)

0 )(

-Y -x X

(v) '( (vi)

-)(
X 0

x�
-'t

420
REGIONS, FUNCTION NOTATION
(a) In each of the sketches above, state the domain and
range of the function
(b) Further, mark the regions for which
(i) y .s 9 - x 2 (ii) y s �. y?.Q (iii) y ?. 2x
(iv) y .s log1 ox (v) xy .s 1 (vi) y ?. 1 xI

�· REGIONS
1. On separat di�grams, sketch th�aphs. of
(i) y = /1-x (ii) y = -/l-x2 (iii) x 2 +y 2 = 1
Which of these graphs does not represent a function?
Shade in the region for which (a) y?. �. y?. 0
(b) y � -�, Y .S O (c) x 2 + y2 � 1
2. Show the region of the (x , y) plane in which
(i) I xI .s 3 (ii) 9 < x2 + y 2 .s 16
(iii) Ix I � 1 and IYI � 2 (iv) y � x2 and y .S f4-x 2
(v) y .s x 2 and x
2
+y2 � 1 (vi) y � x 2 and xy?. 1
x
(vii) y � x2
-4 and y s4 - x2 (viii) y?. x 3 and y .S 2-
x
(ix) y ?. 2 and y .s 4 - x 2 (x) y slog x and y.Sl-x 2

C. FlJIJCTION NOTATION
1. (i) If f(x) = 2x-3, find f(O), f(-5), f(a+b), f(l)
(ii) If F(x) = /2s-x2, find F(3), F(-4), F(3) - F(-4)
(iii) If g(x) = x2 -2x+6, what is the value of
x+h )- g(x)
(a) g(x+h) (B) g( h
2. 2x+3
Given that f(x) = prove that
)x+2 ,
(i) f(l) = % f(O) (ii) f(i) = [f(x)]-l

3. Let f(x) = ax+b, g(x) = cx+d.


Show that f(g(x)) = a(cx+d)+b, and write down the value
of g(f(x)). Show that if f(g(x)) = g(f(x)) for all val­
ues of x, then f(d) = g(b).
4. If f(x) I 2x-9 I for x < 2 Find the values of a, b
__ if f(l) = f(2) = f(3).
lax+f for X 2)
�:ence, evaluate
bx -2 for X > 2
2
f(-l)-f(2)-f(S)

421
TRIGONOMETRIC RATIOS
rnPIC 3
THE TRIGONOMETRIC RATIOS

A. THr SIX RATIOS


1. Find x and hence write down
� (i) sino (ii) coso (iii) tano

:l<
2. In the figure, determine x and y, and
write clown the values of (i) cote
(ii) cosec� (iii) sec0
3. �ithout tables, evaluate
sec 2 45 ° - tan 2 4S 0 •
(i) (ii)
cosec260 ° -cot260 ° +1
° °
tan 45 + tan 30
(ii')
1
1 - tan 45 ° tan 30 °
[Leave the answer to (iii) in simplest surd form.]
A
4. In the diagram, prove *CD = b cos C, ��
and hence that a = b cos C + c ,,os B.�
8
s. An is the diameter of a circle. The chord AC meets the
tangent at Bin D. If *fAC = 0,*AB x, express *AD and
*AC in terms of x and e.
�ence show that *CD= x(sec0-cos0)

B. At-.JGLES OF ANY MAGti I TUDE


1. The diagram shows the circle x 2 +y 2 = 1.
Pis any point on it.
(a) Fxpress cos0, sin0, tan0 in terms
of x, y.
(b) As 0 increases in size from 0 ° to
360 ° , what are the signs of x, y in

@�
each quadrant, and hence decide on
the signs of sin0, cos0, tan0 in
each quadrant.
(c) In which quadrant(s) is 0 if (i) sin0 < O 5 A x
(ii) tan0 > 0 (iii) sin0 < 0 and tan0 > 0 T c
(iv) sin0 < 0 or tan0 > 0

2. Express each of the following ratios ir. terms of e


(a) (i) sin(l80 ° -0) = (b) (i) sin(180 °+6) =
(ii) cos(l80 ° -0) = (ii) cos(l80 °+e)a
(iii) tan(l80 ° -6) • (iii) tan(l80 °+a) =
(c) (i) sin(360 ° -0) "' (d) (i) sin(-0) =
(ii) cos(360 ° -6) = (ii) cos(-0)
(iii) tan(360 ° -0) • (iii) tan(-0) •

422
TRIGONOMETRIC RATIOS
(e) (i) sin(360 ° + 9) (f) (i) sin(90 ° - 8)
(ii) cos(360 ° + 8) = (ii) cos(90 ° - 8)
(iii) tan(360 ° + 8) = (iii) tan(90 ° - 8)
3. t!ithout tables, determine the value of
(i) (a) sin 120 ° (b) cos 210 ° (c) tan 315 °
(d) cos(-60 ° ) (e) sin 270 °
(ii) (a) cosec 330 ° (b) sec 495 ° (c) cot(-240 ° )
(d) cqs 540 ° (e) tan 720 °
4. (i) If� is acute and sin a ·6, find without tables,
cos a and cot a.
(ii) If tan 8 = and 90 ° < 8 < 360 ° ,
2r +-1:....,;=4'-,1-!----4�
(a) justify the sketch
(b) find x and (c) hence find cosec8
(iii) Given that secs - ll and
15
wi ti10ut tables.

5. From the figure, prove that


sinS
(i) tans= (ii) sin 2 e+cos 2 e=f�
cos8
(iii) sec 8=tan 6+1 (iv) cosec 2 8=cot 2 8 +1
2 2

6. Simplify each of the following


1 _l__
(a) + (b) l(l-sin2e)(l+tan2e)
sec28 cosec2e
(c) � � (d) tana.cosa.coseca
1-sina - l+sina
(e) cos 4 e+cos 2 e sin 2 e (f) (ta�8 +cot8 )/cosec8.sec8
7. (i) If x = cos 2 e, express 5 - 3 sin 2 e + 7 cos 2 e in
terms of x.
(ii) Show that if x = cota+tana, y cosa+sina, then
x(y 2 -l). is independent of a.
8. T::liminate e if (i) X = 2 cos e, y 3 sin e
(ii) X = 2 cos e, y 3 sec e
(iii) X 2' cos e, y 3 sin 2 e
(iv) X 1 - sece, y z 2+tan8

9. Find e in the range 0 ° to 360 ° (both inclusive), if


13 13
(i) (a) sin e .. (b) sin28 =
2 2
(c) sin36 = I! (d) sin e 13
2 2 =2
1 1
(ii) (a) cose • - 7i (b) tane = 73

(c) 3 sec 2 e .. 4 (d) cot 3 e .. 1


(iii) (a) (cose+l)(cosece-1) = 0
(b) cos 2 8-cose = o (c) 2 sin 2 e + sin8-l • 0

423
SOLUTION OF TRIANGLES
(iv) (a) 3 sec 2 e - 4 cane - 2 = o
(b) 6sin 2 e-cos e-5 = O (c) 5s ine+B 4cosece

4°'"
C. SOLUTION OF TRIANGLES
A
1. (a) Fin d x, y with- (b) Fi nd et, a usin g
ou t logarit!-imic logarithmi c
\00
t ables. tables to aid a7·3s.:
the calcula- 21·+2
)+ 18
0
tions.
a. C
A
2. In bot} triangles, prove tha
(i) h = c sin Il ft c
(ii) h = :1 sin C b h
[I n the secon d figure,
use sin(180 ° -C) = sin CJ. B _ a. D � 8 a. O
a . b c (\80°-c)
Hcnae de c,uce tne .
, sine ru1e = =
s inA sinB sinC

3. Find x, witho ut the use of tables


(i) (ii) (iii)

6 ,.
� �
° X 30°
60
�-----�����....:...::-->.
1 5°
I n 6 ABC, 4
4
4. 2
(i) B=120 ° , C=45 ° find b:c
(i i) a=3b and sin A = 2/5, find sinB.
l' l00sin50 ° sin70 °
5. ( ) Prove x =
sin 20 '-,,
°

---
X.

(ii) 70 So
' 0
x sin e sinljJ
Prove y sin ( e+ij,)

6. From a point A o n the deck of a sh i p, the angle of ele­


vation of an aircraft is 34 ° , and from an observation
point 80 III. vertically above A, the angle of elevati on
is 32 ° 30'. Calculate, to the neares t 10 m , the height
of the ai rcraft above the deck.
7. Two points A and!: on one bank of a straight stretch of
river are 73 m apart. If C is a point on the other
bank and *Aac = 63 ° 21', *BAC = 51 ° 47' calculate
(i) *AC (i i) the width of the river.

8. (et) I n fi g (i), prove (,)


c 2 =h 2 +(a-x)2 =a2 +b2 -2ax
= a 2 +b 2 -2ab cosC

424
SOLUTION OF TRIANGLES
In fig (ii), prove c 2 = h2 +(a+x) 2 = a 2+b 2 -2ab cosC,
noting cos(l80 ° -C) = -cosC.
(B) Write down similar forms for the cosine rule begin-
ning successively with
(i) a2 = .. . (ii) b 2 (iii) cos A = ...
...
A �·
(iv) cos .3 •.• (v) cos C =

"•""'
=

9. (a) (B) ,.-Find c in surd form in


<•>. <•>.
A
D
� e "" c 4
10. In figures (y), (o), determine
the magnitude of A. 3 �
n
B 6 c ) S ,
11. In t:, ABC 1
(i) b=5, c= 4 ,cosA = ; find a B t '
8
(ii) a=3, b=4, c=/37; find the
12. In the ouadrilateral ABCD shown,
determi�e (i) *ACB
(ii) *ACD (iii) *BD
A
13. A ship steers a course N 35 ° E for 20 klll and then
a lters course to S 15 W for 30 km.
°

6�
(i) Calculate the distance and bearing of the ship
from its first position.
(ii) Calculate how much further the ship will have to
travel on its second course to be due South of its
first position.

l.'.i. In the figure, show �


(i) *QR2 =8(1-cos2B) by the cosine rule and
(ii) *QR2 = (2.*QS) 2 = 4(2 sin B)� Q R
Hence, prove 1 - cos 28 = 2 sin 2 e. s
15. ABCD is a cyclic quadrilateral.
Prove that (i) x2 = a 2+d 2 -2ad cos6 O/C....--,...--'!1.1
(ii) x 2 = b 2 +c 2 +2bc . cosB
a 2 +d 2 -b 2 - 2
i!ence, show that cosB = 2(ad+b c
c)

16. (a) In both


= \ah
(B) Write down two similar
17, Find the area oft:, ABC in which a =
(i) C = 45 ° or (ii) C = 135 ° .
18. ABCD is a parallelogram, in whic:1 *AB = B·crn.*BC = 6"cmand
*ABC = 60 ° . Determine as surds
(i) the· length of the diagonals AC, BD
(ii) the area of the parallelogram.

425
THE STRAIGHT LINE
TOPIC 4
THE LINEAR EXPRESSION AND THE STRAIGHT LINE.

1. On separate sketches, show the following lines.


(The form of the given equation should be studied
first.)
(a) X = 0 (b) y 0 (c) X = 3
(d) y -2 (e) y = X (f) y = 2x+5
(g) y -/Jx (h) y -/ix-7 (i) � 3
+.Y.= 1
2 4
+3 - +3
(j) y-3 = l(x-5) (k) y+3 = -2(x-1) (f..) x-1 = 7-1
.Y:!:.J.
2. �rite down the equations of the following lines
(indicated by p,q, ... ,v).
(i) (i) y t<\.(ii) (iii)

(o,b)
� ,..
t
-x
i b
X
(-Cl,O 0 )(

-Y 0 Q

Tancx"'m
Also state the g radient result
for the line AB.

-y X
3. A,B,P,Q are the points (-1,-3), (2,-5), (2ap,ap 2 ),
(2aq,aq 2 ) respectively. Find the gradient of
(i) AB (ii) PQ in its simplest form.
4. Find the gradient of the following lines
(i) Ax+By+C = 0 (ii) 2x+3y-7 = 0
Hhat is the measure of the angle of inclination of
line (ii) to the x axis?
5. (i) Prove that the points (-4,5), (2,-3), ( -1, 1)
are collinear·.
(ii) Which of the points A(3,-2), B(-2,1), C(-1,-3)
belong to S where S = {(x,y) : 5x-3y-4 = O}?
Which points of S lie on the x, y axes?
6 •. The line ax + by = 7 passes through the points
(1,-1), (3,4). Find the values of a, b. ·

7. Find the equation of the line


(i) through (-1,3)
(a) parallel to the x axis (b) with gradient 2/3
( c) and inclined at 135 ° to the x axis
(d) and through (-4,-2).
(ii)- having an intercept of 7 units on the positive
y axis (a) and an intercept of 5 units on the

426
THE STRAIGHT LINE
negative x axis (b) and with slope -3/5.
8. (a) Show that the equation
(i) Ax+By+K = O, where K varies (K 'I- C), represents
a straight line parallel to the line given by
Ax+By+C = O.
(ii) A(x-x, )+B(y-y,) 0 represents a straight line
through the point (x,,y 1) parallel to
Ax+By+C = 0,
(b) Use both methods of (a) to find the equation of the
line through the point (2,-5) parallel to the line
7x - )y - 4 = O.
9. (i) By consideration of gradients, show that the
points A(-3,-13), B(-4,-7), C(-6,8), D(-5,2) are
the vertices of a parallelogram.
(ii) Find p if the join of the points (5,1), (2p,p-2)
is parallel to line 2x-3y-8 = O.

10. (i) Find the coordinates of the points of intersection


of the lines 3x-5y = -7 and·7x+2y 11.
(ii) Prove the lines 3x-5y = -7, 7x+2y = 11, 8x-3y = 2
are concurrent.
11. Show that the point (-4,3) lies on each of the lines
(i) 2x-5y+23 = 0 (ii) 3x+2y+6 = 0
(iii) (2x-5y+23)+A (3x+2y+6) = 0.
12. l/ithout finding the coordinates of the point of inter­
section of the lines 4x+7y-6 = O, 5x-8y+3 = 0, de­
termine the equation of the line through the point of
intersection of the line 4x+7y-6 = O, 5x-8y+3 = O and
(i) passing through the origin
(ii) passing through the point (1,-2)
(iii) parallel to th_e x axis
*(iv) parallel to the line 2x-3y = 5.

;Ax. +B'j +<.-::o

-��
Ot I
\,
Ax. + &x.+t :o
I •

,o
---�:,----
-1.,'

,-i-
Geometrically, two lines may have (i) ....,(ii).O..,
(iii) ..•.............. number of points in common
according as to whether the lines
(i) ..............•.•••.·(ii) are .•.•...•..•... • or
(iii) .• co��ci..d�. respectively.
(b) Algebraiaally, the aol'l'esponding pairs of equations
which have (i) a unique common solution
(ii) .••....• common solutions (iii) a ••...•number
of common solutions are said to be
(i) ...•......•....• (ii) .......•. (iii) d�pudu.t.

427
THE STRAIGHT LINE
(c) Derive the condition AB' - A'B i O for the lines
Ax+By+C = 0 A'x+B'y+C' = 0 to have one point in
common. What can be said about the line·s when
AB' - A'B = 0 and (i) C = C' OP (ii) Cf c' ?
14. Investigate the pairs of equations below, using the
method _of question 13
(a) f2x-3y-4 = 0 (B) f 2x-3y-4 0 (y) f2x-3y-4 0
.4x-6y+l = 0 l4x-6y-8 0 l4x+6y+l 0
15.
(a) In the figures, mark in
the regions for which
(i) 2x-3y+6 < 0
(ii) 2x-3y+6 > 0
(iii) 5x-2y-7 < O
(iv) 5x-2y-7 > 0 __."-'..__...,___�
-l 0

(b) The lines 2x-3y+6 = 0, 5x-2y-7 0


divide the (x,y) plane into 4 re­
gions, P,Q,R,S.
Verify that in Q, 2x-3y+6 > 0
and 5x-2y-7 < O.
Write down similar results
for P,R,S.

16. On the same diagram, sketch the lines represented by the


equations x-y = O, 3x-y = O, 5x+y = 24. Find the coord­
inates of the vertices of the triangle formed by these
3 lines, and determine the inequalities which are true
for the interior of this triangle.
17, Sketch, on the same diagram, x = 1, 2x+y = 9, x-y+5 = 0
and mark the region for which the 4 inequalities x? 1,
y � O, 2x+y � 9, x-y+5 � 0 simultaneously hold,
18. If R = {(x,y) : y? x 2 }, S = {(x,y) : 0 .:Sy .s �}
T = {(x,y) : y? -x} show clearly the regions defined by
(i) s n T (ii) R u s (iii) R n T

19. (i) State the condition for the lines y m1x + b1,
y = m2x + b 2 to be perpendicular to each other.
(ii) Show that the lines
(a) 3x-4y+7 = O, 8x+6y-1 = 0 are at right angles
(B) 3x-4y+7 = 0, 8x+6y-1 = O, 6x-8y-13 � O,
12x+9y-7 = 0 enclose a rectangle.
20. (i) Find k if the line kx+4y+l = 0 is perpendicular to
the join of (1,3) and (4,5),
(ii) The line cx+dy = 10 passes through the point (1,2)
and is perpendicular to the line 3x-4y-1 = O; find
c, d.
428
THE STRAIGHT LINE
21. (a) Show that the line represented by
(i) Bx-Ay+K = 0, for varying va1ues of K, is perpen-
dicular to the line given by Ax+By+ C � 0.
(ii) B(x-x1 ) - A(y-y1) = 0 passes through the point
(x1, Y1), and is perpendicular to the
line Ax+By+C O.
(b) Use both methods of (a) to determine the equation of
the line through (-4, 3) perpendicular to the line
Sx-2y-9 = 0,

22. The perpendicular distance of t:1e point (x 1, y1) from the


line Ax+By+C = 0 is
+B
I Ax1 Y1 I The corresponding displacement is
+C

1 /A2+!f2

and this may be greater than, less than or equal to zero.


A,B,C,D are the points (-1,1)_, (7,4), (-8,-1), (4,3) re­
spectively, What are the perpendicular distances of
A,B, C ,D from the line 2x-Sy+8 = O? Which of these points
are on the same side of the line?
23. Show that the line
(i) /Jx-y+8 = 0 is a tangent to the circle x 2 +y 2 = 16
(ii) 6x-2y+ll = 0 is wholly outside the circle y=±ll-x
2

24. (i) The line 3x-4y+k = 0 is distant 2 units from the


point (1,-3). Find two possible values of k,
(ii) Show that the point A(l,2) is equidistant from the
lines Sx-12y = 7, 3x+4y = 1. Wha·t is the geo-
metric significance of this result?
(iii) If the line y mx+b 2 is 1 unit from the origin,
prove that b 4 = m 2 +1.

25. (a) A, B are the points (x1,Y1), (x 2 ,Y2 ) respectively.


Write down the formulae to obtain the
(i) I *AB I
(ii) coordinates of N where M lies on 'AB' and
*AH= *MB.
(iii) coordinates of P where Pe: 'AB, and *AP:*PB =k:l
(B) If A = (-7,2), B = (S,8), use the results of (a) to
obtain
(i) l*ABI (ii) the coordinates of M
(iii) the coordinates of P where *AP : *PB 4 : 3
and the coordinates of Q where *AQ: *QB = -4 : 3
26. Show that the points A(4,6), B(-3,4), C(-1,-3) form a
right-angled isosceles triangle, and find its area.
27, A,B,C have coordinates (-1,-6), (2,-1), (5,3) respect­
ively. Find (i) l*BCj (ii) the equation of BC
429
INDICES
(iii) the length of the perpendicular from A to BC
(iv) the area oft:. ABC.
28. The points P, Qare on the sides AC, BC respectively of
a triangle ABC, such that *AP *PC 3: 1 and
*CQ : *QB= 1 : 3, If A= (-3,7), B = (11,3), C = (-1,-9)
fin d the coordinates of P, Q, and hen ce prove that PQ is
parallel to and on e quarter of AB.
29. (i) rRT, is the diameter of a circle, centre s. If
R = (-1,S), S = (3,4) find the coordinates of T.
(ii) A(-4,2), B(2,l), C(-1,-5) are vertices of a paral­
lelogram ABCD. Using the fact that the diagon als
bisect each other, find the coordin ates of D.
30. The cen troid of a triangle divides each median in the
ratio 2 : 1.
(i) For a triangle ABC in which A= (-4,-5), B = (2, 7),
C = (S,-8), fin d the coordin ates of the cen troid G
(ii) Obtain the coordinates for fJ. ABC with vertices
A(x1 , Y1), B(x2 ,Y 2 )' C(x3,y3 )
31. The trian gle ABC has vertices A(0,4), B( 3,0), C(3,0).
Find the coordin ates of -
(i) the orthocentre
(ii) the centroid (iii) the circumcen tre.
[For (ii), you may not assume the result in question 301
[Note the (a) altitudes, (S) medians, (y) right bisect-
ors of the sides of a triangle meet respectively at the
(a) orthocentre (b) centroid (c) circumcentre.J
32. I n a trian gle ABC, 'AD, is a median .
Prove that *AB 2 + *AC2 = 2*AD 2 + 2*BD 2
[liint: let B = (-k ,O), C = (k,O), A = (p,q)J
33. Prove that the angle in a semi-circle is a right-angle.
[Hint: If All is the diameter of the circle, say x2 +y=1, 2

and C is an y point on the circle, let A,ll,C have coord­


in ates (-1,0), (l,O), (cos0,sin 0) respectively. Use
gradients. J
34. Prove that the altitudes of a triangle are concurrent,
[Take the vertices at A(O,a), B(-b,O), C(c,O).J
TOPIC 5
SEQUENCES, SERIES
[Note to level 2F students: Revision on the topics
Mathematical In duction, Limits of Sequences has not been in ­
eluded in this section. They are revised in the Appendix
revision section.)
A. INDICES
1, Assuming that a� O, and m, n are ration al, complete the
following index laws and properties:-
n
m n
(ii) a + a (iii) � ) =
m n
a >< a
m
(i)
(iv) a O = ••• (v) a-n"' ...
(vi) a /n =
m

2. Use these results in the following


-
(a) Simplify r2; 30; 16\ 810•1s; 27 213; (6\) -%
2 1
(b) Simplify (i) y 3/7 >< y-2/7 >< /h (ii) (l0-'9 ><10 } ) ; 10-
2

430
SEQUENCES AND SERIES
2 -
(c) Fin d if (i) 3 = 9 4y527 ;.5
x
x X
-31
6 2 X 12 /2
I/
(ii) 2 , 3Y =
x

18-!,
x-1 2x -3
(d) Simplify (i) B _ _; 4 .
l x +l
3 xx -
16 ( )
2
(e) Arrange in ascending order of size 2 4/s ; 3 112 ;
[Hint; Raise each to the power 10]
3. Solve for 2x +l x 2
27 - (b) 53x-4 = 1
X
(i) (a) 9
x +2
(c) (l) = 12. 16x
4
-
(ii) (a) X 413 = 16 (b) X I12 = 2 113 • //4 • 5 1/G
I

x -1
(iii) (a) 2 = 53
x
(b) 3 = 0·22
[Logarithm tables needed in both)
(iv) (a) 9 - 4, 3 X + 3 = 0 [Let V = 3x )
X

x+l
(b) 4 - 3
X
2 = 16
4. (i) Solve for x an d y: 4 . 8-y = 1 and 25 = 5Y ,125
x x

(ii) Simplify (a) (x l/2 + y 1/i/


(b) (x l/3 - y¥3)(x 2/3 + xl/3 y1/3 + y%)

8. SEQUENCES AND SERIES


1. Obtain the first 5 terms of each of the followin g se-
quen ces
(i) {3n +2} (ii) {n 2 } (iii) {n(�+1)}
(v ) {(-l) 3n -1}
n

2. The nth term of a certain sequence {u } is given by


n
u n = 7n-5.
(a) Fin d (i) u100 and (ii) (u31-uG)
(b) Are the numbers (i) 213 (ii) 520 members of the
sequence. If so, which terms are they?
3. The sequence {un} = u1, u2 , u3, whilst the series

l: u = u1+u2+u3+••• Determine the value of the follow­


r= l r ing sums.
5 6 4
(i) l: (3r+2) (ii) l: 2 r (iii) l:
r=l r= l r= l
10
(iv) l: (-1) r r
r= l
431
A • p I s AND G . p I s

4. Express the f oll owing in .sigma notation


(ii) <%>+<%> 2 +<%> +
3
1 3+2 3 +3 3 + ...+ 42
3
(i)
(iii) 1.2-2.3+3.4-4.5+5.6
(iv) f(x1)h+f(x2 )h+f(x3)h+.•. +f(x0)h
n
5. For any series E u r =
r=l
denote the SUF.l of n and (n-1) terms respectively of this
series, then u = S -S _ .
n 0 0 1
If S 3n 2 -4n, show that S =3(n-1) 2 -4(n-1)�3n 2 -10n+7 '
n n-1
and hence that u n = 6n-7. Find the value� of u 1,u 2�u 3, ...
b. The sum of n terms of a certain series is given by
7n-2n 2 . Calculate
(i) the sum of the first 10 terms
(ii) the sum of the first 9 terms
(iii) the tenth term (iv) the nth term using the result
above and hence verify the result in (iii).
7. For a certain series, the sum of the first n terms is
2°-1. Find the formula for the nth term.
C.
= ARIT�METIC AND GFOMETRIC SEQL:NCES
- l
AND SERIES
AR ITH�1ETIC SEQUENCE GEOMETRIC SEQU�NC�
(or A.P.) \a+(n-1 )di (or G.P.) \ ;:irn- .{
I. Scqucnct:) is a,a+d,a+2d,.. Sequence is t1,ar,ar 2 , ...
�. .. a+(n-1 ld = arn-1
Un Un

3. Test for A.P. Test for G.P.


uz-u1 = uru2 = .. = d uz/u1 = u3/u2 = . .. = r

4. If a,b,c are in A.P.,then I f a,b,c are in G.P., then


b = C
b-a = c-b, i.e. b=\C a+c l i.e. b 2 = ac.
a b

5. A.M. of a and b is .L!....E. G.M. of a and b is t/ab


2
n n
ARITHMETIC k-1
E (a+(k-1 )d] GEOMETRIC SERIES E [ ar J
SERIES
k=I k= I
n
6. sn = n(a+ ) where .e = u s = a(1-r ) = a-r£ ,
2 .e n n 1-r 1-r
n for Ir l < I
> f/0+(11-l)d] n-1)
.. = a(r = r.e.-a
r-"I for I rl > I
r-1
I im = _a_ if and only
7. Sn = s
n-- 1-r
n if Ir I<Iarn
s-s = _a__ a( 1-r ) = --
n 1-r ·1-r 1-r
432
A.P.s AND G.P.s
[Note: The proofs of the results 6, 7 should be learnt, ]
1. For each of the sequences (i) 3,7, 11, ... (ii) 3,-6,12,..
determine the values of
(a) the eighth term (.b) the nth term (c) the sum to
8 terms (d) the Sl.llll of 2n terms.
1 2 5
2. For each of (i) {1, 2 , 8, ... }and (ii) { , , , ..} find
2 3 9 2 3 6
(a) the fifth term (b) the (n+l)th term
(c) the sl.llll of 5 terms (d) the sum of n terms.
3. Calculate x if the terms x+20, 4+Sx, x+4 are the first 3
terms of (i) an arithmetic sequence
(ii) a geometric sequence.
Find the next term of the sequence in each case.
4. If x, y and 9 are the first 3 terms of a geome-t�ic se­
quence, and y, x and 2 are the first 3 terms of an
arithmetic sequ�nce, find the values of x and y.
5. (i) For the sequence {u = a+(n-l)d},
n
u2 = -4 and
15
u6 = -24. Find the sequence, u15 and E u
r=l r
n-1 4
(ii) For the sequence {u = ar }, u2 • us = 27•
4•
n
5
Find the sequence, and E �·
k= l
6. The number indicated belongs to the given sequences.
Find which term it is.
32
(i) -2,1,4,.... 163, •.. (ii) 81,54,36, ..., , ...
3
7. (a) Find the (i) arithme.tic mean
(ii) geometric mean, of (a+b) 2 and (a-b) 2
(13) Insert (i) 3 geometric means between 2 and 162
(ii) 4 arithmetic means between 2 and 17.
8. (a) Find two numbers whose arithmetic mean is 39 and
whose geometric mean is 15.
(b) If 3,x,y,375 are in (i) GP (ii) AP, find x and y.
9. Consider the multiples of 7 between 100 and 500.
Determine (i) f,l, the first and last of these
(ii) n, the number of these (iii) s, their sum

10. (a) With the usual notation for a geometric series,


a = 6, lrl = ·4, find the limiting sum of the series
for (i) r > O (ii) r < O
lim
(b) Find the (i) sum to n terms (S )
n
(ii) s
n-- n
of the series 4 - 2 + 1 - \ + .••

11. (i) If k.
E 3r • 5, find r
(I)

k•O
(ii) The second term of a geometric series is 6 and the
433
A. P. s AND G. P. s
limiting sum of the ser ies is 24, find the series.
12. Express each of the following recurrin g decimals as in­
finite gl'ometric series, and· hence obtain each in the
form p/q, 1-1here p, q · are integers with no conunon
factor,
(i) •7 (ii) · 17 (iii) ·17 (iv) •203 (v) ·407

13. (i) The sums of the first n terms for each of the
se r ies 23+25+27+... and 1+4+7+... are equal;
find n.
(ii) The sum of a cer tain finite a rithm,etic series is
135. If the first te r m is 3 and the last term is
24, find how many terms there are.
(iii) A certain fi�ite geometric series of 10 terms has
sum 3069. If the common ratio is 2, fin d the fir st
term.
14. ( i) Find the sum of
(a) 100 (b) 10 1 terms of the series 1-3+5-7+ ....•
30 10 r -l
Evaluate (a) l: (3n+l) (b) E (-l) 2
r
( ii)
n l
= r l
=

r r
(c) ;{(i) -1 + (-�) }
1
15. (i) The first and third terms of an aritlunetic se-
quence are 25 and 19 respectively.
Find the sequence, and the number of
terms required to make the sum of the asso.ciated
se r ies equal to 82.
(ii) How many terms must be taken for the sum of the
series 8 1+78+75+... to !ie zero?
16. (i) For the ser ies 1+·6+·6 2 +... • find how man y terms
must be tal:en for the sum to be greater than 2·49.
Can the sum equal 2·5?
(ii) Find th<: difference between the limiting sum and
t'.;c, sum of 10 te r ms of the series l+· 2+,04+..•
17L Find the sum of n terms of the geometric series
7 + 3� + 1� + ... , an d deduce the limit of this sum as
n tends to in finity. Find. the least value of n if the
sum to n terms differs fr·om this limit by less than 0·01.
18. Show that there are two geometric prog ressions in which
the second term is -4/3, and the sum of the first
3 terms is 28/9. Show also that one of these progress­
ions has a finite limiting sum, and in this case, find,
the sum.
19. The nth term of a sequence is a(\) n+bn. If the first
434
3 terms are 11,10,ll, ••. fin d a and b, and hence find
the fourth term of the sequence. Fin d the sum to
(i) 10 terms (ii) n terms of the correspondin g series.
20. The sum of the first 4 terms of an arithmetic series is
24, and the sum of the next three terms is 39. Fin d the
first term and the common difference.
21. The first, third and sixth terms of an arithmetic se­
quence form a geometric sequence. Find the common ratio
of the geometric sequence.

22. The midpoin ts of the sides of a square of side 2a are


joined to give a second square, The midpoin ts
of the sides of this square are join ed to give r
I , ·- :
a third square. and this process is repeated I ; �
"-
in definitely. Calculate the limit of the swn of r ', L __j / I
the areas of all such squares. ___ ' ,., __ /
23. A polygon has 25 sides, the lengths of which starting
from the smallest side form an arithmetic sequence. If
the perimeter of the polygon is 1100 cm and the
length of the largest side is ten times that of the
smallest, find the length of the smallest side and the
common differen ce of the arithmetic sequen ce.
24. ·• According to one legen d, the in ven tion of the game of
chess so pleased King Scheran that he promised the
in ven tor Sessa, anything he asked for as reward. Sessa
chose 1 grain of wheat for the first square, two grain s
for the second square, 4 grains for the third, an d so on.
Calculate the total number of grain s he should receive
(a chessboard has 64 squares), an d the side of a cubical
barn required to contain them, assuming that 1 cubic
(:ffi of space contains so· grain s of wheat.

D. THE I DENll TY x n -c n
1. If S = x n-1 + x n- 2c + x n-3c 2 + x n-4c 3 +..• + xc n -2 + cn -1
determine results for xS and cS, an d hence ?Y subtrac-
tion grove that (x-l)S = x -c
n 2 n 2 n 1
i.e. x -e = (x-c)(x - +x - c+x -3c 2 +.•.. +xc - +e - )
n n
n n n

2. Use the result of question 1 to verify that


(i) x 3-c 3 = (x-c)(x 2 +xc+ c 2 )
(ii) x 5-c 5 = (x-c)(x 4 +x 3c+x 2 c2 +xc 3 +c 4 )
and write down correspon ding results for x 6 -c 6, a 7-b 7 •
f(x)-f(c)
Simplify for x 1 c, when
x-c
3.
(i) f(x)=x (ii) f(x) = x 6 *(iii) f(x) = x o/2
3
*(iv) f(x) = x- 5

435
CONTINUOUS FUNCTIONS
TOPIC 6
THE TANGENT AND DERIVATIVE
t• BASIC IDEAS ON CONTINUOUS AND DISCONTINUOUS FUNCTIONS
Read the following carefully

POLYNOMIALS
ofthe f orm f(x) = ax 2 +bx+c, f(x) c ax 3+bx 2 +cx+d, etc
(where a,b,c,d, .•. are real) are always continuous.
That is, g
raphically, they represent unbroken curves;
f
or a O, y = ax +bx+c has the shape\._;/
>
2
whilst y= ax 3+bx +cx+d has the shape or ;
[When a< O, the curves
2 are inverted.] ;J
f(x)
(i) TYPBS SUCR_A1 y = gx y= _L_ , y = k + _j.__
b ( ) , gx ( ) gx ( )
y= kx + 1 ) etc, will be continuous, D1'ovide1 g(x) IO.

4
g(x
Thus y - x2+1 · p and y -_ 21 _
�· x +1
_ X .,: s I

, :
represent unbroken curves, since x +1 IO for real values
ofX, 2

i( i) If g(x)=O for real values of x, then discontinui-


ties occur in the curve.
1

Thus y= -2 y 1 +
'\
� .....1...2
x- ---' ___ J_�_
X
·x. 0 �

x-2
/

y= + l_
x( -1) (x-3)
X
' 1 ). �
X

I
/ I
I
have discontinuities, since g(x= ) O for some value s( ) of x.
x
i( ii) A SPECIAL CASE of the form f ( ) arises when f x( )=O
gX ( ) x2 +x -6
and g(x) = 0 for a common value of x. Thus y=�is such
( ) = x +x-6 and g(x) = x-2 both equal zero when x= 2,
that f x
x +x-6 2 0
i.e . � 2
is then of the form o'
and has no meaning.
x +x-6 = (x+3) (x-2) x+3, p1'ovided x I 2. How-
Thus F(x) = x-2 x-2
2

. ·�Xx-�
ever F(2) is undefined. '( �:i<+ 3 \j"� x-a) Y
The graphs of y= x+3
and y=F(x)
x +x-6
are identical, = 2 x-2
_.,...�......,,........��:z.
Ix
except when x= 2. .......,. 0

436
LIMITS
1, Many other types of functions, both continuous and· di�­
con tinuous, occur in our work on Calculus. [.Further, if
f(x), g(x) are continuous fu�ctions of x, then f(x)±g(x)
and f(x).g(x) are also continuous functions.]

8. LIMIT OF A FlJIJCTION
Read the fo 11owing carefu11 y

9
(i) It can be easily seen that as x -+- 3, x 2 -+- . [In fact,
when x = 3, x 2 = 9.J Thus lx 2 -9I can be made as small
as we please by taking x near enough to 3. [The absolute
value is taken since we may make x near to 3 either
from less than 3 or from greater' than 3.]
Another way of saying this, is that if you ive me any
9
small number, say 0•00001, then I can make Ix§- 1<·00001
by simply taking x near enough to 3. (We aan determine
just how near this has to be, i.e. withtn what "interval
x must lie.) [In this case, we aan show that if
-00001 -00001
- < x < 3 + - - -, then I x2 -9 I < ·00001. [How,
3 - -- 6 6
in actual fact the interval was obtained, is not dis­
cussed here, )
(ii) Now consider f(x) = x -9 = Jr--3x-3
)(x+3) = (x+3) if x; 3;
2

x-3
f(3) is undefined. Here as x-+- 3, f(x) -+- (3+3) 6.
This means that lf(x)-6j can be made as small as we

l:�;9
please by taking x near enough to 3. That is, given a
small number£ say, then we can make - 61 <£, by
simply taking x near enough to 3. As before, we can de­
termine just how near to 3 this must be, i.e. the inter­
val within which x must lie. Denoting the width of this
interval by o (which of course depends on£),
then jf(x)-61 <£, provided that 3-o<x<3+.S.
lim
*(iii) If the limit of f(x) as x -+- a is L, i.e. if f(x) = L,
x-+-a
then the formal definition states:
Given £ > O, then we aan find .S, such that lf(x)-LI < £
whenever x lies in the interval a-o<x<a+o,
i.e. for Ix-al < o. y
Graphically, if we want jf(x)-Lf
i.e. l*DEI or l*FGI to be less
than£, we simply have to take
x within the interval [a-o,a+o) L+E 1--�-----'11F-­
whence automatically D, G lie -L& 1-----1...,,.'------
on the arc PQ. L

Note The definition of


lim f(x) = .L makes no mention
'
x-+-a
of the value (if it exists) of a. o+G X X
the function at x = a.
0

437
LIMIT THEOREMS
x 2 -9 x2 -9
ntus lim • 6, although when x • 3, ! does not exist,
x+3 x-3 x-

C. LIMIT THEOREMS
lim
If f(x), g(x) are functions of x, and +a f(x) = L,
x
lim
x+a g(x) • M, and k is a constant, then
lim { lim
(i) k.f(x)} • k . + f(x) • kL
x+a x a
(ii) lim {f(x) ± g(x)} • lim f(x) ± lim g(x) • L ± M
x+a x+a x+a

��:n
lim { l im lim
(iii) f(x) .g(x)} • + f(x) • x+a g(x) • LM
x a
!:� !:� f(x) I!:� g(x) .. �. provided that M,'O
x+a
(iv) =

Thus (a) lim x -5x+6


2
lim
-5x+6) / ;:� (x 2-4) = 2/-3 = -1.
x+l (x
2
x+l x2-4 3
lim
(B)
lim x -5x+6
2
lim (x-2)(x-3)
x+2
(x-3) = -.1
x+2 x2-4 x+2 (x-2)(x+2) 4
lim (x+2)

"
x+2
lim
lim x 2-5x+6 lim x-3 (x-3)
(y) x-+-2
x-+•2 x2-4 x-1-2 x+2
lim (x+2)
x+-2
this limit does not exist.
[Note. The limit theorem (iv) can be applied immediately to
li
(a), since � (x 2 -4) f O; in (B) after the factor (x-2)
x+
disappears; and not at all in (y), since lim (x+2) = O.]
x-+- 2

1. Determine the following limits, (use the theorems above)


. (a) lim (x 2 -5x+7)
(i) lim 9
x+2 (b) x+Z (x-1)
2
(c) lim x -Sx
x+2 x-1
- (x-l)(x+2)
(ii) If f(x) - x(x-1) determine the U.mit of
f(x) as (a) x + 3 (b) X + -2 (c) X + 1
(d) X + 0 (e) x +�(divide numerator
and denominator first by x2 )
x 2 -x-2 x 3-s when
2. Find the limit of (a) x2 (B )
-Jx+Z x2-4
(i) X + 1 (ii) X + 0 (iii) X + 2 (iv) X + -2,
lim f(x)-f(c) where f(x) = xS
3. Find (i) x-c
x+c '
lim f(x+h)-f(x) [Note factors of x S-c SJ
(ii) where f(x) = 2x2 -3x-7.
h+O h
(iii) lim 2-� [First rationalise the numeratorl
x-3
x+3

438
CONTINUITY
£, C�TINUlTY
Definition f(x) is continuous at x • a if
lim
� (i) f(a) exists and (ii) x+a f(x) exists
and (iii) lim
x+a
f(x)
'(

lim_ f(x) � f(a) lim f(x)


x-+a .. x+a+
X2 -9
Thus (i) !f f(x) "' x-3' t hen limx+3 f(x) 6,-o---o..""--+
but since f(J) does not exist,
=
')(

then f(x) is not continuous at xc3,


x2 -9
(ii) If f(x) "'-- ,x,,13 Here lim f(x) = 6 = f(3) and
x-3 } x+3
= 6, xc 3 thus f(x) is continuous at
x c 3.
(iii) If f (x) 3x+ lxl, x<- Here lim_{f(x)}• -J+l-11
-8 x+-1 .. _2
=x+5 x=r-13
"'x2 -2x-5 x>-1 limf(-1) "' ....=L
-1+5 =-2
x+-l+ {f(x)}a(-1) -2(-1)-5
C -2
2

and thus f(x) is continuous at x= -1


1, Determine whether the functions given below are contin-
UOUS at X "' 2,
x-2
(i) f (X) = ::-z-;:- (ii) f(x) =x-2
x2_4,x; 2
,. 1,X '"' 2
X -4

4
(iii) f(x) = x-2
2_ , x ,,I 2 (iv) f(x) x 2 -2x
x 4 ,. x2+2x-8
=l x = 2
2'
x 2 -2x
f(x) x2+2x-8' x ,,I 2 (vi) f(x) =x 2 +3, x ! 2
=9-x, X > 2
(v)

=l X = 2
3'
(vii) f(x) =x 3 +3,x < 2 (viii) f(x) - � x; 2
= 6x-l,X > 2 x-2 '
= 1, x = 2
(ix) f(x) 15/ (x +1), X < 2
2x-l, X =2
2

"' lrl'f, X > 2


If f(x) =� x 2 -5x, what value must be assigned to
X - 2X
2. (i)
f(O) for f(x) to be continuous at x = O?
(ii) If f(x) =x 2 -3x, x < l and f(x) is continuous
=ax+b, 1<;<2J everywhere, find the
=x+lxl,x�2 values of a,b.

439
SECANTS AND TANGENTS
f· GRADIENTS OF SECANTS AND TANGENTS
1. (i) In the figure, show Y
that if the abscissae of
P, Q are respectively x
and x+h, then the gradi­
ent of the secant
PQ= f(x+h)-f(x) , and
h
deduce that the gradient
of the tangent
( f(x+h)-f(x) 1.
PT = lim
h-+O l h j X
(ii) Determine correspondin g results when the abscissae
of P, Qare (a) x, c and (8) x, x+6x
2. P, Qare neighbouring points with abscissae 3, 3+h on
the curve y = 2x 2 -5x-2. Use the result above to show
that the gradient of the secant PQis 7+2h, and hence
deduce the gradien t of the tangent at P.
3. The points M, N with abscissae x, c are taken on the
curve y = x 6 . Determine the gradien t of the secant MN,
and hence show that the gradient of the tangent at Mis
6x 5 • [Use factors of x 6 -c 6 ]
4. Determine from first principles (i.e. from the definition
f(x+ -f(x)
f'(x) = ��{ �) 3 the gradient of the tangent at
the point (1,1) to the following curves
(i) y 5-3x-x 2 (ii) y = � (iii) y = &

*(iv) y = x!� [Hint, in (iii) rationalise the n umerator]

,�-�,
5. If (x,y)' (x+6x, y+6y) are neighbouring points on the
curve y = x 3-x 2 +1, find � and E.Y. and hence evaluate
6x dx'
when x = 1, 6x = •01.
6x dx

1· RESULTS FOR DERIVATIVES


1. In the following results, u, v are functions of x; n is
a rational number and k is a constant.
Check them careful I}'.!
n-1
(i) _£ (x ) n
nx (ii)_£ (k) = 0
dx dx
( iii) _£ (ku) k du = du t dv
(iv)_£ (u ± v)
dx dx dx dx dx
_g_ dv du
(v) (uv) u dx + v dx ["Product" Rule]
dx
du ["Function of a Function"
(vi) (a) ! (f(u)) = f'(u)
d dx Rule]
440
DERIVATIVES
d n
(vii) (b) dx (u ) = nu
n-1 du
(c) .!!Y. = .!!Y. £ z .5!.Y.fu
, dx dx dt' dx dx/dt
du dv
v-dx - u- dx
(viii) � (�) = ["Quotient Rule".)
dx v v1

2, Verify the following results for 1;-


(1) y = 2x 7 + .L2 -
5LX
arx
+ · .!!Y_ = 14x 6 -
' dx X
3 X
§ - 7x'
x -7x Q - 1
3 3k 1
(ii) y = = x 2 - 7x
x - ;; .!!Y. = 2x - 21 x\ +;.,.
1
dx 2
(iii) y = (x + 3)(7-Sx);
2
1;- = (x + 3).-S + (7-Sx).2x
2
= -1Sx 2 +14x-15

(iv) y = 3u 2 -9u+8 where u = 2x-5; 1; = (6u-9).(2)=24x-78


(v) y = (Sx -7x+1) ;
2 9
1;- = 9(Sx -7x+1) 8.(10x-7)
2

/zt+3• _gy_ = ';i(2t+ 3) 2,2


1 -!:::
�..-t�r-==
2
(vi) = t + 1 y 1 - 1/t2 = (t2-l)i2t+ 3
X
t' 'dx
(vii) y =
3x-2, .!!Y. = (Sx+7). 3 - (32x-2).5 ____31-'--__,,.2
Sx+7' dx (Sx+7) = (Sx+7)
(viii) y = (Sx-1)3(7x+2) 2 ; .!!Y.
dx = (Sx-1)3.2(7x+2).7
+(7x+2)2. 3(Sx-1)2.S
= (Sx-lf(7x+2)(175x+16)
3. Find 1;-
in each of the following
(a) y = x 3+3x2 -Sx-2 (b) y = 2x % - Sx % + 1
2
(c) y = ; 3 7
- � +�
x2+x-4
(d) Y = -
x--
1
(e) y = (2x+1) 7 (f) y -
(3x2 +1)5
(g y = (3x -2x)
) 2 !,:
2
(h) y = 1 ;:+3
1- 3 x
(x-1)(2- 3 x+x2 )
X 2
(i) Y =
x 2+l (j) Y = ( )
2+4x2 k Y =
(l) y = x 2 (1- 3 x)3 *(m) y =
(1-x)ll+x2
X fl+x
*(n) Y = ll+x2 *(o) Y = I 1-x
(p) y= 2u LSu 2 +7, u= x 2 + 3 (q) x=t2 +3t-1, y = St 3 -8t+9
4. (i) If f(x) = x 2 -Sx- 3, find (a) f I (1) >
(b) the value of x for which f I (X) = 3 and
(c) f(x) when f' (x) = -1
4
(ii) If y = x + :-z-, determine ..!!Y. when x = 3 , and find
X dX
d
the value of x for which� is zero,
(iii) If f(x) = { 3 x-4) lo , find
(a) f'(l) (b) f(x) when f'(x) 0,
5, (i) If u, v are functions of x, obtain simpler
441
QUADRATIC POLYNOMIAL
expressions for
d
(a) u dv
dx - dx (uv) (8)
u dv
+ ..J!.
dx (.Y.)
7 dx V
du
(ii) If u = f(x)/x and v = f'(x), show that x . dx = v-u.

6. (a) For the curve given by y = x 2 +2x+3, obtain the


gradients of the tangents to the curve where
(i) x = 6 (ii) y = 6. Determine the equations of
each of these tangents.
(S) Find the coordinates of the points on y = x 2 +2x+3,
at which the tangent is (i) parallel to the x axis
(ii) inclined at an angle of 135 ° to the x axis
(iii) perpendicular to the line x + By - 1 = 0.
7. (i ) Calculate the gradient of the curve y = -x 3 +4x 2 -3x
at each of the points where it crosses the axis
of· x.
(ii) If the abscissae of .3 points P,Q,R on the curve
y = x2 �+1 are x, x-h, x+h respectively, show that
the chord QR is parallel to the tangent to the
curve at P.
, ax+l
8. (i) At the point where x = 3 on the curve y = , the
x-2
gradient of the tangent is -5, find the value of
the constant a.
(ii) The tangent to the curve y = � + bx at the point
X
(2,9) on it, is parallel to the x axis. Determine
the values of the constants a and b.
(lii) The curves y = x 2 +ax+b, y = cx-x 2 touch each (i.e.
have a common tangent) at the point (1,0). Find
the values of ·the constants a, b and c,

TOPIC 7
THE QUADRATIC POLYNOMIAL!\NQ__ _IHE PARABO��

�. QUADP�TIC EQUATIONS
1, Solution of quadratic equations of form ax 2 +bx+c Oby
·-bi�
(a) factors (8) the formula x =
Za
Determine the roots of (i) 2x 2 -3x-5 = 0 (by the method a)
(ii) 2x 2 -3x-8 = O (in irrational form, by the method 8),
2. (i) If a,B are the roots of ax 2 +bx+c � O, prove that
a+8 = -b/a, at.8 = c/a.
(ii) Assllllling a,O are the roots of 2x 2 -3x-l = O, find
442
QUADRATIC EQUATIONS
(without solving the e uation), the values of
(a) a+B (b) aB (c) a �+B 2
[Use identity a 2 +B 2 = ,a+B) 2 -2aB)
-1 -
(d) a +13 l (e) � +
°
,h- (f) �+ Ji (g) (a-1) (B-1)
a l
(h) (a + l)(13 + -;;1 > '"' '"'l
(i) � + �
13
(j) a 2 B 3+a 3 S 2 (k)(a-B) 2 [Note (a-B) 2=(u+B) 2 -4uB)
t(iii) Repeat (ii) for the equation 3x 2 +4x-2 = O.
3. State the sum and product of the roots of x 2 -px+q = O.
If the roots are k and k+l, find the relation independ­
ent of k which must hold between p and. q. If q = 6, find
the possible values of k.
4. If p,- q are the roots of the equation 3x 2 +7x-k= O, find,
without solving the equation, the value of (i) (p-q) 2 in
terms of k (ii) k for which the roots of the given
equation are equal.
5. SUMMARY
DI SCRIM-
ROOTS PARABOLA y = ax2 +bx+c
INANT
ti > 0 Real, unequal cuts x-axis at
2 d istinct points
,zp 0<o\x
/',.. �

ti = 0 Rea I, equa I

ti < 0 Unrea I,
(complex)
touches the x-axis

does not touch or


cross x-axis
y.·,�
�uG)O
\l

A.
0

Note. The equation = 0 has real roots if


ax 2 +bx+c b 2 -4ac
�· 0
The roots are then said to be rational or irrational
according as to whether b 2 -4ac is or is not a perfect
square.
(i) Show that the equation x 2 -Sx+6 = O has 2 real and
different roots, and find them. Hence (a) state
the zeros of the polynomial x 2 -sx+6
(b) sketch the parabola given by y = x 2 -Sx+6.
(ii) Repeat the ideas of (i) for the equations
(a) x 2 -6x+l0 = 0 (B) x 2 -6x+9 = 0 (y) S-2x-x 2 = 0
6. Without solving the equations, discriminate between the
nature of the roots of (a) x 2 -4x-3 = 0
(b) x 2 -4x+3 = 0 (c) x 2 -4x+4 = 0 (d) x 2 -4x+S � 0

7. For what values of k, will the equation 3x2 -4x-(l+k) 0


have (i) equal roots (ii) real and different roots
(iii) real roots (iv) unreal roots.
443
QUADRATIC EXPRESSIONS
8, Find the values of A for which the equation
(5A-4)x 2 -6x+(6A+3) • 0 has equal roots. Determine these
roots for each value of A,
9. Show that the roots of the equation
(i) (a-2b)x 2 -2(a+b)x+(a+4b) • O are rational
(ii) (x-a)(x-b)-k 2 •O are real. [What is the sig­
nificance of the case when simultaneously a • b
and k = O?)
(iii) 2x 2 -2(a+b)x+(a2 +b 2 ) = 0 are complex, unless a = b
when they are equal.

10. SUMUARY. Prove the following results, for the quadratic


equation ax 2 +bx+c = o; if
(i) one root is zero, then a8 = O, c = 0
(ii) the roots are equal but of opposite sign,
then a+S = O, .•. b = 0
(iii) the roots are reciprocals of one another,
then a8 = 1, .·. c = a
(iv) the roots are the same sign, then a8 > O. [If both
roots are positive, then a+S > 0 and a8 > O; if
both roots are negative, then a+8 < 0 and aS > O]
(v) the roots are of opposite sign, then a8 < 0
(vi) one root is k, then x = k satisfies thtc equation,
• ', ak +bk+c = C
2

(vii) one root is three times the other; let roots be a,


3a and show 4a = -b/a, 3a 2 = c/a
and thus 3b 2 = 16ac.
11. For what value(s) of A does the equation
(H3)x 2 -(.3H7)x+3(2Hl) = O have (i) the sum of the
roots eqtial to 2 (ii) the product of the roots is equal
to twice the sum of the roots (iii) one root equal to
zero (iv) one root equal to unity (v) equal roots
(vi) roots equal but of opposite sign (vii) roots which
are reciprocals of one another.

�· QUADRATIC EXPRESSIONS
1. (i) Show that x2-6x+l4 = (x-3)2 +5, and that the rnini­
value of the expression i2- -6x+l4 is 5, when
mUl'l
X = 3,
9 +- 9
(ii) Prove that 7-3x-x2 = 7-(i2-+3x +-)
4 4
= .1l _ (x + 1)2
4 2 '
and that the.maximum value of the expression
7-3x-x 2 is 37/4 when x = -3/2.
2. Prove that ax 2+bx+c = a[ (x + Za b) 2 ] + 4ac-b 2 If a ) O,
� · ·
Liac-b 2
show that the least value of ax2 +bx+c is ---;;;;:--, when
-b show that the greatest value of
= 2a' If a < 0
X
4 44
POSITIVE DEFINITE
2
4ac-b -b
ax 2+bx+c 1..q �· when x = Za.
3. Determine the maxim1.m1 or minim1.m1 value of the following
quadratic expressions, and ti1e corresponding value of x.
(i) x 2 -4x+9 (ii) 2x 2 +10x-3 (iii) S-6x-x 2 (iv) Z-3x-3x 2
i
��
:_� _ ><
4. The expression ax 2 +bx+c is + + • -
(ex) (i) positive definite if a > O + · ++__-- --.
and b 2 -4ac < O +
0 >(

JP'(''-..:::./�
(ii) negative definite if a<O and b 2 -4ac<O
(iii) indefinite if b 2 -4ac > O
(B) the same sign as "a" for al1 real values of x if
b 2 � 4ac; and also for all real values of x except
those lying between the roots of ax 2 +bx+c = O if
b 2 > 4ac.
(y) a Derfect square if both a,c are perfect squares and
b 2'...4a� = O. ·
S. Which of the following expressions are positive definite
(i) 3+2x-x 2 (ii) 5-3x+x 2 (iii) 4+8x+3x 2 (iv) (x-1)(x+2)
6. Classify the following into the categories (positive de­
finite, negative definite, indefinite)
(i) x2 -5x-3 (ii) x 2 -Sx+7 (iii) 4-x-x 2 (iv) 4x-5-x 2
7. Use a sketch to find the range of values of x for which
(i) x(x+2) 5. 0 (ii) (2x-l)(x-3)>0 (iii) (3-2x)(4+x)�O
8. !·: rite down a quadratic expression in x which is
(i) negative for l<x<2 (ii) negative for x>3 or x<-5
9. Use a sketch to find the range of values of x for which
(i) (x+2) (x-1)(x-3) � 0 (ii) (1-x)(x-2) 2 < 0
(iii) (x+2) (x+l)(x-1)(x-2) > 0

10. Find the range of values of k for which the expressions


below are positive definite
(i) x 2 -2x+(3-2k) (ii) 2x 2 -Skx+2 (iii) x 2 -(Sk+l)x+9k 2
11. Find the values of ). for which the expressions below are
perfect squares
(i) x 2 +x(>.-l)-(2>.+l) (ii) (2>.-S)x 2 -6x+3>.
1
12. Find k so that the graph of the paral:-ola y 3x +4x+l-k
may cut the x axis.
13. For what range of values of k will the expression
kx 2 -(sk+l)x+9k be
(i) positive definite (ii) negative definite
(iii) a perfect square.
14. Determine the range of values of k for which the
equation (i) x 2 +(3k-7)x+(2k+6) = O has real roots
(ii) x 2 -x+l = k(x 2 +x+l) has real and different roots
445
IDENTITIES, v METHOD
2 +(3+k)x+k(3+k) = O has unreal roots.
(iii) x

15, Prove that the roots of the equations below are always
real and different, for real values of k
(i) kx 2 -(3k+2)x+(2k+3) • o (ii) x2 -2kx+(3k-5) = O
16. Find the range of values of p for which the roots of the
equation x 2 -px+4 �O are (i) real
(ii) real and positive (iii) real, positive and in the
ratio 3 : 1.

�; IDENTITY OF QUADRATIC EXPRESSIONS


1, Theorem If a quadratic equation is satisfied by more
than two different values of the variable, then it is
satisfied by all real values of the variable.
Show that the following equations are true for all
values of x, [First prove that .there are more than 2
values of x which satisfy each equation,]
x (x-a)
+ (x-b) .. x
x
(i) b-a a-b
x-a)(x-b ) x-b)(x-c ) x-c)(x-a) ,
ii
( )
( + ( + ( 1
(c-a)(c-b) (a-b)(a-c) (b-c}(b-a)
2. If two quadratic expressions are equal for more than
2 values of the variable, then the coefficients of like
powers of the two expressions are equal,
Find the constants A,B,C such that
(a) 2x2 -9x+l4 = Ax(x-l)+Bx+C
(8) 2x2 �9x+l4 = A(x-l)(x-2) + B(x-l)+C
3. (i) Express x2 in the forms
(a) A(x-1)2+B(x-l)+c (S) A(x-1)2 +B(x-2)2 +C(x-3)2
*(ii) Calculate the values of a,b,c,d in order that
4n 3-n 2 +3 = a_n(n+l)(n+2) + bn(n+l)+cn+d
4. If f(x) = ax 2 +bx+c, and f(l) = 8, f(-2) = 11, f(O) 1,
find f(x). What value of x will make f(x) = 2?

Q,, EQUATIONS REDUCIBLE TO QUADRATICS


Use the "v" method to solve the following equations
(i) xb +26x 3-27 = 0 (ii) x 4+48 = 19x2
(iii) (2x 2-x)2 -9(2x2 -x)+18 = 0
(iv) (x + l) 2 + 2(x + l) "' 8 (v) x2 +3x-2 = �
X X X +3x
(vi) t+3) - 10 (x -l ) = 3
x -1 x+3
E. THE PARABOLA x 2 = 4ay
1. Definition A point P(x ,y) moves so that it is equidist­
ant from the point S(O,a) and from the line y = -a.
2 = 4ay.
Show that the locus of P is the parabola x
Prove that the length of the latus rectum is 4a units.
2. Determine the equation of the parabola, ·with vertex the
origin, axis the line x = O, and (i) with latus rectum
446
PARABOLA
of length 12 units (2 parabolas) (ii) with focus at
(0,-6) (iii) through the point (6,-2)
*(iv) such that x + y + 2 a O is a tangent to it,
y •
3. The general form of the equation of a
parabola with vertex (h, k), axis
parallel to the y axis, and focal
length "a" units is (x-h)2 • t4a(y-k).
Use this result to find the equation
of the parabola, with
(i) vertex (4,2), focus (4,8)
(ii) vertex (-2,3), directrix y
(iii) vertex (-1,-5) and_passing
4. Find the coordinates of the focus S and vertex V, the
equations of the ·axis (n) and directrix (d), and the
length (l) of the latus rectum
(i) x2 = Sy (ii) y = _ l x 2
6
(iii) x -2y+8x+l0 = O
2
•(iv) 3x2 +8y-12x = 4

5. Show that the point (2at, at 2 ) lies on the parabola


x 2 � 4ay for all values of the �arameter t. Sketch the
curve given by x = 2at, y = at for varying values of t,
marking clearly the points where t=O, t=t\, t= tl, t=t2.
6. The point P has coordinates x = t+3, y = t2 -l. Eliminate
t and find the cartesian equation of the locus of P. De­
termine the coordinates of the vertex V and focus S of
this locus.
7. The coordinates of a point P are given by
(i) X = p+q, y = \(p 2 +q 2 )
(ii) X = �pq(p+q), y = p7. +q 2 +pq+2
Determine the cartesian equation of the locus of P, if
in each case, pq = 2,
[Hint, use the result p2 +q 2 = (p+q)2 - 2pq.J
8. Find the coordinates of the point of intersection of the
two lines given respectively by
5 u, y = 2-u,
X = - \ + 3t, y = 1+2t and X = 1 +
4
9. Pr>ove, from first principles, each
of the following results for the·
parabola x 2 = 4ay.
(i) The chord PQ has
\(p+q) and
equation y-�(p+q)x+apq = 0 )(

(ii) The tangent PT has gradient


p and equation y�px+ap 2 = O;
the tangents at P, Q meet at
[a(p+q),apq]
447
PARABOLA - PROOFS
(iii) The normal PG has gradient - 1 and equation
p
x + py � ap 3
+2ap; the normals at P, Q meet at
[-apq(p+q), a(p2 +q2 +pq+2)]
(iv) rne tangent and normal at P(x1,Y1) �ave equations
xx1 = 2a(y+y1), y - Yl = -2a � (x -x1) respectively.
x1
(v) If PQ is a focal chord, then (a) pq = -1 lB) the
tangents at P,Q intersect at right angles on the
directrix.
(vi) The principle of the parabolic reflector; namely
that PT is equally inclined to PS and the axis of
the parabola Ol' alternatively, that if rp1 is
parallel to the axis, then 'PL and rps' are equally
inclined to the tangent PT.
(vii) The line y= mx+c is a tangent to x=4ay
2 if am=-c
2

(viii) The chord of contact from (xo,Yo) to x 2 = 4ay has


equation xxo= 2a(y+yo)

10. For the parabola x 2 = 4y, find the equation of the


(i) two tangents through the point (5,4)
(ii) three normals tr.rough ti1e point (6,9)
[Hint: show that p 3 -7p-6= (p+l)(p+2) (p-3) J
11. Find the equations of the tangents and normals to the
parabola x 2 = 16y at the points (16,16), (- 4,1). The
tangents intersect at the point T and the normals inter­
sect at R. Prove that fae line TR is parallel to the
axis of the parabola •
12. . Find the equation of the normal at P(2,l) on t:1e para­
bola x 2 = 4y. This normal meets t!ie y axis in ·G and :{ is
the midpoint of �d'. A line through M parallel to the
x axis meets the y axis in N and the parabola in Q.
Prove l*QNI= �PG�

13. P is any point (2ap,ap2 ) on the parabola x 2 = 4ay whose


focus is Sand vertex O. 'l>N� is perpendicular to the axis
of the para!iola meeting it at :•1, The tangent and normal
at P meet the axis at T, G respectively. Prove that
(i) *TO= *ON (ii) *NG= 2a (iii) *PQ2 = *ON.*SP
where Q is the point of intersection of the tangents at
P, O.
2
14. I! P(2ap,ap ), Q(2aq,aq2 ), R(2ar,ar2 ) are points on
x = 4ay; if the tangent at P is parallel to the chord
GR, sbow that q + r= 2p. If }! £ 'RQ' such that *RM= *l-;Q,
prove that PH is parallel to the axis of the parabola,
and that the tangents at �, R meet on 't1P.
15. In the figure, PQ is a focal chord; PU, QT, RT are tan-
gents; prove
(i) pq= -1 (ii) *PL , *QM constant
448
PARABOLA - LOCUS
(iii) PK passes through 0
(iv) *USK., 90 °
(v) ST is parallel to the
directrix,
16. Shew that the chord of con-
tact of the tangents from
any point (T say) on the
directrix of the parabola
x 2 = 4ay, passes througL
the focus s. K
*If this chord also passes through the point
prove t:1at the chord of contact of tangents
passes through T. [Uint: let T be (x0,-a)J

17. The tangent at the point P(2ap,ap 2 ) on the parabola


x 2 = 4ay with vertex O meets the X axis in R and the
y axis in T.
(i) ORQT is a rectangle, find the locus of Q,
(ii) If }1 lies on •"pR' so that *RM = *HP, determine the
locus of 11.
(iii) The normal at P meets the axis of the uarabola in
G, If N is the midpoint of '"pc', show the locus of
N is a parabola and state the coordinates of the
focus and of the vertex.
(iv) On the ray GP a point U is taken so that *GP= *PU.
Prove that the locus of l' is the parabola
x 2 = 16a(y+2a) ,
18. Perpendiculars are drawn from the vertex O and focus S
of tlie parabola x 2 = By to t'.1e tangent at the varia::ile
point T(4t,2t 2 ). If Q, R are the feet of these perpen­
diculars, find the locus of R, and show that the
equation of the locus of Q is y(y 2 +x 2 )+2x 2 = 0,
19, The points T1(2at 1·,at/), T 2 (2at2,at 2 2 ) lie on the para­
bola x 2 = 4ay. If the chord T1 T 2 subte�ds a right
angle at the vertex O, prove that t1t 2 = -4, Find the
locus of
{i) the point of intersection (R) of the tangents at
T 1, T 2 ,
(ii) the raidpoint (N) of the chord T1T ,.
(iii) the point of intersection (N) of the normals at
T1,T 2 ,
20. The normals to the parabola x 2 = 4ay at the points P, Q
intersect at :.;. If the chord PQ varies so as to always
pass through the point (0, -2a); si1ow that N lies on the
parabola.
21. The tangents at P1,P 2 on the parabola x 2 = 4ay meet at a
point on the parabola ,x 2 = -4ay. Show that the locus of
the midpoint of the chord P 1 P 2 is the parabola 3x 2= 4ay.

449
LOCUS, CURVE, REGION
F. LOCUS, CURVE AND REGION
A, B are the points (4,0), (-4,0) and Pis a variable
point, Find the equations of, and describe geometrically,
the loci of the points P which satisfy the conditions
(1) *PA= *PB (ii) *PA2 +*PB 2 = 64
(iii) tan PAB + tan PBA = 1
2. A is a fixed point, and c a fixed circle, centre E. A
variable line through A intersects the circle at R, S
and M is the midpoint of 1RS.,, Draw figures to illustrate
the locus of M for the 3 possible cases
(i) A is outside c (ii) A is on c (iii) A is inside c
3. On the (x,y) plane are the axes X'OX and Y'OY. A point P
moves so that the shortest distances from P to
(i) the lines X'OX, Y'OY are equal
(ii) the line X'OX and the ray OY are equal
(iii)° the rays OX and CW
Illustrate the locus of P in each case,
4. foR and 10ff are 2 fixed intervals, such that *OB= 2.*0A
and *AOB = 60 ° . A point P moves so that its shortest
distances from roR and roB, are equal. Show on a careful
diagram, the locus of P. [Hint: the locus consists of an
angular region, an interval, a parabolic arc and a ray.)
5. The points P, Q lie on the parabola x 2 = 4ay. The chord
PQ moves parallel to itself, (i.e. the gradient of PQ is
a constant, m say). Prove that the locus of R, the mid­
point oflp1 is a ray parallel to the axis of the para­
bola. Show the exact locus of Rin a sketch.

TOPIC 8
GEOMETRICAL APPLICATIONS OF THE DERIVATIVE
A. SIGN OF f' ( x)
(ii) (iii)

7 � «:'
a
The curve y= f(x) is
��
(i) rising (i.e. increasing) if f'(x) > O,
i.e. if�> 0 [See fig (i))

(ii) falling (i.e. decreasing) if f'(x) < 0,


i.e. if�< O [See fig (ii))
450
GEOMETRICAL INTERPRETATION OF DERIVATIVE
(iii) stationary (i.e. neither rising nor falling) if f'(x)=O
i.e. if .!!Y =o [See fig (iii)].
dx
[t,ote the 4 types of stationary points.)
If _gy = f' (x) has the same sign over t?,e interval a.:Sx.:Sb,
dx
then it is called a monotonio function {i.e. for the interval
[a,b], f(x) is monotonic increasing if f'(x) > 0 (see fig(i))
and f(x) is monotonic decreasing if f'(x) < 0 (see fig(ii))}

1. For �ach o[· tlle following, find whether the function is


increasing, decreasing or stationary at the points given
(a) f(x) x 2 -7x+3 (x = 1) (b) f(x) = 3x - .!. (x = -2)
(c) f(x) (3x-1) 8.(x = f> X

2. Determine whether each of the following functions are


monotonic, for the intervals stated
1
(a) f(x) x 3 -12x-5 (1,5) (b) f(x) = ' (0,3)
x+l
2x-3
(c) f(x) =
x+l' (-7,-2)
3. (i) Fine for what values of x, the function
f(x) = 3x 2 -6x+2 is
(a) monotonic increasing (b) mor,'.ltonic decreasing
3 3x2 .s!Y
(ii) If y = x - --i- -6x+S, show that = 3(x-2)(x+l),
dx
and by considering the sketch of
d><
3(x-2)(x+l) shown, state
set of values of x for which
(a) E:Y
dx
> O
dx
(c) _gy = 0 · -"T-"�--t---1:,-�
(b) .£Y. < O
dx
Hence determine the values of x
for which (a) y is rising
(;3) y is falling (y) y is statior.ary. Hence sketch
the curve y = x 3 - l x 2 - 6x + S.
2
Prove that x 2 + x + 1 = (x + ) 2 + 3. :.rhat can you
1
4. (i)
2 4
say about the sign of x 2 +x+l? Hence show that the
curve y = 2x 3+3x2 +6x+4 is monotonic increasing for
au values of x. . ft•)

(ii) Show that if f(x) = x -8x +10, .\ (-ior(/2 �
4 2 1
then f' (x) = 4x (x-2) (x+2), and I '--"'
uy considering the sketch of 4x(x-2)(x+2), find
for what values of x, f(x) is
(a) rising (b) falling (c) stationary.
Use these facts to give a sketch of y = f(x).
s. For f(x) = x+4x-1, show that f'(x) = (x-2)(x+2)/x 2 , and
hence determine whether f(x) is monotonic increasing or
decreasing in the intervals (i) -co < x < -2
(ii) -2 < x < O (iii) O < x < 2 (iv) 2 <.x < co
Evaluate f(-2), f(2), and state the value of x for which
451
SIGN OF f"(x)
f(x) is discontinuous. Renee sketchy= f(x).
6, Sketch a continuous curve y= f(x) which satisfies the
following data
f'(x) <0 for x <3; f'(3) O; f'(x) > 0 for 3 < x<4;
f'(4) O; f'(x)>Ofor4 <x <5; f'(5)=0
f'(x) <0 for x > 5.
y
8. SIGN OF f"(x)
1. fo fig(i), the gradient
f'(x) is decreasing as
x increases.
d
Thus [f'(x)] < O for
dx
arc POR, i. e, f" (x) <0 ""o-+---------------+
then. Determine a similar result for the arc ABC.
Hence the curve y = f(x) is
(1) cqncave do1,.,-n if f"(x) <o, i.e. if �:} <. 0
[See fig(i)]
(ii) concave up �
if f"(x) > o, i.e. if � �> 0
y
[See fig(ii)]
z. In the sketches, L,�
are points at which
(i) the curvature
of the curve-+-,........-----------------a,x
0
y = f(x) changes from concave down to concave up,
(at L) and vice versa (at u).
(ii) f"(x) changes sign from negative to !JOSitive
(at L) or vice versa (at 10
d2y
(iii) f"(x) = 0 i.e. � = 0
(iv) the curve crosses the tangent at the point.
L, M .are called points of inflexion, and the tangents at
L,M are called infZexionaZ tangeP.ts.
3. (i) Sketch the curves f(x) = (x-1)3, f(x) = (x-1)4 on
separate diagrams. Show that· for each case,
f"(x) = 0 at x = 1, but that f"(x) does not change
sign in passing throogh x = 1 for one case. On
which curve is there an inflexion at x = l? Is the
fact that f"(x) = 0 for a certain value of x suf­
ficient condition for us to claim that there is a
point of inflexion there?
(ii) State the precise conditions for y f(x) to have
a point of inflexion at x = a.
4. Sketch examples of points which are
(i) stationary points but not points of inflexion
(ii) vice versa (iii) stationary points of inflexion.
452
STATIONARY POINTS; POINTS OF INFLEXTION
�. STATIONARY POINTS AND POINTS OF li�FLEXION

E
B

0 0.
1. In the above figure, i.e. y = f(x) defined for a� x � b
select the (a) turning points (i.e. extrema)
(b) stationary points (c) relative maxima or relative
minima (d) absolute maximum and absolute minimum points
(e) stationary points which are not maximum or minimum
turning points (f) points at which f'(x) does not exist
(g) critical points (h) points of inflexion (i) points
of inflexion which are not ·stationary points, and
(j) vice versa.

V1t\.
2. The diagram y
illustrates
the 4 poss­
ible types
of station-
ary points.
Verify the L
c o nditions 0 ><, )(

given below. For a


(i) Minimum turning point at x = x1

f'(x1 ) = 0 and f'(x1-) < 0, f'(x1+) > 0


o:r> f' (x1) = 0 and f" (x1) > 0
(ii) �aximum turning point at x = x2
f'(x2) = O and f'(xz-) > O, f'(x2+) < O
o:r> f'(x 2) = O and f"(x 2) < O
(iii) �orizontal point of inflexion at x = x3

f'(x3) = 0 and f' (x3-) > O, f'(x3+) > 0


or f' (x3) = 0 and f"(x3-) < O, f"(x3) = O, f"(x/) > 0
There is a airr:ila:r> result for the horizontal inflexion
at X = X1+,
3. The die.gram illustrates the two possible (non-horizont­
al) points of inflexion. Verify the conditions given
below.
453
STATIONARY POINTS; INFLEXTIONS
For a point of inflexion

'.y
at
(i) X • XS, f 11(x5) "' 0

A
M , T/\

J I
and f11(x5-) < O,
\_A
I s I
f"(x5+) > 0
(ii) X • XG, f"(XG) • 0 j�R I
and f"(x6-)> O, I
f"(x 6+) < 0 0 ><, )(6 X
Le. in general, there is a point of. inflexion at x • a if
f"(a) • 0 and f" (x) changes sign in passing through x=a.

Q. EXERCISES ON STATIONARY. POINTS,


INFLEXIONS ETC., CURVE SKETCHING
1. (i) Use calculus to find the stationary point on the
parabola y • x 2 -6x+5, and show that the curve is
concave up for all values of x.
(ii) For the parabola y = ax 2 +bx+c, prove by the cal­
culus, that the coordinates of the turning point
are (-b' �4ac-b 2 ), Show that this is a maximum if
2a
a< 0 and that it is a minimum if a> O. Prove
that there is no point of inflexion on the curve.
2. If y = 24x+3x 2 -x 3 ' prove that .!!Y = 3(4-x)(2+x)
dx
.c&
dx2" .. 6(1-x). Find the coordinates of the maximum and
minimum turning points, and the point of inflexion on
the curve. Use this information to sketch the curve.
(i) Verify that y has a maximum value 80 when x = 4.
When x -5, y again has the value 80, Explain
this.
(ii) How many real roots has the equation 24x+3x2 -x 3 =0?
(iii) For what values o.f x is the curve concave upwards?
(iv) Obtain the equation of the inflexional tangent.
Show that the cubic equation giving the abscissae
of the points of intersection of this tangent and
the curve has 3 real and equal roots.
3. Find the relative maximum and minimum values of the
function y = %<4x 3 -3x-l). Sketch the graph of this
function, and find the greatest and least values of y
for values of x in the rang·e -1•5 to 1•5 inclusive.
4. If f(x) = 3x 4 -4x 3 +1, show that f'(x) = 12x 2 (x-l),
f"(x) = 12x(3x-2), Determine the coordinates of the
stationary and inflexion points on the curve y = f(x)
and hence sketch the curve. Show that the curve y = f(x)
is concave down only if O < x< 2/3, and that it is
rising only if x > 1.
5. Find the 3 values of x for which the value of y=x 3(x-1) 2
is stationary. Without determining d 2 y/dx 2 , distinguish
454
STATIONARY POINTS, INFLEXIONS
between these stationary points, and sketch the curve.
6. Each of the following functions is defined over the in-
terval -1 � x � 3. Find the great�st and least values
of this interval. [Hint: draw a sketch.)
(i) f(x) • x-2 (ii) f(x) • jx-2j (iii) f(x) • � 2 -4
*7. (i) Given that f (x) = 4/x + fx,what is the domain
of x? Show that there is only one stationary point
and tha·t it is a minimum. Sketch the graph of
y = f(x).
1/ 2 dv
(ii) Show that if y = 32x 3 - 1 x , then � = O when
6
x o/3 = 32 and hence find the stationary point on
the curve.
8. Show that there are no turning points and no points of
'
x-1
inflexion on the curve Y = x+i • At what points does the
curve cut the axes? For what value of x is y discontin­
uous? What is the limit of y as x � 00? Sketch the
curve, and hence determine the greatest and least values
of y in the interval -�� x � 2.
1
9. The curve y =
x(x-l)
is discontinuous for 2 values of x;
state their values. Show that there is only one station­
ary point, and determine its nature. Sketch the curve.
3x -3( x-2�(x+2)
*10. If Y = prove that .!h'.. = and
(x-l)(x- 4) ' dx (x-1) (x-4)2
hence find the relative maximum and minimum values of y,
discriminating between them. Sketch the curve.
*11. If y = Z 'prove that
X
x +l
.!h'.. _ -(x-1) (x+l) fi - 2x(x:l3) �x+/3)
- Hence find the
dx - (x:t+l)2 ' � (x2+1)
stationary points and points of inflexion of the curve.
Prove that the graph lies entirely within the region
bounded by the lines y = ± �- Show that the curve is
continuous for all values of x, and find the li�it of y
as x � ±00• Sketch the curve.

12. (a) If y = x 2 + l determine the value of b if


x'
there is a (i) relative minimum when x = 1
(ii) an inflexion at x = 1.
ax+b
(B) The curve y = x-l has gradient equal to -5 at the
point (2,8) on it. Find the values of the con­
stants a, b.
13. A curve y = x 3 +Ax 2 +Ex+C
(i) passes through the point (0,1) and has stationary
values at the points where x = 2 and x = -2.
(ii) has a relative maximum at (-1,5) and a relative
4 55
PRACTICAL PROBLEMS
minimum at x = 3.
(iii) has a relative minimum at x 4 and a point of in­
Hexion at the point (1,0). Find the values of
A, 3, C in each case,
*14. Calculate the coefficients a, b, c, d so that the curve
y = ax 3+bx2 +cx+d has a. maximum turnir.� point at (-1, 10)
and a point of inflexion at (1,-6).

15. Sketch the ti)ree continuous curves y = f(x) having the


following cha·tacteris tics:-
In each case, f(-3) = 10, f(O) = 4, f(3) = 2,
f'(-3) = f'(3) = o.
fur>t.�er, in case (i): f I (X) < 0 for !xi < 3, f I (x) > 0 for
!xi > 3, f"(x) < 0 for X < 0,
f" (x) > 0 for X > o.
i1hi lst in case (ii): f '(x) < 0 for lxl < 3 and for X > 3,
f'(x) > 0 for x < -3 f"(x) < 0 for
X < 0 and for X > 3, f" (x) > 0 for
0 < X < 3.
and in case (iii): f'(x) < 0 for all x except x ±3,
f"(x) < 0 for -3 < x < 0 and for x > 3;
f"(x) > 0 for x < -3 and for O < x < 3

E. PRACTICAL APPL! CATIO�JS CF �lAX I �1A AND MIN I MA


1. Find a quantity x which, when added to four times the
square of its reciprocal, gives a minimum sum. If x is
restricted to the interval 1 5 x 5 5, find the greatest
sum possible.
2. A rectangular field is bounded on one side by a straight
river and on the other three sides by a fence whose
total length is 160 m. Show t;1a t the area of the
field cannot exceed 3200 sq, m.
3, A piece of wire, 4 m long, is cut into two parts. One
part is bent into the form of a rectangle of sides x m
and 3x m, and the other part into the form of a square,
Show that the total area, A sq rn, enclosed by the rect­
angle and the square is given by A= 7x 2 -4x+l. Find the
value of x when the area A has the least possible value,
4. Find the !-ieight of t:1e right circular cone of maximum
volume, the sum of the height and radius of the base
being 12 an.
[First prove V = 1
nr 2 (12-r), where rem-is the radius of
the base and V cu tm , the volume of the cone.)
5. T,1e point A is 10 km due North of a town IL One man
starts from A and walks due F.ast at 3km/h. Simultaneous­
ly c>. second nan starts from l3 and wall:s due North at
4kmJh, Shew that, t hours after starting, the distaP-ce
456
PROBLEMS ON MAXIMA, MINIMA
(D km ) between the two men is given by
o 2 = 25t 2 -80t+lOO. If S = D 2 , show that the least value
of S occurs after 8/5 hours, and he-ace find the minimum
distance between the men.

6. A cylinder is open at one end.


(i) If the surface area of the cylinder is 300rr sq�cm,
show that the volume is a maximum when the radius
is lQC:m and find this maximum volume. Determine
the least and greatest volume if 2 Sr � 12.
(ii) If the volume is lOOOrr cu. cm show that the
surface area is a minir.1um when the radius is lQ �m;
and find the minimum surface area. Determine the
least and greatest surface areas if 2 !, r � 12.
7. From a square sheet of cardboard of side
12Cfl\ the corners are cut as shown
(i) If the remaining cardboard is
folded to make an open rectangular
box, prove that its volume
(V cu.,cm ) is given iJy
V = x(l2-2x) 2, and hence find the
values of x for the box of great­
est and least volume. -12c..y.-+
(ii) If the remaining cardboard is folded to form an
open rectangular box, and the corners that were
cut out are put together to make an open cube,
prove that the sum of the volumes (V cuicm) of the
two boxes is given by V = 5x 3-48x2 +144x, and hence
find the values of x for the sum of volumes to be
greatest and least. [IUnt: sketch the graph of V
against x.)

ii ''
8. A (i) A despatch rider is at A in
open country at a distance of 6
km from the nearest point P
of a straight road. lie wishes
6�r ..J3H;;.-- ....._

! ____..____
to proceed as quicl(ly as poss­
ib ZP. to a point Q on the road
___a,,Q
p X """ 8 ( 20 - •) - 20 kffl from P. If his maxi-
..____ :to k mum average speed, across
country, is 40km/h, and along the road, is 5Clkm/h,
find at what distance from P he should strike the
road, and determine the least time for the journey
[If the total time for the journey is T hours,
l36+x2 20-x
prove that T = � + .]
50
(ii) Answer the question of (i) if
(a) *PQ = 8 k.m (b) *PQ 2,km

457
TANGENTS AND NORMALS
r, TNIGENTS AN[' NORiJALS
1, For the curvey = x 3 +1, find the equation of the tangent
to the curve where x = 1, and the equation of the normal
to the curve where y = 0, Deternine the coordinates of
the point of intersection of the tangent and normal,
2. (i) The tangent at the point P(a,a 4 ) on the curve y=x 4
meets the x axis and T, and PN is drawn perpen­
dicular to the x axis meeting it at N, Prove that
*OT = 3. *TN, where O is t!1e origin,
(ii) Find the equation of the normal to the curve
y = zx 2 -12x+S which is parallel to the straight
line x + 4y = O.
3. Find the equation of the tangent to the curve y = 2x + l
X
at the point (li,3) and show that it does not meet the
curve again, Find the coordinates of the point at which
the normal at (1-,,3) meets the curve again.
4. The tangent at the point P 11here x = -1 on the curve
y = 2x+x- 2 8eets they axis in Q, Find the coordinates
of�. Prove that, if R is the middle point of 'o1,
0 being the origin, the gradient of PR is 5/2.
s. If the point P(xo,Yo) lies on the curve y = x 3 -3x, show
ti1at the equation of the tangent to the curve at P is
y - (xo 3 -3xo) = (3xa 2 -3) (x-xo) and hence find the co­
ordinates of P if this tangent passes through the point
�(0,-16),
6. Find the equation of the tangent toy = ax 2 at the point
where x � 2, and the equation of ·the tangent to
y-9 = b(x-6) 2 at the point where x = 5, Calculate the
values of a and� if these straight lines are coincident.
y
*7. The diagram shows the tangent I
PT at the point P(x,y) to the-+- o.

curve y = ax·
P.rove that
��x
(i) *TN = �
X
(ii) a.*TO *ON2
(iii) *area /J. PNT = i:i, *ON 2 ,

PRP'ilTIVE FUMCTIO'.IS A�ID THEIR


GF:OMETRICAL INTERPRETATIONS

If ...J! F(x)
dx
= f(x), then F(x) is called a primitive
fw�ction of f(x). Thus ...J!lx 3 -4 x 2 +7x-91 = 3x2 -8x+7, and hence
dx
a primitive function of 3x 2 -8x+7 is x 3 -4x2 +7x-9.
5
4 8
PRIMITIVE FUNCTIONS
d d
"te thc>.t
.,o f F(x)+C1J
- = f(x) = dx -\ F(x)+C2I, where C1, c 2
dx
are constants, and thus the primitive funotion of f(x) is not
unique. Each of the possible primitive functions vary by a
constant.
CeometriaalZy, if f = 3x2 -ax+7, i.e. the gradient .:-.t any
point (x,y) on the curve is known, then the primitive
funotion rerresents a family of ourves of equation
y = x 3 -4x 2 +7x+C, for different values of C.
1. Write down the primitive functions F(x) such that
a: (Ffx)I f(x{, where f(x) is equal to
=

-
(a) xh (b) x h (c) x 7 (d) x-7 (e) 1 (f) x 2 -Sx-7
(g) 8x -7x +3x
5 4
(h) x-3 + j - -;fr
(i) >fx + X % - 7x
x -7x +3
4 2
(j) (x-2) (x+3) (k)
X
2. The slope at any point (x,y) on a farr.ily of curves, is
given bv
. ..QY. 2x-4. Fxplain why the family has equations
=
dX
of the form y = x 2 -4x+C, where C varies. Determine the
value of C, for each of the cases below, and hence
sketch the curve. The curve
(i) passes through the origin
(ii) touches the x axis where x 2
(iii) crosses the y axis at y = 3
(iv) has a minimum value of 5.
3. Find the equation of the curve y f(x) in each of the
following cases
(i) f' (x) = 1+3x 2 and it passes through the point
(2,11). Show that it passes through the
point (0,1). ·
(ii) f'(x) = kx 2 , where k is a ccnstant, and it passes
through the points (1,8) and (-1,6).
(iii) f'(x) = ax+b, where a and b are constants; the
curve has a stationary point c>t (2,-4) and passes
through the orilin.
*(iv) f'(x) = x - 1/x , and the normal to the curve at
the point whose abscissa is -1 passes through the
origin.
4. (i) At a point (x,y) on a curve, the product of the
slope of the curve and the square of the abscissa
of the point is 2. If the curve passes through the
point ( 1,-1) , find the ordinate when x = 5.
(ii) If ..QY = kx+2 ' where k is a constant, express y as
dx
a function of x, if when x= O, y=4 and� fi = 6.
(iii) Find the value of the consta,1t k, given that
� .91. = k-x, and that y=O when x=2 and when x=6,
3
dx
459
AREAS BY SUMMATION
d2
s. (i) Given tha-t � = 2+3x, find y when x = 1, if it is
known that when x = 1, � = 10, and when x= O, y = S,

(ii) If:��= 10+ 6x-3, and when x = l, � = 5 and y = lS,


find y in terms of x, and determine the value of Y
when x = 3.
6. (i) If f"(x) =
6x-6, find f(x) if f(l) = 1 and f(2)= 3.
(ii) For a certain curve,
d2 =
ax
.g.
6x-4, and y has a mini­
rnum value 5 when x = 1. Find the e(!uat ion of the
curve, and the rnaximum turning value of y.
*7. For a certain curve, y = f(x), fi
the concavit y� is de-
termined by the result � b = k
7;• where k is consta nt.
If the curve passes through the points (1,12), (4, 64 ),
d�_
{O,O), find y, E.Y.' � when x = 9. Interpret these re-
dx
sults geoLletrically.

TOPIC 9

1
INTEGRATI O N

A. OETERl/ilNHJG AREAS BY SUl.1MATlm,


1

1. :::i:::: � �:\�::::::� the area y


�etween the curve y = f(x)= x 2 ,
the x axis and the ordinates
at x= O and x = 3.
U·le � the results
1 2 +2 2 +3 2 + .•.+n 2 = % n(n+1) ·(2n+l), -0.J-""'11!16£:::��-��-
,,
"1,,
""',--_
x

and thus 1 2 +2 2 +3 2 +..• +(n-1) 2 = % (n-l)n (2n-l)


1 2 +3 2 +S 2 +... +(2n-1) 2 = 3 n(2n-1)(2n+l�
Vivide tile interval [o,31 into n equal su:>dlvisions, each of
3 -o =
=
1-1idth h, wt.ere h l Construct ordinates at x=h, x = 2h,
n n
x= 3h, •.. , x= nh.
:.:etrwd 1. Using insa'l'ibed rectangles as indicated, prove that
the su�,(s ), of their areas is given by
n
2
s h[f(h)+f{2h)+f(3h)+... +f(n-1 h)l = h[h 2 + 4h 2 +9h +... _
n -
+(n-1) 2 h 2 J
3 3 · 12 +2 2 +3 2 +...+(n-1) .
1
21 27 1
(�) =
6
-;:;"!. (n-1) (n) (2n-
1)

4 60
AREAS BY SUMMATION
_ .2.
- 2
(n-1) (2n-1)
!l =
9
Cl_ l)(l)(2 _ l)
n n n n 2 n
lim 9 .
Hence
· show s
n-+"' n

Method 2. Using airawnscribed rect-


angles, prove that the sum,
(S ), of their areas is given
n
by
S h [f(h)+f(2h)+f(3h)+., .+f(nh)]
n
= f (1 + 1)(1)( 2 + l), proceed-
n n
. 1� x
ing as in method 1, and thus ->«> S =9 ->-tl"""LUJ<JJ.Z,L.L..A,--'"-.a.....-__;;
n n

Method 3. Using rectangles,each of


height the value of the mid­
ordinate of each strip.
Show their sum, (A ), of areas, is
n --
given by
h
A =h [�<t>+f( �)+f( �)+ .. +f( -� ) l
3 5 2n
n
= * [12 +3 2 +5 2 +, .. +(2n-1) 2 ]
27 n
=
4n 3

3 (2n-1)(2n+l)
= .2. (2 - 1)(2 + .1)
4 .n n'
lim
and thus A = 9.
n->«> n

Method 4. Using trapezia. Show that


their sum>(B), of areas, is
n

t
given by
I

[{f(O)+f(h)} +(f(h)+f( 2h)}


I
B = I
n I

+{f( 2h)+f(3h)}+ •.+{ f(n-1 h)+f(nh) }]


I

= � [ 2 (1 2 +2 2 +32+ ..+n-12)+n2 ]
3 I

27 1 2
[ 2.
=
2n 3 6( n -l)(n)(2n-l)+n)

9
= _ (1 - .1).1. ( 2 - _l ) + ll and
2 n n 2n' "
1.im .
thus B = 9
n-+"" n 0

,'1ethod 5. Using the definite integral. The required area A


is given by
A = (x 2 dx
0
[ff 0
= 9.

Note that although s , S , A , B , A


n
are unequal, and in
n n n
fact s < A < A < B < S , when the number n of strips is
n n n n
461
MID-ORDINATE RULE; SIMPSON'S RULE
increased without limit, then
3
lim
n- n
s c
lim S
n- n
= lim
n- n
A lim B
n- n
=A= f x2dx , and the re-
quired area is 9 sq. units. O

2. Repeat question 1, using the five methods illustrated,


for the area bounded by the curve y = f(x), the x axis
and the ordinat�s at x = O, x = 2 for the cases
(i) y =x (ii) y = x 3 ·[Here omit method 3,)
[You may assume the results
n n-1
1+ 2 +3+.•• +n = 2 (n+l) and thus 1+2 +3+ ... +(n-1) = �2� (n)
n
1 3+2 3+3 3+.•.+n 3 = 4 (n+1) 2 and thus 1 3+2 3+3 3+,,,+(n-1) 3
2

(n-1)2

-
= 4
B. APPROXIMATION METHODS
MID-ORDINATE RULE AND SIMPSON'S RULE
1. Problem: 5To find the approximate value of the definite
integral I 2 dx, i.e. to determine an approximation to
x

-1
the area bounded by the curve y = 2 X , the x axis and the
ordinates x = -1, x = 5. [The method will be illustrated
hy using.§. stri.ps, The diagram is not drawn to scale.]
ta) Mid-Ordinate Method
The ordinates at x = -1, x = 0,
x = 1, ... , x = 5 give the 6
strips, and the mid-ordinates
occur in each strip at
1 X = 2'1 3
X = - 2' X = 2'',, X =.2.
2
Show that the sum, (An), of the
areas of the rectangles (as .,11.1.<uu..��'4LL.Jf..l.-4L''-¥.l-'!--�Ll-<U¥-�
shaded) is given by
_!,, !- 3/ 9/ 1
1[2 2+2 2+2 2+., .+2 2) =[72 + fi + /a+ {32 +..+ ,51 2)
r,:-::-;;
An
= ·707 + 1·414 + 2 ·828 + 5'657 + 11·31 + 22 ·63,
44·546 from square root tables
(8) Simpson's Rule
that the approx imation
to the area, by Simpson's
Rule, is given by
h
Hn = 3[�o+Y6)+4(y1+y3+y5)
+2(y2+Y1+> J
= 1: 5
(2 -1+ 2 )+4(20+ 2 2+2'+)
3
+2 ( 2 1+2 3 )]
)(

= 45·5
462
MID-ORDINATE RULE; SIMPSON'S RULE
5
[i!:e21
5

Note (i) the actual area• f 2x dx m since


-I J... �(2x
)= 2 log e2
x
dx
• 31·5
':' •6931
+ .il=.M. s�. units

(ii) Simpson'.s Rule gives a much better approximation


than the Mid-Ordinate Rule. The Mid-Ordinate Rule
may b e used for any numb er of sub-divisions b ut Slnps­
on's Rule may only be applied for an even numb er of
sub -divisions.
-x
2. The tables b elow give the values_ of y = �(ex - e ) fo·c
differing x .
•25 •75 I •25 'l•f5
1;1g1 :;21 I/· 175,�:�29,;·6271 ·253 •822 I •602 2••7 0
X

Hence evaluate 1 i
-x
�(e - e ) dx approx imately, using 4
x

strips, b y (i) S�pson's Rule (ii) the Mid-Ordinate Rule


Find the actual value of the definite integral b y direct
integration, and evaluate it. using tables,
3. Sketch the graph of y = .fi for 2 .S x .S 6, Hence find

approY.imations to f rx dx, using 4 strips, b y


2
(a) Simpson's Rule ( b) the Mid-Ordinate Rule.,
Determine the actual area from the definite integral,
4. (i) use Simpson's Rule (and 6 strips, i.e. 7 ordinates)
to determine an approximation to the area b ounded
'/ the curve y = 12/x, the x axis and the ordin­
ates at x = 1 and x = 4.
(ii) Obtain a corresponding approximation to the ab ove
area b y the Mid-Ordinate Rule.
(iii) Calculate the actual area from a definite integral

�· THE RELATION BETWEEN THE INTEGRAL


AND THE PRIMITIVE FUNCTION
(a) Revise carefully, the proof in the text; that if A(x )
denotes the area b etween the curve y = f(t), the t axis
and the ordinates at t = a and t = x, (where f(t) is a
continuous positive increasing function), then
� {A(x )} = f(x). Hence, if F(x) is a primitive function
of f(x ), i.e. d [F(x )J = f(x J i.e. F'(x ) = f(x ), show
dx
that A(x) = F(x )+c, where C is a constant. Deduce, b y
sub stituting successively x a, x b, that
b
A(b ) {i.e. f f(t) dt} F(b ) - F(a)
463
INDEFINITE, DEFINITE INTEGRALS
(�) Indefinite Integrals
1. Integrate the foll"owing functions with respect to x
(a) x 2 + 2 + 1 2 (b) (3x2 -1) 2 (c) � (d) (2x-3)(3x+4)
(e) Ix-xix (f) (>'x-2)(/x-3) (g) (x 2 +-- 5)/x
X X
2
X

(h) x(l-x 2 ) (i) x(l-x/ (j) (2x + X


1)(Sx - 1)
X
(k) (5x 3 +7x2 -4)/x2
• 2.
(a) I
Perform the following integrations
(x+3) 9 dx (b) I 14x+3 dx (c) I (2x+3) 5
dx

J <3-4x) 5 dx (e) J<l-4x) 3 dx (f) J � �� 2


v
(d)
( x 1)
(3x 2 -4) dx
(g)I x 3(3x 4 -5) 3 dx (h)f
/x 3 -4x-l
(y) Definite Integrals

I I
Evaluate the following definite integrals
0 9 -1
1.

(a) (x -5) dx (b)


2 I;: dx (c) (x - �2 ) dx f f7
-2
-3 2 4
f (3x -2x+l) f (x - �) (x+l��x-l) dx
2
(d ) 2 dx (e) 2
dx (f)

i4) dx
-1 4 l l 4
(g) f x(x-l)(x-2) dx (h) f x2 (1 -

f Ux--1) dx f 11x (1-x)


0 l 2 l
(i) 2
(j) 2 2 dx
O

2. (i) If rx-l.idx = 2, find the va�.ue of a

(ii) If
l
y = x 3, evaluate
0 0
I 4
y dx + f
64
x dy

Determine the value of the foll owing

I I I2
*3.
dx
f 14x+l dx
l 2 2
(a) (1-x) lO dx /9�4
(b)
x (2x-1)3 (d)

I I 0
(c)

z 1! 3 (2 + z4/3 )6 dz O Fu-S)du
0 l O 1 l
(e' J yh+y 2 dy (f) (g) (u -Su-4)3
-1 0 5

D. AREAS ANO VOLUMES OF SOL I OS OF REVOLUTION


1. Revise carefully, the proof of the text, that the volume
of the .solid generated by rotating the arc from x = a to
x = b of the continuous positive increasing function
b
y = f(x) ab out the x axis is given by f 11[f(x)] 2 dx.
464
AREAS AND VOLUMES
State the result for the solid of revolution, for the
arc joining y = c, y =d on a similar function x =g(y),

E
�������������-
about the y axis.
2. (a) (b) {c) (d)

0 I
Represent
(1) each
nine
• )l
by defini�e integrals
X

area indicated in (a), (b)' and hence deter-


these areas.

(ii) each volume indicated in (c)' (d),and hence de�er-
mine these volumes.
3. Sketch the curve y = 4-x 2 , and find t:,e
( 1) area bounded by the curve and t!1e x axis
(ii) volume generated when this area is rotated about
(a) the x axis (h) the y axis
4. Calculate the points of intersection of the curve
y = x(4-x 2 ) and the x axis, and hence sketch the curve.
Determine the
(i) area bounded by the curve and t:·,e x axis between
x = 0 and x = 2.
(ii) volume generated by rotating this area about the
x axis.
5. Sketch the curve y = x2 + i2 , for values of x such that
1 .$ X .$ 2,
(i) Determine the area bounded by the curve, the
x axis and the ordinates at x = 1 and x = 2.
(ii) TI1is area is rotated about the x axis. Calculate
t:1e volume generated.
6. Calculate the area enclosed by the x axis and that part
of the curve y =-x2 +5x-4 for which y is positive. [First
find where y = 0, and sketch the curve.]
Determine the magnitude of the areas defined as follows:
(i) {(x,y) : y � 2} n {(x,y) : y � -x 2 + Sx - 4}
(ii) the boundaries are y = x-1 and y = -x 2 +5x-4
(iii) the boundaries are y = x-1, y = -x 2 +5x-4, y = O.
7. Find, by integration, the volume of the
(i) cone generated by rotating portion of the straight
line y = 2x from y = 0 to y = 6, about
(a) the y axis (8) the X axis,
(ii) sphere obtained by revolving the ser.ai-circle
y = � a!.,out the x axis,
(iii) paraboloid deterr.iined by rotating the arc of the
parabola x:'?, = 4y from x = 2 to x = 4 about
(ex) the x a:ds (S) tl;e y axis_.

465
AREAS AND VOLUMES
8. (i) Find the area enclosed by the curve y = 4x-x 2 , the
x axis and ordinates at x = 0 and x = 6.
(ii) The line y = 2x cuts the curve y 4x-x 2 in R, S.
Find the coordinates of Rand S. Calculate the
volume generated when the area enclosed by the
line and the curve is rotated about the x axis.
9. Find the area enclosed by the curves x 2 = y and y 2 = x.
:ihat is the volume generated by rotation of this area
about the y axis?
10. The gradient at any point (x,y) on a curve is 6x-3x 2 , If
the curve passes through the origin, find its equation
and show that it cuts the x axis again where x = 3. Find
also the area bounded by the curve and the x axis.
k
11. If y show that .QY is undefined at x = O, and
x 2(5-x),
dX
that a stationary point occurs where x = 35. Determine
its nature. Use these results to sketch the curves
L
2
(a) y := x' (5-x) (B) y 2 = x-(S-x) 2 . Determine the area
enclosed by y = x;2(5-x) and the x axis (leave answer in
irrational form), and calculate the volume generated
when the loop of y 2 = x(S-x)2 is rotated about the
x axis.
12. The curved surface of an open bowl 11itlt a flat circular
base may be traced out by the complete revolution of�
portion of the curve ay = x 2 about the vertical axis Oy.
The radius of the top rim of the bowl is twice that of
5
the base, and the capacity of the bowl is 11 a 3
units. Find the vertical height of the
6 bowl. cubic
13. If f(x) is defined as follows: f(x) x 2 +2, 0 .5 X < 1
3, X = 1
2x+l, 1 < X 5. 3
show that f(x) is- continuous at x = 1, and determine
(i) the area boun�ed by y = f(x), the X axis and the
ordinates � = 0, x = 3
(ii) the volume generated when this area is rotated
about the x axis.

TOPIC 10
LOGARITHMIC EXPONENTIAL FUNCTIONS
Note The results in the following sum:nary should be rev1,sed
from the text.

t
f;,. Tf:E i:ATUR/\L L0GAR I TH!J. IC FLJ'·ICT I Oi'J loa X

du
1. (a) Defin:tion log X X > ()
u'
Log X is also 1
written as log x, .en x, log lx l • "
e ,a

466
LOGARITHMIC AND EXPONENTIAL FUNCTIONS
If x=l, logx=O; if x>l, log x>O; x<O
du
e

(8) Definition of e,f = 1


u
l
e is irrational; e � 2·718

2. Looariti1r1 Laws (These may be proven directly from the


0 • ....

definition.)
If x, y are positive and n is rational,
(i) log xy = log x +log y (ii) log(�) = log x - log y
. y
(iii) log (x ) = n log
n
x

3. Derivatives and Integrals


d dx
(i) (log x) = land . ·. f = log + C
dX
x
X X

(ii) log{f (x)}J f x


d�[ ;��)) and .', f log{f(x)}+C
)c�f
=

4. Graph of �: Zoq x

log x + +oo slowly as x + +oo

log X rapidly as x + 0+· domain: x > 0


range: -oo<log x < oo

5. FlU'ther Results
(i) x, X > 0
og x
(iii) log = ---
l e Change of base law.
a
x
log a
e
i lim � =
*(iv) e = l m (l +l)n *(v) 0
n+oo n x-- x
lim 0 lim a - x O a real
*(vi) = O, a> 0 *(vii) +oo x e "' '
x-+() x log x x
[Results (v), (vi), (vii) are often useful in curve
sketching, ]

B. THE EXPmmmAL FUNCTION e


x

1. Definition If y "' log x ' then


e
the inverse function
i.e.

2• Properties
X +
e > O for all x; as .. -00, e 0 slowly;
x
(i) x +

as + +oo, e + +oo rapidly.


x
x

1 (i.e. y = e crosses the y axis at y � 1)


x

467
DIFFERENTIATION log x, ex
dx 1
(iii) ..J!(e ) = e ' (use made o f resu lt = ),
dx dy dy/dx
x X

and
f
e
x
dx .. e
x
+ C

..J! { f(x) } = f(x)


( iv) e f' (x) ' e
dx
and , •• f f' (x) e dx = e ( x) + C
f (x) f

d X X a
( v) -(a ) = a log a, and a dx=--+c
dx log a
X
X

y x+y x-y
(a) e .e
I

(vi) e ( B) e 1 eY = e
x X

nx -x
(y) (e e (o) e = .L
e
x n
) x

3, F,xponentiaZ W:..J of gr>o1,1th or> of deaay


I f N is t he amoun t o f a qua t i t y present at t ime t, t hen
if t he rate o f growth o f N varies as N, then
n

�; = kN, where k is a cons t ant.


It can be shown thet N = Ae , where A is a constant.
-kt
kt

}l = A�
dN =
For decay, t!:e la\/ is o f the fcrm -kN, and
dt

DI FFERENTI ATI ON, I ltCLUDING APPLICATIONS


! {log(3x-5 ) } = �_ ,
�-
1. (i) Show that
d 3 5
d 3
X -7
3
and 2 . eX -7
(e ) = Jx
dx
(ii) Prove the foilowing -results fot 1;
(a) y x
3
1og x; �= x 2
(1 + 3 log x)

e dv
(b) y e· (x-3 )
x
x
= �; � = I x
4

7
( c) y ( log x) = 7(1og x)
;
dx
.21. I x
7 d 7
6

( d) y log x = 7 log x; .ll


dx
lzx-5
= -

( e) Y log ( ] ) = \log(2x-5) - log{Bx-7 ) ;


B x-
X

= _1___
2x-5
.21.
dx 8x-7
s__
( f) y log{(3-4x ) 3• ( 5-7 x) } = 3 log(3-4x) + 1og{S-7x );
.21_ .. -12 _ _ 7_
dx 3-4x 5-7x
-x 2 dv -x 2 1
(g) y e • log Sx; � e [ - 2x .log Sx]
=
-3 log x _ log x-3 3
e e .. X-3 i .21_ "' ..
-;

{h) y dx
;i+
(i) y = 10 ,
X
f = 1o
x
log 10 (j) y • log 1 0x • ��:
e�0
e
468 1
_!!l. ..
dx x log 10
e
DIFFERENTIATION log x, ex
2x-3
*(k) y= x , tak'ing natura-l 1 ogari· thms,
1 _gy_ 1
log y= (2x-3) log x and .. - = (2x-3).-+ logx .2
dx x

i.e. _Qy_ _ x -3[ -3 + 2 log�]
2x 2x
dx - x

2. Differentiate each of the following


(i) log 2x; e
2x S xS
log x ; e ; � log x; lex
-log x,
(ii) log e-
x
e ' (l+x3) log(l+x 3);(log x)/ x 3 ;
x X
x3 e- ; (2x+l) I e
3x+·4
(iii) log/2x-7; log{(2x-7)(3x2 +4x-l)}; log(7x_ 2);
log(S x+9) 8

(iv) logm,; log(


l+:x); e
x
log x; e
2x
I log 3x;
log(log x)
-x2
(v) log 3x; 10 x log1o x

*(vi) (log x)
x
; (x-2 )
x-2 il-3x
; (l-x)3; /i+x
lk
4x
3. (i) If f(x) = log(2x+l) + e , find f(O), f'(O), f"(O)
Interpret these results geometrically.
2x dV
. dt' - dx
(ii) If V e , find when x= 0 given that -= 3
dt
when x= 0.
(iii) If y = lo� x2 , for what value of x
x is�= O?
Show that�� IO for finite values of x.

-x
4. (]..) If y xe prove that E.1
dx2
+ 2 .QY. + y= 0
dx
kx
(ii) If y e is a solution of the differential equa-
d2
tion � - 41; + 3y= O, find the values of k.

5. Find the equation of the tangent and normal at the point


P where x = 1 on the curve y = log x. If the tangent and
normal meet the y axis at A, B respectively, find the
area of tJ. APB.
Also find the coordinates of the point of intersection
of the tangent at P and the tangent at Q (where the line
through B parallel to the x axis meets the curve.)
x
6. For the curve y= e , determine the equation of the tan­
gent at the point P with abscissa a. If this tangent
meets the x axis in T and N is the foot of the ordinate
from P, show that *TN= 1 unit. The normal at P meets
the x axis in G; find *NG (in terms of e).
7. What is the domain of the function log( 7-Sx)? Prove that
4 69
APPLICATIONS OF DIFFERENTIATION
the graph of y = log(7-Sx) is (a) monotonic decreasing
(8) concave down, for all values of x.
x
8. (i) Find the intervals of x for which the function xe
is (a) increasing (B) concave upwards.
(ii) Determine the coordinates of the stationary point
and point of inflexion on the curve y = xe . Hence
x

sketch the curve.


(iii) Use your graph to determine the nwnber of real
roots of the equation xex k, where
(a) k = -e (B) k = e
]
(y) - ; < k < 0
(iv) Find the equation of the inflexional tangent, and
find where it cuts the x axis.
d2y
9. If y = �, find � and �· Hence show that there is
x dx
one stationary point, and determine its nature. Obtain
the coordinates of the point of inflexion. Show that
the curve crosses the x axis in one point, and that if
0 < x < 1, y < O. Use these results to sketch the curve.
- 2 d2y -x2
10. If y = e x , find � and prove that�= 2e (2x2 -1),
dx
Hence determine for what value(s) of x, the curve is
(i) (a) rising (b) falling (c) stationary
(ii) (a) concave. downwards (b) concave upwards
(c) changing curvature.
Sketch the curve, and state the greatest and least
-x2 1
values of e in the interval - 72 � x � 1,
*11, A submarine telegraph cable consists of a copper core
with a concentric sheath of non-conducting material, The
ratio of the radius of the core to the thickness of the
sheath is x, and it is known that the speed of signal­
ling is equal to kx 2 log(l/ x), wher.e k is a constant,
Show that the greatest speed of signalling is reached in

f
a Cab le for which X "' 1/ /e,
12, If y "' 2h - log X, prove that, for X > 1, > 0, and
find y when x • 1, Interpret these results geometric­
ally. Sketch the curve y • 2h - log x, for x > l, and
thus explain why 2h > log X for X > l, Justify the
inequality O < log x < Zlx, and on dividing the
l
inequality by x, prove that im (1£&...!,) • 0, Deduce, by
x x
1
letting x • ;, u + +a>, that- lim
-+0 + (x log x ) • 0,
x

g. INTEGRATION1 INCLUDING APPLICATjONS


l, Find the primitive function of the following functions,
1w
(a) Sex + !
X 2 l
(b) e 3x - -1 (c) le- + (e) + -x
X Jx e
x
(e) 12 + 1 - 13 ( f) J!:ll.2 (g) e + e + 1
3x
(d) 3x-2
X X X X X 2x
e
470
INTEGRATION; LOGS AND EXPONENTIALS
(h) (e - e-x) 2 (i) (x + 1 )2 l
(j) e -3 + _ _
x-3
x x

(k) eSx + 2 7 (l) __ _ e 2 9- x + _l_


X2

- Sx -2 3-4x e3-4x
2

3x 2 2x-3
(m) (2x -3)-1 + (2x-3)-2 (n) 2 (o) (p) 2-
x +8 4-x 3 3x -5
2x

Sx 4-6x 2 +8 x 2 -7x +l
(q) 5 2x 3 +8x-l (r) 2 x e (s) ( 2x-7)e
x

x -
x2

x +x- 7
(t) (3x 2 +1)e (u) B + x 5e -3 (v) X 2 e -
x +2
3
x 7
x6 x3

eX
(w) x '+(1+ xs)-1 (x
) 1+ 2 e�
2x-l 4
2. (i) 4Show that 1 + d hence eva).uate
2x-S = 2x-S'
I 2x-l
an

2x-5
xd

(ii) Prove that - ,and hence obtain


(x +l)(x + 2 ) = x +l x+ 2
3 1 1 1

r (x!)(x+2 )
-x
3. (i) If y = log(e + e ), fin d 1; and hence evaluate
l X -x
x

J e - e
X -x xd
e + e

log{x +lx2+1},
""l

(ii) If f(x ) fin d f'(x ), and hence ob-

f 1 ll+x2dx
=

tain

Eva luate the following definiteintegrals. [Give answers


in terms of e an d n atural logarithms, an duse tables to
4.
give approximations.)

( a) I� 3

-1
(b) f \2
k
x
xd

l
(c) f e
2
2x
- e -2 x dx (d)
I
I+

O l log 2
x�
dx 2x- 2 - 2x xd
(e) I 1_ 2x (f) f e dx (g) I e
2 2 2

(h) 1
7
I (- 3- + -1-)
l -2

4- 3x l+x
0
xd (i) f (el
x -x xd
- e )z
0

l
x 3 dx x2 l
(j) I. ( - ) ( - ) xd (k) f (!) I xe - xd
0 2 0 2

X xZ + 2
I
x 2 x

e2
(m) I
l
(x + 2 ) dx
3

x2 +4x +3
(n) I
0
l
LJ!!.
2 e +l
x
X
(o)
O
I · x
xd
log x
*(p) f !O
0
x
l

0
dx
e

5. The slope at any point (x ,y) on a cert a in curve is given


d eX + 1 2
by (i) _gy_x -- ( x ) If the curve p asses through the
d e
471
APPLICATIONS OF INTEGRATION
point (0,2), find y in terms of x.

(ii) x .!!Y = x - 6. If the curve intersects the line


dx
X = 1 at y = 2, find where the curve cuts the
line x = 3.
������������-
6. (i) Find the magnitude of the area bounded by the
curve xy = 4, (a) the x axis and the ordinates x=l
and x=4 (b) and the line x+y = 5.
(ii) Calculate the volwnes obtained when the areas of
(i) are revolved about the x axis.
_1
(iii) What is the size of the area bounded by y = 4x
y = 0, X = -1, X = -4?
7. On a sketch, mark the region satisfying the following
inequalities y � ex , y � x, X � 1, X 2: O. Calculate
the (i) area of this region (ii) volume generated when
this region is rotated about the x axis.

8. The sketch shows the curve
-x
y =\(e + e ), called a
x

Determine the
(i} size of the area
aatenary.

shaded
(ii) volwne generated
when this area is�����---�o���A����
X

rotated about the


x ax is.
*(iii) length ot the arc PQ assuming that its value is

given by definite integral r Ii+ 2 dx <f>


0 1
Find the turning points of the curve y = x + , and hence
;
sketch the curve. Find the (i) area bounded by the curve
y = x+x-1, the x axis and the ordinates at x = 1, x = e,
(ii) volume generated when this area is rotated about
the x axis.· [Leave answers in terms of e.]
10. (i)
6-x- -x -
= 2e - 3e , and when x = O, y = 4, .!!i = 5,
dx
If

find� when y = O.
-x
[Hint: First pr.ove y = 2e - 3e + SJ.
x

(ii) Given that f"(x) = - 12 - �3• find f(x ), given


X X
that f(l) � -9 and f'(l) = 2.
11. The gradient at any point (x ,y) on a curve is given by
(i) .!!Y = y 2 (ii) .!!Y = -2y
dx dx
(iii) f .L
eY
(iv) .!!Y = y-1
d1c
If each curve passes through the point (0,1), find y in
terms of x . ------··-··-·- ----···-·-
472
CIRCULAR MEASURE
12. The popula t ion n of a cer t ain coun t ry is increasing at a
dN
rate always propor tional to N. SIiow tha t dt = kN where
k is a const an t, and hen ce tha t this can be expressed in
kt
form N = Ae where A is a const an t ,
t he If t he pop.u­
lat ion is increasing a t t he ra te of 2% per year, and t he
popula t ion is now No, show tha t when t = O, N = N0- and
when t = 1., N"' No+ •02N0 •
·0198t
Hence show t ha t N = Noe , (using t ables).
If No = 1 million, find
(i) the expec ted populat ion in 20 years t ime
(ii) af ter how many years the population would be ex­
pec ted t o exceed 3 million
(iii) in what time the popula t ion should double it self.
13. In t he inversion of raw sugar, t he t ime rate of change
of t he amoun t of raw sugar present varies as the amount
of raw sugar remaining. If after 10 hours, 1000 lb of
raw sugar has been reduced t o 800 lb, how much raw sugar
will remain aft er 24 hours?
14. A bac terial popula tion of size B is known t o have a rate
of growt h propor tional to B. If .bet ween noon and 2 p.m.
t he population t riples, a t what t ime would you expec t B
t o be 100 t imes the noon population?

TOPIC 11
TRIGONOMETRIC FUNCTIONS

A. CIRCULAR MEASURE
The angle at the centre of a unit
Slfi.f!.JARY: c ircle (i.e. a
circle of radius l unit) sub tended by a unit
arc (i.e. an arc of length l unit ) is called
a rodian.
1. In the figure, POR is a diame ter of a
unit circle. Show tha t *arcPqR=n unit s,
and .·. *POR = 11 radians, and hence
n c = 180 ° .
2. In the diagram. show that
(i) *arc ADB : Circumference of cir­ �
cleaS:211, and hence show .t: • re •.
(ii) *area sector AOB : *area of circle•0:2n,
and hence show *area sector AOB = \r 2 e �
(iii) *area minor segment ADB • *area sector Aon - *area A AOB
\r 2 (e - sin0)

3. (1) Two points one k• apart are connected by a. rail-


473
TRIGONOMETRIC GRAPHS
way line in the form of a circular arc of radius 3
km.. Find the length of the track to the nearest
metre.
(ii) A sector of angle 20 ° is taken in a circle whose
circumference is 90 m; find the area of the
sector in sq. m , as accurately as your tables
allow.
4. The centres of two circles, which intersect at A and B
are 6cmapart, and lie on opposite sides of the chord AB.
If rAB' subtends angles of 60 ° and 120 ° at the centres of
the two circles, find (1) the radii of the circles
(ii) the area of overlap of one circle on the other.
5. The angle of a sector AOB of a circle (centre O) of
2
radius 12 cm is ; radians. The sector is formed into a
hollow cone without overlap (by bringing OA, OB to­
gether). Find the surface area, the semi-vertical angle
and the capacity of the cone.
6. A star whose distance from the earth is 10 9 1<. m , sub-
tends at the earth an angle of\ minute. Find the
approximate length of its diameter.
*7. Two wheels of radii 12cmand Scr,,lie in the same plane and
have their centres 2Scmapart. Find the length of the
belt which passes tightly round the wheels
(a) witbout crossing and (b) crossing between the wheels
8. Find the solution,for 0 S ei 211' of the following
equations
(1) (a) sin e= -\ (b) cos 26 = "f2' (c) tan
e
2
= -15
(d) sec 2 e= 4
(ii) (a) 2 sine= cose (b) (sin6+1) (cos6+1) = O
(c) tan 2 e+tane= O
(iii) (a) 2 sin 2 e + 3 sine- 2 0
(b) 9 cos 2 e- 9 cose + 2 o

B. THE TRIGONOMETRIC FUNCTIONS AND THEIR GRAPHS


1. Give sketches (not on graph paper) of the following, for
0 ix i 211. (a) y = sin x (b) y = cos x (c) y=tanx
(d) y = cosec x (e) y = sec x (f) y = cot x
2.
FUNCTION DOMAIN RANGE PERIOD DISCONTINUITIES
FEATURES
AND OTHER

Complete the table above �or eaoh of the 6 trigonometric


functions. (Your sketches from question 1 are useful
here.) 474
TRIGONOMETRIC GRAPHS
3. For the interval O � x :S 2TI, on
the same axes, sketch
the curves (a) y = 2 sin x
(b) y = - sin x (c) y = 3 + 2 sinx (d) y = 1 - sin x
4. On separate axes, for -TI� x :S nsketch the graphs of the
pairs of curves below
(i) y sin 2x and y = cosec 2x
(ii) y = cos x/2 and y = sec x/2

5. With the same scale and on the same axes, draw graphs of
(i) y = sin 3x and y = cos 2x for values of x from
-90 ° to +90 ° inclusive and, with the help of your
graphs, solve the equation sin 3x = cos 2x for
values of x within the given domain.
(ii) y = tan -ix and y = cot 2x for value�, of x from
0 torr inclusive, and use these to solve the equa­
tion tan 3x = cot 2x in the given domain. For
what intervals of x is tan 3x > cot 2x?
6. Draw the graph of y = \TI sin 2 x for -2TI :S x :S 2TI, and use
it to solve the equation 2x = TI sin 2 x. a Shade in the

region represented by the definite j (\Tisin 2 x-x) dx


2

integral .:!..
4
7. On the same diagram, over ti1e interval 0 :S x .S TI , draw
graphs of (a) y=sinx (b) y = f3 cos x
(c) y = sin x + /J cos x
Hence (i) determine the maximtun value of sinx+ ficosx,
and the corresponding value of x.
(ii) find the !J>proxirnate solution of the equation
sin x + 13 cos x + \ = O, which lies between
0 and TI (leave your answer as a rnultiple of TI).
On the same axes, draw graphs of (i) y = sin x
t 1
8.
(ii) y = sin 3x (iii) y � sin x + sin 3x
for values of x such that C 0 x :S 180 ° .
2
' x +_ l sin

Hence so 1ve the equation sin


domain. 3 . 3x - 3 = 0 in
. t his

y
EXPRESSIONS FOR
sin (a ± B), cos(a ± B), tan(a ± B)
P, Qare points on the unit circle
x 2 +y 2 = 1 such that x-+---::? -'-'-+-x
*XOP c a, -...
1.
P
*XOQ= B.
Show that Q has coordinates
(cos B, sin B) and write down the
coordinates of P.
By obtaining values for PQ 2 by (i) the cosine rule in
6 QOP and (ii) the formula for the distance between
2 points, show that 2-2cos(a-B)=2-2{cosacosB+sinasinB} ,
475
a 1 1:1 RES UL TS
and thus prove cos(a-1:1)= cosa cosf:I + sina sinf:I ••.• (S)
In (S), put y= -f:I and obtain a result for cos(a + y).
In the result for cos(a-1:1), put a= ..!!. - o and in the re-
sult for cos(a+y), put a= 2 - E; hence obtain results
for sin(o+f:I) and sin(E-y).
2. Using results of the form proven in question 1, obtain
expressions for tan(a+f:I) and tan(a-f:I) in terms of tana
and tan f:I,

3. Without tables, obtain expressions for the following (in


simplest irrational form, where applicable).
sinl68 ° cosl08 ° - sinl08 ° cosl68 ° tan58 ° - canJ3 °
(a) (b)
cos41 cosl9 - sin41 sinl9
° ° ° °
l+tan58 ° tanl3 °
lT 711
(c) sin 75 °
(d) tan (e) cos
12 12
4. (i) If a,f:I are acute angles such that S in a= l
5
5'
cos f:I= , find without tables, the value of
13
sin(a+f:I).
(ii) If A,B,C are the angles of a triangle and
-4 12
cosA = , cosB= 13, find sinA and sinB without
5
tables. Show that cosC = -cos(A+B), and hence de­
termine cosC in rational form.
1
5. (i) If tanx= -2, where O < X < lT' and tan y = , where
3
lT < y < 211, prove that tan(x+y)= -1, and hence
find (x+y), in terms of 11.
3 and tan 0= 2, obtain tan 1jl with-
(ii) If tan(0-1jl)=
4' 5
out tables.
*6. The triangle ABC is right angled at B, and *BC s 2.*AB.
If P, Q are the points of trisection of rBC"', without
using table�, calculate (i) tan PAB (ii) tan PAC
(iii) tan PAQ.
lT lT
7. Write down expressions for_ sin(x + ), cos(x + ),
4 4
tan(x + f>. Hence simplify
(i) sin(x + f>
11
sin(x - {>
11
(ii) cos(x + f> cos(x - f>
(iii) tan(x + ) tan(x - 4)
4
8. Simplify211 211
(a) cos( - 0) + cos0 + cos('"""j' + 0)
3
sin(ml + sin x·+ sin(x-:z'.2
(b)
cos(x+y) + COS X + cos(x-y)
476
ANGLE BETWEEN TWO LINES
9. Prove that
(i) tana+tana = sin(a+a) seca seca
(ii) cos(A+B) cos(A-B) cos2 A - sin 2B
(iii) sin(6+1j,) sin(e-lj,) - cos(0+ij,) cos(0-ij,) is independ­
ent of ij,,
(iv) if k cose = cos(0-a), then tan0 = k coseca - cot a
a
10. (i) Prove that tan(A+B+c) = t nA+tanB+tanC-tanAtanBtanC
1-tanAtanB-tanAtanC-tanBtanC
[Hint: write A+B+C as (A+B}+C)
(ii) If A,B, C are the angles of a triangle, and
tanA tanB tanC = k 1
4 = 5 = 6 ' show that k = Vz· and
hence use tables to calculate the size of the
angle A.
11. (i) Express 13 cose - sin0 in the form R cos(0+a),
where R > 0 and O �a � 2rr, Hence determine the
maximum value of 13 cos0 - sin0 and the corres-
ponding value of a.
(ii) Express 13 cos0 - sine in the forms
(a) A cos(0-B) (b) B sin(e+y) (c) C sin(0-6),
where A,B,C > 0 and O 5 8,y,6 � 2rr.
12. Determine a positive number R and an angle a between
0 ° and 360 ° , such that
(i) 3 sine - cose R sin(0+o)
(ii) 24 sine+ 7 cose = R cos(e-a)

0. ANGLE BETWEEN JWO LINES


1. (i) In the figure, the lines
y = m1x+b1,
y = m2x + b2 intersect
at an acute angle ij,.
--,,.""--,.+-��-r==--���-....x
Prov� that lj, � a-8, and
m -m
hence that tanlj, = i 2 , m1m2 ¥ -1.
l+m1m 2
(ii) Deduce from this, the Gondition for the lines to
be at right angles to each other.
rr c
(iii) What does result in (i) become if a =
2
2. Calculate the acute angle between the lines
(i) 3x-4y+2 = O, 2x+3y-9 = 0 (ii) x+3y = 9, x = 2
3. The vertices of a triangle ABC are A(3,3), B(-1,-1),
C(-1,S). Find the angle made by the median from B with
the side AC.
4. If the lines through the point (2,3) which make angles
of .f m,with the line x-2y .. 1,
C
have gradient m, prove
that - \; - tl. Hence f,ind the equations of these lines
l+r.i/2 -
477
APPROXIMATIONS FOR SMALL x
·and show that they are perpendicular to one another.

5, Find the points of intersection of the curves y = x 2 -x+l


and y = 2x 2 -4x+3, and hence determine the size of the
acute angle between the curves at each point of inter­
section.
6. Sketch the curves y = x 2 and y = x�3. Find the angles
between these curves at each point of intersection.

�PPROXIMATIONS TO sinxt cosx,


when x is sma 11 ,
Limit sin x � I �
X -+0 X
A
I

1. In the diagram, 0 is the centre of a unit circle; AC is


11
the tangent at A, and *AOB = x (where x < ).
C
2
Noting that *area of MOB< *area of sector AOB< *area
MOC
prove that \sinx < �vt < \tanx, and hence that if x is
an aaute angle, sin x < x < tan x • . • . .• , •.•• , • . • • . (L)
2. If is small, then
X
(i) *area MOB� *area sector AOB i *area MOC, and
thus show sin x � x � tan x •
. *OA
(ii) cos x "° = 1.
*OB
-1--
3. From (L) above, show 1 < _x__ < and thus deduce
sin x cos x'
lirn �x- lirn 1 ]
that
x-+0 sin x
= 1 [First obtain x-+0 � •
lim
Similarly, from (L), show that x-+0 � = 1
X

1
4. In the figure, prove that - - < e < 1
rb5' 8
5. If X is small and measured in radians,
(i) arrange in. asaending order of rnag11itude, the
quantities cos x, x, sin x, cosec x, tan x.
(ii) prove that $in({+ x) � -;tz"(l+x) � cos({- x), and
311 x+l
that tan(-;- - x) "°• -- • Use one of the results of
.. x- 1
(ii) to calculate the approximate value of sin46 °,
given Ii= 1•414 and 11 = 3·142. (Answer correct to
2 decimal places.)
6. (a) Without trigonometria tables, (given 11 • 3•142),
determine the n1111erical value of
(i) tan 1 °36' (ii) cos 89 ° 48 1 [Note cos0 = sin(.!!.-e)J
2
(b) Given that 0 is a small angle, solve without trig­
onometric tables the equation sin 0 = •02346, giving
the answer correct to the nearest minute
7. The angle of elevation of a vertical tower from a point
478
DIFFERENTIATION OF TRIGONOMETRIC FUNCTIONS
distant 2400 m from it is 1 ° 36'. The point of observ­
ation and the foot of the tower are in the same
horizontal plane. Calculate the height of the tower,
(in metres,correct to 3 significant figures).

8. Determine the limit of each of the following, as X -+ 0.


2 sin x 3 sin 2 x
(a)
x2
(b) _x__
X sin 7x (c)
3 sin 2 x 3x sin Sx
(d) (e) (f)
X tan x/2 sin 4x
sin 3x + tan Sx 2x cos 3x + sin x/3
(g)
2x
(h)
X

9. Investigate whether f(x) is continuous at x = 0 in each


of the following
sin x sin x
(i) f(x) = (ii) f(x) = --, X f 0
X
= o,
X
sin x X = 0
(iii) f (x) = ' X ,;. 0
= 1, X = 0

F. DIFFERENTIATION OF TRIGONOMETRIC
FUNCTIONS AND APPLICATIONS
1. (i) Write down the results for sin(a+S), sin(a-8) and
hence prove that
sin(a+S) - sin(a-8) = 2 cosa sin8 ......•.••.. (T)
By letting a+S = x+h and a-8 = x, show that equa­
tion (T) becomes
sin(x+h) - sin x • 2 cos(x + ) sin h.
h
2 2
(ii) From the definition for f'(x), prove that if
d lim cos(x + �) sin
f(x) .. sinx,then (sinx) .. h-+O
dx {
tJ
h/2
and deduce the derivative of sin x.
(iii) Noting that cos x • sin(
1T
- x), deduce the deriva-
2
tive of cos x, using the resuZt of (ii).
sin x
(iv) Using the fact that tan x •--,and the results
dcos·x
of (ii),(iii),prove that d (tan x) • sec 2x,
x

2. Prove the following results for�


x x 1
(a) y • sin ; EL dx • cos 3 . 3 (b)p•cos(logx);
3
(c)f�..• e tan2x
l� • -
sin(log x) l
X

� • e tanZx . sec 22 x. 2 (d) y • tan(sin x);


{EL • sec2< sin x
(e) y•log(lcos 4x) • �log(cos4x); dx > , COS X

lEL • .! --1- • - sin 4x • 4 • - 2 tan 4x


dx 2 • cos 4x
479
TRIGONOMETRIC DERIVATIVES
0
(f) y = s in x = s in ; = cos �� cos x
0
��0 � 1;0 0 1;0
(g) y = cos 5( 2x -f) = [cos (2x -f)/;
11 l;
ir = S[cos ( 2x - -)) 11
, - s in(2x - -) . 2
{dx 7
1 d 7
(h) Y = lcot 3x = (tan 3x)-72 � = - 3 s ec2 3x / 2 (tan 3x) 3/2
e- sin 3x;
x
(i) y ( ) y = ;
ir = e-x(3cos3x-srn3x)
. .
ldx ir=-(s1.nx.logx-+{cos xV xJ
j ldx �:::(log x)2

3. Find the derivative s of ea ch of the f ollowing function s .


x cosx
(i) s in 2x ; cos(�); ta n(x 2 ); sin(e ); e ;
0
(ii) tan(logx); sin(Sx - {); cos x ; sin(sin x):

. iii) s in 7x '· lcos( 2 x+l) · --1_,..._. (3 - 2 sin x /2) 6;


(
' cot�4x '
(iv) log(l+3s inx); log(l+cosx) 7 ; x3 sinx;
-ax x cos Z x
(v) e cos bx; s in3x log 2 x; log(tan3x) ; ; X
sinx - cosx . lo g(--�-
l+cos3x), sin x
( vi)
s inx + cosx ' s in4x ' 2+3cos x
2c 2 2x
x os 2x ; e ( 2 c os 3x + 3 sin 3x)

4. (i) If y = 2 cos 3x or if y = Acos 3x + Bsin3x , prove


in both case s , that <lx7 + 9y = O.
(ii) If y l og{(l-cos x '1 I (l+cos x)}, prove that
ir = 2 cosec x.
dx
(iii) If y = e
x
sin x, prove that�= ti e
X
x
sin(x + f>,
X 11
and that _.£y
dx2 = 2e sin(x +z).

s. If f(x) = J..p< 2 + s in XI O.:, Xi 211:find f'(O), f"(O),


and interpret these result s geometrically.
(i) Find the equation of the tangent to the curve
y = f_(x) at x = O.
(ii) Sketch the graph of y = sin x, and use it to de­
termine the values of x in the interval O � x 1 211,
for which�> sin x.
Hence calculate for what values of x, the cµrve
y = J.,x2 + sin x is concave upwards.
6. (i) Show that the curve y = x + sin x h�s no relative
maximum or minimum turning points, even though
d
�here are points on the curve where .!!Y. s O.
dx
(ii) Find the coordinates of the stationary p oints and
p oints of inflex ion on the curve y = x +sin2x , for
480
INTEGRALS OF TRIGONOMETRIC FUNCTIONS
0 � x 5 2n. [Leave answers in irrational form.]
7. For the curve y l·+ 2 sin x - 2 sin 2 x, show
f = 2 cos x(l - 2 sin x), and hence find the stationary
points in the interval O 5 x 5 f. Sketch the curve in
this interval, and find the greatest and least values of
1 + 2 sin x - 2 sin 2 x for O �
< n
X - 2'
8. Tile length of the perimeter of a sector of a circle is
12 cm. Express its area in terms of the radius of the
circle, and hence find the maximur,1 value of the area.
*9. Tile illumination I at a point Pon
the edge of a table of radius r,
is ;c:iven by I= where k
k COS X
di
is a constant. The light is Table
directly above the centre O of the
table. How far should t�.e light be
above the table in order to give
the strongest illumination at P?
[Hint: first prove I = 2 sin 2 x cos x]
r

g,, INTEGRATIW AND APPLICATIONS


1. Complete the results
(i) f. sin ax dx = (ii) J cos ax dx

(iii) J sec 2 ax d:t

(iv) J tan 2 ax dx'= J(sec 2 ax - 1) dx ,,•


(Note sec 2 a 1+ tan 2
o]
2. Find the primitive functions of the following
(a) 2sinx+3cosx+4sec 2 x (b) cos3x-sin4x+sec2 8x
(c) cos� - sin� (d) sin(.!! - 2x) - sec (3x-l)
1
3 2 3
(e) tan1 2x.
3, Evaluate the following definite integrals (leave answers
in irrational form where applicable).

ff (b) ff sec x dx
t'
cos2x dx 2

-fr
(a) (c)

f
, �
d)
f
si� 3x dx ( )
: ;
(sin46+sin e) d6
:
f 6 f
Ir (f) :(sin3x+cos5x) dx (g) sec 2 2t dt (h) \ [l+cos2(x-a)]dx

4 12n !_
12
0

*(i) [sin(m+n)6-sin(m-n)6]de; (m 1 ni m, n integers)


O 481
APPLICATIONS OF TRIGONOMETRIC INTEGRALS
4. Use the relations cos 2 x = \(l+cos2x), sin 2 x = \(l-cos2x),
sin x cos x = { sin 2x to obtain

f
(i) ;cos 2 x dx J:sin 2 2x dx f
11

(ii)
.
(iii) !sin f cos f d x

"J"
s. (i) Use the derivative of (x sin x + cos x) to evaluate

{fx cos x dx .
d
(ii) Prove that [-cos x + l cos 3 x] sin 3 x ; hence
dx 3
find f:sin 3 x dx,
6. (i)

(ii)
d
Find d [log(3 + 4 tan x) J; hence obtain
x

Show that the derivative of -j-e


x
(s.
i nx +
O
cos x ) ,l'ith
j*sec 2 x dx
3+4tan x

respect to x is e cos x, and hence evaluate


n
x

j2xe cos x dx .
0
7. (i) The gradient at any point (x,y) on a curve is
given by -2Y
dx = sin2x + cos �3 + sec sx. If the curve
2

passes through the point (0, 1), find the equation


of the curve,
(ii) Given that�= - 4 cos 2x - 9 sin 3x, and when
x O, � • 1, and y • 4, find y in terms of x,
X
= d

and calculate the value of y when x-• 2.


8. Sketch the curve y • sin x for O ! x � 211. Find
the area enclosed by the curve y • sin x, and the
( ex)

x axis between
(i) X c O and X a (ii) X • 0 and X • 211,
2
11

p t h• ,,,ult in (ii) with tho valuo of


=� : :x:
i:

(S) Find the points of intersection of the curves


y • sin x, y • cos x and determine the size of the
"triangular" area bounded by y • sin x, y • cos x
and the y axis.
11 < < 11
9. Sketch the curve y • l+cosx for -
the 2 - x - 2. Determine
(i) area bounded by y • 1 + cos x, the x axis and the
ordinates x • - 2, x • .
2
11 11

volume generated when this area is revolved about


the x axis. [Hint: Use one of the results of
(ii)

question 4.]
482
THE INVERSE TRIGONOMETRIC FUNCTIONS
10. Prove that Xd[log(cos x)J = - tan x,
(i) d
(ii) Obtain the coordinates of the point of inter­
section, of the normal at the pO'int where
11
x = on the curve y = tan x, and they axis.
4
(iii)Determine the magnitude of the
(a) area. bounded by y = tan_ x, the x axis and the
ordinate at x = 11/4. [Use the result of (i))
(B) volume generated when this area is rotated
about the x axis.

TOPIC 12
THE INVERSE TRIGONOMETRIC FUNCTIONS
1. Sketch the curve y = sin x for the interval -211 .:: x .S 211,
Hence, sketch the graph of the inverse relation x = sin y
for IYI � 211, Mark clearly on this graph, the portion
taken by convention, to denote the inverse function
y = sin-1 x. State the domain and range of y = sin-1x.
2. Repeat the procedure adopted in question 1, for the
functions (a) y = cos-1 x (b) y •. tan-1 x.

3, Ex;, lain from your


s ket cl1es, why
(i) sin-l(-x) •
-sin- l(x) (ii) cos-l(-x)•11-cos-l(x)
(iii) tan-1 (-x) •- tan - 1 (x)
4, Without tables, write down the values of
(a) sin-1(�) (b) cos -l<f> (c) tan-l(7i)

(d) sin-1 (-1) (e) cos-1(- 13) (f) tan-l(-/3)


2

5, Evaluate, correct to 3 - decimal places,


(i)
2
sin-1 <3) (ii) cos-1(-
3
s> 1
(iii) tan-1(- 2>

d
6. Assuming that u (sin u) • cos u, prove that
d l
d (sin-1x) •�·where
°d"'
X -X lxl < 1. Hence determine

...S. [ain-1(�)] and ...S. where 'a' is a poai•


dx a dx [ain-l(ax)] '
tive constant,
7. Obtain similar results to those in queation 6, for
y • coa-1x and y • tan-1x, etc, aasuming that
d
- sin u, and � (tan u) • aec 2 u.
du (cos u) • - d
8. Use the results of questions 6 1 7 to obtain the deriva­
tives of
(a) sin- l(Jx) (b) coa-1(3) (c) tan-1(2;).

483
INVERSE TRIGONOMETRIC FUNCTIONS
9. Prove the following
(i) ....!! [sin-1(3 x- l)]
1 d
- .-(3 x-l)
3
dx ll- (3x-l)2 dx
=

....!!{(cos-12x) - 1 }
d · 1 )
(ii)
dx lcos-12x dx
=
_2 d _
1
-l(cos- 2x ) ·dx -{cos 12x }
1 1
-l(cos- 2x)-. - ( ( ) 2 ·
2

1- 2x
2
(cos-12x )2./l-4x2
X2 • "i+x'2"
1 - tan X. 2 X
-1
..J! [tan 1x
; J
(iii)
dx L x (x)
x-2tan- 1 x .(l+x 2)
x (1+x )
3 2

10. Find� in each of the following cases


(a) y = sin-I (ex) cos-I x
(b) y = e
(c) y = tan-1 (1) (d) y x
2
sin-1 x
cos-1 x
(f) y log X, tan-12x
=
(e) y
(g) y /sin-�� (h) y log(cos-1 f>�
( i) y tan-I (2 tan x) (j) y = 2x tan-I x - loge(l+x
2
)

11. Find the equation of the


(i) tangent to the curve y = sin- 1 f, at the point on
= -1.
it 1-:here x

(ii) normal toy= tan-1(2x+l), at the point on it


where y .. .
11
4

12. Use the results of questions 6, 7 to write down primi-


tive functions for
1
(a) 1 (b) 1 (c) ( )
� l+x2 � d a2!x2
1 1
(e)
h-a2i'.2 (f
)
l+a2x2
[In thfs question a is a positive constant. There are
two po�sible results for parts (a), (c), (e).J
1
Noting that ;:z.:t 1 1
13. (i) zxz-Z lb2(a2lb2 xzr b/fa/b)2=i'2
-
1
write down the primitive function of l.i1=1>2x2

(ii) Similarly determine the primitive function for


1

484
INVERSE TRIGONOMETRIC FUNCTIONS
14. Stat e in defi n ite i nt egrals fo r each of t he followi n g
dx dx dx dx
(a)
I � (b) J 4+x2 (c)
I 1+4x2 (d)
I �

(e) f (25-9x )-:l:dx


2
(f) f (25+9x )-1dx
2

(g)
I dx
16+(x-2)2 (h) I dx
116-(x-2) 2
[There are 2 possible results for pa r ts (a), (d), (e), (h))

(b) P rove

16.
wers
i,

(a) f �
::

rrational o

.

(b) f 9 x2
+
(c)
-k
i�
Evalua te the follow ing defi n it e int egrals, leavi n g

(1-x�) 2dx
ans-

l -k 1
(d) 0 r3(1+x 2 )-ld: (e) j3 11���2 (f) f 1!;x,. n
_l/6 1 _lh
f -1
2/13 dx
4+9x2 (h}
2
f
IJ dx .
(i) f
dx
l -(x+I)2
% _1 !
(g) fl�-9x2
4 4
11
dx dx dx
(J.) f (x-2 2 4 ( l) f
3
'h.
) + 3+4x2
1% .'3-4x2
fi/
2 2
(k)

[ Ce rtai n of the al:ove exe r cises may be do ne by 2 methods,


an d should be done so.]

17. Show tha t

r 19��2 .. [sin-1 �( = sin -1 (�) - sin -1 <t>·


1 {41 ° 49' - 19 ° 28' = 22 ° 21 1 }
· ·3901, fron radian tables.
=

485
INVERSE TRIGONOMETRIC FUNCTIONS

o, ( ,,�:, =-[cos-1 11: = -[cos-1(1)- cos-l (t) J


l +-{ 4 8 ° 1 1 ' - 70 ° 32' = -22 ° 21'}
� •3901 , as before.

l �f�
1 8. Ev aluate the following, giving answers correct to 3 sig-

4.�
niii cant figure s .
2

1o 23
dx dx dx
( a) (b) l+x2 (c) -x
_ f
lI
l
3
dx dx dx
(d ) 16+x2 (e) (£)
-1
4+25�
f
-2 l

1
Sket ch the graph of y •
1 9.
l+x2 for the domain -3 � x � 3 ,
and calculate the are a bounded by the curve, the x axis
and the o rdinate s at (i) x= -13, x= +/3 (ii) x= l, x= 3
1
20. Sket ch the graph of y = �· for the domain lxl < 3,
and calculate the are a b ounded by the curve, the x axis
and the o rdinate s at ( i) x= - 1 •5, x=+l•S ( ii) x=l, x=2

21. 3 4 3x + 4x+7
2
(i) a
1 )(

Jio
)
::::: a:: r '��;� ·,�� :::::� �.::: ,:·:::
ratio nal form.]

(11 ) Show that


d [ 1
dx sin- ( �s inx)
J . cos
{4-s in.: x•
x and hence

(2 COS X
dx,
Jo
evaluate
l4-sln2x
22. (i) Prove that d! [tan-1 1 + J�4J ()!4) 2• and cal­
2

culate the value of f fo rm,


(x�:4 ) 2 in irrati nal

�f
o

))Show that d!["t + sin_;�] • cos-1/1-xZJ, x>O and


,
hence prove that sin-lx • cos-1,li':i'Z",
\

486
SUMMARY - INVERSE TRIGONOMETRIC FUNCTIONS
23. STUDY AND LEARN THIS SUMMARY
( i) --- _Y.
'I= s,,,·' x domain lxl S I,
range lsin-lxl s ¥"
sin-1(-x)=-sin-l(x)

--P.L._J -;,: - . -'


1�
�(s'1n-lx) -
dx 7f-.xZ'
1
-+ 7i"=x2" = s I n • x+ C

1 -+ = sin-1� + C
� sin-1�)
dx a = � va·- x· a
a
-+
dx l. i -
IJ-a2x1 = a s n lax+ C

I
IJ-a 2x'2 I

(ii)
domain I x I :: I;

I range OS cos-lx Sn
c.&6'(-x) I
'-1
I

I
:?' _, -� 0

1��
-+ {I��2 =

-+ = - cos-1 a� + C

�(cos-1ax)
-a dx
dx l1-a2x2 -+ I l1-a2x2

I dx
Ta"l=tzX"2 = - l.b cos-1 bx +· C ·
a

487
SUMM/\RY - ArlALYTICAL GEOMETRY IN 3 DIMENSIO�lS

(iii) _Yr_ - - -- - domafn -oo < x < oo;


= 7o.."'-· )(
y---···-
I<- -;lo.,.;:' x.
range ltan-1 xl <·f
I
�--. --�---i '-----:I---�
-'f'. tan-1(-x) = -tan-1x
-�
0 x. )<
I
d«.;:' (:,:)1
I
-rr

r!x2 -> / 1 �:z = tan-;x + C

-2.(tan-1�) a�+xz �.,/ 1c·


� = J_ tan-1 � + C
dx a ··+x·· a a
a
__Q_(tern- I ax) � 1 <I� .....,.. = J_ tan-I (ax) + C
dx T+alx2" +a•x- a

_!_ tan-1 bx + C
ab a

TOPIC 13

ANALYTICAL GEOMETRY IN 3 DIMENSIONS

S1M"1ARY
I. COORDINATES "P,(;(,.�.•}�
d I
13-,
Cb) Direction of ray OP 1 in space x I / �
is defined by
I /
( i) the direction angles -- - -; __ .;/
where O � a,6,y � -
a,8,y < n
Xl
the' direction cosines cosa
01' ( i i)
Vl n = cosy
d'
m = cosB = .:._
d ,
[Note cos2 a+cos28+cos2 y I]
or (iii) the direction ratios x1:y1:z1
(c) l*P1P 2 I = lcx 2 -x1> +Cy 2 -y1) +Cz2-z1) = D
2 2 2

(d) The coordinates of the midpoint of


the interval P1P 2 are
(ic,,'l.a,l�
t\< x1+ x2l, �<y1+Y 2 l, �Cz 1+ z 2 >J pi
Ce) Direction of ray P1P2 in space I

is defined by the direction of 0

� 1-
SUMMARY - ANALYTICAL GEOMETRY HI 3 DIMENSIONS

__
the para I lei ray through 0. The direction
(i) cosines of P1P2 are .l = co sex X2-Xl

__
m = cos6 = Y2-Y1 , n
D
z2-zl
cosy = -0 -
D

(ii) ratios of P1P2 are < x2-x1): <y2-Y1 >: ( 22- 21 )


( f) The equations for the change of axes to !p,q,r) �ith­
out change of direction are x = X+p, y = Y+q, z = Z+r
(g) Angle between I ines L1, L 2 is L; /L.2
the angle between para! lei �
lines through O � ____....'
cos8 = .l1.l2+m1m2+n1 n 2 9 �L,
(h) The I ines are
0 )1
".(.
---------------

(i) parallel if l1:l2 = m1:m2 = n 1:n2,


or if a1:a2 = b 1:b 2 = c1:c 2
(ii) perpendicular if lil2+m1m2+n 1 n 2 = 0
or if a1 ai+b:b�+c1 c2 = C.
2. EQUATIONS OF LINES
The equations of the line through
(al O and P1 in (i) cartesian form are�= y__ = £_
x1 Yl z1
(iil parametric form are
x = tx1, y = ty1 , z tz1.
(bl P1, in a given direction in
x-xl y-yl
symmetric form are -l- = -m-
= n
( i)
x-x1 y-yl
( i i ) direction -ratio form are -- = - - =
a b C

(iiil parametric form are x = x1+a>.,


z = z1+c>..
x-x2 y-y
< cl P1 , para! lei to the I ine --
x-x1 y-yl z-z1 a = T z-z
= C 2
are -a- = b = c •
(d) P1 and P2 in x-x1 y y z-z1
(i) cartesian form are -­ = - l =
x 2 -x1 YrY1 z2-z1
(ii) ratio form are x = x1 +t(x2-x1), y = Y1+t<y2-Y1l,
z = z1+t(z2 -z1l
where t : I = *P1 P : *P1P 2 .
l· SURFACES
(al Sphere. The equation of a sphere
< i) centre 0, radius a, is x2 +y 2 +z 2 a2
(ii) centre P1 radius a,
is (x-x1 )�+(y-y1 )2 +(z-z1)2 = a2 .

489
SUMMARY - ANALYTICAL GEOMETRY HI 3 DIMENSIONS
(bl The aeneral eauation z = f(x,y) represents a surface
in / dimensio�s.
For exam�le z = x2+y2 is a paraboloid of revolution,
z = 11-x -y� is the curved surface of a hemisphere,
z = 8-2x-3y is a plane.
(cl '.l'he equation f(x,F) = O represents a cylinder in
space.
Thus x2+y2 = a2, (x-x1 )2+(y-y1 )2 � a2 both represent
right circular cylinders; y x 2 is a parabolic
cylinder, etc.
(dl Planes. The equation forms
(i) x = 0 (yz- plane); y = 0 (zx- plane);
z = 0 (xy- plane).
(ii) x = ±a, y = ±b, z = ±c, (Planes para I lei to the
yz-, zx, xy- planes respectively.)
(iii) y = A 1 x+9 1 • (Plane para! lei to the z axis)
(ivl ix+my+nz = p (Plane distant p units from O;
the normal to the plane having direction co­
sines f,m,n.)
(v) Ax+8y+Cz = D. CThe general form of a plane;
the direction ratios of the normal are A:8:C.)
(vi) A(x-x 1l+8(y-y1l+C(z-z1l = 0. (Plane through P1,
and the direction numbers of the normal are
A,e,c. >
(vii) rx+8y+Cz = K, where K varies. (Planes para I lei
to the plane Ax+8y+Cz �.)
(el Condition for the planes
S1 : A1x+i31y+C'1z = 01, S2 : A2x+82 y+C2 z be
A1 81 C1
( i) para I lei is
A2 = 82 C2
(ii) perpendicu I ar is A1A2+8182+C1C2 0
(fl tngle between the
(i) pianes S1,S2 is given by
A1A 2 +81S 2 +C1C2
cos a
f:'12 +81 2 +c12 , IA2 2 +s22+c 2 2
(ii) I ine L1 (v1ith direction ratios a1 :b1 :c1l and
the plane S1 is given by
a1A1+b1S1+c1C1

490
ANALYTICAL GEOMETRY IN 3 DIMEi�SIONS
EXERCISES
1. 0 is the origin and P1 the point (3,2,-6)
Find
(i) t:,e length of ti-.e interval 0P1
(ii) the direction cosines of the
-i- -
ray OP1, and a set of -,
direction ratios for OP1. I
1- G,
(iii) the coordinates of the I
(a) midpoint M of 'OP1'·
(b) point Q on OP1, such that *OQ = l*OPpp, (3,�.-lo)
3 "
(iv) the angle between the line OP1 and
(a) the x axis
(b) the xy- plane
(v) the equations of the
(a) line OP1 in (a) cartesian (S) parametric, form
(b) two lines through O making an angle of 60
°

with the x axis and an angle of 135 ° with the


y axis.
-( c) line through P1 perpendicular to the plane
x+2y+5z = 13.
(vi) the equation of t!1e plane througl·,
(a) P1, parallel to the zx- plane
(b) P1, to which OP1 is the normal through 0.
(c) P2(4,-4,7) parallel to the plane 5x+7y-3z = 8.
(vii) the equation of the sphere, centre 0, radius I *OP1 I.
2. O,P1,P2 are points in space witl. "f.:.(y.,-'f,?)
coordinates (O,O,C), (3,2,-6),
(4,-4,"7). Determine I
(i) the distance from P1 to P217
(ii) the direction ratios, and I 3
0

the direction cosines for L __ _


the line P1P2. _,,_
(iii) the coordinates of the x
(a) midpoint of the interval P1P2.
(b) point dividing the interval -P, (3,.1,-b)
P1P2 in the ratio (a) 3:2 (S) 3:-2
(c) point P2, when the axes are translated to the
point P1, without change of direction.
(iv) the cosine of the angle a, between the lines OP1,
OP2.
(v) the equations of tlie lin� through
(a) P1, with direction numbers 5:1:-8 in paramet­
ric form, and hence find the coordinates of
three points on the line.
(b) P1 and P2.
(c) P2, parallel to the ray OP1.
(vi) the equation of the plane
(a) S1, through P2, if OP2 is a normal to S1.
(b) S2, through P1, parallel to the plane S1.

491
ANALYTICAL GEOMETRY IN 3 DIMENSIONS
(c) S3, bisecting the interval r1 P 2 at right
angles.
(vii) the acute angle between
(a) the planes S1, S3 (Leave as cos6)
(1:,) the line OP1 and the plane s1 (Le.'.lVe as sin ij,)
(viii) the equation of the S?here, centre P1, and radius
l*P 1P2I•
3. A,3,C are points in space with coordinates (2,-1,3),
(1,1,2), (4,2,1). Show that triangle ABC is right
angled, and hence detennine its area.
4. Show that
(i) the three points (-2,3,5), (1,2,3), (7,0,-1) are
collinear.
(ii) the join of the points (1,2,3), (4,5,7) is paral­
lel to the join of the points (-4,3,-6), (2,9,2).
(iii) the line through the points (1,8,4), (4,5,-2) is
perpendicular to the line through the points
(O,-ll,4), (2,-3,1).
(iv) the point of intersection of the lines joining
A(6,-7,0) to B(l6,-19,-4) and C(0,3,-6) to
D(2,-5,10) has coordinates (1,-1,2).
5. Express the equation of the plane S1 : 2x-2y+z = 6,
in the form ix+r,1y+nz "' p, and hence determine the
perpendicular distance frorr, O to the plane, and
the direction cosines of the normal to S1 .
(i) Determine the equations of the line through
D(S,-3,-4) �erpcndicular to S1, and the coordin­
ates of the point where this line meets the xy-
plane.
(ii) Find the equation of the plane through D parallel
to the plane S1. Does t·he point (3,-4,-1) lie on
this plane?

6. Find the value of k for the


x-1 � z-4 x+3 :t.=]_ .. z+l
(i) lines = = = to be
4 k+l 6' 3k 9/2 9
(a) parallel (b) perpendicular, to each other.
(ii) planes kx-3y+5z = 8, 4x-6y+lo·z. = 3 to be
(a) parallel (b) perpendicular, to each other.
7. fletermine the angle between the
x+3 J.::!! z-3 x-2 l'.±.L_ z
(i) lines = = ,
-1- 1 -2- . 7f.:I ,. -l!-1 "' 4
(ii) planes 2x-y+z "' 6, x+y+2z = 7.

8. Express t!1e equations of the line passing through the


points (-1,-3,2), (2,6,-4) in parametric form, and
he'nce determine the coordinates of the point of inter­
section of this· .line and the plane 3x+4y+5z s 5.
492
ANALYTICAL GEOMETRY IN 3 DIMENSIONS
9. Sketch the following surfaces
(a) 3x+4y+2z = 24 (b) z = 2 (c) IYI = 3
(d) y = 5-2x (e) x 2 +y 2 +z 2 16 (f) z ·/16-x2-y2
0

(g)_ = 25 (h) y z -x
x 2 +y 2 2
(i) (x-3)2 +(y-2) 2 = 49
(j) z = x 2 +y 2 (k) x 2 +y 2 +z 2 9, z = 2 (l) x=2 and y=3
(m) x = 2 or y = 3.
10. Sketch the following regions in space.
(a) 3x+4y+2z � 24 (b) x 2 +y2 +z2 < 16 (c) 05z5il 6�x2-y2
(d) x +y2 < 25, l<z<2
2
(e) x 2 +y 2 +z 2 5 16, lzl 2: 2
(f) x > 2 and y < 3 (g) z 2: x 2 +y 2 , O 5 z � a (a> 0)
(h) y < 5-2x, z > 0 (i) y < -x 2 , - 2 < y < 0
11. If A = {(x,y,z):!xl 5 l}, B = {(x,y,z) : !YI 5 2 },
C = {(x,y,z) : lzl .!: 3}, sk�tch the region defined by
A n B il C.

12. Use the general form Ax+By+Cz = D for the equation of a


plane, to do the following exercises. Find the equation
of the plane
(i) through the point (1,1,1) and perpendicular to the
planes x-2y+z = 2, 4x+3y-z+l = O [Hint: show that
A+B+C = D, A-2B+C = O, 4A+3B-C 7 O, and hence find
the values of A,B,C in terms of D.J
(ii) through the points (-1,1,1), (1,-1,1) and perpen­
dicu l ar to the plane x+2y+ 2 z = 5.
(iii) through the points (1,1,2), (-4,1,3), (-1,-3,4)
(iv) through the origin and containing the line
x l Y 2 z
; = � = =�· [Hint: first find two points on
the line.]
(v) through the points (-2,1,-3), (4,3,3) and parallel
to the join of the points (3,2 ,-5) and (0,-4,-11),
13, Find the equation of the plane through the line of
intersection of the planes x+2y+3z = 4, 2x+y-z = -5 and
passing through the point (1,-1,0). [Use A technique.]
14. A(2,2,-l), B(3,4,2) are fixed points in space, and
P(x,y,z) is a variable point, Determine the locus of P
if (i) P is equidistant from A, B.
� nc
(ii) *APB = .
2

493
TOPIC 14
A. VELOCITY AND ACCELERATION AS DERIVATIVES
1. Tile dispiacement x cm ox
o f a particle from a·�---�1�
X
----�---...,,.L..-.
---- X
,r- flQ
X
.._��
fixed point O a fter time t sec,
0 is
given by the formula
x • f(t). Show that the displacemen t (x+ox) cm, from O a fter
time (t+ot) sec is f(t +ot) CM and the average velocity for
the ot sec at the end o f t sec (i.e. over the int erval PQ in
f t)
the sketch) i s �� =
f (t+ot
�� ( cm /sec.

Tilus the instantaneous velocity v cm /sec a t the end o f


v = �!:O (��) �!:O { f(t+o��-f(U} = :: = x
s s
� s =��:� by
X
2. I f the particle passes p Q

the point P x' o - - X


"1 ,,.,.,Jsec (v+Sv) t.9'\140.c.
(where *OP = x cm) with velocity v cm./sec, and over the
in terval PQ in the sketch, it increases it s velocit y to
(v+ov) cm/ sec, show that in the time interval ot sec in going
from P to Q, the average acceleration of t he particle is
ov ov
6t cm'/sec per> second, i.e. 6t cm /sec 2.

Hence the ins tantaneou s accelera tion (a or f cm/ sec� at


the end of t sec, is given by

lim ov dv d dx = �d2x
a (or f) ( ) = ( dt�
= X
6t-+O ot "' dt dt d t )
)(+, ...I
dv dv dx = -d (
3. Prove tha t -=
dt dt v v = ..J! (\v2) I
dx dx dx I

4. A s tone is thrown upward s into the air


from the top o f a tower. The height
x m of the s tone . from the ground t sec
after it is thrown is x = 400tl20 t-16 t 2. _............_..._...,,__�
(i) Find the height of the tower.
(ii) Find the displacement of the stone from O a fter
1 sec, and a fter 3 sec. Hence, determine the
average velocity for these two seconds,
(iii). caiculate the average velocity for the first 3
seconds of motion.
(iv) After what t ime is the stone 176 tJ' above the top
of the tower? Explain the two answers .
(v) When does the stone return to its initial height?
(vi) Write down the displacement of the stone from O
after (t+6t) sec and hence determine its average
velocity for the 6t sec at the end of t seconds.
From this, deduce the instantaneous velocity
v m/ sec of the stone at the end of t sec.
494
VELOCITY - ACCELERATION; DERIVATIVES
(vii) How fast is the stone going after 1 6econd and
after 3 seconds? Hence calculate the average
acceleration over these two seconds.
(viii) Find the initial velocity, and determine the aver­
age acceleration for the first 3 seconds.
(ix) Write down the velocity of the stone at the end of
(t+ot) sec, and hence determine its average accel­
eration for the ot sec at the end of t sec, From
'this, deduce that the instantaneous acceleration
a m/sec 2 of the stone at the end of t. sec, is
constant.
(x) What height above O does the stone reach, and
after what time?
(xi) Find when and at what speed the stone hits the
ground.
(xii) At what height above ground level is the stone
moving at a speed of 56 m/sec in the
(a) upward direction (b) downward direction?
(xiii) Sketch on separate axes, the graph of
(a) x against t (8) v against t (y) a against t
(xiv) Determine the equation of the tangent to th� graph
of x = 40o+l20t-16t 2 at the point where v = 88.
(xv) What is the geometrical significance of v and a in
the graph of x against t? What geometrical mean­
ing would you attach when v = 1, a c -32?
(xvi) From the sketch of v against t, determine the
greatest and least speeds of the stone during the
motion.

5. A particle Pis moving in a· straight )(

line so that after t seconds, from o X


the start, its displacement from a fixed point O is
given by x = t 3-6 t 2 +9t+56 ill
(i) Write down expressions for the velocity and acceleration
at the end of t seconds.
(ii) Find the velocity and acceleration of P after 4 sec,
(iii) Calculate at what time the particle changes direction,
and its distance then from o.
(iv) Determine the velocity of Pwhen its acceleration is
zero,
(v) For what range of values of t, is the particle moving
(a) tcwal'ds O (B) (Zl;)ay from 0
[Hint: sketch the v - t graph]
(vi) For what values of t is the acceferation
(a) positive (B) negative?
(vii) Find the acceleration of P when it returns to its start,,.
ing point,
(vjli) Use the above results to sket_s:h the graph of
x • t 3-6t 2 +9t+56, and find the equation of the inflex­
ional tangent to the curve.
495
VELOCITY AND ACCELERATI0N; DERIVATIVES
(ix) Use the x - t graph to find the furthest distance of the
particle from O for the time interval O � t S 5.
(x) Sketch the velocity-time graph, and use it to find the
maximum and minimum velocities of P for O � t !=. 5. What
is the greatest negative velocity of P?
(xi) On the v - t graph; shade the area representing the
distance covered by P for the fifth second, and ev.aluate
this by Calculus. What is the average velocity for the
fifth second? -·
6. If x = at+bt 3, and at time t = 4, the velocity is zero
and the acceleration is 12, determine the values of the
constants a and b. Also find the initial velocity, and
the average velocity for the period t = 2 to t = 4.
7. A train runs non-stop between 2 stations
A and B, and its velocity t hours after A B
leaving A is (60t-30t 2 )k�/h. Find the (i) time of the
journey from A to B (ii) distance between A and B
(iii) average velocity for the journey
(iv) maximum velocity attained for the journey.
8. With the usual notation, the displacement-time equation
of a particle moving in a straight line is given by
x = t sin t +cost, Find
(i) at what times in the interval OS t $ 2n, the
velocity is zero, and determine the corresponding
accelerations.
(ii) the initial displacement, velocity and acceler­
ation of i:he particle.
TT
( iii) the average velocity for the interval O -< t -<
2
9. If x-= a sin 3t +b cos 2t, and when t= O, x = 12, �= -12,
find the values of the const ants a, b •
Hence find x, x and x when t ..!!.
6
t
10. Given that x = Ae 2t + Be-2 , prove that the acceleration
f = 4x.
dx
If x = 12, dt = 4, when t = O, find �he values of A, ·e.
dx 5
Hence prove that = O, when t = \ log ( ), and show
dt e 7
that f; 0 for real values of t.
11. The displacement xcmof a particle P moving in a straight
l+t
line at the end oft seconds is given by X = log(--)
cost
TT
where O < t < . 2 Show that the velocity (v cm./sec) is
given by v =� +tan t, and hence derive an expres�ion
+t
for t he acceleration (f cm/sec 2 ) in terms of t, Find the
initial displacement, velocity and acceleration of P.

496
INTEGRATION - VELOCITY AND ACCELERATION
8. l�HEGRATION PROBLE.f1S ON VELOCITY AMO ACCELERATl·ON
1. Tile veloc'.ty of a particle . moving in a straight line is
giyen by
(i) dx = 6t-S. \fuen t=2, x= 4. Find its displacement when
dt
t = 1.
(ii) x .= 1 - 2 sin t, l'lhen t = O, x = -7. Find the value of
x when the particle is first at rest.
dx t2 +1
(iii) - =- --. l·:hen t = 2, x = S + leg 2; !'ind the value of
dt t 2
x when
d x =
W 0.
dx = t - -t ,,,_
(iv)" e e , l,:uen t = o·, X ·= o . :1 •
cetermJ,.ne " Va 1 UC 0f
t.-e
dt
d2 x
(� - x), and show this is independent of t.

(v) v = sec t - 4. When t =


2
f, x = 2--rr. Find the value of x

when v = 0 for the first time.

2. The accelera tion of a particle moving in a straight line


is

(i) dt2
d2 x
= 6 t--6. When t 1, X 3,
dx
= 3. dt
Find its posi-

tion when t 2.
(ii) dt
dv =
2. When t = O, x = 6 and when t = 1, x = 2. Cal-

culate the value of v for each time when x o.


(iii) x = 6(2t2 -1). When t = 1, x -6, x = --2. Determine
the value of; when x = O.

d2 x =
(iv)
W - 4 sin 2t + 9 cos 3 t , !.'hen t = o, x -1, : � 2,
Find x when t =
6.
(v) dt
dv "'
4t- + 3t-l
2
+ 2t. When t 1, V = -j, Find the

value of v when t = 2.
d2 x = -7 dx 3
w
t
(vi) (e -- 1)2 . \,'hen t = O, x =
4, dt
=
2. Find an

e.xpression for� in terms of e when t = 1,

3, A particle Pis projected from the point O along the


s traigh t line X'OX with ini tial velocity u :cm/sec in the
posi tive direction. If the accelera tion of P is
a ·cm/sec 2 , where a is cons tan t, prove by integra tion
that after t seconds, when the displacemen t and veloci ty
of Pare x cm, v •c(IJ./sec respectively, then

(i) v = u+a t (ii) x- = u t + !-,at2 (iii) v2 = u2 +2ax

1ior (iii), star t !(l,v ) = a]


2
d

4. A ball falls from rest from O towards the


ground. The equation of motion of the
ball is given by • f 32 - k t2 , where
flltsec 2 is the accelera tion of the ball ___...___�
after t sec, and k is a cons tant. After seconds, the 13
497
VELOCITY - ACCELERATION; INTEGRATION
acceleration of the ball is 8'/sec 2 ; find k, and deter­
mine the velocity and displacement of the ball after
2 sec.
5. The f, t equation for the particle is given by
f - a n 2 sin nt, where a, n are constants. If v an
and x = 0 when \: � 0, find the x, t equation for the
particle.
6. r:1e function v = 4 cos 2t represents the velocity
v cmifsec of a moving body at time t sec. Sketch ths
graph of v versus t, for the interval O � t � rr, and
find
(i) that portion of the time interval in which the vel­
o�ity is positive.
(ii) the total distance travelled by the body between
t = 0 and t = rr.
7. ft particle moves in a straight line X'OX starting from 0
with a velocity of 4 m per sec. At time t its accelera­
tion is 2(3t-4) rn per �ec 2 Find
(i) how far the particle moves before first coming t�
instantaneous rest.
(ii) the total time that elapses before the particle
returns to its starting point.
(iii) the greatest velocity of the particle in that in­
terval.
3. A car is travelling along a straight road, When it is
passing a certain position O the engine is switched off.
At time t sec after the car has passed O the speed v m/s
1
is given by the formula v = A+Bt, where A, Il are posi-
tive constants. Show that the retardation is proportion­
al to the square of the speed, If when t = O, the
retardation is lm/sec2 and v = 80, find A and B. If x m
is the distance moved from O in t sec, express
(i) x in terms of t *(ii) v in terms of x.
9. The velocity-time equation for a particle moving in a
straight line is :� = cos 2 2t + sin 2 tt + tan 2 3t,
Initially, the particle is at the origin, Determine the
value of x when t "' rr. [Hint: use the relations
cos 2 e • \(1 + cos29); sin 2 e = �(1-co�29) and
tan 2 e = (s�c 2 e-1))
��-'--'--��������������

10, (i) Find �(/t+4) and :(t sin t + cos t)


d d

-
(ii) If:�= /t!4 + t cos t, and when t = 0, x = 2, find
x when t "' rr.
p
11. A particle P moves in a strai�ht line
under the influence of a retarding 0
force which acts so that the law of

498
� = g(x)
dv
motion is = - kv where k is a constant, and v is
dt
measured in cm/sec, and t in sec. [ This result corres­
ponds to the exponential law of natural decay.]
-kt
Prove that v = Ae , where A is a constant.

The particle starts from the origin with velocity


10 cm/sec, and after � sec its velocity is (10/e) cm/sec
Find the values of A, k, and determine the formula con-
necting v, t. Find
(i) at what time Pis moving with half its original
velocity
(ii) the velocity of P after 2 sec
(iii) the formula connecting x and t, where x is the
displacement of P from Oat time t
(iv) Use the formula of (iii) to find how far the par­
ticle is from O after 1 sec, and when it is 4 cm
on the positive side of 0.
(v) What is the greatest distance from Oreached by P?
(vi) the formula connecting v and x
[Hint: dv = v dv ]
dt dx
(vii) Use the formula of (vi) to find how far P is from
0 when the velocity is 3 cm/sec.

12, A body P moves in a straight line.,.,..,..--1l;::::===x====3'11���


"1 �
tCMards a fixed point O , being x• o
initially at rest 40 m en the positive side of 0. P
moves under the influence of a force which produces an
acceleration towards Oof l00/x 2 m/sec2 where x is the
displacement of P from O. Find the velocity of P when it
is halfway to O. [Hint: f = -100x- 2 • �(�v 2 )]
13. A particle P moves towards a fixed point Oin the line
X'OX under the influence of a force, so that its accel­
eration f cm/sec 2 towards O and its displacement x cm
from Oare related by the fact that
A (i) f varies direatly as x, � +
---1:1====•*----+J
i.e. f = -kx X.' 0 P X.

(ii) f varies inversely as x, i.e. f • - ls.


X
When P is 1 cm to the right of O , its velocity is
4 cni/sec and its acceleration is 8 cmy sec 2 • Find at
what distance from Oit started from rest.
[Hint in (i), first show v2 = 8(3-x2 );
and in (ii), prove v 2 = 16(1-logx)].

]! (i) f varies direatly as the cube of x, i.e. f = -kx 3


(ii) f varies inversely as the cube of x, i.e. f=-k/x 3 •
Initially P starts from res·t 2 cm to the positive side
of O, and is then accelerating towards Oat 24 cm/sec 2.
Determine its position when its velocity is 4 cm/sec
towards O.

499
d2x
W = g(x)
14. The acceleration of a particle P
travelling - along the line , r x -+ , ,..
x 1 Ox varies inversely as its·.x. � 1 b P x
distance from the point Q on the line 2 cm to the nega­
tive side of O . Initially, it is at rest at O and is
undergoing an acceleration of 1 c:m/sec. Determine its
velocity after moving 20 �m . [Hint f = k/(x+2)],
15. A particle P travels in a straight�l;;==x==�� ==== t• -��
a.
!:-;;- =::; ��
"';- x
line x'Ox so that its accelerationO�
is directly proportional to its distance from the fixed
point Q, distant a from O. [Hint: f = k(a-x)].
At O its velocity is J m/sec and its acceleration is
2/5 m/sec 2 • When it has travelled 10 m towards Q, its
acceleration is 3/10 m/sec 2 • Find the distance
travelled before the velocity is a maximwn and the value
of this maximum velocity,

16. (i) For a particle P (initially at the origin), with


2
the usual notation, we have Prove that
V = l+Jx'
(a) f varies as v 3

(b) the time t for p to travel a distance X is


given by t = \(2x+3x2 ),
(ii) For another particle Q (also initially at 0) the
corresponding equation is v = (l+Jx)/2. Prove that
(a) f varies as v
(b) the time t for Q to travel a distance x is
given by t = 3
2
log(l+Jx).
(iii) For both particles P, Q above, find x when t 2.
17. A rocket projected from the earth,
when put of the earth's atmosphere,
has a retardation of approximately
91000/x 2 k m/sec 2 , where x K�
is the distance from J::.l:le _centre. of
the earth, If the rocket is moving
at 4 k r../sec after it is 6 00 k m above the earth's
surface, find its speed after a further 1000 k'.m. Find
also the total distance travelled before first coming to
rest. [Take the earth's radius as 6400 km l
*18. A particle P moves from rest at a distance a towards .,
centre O of a force which produces an--------
acceleration µ2 /x 3 , where µ is a .-p
positive constant and x is the distance from O to P.
1 -12) and hence that -dt = - r?=::-2
ax
Prove that v2 = µ2(-2
x a dx µv'a--x
Find the derivative of /a2-x2 with respect to x, and use
this result to prove that the time for P to reach O is
a2 /µ.

500
SIMPLE HARMONIC MOTION
C.
= SIMPLE HARMONIC MOTION
1. (1) A particle P is executing simple harmonic motion
about a point O in a straight line, according to
the law x= -n 2x. If initially the particle is at
0� and when at d.istance a to tlie right of O then P
is at rest, prove by integration that v 2 an 2 (a 2 -x2 )
and that x = a sin nt. Prove that the time of a
complete oscillation is 2n/n.
[These results are used in (ii))
(ii) If when P is5..mm'to the right of O, its speed is
6 cm /sec, and th� acceleration towards O is
128 cm/ sec2 , find the time of a complete
oscillation and the amplitude. D.etermine also the
least· time in • hich the· particle P can move from a
1
point distant3aUllllfrom O to a point at the same
distance from the centre but on the opposite side
of it,
2. A particle P can move in a straight line OX in which it
is attracted towards O by a force producing an accelera­
tion n 2x, where n is constant and x is the distance of
the particle from O.
Show that the equation x'= A cos nt + B sin nt, where
A, B are constants, satisfies the equation x
-n2 x.
(a) The particle is held at rest at a point P in OX
where •OP= k and is then projected in the direction
Ox with speed nk. Prove that A = B = k.
(b) Find an expression for the time taken by the
particle to reach O for the first time, and calcu­
late the velocity of P then.
3. The particle P moves according to the equation
x= a cos(4t+e:), where a,e: ar·e constants. [a>O, O!>e:<2n)
Prove that P executes simple harmonic motion about a
fixed point. When t O, x= 4 and when t= n/8, x= 2.
Prove that 4·= a cos e: and 2 n
a cos( + e:) = - a sin e:.
2
Hence show that a 2,5,r. cos e:= 2 , sin e: = - 1 · When
f,
7s 7s
t= x
prove that x·= /3-2 and = -16(/3-2).
4. A mass is vibrating in simple harmonic motion from A to
B, and back to A in 4 sec. •AB = 10 m , A O 5
If the egua tion of the motion of the 10.,.. _
mass is ·x = -n 2.x, write down the period n
of oscillation of the mass, and prove n = . Prove that
2
the velocity v m/sec of the mass is given by
V = t 2!_ {25-x2 , and show that its speed when it is
2
passing through the point C, 2\m from A, is 6·8 m/sec
approximately. What is the acceleration of the mass when
it is at B? [Take n as 3·14 and n 2 as 9·87.)

501
SIMPLE HARMONIC MOTION; HOOKE'S LAW
5. A particle P moves in S.H.M. according to the law
x • a cos(nt+e), where a,n,E are constants (a>O, O�eSn).
The period of oscillation of Pis 4 sec. At time J� sec
after the motion has started, Pis at its mean position
O and 1\ sec after Phas passed through O in----. it is
moving with speed 2 m/sec. Show that the amplitude is
4f'2/n .m, and find the lll!iximum and initial veloci­
ties of P .
6. A particle moving in a straight line is attracted to­
wards a fixed point O by a force producing an accelera­
tion n 2 x towards O, where x is its displacement from O
� ••d n is a constant. By integration, find the velocity
as a function of x, given the initial conditions v • vo
at x • O. A lso find the equation giving x as a function
of time for this motion.
A particle having such a motion has a velocity of
8 cm/sec towards O when it is distant 1•5 cm to the
positive side of O. The period of the motion is n/2 sec.
Find the velocity at O and the maximum displacement from
o.
7. A particle Pis initially distant b on the positive side
of a fixed point O and moving with velocity V towards 0.
When OPax, the acceleration of Pis n 2 x towards O,
where n is a positive constant. Show the time for Pto
1 nb
reach O the first time is ; sin -1 (fil+ill What is
ol).
this time for the case when the velocity is nb instead
of V?

D. HOOKE'S LAW
1, A uniform elastic spring AB of A a. o x 1'\.
natural length a lies on a TB
smooth horizontal table. A mass m is attached to the end
Band the spring is stretched a distance x, and allowed
to oscillate.
(1) Show that the equation of motion of the maAs m is
given by mx • -kx, and noting k • )./a where). is
the modulus of elast:icit:y of the spring, prove
.,.
am
X • - - X.

(ii) Inltially the particle is drawn a distance xo to


the right of O and is projected CQ.Jay from O at a
velocity� xo, Show that. the speed in
position xamis /:a (2xl-x 2 ).
(iii) (a) Express x in tena.9 of t.
(b) What would be the corresponding expression for
.x in terms of t, if the particle were in­
itially projected towards O not CTJ,Jay from O?

502
HOOKE'S LAW; PROJECTILES
2. A light spring of length land modulus :>. lies on a
smooth horizontal table, One end of the spring is fixed,
and to the other is fastened a mass �t. If the spring is
compressed until its length is � and then released,
show that
(i} the period of oscillation is 2nl.1-l/:>.
(ii) when the length of the spring is 5£./4 the ve-locity
>
of the mass is /3:>.l/16M.
[Hint: first prove Mx = -:>.x/£., v 2 = .ei�<f' -
x2)
). One end of a light elastic string, of natural length £.
and modulus:>., is fastened to a fixed point O on a
smooth horizontal table and a particle of mass m is
attached to the other end. If the particle is pulled
along the table until it is at a distance 2l fro1n O and
then released from rest, find the time that elapse, be­
fore the partic1e first reaches its equilibrium position.
[Hint:..
prove x
:>.
= - Cm p�(4£.2 -x2 ),
x, v2 = .un t 'l.if
= /� cos-1(2�))

4. One end of a light elastic string of natural length a


and modulus mg, is fixed at a point O on a smooth hori­
zontal table; to the other end is attached a particle
of mass m. Initially the particle is held at rest on
the table at a distance (a+b) from O. Show that, if the
particle is released, it reaches the equilibrium
position after time
[Hint:
g i A,
prove x = - � v = .&[ (a+b) 2-x2) t = � cos-1 (__lC_)),
2
a' a • /g a+b

ii. PARABOLIC MOTION UNDER GRAVITY, PROJECTILES

!, A particle is projected with a velocity


of 25 m/s in a direction making an
angle a = tan- 1 (3), wfth the horizontal.
Assuming that x z O and y = -g, where g
is the acceleration of the particle due
to gravity, obtain the following results by fntegratfon.
(f) x z 25 cos a, j = 25 sfn a - gt
(ii) x z 25t cos a, y = 25t sin a - Jsgt2
Taking g = 10 rn/s2, show that

4t
(f) ft rises to a vertical height of 225/8 m
(ff) its time of flight is about sec, and its range is 37� m
exactly
(ffi) after lllf sec, its direction of motion is inclined at 1350
to the horizontal, and its velocity is 5,15" rn/s.

503
PROJECT! LES
0
2. (A) An aircraft flying horizontally
at an altitude of 2880 m above
sea level releases a bomb. The
velocity of the plane at the
moment of release is 504 km/h oliio-wt
(=140 m/s); (this is assumed to
be the initial horizontal
velocity of the bomb). Take
g =10 m/s 2. A
Prove that after time t, the �osition P(x, y) of the bomb
is given by x = 140t, y = -St
Hence determine the
(i) time for the bomb to reach the ground
(Ti)
� horizontal distance of the point of release O from
the point of impact Q.
(B) The aircraft then reduces altitude to 1620 m and continues
to fly horizontally but at a different speed to previously.
It releases another bomb, this time aimed at a target (on
the ground) whose horizontal distance from the plane at
the moment of release is 2700 m. Calculate
(i) the time for the bomb to hit the target
(Ti) the velocity of the plane at the moment of release.

3. A golf ball is to be hit from a point· 0 so that it rises at an


a�9le of 30 ° to the horizontal. Take g = 10 m/s 2.
(A) In its descent it just clears a wall 11� m high, and
takes 4� sec from the moment of projection to clear the
wall. Calculate the
(i) initial velocity
(Ti) greatest height reached by the ball
(TTi) horizontal distance from the wall to the point of
� impact of the ball with the ground.
(B) The initial velocity of the ball is 50 m/s and the ba11
lands on a horizontal green 20 m above the level of the
point of impact. Calculate the
� So�I� ----
(i} hori zonta1 di stance of the
point of impact from the
point where the ball lands;
(jj_) time of flight of the ban . J..:.�--------l� ;,c.

4. A particle is projected from a point O with a speed of 35 m/s


at an angle a (where a = tan-1 4/3) to the horizontal. After
t sec it is at a point Q and is rising at an angle of 450 with
the horizontal. Find t and the vertical and horizontal
distances of Q from 0. (Take g = 9.8 m/s 2)

504
26, 36, t RESULTS
TOPIC 15 : FURTHER TRIGONOMETRY AND CALCULUS

A. Revise carefully the (a t 13), 26, 36, t resu Its


summarised at the beginning of Chapter 4.
11 -1 and O < y < 11, find
l. If tanx = 2 and O < x < , tany • """"'j"
2
without tables, the value of
(i) tan2x (ii) tan3y (�ii) tan y/2 (iv) tan(2x+y)
(v) x+y (vi) sin2y (vii) cos2x

2. If tan(a-13) = 5 and tan-6 = �. determine (without taJ. .?S)


3
the value of (i) tana (ii) tan2a (iii) sin213.
3. Two acute angles a and 13 are such that a + 13 , and
4
tan a = �. Hithout tables, prove that tan 13 = 1 . Hence
3
calculate the value of
(i) sinl3 (ii) sin213 (iii) sin313 (iv) sin413
4. Without tables, find the value of
(i) cosx when cos2x = 81 , and x is acute
6 4 311
(ii) tan 2 when tan 6 = 3 , and 11 < 6 <
a 3 2 -3
(iii) sina when tan = (iv) cos213 when sinl3 =
2 4 s
-5
(v) sin26 when cos6 = and 0 ° < 6 < 180 °
13
(vi) cos31jl when cosljl = \.

s. In the resu lt, tan6 = 2�tan26/2, let tan6 = 1, and show


l tan 612
that tan22� 0 is one root of the equation t 2 +2t-1= O, and
hence obtain tan22� 0 in surd form, Explain the other
root of this equation.
6. Find expressions (as surds) for sin75 °, cos75 ° ,
A 1 -cosA
Prove that tan 2 =
sinA
and deduce that
0
tan37� = 16+13-fi-2.
Show that tanl5 ° = 2-/3. Prove that tan7� 0 is one root
of the equation t2 +2(2+/J)t-l = O , and show that
i.: i.:
tan7� 0 = (2+/3) 2, {2-(2+/J) 2}, Explain the significance
of the other root of this equation,
8, If 6 is acute, prove that (i) sin ! = /l-cos6
2 2
(ii) cos ! = /l+cose
t:'"
(iii) tan ! = Ji --c-o-s"""a-
2 2 2 /i+cose
Use these results to find sin a , cos a , tan a
2 2 2
when cos6 = 12•
13
9. In ti. ABC

a::.
l
(i) tanA = �. tanB = . Find without tables, tanC
3
*(ii) *A= 3.*B. Prove that cos2B =

505
20, 30, t RESULTS

10. Now work Set 4B, question 2 (xiii-xviii) and question 3,


[These were left until this stage for revision purposes]

Prove the following: (questions 11 - 15)


sin39 cos39
= 2cos20cosec20
11. (i) cos0 + sine
sin40-cos40tan30
(ii) = tan0
cos40+sin40tan30
12. (i) (l-sin20)(1+2sin20) = sin20+cos40
(ii) 2sin20-sin40 = 8sin 3 ecose

13. (i) cosec2A=cot2A+tanA=\(cotA+tanA)


l-sin2A
(ii) tan(.! - A) =
4 cos2A
l+tan20tan30 = cos 2e
14. (i) l+tan0tan30 cos22e
2tan20 4tan0-4tan 3 e
(ii) tan4e = =
l-tan220 l-(itan20+tan4e
sinx+sin2x cos2x+sin2x-l l-cos4x
15. = tanx = =
l+cosx+cos2x cos2x-sin2x+l 2sin2x+sin4x
16. (i) Show that tana+tan(a+n/3)+tan(a+2n/3) = 3tan3a,
_
(ii) Prove that cosB+sinB _ cosB-sinB - 2tan2B,
cosB-sinB cosB+sinB
(iii) If 2A+B = n/4, prove that tanB = l-2tanA-tan A
2

l+ZtanA-tan2A
17. AB is a diameter of a circle centre O and C is a point
on the circumference such that the an�le AOC is acute.
'oc meets the tangent at A in D, If CBD = ,Ji and ABC = a,
prove trigonom����ca_!!y, that tan(e+,J,) = �tan20.
Expanding both sides of this result, sh.ow that
tan,J, = tan 3 0, Calculate the val� of ,Ji when *AD = *AB.
18. If 01,0 2 are roots (not in the same quadrant) of the
equation tan20 = c, show that tan01,tan0 2 = -1,
19, If a,B are two values of a satisfying the equation
acos2e+bsin20 = c, and such that tana, tanB are unequal,
prove that (c+a)tan2 e - 2btane+(c-a) .. O. Hence show
that (i) tana+tanB = 2b/(c+a) (ii) tanatanB'" (c-a)/(c+a)
*(iii) ctan(a-�±/a2+b2-c2 ,
20. Prove, using the t results, that
2t l-t 2
(i) sin0 '" l+t2 (i1.") cos0 = l+t2
3t-t 3 l-3t 2
(iii) sin30 '"' (l+t2) 3,'z (iv) cos30 = (l+t2) o/2
where t .. tan0/2
506
TRIGONOMETRIC EQUATIONS� DERIVATIVES
B. TRIGONOMETRIC EQUATIONS
[Note: Take the working only as far as you need.]
1. Work Set 4D, question 3.
2. Work ·set 4F, quest ions 1(A-I), 2 (ii), 3( iii)
3, Work Set 4G, questions 3,4,9,

£:_ DERIVATIVES OF sec x, cosec x, cot x


1. (i) Verify the results for the deriva(ives of sec x,
cosec x, cot x by treating each as a quotient of
two functions.
(ii) Prove that .4Y = y sec x for y = sec x + tan x
. dx
(iii) If Y = (tahx+secx) m

f
and m is a positive integer,
prove that = my sec x.
2. (i ) If y = cotx, prove that f + y 2 +1 O
*(ii) Given that y = cosec x + cot x,
prove that fi + y .4Y = o.
W dx
3. (i) Determine the equation of the tangent to the curve
y = sec2x at the point where x = 11/6, and find
where this tangent cuts the y axis.
(ii) Find the equation of the normal at the point x=11/4
on the curve y = cotx, If this normal intersects
the x axis at P, calculate *OP, where O is the
origin.
4. (i) If f(x) = tanx, O 5 x < f,
show that f' (x) = sec2 x,
f" (x) = 2sec2x tanx, f'" (x) = 2sec'-"x+4sec2 x tan 2 x.
(ii) Find the first three derivatives of sec x,
5. Equate the derivatives of both sides of the .identities
given below, and show that we obtain the identity indi­
cated.
(i) sin2x = 2sinx cosx + cos2x cos 2x-sin 2 x
(ii) cos2x = cos2 x-sin 2 x + sin2x 2sinx cosx
(iii) cos2x+sin 2 x = 1 + 0 = 0
(iv) sec2x = l+tan 2 x + sec2 x tan x = sec2 x tan x
(v) cosec2x = l+cot 2 x + cosec2 x cot x = cosec2 x cotx
6. X dC
(i) If c = cot , show that = L( c 2 )
2 e dx -'2 1+
Noting e =- 1/ x' prove that dx
dx d -2
= + z
d d de l c
*(ii) Show that if f(x) = cotx, then
lim {j(x+h)-f(x) a lim Lcos(x+h)sinx-sin(x+h)cosx}
]
h + Ot h h + Ol h sin(x+h) sinx .
.. lim f .;-sin(x+n-x)_l and hence find
h + O\tsin(x+ h)sinx}' f'(x)
7. Now do Set 4I nos. 7,8,9.
507
TRIGONOMETRIC INTEGRALS
Q, TRIGONOMETRIC INTEGRALS
1, Verify the following results (constants omitted)
(a) J sinx dx • -coax (b) J cosx dx • sinx
(c) J sec2x dx • tanx (d) J cosec2 x dx • -cot

J J
x

(e) secx tanx dx • secx (f) cosecx cotx dx = -cosec

I I
x

(g) tan 2 x dx = (sec2x-l)dx = tanx-x

(h) I cot 2x dx = I (cosec2x-l)dx • -cotx -

J
x

(i) sin 2x dx • �(l-cos2x)dx • k2[x - ---


sin2 J
I 2
x
---

(j) J cos 2x dx c J�(l+cos2x) dx = �[x + si� ]


Zx

2.

(a) f:
Evaluate the following

secx tanx dx; f�


"B"
cosec2x cot2x dx;
f!.
TT

2
cosec ) X dx
2

3TT
J cos 2 (x-a)dx; Jatan2(ax+b) dx i

( )
b f6 in� d "i-cot� dx
a

,r 2
-TT/6
5 l x,
0
TT
0

(c)
f!.(si°2a + co5z)a de;
3 2
J2(2cos 2x+3sin 2x)dx;

2 x and hence find the value of


0 O
(i) Simplify t x ���
1-
TT
3.
3 l-cos 2x
f -dx
tan2x
TT/6
(ii) Determine constants k,£ such that cos 30=kcos3e+£cos0
for all values of 0,
Hence obtain the value of �cos 3 x dx,
I!.
4. Show that
(i) �{log(secx+tanx)} a �log(secx-tanx)-1} = sec x
d
(ii) {log(cosecx-cotx)} = �log(cosecx+cotx)-1}
dx
d · X
= d/ log tan ) cosec
2
x

(iii) �{log(secx)} = :{log(cosx)-1} = tan


d
x

(iv) "i<log(cosec x)} = d!{log(sin x)-1} cot x


TT Jtence evaluate TT TT
(a) J3secx dx (8) ftco�ecx dx (y) fbtanx dx (6) f'3"cotx dx
TT/6
0 TT /3 0
508
TRIGONOMETRIC INTEGRALS
5, Prove that
d TT
-cos X ' COS X
dx{tan-l(cosec sin2x+1• and
X)} '"'
' ]
sina x+l dx
hence evaluate./ -1 1)
. 2 sin (7!
6. Find the derivative of sec,/x TT
and hence find f9sec/i tanlx
dx rx
TT 2 /16
Prove by direct differentiation, that

;ii
7. (i)
ftanx dx = log sec x + C and fcot x dx = log sinx+c

(ii) The sketch shows portion of


y = t anx , y = cotx , .,
, -..t:.<
(a) Find the size of the shaded area y•tnnx 1
{First obtain the coordinates of P} \, 11,

(b) If this area is rotated about the


x axis, find the volume of the
solid so obtained,
p
l
I
(c) Calculate the magnitude of the ,t:""":.=..::..:,.::...'i;;.!J1�·
angle between the curves at P. 1�

Find the area bounded by y = secx tanx, the x axis


I
8. (i)
and the line x = rr/3,
(ii) The area of (i) is rotated about the x axis. What
is the volume of the resultant solid,
[Hint: what is !(tan 3 x)?)
d
y•fo<l
9. The length of an arc of a curve y f(x) '\
x2 /i
is given by s = J �
+ ( ) d
2
x,
x1
X

,
0 "· "•
where x 1 x2 are the abscissae of the end-
points of the arc. Use chis result to find the length of
the ..:urve y ·= log (sec x) rrom x = 0 to X = TT/ 4,
{Use the result in question 4 (i). }
10, The gradient of a curve y = f(x) at the point (x ,y)
on it is cosx + cos2x, and the curve passes through
the origin. Find the equation of the curve.
(i) Find the coordinates of the points at which the
curve cuts the x axis in the interval O�x�2TT,
(ii) Obtain the stationary values of y within this
range, distinguishing between them,
(iii) Sketch the curve for O �xi 2rr.
(iv) Determine the magnitude of one of the regions
bounded by the curve and the x axis.
*(v) Calculate the voltm1e generated when the area of
(iv) is rotated about the x axis. [First show
2 sin A sin B = cos(A-B) - cos(A+B)).
509
FURTHER POLYNOMIALS
TOPIC 16. FURTHER POLYNOMIALS
1, For the polynomials (7-2x 3 )(1+2x)+x2 (4x2 -5), state the
(a) degree (b) leading term (c) constant (d) coefficient
of x2 • ls it a monic polynomial?
2. Which of the following are polynomials?
(a) x 3-7x2 +5 (b) x 3 -7x�+5 (c) x�-l (d) log x 3

3, For xk-J - 5x2 + 9, find the value(s) of k for which the


expression is .a polynomial of
(i) degree 8 (ii) degree 2 (iii) any degree.
4. Consider the polynomial (.t-4)x 3 -(m+6)x2 +(n-7), Find th.e
values of t,m,n for which the polynomial is
(i) monic, of degree 3 (ii) of degree 2
(iii) of zero degree (iv) a zero polynomial.
5, State the number laws obeyed by the
(a) integers (b) rationals. Which of these are obeyed
by the polynomials?
6. Without calculus, sketch each of the following polyno­
mials p(x). Hence find
(i) the real roots of the equation p(x) • O
(ii) the real zeros of p(x)
(iii) the set of values of x for which p(x) � 0,
(a) p(x) a (x+2)(x-l)(x-3) (b) p(x) • (x-1) 2 (x+3)
(c) p(x) m x2 (x-1) 2 (d) p(x) • x2 -x 4 (e) p(x) c (x-2) 3

7. Perform each of the following divisions;give each result


in the form P(x) • A(x) Q(x) + R(x).
(a) P(x) • 5x 3-7x2 +8x-3; A(x) • x+l
(b) P(x) • 5x3-7x 2 +8x-3; A(x) s x 2 +x-l
8. Find the quotient and remainder when the polynomial
3x 4 -ax 3+sx2 -2x-9 is d ivided by x 2 +2. Write down the
identity obtained from this division.
Determine the values of the constants a, b such that
(3x 4 -8x 3 +5x 2 +4)+(ax+b) is exactly divisible by x 2 + 2.
For these values of a, b, find the factors of this poly­
nomial.

9, Without division, find the remainder when


p(x) • 2x 3-sx2 -4x+3 is divided by
(i) X - 2 (ii) X + 2 (iii) X + 1
Which of these is a factor of p(x)?
Hence express p(x) as the product of 3 linear polyno­
mials,
10. (i) Use the factor theorem to find one factor of each
of the following polynomials. Thence find the
other rational facton.
(a) x 3 +5x-6 (b) x 3-6x2 -13x+42 (c) x 3 +2x2 -4x-3
(ii) Find the real roots of the equations
(a) x 3 +sx-6 • 0 (b) x 3 -6x2 -13x+42 • O
(�) x 3 +2x2 -4x-3 • O
510
FURTHER POLYNOMIALS
11, Show that X"'� is a root of the equation 2x 3 -5x2 -6lt+-4 • o.
and find the other roots in irrational form.
12. (i) Given that (x+l) is a factor of x 7 -5ax 3 -9, find a.
(ii) If (x-2) is a factor of 2x 3-5x 2 +ax+b. and there is
a remainder of -85 when divided by (x+3), find the
values of a, b.
13, When the polynomial ax 2 +bx+c is divided by (x-1). (x-2),
(x-3), the respective remainders are o,-1,2. Find the
values of a,b,c.
14. Show that p(x) = x 4 -9x 3 +2lx2 +x-30 has distinct zeros
3, -1. Hence find all the zeros of· p(x).
15. Find 11
(i) monic polynomial of degree 4, with zeros 1,-2,3,3
(ii) cubic polynomial with integral coefficients, with
zeros -�. 2/3, 4.
16. (i) The polynomial (a-2)x 2 +(b+7)x-(c-8) has 3 distinct
zeros. What are the values of a,b,c?
(ii) The polynomials ax(x-l)(x-2)+bx(x-l)+cx+d and
3x 3-4x2 +7x-9 are equal for all values of x. Find
the values of the constants a,b,c,d.

17. If a,6,y are the roots of the equation x 3 -2x2 -3x+4 = O,


determine the values of
(i) a+S+y (ii) aS+ Sy + ya (iii) aSy
(iv) a -l+s-I+y- 1 (v) (a-1)(8-1)+(6-l)(y-l)+(a-l)(y-1)
(vi) (a-1)(6-1)(y-1) (vii) a 2 +a 2 +y 2
18. If a,S,y,o are the roots of 2x 4 -x2 -5x-3 = O, find
(i) Ea (ii) EaS (iii) EaSy (iv) aSyo (v) E(aS) -1
19. If x = 2 is a root of the equation x 3 -7x-k = O, find the
value of k, and the sum and product of the other two
roots of this equation�
20. In the equation x 3 -4x2 +ax+4 • o, one root is equal to
the sum of the other two roots. Find a, and solve the
equatfon.
21, Show that the polynomial x 3+x+2 has only one real zero.
Find it, and hence sketch the curve y = x 3 +x+2. Obtain
the equation of the tangent to the curve at the point P
where the curve crosses the x axis. Show that this tan­
gent is parallel to the tangent at another point Q of
the curve, and find the coordinates of R, the point in
which the tangent at Q meets the curve again. If S is
the point of intersection of the cur.ve and the y axis,
calculate the area bounded by the arc SQ, the tangent at
Q and the y axis.

22. (i) Show that the equati�n 2x 3-3x 2 -7 • O has at least


one root in the interval 2 < x < 3.
(ii) The equation x 3 -3x+6 • 0 has one real root.
Oetween which two consecutive integers does it lie?
511
FURTHER POLYNOMIALS
23, The curve y • P(x) crosses the x axis
at x � r, The tangent at x • z1 on
the curve (where z1 is near r) crosses
the x axis at x = z2• Obtain the
equation of the tangent at the point
where x c z1, and hence prove that
P(z1)
z2 = z1 - , This result is the -:,r:
P' (zi)
basis of Newton's Method to approxi­
mate to a root of the equation P(x) = O.

(i) Show on various sketches, cases where z2 may not


be nearer to r than is z 1,
(ii) What happens to z2,z 3, if z1 c r?

24. Sketch the graph of y = P(x) for a ix� b in each of


the following. Show that P(a), P(b) have different
signs, and use (CI) The Halving-the-Interval Method
(a) Newton's Method
to approximate to the root of the equation P(x)�O which
lies between a, b. Give the answer correct to 2 decimal
places. [The methods should be used successively no
more than twice.]
(i) P(x) x3 -3x-4. a = 2, b 3
(ii) P(x) = x4 +x-3. a = 1, b 2.

TOPIC 17.
THE BINOMIAL THEOREM
[A. Revise the work on Pascal's Triangle. i
1. Determine, as far as the tenn in x2 , the expansions of
(1-x) 7 and (1 + 3;) 6 in ascending powers of x. Hence
calculate the first three terms of the expansion of
(1-x) 7 (1 + 3;) 6 in ascending powers of x.
2. The first four terms in the expansion of (1+2x)2 (1-x)S
in ascending powers of x are a+bx+cx2 +dx3 , where a,b,c,d
are certain integers. Find the values of a,b,c,d.
3, (i) In the expansion of (1+2x)S(l-y)7 the coefficient
of x2 y 3 is a constant k; find the value of k.
(ii) For the expansion of (l-x)3(1 + .l)s calculate the
X
(a) coefficient of x (b) term independent of x.
4. (!) Expand (1-x) 5 and determine correct to 3 decimal
places, the value of . <49
so) s.
(ii) Calculate the first 3 terms in the expansfon of
(1+�)4 and by putting x = O·l, find (1·025)4
correct to 4 significant figures.
512
THE BINOMIAL THEOREM
5. Evaluate eaeh of the following correct to the number of
decimal places indicated in the brackets.
(i) (•9997) 4 [4] (ii) (1•01) 7 [5]

(), The value of a is such that in each of the expansions


(l+a) 6 , (l+a)8 the fourth term is greater than both the
third term and the fifth term. Prove that 3/4 <a< 4/5.

7. (i) The expansion of (l�ax+bx2 )(1+x) 3 in ascending


powers of x begins with l+x-2x2 ; find a,b,
(ii) The coefficients of x, x2 in the expansion of
(l+x+ax2 )(1+bx) lO are both zero; find the numeri­
cal val�es of a, b and the coefficient of x 3 ,

* 8, (i) In the expansion of (l+x+kx2 ) 9 in ascending powers


of x, the coefficient of x2 is zero; find the
value of k.
(ii) Expand fully (1+3x+x2 ) 3 ,

Now work Set 6B nos, 1 (l-p), 6, 8, 9(b), 11, 14.

C. !Revise the Binomial Theorem and Pascal Triangle Relations]

1. Find the ratio u 9 : u6· for the expansion of


(1 - f> 12 in ascending powers of x, given that x=2.

2. 3 10
Use the binomial theorem to find the value of (1
+ 100>
correct to Ii- decimal places.
r n
Hence use the formula A= P(l ) to calculate the
+ 100
compound interest on $1000 for 10 years 2t 3% p.a.,
interest paid annually. Check the result by using logar­
ithm tables.
1 11
3, (i) Find the coefficient of x 7 in (ax2
+ bx
) If
this coefficient is equal to that of in
(ax + -1..,,..)1 1 ' prove that ab = 1,
bx"
(ii) Show that the expansion (x2 - l)n contains a term
X
independent of x if n is a multiple of 3, and find
the value of this term (in index form) when n = 12.
(iii) Find the coefficient of x� in the expansion of
(2x2 +kx- 3 )10, where k is a positive constant, and
hence calculate the value of k if the coefficients
of xS and_ x lS in this expansion are equal.

513
THE BINOMIAL THEOREM
4. Write down u ' u of the expansion in ascendin g powers
r r+l
of x of
(i) (l+x) 20 , and hence fin d the ratio of their co­
efficients. Which two consecutive terms of the
expansion have the ratio of their coefficients
equal to 16 : 5?
(ii) (x + t> l9, an d hence find which two consecutive
terms have equal coefficients.
[Note (x + \)19 = (\ + x)l 9 J
5. (i) If the coefficients of the secon d, third and
fourth tenns in the exp ansion of (l+x)n in ascend­
ing powers of x form an arithmetic sequence,
fin d n.
(ii) If (l+x / (l+x) = a 0 + a1 x+a2x + a3x +... where
1 n 2 3

a 0, a 1, a2, ••• a re const ants, an d a o, a 1, a 2 are in


a rithmetic progression , show that there are two
pos·sible values of n and find them.
n +l'
6. (i) In the expansion of (l+x) where n is a positive
integer gre ater than 2, the coefficien t of x 4 is
six times the coefficient of x2 in the exp a nsion
of (l+x)n -1, Find n .
(ii) Fin d n if the coefficients of the s.ixth and six­
teenth terms of the expansion (l+x)n , in ascendin g
powers of x, are equal. [Hint; nCr c n Cn-rl

7. Write down the formula for


n
c , and hence prove the re-
r
n +l
sult c + c = c .
n n
r-1 r r
8. If n is a positive integer·, write down the first 3 terms
in the expansion of (1-x)n in ascendin g powers of x.
Hence, show that the first 3 terms in the expansion of
(��:) in powers of � are 1_2n (..lL.) + 2 n(2n-2) (-1L) •
n 2 '
l x l+x 1.2 .l+x

TOPIC 18
PROBABILITY
State wh a t you understan d by the terms
(i) a ra ndom experiment (ii) a simpl
e event (iii) com­
posite events (iv) sample space (v) randcm
Illustr ate your answers by actu al examples. variable

514
PROBABILITY - SAMPLE SPACES
2. A random experiment consists of selecting a card from
one of the cards 5 ) 6,7,8,9 of hearts. If the cards are
denoted by their nUll\bers, state the sample space in set
notation, and represent this on a diagram. What is the
probability of obtaining (a) the 7 of hearts
(b) the queen of spades (c) a card bearing an even
number.
3. (i) From the 5,6,7,8,9 of hearts, two cards are drawn
at random, the first being replaced before the
second card is drawn. Plot the set of points rep­
resenting the sample space for this experiment.
State the sets corresponding to the event of draw­
ing
A: two even numbers;
B: an even number and an odd number;
C: the same card twice; D: two cards totalling 14;
F.: the 7 of hearts followed by the 5 of hearts.
Determine P(A); P(B); P(C); P(V); p (E)
(ii) Repeat (i) if no replacement is allowed.
4. Three unbiassed coins are tossed. Indicating a tail by 0
and a head by 1, state the set of points forming the
sample space of this experiment. Find the probability
of each of the following events
A: three tails, B: 2 tails and 1 head;
C: 1 tail and 2 heads; D: more heads than tails;
E: at least one head.
5. A bag contains 13 counters marked 1 to 13. One counter
is s.elected at random; if X denotes the number on the
counter, determine
(a) P(X is even) (b) P(X is odd) (c) P(X < 5)
(d) P(X � 10) (e) P(X is prime)
(f) P(X is a multiple of 17)
(g) P(X is not divisible by 3 or by 5)
(h) P(X > 6 and divisible by 4)
(i) P(X > 6 or divisible by 4)
(j) P(jxj > 3) (k) P( lx-51 s. 3)

6. (a) Let A and F represent two finite sets of elements.


Show on separate diagrams
(i) A (ii) A U B (iii) A n B.
[In (ii), (iii) consider the possibilities, A is a
subset of B and vice versa; A, B are disjoint;
A, B have some elements in common •. )
( 8) Corresponding to the results in (a), let A, B now
denote composite events. What meaning do you
attach to P(A), P(AUB), P(AB)? What are mutually
exclusive events?
(y) Deduce from your diagrams, the probability laws
(i) P(A) = 1 ·· P(A)
515
PROBABILITY THEOREMS
(ii) For mutually exclusive events,
(a) P(AB) = 0 (8) P(AUB) = P(A) + P(B)
(iii) In general, P(AUB) = P(A) + P(B) - P(AB)
(a) If A,B,C are mutually exclusive events, show by
a sketch that P(AUBUC) = P(A)+P(B)+P(C). Is this
result true whea A,B,C are not mutually exclusive?
Illustrate by a sketch.
7. Use these theorems to determine. the following:
(i) If two coins are tossed, what is the probability
of ( a) obtaining two tails (8) not obtaining two
tails.
(ii) A group of 40 sixth form students contains 12
students with fair hair, and 13 students with dark
hair. What is the probability that a student,
chosen at random from the group, has either fair
hair or dark hair?
(iii) This group of 40 students contains 18 students who
play tennis and 13 students who play squash. In
the group are 7 stude.nts who play both tennis and
squash. What is. the probability that a student,
chosen at random from the group, plays either
tennis or squash?
!l. If A, Bare the respective events of rolling sums o'r 7
and 11 with two dice, find (i) P(A) (ii) P(B)
(iii) P(AB) (iv) P(AUB) (v) P(AUB) (vi) P(X)
9. For events A, B it is given that
4
(i) P(AUB) = �' P(A) = p (B) = 1.. find P(AB)
5' 3'
(ii) P(A) = . 4, P(B) = •3 and A, B are mutually exclus-
ive; find P(AUB).
10. Let X be the random variable whose sample space

t,
S = {0,1,2,3,4,5,6}. If P(O) = P(6) _l,
32
P(l) = P(2) = P(3) = �' P(4) = P(S) = verify that
l
P(S) = 1. Find
(i) P( X 5 2) (ii) P(X=O or X=S) (iii) P(X # 3) :�
11. In a bag are 4 white, 6 black, 3 red and 7 yellow )R
balls. One ball is selected at random What is
]Y
the probability that it is (i) white (ii) yellow �
(iii) white or yellow (iv) neithec white nor yellow.
12. From a pack of cards, a card is drawn out at random.
Find the probability of drawing,
(i) a diamond (ii) a heart (iii) a diamond or a heart
(iv) a picture card (v) a dirunond or a picture card
(vi) a diamond or an ace (vii) a queen or a ten
(viii) a queen or a picture card (ix) a red card or a
jack (x) a black card or a red ten.

13. (i) What are independent events? Illustrate your


answer by examples.
516
PROBABILITY
(ii) If A, B are independent events, what can you say
about P (AB) ? Extend this resu).t to the independent
events A,B,C.
(iii) If A is the event of drawing a red ball from an
urn containing 5 red and 4 black balls, and B is
the event of tossing a 3 with a die, what is P(AB)?
(iv) If E, F are the respective events of rolling sums
of 7, 11 with two dice what is the probability of
rolling a 7 on the first roll and an 1 1 on the
second �
roll?
��������������
14. In a bag, there are 5 white, 4 black and 3 red balls,
(a) Two balls are drawn out one at a time; the first
ball being replaced before the second ball is
withdrawn. �That is the probability that
(i) both balls are white
(ii) the ffrst ball is white and :.he second ball
is red
(iii) both balls are the same colour?
(B) Three balls are withdrawn, with replacement after
each draw. What is the probability that
(i) all balls are red
(ii) the balls are respectively white, then black,
then red
-(iii) all balls are the same colour?
tlS, From a pack of cards, two cards are drawn at random
(a) with replacem�nt (b) without replacement. Find the
probability of drawing
(i) a heart then a club (ii) a heart and a club
(iii) two spades (iv) two cards of the same suit
(v) a black card followed by a red card (vi) two car�s
of different colours (vii) two red cards
(viii) two cards of the same colour (ix) the two of
spades then the three of clubs (x) the two of
spades and the three of clubs (xi) a two then a
three (xii) a two and a three (xiii) two queens
(xiv) the same two cards *(xv) two cards of the same
denomination (xvi) the ten of clubs then a heart
(xvii) the ten of clubs and a heart
(xviii) two cards of different suits
16. One bag contains 3 red and 5 yellow balls; another bag
contains 4 red and 6 yellow balls. One bag is selected
at random, and
(i) one ball is withdrawn.
What is the probability that it is red?
(ii) two balls are drawn in succession with replacement
after each draw. What is the probability that they
are (a) both red? (B) both the same colour?
(iii) two balls are drawn in succession without replace­
ment after each draw. What is the probability that
th�y are (a) red then yellow? (B) of different
colours?
517
PROBABILITY EXERCISES
17. In one bag there are 7 black and 3 white counters, and
in another bag, there are 5 black and 4 white counters.
A boy chooses one counter at random from each bag. What
is the probability that both balls are
(i) the same colour (ii) different colours.
18. A I.Joy X draws two balls simultaneously from an urn con­
taining 7 red and 4 white balls; a boy Y draws two halls
simultaneously from another urn containing 6 red and
5 •.,hite balls. Find who is more likely to draw a ball
of each colour.
19. From a bag containing 13 counters marked 1 to 13, two
counters are selected at random (the first counter is
replaced before the second -is selected). Find the proba­
bility that (a) both are even (b) both are odd
(c) the first is odd and the second is even
(d) one is odd and the other is even
(e} �oth are multiples of 4
(f) the first is less than ,:i and the second greater than
or equal to 11 (g) their sum is 10
(h) the first is 3 and the second is 7 (i) one of them
is 'i and the other is at least 8 (j) they are equal
(t) nt least one of them is divisible by 3
20. At a conference of 60 adults, 39 were right-handed,
34 were fat and 33 were bald, 20 were right-ha1ded and
fat, 23 were right-handed and bald, 15 were fat and
balci, r, were rig:1t··handed, fat and bald, Draw a Venn
"iagram and determine the probability that an adult
chosen at random at the conference will be
(i) right-handed or fat or bald
(ii) fat but not right-handed or bald
(iii) neither right-handed nor fat nor bald.
21. If the probabilities that three students A,B,c· may pass
the Higher School Certificate fxamination are 4/7, 3/5,
2/3 respectively, what is the prohahility that
(i) all 3 pass (ii) all 3 fail (iii) A and 3 pass
but C fails (iv) two of the students pass but
the other fails (v) A,B fail but C passes
(vi) two of the students fail, but the other passes
(vii) at least two students pass (viii) at least one
student passes.
22. Three sixth form students agree that if as many as two
are alive on the fiftieth anniversary of the Higher
School Certificate Examination, they will have dinner
together. If the probabilities that each will be alive
SO years hence are 2/5, 1/3, 9/25 respectively, find the
probability that (i) all three will have dinner together
(ii) only two will attend the dinner
(iii) the dinner will not be held
23. 'nlere are 3 bags of fruit. In the first bag are 4 red
518
PROBABILITY TREE EXERCISES
and 3 green apples; in the second bag are 5 red and 4
green apples; in the third bag are 3 red and 7 green
apples. One bag is selected at random, and
(a) one apple is selected.
What is the probability that it is red?
*(B) two apples are selected (without replacement).
What is the probability that they are both red.
24. 'lwo people are each asked to write down a positive num­
ber less than 20. Find the probability that
(i) both numbers are less than 10
(ii) both numbers are equal
(iii) the sum of the two numbers is 10
(iv) one number is a multiple of 3 and the other number
is prime.
25. To drive to another town A, there are two roads avail­
able, one to the left and one to the right. If a driver
selects a road at random, what is the probability that
if 3 drivers go to town A, then
(i) exactly two take the road to the left
(ii) at least two take the road to the left
(iii) at least one takes the road to the left,

[TI1e following questions may be done using a tree diagram.]


26. In a series of motions in parliament, an independent
member may vote either for (F) or against (A) the motion.
If it is assumed that the member is as likely to vote
for as against, form a probability tree covering four
votes. From this derive the probability that
(i) he approves of all the motions
(ii) he approves of exactly one motion
(iii) he is in favour of at least 1 motion
27. A motorist in travelling to work passes three traffic
lights. If the lights may show red, amber, green with
equal probability, draw a probability tree to determine
the chance that
(i) each light is red (ii) each light is not red
(iii) at least 1 light is red (iv) at most there are
2 red lights (v) there are exactly 2 red lights
28. 'lwo tennis champions, A, B, play a practice match of
3 sets. If the probability that A wins any one set is
3/5, find the probability that
(i) A wins all 3 sets (ii) A wins only the first set
(iii) A wins only one set (iv) A wins more sets than B
29. In a survey taken in two large towns A, B, it was found
that in town A, 75� had both T.V. and radio, and 251
only had radio, whilst in town B, 70% had both T.V. and
radio, 20% only had radio and 10% had neither T,V, nor
radio. A P.N.G. licence inspector selects one of the
towns at random and approaches
(i) one citizen (also chosen at random). What is the
519
PROB,'ID I LITY
probability that the citizen should have a com­
bined T.V. - radio licence?
(ii) two citizens, (also selected at random). What is
the probability that these citizens
( 0 ) both possess radios but not T.V.s
(13) at most one should have a combined T,V. -radio
licence?

EXERCISES ON BOTH ORDERED AND UNORDERED SELECTIONS


Note. The fol lowing exercises are further practice on
both ordered and unordered selections. The first problem,
of course, is to decide whether the order of selection is
crucial or not. If it is imp ortant, then we use either a
space-occupying counting technique or the formulae for
np and np . If it is of no signifiaanae, then we use the
r n
formu I a for nc •
r

EXAMPLE. A bag contains 3 red, 4 white and 5 blue bal Is.


In how many ways can three ba I Is be drawr from the bag
( i) simultaneously (ii) one at a time. For each case,
on how many of these occasions are the bal Is al I blue?
Method.
(i) If·the.balls are dr�.m out all together, we are only
aonaerned with the content of the set not the order in
whicl'. it was obtained. The required number of ways is
12c = ��; =
220, 0f these 220 possibilities, there
3 3
5
·are c 3 10 occasions on which the 3 balls will be
all blue.
(ii) If the balls are withdr(IJ;)YIone at a t ime, then this
implies that the order of appearance of the balls is
vital. In this case, the required number of ways is
12P 1;;
= = 12.11.10 = 1320, ·[Alternatively, there is
3
a selection of 12 balls for the first draw, 11 for the
second and 10 for the third draw.]
5
Of these 1328 possibilities, there are P = 5.4.3 = 60
3
occasions on which the 3 balls will be all blue,
1. A group of 5 boys is to be selected from a group of
9 students. Once selected they are to meet the Mayor.
(i)' In how many ways can the group be selected, and
when selected,

520
ORDERED AND UNORDERFD SELECTIONS
(ii) in how many ways can they shake the Mayor's hand?
2. The set S = {a·,b,c,d,e,f,g,h,i,j}.
(i) How many unordered sets of 6'letters are possible
if each set contains 4 consonants and 2 vowels.
(ii) One of these sets is chosen at random and the
letters ordered. How many su_ch orderings are
there?
(iii) Deduae, from (i) and (ii), how many 6 letter
"words" are poasible from S, each containing
4 consonants and 2 vowels.
3. From an urn containing 5 red, 3 green,

lliJ
2 yellow, 1 p·ink marbles, four are G
selected at random. How many selec­ y
tions are possible, if order is p
(i) important (ii) immaterial?
Of these selections, how-many are (a) all red
(b) of different colours?
4. A heap of 12 cards contains 4 hearts, 3 diamonds,
5 clubs. How many ways are there of selecting three
cards (i) simultaneously (ii) one at a time? ·
Of these numbers, how many consist of (a) all diamonds
(b) one of each suit?

5, In how many ways can


(i) a tennis team of 4 and their coach be arranged in
line for a photograph if the coach is to be
(a) in the centre (b) on one of the ends
(ii) A tennis team of 4 be cho$en by their coach if
7 persons are available, assuming
(a) each one is equally likely to be chosen
(b) the captain A of the team must automatically
be included.
6. In how many ways can
(i) 5 runners finish first, second and third in a race.
(ii) 5 runners finish the race if there are 9 compet­
itors?
7 On a library shelf are 4 fiction and 5 non-fiction books.
(i) How many sets of 4 books can you select, if 1 is
to be fiction and 3 non-fiction?
(ii) The 9 books on the shelf are to be arranged by the
librarian so that the ficLion and non-fiction
books alternate. In how many ways can this be
done?
8. A team of 11 girls are to play hockey at a distant oval.
The mother of one of the girls calls for them in a car.
However, she has room only for 5 girls.
(i) How many different combinations are possible for
her to take in the car?
521
ORDERED AND UNORDERED SELECTIONS
(ii) Having decided on which group to take, in how many
ways can they be seated, if a particular girl G of
the group must travel in the back seat? (Assume
there is room for 3 in the back seat and 2 in the
front seat,)
9. In how many ways can 4 girls and 3 boys be selected to
form a committee if there are 7 girls and 6 boys avail­
able? After a subsequent meeting, the committee adjourn
to a cafeteria for lunch. In how many ways can they line
up, if the President goes first?
10. From the digits 1,2,3,4,5,6 four-figure numbers are to
be obtained, no repetitions allowed.
( i) i!ow many such numbers are there, and
(ii) of these, how many have the�r digits in descending
order?
Determine for (i), (ii) how many numbers end with 2.
11. Six coins are tossed in air. In hm, many ways can
2 heads and 4 tails be obtained?
Do this (i) by selections (ii) by arrangements of 6
letters, of which 2 are H and 4 are T.
Similarly, determine the number of ways of obtaining
k heads where k = 0,1,2, ••• ,6.
12. A girl is required to memorise and recite 4 passages out
of 10 passages in Shakespeare. In how many ways c·an she
(i) select the passages?
(i'i) present th.e passages after selection?
13. (a) Solve for n,
n n n
(i) c = 7 (ii) p2 30 (iii) c = 1
1 2
n n n n
(iv) 21 • p = 7 • p (v) c = c
4 5 9 8
n
(vi) c = 5 • n-2 c (vii)
20P
:
20P
132:1
4 3 n n-2
n n
(b) If p 12 and c 6, find n and r.
r r
14. Before packing for a business trip, a man decides to
take 2 suits, 3 shirts, 4 ties and 2 pairs of shoes, If
he has J suits, 6 shirts, 8 ties and 4 pairs of shoes
from which to choose,• in how many ways can he make his
selection?
15. (i) In how many ways can at least 4 boys but not more
than 7 boys, be selected from a group of 9 boys?
(ii) /1.t the school library, a senior girl may borrow as
many as� books at a time. How many selections are
possible if there are really 8 books in which she
is interested?

522
PROBABILITIES INVOLVING COUNTING TECHNIQUES
£, PROBABILITIES INVOLVING COlJIJTING TECHNIQUES
1. A group of 3 persons is chosen from 6 boys and 5 gi�ls,
by putting the 11 names in a hat, and drawing out J sim­
ultaneously at random, What is the probability that the
group is exclusively female?
2, Five women and six men are arranged in a row so that
each woman sits between 2 men. How many such arrange­
ments are possible? What is the probability that two
specified men A and B should sit at the ends of the row?
3, Three balls are drawn at random simultaneously from a
bag containing j red, 5 black and 4 yellow balls, What
are the probabilities that the three balls will be
(a) all yellow (b) all of one colour
(c) all of different colours?
4. Words are constructed using the 5 vowels a e i o u and
the 4 consonants b c d f, Find how many different words
are possible, each containing the 5 vowels and 4 conso­
nants. What is the probability that the vowels come
together (i) in alphabetical order (ii) in any order.
5. The digits 2,3,4,5,6 are written in every possible
order, Find how many numbers formed exceed 34000, and
the probability of this to occur.
6. 7 girls and 4 boys wish to form a committee of 4. I.f
each of the boys and girls has an equal chance of being
selected, what is the probability that the COllDilittee
will contain (i) all boys (ii) all girls
(iii) 2 boys and 2 girls (iv) a particular boy.
7, How many numbers less than 3000 may be formed from four
or fewer of the digits 1, 2, 3, 4, 5 (no repetitions
allowed)? One of these numbers is selected at random.
What is the probability that it is divisible by 5?
8. Nine cards each bear one of the digits 1,2,3,4,5,6,7,8,�
nie cards are shuffled and 5 are selected one at a time.
Find the probability that
(i) the card marked "7" is not drawn
(ii) the digits on the cards drawn are in ascending
order of size.
9. In a batch of 10 similar articles, there are 4 faulty
ones. If three articles are selected at random, what is
the probability that (i} all three are faulty
(ii) at least one of the three is faulty?
10, How many 5-digit numbers are possible from the digits
0,1,2,3,4,5,6,7,8,9 (if no repetitions are allowed} if
(1} there are no restrictions
523
PROBABILITIES INVOLYING COUNTING TECHNIQUES
(ii) each digit is greater than that which follows it
e.g. 87532 (iii) each digit is less than that
which follows it, e.g. 12459.
If A, Bare the events that (ii), (iii) may occur, find
P(A), P(B), P(A), P(B),
����������
11. (i) How many different arrangements are there of
HHTHHH? If one of these is selected at random,
what is the probability that it · ends with T?
Hence, state the probability that if 6 coins are
tossed, then the tail is on the end coin on the
right whilst heads appear on the other 5 coins,
(ii) How many different ordered selections are possible
from the letters of ATTAIN? If one of these selec­
tions is chosen at random, what is the chance that
it begins with N?
12, (i) If 4 Maths, 5 Science and 2 English books are to
be arranged at random on a shelf, find the proba­
bility that books of the same subject· are together.
(ii) From a bag containing 5 balls, any number of balls
may be drawn. What is the chance that an even
number of balls will be drawn?
13, Three cards are drawn one at a time from a pack of 52,
each card (a) being replaced (b) not being replaced
�fter each draw. Find the probability that
(i) all are ten's (ii) they are the ten of spades,
ten of clubs and ten of hearts in this order
(iii) all are hearts (iv) all are black cards
(v) all are the same suit
(vi all are of different suits
14, (i) In a bus are 8 passengers; 5 women and 3 men. If
4 passengers get out at the next stop, what is the
probability that at least 3 of them are women?
(ii) If committees of 5 are to be chosen from 7 adher­
ents of party X and 4 followers of party Y, what
is the probability that party X has a majority but
at least one person belonging to party Y is in­
cluded.
15. An um contains 30 black, 20 white and 10 yellow balls.
Three balls are withdrawn one at a time. What is the
chance of drawing
(i) in order, a black, white then yellow ball
(ii) a ball of each colour (iii) 3 black balls
(iv) 3 balls of the same colour (v) two black balls
then 1 white ball (in this order)
(vi) 2 black balls and 1 white ball.
C0nsider the two possibilities (a) the ball is replaced
after each draw (b) the ball is not replaced,

524
BINOMIAL DISTRIBUTION

Revise carefully the Section on the Btnomlal Distribu­


tion· and Binomial Probabi I ities.

1. On an average a certain student can solve 60% of prob­


lems. In an examination there are 5 problems, and to
pass the examination, he must solve a minimum of 3 prob­
lems, What is the probability (i) of his passing?
(ii) of him scoring full marks?
2. I have a 25% chance of hitting a target. What is the
probability that in 6 shots, I hit the target
(i) as often as I miss (ii) more often than I miss?
3. It is known that a certain species of· rabbit has either
brown or white fur. If we can expect, in the long run,
that 3 out of 4 of these rabbits will be brown, what is
the probability that at least 2 rabbits in a litter of 3
will be brown? ���������,--��
4. For a certain type of seed, the probability of a seed
germinating is 0,6, If 5 seeds are planted, what is the
probability that (i) exactly 4 seeds
(ii) at least 4 seeds (iii� at most 4 seeds
(iv) most of the seeds, will germinate?
5. A multiple-choice quiz consists of 5 .·questions each with
4 different choices. If each question was answered at
random, what is the probability of getting
(i) 1 question correct (ii) 2 questi.ons wrong
(iii) at least 4 questions wrong
(iv) at least 4 questions correct.
6. If a 'pair of dice is rolled, show that the probability
of getting a total of 7 or of 11 is 6/36 + 2/36 = 2/9.
Now, if the pair of dice is rolled three times, find the
probability of having a 7 or an 11 at least once.
7. At an election 60% of the voters favoured canclidate A.
If 5 voters are selected at random, what is the proba-
bility that a majority favour A?
8. Seven students are chosen at random from a University· at
which 10% of the students are married. What is the prob­
ability that
(i) all are married (ii) no more than 5 are married?
9. (i) A coin is tossed n times. What is the chance of
obtaining at least one head? How large should n be
to make this chance about 0•9?
[Hint 1 - ;n i •9].
(ii) How many times should a die be thrown in order
that the probability that a "3!' appears at least
once shall be greater thim �?
[Hint: Use the method of (i)]
525
BINOMIAL DISTRIBUTION
lo. If on the average in a shipment of 10 cases of certain
goods, 1 case is damaged, what is the probability that
out of 5 cases inspected, at least 4 will not be damaged.

11, In a factory three operators A, B, C work in shifts on a


certain machine. Records show that the number of parts
produced by A, B, C respectively are in the ratio 5:6:7,
Of the parts produced 1% of A's, 1fr. of B's and 3% of
C's are defective. What is the probability that a part
drawn at random from the output of this machine will be
defective? The owner of the factory has a large stock
of parts made from this machine and makes them up in
parcels of 10. What is the probability that a parcel
chosen at random will contain at most two def�ctives?
E. EXPECTATION; EXPECTED VALUES; RANDOM DIGITS
!Revise this work from the text. I
1, A factory mass-produces bolts. If it is known that 10%
of the items produced are defective,
(i) What is the expected number of defectives in a
sample of 5 bolts taken from the factory?
(ii) Calculate the probabilities of obtaining O,l,2,3,
4,5 defective bolts in our sample.
(iii) If 100,000 samples (each of 5 bolts) are consider­
ed, what would be the expected number of samples
in which there would be 0,1,2,3,4,5 defective
bolts?
(iv) Verify that approximately 92% of our samples would
have at most 1 defective bolt, and that approx­
imately 99% of our samples would have at most 2
defective bolts.
l. An electronic machine generates the digits 0,1,2,3,••• ,9
at random. It is operated 5 times (i.e. 5'digits are
generated),
(i) What is the expected number of times a 9 arpears?
(ii) Use the calculations of question 1 t o calculate
the probabilities that a 9 appears o, 1, 2, 3, 4,
5 times in the 5 operations.
(iii) If the machine produced 100,000 groups of 5 digits,
what would be the expected number of groups in
which a 9 (a) does not appear (B) appears at
least once?

526
CHANGE OF COORDINATE SYSTEMS
TOPIC 19 (SYLLABUS TOPIC 9)
CHANGE OF COORDINATE SYSTEMS. TRANSFORMATIONS
1. A point P has coordinates (x,y) relative to the Ox, Oy
axes, and coordinates (X,Y) relative to axes O'X, O'Y,
where O' has coordinates (p, q) and O'X, O'Y are paral­
lel to Ox, Oy respectively. Prove that x•X+p, y•Y+q.
2. (i) 11\e vertices of a quadrilateral ABCD are A(-1,2),
B(3,-4), C(-2,-5), D(l,6). What do these coordin­
ates become when the axes is translated to the
point A, without change of direction of these
axes.
(ii) 11\e coordinates of the end points of an interval
AB are A(-5,-4), B(3,-6), relative to axes through
0 1 (2,-5), What are the coordinates of A, B rela­
tive to the parallel axes Ox, Oy?
(iii) A point P has coordinates (-5,7) relative to the
axes Ox, Oy and coordinates (-1,-8) relative to
parallel axes through O'. What are the coordinates
of O'?

3. What do the equations of the following curves become


when the axes are translated to the point O', without
change of direction?
(i) x2 +y 2 -6x+4y-51 .. O; o'(3,-2)
(ii) xy+7x+y+3 • O; 0 1 (-1,-7)
Hence identify and sketch these curves.
4. Transform the curve given to parallel axes through the
point stated.
(i) y • x l - 6 x 2+12x-8; (2,0)
(ii) y '"' 2x 4 -5x-3; (1,-1)
(iii) y '"' 3 + 5 sin(2x - rr/4); (rr/8 3)
b -4ac
(!.v) y • ax2 +bx+c; - Za' - �
1- 1
2
b

Use the method of completing the square and translating


.

the axes to find the coordinates of the vertex and of


5.
the focus, the equation of the directrix and of the
axis, and the length of the latus rectum of each of the
parabolas (1) x2 +2x-12y+61•0 (ii) y • 2+x-x 2 .

TOPIC 20 (SYLLABUS TOPIC Sb, 5d)


MATHEMATICAL INDUCTION; SEQUENCES
MATHEMATICAL INDUCTION

Prove that E (2r-l)2 •12 +32 +52 + •• +(2n-1)2 • .!!.(2n-1)(2n+l)


Pl 3
1.

When n • 1, E (2r-1) 2 al2 • 1(2-1)(2+1)


Method
1

Pl 3 which is true.
Step 1

527
MATHEMATICAL INDUCTION
Step 2, We now assume that the result is t rue for n � k, and
prove it tru�. for n • k+l.
k
Thus we assume that : (2r-l)2 = f<2k-l)(2k+l),
r 1
k+l
and prove that E (2r-i) 2 • k+l[2(k+l)-1][2(k+l)+l] ••(X)
3
r=l
T o do this, we n ote that
k+l k
I: (2r-1) 2 I: (2r-1) 2 + � [i.e. S k+l S k + u k+ll
r=l r=l
+l

= 1<2k-1)(2k+l) + [2(k+l)-1] 2
{Note this step!}
= .1(2k-1)(2k+l)+(2k+l)2

<
= 2k;l) [k(2k-1)+3(2k+l)] on factorising
(2k+l)
= [2k 2 +5k+3] on collecting terms,
3
= 2k+l. (2k+3)(k+l) on factorising
3
k 1 2(k+l)-1][2(k+l)+l], re arr anging
= ; [
[ See line (X) ]
Henae if the result is true for n = k, then it follows
,t!Yat it is true for n = k+l.
However, it is true for n = 1 (proved above), and thus
is true for n = 1+1 i.e. n = 2.
Similarly, n ow being true for n = 2, it follows that it
is true fo r n = 2+1 i.e. n = 3, and so on for all positive
integral values of n.
[Note (i) the two steps in volved must be both proven.
(ii) the method must be adapted in certain cases.]

2. P rove, by the method of mathematical induction, that


n
(a) 3+7+11+...+(4n-1) [i.e. E (4r-1)] = n(2n+l)
r= l
n
!: r 2 ] = ti(n +l)(Z n +l)
r=l
n
(c) 1+5+5 2 + .••+sn-l [i.e. !: sr-11
r= l

(d) 1 + 1 + 1 +. '.+ 1 1 n
n

1.2 2.3 3.4 n (n +l)


[i,e, " r(r+l)] = n+l
r=l

(e) /s + s\ _., /13 + ...+ (4n:3)(4n+l) = 4:+1


528
MATHEMATICAL INDUCTION
3. In the fol lowin g, the method of in ductio n is to be used,
but it needs careful consideration. Prove
d n n -1 [H' t d k+l . d k
(1') (x ) = n x i.n (x ) .. dx(x , x) and use
dx dx
the Product Rule for differentiation.]
n
d n- l �
(ii) (l og x) = (-l)
dxn k+ l
xn k
d d d
[Hint: (log x) = <l r_ (log x)] )
k+l xLbf
dx
n n n-1 n -2 n -3 � n -1
(iii) X - C = (x-c)(x + X C + X C +•.••. +c )
k+ l k+ l k+l k k k +l
[Hin t: X - C = (x - XC )+(xc - C ))
(iv) S � 1+4n. [Hin t: S
n k+l k
= 5.S )
2n +4 2n
(v) 3 is divisible by 5.
- 2
2 k+4 _ 2k
3 2
[Hint: assume is an in teger I (say),
5
k+4
and thus i 51 .+ l
k

Then co sider
n

i k+6_i2k +2 2 2k+4_ 2 k+2 2k _ 2 k+2


3 .3 2 3 [SI+2 j 2
2k
4sr + 2 C3 2 -2 2 )
k
5 [91 + l J,
which is divisible by 5.J
7n n
(vi) + 19is divisible by 13 if n is odd.
7k k
[Hint: Assume = 131 - 19 , an d then prove
7 k+2 k+2
+ 19 is a multiple of 13.)

!1· SEQUENCES !Revise this sect.ion from the /\ppendixl


1. Write down the first 6 terms of each of the fo llowin g
sequences; graph these terms, an d in dicate if the se­
quence appears to con verge, diverge or oscillate
1 n 11
(a)·{7} (b) { } (c) {logio n } (d) {cos n }
2n +l 2
n -l
2
1-l' n
(e) { z } (f) {5-2n } (g) {3 + �} (h) {4+(- l)n}
n +l

sin90n °
n
(i) { } (j) {L}
n n
State the limit for each of the convergen t sequen ces.
b
2. Given that u = a.4 + n , n = 0,1,2,3,•.•; in each of
n
n 4
the fol lowin g cases, find a, b, u and the limit of u . 2
n

(i) uo = 3, u1 12 (ii) uo = 8, u1 = 2

(iii) u0 = 9, u1 21

3. Select the correct answer in the followin g:


5 29
SEQUENCES
{u } is a convergen t sequence if
n
2n 2 +1
(a) u = (b) un • (2l)n -1 1T
(c) u � tan(2n +l)
n n n 4
(d) u = (-l)n , 2n
n
(e) un = (-l)n , 2-n
1 1
(f) {l +i Tn'}, [Hin t: Ration a lise un first,] (g) 0.og(;)}
n -
4. The limit of the sequen ce
(a) {;��i} is (ex) 0 (B) 1 (y) 1 (IS) n o limit (c) 1?
3 2
(b) {tan-1n} is (ex) O (B) 1 (y) 2!. ( IS)
1T
2 4
( e:) n o limit; oscillates?

(c) {7+4l /n } is (ex) 7 (B) 8, (y) o (IS) 11 (e:) no limit?


5. For what set of values of x will the sequence {xn}
(i) con verge to O (ii) diverge to ± �
(iii) oscillate from -1 to 1
What does the sequen ce become when x = 1 and when x = O?
6. Determine the limit U of the sequen ce {u = �5 n +31},
n 3n-
(a) Show that I U - un I = �14 , and hence if I U-u I < O•l,
n-3 n
prove that n > 15 8 . Beyon d which term do al l terms
9 5n +3
of the sequence { _1} differ in absolute value
3n
from its limit U by less than O•l?
(b) What is the least value of N for which lu-�I < •001?
(c) Given c > O, show that lu-u I < e:, when ever
n
n > i<3 + 14e:-1),

530
\

ANSWERS
• I
ANSWERS
CHAPTER l
� (page 18)
11 11 rr 11 511 n 11 2n
(i) -4 (ii) -�,\3(iv) - - (� -
6
(vi) (vii) - (viii) -
3 6 3
- 11- 3-11 - 2
""T,;"""
(ix) -
4- ( - 2. (i) .!!.
3 (ii) - .!!. (iii).!!. (ivl_ - -
�) 2 •
11 ·n n n - - 1 1 - 12 ""'73 13
2
(.!_) 3; - 3; 3; - 4 (ii) O; - 2 (iii) - l
; ; 2; TJ; -
5
; ·p 72
3111
(i) .!!. + 0 + .!!. 4 -
.!!..)kii) 3(.!!.)- 2(..'!.)+7( )-4( ) =
311 511

- 3 2 1l - 2 6 4 6 _5 1 2 ·
11 11 11 211 2
(iii) .2(-)+3(- -)-7(0)+2 (- ) ; -. � (i) - -; 0• 537; -
8; 7
3

-. - -4 3 1 - 2 6 12 . .4 1 5 -9 5
� 24 336 - 1
·s; -
3
(� 9; 2; 3; 13 (iil) 0"8; 17; 41 (iv) 5; ; 0'68
2 625
2 0 -4 24. 3 n Sn 7n lln . _1 I n
( 3; ( - sin
� 21; 7; 5 • J. .!) 6; 6; 6 ; - -6-; <2> • 6
(ii) tan-l(i) ; f; Ii --7fl = 11 (iii) !f + (- f> - !J = - f
n 5n _ ]]! (ii) - 7n 5n 5n 7n 5n
( ) - 6' - (a) (b) .!!.
.!_ 6; 6 - 6' - 6' 6' 6 ; 6 6
3n 3n
(iii) x1 ; , x2 = - n
2 .
4 4;

-3x 2
�;

(_!) sin-Ix; cos-12 x; � - a J . (ii) 1 +


J_. t n x (l+x2) (sin- 1�x).{1-gx2;
2x tan-1 x; x 2 [� + 3 sin-1x 2] (iii)
11=!2; O;
[3 x sin-1 x 2 -x 2 [ 1
7(sin-1 x) 2 � -L + -- -J (iv) - 2e l� + x cos-1 2 xJ ;
x x xz+l;
1 • sin(sin - 1 x) 1,. _ • 1 •
- f' (x )
(tan-12x). (1+4xZ)' ,1-xz .,, 7f::xl•
x 11 f' (x) 1
£" (x) • 1 ; IS 5
(4-x2) 72 ; 3;
31 • :a -x-,/1'"-"'<"'io-g"'x;,)�2;;
1 2e� 12 Be� 1
g' (x) • sin-Ix. ;;t::x,; 7'f"' · ffi • - - (b) 7j'
11
l 2 log2
(ii) (a)
2
(b)
5
(Note e

2I J.
6
(iii)
- <";''
1!
3
> _8. l ; x-y+l

o; If
ANSWERS

1.

211•'.! . .!
- .!.
(a) 6' J'4
- .!.
(d) 11 • .!. .!I..
4' 'J ' 12
I7J
" " " ff ff ff " ff
(.&,) 8' 36' 6' 2 <.!l> 120'44'8'
x
e 11 �3 .t'.n/3 r.: 1 11
- --2X-; -12 [Note e
(i) • e • t3) (.!!) 7_1 0x x2l -2

"kJ�1!•"""''ic
l+e -

:; (.!) 2 tan-1 x I (1�2) (ii) 4x .!!.n:1 (x 2 ) I tl-x4


�511
s. -l • ..! -1. - l 6.
- sin x'12+ 2
(i) - 12
' _
-x ?'11 I ho.
• 13-1 511
.?.· --r,,r;
TI' - 12• 3
ff II

(_!!) ,m-•,, •<•fi-Jl - 6 1012 !, <!) log 2- 2+ f


-
3 11(9-413)
-i•A..'it , (ii) 'sl.011<2> -+- 36
9. (.!!.) 1 ; + �log(2/3-l)

� (page 32)
1. W y • sin-1x + .! or y
2 X • -cos-1x + 11 W y • tan-Ix + l!!. 4
5'11
; [y • sin-1 2 + 11 or y • -cos-1 X + 511 J (.il) 5 + II
i• (j) 6
3 2 6 8;
(y • 5 + �tan-I
t • -cos-1 �2 ) .
JJ.
J,;
(!!) x
<.!) x • 2 cos t (t • s1n-l
• -l
I-. f
cos 3t; O (t • l ,1n-l(Jx) + .! or
or

3 6
f>;
3
t • -
3
l
cos-1(3x) + t1 Uil) x • 4 tan(4t + 4
x
(t • �tan-1 �4 - ..!J
16 -
4. ; W I '4'
1' - !.
(l -x2) f2 -_, 1
2aqUDU
5.
-2x
(l+x2) 2 �-1

1 2
aq. un its
I I
_...,.,;...: ��;:,;:,,;�.,.I ,.
-IL-.-------------'..
(j) C (.il) D Uil) A
II
ANSWERS
9112 cu
J_ .!) Vol •
S
I

11�
____.,
---- �its �
3 11 �-��---f)�"--+-4"
-
*""x-- �
"' -
(ii) 2 sq units
(iii) tan- 1 (�) • .lt
lim I
_1 - x- a 2 I _11
J. (_!) • 7867 (£.) • 71154 (.£) actual value •{ + • 7854 I

.12, l•I , 1 <!> If; , - f · lfc, - "Jt>;


-:t 11•...:'• j sin-1 xl � I Ii' !I 2
+

�- -:� -- $-
- Jt -, 0 ' .,, 1 ( ii) 1 . _.!!.

,u:_ -- -!\•CM_,x 11 13
3n/, Y - 3 • z(x - \)
2
X= to<!� 11 *ML•
l
+ l3
··X. -, X: (!) 7j;,,. - "
:·:-;·:
4
-x----+---4---+_' O � cos-Ix S 11 (ii) y • { - cos-\;:
- I
derivative is x • 12 sin y.
1
-rt
-� ---- �- -

---., -.:c
=;{,,,,.'.,,
("fl,,f,o� o\ X•�
tvrvt.J 'll::

-�--

CHAPTER 2
� (page 40)
(.!_) A(3,0,Q)_; B(3,4,0); C(0,4,,_Q.); D(0,,46); E(0,0,6); F(3,0,6)
(ii) (a) � (b) 2� (c) {61 (d) {61
(iii) (a), (b), (c), (d) all (3/2, 2>.- 3).

t-o. (i) a cube (ii) 3;3; If ..l, 11+9+i6 • 2o+l6


<I>
11 15
(5,2,-3); 1166 (ill
1 1
Im
11 -11 �5 -
m-,
5, ill!.) �,'152. Ire• 6119 sq
C4, 4' -2> • <2• 2• -2> • C4, 4' 2> i• <2•2,11,
3 � o, O)
10. (1, 1, •/6) JJ.·
lx1 2+y1 2+z12; �; �; �

� (page 51)
1. <.!> o•,90•,90• CJ?.) (,o•,:10 ° ,90• (£) 45 ° ,90 ° ,45 ° -
2. (.!) * ./6
1 4 angles
(.!!) 60 ° , 120 ° J_ (.!.) .Pe
(ii) cosa • cosB •cosy•• 73;
ll I
are 54 ° 44' or 125 ° 16'.

I I
0

I a..._,
2 3 6
�11, 1. upwards d �cs sre 7' - 7' 7
2 3 6
I_ __ dot.mlJards d,cs are - , 7, -
7 7
(!) 9; 5 (.!!) d.cs are - 9, 9, 9 and 5, S' 0, D,rs are -8:1:4 and
-3 "
,ii
8 1 4 3 4
3:4:0 (iii)
- for OA: 152 44', 8 3 37', 63 37'; for OB: 53 8',
° ° ° °

36•52•, go• 4 1. -2 2 -4 -12 3


<iv> 21 <!.> 3' 3' 3; l:-2:2 <!!.> 13' TI• 13;
J.
-4:-12:3 (.£) 572, �' O; 1:-7:0 (.!!) 1,0,0; l:O:o
1 JO? 1 2 -1 1
for L1, d.cs are 76• 76• 76 and for L2, d,cs are 76• 76• -76;
a • 80 24', Ji (.!) {ii) 30 � (.!) d.cs for AB are 76•76•76•
4 13 1 -1 -2
21
°

and for CD are 76• 76• 76 (.!!) 2 (iii) 3


2 1 -1 1

1f! � "7t, "ft,'7t,


11

11, (.!) � (ii) (.!) 16; 2J"j (ii) for All, d.cs are
6
-1 1 1 r;; sq.
for AC, d.cs are/!' ' 7f (iii) O (iv) 3,2
A
1 i: -3 - 1 7rr,
3 -1
(.!) d. cs for AB are l'fi'• l'fi'• 7ff and for /.C are 7rr, 7'fi'
J
13.
cos BAC • u5 (ii) 11 (iii) 2 ..,,
4/6
6 sq.
14. d.rs for AB are 2:1:4 and for CD are -4:-2:-8 (i.e. 2:1:4)
�- (.!) Parallel (ii)
14
3
and for AC are 1:6:-5
Jl: (ii) k • 3 18, d.rs for AB are 2:3:4
21, (i) k • 3 (i;:;- k • - t
13 16
(i) 1-_2s� 2:-3:5
.ll· - 3 ·(ii)- 3 (iii)
-- 3

SET _2C (page �3)


-8 2 -10 X ...:J. Z
1. sX m
:/. Z
2 • 3; oo,4 , 6 l:. < 3' 3'
> W I • -1 • 7i3> l:
...:J.
CU) -=I • 3 • 4 <.!W I • -3 • I 2 ; 7j • 1 • o
X Z X Z ,rC X Z
.i:
:/. :/.

5. x • X+p, y • Y+q, z • Z+r (!) (3,-9,11) (ill p • 9, q • -4, r•2


( ill)-(1,11-4) .2.J. (!) 2X+3Y-4Z.• 8 (.il) JC4y2+z2 • 25
(ill) x2+yz+z2 - n
{ll) z • x2+Y2
x a x1+t(x2-x1),_y • Y1+t(y2-Y1), z • z1+t(z2-z1) (.1) (4,1,-2)
(!!) (-4,-6,-8) (ill) (14/5, -1/5, 22/5) (ll) (14,8,-�)
1,2 9, (A) (!) x-2 • rl. rl (!i) tl.rl. z-4
- 3 1 -7 '3 -1 O
- 2
i .e. x- - - rl z--4 o (� 'i" x-2.rl.ti1 .e. y-5•0 ' z-40 a
3 -1• - 3 0 0
" x-2 rl.� ,,,, tl.rl. z-4
(!!) ( •
2 -2 -1 6 3 -2
�, (.1) � • rl. z-4 ,,,, • 90°•' tl • Z.:i.' z-4 • O
-"J \AAf

(:u
-o. -1 1 _...., y 1 -13
4:3:-2; � {jJ Points for (say) A• 0,1,2 are (1,-2,5);
72J• �
4 3 -2

(5,1,3), (9,4,1) 11. (Points given are for A• 0,1,2)


{jJ -5:3:4; (-1,2,"n", (-6,5,7), (-11,8,11) W) -5:3:0; (-1,2,3) l(-,11 .•�,
1-,,, ,:i.)i
s

7 4 -9 11 -7 -5
(ill) 3:-1:1 • 7:-3:3; <3, -3, -4, (6, -s, 2>. <3, 2>. 2>
IV
ANSWERS

=
lli) 5:2:-3; (1,-4,2); (6,-2,-1); (11,0,-4)
(i, x-1• �. z-5 (ii ' x+2• :L±i O z-3 -3
(i"ii' x • rl . z-5
.!,.I 2 6 ..:3 oil/ 1 1 -5 -1 -1 O
(i v'� • rl• ....! 13 (i ' x-5• l'.il � z-7• yes (ii' (5 -1 1)
-2 -· .!,.I -2 3 -2'

=
=:tJ 3 -2 oil/ ' 3' 3
14. (i ' x+2• .I±!. z-3 (ii' x+2. :d:.!. z-3 (iii' x+2• .I±! · 2_3• 0
.!,.I 7 -2 1 oil/ 4 5 -8 3 5 '
x+2 ..ti:L__ z-3
( • ° ·�i .e. y+l• o, z-3• O
_!!) cos0° cos90
15. x-4• �
-:S _ • -z+5- 16. 3:4:6 and 6:8:12 ll: 3, 6, -2 and 2, -1, 2:
3 7 - 7 7 7 3-183 3
cose •
4
21 ;
e • 79 ° 1'J2.· 10
1 l2,· (i) k• 2 (ii) k•
5
1 17 17 X • ::t.
Point is ( ,
4 4 2
, ) .ll· _ 24 7, z s
0

24. l:m:n• 26:21:-24; eqn is� x+4• cl• z-3


21 _24

SET 20 {page 70)


� 2+z f• 36 (ii) (x-3) 2+(y+2)2 +(z-4)2 • 49 (.ill) x 2 2+z 2•9
<Tu (x-1)2+(y-2) 2+(z+6)2 • 30 l: (j) centre O, radius 10 units
+y +y

-
(ii) centre (3,-1,2), radius 7 units (.ill) centre (3,5,-4), radius
6 units.
4.
� 1

V
ANSWERS
� (page 75)

WJl
.l: ill l'

. .,.

(!) x•O (.!?) y•5 (£.) z•6 @ z•-7 (!) 3x-2y•6


(!) x cos 120 ° + y cos 60 ° + z cos 135 ° a 2, i,e, x-y+lfz+4 • 0
(.&) cos¥ a -123 r.. 2 3 1
6-; 3x+2y+•2lz a 7 30 (!!) X , {14 + Y•7i7: + Z,7i7: • •14,
r.,

i. e, 2x+3y+z • 14
9
J.:
4 x + y - 4 z • 3;
9 9
l = i m • l n = -4' p=3
9' 9' 9
7
24
units; -1' .§.' -1
7 7 7
-1 1 2 .!!) -6;6;3 units -:c 4
<.!) -1:1:2, 76• �· � (
(ill) meets xy, yz, zx planes in lines x-y+6a . O, 6, x-2z+6•0
(.!:,v �6 units ii 1 2 3
(a) (i) 1:2:3; �· ?If• 7i7: (ii) O;O;O
(ill) x+2y = O, 2y+3z • O, x+3z • 0 (.!:,v O units
(.!?) (i) 3:4:0; !• !•
0 (!.!) 20;15;0 (ill) 3x+4y • 60, y • 15,
x • 20 (.t,v 12 units, (c) (i) 1:0:0; 1,0,0 (ii) 3;0;0
(iii) x • 3; does not meet yz pTane; x • 3 (iv)� units,
1.: -r-
-
x + 3 y + 6 z = 9, i.e. 2x-y-2z+27 • o
9 9 9
8. (i) 5x+4y-10z • 20 (ii) 5x-4y+3z • 15 9, (i) x-3y-7z+45 • 0
(ii) 9x-2y+6z+4 • 0 ,li,
(.!) x-2y+3z+l5 ':" 0 -(!.!) 7x+2y-4z • 13

SET 2F (page 80)


;-- 25 2 8 15 I 25
-3 � (.!) 3 8 . (!.!) T,7 (iii) 7"7r)f)
5 -
6. (i) -
>. • -
8 - ii) >. • - -6
1
l.: sin ,i, • ; ,i, • 19 28
1
° I
!: sin,j, • l; ,i, • 90 °

.
9. sin,j, • � ; · 1/1 • 10 59' �- (.!) 5x-7y- z • 0
° 8 (.!!) 4x+2y-7z+40-6
1
l
2) I
w
·-
)
�.:. � I I
I
I
J
I
I "
4 i ,/

£���
µ
l �
I,
it�.,'
origin side of
"
between and on

interior of cube interior and surface
plane. planes. of side 4 units, of rect.prism 6x8xl0
VI
interior of sphere, region outside sphere, inside and lurface of
centre (2,-3,1), above plane z=2 or para�olic cylinder.
radius 4 units. below �lane -2. 3-
� 10
w

side interior and surface of region between 2


sphere, excluding cylinder concentric spheres.
7

� ,t �
surface and re­ interior of para­ surface and region surface and in­
sion between two boloid between 2 inside sphere on terior of
cylinders, radii parallel planes non-origin side of triangular-prism
3, 4 units z •l, z•3. plane x+y+z• 1. shaped region.

SET 2H (page 83)


' x-1 •_ti:1•z-4 (b) x-5 •:£.ti. •z+4
(a
!!.I 1 -5 2 3 -7 8
(.!) 5(x-2)+1(y+3)-4(z-l) a O, i.e. 5x+y-4z•3
(.!!) l(x-2)+5(y+3)-6{z-l) •O, i.e. x+5y-6z+l9 • 0
(iii) 7x-4y-10z •16 (iv) 7(x-2)+1(y+3)-5(z-l) a O,
i.e. 7x+y-5z � 6 J: 2x-3y+2z•14 � (!) 3x-6y-2z•t 21
VII
ANSWERS
(.1!) 2x-y-3z • 14 (iii) llx-33y+4 4z • 12
W k 2 ; 15x-2y+4z • 25 6
5
O
(.1!) k • - 1; 33x+6y-12z • 55
1
-4 , 2 units. 2 + - 4 • -W unitslO,i\1nonopposit
. e sides of the planes)
3 3 3
(.1! ) (0,-1,1) .!!.· {.!!.) (0,-7 13
2' 2> (]2) (7,0,3) (£) (-9, -8,11)

c.,v
(2,1,7) 10. (j) X • 5-3t, Y • -6+6t, Z c 5+3t (.1!) (3,-2,7)
cw 3:-2:1, -3:6:3 12. c1,2,3> w c-2,4 ,-1>
P(4,3,4); 3x+2 y+z • 22 15. {.!!.) >- • 5/11; 4 3x-24y+l2z • 31
� 10 5
�) 15x-3ly+3Sz • 74 �· U) ( , , 1) (.1!) (0,10,16)
3 3
(111) -2:5:9, 17. (1) x-12y-10z • 5 (11) 2x-y+3z+4 • O

x;:
(111) xt3yt6z !To (iv) 4x-y-2z • 6 (v) 4 x+y+9z • 29
2 •� 2 4
• � 19. (1) 5x.+y+3z • 15; plane bisecting "AB' at
r.!ght angles (11) x 2 +(y-1) 2 +z2 • 1; sphere centre (O,l,O), radius
h units. (111) x2 +y2 • 4z; surface of paraboloid of revolution.
(vl (x+6)2+(y-6)2 +(z+6)2 • 108; sphere centre (-6,6,-6), radius
6,'3 units.

CHAPTER 3

� (page 90)

J., {.!!.) (1) ·ocm (11) 4 Cm(iii) 1ocm(1v)l8cm (v) {T 2 +2T(6T)+(6T)+3T


(1!) (1) After 2 sec (ii) after \(ill-3) sec +36T} cm
(111) no possible time (iv) 1 sec; 4Cmto right of O; never 4 Cm
hO t•I t•1 t,c) V+ t•S to left of O.
(£) (i)I I (ii) ><
.+
1
>t' 0 4 10 It :lD 1t JO "lq!,. l
xat+lt
(d)(i) 3QCID(U) 8 CID(iii) i8Cnt ,
- 2
(iv) 2T(6T)+(6T) +3(6T) :
(£.) (1) 10 cm/sec (ii) 8 cm/sec (iii) i
(iv) [2T+6T+3) oo/sec , • > 4 , t
(!) (1) -7 Cm(11) -13CJn(]!.) (1) 15 Cm(ii) 3 .cm(£) (1) after 6 sec
(ii) afte 1/ � 7/2 secs (iii} v es of t x
f.1. 1 l .1 ' �� ���
.:.-....:-------.;...;;�:....::i.·--· __._._......'"""�
W (1) , ,., t•o (ii) :
a.•:b l,1 .9 o' a 1
2 -Bh)"Clll
(1) (i) -6 Cm(11) 16:m(iii) (4th + 2h !
{A) (1) -6 cm/sec (11) 8C!Jl/sec(11i) (4 t+2h-B)cm/sec
(!!) (1) 6CID (ii) 8+2•10cm(!) (i) 6cm/sec (11) 1 cm/sec 3t
1 13 ,r 2 ,r
(!) (i) o (ii) \ (iii) 7i (iv) 1 (v) 2 ® (1) , sec
3 3
4,
(ii) ,r 3,r 5,r 7,r
• •
""i; ""i; ""i; sec (111) when t • O,w, 2,r sec
3/i
£) (i) 2ff 1 1 ,r ,r �
c mlsec (ii) (72 - 2) t (4 - 6) • 6(,2-l)/11cm/sec

=--
(
r------t.•lft'

(
i)
: .•.
�-,4
1·-- !:" ,• __-41;'i (ii) -�--�----••
.i°"lll__
' ;c

(i) �Cl!\lsec (11) f,,c m/sec •I

(1) o cm(ii) 13 rm® after I sec (£) 1cm@ 14:m/sec


,r
VIII
ANSWERS
(!_) X -+ +<,, (!..) (i) � ,t.•'Ji �··Jlj' t.•11' (ii)
� •111: Jj : "
<.!!.> (i) o cm(u) 1-::m (iii) -1cr,l(iv) lof 10 2• 30cm +
(v) - 4 log 10 +
-9·2lcm .As t -+ O , x -+ -
(!1.l (i) after e 3 +
20·0� sec (ii) e-2 ·135 sec +
(iii) e +
2•72 sec and e-1 � ·368 sec (iv) 1 sec
6 t+ot cm
(s.> (1) log(2) +
1·09 cm(U) -2+3 • 1 cm(tii) log(--) 0·c,c;.--'_......
t
(.!!) (1) l.; log J .i, ·27 cin/sec (ii) l+- (e-2 - e-3)•e 3 /(e-l)":"ll·70 cm/ sec
1 t+6t crn'sec .x
(iii) 6t 1Og(- -) 1 (!.) I--------
t --. '
(a) (i) lr.r- (ii) e +
2·72Cm(iii) e 3 20·B6Cm l4 + :
(iv) el, ; 1·65cm Cl!.) (i) 0 sec ,
CH) log J +
l • 099 sec (iii) no values since ., _,
t • -log J, which is< 0 (iv) 1 sec

L_
t
(s) e � O has no real roo t s
h
(.!!.) (i) ',(e Le) +
8•68m/HC(ii) e (e -1)/h �,.,.,._
t
"
(e)
- x 7. (a) (i) (a) 1/3 cm (a) l/5cm (y) 2
- (ii) (a) 1/3 sec (B) 1 sec c, x,'h
(111) -1/12 cm/sec
�·e' -
ill (i) (a) 4 cm(a) ocm
' 1 t

,�:�
(y) J/IT/2 f 4·98CDI
(ii) (a) 4 sec
(a) 216 4·90 sec +
(iii) -lm/aec
0

� {page 95)

If") 2. v = 4t-3 (j) (a) -Bm/sec (b) OID/sec (c) +12mfsec


(i!) 2-sec; -15 m (iii) (a) Jl; sec (h) !; sec (c) 1, 3 sec
(!y) (a) [ 2 0-(-4)) • 24ID/sec (b) 4(6 t )n/sec
(y) (a) 4m/sec 2 v .l· v • -3/ t 2 (!) (a) -3/4m/sec
(b) 4m/sec2 "•4t-l (b) -121D/sec (ill) (a) 1 sec
(t) never v •

(.!.Y.) (a) J61Illsec2 :'


� 6t+Jh "" ·Jft,
(a ) Z e _, , ..i·J
V • et -I t (t+h)2m-/S ,:-...:t.
(!) (a) Jqifsec (b) e2 '5 +
12•18111/sec CU) (a) 1 sec
(b) log 2 +
·69 sec (!.!!) (a) e +
2·72m (b) 2m
(iy:) (a) (e-1) f 1·72m/sec; (e-1) 1·72m/serf + V••'
(b) (e�-e') +
47•2lm/sec;\(e -e )
4 2
+
23•6Cll1/sec2 �
I t
v = cos t (!) (a) +la>/sec (b) -1/fi:,Vsec
5 7
(c) +/J/2mtsec (!!) (a) n/3, 5n/3 sec (b) ;, � sec
311
(!!!) after
2, �
sec; corresponding
ff

-2
displacements +l m �-1m (ll) (�) -wlll/sec 2
0
(b) -JIil( sec2

1'
2n
-f.
SET JC c,... 98)
=
l.· f • 7 (a)
0
f: 7

IX
;. " t
(b)
t
ANSWERS
2
2
(b) -6:m/sec (!!) l's sec;
l· f • 6-4t (i) (a) 6 cm/sec
..:;!µ�-----�
sec (iv) after 3 sec when

t
'Hi) (a) !-sec (b) �

f
4 4 velocity is 7 cm/sec
'=
2 (u) - (c) -·0001 f·6-•t
f. -l/(t+2) Ci) (s) - V
l
ill) (a) 3 (b) 8 (iii) (a) 4
!
0 6 t
-t -
I .t
f--�>'
(b) _!
81.lll-���;;;;;:;ic::.....
32 2
i• (_!) l:ro/sec ;1 C!ll/HC (iii) ffi cm I �ec

5. 2
f • -cosec 2 t (!) (a) +/Jcm/sec\-4cm/ sec (b) -lcm/sec; -2 <;1D/aec 2
n 3w
ill) after
4sec, v • +lcm/sec;after """'i; sec, v • -lcrnI sec
z

Yi
2
(ill) (a) - icm/sec (t • .l!.) (b) - i cm/&ec (t • l!!.)
3 3 3 3
(iv)
· -lcm/sec2(t • .l!.)v
2 v c

-c.,,lt
: "t
*
0 -�

, I /

\If•
t

� (page 99)

Expnple 1. A. ill) (a) Jm (b) om (c) 3 m (iv) -5.n ;.-5ID/sec


(v} (a)-1, 5 sec (b) 3•/s sec (c) 3 sec (.Yi!) (a) 2, 4 sec
(b) 6 sec.
!!. (.!.!.) (a) -2Dl/sec (b)+2Dl/sec (fil) -1(1) (ly) (a) 31-, sec
(b) 2ls sec (y) after 41-, sec, displ. +5/4m; after 11-, sec,displ.+5/4.ID
(viii) (a) 2m/sec 2 (b) 2m/sec 2 (ix) +15m(t m 7) �) lomlsec (t•8)
(xi) 3-(-1) • 4 m; 2m/sec
Q. (y) (a) x-2t+8 • 0 (b) x • -1

cef
Ef!i1!!!!2. le 2
(page 102 )
-25011\ +285m/sec; -72mlsec 2 (ill) (a) +184m; +15301/sec;
-60mlsec2 (b) -l401D; -144 m/sec; -6°o/sec2 (!::'.) 250m; 250m/sec
(.y) (a) three times; after 1,10,25 sec (b) twice; after 5,19 se
(c) once; after 12 sec <.J!i) (a) 78m/sec 2 (b) +4oom (c) -14711l/sec
(vii) 45-(-39) • 84mlsec; 42m /sec2 (viii) after 4 sec, displ.+378 m
after 20 sec, displ. -950.m (ix) -72ffl/sec (t•l7)
(xiii) (f) (iv) x > O for 1 < t < 10 or t > 25 (v) v > 0 for O S. t < 5
or t > 19 (vi) f > 0 for t> 12 (j) 400
�le J (page 106)
·t�(a} at O; 80:n/sec, -32m/sec 2 (b) +96m; l6m/sec; -32m/sec2
(c) -96� -ll2mlsec; -32.nlsec2 (iii) after 1 sec (on upward path)
and after 4 sec (on downward path) (iv) 1oom (t • 2ls)
(.y) (a) 8 sec (b) 5 sec (.l!.!) (a) -176m/sec (b) -80mlsec
(!!!) (a) 64 m. (b) (100-64) • 3 6 m (viii) (a) 24m/sec (b) -16111/sec
(c) max vel l76mlsec + when hits sea; min vel. is zero at highest
point (d) p�&+(-16)) • 16IDlsec (.W (a) after l� sec
(b) after � sec (b
)
(c) P'a
It o I t.
I I

I -,a.
-176

ll
ANSWERS
(a) S/2 < t ! 8 (b) 0 < t < S (c) for all t � (a) (a)-32m/sec
(b) -48111/sec (B) (a) -32m/aec 2 (b) -32Ulsec 2 (xiii) after S sec
when at O on return journey (b) after 6 sec when vel is 1121111sec •
and acceln is 321!1/aec 2 • �) (a) h-96 • -16(t-3) i.�. h+l6t • 144
(b) h • 100
E:z!r'Ze
4 (page 1 07)
1
+
{i�(a) At O; -2 cm/sec; 9 cra/aec2 (b) -(e-4) -·43 cm;
(e-3)/e +
-·104 cm/sect9/e2 +1•22cm/alc 2 (111) (a) t • 2/3 (b) t > 2/3
(c) 0 � t < 2/3 (� (�) l+log(7/10) +
·64 cm (b) 9/70 cm/sec
(y) (a) l+log(7/l0) +
•64 ca/sec (b) 9/70 cm/sec 2
(c) \(4/7 + 7/10) • 89/140 cm/sec (d) 11/17 Cfll/sec
�) (a) x • l0-log31 +
6•57 (t • 10) !b) v • 13/16 (t • S)
(c) f • 9/625 (t•8) (vii) (a) x+2t •.O (b) v- 9t+2 • 0
(c) f+54t • 9. -
E;egrrple 5 (page 107 ) (vii) 1f sq units
� 6 (page 109)
(!!) 1 °-;;;-; Ooa/HC • 4 cm/sec 2 (!!!) -lcm ;-211cm/sec ;-4 cm/sec 2
<.11!) t • 11/4, 311/4, Sw/4, 7w/4. Pis at rest initially and after 11/4,
3w/4, 5s/4, 711/4 sec. W when t • O, x • l cmJwhen t • w/4,
x • 11/2cm when t • 311/4. x • -J11/2 cm;when t • 5Tr/4, x • 5n/2c:a;
when t • ; 711/4, x • -7w/2 cm
(vi) It\

·"' & , ffta ' Na. 'r.

·>\ ·-------- ------- --------- - -- -- -


(� (a) & (b) hoth 4 times: (a) & (B) both on 4 occasions.
11 3 5 7
(viii) (a) < - 1 >+<1!!. + 11>+ cs" + 11>+< + 11>+<1 + 11> • 1611cm Cb) l.!!.cm
111
2 2
����---------
2 2 2 2 2 2 2
(ix) X - l • 411(t-n.)
Ezampie 7 1page uo)
(fil) (a) l km/min • 67'ikm/h; I Knl/min2 (b) 0 km/min;
f
"ti
km/min (!,u (a)� km/min • 30km/h (B) '.km /min • l�km/h
1
2

t
(Y) km /min • 20km/ h (y) (a) km /min •16 \km/h
�) l,ikm
t
(B) O km /min ; k.GI /min • 221,km/h
(ill) a fter 2 min, v • k.m /min • 22'tkm/ h and f• 0 km /min2 ; after
(2-/J) min, v • -i km /min • -45km/ h and f • ill km /min2
(viii) (b) " •·•)(M)(c)' 4 >i
-- (a) "
x•t(•·f(M) _\�--;IF---�-
-�
<.!!!.) (a) - 1 km/min • -67'tlutt/h (t•O) (b)
9 8
t
km /min • 221:l,.m/h (t • 2)

(c) i km /min • 671,kla/h (t•O) (d) O lc.m/111in (t•l,3)

(A) After 11, min, car is accelerating after leaving A; and after 2'1 min
car ia decelerating before reaching B,
XI
ANSWERS
5 -� 7 27 9 7
(.1!..!) (a) km , k.m/min • 16akm/h (8) km ; km/rain•l6 km/h
64 32 64 32 8
(� (a) OS ts 3 (h) 1 St .S 3 (c) 0 St .S 2 v
(xiii) x - t = �(t-2) (xiv)

CxCl!11ple 8 (Page 111)


.,
(.!!) (a) 4 cm/se c ;-3 c![!/se c 2(�) 3cm/s1;c ,4cm/sec 2 (ili.) (a) x = o, v = 5
(b) X • •5, f c; J (iV) (a) X = -2 ; V = nr (!,) X = 2 ;
v • 121 (y) domain -5 s x S 5, range OS v � 5; max vel 5 cm/sec
when x c O; min vel ocm/se c when x = •5.

SET 3E (page lll)


3
!· (!) 41,cm 4cmjsec2 (after 2 se c ) (!!) 6cm/&ec(t=3) (ill) 2cufse c (t=l)

"' •. ,..,,.,, • ''.'. ......... ,•


and -2Cmfsec (t=2) (iv) (a) (4-0)/1 = 4 cm/sec (b) 1,(6+2) = 4 cm/sec

<;o <•> l,
(v) (a) x = 2!:;CID(t=l,) (b) x = 21,cm(t = 21,)
,'°]I

t·• :' I
(vii) x-4 = 2(t-l) (viii) zreatest velocity is 6C11/eec(at O on way
up and on way down); least velo city
is Ocm/secat top of in cl�ne (ix) 6 dist = 41, c-m
[
.t
a = -3, b = 24; (i) -6,1!l/eec 2

(.!!) after 4 sec; +48 cm


(!) Pis momentarily at rest at O; the positive ac celeration
(3n 2 Cll!/se c2 ) produces motion in -(ill) +6Cm (y) 3n cm /se c (after

::,n:,�JS:·�:: ··u
tA
f\ I\
1 3 5

0
J '\T\TV r
, ,. > ,.. -·;,
(vii) 3rrcm/secwhen 3.- ,ufrom O· (ix) 12cm
·
(�) (i) 2 - l.og 2 � l· 3lcmJJ,1"m/sa c; li.cm/se c2 (ii) lcm;lcnt;e c2 (t = 1)
( )

11 ' . .
(ii) f > O but f � 0 as t � +<». X•t-�t
• -�
H< le'!'
(!!) ' • -4
'"'cm/sec
) ; ) 16,_ 2 (iii) Ocnfsec; oc
cofsec • '•> '"'
_, •• , ... ,. - _ "t:e c 2 � , t. ·
(iv) -·07:lcol,ec; ·29 crrtsec 2 . (y) (a) -4(1-e 1) ,; -2·53 .cm/sec
(b) 16(1-e-1) � lO·llcm'lsec l
(vi) t (vii) (a) --i,.cm /,sec
, x- .·•t �t:::t.: (b) (2 log 2 - log 2) 69 se c

t -4
V'·-··-·"
O t, •
(i) c=4, k•-1; l�ec 2 (ii) (a) when t•O, f•32 when t • 8,
f-. -32 (b) t(B·t) 5;2 cm (fil) 96·3 cin-/sec 2

�,!t
t
J.· ) (h)
(!) (a) f • •600Cllf;ec 2 (when x • i
3 max vel 2�ec(when x•O)
}>
t t>
min vel �e c (when x • • (c) •60Gcnfiec 2 when x • + (Note v2
exists only if - .S x S. (.!!) (a) c • 0 (b) x • -i, ·v· • ' 3
8. (i) (a) b(a+b) (8) ab
2 3
(!) (a) [3(6t)-6t 2 (6t)-6t(6t) -2(6t) Jcm; [3-6t -6t(6t)-2(6t) ) cm/
2 2

sec; (b) [3-6t2) cm/sec (.!!) (a) 6v • [-12t(6t)-6(6t) ) CtA/sec;


2

[-12t-6(6t)) cm/sec 2 ; (b) -lit cm/sec 2 (iii) (a) [-6t(6t)-2(6t) )•


2

-•1202 cm/sec (b) -6(6t) • -·06 cm/sec 2


XII
ANSWERS

.!£· (a) f = g(l-x) (b) (a) max vel /scm/secwhen t


g
= � and xal
n
fl!) max acceln gcm/scc2 when t=O, x•O .2!'. -@cm /sec2 when t = 7g• x•2
E:ra1trple 5 (page 119)
(l) C1 = 3, Cz = 8 (ill) (Je-4)/(e-'1) C111/sec (iv) (a) v = 3 - t;
1 1T
B is (
3,
0) (b) - - - - - - (c) (a) v -+ -m [particle is in-
itially moving towards O with very
..,+¥--"'"'---'"-L--+&reat (infinite) velocity]
t (B) at rest (y) v -+ 3 [particle
is moving away from O with ever­
increasing velocity, but never
attains velocity of 3cm/sec. (d) l/t 2 > O; in positive direction.
dv dv
Slope of v-t curve is dt'
and since dt
> 0 • ·. v is always increas­

ing as t increases.

i,;xample 6 t<t•-,)
t
(page ·119)
(fil) -2 cm/sec :,.41,cm (iv) (5 - log 2) cm (y)
;i,.,o
o c,./al"

Ex=le 7 (page 120)


(!) (a) 0 � t < 2 or t > 3 (b) 2 < t < 3 (c) t = 2 or 3. When v > O,
body moves in_,.; when v LO, bod� moves in..- , and when v • 0,
body is at rest. (.!!) (a) max vel 18cm/sec(t c O); min vel ocm/sec
(t =
2 or 3) (b) -3/4cm/sec(t = 2\) (iii) -3cm/sec2
(iv) (a) �ec• (t=4) (b) 15c!lfsec' (t•O) (v) (a) 14 cm
(b) 14+1, a 14!., cm (c) 14+1,+-21, a 17 cm Area 17 sq units.

Example 8 (page 120)


(!) C1.= -1/24; 85/8 (ii) Cz = 2 - 1/n; x = - l,cos(2t-l) + l sin TTt+
ff
(2-1/n) (iv) (B) x-; 332
================
� (page 122)
2.
l· after 13 sec l·
(2_) -15/4 cm/s ec (b) after 4 sec; 30cmto rieht of 0
(£) after 10 sec when vel is -221, C111""/sec
4. (i) 5mfmin (ii) 15m/min(t=2) (ill) (/ii-1) sec (iv) 20.m
I- (i>
(l)
(i) afte12ohsec, height 4 0m (ii) afte, )jsec, 28:m/sec +
4<

(i) after llsec; !2_:m.above O (ii) after 10 sec, 77'm/sec +


6. k • 4; 15m 7 2
!- When fan, v-2Jcm/sec;26cm;after 9 sec returns to starting point;

�-
2 • 108 = 216 cm (!:!'te particle stops after 6 S<!C)
p • 1/3, q • 2; 27cm/sec2, 13/12Cm. (!) after 1 sec, }cm/sec.2.·
(.!!) 4 cm (.!!!.) 6Cll1Sec 2 when t • 0 or t • 2 (iv) 2 cm/sec
2 -1 (ii) 0 ( ) 2 (iv) 25 (y) 3
.!£· (!) 1 - j; fil 6°J
- l,log 3 (.Y,!) 0
9
11. (!) l,(1-e-l) (ii) lcn+2) (iii) 3+e (!y) e-e-1+21 11'
8 ff13 v. Ht-zf
3
ll· /!> (a) 2n (b)
1;
(c)
2 ).'
\(.!! ) distance (28 ) is magnitude f area. t.....,.�..,/
';,
9
ll· (2_) after 2, 5 sec;
2 m. <.!!.)
(i) 10:m/sec�n � (when tzO)
(ii) O m/sec(when t•2,5) ·4
(iii) 9/4 m/sec(when t • 7/2) -10
2
(£) (i) �m; (ii) f (iii) 1 Illa m. ! (iv) c
26

3
+
2+
2
11)
6
• 15 m. Area

is 1 5 sq \lllits. (J!.) 3 m-/sec •0-3 W/sec2 2

XIII
ANSWERS
511 511 /j
(A) Wher,. t • ....l!. X • ....l!. + l3 f • -213•' when t • , x •
12' 12 2' 12 12 - 2 ,
r::: 1311 1311 13 1711
f • 2,-3; when t • -, x • - + -, f • -2,3; r;;
when t • ,
12 12 2 12
1711 /j
X • l2 - , f • 2/J (.J1.)
2 )(

Aver ve1 • (4+11)I 11 cm I sec .,.' +,+.-.;.;:;....,--...,,..........-�;.,_-_..,.,,.......


Iii • ' ,l
.
(!) 11/6; 5; -1 (ill (a) 17f units/sec (b) 94/15 units (fil) 7
.ll· 89 35
(iv) 9t Ji·
3
(J) -3 (!!) (a)
2

(b)
15
(ill)
4
- 2 log 2
(iv) x• -\e
-2t
+21og(t+l)-2(t+l) 2+tant + �(1-5t) 10 + ��
5
(i) after 4 sec; -64cm/sec (ii) 27cm(after 3 sec)
<I!.!.> max positive velocity I; 16CID/s ec(after 2 sec); greatest
speed is 64cm/�ec(after 4 sec)
40 6400m 8ol!y' ,.m .
(j) 150m. 1 25D, I sec (!..!.> sec; ; sec J.2.· 12111; 1-.1sec;
3 27 3/
16, I sec 2
lQ.. 162 cm
��=================-==
� (page 130)

-_!. (i)
- t • .!( x -1)
2
(_ii) t • � log X (iii) t • �i(..!2 - 1)
6 - X
1 1 -I -X -J 11

(iv) t. 3lsin x-sinl) � ( ) t • al e -e ) (�i) t • L
,,sin .x - or
4
t • - l;cos -lx (vii) t • tan-Ix - .!!. (viii) t • tanx - tanl --
4
(ix) t • tan x - x + 1 - tan 1 2. (i) v2 • 2x 3 (ii) v 2 •log(2x+l)
-x
(iii) v 2 •2(e -e ) (iv) v 2 • sin 1 (2;) (y) v 2 • tan- 1(2x) 3h "-
x ':"
- - 5, ' i
4
(vi) v 2 • 1 - cos 2x (vii) v 2 • (x + 5x - Sx ·,
5
-1, •2e � •5•4 units/sec 4. -8 = (Note: starts from x • 5 and is
moving towards C at • 3) l.· (9 k •
9
2
x

(ii) (100-75) • 25 6. (i) at x • 4 (ii) max B when x • 2


- 2 = 4(1 - -),
1 - - · 1 - 4/; If;
J· Note v
2
X
2/6
As
X
x 8. (a) � � (b) � �
increases, - + 0
3 3
%: v 2 • l-cos2x - - sin 3x; • -
3 4
3
4 1
3
10. v2 • e4 +'log 2 (+ 58• 76)
-= 1
.ll• [k • ; C • 3; t • (1 - i7)J; X • •/'i 12. [t • 3x 2 +1]; x • 1
8 3
ll· i 3

t
13. t m 1 + l,log2 (+ 1•35) sec for P; � sec for Q.
2
16. (i) - (ii) 1(1 -
e-3) •63; ·+ 1
sec since as t + "'• e
-3t
+ o

� (page 145)
(.:!) (.!) 116cm/sec (.!.!) •813cm/sec
(i) (.!) 16i:m/sec,;64�m/sec2 (ill 4cm; f sec
(ill) +21'icm.-81'i cm/sec
11
(iv) (a) after sec (b) after sec.
11

12 6
3. (iii) (a) (a) •61'f cm/sec (B) -12cm/sec (b) (a) 12fi·c m/sec
2
(;;-36ficm/$ec2·(y) 4/'i·cu, (£) (a) zl'i cm (;) 412cm (!!) ; sec
1 1111 11 1911
(!_) first time is after (a) ( - ) • sec
3 6 4 36
1 Sn 11 7-n 5 (iii) 11 sec
(B) ( - ) • sec �· (.!) •S (.!!)
3 6 4 36 2
32·m,•in !·
3cm/sec ;6c� (For second case,�2 • - i.x 2 + 8, since

XIV
ANSWERS
v • 4 when x • 0); 8 cm J..· 10/inv.mtn; 7�1Va1n2
8. a • 16 - 9. (H) •1s·c'!l/sec - 11. (a) 1! (b) (1) I-an sin(nt+t) I
(U) n� (c) ii • -n2 x ,U_. (i) ii • -n2 x; v2 • n 2 (a2 -x 2 ); 2: sec
n

(iii) a • sr.11�11 sec; 1ocm/11ee ll· (i) O; -1211 2 (U) 3; 611; 1211 2
.. .• !11
14. x • -9(x-ll, i;e. X • -9X, where X • x-1; at x • l; sec; 4 cm
211 s 2 1 3
15. (a) 73' sec (b) at x • - (c) lt("j) • 2:j· ca
3

Set 31 (page 154).


f
L. (.!i) 24 m (fil) 1 m (b'.,) 1:3 � 2-rrff sec; sec l:.(!:!'..H!.) 3n m
(b) 6n m/s (c) 3 m to left of O (d) after 11/8 sec ii. 4n/6 sec; *PQ=
2/'l m � dist;nce = (5 - 3) = 2 m; time = ! cos-1 (i) � 0.12 sec
§... (j_) k = 72 g = 720 N/m (.ill.) (!,) 2n/v'l!f sec (!!,) v'l!f/4 m/s (£.) x = \ cos
(v'lll't) (.!!) � m z. Remember to take axes at A not O; time to return to A
is It time of one oscillation=�. 2n where n2 = A/(am). Greatest extension�
v.,ff when v = O; x =,ff. V sin.fa�. t; V .jff + a

Set 3J (page 167�


1 (i) H = u sfn a (ii) T
2
= 2u sin a (iii) R = u sin
2
g
2a
-- Zg- g -
t 6 _ u sin a - gt
(b'.,) (!,) v = r'(u cos a)2 + (u sfn a - gt)i' an - u cos a
(!!,) v = r'u' - 2gh, use result of (a), noting h = ut sin a -�t 2 ; tan,=
/u2 sin' a - 2gh/u cos a. (y) R max = u2 /g when a = n/4
A... (�) 5 m; 2 sec; 20.'r m (!!,) 45/4 m; 3 sec; 60 m (note sin 2a
= 2 sin a cos a)
I!... 45/2 m; 45/8 m (note a = 45 °)
J;.,, sin 2a = �•• ·. a= 15° or 75°; 90(sin 32° - sin 30°) � 2.693 m
LL.a(!,) 490..-TI' cm/s; tan e = 1/(2,'J) ie e � 16° 6' (!!,) same answers as (!,)
L (!) 28 m/s; 45 (ii) 20n17 sec (iii) 20 m L, 200 m/s; 120 m/s (here
°

y = 0) ,(k, (!) 45° {j_!) 15° or 75° !!.,. (!,) 1 : Ir (!!,) 1 : 1


,L. (!,) 1000.'r m (!!,) 24° 18' or 65° 42' .il.r 100134' m/s; 14° 2'
JS... The two elevations give the same �ange; required area is between 2
concentric circles, radii u2 /g (when a = 15°) and u2 /r/2g (when a = 30° or
60°); area nu4 /4g 2.
� (�) (j_) 13 sec (jj) 1040 m (.!!.) (j_) 420 m (jj_) v = 75 m/s; e = 100° -
tan-1 (3/4) � 143° 8'
� (�) 62� m from A; 25n m/.s (.!!,) 50 m/s; 10174 m/s; 180° - tan·l (7/5) �
125° 32
i,. (j_) 30 m/s; 40 m/s; 50 m/s (ii) 5/S'5'm/s (iii) after 4 sec,80 m (iv) - 1800
- tan-I (4/3) � 126° 52' (y_) y = m{240 - x)
.§... (j_) 500 m (ii) 2000/rm; 20 sec (fil) 20lrrm/s; tan-1: (4/3'/15):
24° 47' in both cases when ·the height is 180 m
§.., 180 m; 15° or 75 ° � tan-1 (4/3) � 53 8'; 5/2 sec
°

§... U..!) x= 30, y = 2(15/3' - 10) 2.., x = 10, y = 0

xv
ANSWERS
CHAPTER 4
� (page 178)
a
l• (!) sin(7x-6x) • sin x (�) sin 2.(36) • sin 66 (_£) cos 2( ) cosa
2 •
�) cos(5A-3A) • cos2A (e) cosl06 (f) sinl26 (.s_) cosa (h) sin 2 3A
(l) cosl4B (j) tan(4C-3D) �) tan6x -Ci) tanA (m) tanl66-
(n) sinl6 0 (2) cosa
l
(.f) cos (2A+2B) (.9.) l,sina -(_!) 2 cos 2
13 n /j
I
(!) sin45• • 7-r_ �) sin60 ° • (..£) cos = (_!!) tanl35 ° • -l
2 6 2
·73 l
(£) cos30 • - (!) tanl50 • - 7:i (&) tan225 • l
2
° °
3
°

1 sin30 ° l
(l!) sinl35• • -n_ (l) o = 7.i' (i) cos60 • \
°
cos J •
n 13 13 cosl60 °
(1) cos( + ) • cos = (l) cos210 = - Cm) -1
10 15 6 2 2 cos 20• =
.l[
11
0 0

cos 5n/6 -16


( ) • (.Q) sec 1!. • .fi (2) sin330 • -\ °
n 4
�in 1.!!.
4
{g) (�inl35 ) 2 • l (.[) l I (�in45 °) • 2.fi
°
8
l 2
(al sin 75• cos 2 75• • � sin 2 150 ° • <!!) 2tan30 ° • 73 (£) 2
16
2

u1> a • %•
b • -�
W 1 !:.: ) )
UJ. l� ill! �� (!y
)
!�
ur) �� W>. �; W!l �!�
{v iii)
1
; (g) 1�;
4
� �> -
5
W> 7irf Cxli) 75'
l
Cnii) I,( '13-3) =
7 23 17 12 l l
W - 9 W.> - 21 CW> sI lli> / "j' 0!> 73 W> <.!> 7i.
1 1 . 120
Oi>rz i· Ca> W W> - 5
s 01> 119
(J.) fi-1 W.) other root (-namely -(.fi-1)} is tanl57l, 0 since
cos315 ° • "'fz also. (ill) 2+13 �· (J) 48 ° ll'

,< -�-
� (page 181)
(i) y • 2(1-x2 ) (ii) x • l+y (x > 0) (iii) y • 2x/(l+x 2 )

�- • ;h . - -· r
� � ·i
l: (Your answers may have different fotll18 to those below.)
(l) y • �(2x 2 -1) (!!) x • 1 - -:t::.. ill!) y • 2x/(1-x 2 )
2

8
3 5(3x-x 3)
(!y) y • 3x-4x (!) 2x • y -Jy W> Y • l-Jx2
3

<.i.:i!) y 2 • 4x 2 (1-x2 ) (viii) y 2 • � 2 (4-x 2 ) (g) x • a(� - 1)

(x) x 2 • 4a 2 y(b-y)/b2 (z!) x • a(l - �) �) x = ��!Y


a(b2 -lr"2)
2

�) X • .!1!!..:::tl. ur,!) b 3x • a(Jb2 y-4y 3)


Y
(MU) X • bz z b+ Y 8 4x
(m) a 3y • b>1(4x 2 -Ja 2 ) (xvii) y • i2' - 1 (xviii) y • 2
+y

x _4
(i) (x-2) • 2(1-y)2 (ii) x-4 • -2(1-y) 2 <1!.1) x • i!:t=!l. 2
4 ( -l)
x-1 1-x 9- 2 64 - y
lli> - -. 32y 3-6y C:i> -s - . _.;2 W> y2 • e1<x-1)2 (9-Cx-1) 2 l
2 9
4. (i) x • �(a+b) + �(a-b)cos20; y • �(a-b) sin26. Use cos 2 2e+sin 2 26•l
2
:.
giving [x - �(a+b)J2 + y 2 • �(a-b) 2 (11) x 2 + � • 2
X

XVI
ANSWERS
SET 4C (page 184)
(.!) k • 2 (!!) a O
i, b • I,, c =i (.!.!i) 4s2 (1-s2 )
(i) 2 13. (i) b • 9 (ii) k • !
- (l+t r'. - 2t 2 2(2t ) 2 4 l-t 2 1-t l
(!) l+t� (!!) l+tz (iii) l+tp (iv) � (y) l+t (vi) t
. ·t 2 +5 . 1-t 2 Ci+t>O+t)
(vii) t2_8t+l (viii) l+t (ix) 2(2t +2t-l) (� l+tz

�ET 4D (page 188)


[To save space, answers are given alternately in degrees and
minutes, and in radians (correct to 3 dee. places).j
co 45•,135•,22s•,315• cm> .201, 2·94o, 3·872, s-5s3
° ° 11 Sn
(!!) �3 k', 306 52'; 120 , 240
° °
(Q.) 4 4; 2·897, 6-0 3 8
<iv °
60 . 300· <su2
t,
11 411 Sn n Sn.'711 .311 -' 9 o
;
3'3'3 4' 4'4 ·�(J.J
•666, 2· 476 (!) 4s ,22s
0 0 ° °
O • 270 o
63 26', 243 26' <0 ·245, 3·387;
;

2 •159, 5• JOO
J: Cv 30°'330° ("' !.1'!. � lJ!. (� 26° 34', 206° 34'; 1 35 ° ,315°
lln "' 4'° 4' °4' 4°
!. 11!.
6' 6' 6 (-' "-' 0 ' 180 ' 360
3
(1) ; <v
45°,225° ;18 ° 26' ,198°26'
2n,_ 411; 1·231, 5•052 (i) . . n Sn 711 ,-lln ; O,n,211
no solutions (j)
(h)
3 3 6,6,6 6 411
(k) as for 1 Cl) ("' !. � (m) 120 58' ' 300 58' (u) !.
° ° .fl!. �
!&.14•43 3'3'3'3
YJ as for 1 Cl)
(_, -' !. "· !. 211 (-' 0° ,180° 360° ,· 5 3°8',233°8'
(1t1J 2' 2'3'3 � '
(.r) as for 2 (n) (.ILJ 210°,330° ; 14° 29', 165° 31'
° ° °
(.0 0,2n,2•214,4•069 (y) 161 34',341 34'; 71 34', 251° 34' (y)n
(l!l) 180° ex> as for 1 (q) (:£) 60°,300° ; 120" (� �. �.
3 7
° ° °
1·249, 4·391 (A) 135 ,315 ; 63 26',243 26' OD °
f,
5�; •675,3·817
3
(� 204° 27', 335° 33' (12)1·263, 5•020 CID 30°,150° (I) O,n,2n; l"
°
C.G) 63 26', 243 26'
°

J.· (1) as for 1 Cl) (.il) 45°,315° ; 60°,300° (ill) 90°,270° ; 45°,225°
Cm 71° 34', 251° 34• C0 60° ,300° ; 180° Cill 60° ,120° ,240°,300° ;
45°,135°,22s 0,315° Cill.> 135• ,315° ,18° 26',198° 26'
(Yiil.) as for 3 (v) (ix) 30° ,150° ; 19° 28',160° 32' (iu as for •
2 (q)
(xi)0°,360° (xil) as for 1 (t) (iu.JJ.) 45°,225° ; 26° 34', 206° 34'
° ° °
(xill) 135 ,315 ,120 58' ,300 58' (x:,z) empty set (.lllli.) as for 1 (r)
°

(.ll.ldJ.) 315° (mil) 45•,22s° C.lliJu o•,360°; 45•,135• ,22s 0 ,315°;


90°,210° cw 45• ,22s•; 30° ,1so•; 19° 28',160° 32• (x.ll.1) 45•,225°

� (page 191)
(.!) 1s 0,1s 0,195•,255°; 45°,225° (!v 1s 0 , 75•,135° ,195•,255° ,315° ;
48° 46 1 , 108°46', 168 ° 46', 228° 46', 288° 46', 348° 46'
C.!v 60°,240° ; 1so•,330• .1, C.!) o· ,180° ,360° ,109° 28',250° 32•
<!!.> 180 °; 60°,300° (.£) 90 ° ,210°,330°; 20° ,100•,140•,220 ° ,260 ° ,340°
<.!!> 15•,15°,105°,165°,195•,255•,25s•,345•
(.!) 60° ,120 °,240•,300 ° <.!v 104° 29• ,255° 31' (£) 221i•,67\ 0,1121, 0 ,
151i,0 , 2021, 0 , 2411, 0 ; 2921,•, 3311, 0 @ 210° ; 14 ° 29•, 1 65"31'
(�)no solutions in domain l• (.!) 0 °,180°,360° ; 30 °,150°,210° ,
330• (.!v 0°,60 °,120•,1so•,240•,300•,360"; 10•,50•,10° ,110° ,130•,
110 °,190•,230•,250•,290•,310•,350• (£)35•32•,215•32•·; 161 ° 34',
341° 34' .l!.• (.!) As for 7 (a) (.!v 90 °,270 ° ; 45 ° ,135°,225 ° ,315°
(£)(.:!)as-for 5 (d)+ 0 ° ,90 ° ,180 °,270 °,360 ° · (.!!) 270 °,_1 35 °
XVII
ANSWERS
C.!!!) o•,60 ° ,120•,1ao•,240•,300•,J60 ° �·,1so 0 ,J60",15•31•,
284 °29' (!,) o•,1so0,J60•,45•,135•,225 ,315 (iv).�·.2n
<.!> Jo•,90•,1so•,210•,210•,330•; 10•,50•,130•,110•}250•,290•
(.!?) 60•,300•; 1ao• W 90•,210 ° <.!!> 0 °,360 ° ; 90• (!) 63 °26',
243'26',161° 34',341 °34' (1) 45',225 ° ,63'26 1 ,243 ° 26 1
!!· (.!!,) 1·88,-•35,-1•53

� (page 197)
<!.> 60 °,Joo• (!?) f. 5
!"
1 1
;, ;, (£) 20•,100• ,140°,220•,260 °,340 °
(!!_) n (!_) 135 ,315 (f) ° °

3
3 1
;. ;. ! 1 11 1 11
, � (&) 45 ° ,105 ° 165 °
225 ,285 °,345 ° (!!) ; (.!.) 45 ° ,135 °,225 ° ,315 ° (j) as for (f) +
°

n Sn 9n lJn
ii•'"'"'ii•'"'"'ii•"""if"" (k) - as for (g) + 15 ,75 ,135 ,195 ,255 ,315
° ° ° ° ° °

11 Sn
(-o
...) ..!!.
2 2 'l! (m) 90 °,270 ° ,30 °,150 ° (!!,) 0,11,211,1,3
2 4
<.2> o•,90•,1so•,270•,360° ; J0 °,15o•,210•,330• (JU o,2n, ;, ;
Jn
(s) as for (i) (r) O,n,2n; 2 (.!) 90 ° ; 216 ° 52',323 ° 8'
<!.> {.2;. 4;.s; c�> 0 °,1ao 0 ,J60 °; 11•34•,2s1•34•,1oe 0 26',2ee•26'
(�) ss for (b) (�) o",90 ° ,180 °,270 ° ,360 ° (�) 8& for (r) + f
f.
z.
5
Cl,) 45 °,225 ° <!.> ; C�> 15' ,75' ,135 °,195 °,255 °,315 °
(!) o,f.f. :,11, 3
s:. ;. 1 2n;�(f.) 45 °,225 °,63'26',243 °26'
3w 711
3
• l!. 711
(Q) •464,3•605,4,4 (�) 90 °; 30 ° ,150 °,210',330". 12' U'U'
4,
n 3n Sn 7n
F) 88 for (b) + , 4 (g_) O 0, 36O • ; l 20 O,240.• ll!,ll!' 17•·,
;lff.12
(_ 4,
n 5n 13n 17n 4 3n 7n 12. U
<!!.> TI• TI• 12' 12; 4' 4 (!) 88 for (i) • l�

2. n 311
"" (!) 2' 2; •314,2·827; 4•084,5•341 (ii) 0 ° ,360 °; 72',288 °;144 ° ,216'
-i�-3-.: � ;J·� Ca.IV<
,•f\
I
I \
-:-!+.T.1rl-;; ��1r't;.-=-'...,.....,,,-�,,- (\ "'
,
··I

(1) 2 sinl0 ° , •3472, 2 sin50 ° , 1•5320, - 2 sin70 ° -J.•8794


(!!) 2 cos30 - l; 1 + 2 cos20 ° , 2•8794, l - 2 coe80' + ·6528,
-,

1 - 2 cos40 ° + -·5320 (.!!!,) x • 0•241

� (page 202)
(!) (�) co118 + sine • lf[cos(a - f>
J (.!!,) (j) 12, f
(.il) -12, s :
(£) o, f,211 _l· (..!) (!) 7 cosa + sine • llz sin(e+Sl 0 52')
<.!?) <.!) 5/2, e•e• <.il> -s/2, 1se•e• cs>
53•e•, 323•9•
l· (.!.) �) efn8 - 2 coe8 • /s ein(8-63'26') (.!?) (.!.) ls,lSJ.0 26
(.!..!.) -Is, 333 26' (£) 270 36 52'
° ° °

:, (1) (a) 4 cose - J sine•


<Ii> ::-5, 143•s• <.£.>
210•, 16 ° 16 1
/s'
coe(e+36 ° 52') (!?) <!>
5,323 ° 8'
s. (!) (.!.) t • S; e • 157"23' (.!!) t • 5 12 ; a • 134 ° 46'; and 180" (bJ
trial) (!!,) S sine - 12 cose• 13 sin(&-67 °23')
6. (!) (!) 2 cos(& + 1>
(!?,) 2 cos(& - s;>
(£) 2 sin(& - ;)
7

XVIII
ANSWERS
5
(d) 2 sin(9 + s;) (ii) greatest is 2 when ij • ;; least is -2 when
2n 4n 2n Sn
e • (.ill) 0,211; 3 (.!!) (! ) o,3,n,3 ,2n (!a) o
3
2.• <.!.> 86°34',326° 34' (!!) 10s• ,195 ° (.ill) 76°43 ',209• 33•
(.!!) 82° 3 7', 322•37• w 8"42', 277•34•
!: (!) 90• ,143°8' (!!) 90•,306°52' <.!!!> 323"8' <�
180°,286°16'
{:!) 69 12', 327 40'
° °
1.· 3 3
(!) o, ;,2n (!!) ·644, ; (.ill) •644,2•214
(.!!) •284 W n,3 •710 .12·
(!) 3 cos29 + 4 sin29 + 3;
3 + S co s(28-53 8'); greatest value is 8 when 0 • 26 34'; lea st
° °

value is -2 when 9 • 116 34'. (!!) 65°48',167° 20',245°48',347° 20'


°

11. (.!!) 36 52' (�270 since sec 270 is undefined) (hl 36 S2'
° ° ° °

l!· (!> 323°8' (ii) 0°,216°s2• ,360° (.ill) 97•19• ,230•9•

� (page 209)
(.!!_) 9 cos9x; -9 sin9x; 9 sec29x; 9 s ec9x tan9x ; - S cos ecSx cotSx
(-b) .!. sec l!. tan l!..
9' - .!.
5 cosec l!. cot l!..
5' - ""
L-osec2 l!..
2'
9 9 5
�"� sec x0 tan x 0
180 � 1...
tc) 3x2sec(x3) tan(x 3)·' x ' cosec .! cot .!.
x'
.
ft
x

cosec2 /;;-; 12!+s secl2x+5.tanl2x+5 (JV cosec x(l-xcotx );


-( x c osec2x+c ot x)/x 2; -cos ec x(2+xcotx)/x3; x6 sec x (7+ xtanx)
(!) e
secx x x
.sec x tanx; 3 tan 3x ; e sec(e )tan(ex );
-c se
se c(logx) tan(l ogx)/x (!) e
o cx
cosecx cot x; -2 cot 2x ;
e-x cosec(e-x).cot(e-x) (&) - 3 cosec2 3x . ecot3x; -3ccooset3x c23x
-cosec2(log3x>, 3 cosec23x. cosec2(cot3x )
sec x -cosec x
(!l) --[l+xlogx tanx ]; x(l og )Z [l+xcotx.l ogxj;
x x
x -c osec2x l tan
e (cot x � co sec2x ); 1 + 2 tan x (i)
- co t x
- x; og xsec x '· -2
-2 1
2cosec2 x
"" 3 sec3x tanx; -cot2x . (cosec 2x)J,;
(;) x
(cot + 3)3; 0 co
14 sec7x cot7x
(]0 -2lcosec1 3x . cot 3x; 60(2-5cot2x)5, cosec22x;
(3+4co sec7 x) ¥2
�ecx.(secx+tanx) J,
Answers given are after reducing to simplest terms
(!) J!.
a cosec� (!!) -]!. cot3 1j, (!!!) tana (.!Y) �� cosec 5 e
� 2�
(y) -sec2 e/e 2 3 . (!) c21311+6) (!!) -1;2 (!.!!) -- (!!'..} -1
3 3
II
.!!:£ • o , fi
cix2' • =
i· dx 8; Curve is stationary and concave up at x • 4

t
(i.e. is a min.turning pt) Eqn of (i) tan is y • 2 (ii) normal is
X • * l· (!) 13 (!!) l: (!!) C • 0 (Hint: put X • f>
SET 41 (page 211)
" .!' l!' 511' 711 (!!t 311 SIi 1) (711 1)
.!.· H\ -2 (iii) (4' II
(� 4 4 1) • (4,-l) • (4' • 4'-
11 511 4 4 II ,::. 511 r.. r. 2(x II);
l· (!)
r. 6 6
, ( !!) (6, 2d), (6,-2f3) (ill) y-2f3 • - 6
y+h3 • 2(x - 511 )
6 11
3. 125
II r. !11 r. 711
i· (!) (!) (6 , -fl + 1>. ( 6, fl -
3>
11 511 II 511
(!!) O < x < 6 or
II 6 < x < n.TT (!!) 0 < x < 6 or 6 < x < 11.
511
(!) 6 < x <6 (ill) At x • 6, max t.pt. (grad+o-);
XIX
ANSWERS
5ff
at x • , min,t,pt. (grad -o+)
6 n n
(Q.) (!!) (� 0 < x < 2 (g) 2 < x < n, (c)
ff ll(X
Concave up when 2 < x < ff and concave
I· I ) I
I I
down when O < x <
6
(!!.!) Yes ("iixZ • O and changes sign
through x • 2) I
I
i• sec 2 ;,-3cosec 2 x; 2sec 2 x tanx+6cosec2 x cotx, x •
211 ff ,.,
1
(min);
x a Sn ,.,
3 (ma:·.) l• (!) (6 ; t3) min; (6, -,3) max 311
(ii) (a' .!!. w S l.:!!_ (b) .!!. < x < .!!. .!!. < x < 311 ' Sn < x < 2'
3
:'!/ 4' 4' 4' ,,
n
- - 4 2' 2 4 4
3ff < X < 7n
C' 0 < X < .!!_ ff < X < l!!.
2 4 (:::..t 2' 2
tanx-2x+3; sec 2 x-2; -.!!_,.!!.; both inflexions 2.: 7i' l
4 4
.!2· dl • - 8 cosec0 cote+ 27 secO ta d2 l
dO �
de2" • 8 cosec0(cot2 B+cosec 2 0)+
3
27 sec0(sec 2 o+tan 2 a); cosecO • ; ,

� (page 220)
J.· (Constants have been omitted)
(!) - l 7 cos7x· ' 6 sin 6'. l 9 sec 9x·' - 5 cosec .! 5'. .....!. tanlOx·' -2 cot .1!.
.!

1 l 10 2
(!!) l;[x - sinlOx); l,(x + sinx); tan3x - x; - 1..cot4x - x
10
(!!.!) [Alternative results possible) 3 log(sec
l log(cosec8x - cot8x); 2 log(sec X + tan X
3
{>; f log(sin3x);
8 2 2)

(iv)
180 60 tan3x•· i.[x sin(2ax+2b)J· x
- w
secx•· ' n '
- 2a '
sin2x sin2x
(y) tanx; -cotx; l2(x + -2-); l;(x - -2-); tanx - x
3 4x 1 x+l
(� -tan(l-x); \[x + 4 sin 3J; - cot(3x+4) - x; -2cosec(� �)
3 2
(.!_) O; l;; l; 2-./f (!!) "'3': 1<
l; lf-1); 0 (.!!!) !<n+2); f;
l ,;; 11 n 11 l ,;; l
u<4n+3d); '><2 + l) (iv) l - 4: l - 4: 6(2t3-n); 12<4-n)
ff n n ,.,- ,;; l
(�) l + ; 2 + l; 4 -t3+1 (� log(2+t3); -log(73) • i.1og3;
4
l,log3; l,;log2. J., (.!.) 3 - "fz (!!) J/3+2
(!) C1 • Cz - I,; C1 • C3 - I.. (!!) (!)
3
t + i. (Q.) 8
(.!.) log2 (.!!.) 4 sec 4 x tanx; 4 ,2.• (.!.) l(e-l)
(!) 3sinA - 4sin 3 A; 2 4 l3
<wt
( ) (!!)
4 3 !· .!. 3 l 32(n+2)r;; 11
(.!_) ( ii) I..( n-2log2) ,!2• l - 73 .ll· (a) ,2 + 4 - 2
3

� (page 224)
l,., (1) 4-lf (ii) 2+/5 2. (1) f(x) • 2�sin2x+i.sin 2 x+l (on simplify­
ing); l..(2w+7) (ii)�+ 2 - { l·
(i) y • tanx - cotx
(ii) where y •-{ - 1 - 13 (iii) f (x) • sin -1x + tan-Ix - cos(x-1)+1
or -cos-1x + tan-Ix - cos(x-1) + .!!. + 1 2
xx
ANSWERS

,!• (!) 2+log2 (W X • t log(l+ 2t) + tan-1(}) - sin - 1({) �sec2t-\

n
(ill) •3/2 (.!.Y) 1

l•
II 1 � sq
( !) (
4 72
, ) (ii) (!) (2-t2)
i•
(!!_) ( fi-1) sq I --- - - - -
II II II
(ill) (!_) 2<2 - 1) cu (!!) 2 cu
\
(.!.Y) acute angle is tan-lc2fi) + 70 ° 3 2' • �,
_""""",_u,4x: 211 sq units; 311 2 cu units

l· -Pl -(!) (h"+l) �,u · (!!) t(3•+2) '"


· �w:,-,c


8.
- ,c 9. 10. (ii) I
;� ,c - -\o � X
f2 log(/2+1) sq; �log2 sq units

11
/ 211 cu. (ill) a<4-,r) cu units
-,r
..._____.._____.___ .
�;::Q.,�X. 0 11a, ="'==-========
CHAPTER 5.
� (page,231)
.!.· [Parts (i),(ii),(iii),(iv) are given in order)
(!) 2;5x2 ;-l; not monic (]!) l;-3x;2; not monic (£) 7;x 7 •o; monic
(.!!) 0;4;4 not monic W no degree;O;O; not monic CJ) 7;x\o;monic
i
(&) 2;-7x ;O;not monic (.h) 1;4x;o not monic
(.!) 3;-7x 3 ;-10;not monic (j) 4;x4;O; monic
2. Polynomials are (b),(d),(e),(h) (i),(n)
'J. (!) x2+2x-4 (.!!) x-4 (!!!) 2x Lx 2 -6x+3 (.!.Y) 4x 3 -4x2 -3x+2
1. (!) ao • 4, a1 • -7, a2 • -3 (!!) a • 5, b • -1 (!!!) a•l,b•l,c•O
(.!.Y) a•l,b•2,c•-3 i·
(.!) R(x) • -4 (.!!) Q(x) • x-4
ill a • 4 (b, c arbitrary) (!!) a•3, b {, - f (c is arbitrary)
c {, 2 (.!.Y) a • 3, b • 5, c • 2
5
(iii) a • 3, b • -
2 2
.!!.• (!) p • 11 CW p • 4,!>,6 (!!!) p is any integer ;t 4 •
9. (.!) ( x-1) only.
io. Degree of {P(x)+Q(x)} is less than or equal to the degree of P(x)
or the degree of Q(x); Degree of {P(x).Q(x)} is equal to the degree
of P(x) + the degree of Q(x).

�� :
:�v(p t
) ]

• (!) (i) X • -2,1


(ii) -2,1
(11) 1 �
x 2 -,._ ·• I :,t (iii) X S -2
!(, 3
(iii) x ::. or x � 1
2 2
-3

(.£) ....-(i) no real roots "k .!!) -i (i) x • 1,2,4


11 (ii) no real zeros
(111) all x
(ii) 1,2,4
(111) 1 S x S 2
I hl

h
"' or x? 4

,,,.,,.,. •,.
- .•

'·-�Ad
(e) fl .,..(1) x--1,2 ,3,4 (11) -1,2 ,3,4
(iii) x � -1, 2 � x .S 3
- "' � or x? 4
a
-�
XXI
ANSWERS
(!) . -(!)x- 1,0 , l
. (g) ...(!) x-0,0, l
(ill - 1 ,0, l -· ·- d 0 ,0 , l
I ,.<ill

--r
� (ill) - 1.Sx.SO + (ill) x-0 or

.
1
or x 2 1 '- x ?. 1

(h) (!) x•- 1,0,0, l (!) (!) x- 0,0 ,0, l


(ill - 1,0,0,l (.!!) 0, 0, 0, 1

- --
� ·- ·
, ,.(ill) xr-1,X"'O , :.C!.!.!) x s 0
or x � 1 or x 2 1

D
Ci) - (j) x-0, l J.!0 (!) x • 1,1, 4
1
(ill 0,1 · 1---. ·� • (.!!) 1, 1,4



0 I (ill) X .S O p,:- ?C (ill) X • 1
-u
or x l l or x � 4

x•l,l,1,4 (!!!) /(j) xml,1, 4, 4



<W 1, 1 , 1, 4 b<W 1, 1 ,4 , 4
(.i.U) X .S 1 (.ti!) all X
"'
or x 2 4 ,.
wtrf,_.,.(j)
(n) l! /,_(i) X"l, 1, 1 (gj- I lj. �j) X"l, 1 ,1,1
/
_ (.iJ..) 1. 1 . 1 (.1J.) 1. 1. 1 . 1
p1 ,: (iii.) X 2 l (iii.) all X
-I
I

P�--/;:;
o .._ �

"· (.l) ,•1,, (.W 1,, (ill) , ' l o, , ' 5 'l· --


� )
\
('
-1

i-fil::�lr\ o::�i==- .

<.!.> ,3, ,rs <.!!.> <.!!> •1, •2 <E.> •fi


(_!_) X • - 1, 1,2,4 (!!) -1,1 ,2, 4
-1 � x .s 1 or 2 .s x � 4

(1) b 2 - 4ac < O U!) b 2-4ac 2 O


(the zeros are distinct if b 2 - 4ac
(.!) 2 real zeros; 1,5 (.!!.) 2 rea l zeros; 3,3 (£.) no real zeros
(JD 2 real zeros; 3•/f9 9. (x+3)(x-2)(x 2 +x- 1); -3,2,l;(- 1 •"5)
10 . -t

-(.!)7:;�ili!
4\�' .. /
ill)�- '
0 "'

-�:
- -

(0, 7) Max No stationary pts.- (0,2) horiz. in�l exion


(2,-1) Min Inf lexion at ( 0,2) ( 1,5/3) Hin
(a) 3 real zeros ( a) 1 real zero ( y) no real zeros

� (page 244)
All results in questions 1 ,2 are of form P(x) • A(x) Q(x)+R(x).
On ly the right�hand side is given in the answers.
(i) (x-1)(2x-1)+(-2) (ii) (x+ l)(5x- 12)+(15)
(iii) (x- l)(x 2-4 x- 11)+(-1 4 ) (iv) (x+ l)(2x 2 +x-9)+( 13)
(y) (x-2)(x 2 +2x-3)+(-9) (�) (x+2)(3x 3- llx 2 +22x-5 1)+(93)
XXII
ANSWERS
(vii) (x-3)(7x 3+ 2 lx2+60x-2 )+(-10)
(viii) (Jtt-3)(2 x4-6x 3 +1sxZ -4sx+l44)+(-433) (ix) (2 x+1)(3x2 -4x-2)+(1)
(i}"'(3x-1)(3x 3 +x2 +x-2)+(-7)
(I) (x2 +l)(Sx-7)+(-8x+9) (ii) (x2 -3)(2 x2 -3x+6)+(-16ir+l7)
(iii) (x2 -ir+2)(x2 +x+2)+(-5x=s)
(iv) (x2 +2 x-1)(3x 3-13x2 + 2 9x-71)+(173x-78) o
prx) • (x+3)(ir+2)(x-1) (!_) X • -3,-2,1 � . "l .,.
(ii) -3,-2,1 -3 s. x s. -2 or x 2 1 .1,
x 3 +5x-6 • (x-l)(x2 +x+6); only real root is x--1 (x2 +x+6 • 0 has no
real roots) l· (!) Q(x)•x 2 -x+3; R(x)•5 (!!) Q(x)•x 2 -3x-2 ; R(x)•6x

� (page 245)
quotient 2 x2 +5x-5; remainder -10x+l3; a•lO,b•-10; factors are
(x2 +2)(2x 2 +5x-5).
quotient x2 -7x+7; remainder -4x-ll;
x 4 -6x 3 +x2 -4x-4 • (x 2 +x+l)(x 2 -7x+7)+(-4x-11); a•4, b•ll
(.!) a•-1, b•6 (ii) a • -5, b • l �· a a l; (x-2)

� (page 249)

-
1. (a) 4 (b) 3 (c) 30 (d) -29 (�) -13 (!) -82 (§_) -11 (!!.) 12
'!. (a) Yes -(b) No- (c) Yes
(I) P(x) --(x-1)(x::-2)(x-3) (ii) (a) x-1,2,3 (.b) 1,2,3
(c) l S. X .S 2 or X � 3 4, qrx} a-x 2 -x+7 j
k-must be one of •1, •7; ;;'y factor theorem, /- 1�7/i -i:
k a -1, P(x) • (x+l) 2 (2x-7); zeros of P(x) j
are -1,-1,7/2; roots are X"-1,-1,7/2,
_1
1=7
(i) (x-l)(x+l) (x+J); 1,-1,-3 (ii) (x+l)(x+2)(x-3); -1,-2,3
(iii) (x-2)(x2 +x+3); 2 only; other zeros unreal (iv) (x+2)2 (x-5);
-�2,5 (v) (x-l)(x+-4)(3x+l); 1,-4,-1/3 (vi) � (x::-j}(Sx -x-3); 3
2
(other zeros real but not rational)
l:.· (i)· x�l,-2 ,-5 (ii) X"- 2 (roots of x2 +5•0 are unreal)
(iii) X a -1, lt {6 8, X • 2 t nQ 9, (i) 3/2 (ii) 9/2
<III) -1 Civ) -22 To. -B-2 b; b•-4; otiier factors Cx-2)Cx-3)
11. p•2 ; A(x) • (x-2)2 (x� (.!) (a) 2,2,-6 1'
(b) x•2 ,2 ,-6 (ii)
12, p•l2 ; 2x 2 -3x+4 13, (i) a•-5, b•2 (ii) a•-3,o--5 �
l7i. a•-5, b•4 15, �1. b•2, c•-4 .ll· a•l, b•-3 -1, d ><-
17. a•-13, . b•-14.

� (page 255)
(.!) (x-2)(x+l)(x+3)(x+-4) (ii) x-2,-1,-3,-4 i, k•2; x-1,3,3,-2
1 2
l_.
(i) Po(x-1)(x-2)(x-3) (ii) Po(x-l)(x+l)(x+3) (iii)
J.· where. - Po(x+1)
·
2 (x-7),

Po is a constant, -
4. (a) (x+2)(x- 2)(x-1)2 (b) k(2x-1)(3x+l)(x-2), where k is integral
!. (i) a•l2,b•-2, C • 6/5-; d • 2 /3 (ii) a • 2, b • 3, C • 3
!. a;;4, b•-13, c•S, d•3 7, a•2, b•=!, c•4 Jl. p-2, q•lO, r-19, s•l7
9. a•l, b•-4, c•3; Hint: 1':t v•x2+x; x-\(-1•,'5), \(-1•/iJ)
To. a • 2, b • 8.

SET 5G (page 259)


(_i) 3
- (ii) -2 (iii) -1 (iv) - 1 (v) 11 (vi)- 11 (vii) 5
4 � 4
2
8 16
1.

t !
2

(viii) (ix) 6 (_!) -+


(xi) 25 (xii) 4 (1111) 9: (xiv) - �:
(!) 3 (ii) 4 (iii) -2 (iv) 9 (y) 4 (vi) - f (vii) -2
XXIII
ANSWERS
(viii) -6 (.!!:!) 1 l· (.!) O (.!!) -
(� O (@ - 5 3
t
(viii) - 2 (.!!:!) 5
(!ill i (.!Y) 3

5 73 1 53
i• (.!) 2 (.!!) - 2 (!!!) 0 (.!Y) - 2 (y) 0 (Y!) 4 ( @ 7
(viii) -5 l, p•30; sum• -3, product • -10 g,, (.!) -1 (.!!) -1
J.· x• - ti 2 x3+3x2 - 2 9x-60• O §.• a•l, b•-6; x-3

I, a•- 2 , b•-8, sum• 3, product• -5 10. 3x-y • 2 ; (- 2 ,-8)


T
- 1:t 2 x+y • 3 g. x+!y• 4, 2 x-y• 3; (0,2 ); [Note
'i5 (-!-,,4) - factors of 2 x 3 -3x 2 +8x-20 are
� P(,,,) 2 :1 (x-2)(2x2 +x+l0), 2 is not a double
-----·- _ xx+ly .33:a ., (6a,9 a). zero,]
.!1, x+ty � at 3+ 2at� 3 -13t+l 2• (t-l)(t-3)(t+4)]; three normals are
-
x+y• 3, x+3y -�3, x-4y• -7j; 1+3+(-4)• O. J;l, Q is (3/2 , 3/8)
lg, (-1, 20) max; (3,-12) min; R/s (1,4) ··

� (page 263)
Intervals are (.!) (2 ,3) (.!!) (-2 ,-1),(1,2 ),(3,4)
(!!!) (-1,0), (O, l), (1,2)
================
� (page 265)
l.· Successive approximations are 1•5, 1•25, 1•315, 1'312 5; Note best
approxn is 1·25,
(!) 0,625 (.!!) 1,875 (!!!) 1•12 5 (.!Y) 1•125
(y) 0,6 2 5 (� 4.375

SET SJ (page 271)


z,.T • 1· 2 599 3. Intervals are (-2,-1), (O,l), (1,2 )
23 - +
(.!!) Z2 • l5 • 1•533
I J,99·5
(!) 0•68 <.!!) 1•83 (!!!) 1•19
C.!Y) 1·11 (.y) o·59 c� 4 ,33
.2.· o.. 1 1-'-

CHAPTER 6
� (page 278)
(a) 1+4x+<>x 2 +4x 3+x 4 (b) l-4x+6x2 -4x 3+1c 4 (c) 1+6y+l5y 2 +20v 3 +15y 4 +
6�+y6 (g) l-7y+2ly2-J5y 3 +35y 4- 2 1y S +7y 6 -y7 (e) 1+5b+lOb�+lob3 +5b 4 +
b 5 (!) l + 3x/5 + 3x 2 /25 + x 3 /12 5 (A) l-10x+40x 2 -sox 3 +80x 4-32x 5
(l!) 1+18x+l35x 2 +54ox 3+1215x 4 +1458x 5 +729x 6 (i) 2
- l-4x+7x -7x +
3

11 x - l x S + ...1. x - _!, + --lL. ('' l+lox-1+45x- +12ox 3+21ox-4+


4 6
7 8
2 -
8 4 16 16 256 �
252x-5+210x-6+120x-7+45X- 8+10x-9+x-10(1} l - 14 + � - � + � -
2 X X X
� + � - � @ l + _!! + � + �3+!�,::X
X Xl2 X 48 64
:=w - y
asyfor (h)y
withyx-1 for x.
(a) l - ::t;
X + X X 2 (n)
::T
5 5
4 s
l· W 70x U1) 56x filil -;
45 405
4

1120; 1792; -
45 405 9 8
.i: (
!) 2i2 (.!!) 16x4 (!!!) Bz2l 2; 16 i
;
1• (!) 56
Expansion is [l+(x-1)) 7 8. (.!) term indep. of x is lSS
;J., a•
�· (.!!) coefft of z4 is 3 9. 3, b • 14, c • 26
<!> 14 <W 61 <!!!> -!a <.!Y> -12 (l!) 43/4
XXIV
ANSWERS
(.!.) a • 41, b • 29 (!.!)X • 217 t y • -104 (ill) (!_} 14 (.!?) 10/2
(.!) x • 1/5 or 1 (x ,f, 0) (!.!) x • 9/5, (x ,f, 0)
11.
(.!) (.!!) k • 4/5, (k ,f, 0) (.!?) k • 6/5 (!.!) 416
'ff.
(.!.) 1+5x+l0x2+1ox l ; 1•010040 (� 1-12x+6ox2 -16ox l +24ox 4-192x 5+
13.
64x6 ; •88584 -15. (.!) 2[7x+35x +21x 5+x 7 ); 1•47042
J!.
(!.!) 2 ( 1+1ox2+5x'll'j'; 2, 000080
16. 1+·1+•004+·00008+•00000080+·0000000032; $104•08; $.J.04
J!, (.!) l-5x+20x2 -5ox 3+105x 4 (!.!) 19

(a) x l+3x2y + 3xy2 + y 3 (.!?) x 3 -3x2 y+3xy2 -y 3 (c} p 6+6p 5 �+15p 4 q2 +


� (page 281)

20p 3q3+15p�q 4 +6pq 5 6 (d} pS-5p 4q+l0p 3q2 -10p2q3+5pq 4-q


(e) a B+8a'b+28a 6b2 +56a sb"1'+7oa 4b 4 +56a 3b5 +28a2 b6 +8ab7+be
(f) a9-9a 8b+36a 7b2 -84a 6 b 3 +126a 5b4 -126a 4 b5 +84a 3b 6 -32a2 b 7+9ab8-b 9
-tq

(&) x S+5x l +1ox+10x-l+5x- 3 t x- 5 (h) a 4-2a 3x+3a2 x2 /2-ax 3/2+x 2/16


(i) x 3 - 15 + 75x- 3 - 125x- 6 (j) 128x 7-448x6b+672x 5b2 -560x4b 3 +
280x 3b4 -04x2 b 5+14xb6-b 7 (!) 27x 3 +108x2 l44xy2 +64y 3
(!) x 6 /64 - 3x 5/8 + 15x 4 /4 - 2ox 3 + 6ox2 - 96x +64
(m) x 12 + 2x l0 y + 5x 8 y2 /3 + 20x 6 y 3/27 + 5x 4 y 4/27 + 2x2 y 5/81 + y 6 /729
- 3 l, ,, 4
v+

(!!) x2+8x.�y 73+24xy 73+32x\+16y V3 (2) 32x- 5 + Box-2 /3 + 80x/9 +


40x 4 /27 +. lox 7/81 + x 10 /243 (£) a 6-6/Ja 5+45a4 -6o/Ja 3+135a2 54/Ja+
27. l· (§) (.!.) [a+(2b-a)J 6 • 64b 6 (!.!) ((,'3+/2)-12) 5 • 9/j"
(i) b = 5 (!.!)a • 843, b • 589 4. (.!.) 40:3
,-

5. (!) 64+192x+240x2 +160x 3+6ox 4; -20 <Tu (!) -7 (.!?) -10


3.
6. (.!) 46 (!.!)(-2) + (-2) = -4 (.gV, -44 (� 155 l• (!.!) a • 40
(.!.) 3936 (ii) 323 9. (!) (.!.) 5x +1ox l h+lOx h2 +5xh 3 +h 4 (!_p 5,i 4
(.!?) (i) 12x"3:tl8x2 h+l2xh2 +3h 3-15x2 -15xh�5h2 (!.!) 12x l -15x2
!,
.12· a • 64, b • 96, c • -720 !l• a • 20 .ll· (.!.) 32,08008
(!.!) 63· 61696 14. (.!) l• 19405 (!.!)67,937
15. As for 1 (e) where a•x, b•y, A•l, B•-8, 0-20, l)o,-16 E•2
n. (.!.) a2 +b2+c2 +2ab+2ac+2bc (!.!)a l+b 3+c 3+3a2b+3ab2 +3a i c+3ac2+3b2 c+
3bc2 +6abc.

(!) 56 (.!?) 21 (.£) 84 (� 20 (!) 210 (!) 715 (.&) 330 (h) 66
� (page 292)

(.!.) 1 (j) 680 (!) 5/18 (!) 7/U (� 2/99 (!!) 12/55

�· (!) (.!_) 65 (!.!)7 (,f, -6) (ill) 6 (,f, -7) (!.Y) 8 (,f, 1)
(.!?) n ,f, \ o r -2 5, (i} 9 (!.!)11 (ill) 9 (!.Y) 20 (y) 8 (:tl) 5
=6. (.!.) 945x 4 /16 (!.!)--14�y l2/27 (.!.!.!) 495
(!) (.!) 112oa 4b4 (!.!)252/k (� (.!) -35x 4 y 3/81, 35x l y 4 /27
(!.!)462x4 , 462x 7 9, (i} 13x :6 (ii} y l :1
<.!) 21 <!.!.> so cw b1r <M 5oo5kr 11. w220 <!.!.> 192
(ill) 495 12. (.!) 15 (!.!)210 (gij33/2 (!.Y) -35/54
(v) 2,3 7,5 5,.,- (ill -231/8 (� 5103/4 (viii) 40095
(!) -7/18 (!.!) 112 (!.!!) -455y 2 (.!) 1417 (!.!)452 (ill) 7656
(.!.Y) -36 15. (.!) x-3. (!.!)a • -9, b • 46
ll·
(.!) n • 8; '!+8x+28x2 (!.!)n .. 29 17. (.!) c • 2/3, n • 9
(.!!) c • -2/3, n • 6; -160x l /27 ll• (.!) ri • 16 (!.!) n • 8 (,f, 1)
n- r
19. 12th, 13th terms 20. ncr b (ax) 21. x • 1 or 5
n • 7 o r 14 l,i, � (.!) -1 (.!!) 1/512 (iii) 1 (.!.Y) l/i
r

I!, 1

grad PQ is 7x6 +21x5 h+35x4h2 +35x 3 h 3 +21x2h 4+7xh 5+h 6 ; grad tan ie 7x 6 ,
� (page 295)

<� 2 I 6+15h+2oh2+15h 3+6h 4+h 5 J (!.!) 12


,.!, f (x) • nxn-1 4. 1/20 sq unit
, IG-
"ltf
J..
:2.

(0,0) min; (1/3"'; 16/729) max; (1,0) min.\


area • 1/105 sq. unit. I I
A _/
XXV I I I h.
AIISWERS
�· (i) 16/105 sq unit (ii) w/30 cu Wlits
7. (i) 178/3

!· a • 376, b • 168; (/5-1)6 • 576-256,5; �(-25+53,5)

2,, (i) •9135 (ii) 1·1157 ,12., (a) 128/n; 448/n; 672/n; 560/n; 280/n;
84/n; 14/n; 1/n; where n • 2187 (b) (i) 1232/n (ii) 99/n
(iii) 576/n 11, (a) 1/s; 10/s; 45/s; 120/s; 210/s; 252/s; 210/s;
120/s; 45/s; 10-r;;; 1/s; where s• 1024 (b) (i) 56/s (ii) 386/s

. CHAPTER 7
t:xampte 1 (page 310 )
(y) (a) 7/13 (b) 1/13 (c) 3/13 (d) 2/13 (e) 4/13 (f) 8/13 (g) l
(h) O (i) 8/13 (j) 10/13 (k) 4/13 (l) 6/13 (m) 7/13 (n) 0
(o) 3/13 (p) 12/13 (q) 5/13 (,) 5/13 (s) 3/13 (t) 1/13
(u) 12/13 (v) 0 (w) 2/13 (x) 5/13 (y) 8/13 (z) 1/13
Examf'le :J (page 311 )
(iv ) H • ((1,5),(2,4),(3,3),(4,2),(5,l)}; I• ((4,4),(4,5),(5,4),(5,5)}
L • ((2,l),(2,2),(2,3),(2,4),(2,5),(5,1),(5,2),(5,3),(5,4),(5,5)}
M = ((1,2),(l,4),(3,2),(3,4),(5,2),(5,4)}
(y) 1/5; 4/25; 2/5; 6/25
(.YJ.) (a) 1/25 (b) 1/5 (c) 2/5 (d) 4/5 (e, 2/5 (f) 9/25 (g) 2/25
(h) 6/25 (i) 19/25 (j) 4/25 (k) 21/25 (() 3/5 (m) 2/5 (n) 4/5
(o) 1/5 (p) 2/25 (q) 6/25 (r) 4/25 (s) 9/25 (t) 3/5 (u) 19/25
(v) 21/25 (w) 12/25 (x) 23/25
!,':,;ample J (page 312 )
(iY) 11 • ((1,5),(2,4),(4,2),(5,1)}; I= ((4,5),(5,4)}; L • ((2,1),(2,3),
(2,4),(2,5),(5,1),(5,2),(5,3),(5,4)}; M • ((1,2),(1,4),(3,2),(3,4),
(5,2),(5,4)}
(v) 1/5; 1/10; 2/5; 3/10
(;i) (a ) 1/20 (b) 1/5 (c) 2/5 (d) 4/5 (e) 2/5 (f) 7/20 (g) 1/10
(h) 1/5 (i) 4/5 (j) 1/5 (k) 4/5 Cl) 3/5 (m) 2/5 (n) 4/5 (o) 0
(p) 1/10 (q) 3/10 (r) 1/10 (s) 3/10 (t) 3/5 (u) 7/10 (v) 9/10
(w) 9/20 (x) 19 /20______________
_ .;.. _
&:ample 4 (page 312 )
E:,:periment 1 (.!) s • {w,B,R,Y\ (a) 1/4 (b) 1/4 (c) 1/2 (d) 3/4
(e) 1 (f) O (W y 1 ! :
::Xperimen t 2 (!) (a) S• {Im ,WB,WR,1./Y,nw,BB, BR,BY,RW,RB ,KR,RY,YW,YB,
YR,YY } 4 0 O o O (.!!) (a) 1/16 (b) 1/16 (c) 1/8
0 0 0 0 (d) 3/16 (e) 3/8 (f) 1/8
l O O O 0 (g) 1/4 (h) 3/4 (i) 1/16
0 0 0 0
(j) 1/4 (k) 7/16
I l t 4

E:i:periment J (!) (a) S • (WB,WR,WY,liW,BR,BY ,RW,Rll,RY,YW,YB,YR}


(.!!) (a) 0 (b) 1/12 (c) 1/6 (d) 1/6
(e) 1/2 (f) 1/6 (g) 1/4 (h) 3/4
(i) 1/12 (j) 0 (k) 1/2

Experiment 4 W S• (WB,WR,WY,BR,BY,RY) (.!!) (a) 1/2 (b) 1/6


_________(c) 1/2 (d) 1/6
.,_ �
XXVl
ANSWERS
e:i;ample fi (page 313 )
<!!) (a) 1/8 (b) 1/8 (c) 1/8 (d) 1/8 (e) 3/8 (f) 3/8 (g) 1/4
(h) 1/8 (1) 7/8 (j) 1/2 (k) 3/8 (!) 3/8 (m) 1/2 (n) 1/2
(o) 3/4 (ill) (a) 1/16 (b) 1/16 (c) 1/16 (d) 1/16 (e) 1/4
(f) 1/16 (g) 1/16 (h) 3/8 (i) 1/4 (j) 1/16 (k) 1/8 (l) 1/8;
2 boys and 2 gir_ls_ _. __________
E:z:ample 6 (page 314)
(!) (b) l'(T • 9) • 1/9. In 72 throw,;, I should win -k x 72 • l:l ti.mes
and you should win 9 1 x 72 • 8 ti.mes, You "owe" me $4,
(c) Probabilities of totals 2,3,4,5,6,7,8,9,10,11,12 ar� respect­
ively 1/36, 2/36, 3/36, 4/36, 5/36, 6/36, 5/3b, 4/36, 3/36, 2/36,
1/36, (ii) (b) P(T • 12) • 25/216 (c) P(T-13) • 21/216
(ill) 5/6T"": 10/63: 10/6
4

k
==m====================
� (page 315)
,1: (!) (ca) {H, T} (S) ' � ; (a) 1/2 (b) l
(.!!) (ca) {IOl,HT,111,TT} (S)1 (a) 1/4 (b) 1/4 (c) 1/4
0 0
' • 0
(d) 1/2 (e) 1/4 (f) 1/2
(g) 3/4 (h) 1/4
(iii) (ca) 8 si.mple events (y)' ( ) 1/8 (b) 1/8 (c) 1/8 (d) 3/8
(e) 1/4 (f) 7/8 (g) 1/2 (h) 1/2 (1) 3/8
(� (ca) 16 simple events (S) (a) 1/16 (b) 1/4 (c) 5/16
(d) 5/16 (e) 5/16 (f) 3/8
l• (a) 1/10 (b) 1/2 (c) 2/5 (d) 2/5 (e) 1/5 (f) 3/10 (g) l (h) O
(i) 1/5 (j) 0
(a) 1/26 (b) 3/26 (c) 5/26 (d) 5/26 (e) O
(a) 1/6 (b) 1/3 (c) 5/6 (d) 1/2
(a) 1/4 (b) 1/2 (c) 1/13 (d) 1/2 (e) 1 (f) 0 (g) 1/52
(h) 9/13 (i) 3/13 (j) 1/13
!• (a) 9/lO O(including O as divisi�le by 12)
(b) 9/10 (incl. 0 as a multiple oflO) (c) 1/10 (incl, 0 as 00)
(d) 3/10 (e) 2/25 {f) 3/50 (g) 0 (h) 1/10 (incl, O) (i) 1/10
(j) 1/20 (k) 1/10 ({) 1/50 (111) 71/100
}., (!) P(A) • 5/36; P(U) • 1/2; P(C) • 1/4; P(D) • 4/9
(.!!) (a) 1/6 (b) 1/36 (c) 11/36 (d) 1/2 (e) 1/6 (f) 2/3
(g) 1/2 (h) 1/2 (1) 1/4 (j) 3/4 (k) 1/4 (l) 1/12 (m) 1/6
(n) 5/6 (o) 5/12 (p) 7/12 (q) 25/36 (r) 5/18 (s) 1/9 (t) 1/6
(u) 2/3 (v) 5/6 (w) 1/3 (x) 0 (y) 1/3 (z) 5/12
(ill) (a) 8/9 (b) 5/9 (c) 4/9 (d) 5/9
!• (.!) (�) 6/7 (b) 1/7 (c) 4/7 (d) l (e) 5/7 (f) 3/7 (g) 4/7
<!!) (a) 40/49 (b) 9/49 (c) 4/7 (d) l (e) 33/49 (f) 25/49
(g) 24/49
Ez<zmple 6 (page 324) (.!!.) 1/18; 5/18;. No.
lU,l! (page 326')
l· (!) •6. ·65, o, '/), •25 (.!!) •75, •45,-•55 (ill) •6, ·l, •4
l• Corre,
(.!) l <!!) • l (ill) 37 /60 � 1/2
l• . ones (b), (c). (a) is incorrect, A,B are not mutually ex­
cluef e. P(AUB) • 4/6 + 4/6 - 2/6 • 1
i· (h)(.!) ) 4/5 (b) 1/2 (c) 7/10 (d) 4/5 (e) 3/10 (f) 0 (g) 0
<!!) (a) •5 (b) •55 (c) •3
i.• (!) 1/2 (b) 1/2 (c) 1/4 (d) O (e) 3/4 (f) 1/4
( .!!) ,7 (b) •3 (c) •45 (d) • 25 (e) ·9 (f) ·1
(!) (a 5/8 (b) 3/8 (c) 1/2 (d) 3/8 (e) 3/8 (f) 3/4 (g) 0
(h) l .i) 3/4 (j) 5/8 (k) 7/8, A,C are mutually exclusive.
(.!!) ( 1 •4 (b) •6 (c) •55 (d) •6 (e) •25 (f) •7 (g) 0
(h) 1 (1) •85 (j) •4 (k) •7, A, C,
XXVII
ANSWERS
7. (!) 1/2 (.li) 1/2 (ill) o (10 5/6 (:,u 1/3
'ii. (!) 11/15 (ii) 47 /60 (iii) 19/100 2.· (!) 27/35 (.!!) 30/35 • 6/7
io. 3/8'; 2/5 .!!· (i) 5/6 (ti) 67 /120
ii. (i) 46/49 (ii} 1/7 (fil:l(!y)1 sf4g;!3. (i) (a) 1 - 1/4 = 3/4
(:,) 1 - 1/8 = 7/ 8 (c) 1 - 1/16 = isl16 (d) 1 - 1/32 • 31/32
(e) l - 1 /1024 = 1023/1024 (li) (a) l -25/36 = 11/36
(b) l-125V216 = 91/216 (c) 1•625 /1296 = 671/12?6
ll· (_!) 2/5 (!1) 43 : 34
.ll• (a) 1/6 (b) 5/6 (c) 1/3 (d) 2/J (e) 1 (f) 0 (g) 2/3 (h) 5/6
<1 >5/6 <J>
1 <k> 213 · m 5/6
li.· (a) 1/13 (Ii) 1/1� (c) 2/13 (d) 11/13 (e) 1/4 (f) 4/13
(g) 9/13 (h) 3/13 (i) 1/52 (J) 1/52 (k) 1/26 (l) 1/2
(m) 1/2 (n) 1 (o) 0 (p) 3/13 (n.) 3/26 (r) 8/13 (s) 15/26
(t) 11/13 (u) 4/13 (v) 5/26
.U.· (a) 1/6 (b) 1/36· (c) 11/36 (d) 5/6 (e) 5/6 (f) 35/)6
(g) 25/36 (h) 1/2 (i) 1/2 (j) 1/4 (k) 3/4 (i) 3/4 (m) 1/4
(n) 1/6 (o) 5/6 (p} 11/36 (q) 1/9 (r) 8/9 (s} 1/6 (t) 5/6
(u) 2/9 (v)=l7;.,;/,;l,;;.8 ============-
SET 7C (page 332)
�) 1/12' (ii) 1/10 (iii) 13/15 (iv) 1/30 2. 1/30
3. (i) 1/2611 (ii} 1/52 (iii) 21/52 (iv) 14/65 :_:(i) 1/12 (ii) 1/4
5. (i} 21/50 (ii) 9/50 (iii) 21/50 + ')/50 = 3/5 (iv) 3/25 (v) 29/50
-
6. (i) - - - -
15/77 (ii) 24/77 (iii) 1 8/77 (iv) 20/77 (v} 1 8+20 = n
----=n 3 8

7. 1/22 8. (i) 1/156 (ii) 1/16 (iii) 1/13


1- (i) 2/5- (ii} 1/10 (iU) 1/60 (M
(;) 1/30 +�/15 + 1/2� 3/20
- 1/10 + 1/5 + 2/15 = 13/30
10. (;) •132651 (b) •117649. (c} •127449 (d) 3 X •127449 = •3 8 2347
TL (a) (i) 49/100 (ii). 9/100 (iii) 21/100 (iv) 21/100
(;) 29/50 / (vi) 21/50 (b) (i) 7/15 (ii) 1/15 (iii} 7/30
(iv} 7 /30 (v) 8 /15 (vi) 7/15
.ll• (a) (i) 1/36 (ii) 1/36 (iii) 1/3 (iv) 1/4 (b) (i) 1/216
(ii} 1/216 (iii} 1/216 (iv} 1/8 (v} 1/27 (vi) 1/36
.!l.· (a} (i} 1/16 (ii) 1/16 (iii} 1/4 (iv) 1/4 (v) 2/2704
(;i) 1/169 (vii) 1/169 (viii) 1/20 8
(b) (i) 13/204 (ii) 1/17 (iii) 13/51 (iv) 25/102
(;) ° 2/2652 ·(vi) 4/663 (vii) 1/221 (viii) 1/204
ll· (i) (a) 15/64 (b} 75/512 (c) 225/4096 (ii) - (a} 15/56 (b) 5/28
(;> 1/141 15. (i) 1/10 (ii) 1/4
16. (i) 5/18. (ii) l /6 (iii) 5/9 (iv) 1/36
17. (a) (i) 3/8 (ii) 1/2 (iii) 7/8 (iv) J./ 8 (v) 7/8
(j) (i) 9/64 (H) 1/4 (iii) 9/64 + 1/4 + 1/64 = 13/32 (iv) 3/16
� (page 341)
1.
- (i) 1/4 (ii) 5/24 (iii) 13/24 (iv) 11/24 l• (i) 14/55
(ii) 83/165 3. (i) 1/6 (ii) 1/2 (iii) 2/3
f!· (i) 1/3 (ii} 49/120 (iii) 4/9 l· Reasoning ignores probability
of first selecting a particular urn; correct va l ue 1/2
31
6. (1) 9/20 (ii) 2/15 + 1/3. 7 /15 1, (i) 3/35 (ii) 12/35 (iii} 35
!, (i) 18/25 • 727. (ii) 1/50 • 21. (iii) 2/25 • 81. (iv) 49/50-98%
(v) 4/5 • 801. (vi) 7/25• 281.
.2.• Events are not mutually eKc l usive.
Correct method is 1 - P(both fail)• 1 - •24 • ·76
.12· (i) 8/35 (ii) 32/105 (iii) 4/105 (iv} 1 - 4/105 • 101/105
24+32+12+6 ....l!!
(v)
105

105 11 (1) 1/210 (ii) 3/28 (iii).ll•· 1- .1
-· 28 28
�· (a) 5/24 (b} 3/11 13. 4/9 14. (a) (i) 191/360 (ii} 169/360
<.!!.> <1> 11so c11> 1 -=-'<1180 + i78> :- 63/80
XXVIII
ANSWERS
(a) (1) 1/36 (ii) 1/6 (iii) 2/36 + 1/36 • 1/12
�) (i) 1/216 {ii) 6 X 1/216 a 1/36 (iii) 1/216 (iv) 1/36

t t t 1 �
(v) 1/216 (vi) 3 x 1/216 • 1/72 (vii) 2 x 1/2 3 = 1/4
x + • "t
= (!) (a) 1/27 4 (b) 1/4 (c) 1/16 0
(d) 1/208 (e) 1/16 (f) 1/2 (g) 9/16 (h) 7/16 (i) 1/169
(j) 144/169 (k) 1/169 Cl) 9/169 (m) 30/169
(ii) (a) 1/2652 (b) 1/4 (c) 13/2 4 (d) 1/204 (e) 1/17 (f) 26/51 0
(g)l9/34 �h) 15/34 (i) 1/221 {j) ltl8/221 (k) 4/663 (t) U/221
0
(m) 4 /221 18. (i) 1/8, 3/8, 3/8, 1/8; l/4 (.!!) 1/16, 1/4,
3/8, 1/4 l/16 -19.-(i) (a) 1/8 (b) 3/8 (c) 1 - 1/8 = 7/8
(.!!) (a) 27/125 (b) 36/125 (c) 98/125
2 . 0 Respective p robabilities are q 3, 3pq 2 , 3p2 q, p 3.
21. A (i) 27/512 (ii) 1 - 125/512 = 387/512 (iii) 125/512
ii (i) 1/56 (ii) 1 - 5/28 • 23/28 (iii) 5/28
13 1 3 2 3 .
ll· 3 • 5 + 3 • 5 5 ll• (!) 5 (.!!) 31/9
D
0
24. (i) 181/450 (ii) 1 4/225 (iii) 61/45 0 0
25. (a) 11/20 = 55% (b) 7/20 = 35% (c) 1/8 = 121,7. (d) 41/20 201,z 0 C

(e) 1 - 41/2 00
m 159/2 = 79� 00
ll· (.J!) (i) 11/20 55% (ii) 5/12
& 411% (iii) 9/20 45% S D

5 579 3 221 = 275


(!!_) (i) 181/800 • 22tr. (ii) • 72 % (iii)
80 8 800 i' 0
(i) (a) 11/20 = 55% (b) 7/2 • 35% (c) 13/2 = 65% 0 0
0
(.!!) (a) 61/200 = 3 �% (b) 159/200 = 79�% (c) 41/200 = 20�
0
(i) 1/40 = 21-.z (ii) 9/1 • 9 7. (iii) 29/4 = 721-.z 0 0
� (page 353)
J.. w 42 (_!!.) 2 20

(.£) 1680 <�> 12 0
(!_) 12 (!) 1 < i> 1 <�> ! i0 "t
(1) 36 (j)
63
l: (_!) 12. {ab,ac,ad,ba,bc,bd,ca,cb,cd,da,db,dc}.
(_!!.) 24 (c) 24 3, (i) (!) 26.25 (b) 26 2 (.!!) (a) 26.25.24
(b) 26 3 (iii) (a) 26-;-25.24.23 (b) 26 4 4. (i) 6! • 720
(ii) 6.5 -To s.
(i) 36 (ii) 216. 6 3 (i'u)-16. 2 4 = 6. 5! • 12 0
1=.· <!!> <!> <2.> 6
(.!::'.) (_!!) 36 0
6-.ID c..w
<2.> 30 <.2.> 36 < iii) <2.> 12 <.2.> 216
(E.) 1296 (1.), (!) (_!) 6 (_!!) 6 (.!_!) (!) 25 (_£) 30
0
(ill) (!) 100 (E.) lllO (iv) (_!) 30 (_!!) 1 80 8. (_!) 140 (.!_!) 72 0 0 0
(_!!) 6.5.4.3 • 360 (E.) 1.5.4.3 • 60 (.£) 5.4.3.! = 120
(2) (4.3).4.3 • 144 (!) 41 • 24 (.!) 4.(4.3.2) • 96 (g) 1.4.3,1•12
.!2· 0
(!) 71 • 5 40 (E.) 2.6! • 1440 (.£) 2.61 • 144 @ 5.4.(5!)•24 0 0 0
(e) 5! • 120 11, 8! 4 32 (!) 61 - 72 0 0(ii) 2,71 - 10080 0
0
C

.ll· (i) 8.7.6.5.4.-r;72 (ii) 4.3.(6.5.4) • 1440 �


(iii) 4.(7.6.5.4) • 3360 (iv) 4,4,(6.5.4) • 1920 •
13. 0
27;T"°x 10 3 (i) 26,25,24,1 ,9°:-8 (.!!) 26 3 ,10,9,8 (iii) 26 2 X 1 2 0
(iv) 1 3 0 14. �) (1) 8.7.6 • 336 (ii) 672+504+72+9 • 1257
W (!) 9'.8.7.6.5 • 15120 (E.) 4.3.(7.6-:S) • 252 0
(.£) (5.4).(7,6.5) • 4200 15. (!) 61 • 1'2 (.!,!) 6,5.4.3 • 36 0 0
16. (!) 71 • 5 4 00(E.) 4131 • 144 (.£) 2.(4!3!) • 388 @ 2.6! • 144 0
(e) 3!5! • 720 17. (1) 4 (ii) 4.3 • 12 (111) 4.3.2 • 24
(iv) 41 • 24 0
18':"9(a)-(i) (a°f"'l 8+36+6 • 1,o'.°7(b) 6o+3o+6 • 96
(ii) (a) 3.6' +6�6'+7i• (b) 6o+36o+72o+72 • 1860 0
0
( S) (1) (a) 1 8+3o+6 • 144 (hl 6o+25+6 • 91
(ii)(!) 2.6 2 +5.6 3 +5.6 4 +5.6 5-�or exactly 400)
(ii) 4o+Joo+6oo+600 • 1540 -= 19. (1)
- 9! • 362,88 (J.!) 413121 • 288 0
(iii) 31413121 • 1728
\
XXIX

\
ANSWERS
� (page 357)
!.: (!) 120 (ill 60 (!!!) 12 (� 3 (!) 120 (vi) 60480
(vii) 30240 (viii) 45360 2. (1) 3360 (ii) 35 (iii) 90 (-iv) 90
3. (�l,HHHHH (b) 5,HIDIHT, HHIITH:- HHTIDI, HfflHH, THHiiil (c)lO
4. (a) 1 (b) 8 (c) 28 (d) 56 (e) 70 (f). 56 (&} 28 (ii} 8 (i) 1
(1) 35 (ii) 420 (iii) -2520 6:- (i) (a) 20 (b) 56 (a) 20 cm
<fil 140 - - = - - - - -
SET 7G (page 363)
(a) 15 (� 35 (£_) 35 (d) 9 (e) 1 (f) 5/14 (g) 21/44 (!!) 2/7
(!} 45 (ii) 120 (iii) 210 (ivf 252 fv) 210 -
8 7
(!) c • ---;-6 (ii) c • 35 (iii) 7c • -21 (iv) 6c • 20
5 4
6 6 · 9 6 - 3 6 6
5
(!) c • 6 (vi) c • 15 (vii) ( c - c ) or (2 • c + c )•36
5 4 3 4 5
9 3 6 6
5

.i· (!) c • 126 (ii) c x c • 45 (iii) c • 1s


4 2 2 4
3 6
(iv) c x c • 60 ('{} 126-15 or 6o+45+6 • 111
1 3
(i) 330 (ii) 5 (fii) 15 (iv) 60 (v) 150 - (vi) 330- 5 • 325
(vii) �5+100 • 115(viii) i20 6. (if 120 (ii} 10 (iii) 6
(iv) 30 (v) 8 7. ([)To (ii)-15 -(iii) 35-(iv) 2 -
9 10
J!.· (!) c • ;26 (ca::iot-use O t;;--begin number) (11) c • 210 (here
4 4
0 can end number) 9. (i) (a) 21 (B) 6 (ii) (a) 35 (B) - 15
(y) 5 (6) 4 (£) 4+1&+-12-• 34 (iii)-(a) - - 35-(8)-10 -
(iv) (a)-21 (B) 6
52 4 4 13 39
.!2· <,!.> � • 2,0;2s (ii) c x c = 36 (Hi)
13
4
39
2 2
40
c x
3
c • 11154
1
<.!:!> c x c,� 'l,.-u1s4+715 • 11869 <�> c • 91390
3 4
13 13
(vi) c • 715 (vii) 4 x c • 2860 (viii) 13 4
4
44 48
<.!!Y 2. c2 x c
2 ll· (.!). 75 (ii) 1!>+60+10 • 145 (iii) 75+20-95
12. (1) 330 (ii) 60 (111) 15 (iv) 15+5 • 20
'IT. <I> 15-6 •9 (ii) 21-7 • 14 (iii) 28-8 • 20 (iv) 66-12 • 54

� (page 370)
1 l 4+10 7 4+lo+l 3
220; (.!) (ii) 22 (iii) • (iv) 220 •
55 220 110 44
3 3 3
(!) (vi) 22 (vii)
44 11
Sc
1 1 2 5 10 4 65
(_!) (.!!) 22 (iii) TI (iv) TI (�) (vi) 1 - •
66 33 ll 66
10 C
60H5o+loo+l5 6 5 c 4
(vii) o+ • ll (viii) 3 • _!!
330 330 66 -- -- 11
- 330
1 14 1
8.7 • 56 possible outcomes ( !) (ii) 2/56 • 1/28 (111) •
56 56 4
15
£±ll:. 3._ ( ) ..1. • ....! ( 1) hl. ( 11) Llb.ll. 1
2.)
(1 56 (.., � 56 28 � 56
11
28 �
11c 56
7
12 c
1 c 4 4 6 3 3
<! > >
11s <.!! --
3 •
13 <..!!.P 11s • TI <iv> 715 • 13
715
ll
(_y_) __3. • ....! .l· 2 5 • 32 (_!.) 1/32 <.!.!.> 5/32 <.!.!!.> 10/32•5/16
715 13
(iv) 10/32 • 5/16 (v) 5/32 (vi) 1/32 (vii) 13/16

cu
-
2111 (ii> 2•5•6-- ...i
- 11.10 11 (iii)-
2.5.6 +T.4 • ...§.
110 11
-
(_!) 1/221 (ii) 1/11 ( iii) 11/221 (iv) 1/1326 (y) 25/102
-xxx
---
ANSWERS
l· (tl) 26/5 1 (lli) 13/102 (n.11) 8/663 (u) 13/34
(!!) l\��r • �!
!· 61 5 1 • 86400; 1 • !;,!�
1�
8

i' 2, 5 1 !!) 3 1 41 • l
2.· ( � 61 • l 3 ( .61 5
4 2
.!£· (!) (!) �:!:; . t
(!!) 1 - (Ui) t. t • ;,3· t (ilu !:.1...!
60 • 5
l

(v' o (vii' l:.hl • 1 (nil) l • 1 - 1 (;Lx) .2 • ...!


u ...!
60 (vi' 60 5 5 5 60 20
5 5 , .1 (1!) 1 _ 1. l (ill) 5,5,2. l
3
(!!) 15 (hl W 5.,5 .5 (iv) l
ll.l �

5 5 5.5.5 5 -5
4 51
t t ·t
5

0 ·
W J5 <Y.!> <ill> <ill.!)
· 24+ 2•41
f U!> 2� <.!!)
...J.l
ll· \�·
5 + 2o+-6o+l 20H20 • 32 5
' 325 • 325
(i){ C6x c}/ C .l!! 'ii'{ 7 C�8C+7 cx8 C}/15 C • .1§.
.!l· 6 12 65 � b 6 5 12 65
7 8 15
7

10 10
'11' c / c • ...l (iii' c / c • ....i 10
14. (! ) c / c • l �
4 6 3 6 6 ==., 6 15
8 8 8

(10 lOC _Sc l C • l 15 . (!) i (ii) l (iii) l (iv) l


15 5

( 6 4 ) I 6 3- 9
9 -9 -3
O

8 1 . 1 120 1 1 1 11 1+·21
9 • - 9 ,ll, (!!) (!) (ti) 4 (iii) 4 (iv)
W 12 60 • 120
(� 210 (!) ...! '11' 1 fv 45 • lo+3 +1.li
42 � 7 u) ' 45 45
1 1 3 2,3 1 5 1 l
'-l!

(2) 81 • 40320 (!) 56 (.!!) 56 <.!.!!) 28 �) - -


8 1 - • 28
4 2
st •40�20 �> 30; c c2 x c1> / 30 • 2/5
0l>
51 41 30 • l ill> 4 • 1 - l
TIIT • 30 W < 2121 > I s s s
� (page 386)

) �) )
<!!
J., (ii IP{x)l�3�!2pl3!p21!31 IP{X),�414!3pl6!2p2!4!p31:�
lli) �) p 4 <!!) q4 (£) 4q 3p (d) 4qp 3 (!) 4qp 3 (f) 6q 2p2+4qp 3+p4 or
l-(q 4 +4q 3p) (g) q4+4q3p+6 q2pt""or l-(4qp3+p '+), -
l.· �) 4/9, 4/9, 1/9 <!!) (!!) 8/2 7 , 12/2 7 , 6/2 7, 1/27
�) 16/81, 32/8 1, 24/81, 8/81, 1/81
(£) X
where since one of these events
X .B. .l!Q 80 iQ .!Q l n• 243; is certain to occur,
1

P( ) n n n n nn
W 15/64 (!!) 1/64 (!!!) 20/64 • 5/16 (!y) 5 7/64
(y) 22/64 • 11/32 .i· {!) 70/25 6 • 35/128 ill> 1/256
1 9 24 7 210 . 105 45 56 7
U11.> - m•m lOi4 • 5 12 ill> 1024 (iii) lOi4 •
.l· w m
38 6 193 1 ..J2 29 1
lli) 1024 • fil i· (!) 128 (!!) 12 8 (!g) 128 (iv) ffi • 16
8

98 49 63 64 1 35
)
Cy ffi • 64 W> 128 C!!!> 1 28 • 2 (viii) 128
1 1 1 s 405 243
1 - 1024 1 ...M§. 81
J.: 4; <4 + 4> w 1024 w> • <.!!!> 1024 •m
78
1024
(iu 1�: • ili !.· (!) 6��
XXXI
ill> g�
ANSWERS
6 l [since 7 a 2+5,5+2,6+ 500 125
2.· • 1, 1+6,4+3,3+4] (.!)1296• 324
36 6
1125 19 625
(.!!) l -1296 • (iii)1296
144
.!2· (.!) �s • '00
1 1% (ii) ( �) 5• 59 • 049?. (iii) l - 9
1
4
• •856?. ��1
11. 15
(.!)16
1
2 • n.
5

2
16
(ii)16
2

27
10
2 (iii)
1 (iv)
1 1
125
16
2
200
12, (.!) 243 (ii) 243 (iii) 243 (iv) 243 (v) 243
13. (.!) l-(•9) 10 " l-·3483 f • 65 (ii)
15 •
r (•4)7(,6)8 1773
7
+
� (page 391)
In questions 1,2; [q = •96, p = •04]
5
A: <!> ;�� • ·04 (ii) (!) 252q s p s <!!.> q 101oq
l7+33
+ 9
p+4sqsp
2

(c)
- l - {q +loq p).
10 9
2, (i)
= lo+
- 1500 • •04 (ii) (_a) q 20
(!!_� 20q 9p (.9 q +20q l 9p+l90q 8p
l 20 1 2

J, Answers to (a),(8),(y),(6) are


(,!) q� (!!.)10q2p 3 (.£) lOq2p3+5qp 4+p S (!!)1-(5qp4+p 5), where
for (a), p • 1, •0 q• •99 (8) p =14/900, q • 886/900
(y) p • 32/ 1500, q •1468/ 1500 (6) p • 60/2200, q • 140/2200 2
;;,: )
(.! l0q p (.!.!) lOq 2 p 3 +5qp 4+p , where q• 16,
2 3 S
9/ p• 7/
16

� (page 398)
t<l+2+3+4+5) • 3 l• 2'6 J_, (!) 2 i: 2 (np• 3 X 2/3)
X O l 2 3 where (np• 3 x l/6);theoretical frequencies
of 0,
1,2,3 sixes are125; 75; 15; 1
125 1111 ! V• 16
2
P(X)
V V V V

np• 5 1/4
X =1,25 X 0 1
486;8
10;540;
180;30;2 where n•1024
243 405 270 90
15 !
P(X)
n n n n n n

1· ftl!) = a,a, ;, a,8, a·;,


1 2 3 4 5
lnieoret freq: 2, 4, 6, 10, 12,
a, a, a,a,a
6 5

10, 8; 6, 4, 2
4 3 2 1 [a• 36]

490 , 252
(!) � � 6·8 (ii)�• 7 (not a binomial distribution)

(a)14 (b) (1) 20 (ii) 40 (c) (i) 28 (ii) 42 (d) (1) 3 (ii) 2•5
E(X) need-not-be integral; this is-the average number of girls (in
the long run) in a family of 5, (e) 3 (f) (1) 2 (11) 6 (iii) 6•5
10C � 10 100C <�10 0 � l
(.!) 5;
5( ) (ii) 50; ( ii)-500; OOOC \ IOOO
50 500 ( )
lOC· k 10
( ) • ..JJ..' l20•
,!2, l 2 128

APPENDIX
SET APP. lA (page 403)
[Abbreviations; cges for converges; dges for diverges;
-osc. for oscillates)
(.!)
11111
{l, 2' 3 ' 4' s• 1 2 3 4 5 6
6'... } ; cges to O (.!?) {2'3'4'5'6'7'... }; cges to
1

(.£) (1,4,9,
16,25,36, •• • }; dges (.!!) {-1,2,-3,4,-5,6,- ••• ); dges
XXXII
\
A�SWEP.S
(e) {-1,1,-1,1,-1,1,•••); osc. (!) {l,J,5,7,9,11,•••); dges
Ci> {4• 9,4•8,4•7,4•6,4•5,4•4,••• }; dges(h) {1,1,1,1,1,1,••• }; cges to l
(!) {1,2 1,1, -1, 1 -1
3 4 5, 6, ...}; c ges t o 0
(1) {O,lo g2,log3,log4,log5,lo g6,.•. }; dges
1 1 1 1 1
(ll {1'3'9 '27' 81' 243'.. •); cges to O(!) (-3,-!,1,f,1,f, ... � cges to 2
2 4 8 16 32 64 ll 81 243 729 ••• ; dges
(!) 13'9'27'81'243'729'...}; cg es t o O (!Y (1.2.
2'4' 8'16' 32' 64' )
-4 5 -6 7 -8 9
(£) { T•1• 5,7, 9 ,11,.. ,}; oac. (E) {O,l,O,l,u, l,•• J; osc.
(g) {0,1,0,112,0,1/3,•••}�gea .o O(!) {o,o,o,o,o,o,•••}; c gea to o
l l -1
(!) 11,2,1,2,1,2,•••}; osc . <1>
l7z,1,7z,o,7z,-1,•••}; osc.
5 -7 9 -11 13
(.!!) {- 3,z,3•4•-5•6'...}; osc.
5 10 17 26 37 5 19 65 211 655
(.!) {l,3 •4•s·6·1·· ••); dges (� 11•2•4•s•'i6•32• .. · }; dge s
2. (.!) {l,2,3,5,8,13,21,34,•••}
). <!> 11,-1,1,-1,1,-1,..•}; 1 (!!) {l,2,2,4,8,32,256, •••); 256
4. (.!) {l,4,1,16,1,36,•••} (!!) {l,l·l,l•ll,l• lll,l•llll,1•11111,•••)
'3'. u� f or A is 64 sn d for Bis 58._!. 625;601 l•{l,1,2,3,5,8,13, 21, •••)

SET APP. 18 (page 413)


Category A(!) o;(.!!) l;(.!!) l; (.!) o;(hl o;(!) 2; (!) o; (!) l;

�:t
(.!) l; (-2) o;(� o;(1) \;(i) 3; Cr) o
Category B(c);(d);(f );( g);(j);(n) ;(v);(w) ;(y)
Category C(e) ;(o) ; Cg); (q); (s);(t ) ;(u)
o d verges)
: : �
�� ::�: ::�: �: : :,� 1 i
:
(!!!) a•5, b•6; un • 5 .3 + 6/3 ; no limit(diverges)
(!)(!) limit is O (H)(.£) (!!!) (e) 4.(.!) (iii) limit 5/2
(.!Y) limit O (.!!) (.!) w/2 (! !) tt/2 (!!!) 0
1 2 3 4 5 6 7 8
l· (.!) <3,4,5,6,7,8,9,10,•••}; U • 1 (iii) U-un • 0•1 when n•18
I I
t
(.!Y) (!) :� • 1998 (.!!) N • 1,999,998 !· (.!) from 155th tera
3 8 15 24 35
onwards(!!) 155 7, {2,�,2g,2f6,2zs,�,... }; U • 3; n > 100
U • l; n > •4� 7' 8 i.e. N • 8 !· (.!) 0 (!!) 2+1 • 3
71
(!!!) -1 (.!Y) 2/3 (.!) 2/3 .!2..• D > {325•5 > 18; U19,U20 ••••
.M.•l�r (!!!) n > {log1(35xl0
0. 6 6)/log107}·>
t 7• 989 i.e. N•7,8,9,., •
1
12. Seq. ia {l,l+r,l+r+r ,l+r+r +r 3,,,.•• } (.!) - - ��
2 2
(N.B, Sn· 1-r• if
r � 1. I 1-r
(!!) n • 7; 00) > 100, since O < r·< 1;
(Note(JQQ
1-r • log( lOO) > 2).
l-r

XXXIII
ANSWERS

� (page 4161
A• J., Positive integers: 8, log e m 1, lzx la• 4; Zero O; Negative
inteaers
•• -7·• Fractions
. .!. •
7' /. l..
.
10' , 3 • .!
i 'ff.
m 1
3' 3' /9 3'
, 57 • ...1Z.
100'
57
• • ��. l•l • �; Irrationals 12,w, e, log103, ·1,101001. •• ,
7+3v'S, . fi.
-2. A rational number can be expressed in the form !!l.,
n
where m, n are
integers, (in their lowest terms). There are no restrictions of m;
ni'O. • • .
1.,_(11)Ye1;5•0;0•6,0•i, •285714 (� Yes; 7/1; 41/100; t/9; 23/99;
213/999; 7/30 .,i: T,T,T,F,T,F
.l· W ad�ition; cloaed under addition (]_) If a,b are elements of
S, then (i) (a-b) (ii) ax b (iii) a, b, b; 0 are also elements
of S (y) (1) closed under+ x (ii) closed under+
(iii) closed under+ -x
i• (a) (ad+bc) /bd (8) (ad-bc) /bd (y) ac/bd (6) ad/be. Yes
!· l. (1) T (ii) f 2. (a) -1016 (b) 75 (c) 43 (d) 18 - 1116
Te) 11 - 6/2 .1, (a)-1 (b) 150 (c) x • 42 ( y • -24)
(d) x • 7 (y • � 4. (a) 6,928 (b) 3•464 (c) 3•732 (d) 3·732
5, 2nf 10. 6. (a) 'l;cls-7) (b) 6
� (a) (i) Yes (ii) Xo (111) No (iv) No (8) Not closed under
+ - x or; (y) (1) (r+iv+(s+q)/2 (ii) (r-p)+(s-q)/z

.!:. ·
(iii) (rp+2sq)+(sp
----------------
+rq)/z (iv)
Closed under+ - x t
(rp-2sq) /(p2 -2q2 )+(sp-rq)/z/(p2 -2q2)

4. (a) T,T,T,T,F,F (b) T,T,T,F,T,F (c) T,T,T,F,F,F


!. (1) No restrictions (11) z > 0 (iii) x,y,z,w positive
(iv) x,y same sign (v) x,y positive (vi) z < 0
(vii) x, y o_pposite signs i• (1) x S 7

------t-
(ii) X < -,Slff -� .,\ •r (iii)X � 3 ----.-·
(iv) 2 :S X < 4
t
�-,---·•1:-,.
J• PnQ -3 !: x S -2 PUQ -6 :S x :S 2
!· (1) -4 < x < 4
•) -� -i
(11) x 2 5 or .x S -5 --·- -- -..
s ,
(iii) x > 1/3 or x < 0
I • 1 • (iv) 2 < x S 2i. ---�::-
0 V> 1 1 �;,•
l_. (1) 15 (11) 72 (111) 10 (NB. 2 o > 1000)
(No graphs given here; similar results to above 6,7,8)
,l_. (11) T,F,F,T,T,T (iii) l
2. (a) x • •2 (b) x • 2 or -2/3 (c) x • 2 or 4 (d) -4 :S x � 3
'ce) x > 6 or x < -2 (f) Note l3x-41 < 1, l < x < 5/3
(g) x :l.7/Jor x SS/3 (h) -3 � fc � 2

XXXIV
ANSWERS
_TOPIC 2 (page 420)
i (11) range (iv) independent,
(ii) -3 �XS 3, 0 ! y'! 3
real y (v) x ,/, 0 ,-'y�n

B
- l, (a) x2+y2 • 1 is
not a function
I lC.

(ix) (x)

1. (i) -3; -13; 2a+2b-3; (2-lx)/x (ii) 4;3;1


'ciii) (a) x2 +2xh+h2 -2x-2h+6 (S) 2x+h-2 l· c(ax+b)+d
J.. a • 24, b • 1; 11-7-23 • -19
!O!I� 3 (page 422)
�. l• X • 6 (i) 4/5 (ii) 3/5 (iii) 4/3 2. x•25, y•8 (i) 4/3
TU) 17/8 (iii) 5/4 l· (1) 1/2 (iiJ O (iii) 2+,/j'
i• x sece_; _x_c_os_e__________ _
_ _
!: !.· (!) x; y; y/x G!) Quadrant I; x > O, y > O; all ratios posi­
tive. Quadrant II; x< o, y > o, sine > o,_ cos e< o, tane< o.
Quadrant III; x < o, y< Ossine< o, cos e< o, tane > o.
Quadrant IV, x > o, y < O; sine< o, coa e > o, tan e< O
(s) (i) quads 3,4 (ii) quads 1,3 (iii) quad 3 (iv) quads 1,3,4
2. (a) stn e, -coa e, -tan e (b) -sine, -cos e, tan e (c) -sin&,

i.
cos e, -tan e (d) -sine, coa e, -tane (e) sine, cose, tan e
(f) cose sine cote.
(!) 1312, -?J-12. -1, 1/2, -1 ill) -2, -l'l, -1/,/j',' -1, 0
• (!) 4/5; 4/3 ill) (b) 25 (c) -25/7 � cot S • -lS/8
-· (a) 1 (b) 1 (c) 2 tan a (d) 1 (e) cos S (f) 1
XXXV

;.j
AIISWERS
l• (!) 2+10x (!!) expression• 2 !, (i) x 2 /4 + y2 /9• 1
(!!) xy• 6 (iii) x 2 /4 + y/3• l (iv) (l-x)2 • l+(y-2) 2
..2.: (!) (a) 60 ° ,120 ° (b) 30 ,60 ° ,210 ° ,240 ° (c) 20 ° ,40 ° ,140 ° ,160 ° ,
°

(d) 120 ° ,240 ° (!!) (a) 135 ,225 (b) 30 ,210


° ° °
260 ° ,280�
°

(c) 30•,150•,210•,330• (d) 45 °,225 ° <.!!!) (a) 90 ° ,180 °


(b) o•,90•,270 ° ,360 ° (c) 30 ° ,150° ,270 °
(.!Y) (a) 1a 0 26 1 ,45 ° ,19a 0 26',22s 0 (b) 10•32•,120 ° ,240 ° ,2a9•2a 1
(c) 23•35 •, 156 ° 25 1 ; (sin e ; 2)
C. l (!) X• 100 sin 34 ° 18'• 56,35; y • 100 COS 34 °18 1 • 82•61
- - <.!!.) <>• '51 ° 33'; a• 17•01 l: (i) 3/6 (ii) 412 (iii) 30°
4. (i) 16: 2 (ii) 2/15 6. 144om 7. (i) 72·06m (ii) 56•6lm
8.
� (B) (i) a c2 b2 +c -2bc cosA� (ii) b
2 2 2 2 • a2+c 2
- 2ac cos B
b +c _82 8 +c2-b 2
2
8 2 +b2-c2
(iii) cosA• (iv) cos B • (v) cosC • �
2bc 2ac
9. (o) /io (B) lf6 10. (y) 82 ° 49' (6) 104 ° 29'
11. (i) 6 (ii) 120 ° 12. (1) 101 °32 1 (ii) ° 49 ° 27'
(iii) � +
5•83 13-:- 13•12km ; E 73 35' S; 14•33�m
16. (B) )iac sin il,,_ iil,c sin A 17. (i) 12 sq.cm (ii) 12 sq C!Tl
18. (i) 2/oC1D;U37'CM(ii) 24/J sqcm
� (page 426

l· (1) p: y • b;q : x a -a (ii) r: ya mx,s : ya mx+b


(iii) t:
! f + n 1
Grad• (y 2 -Y1)/(x 2 -x1)
(iv) u: y-y1 • m(x-xi);v:
y-y1. Y2-Y1

3. (i) -2/3 (ii) l:;(p+q) 4, (i) -A/B (ii). -2/3; 146 ° 19'
5. (ii) C, (4/5, 0), (0, -413) 6, a05, b
•-2
!, (!) (a) yn3 (b)2x-3y+ll• 0 (� x+y = 2 (d) 5x-3yT14 c 0
(ii) (a) 7x-5y+35• 0 .(b) 3x+5y• 35 J!..
- (I,) (i) (ii) 7x-3y • 29
9. (ii) p •l . 10. (i) (1,2)
'i2. (i) 14x-9y ;""o (ii) 22x+ Sy• 12 (iii) �7y = 42 (iv) 134x - 20ly
+72 = 0, 13. W (i) 1 (ii) 0 (iii) limitless number;
(i) intersect,are parallel or coincide •(!?) (ii) no (iii) limitless
(i) independent (ii) inconsistent (iii) depende.nt
(.£.) (i) lines coincide (ii) lines parallel
14. (a) inconsiste.nt (B)dependent (y) independent
15. (iii), (iv)
//��
�/i-J-i-1<

P .: 2x-3y+6 < O, 5x-2y-7 < 0 R: 2x-3y+6 > o; 5x-2y-7 > 0


S 2x-3y+6 < 0, 5x-2y-7 > 0
17.

For interior of

••
triangle x-y < O,
3x-y > 0 and
-,,Mi--...,..----x 5,c+y < 24 "
(ii) (iii)

•x
�. (!) k • 6 <.!!) C • 4, d • 3
...
XXXVI
ll.· (b) 2x +5y = 7 ll_. �; tzj; l�I; �;
ANSWERS
A,B,D are on same side
of line, 2 4. (i) -5 o r -2 5 (ii) A lies on on e of the bisecto rs
of the angTe'"s between the lines
XJ +Xz YJ+yz
<£!) (i) l(x z-x 1)2+(yz-YJ) (ii) (--, --)
2
2
k y2 +.iY1
2
kx2 +£x1
(iii) (�, .�> (8) (i) 615 (ii) (-1,5)
(iii) P(-1/7, 38/7); Q(4 1,26) 26. 26 \ sq units

�-
27. (i) 5 1mits (ii) 4x-3y • 11 (iii)3/5 (iv) 3/2 sq 1mits
P(-3/2, -5), Q(2,-6) 2 9. (i) T(7,3) (ii) D(-7,-4)
x1+x2� Y1+yz+y 3
(i) (1,-2) (ii) ( ) ,ll• (i) (O,9/4)
J 3
(ii) (0, 4/3) (iii) (O, 7/8)

� (page 430/
m-n
1 n
4. i, (i) a
m+n
(ii) a (iii) a
mn
(iv) 1 (v) 3n (vi) ;;.n
l· (,!) 1/9; 1; 2; 27; 1/9; 8/125 (.!!) (i) y (ii) 1/10
l6x+4 X
(c) (i) 2/5 (ii) X = -2 (y • 0), (g) (i) 2 (ii) 5 /81
�) 5' 3, 3\ 2 '5 J·
(1) (a) -8 (b) 4/3 (c) -3/4
(il) (a) 8 (b) 6 75/16 (iii) (a) 5·728 (b) -•378
x
(iy) (a) x • 0 o r 1 (b) x • 3 (2 ; -2 )
�· (1) x = 9/4, y = 3/2 (.!!) (a) x+2xl:i y\ + y (b) x - y

l• (i) {5,8,11,14,17, ••• } (ii) {1,4,9,16,25,••. }


- . 1 .1 1 1 1
(iii) { , , , , ,..}{iv) {2,4,8,16,32 , •. ,}
2 6 12 20 30
(v) {-1,3,-9,27,-81,.•. } 2. (a) (i) 6 95 (ii) 175 (b) (i) No
(ii) Yes; 75th term 3.-(i) 55 (ii) 126 (iii) -60 (iv) 5
42 n - 5 n
r l
E r (iii) E (-l) - r( r+l) (iv) E f(xr)h
r

3
(ii) E (3/4)
r=l r-1 r•l r=l

1· -1; 5; 11 i• (i) -130 (ii) -99 (iii) -31 (iv) u • 9-4n


n
n-
1: u • 2 l

1. (1) (a) 31 (b) 4 n-l (c) 136 (d) (4n+l) ill) (a)
-384
2n
n-
c°h) ;(-2) l
2n
(c) - 255 (d) l-2
l· (1) (a) 128/81 (b) 2 -l/3
2n n
(c) 781/1 62 (d) ti <1> n
-1)
(ii) (a) 7/ 6 (b) f< n+3) (c) 25/ 6
(d\�(n+5)
l• (1) x•2 . Terms 22,14,6,-l_,.• (.!!) x • 4/3 or -2.
Terms a re 64/3, 32/3, 16/3, 8/3, ..• and 18,- 6,2 ,-2/3, .••

!!., x•4, y•6 or x • 1/4 , y • �3/2


(!) {1,-4,-9,.,.}; - 69; -510 (� {12, 4, 4/3, ...} ; 4 84/27

l· (2.) (i) a +b (ii) • (a 2 -b )
2 2
_§.. (!) 56th (.!!) 6th
(!) (i) 6,18,54 or - 6,18,-54 (H) 5,8,1 1,14
8. (.!) }5 and 3 (.!!) {i) x•l5, y-75 (ii) x•l27, y•25l
i, (!) f•l05, l •497 (.!!) n •57 (iii) s•l7157

XXXVII
ANSWERS

_!2. (.!!_) (1) 10 (ii) 30/7 <J!.) (1) f11-(-!)nJ (ii) f

.ll.: (1) 2/5 (ii) 12+6+3+... 12. (1) 7/9 (ii) 17/99 (iii) 8/!,5

.....
14
(iv) 203/999 <v> 403/99F 13. 45
(.!) (!) -100 [Group series as (l-°3)+(5-7)+,., J
c1> cu>
10 (iii) 3

<J!.) 101 [Group as 1+(-3+5)+(-7+9)+••.] (.!!) (.!!.) 1425 (b ) 341


-3 11 41
(c) 2+( ) • .ll: (!) {25,22,19, ... ); 4(.; ) (!.!) 55
7 7 3
(i) 11; No, since limiting sum is 2• 5 (!.!) ·000000128
1 4 4 4 4
14(1 - nl; 14; 11 l!!.· 4 - + - ... or ? - + 4 - •••
2 3 9 3
Limiting SllD of first series is 3.
!2.• a•l6, b•3; 13; (i)
10 !!
+ 165 (ii) 16(1 - + �n+l) t�)
3.£· ll· 1 (Note a•4d] ll· aa 2 ll·
------------�
3; 2 8cl!I ;3cm �· 2 64 _1;
6_; 13 km approx.

!!.• 2. x -c . (x-c)(x 5+x4c+x 3c 2+x 2 c 3+xc 4+c 5 )


6 6

- a 7-b7 • (a-b)(a 6+a 5 b+a 4 b 2 +a 3 b 3+a 2 b4 +a b 5+ b 6)


_J..(i) x 2 +xc+c 2 (.!!) x 5+x 4c+x 3c 2+x 2 c 3-h:c4 �c 5
(ill) x-+1c\lt+c (!y) -(x4+x 3c+x 2c 2+xc 3+c 4)
's '- xScS
X +c�

TOPIC 6 (page 436)


£· J;/�a) 1 (b) l (c) -12 (11) (a) 5/3 (b} 0 (c) 3 (d) no limit (e} 1
2: W (1) no limit (11) -1 (iii) 3 (iv) 1/3
- (§_) (i) 7/3 (ii) 2 (iii) 3 (iv) no limit
l· (i) 5c4 (ii) 4x-3 (iii) 1/4

!!.· 1. (1) i>ot continuous (ii) cts (iii) not cts (iv) not eta
(v) cts (vi) cts (vii) not cts (note not defined at x•2)
(viii) not cts [Note lx-21 • x-2 if x > 2, but is -(x-2) if
x < 2] (ix) eta
l· ( !) 5/2 (ii) a•61 b•-8
(ii) (a) f(x)-f(c) l lim fCx)-f(c) (S) f(x+tix)-f(x).
!· l· x-c X-OC· x-c llx '
Um f(x+llx)-f(x)
2 7 _3• x5 -h: c+x 3c c 3 +xc4+c5
4

2 2
llx +O llx
-+,c

4. (1) -5 (ii) -1 (iii) 1/2 (iv) 1/2


!- 3x +3x(llx)+(Ax)2 -2x-(Ax)i Jx 2 -2x; •0201
2

Ii 25xV.. -2 6 21 4
!• J.. (a) Jx 2 +6x-5 (b))ll - - - (c) ::2' + :;J - :1i' (d) 1 + -2
2 X X X Ja X
(e) 14(2x+l)6 (f) -JOx/(3x +1)6 .(g) (3x-l)/(3x -2x)
2 2

5 l-x 2 -20x -Sx


(h} (1) "f 2 (j} •
(2x+3)2 x +1) 2 (2+4x2)2 (1+2x2)2
-l+x-2x 2
1
(t) x(l-Jx) 2 (2-15x) (111) ;r;:i (n) --''---f,- (o)
(l+x2 ) 2
(p) 4x(x2 +3)(3x 2 +4) (q) (15t2 -8)/(2t+J)
i• (!) (a) -3 (b) 4 (c) -9 (!.!) 19/27; x•2 (iii) (a) -30 (b ) 0
-5. (1) (a) - v � (i3) l �� 6, W (i) 14; 14x-y • 33
-.
V""
""d d
(ii) at x • 1, � • 4, 4x-y+2 • o; and at x • -3, � • -4,
4x+y+6 • o (A) (1) (-1,2) (11)(-3/2, 9/4) (iii) (3,18)
XXXVIfl
ANSWERS
7. (i) -3 at x•O; 2 at x•l; -6 at x• 3
i (i) a•2 (ii) a•l2, b•3 (iii) a•-3, b•2, c•l

TOPIC 7 (page 4421


� (i) 5/2 or -1 (!j_ ) to•m)
� (ill (a) 3/2 (b) -1/2 (c) 13/4 (d) -3 (e) 13 (f) -13/2
(g) -1 (h) -1/2 (i) 6 (j) 3/8 (k) 17/4 (ill) (a) -4/3
(b) -2/3 (c) 28/9 (d) 2 (e) 7 (f) -14/3 (g) 5/3 · (h) -1/6
(i) -3 (j) -16/27 (k) 40/9
l• p,q; p 2 -1 • 4q; when q•6, p• •5, and k•2,-3 resp.
-

:: �''. �':�'.''.' 2 '.'.''. � ,., Cb> A.t.hJ:';I

LI
(ti) (a) 6 0, roots unreal; no zeros
I
<

(a) 6 • O, ro ots equal namely x • 3 or 3;


0 J 4 X.
ez os
r are 3' 3 , : 0 1 "

(y) 6 >O, roots different,
x • -1 • /6; zeors -1 • ,qi ·•·JJ4 .,.,.
-��! '"
i• (a) real, different, irrational (b) real, ,different, rational
(y) real, equal (6) unreal or complex
7. (i) k • -7/3 (ii) k> -7/3 (iii) k � -7/3 (iv) k < -7/3
I,. A• 1 (when solution is x•3 or 3) or A• -7/10 (whence roots are
X • -2/5)
9. (i) 6 • (6b) 2 (ii) 6 • (a-b) 2 +4k 2 . If a•b and k•O, 6 • 0 and
= roots are equal(__(iii) 4 • -4(a-b)2
ll· (i) -1/3 (ii) -11/4 (iii) -1/2 (iv) 1/2 (v) 1 or 13
(vi) -7/5. (vii) 0

4· (i) liin 5 when x • 2 (ii) Min is - 3 1/2 when x • -5/2, (iii) Max
14 when x • -3 (iv) Max 11/4 when x • �1/2

5. Only pos itive defi�ite expres s ion is 5-3x+x2 (6 < O)
I- (i) Indef (ii) +ve def (iii) indef (iv) -ve def.
(i) (ii) (iii)

,. Jr.

-2 .S X .S 0 ¥ < !i or x> 3
!· (i) k(x-l)(x-2) where k > 0 (ii) c(x-3)(x+5) where c < 0
.2.· (i)
4 I
(ii)
�·
V'\
1 < X < 2
.(iii)

'"' x
&
·," )�v<' .
< -2 or -1 < x < 1
JC

or x > 2 .2r.x>2
.!Q.. (i) k < 1 (ii) -4/5 < k < 4/5 (iii) k> 1 or k < -1-/11
.ll· (i). -:1 or -5 (ii) 3 ( � -t) k > -1/3 ll· (i) k> 0 and
Jl·
k < -1/11 or k> l;· leads to k> 1 (ii) k < 0 -
and -1/11 < k < 1,
leada to k < -1/11 (iii) k • 1 ( � -1/11)
(i) k � 5/9 or k? 5 (11) 1/3 < k < 3 1iii) k < -3 or k > 1
(i) p � 4 or p .s -4 (ii) p � 4 (iii) 8//J

2. (g) A• 2, B • -7, C • 14 (a) A• 2, B • -3, C • 1


l- (!) (a) A • 1, ll • 2, C • 1 (a) A • 3, B • -3, C • 1
(ill a• 4, b • -13, c • 5, d • 3 f(x) • 4x 2+3x+l; 1/4 or -1

(i) -3 ·or 1 (ii) •4 or •fi (iii) -3/2, 2, 3/2, -1
(iv) 1, 1, -2•/J (v) -4, 1, -1, -2 °(vi) 2 or -1/3
.!?.·

l.· (i) x 2 • •12y (ii) -x2 • -24y (111) x 2 • -18y [Let curve be
x 2 • 4ay, aubatitute (6,-2)) (iv) x 2 • 8y [Let parabola be
XXXIX
ANSWERS
x2 = 4ay, solve with x+y+2 = O; equation then has equal roots,)
l· (i) (x-4)2 = 4.6(y-2) (ii) (x+2) 2 = -4.7(y-3)
(iii) (x+l) 2 = 4.2/3(y+5) 4.(i) S(0,2), V(O,O); n: x = O;
d y = -2; £ = 8 units (ii) S(O, -3/2), V(O,O); n: x = O;
d y 3/2; t = 6 units (iii) S(-4, -5/2); V(-4,-3); n: x = -4;
d: y = -7/2; t = 2 wiits (iv) S(2, 4/3); V(2,2); n: x = 2;
d: y = 8/3; t 8/3 wiits C

�. t•·Z (-4c,. ,4o ) Y (+4,-4<,) t•Z £.. (x-3) 2 • y+l; V(3,-l); S(3, -3/4)
7. (i) x2 = 2(y+2) (ii) x 2 = 4y
a. (1,2); lines 2x-,-3y+4 = o, 4x+Sy = 14
To. (i) p•l or 4. Tangents are x-y = 1,
� 4x-y • 16 (ii) p • -1, -2, 3
>( �:orrnals are x-y+3 • O, x-2y+l2 = O,
•O x+3y = 33
At (16,16) tangent is 2x-y = 16; normal is x+2y = 48
At (-4,1) tan is x+2y+2 = O, normal is 2x-y+9 = 0
Tis (6,-4); R is (6i21)
ll: x+y = 3 .!I, (i) x = -ay (ii) 2x = 9ay (iii) x = a(y-a)
2 2

Sa > -
S(O, -i; V(O, a) 18, (i) Locus of R is y = 0 (i.e. x axis)
(ii) Simplest method is to eliminate t between the tangent
y - tx + 2t 2 = 0 and perp, ty + x • 0 19. (i) y = -4a
(ii) x2 = 2a(y-4a) (iii) x2 • 16a(y-6a-r--
J.: (i) x = O; the y axis (ii) x 2 +y 2 = 16; the circle on diameter
AB (iii) If Pis aLove x axis, noting tan PAil = :::t.... , tan PBA
x-4
is�. locus is x 2 • -8(y-2); i.e. parabola,vertex (0,2), focus
(O,O). If Pis below x axis, locus is x2 = 8(y+2); a parabola
1· ,,,·"' ''·-�;�,.._,_�

A y ,;f
y ,,,
(ii) 'P (iii)
\.p /
p/ /p
1i) '\ /f..t... /
x' X
x' )(
/

TOPIC 8 (Page 450)


!: .!· (a) monotonic decreasing (m.d) (b) m.i. (c) stationary
XL
·1
I l'----1 m. I
ANS\!ERS
(a)
.+-1-t--+ m.d. for 1 x 2; (b) m.d. (c) m.i.
< <
}.,
i. for 2 x 5 < <

l.· (i) (a) (!,) x < 1 (ii) (a) x < -1 or x > 2 (b) -1 < x < 2
(c) x • 2 (a) as for (a); (B) as for (b); (y) as for (c)

4. (ii) (a) -2 < x < 0 and x > 2


(b) x<-2 and O < x < 2
x (c) x • -2,0,2 10
'I l /.../ ��
-5 � o �x
(i) m.i. (ii) m.d, /
(iii) m.d. -/.,.-'Y•• i�
(iv) m. i.; f(-2) • -4 ;----4''�-­
f (2) • 4; X ; 0 )(

/ I I
I I I
'"'/�\ � I I

,!/,· y • (x-1) 4

(ii) f"(a) = 0

(iii
��

1. (a) B,D,F,ii (b) B,D,F,G,li,L (c) B,D,F,H,I,K (A,�!) (d) K,F


(e) G,L (f) I;K (g) B,D,F,G,H,I.,C:,L (h) C,E,G,J,L (i) C,E,J
(j) B,D,F,H.

1. (i) (3,-4) Min


I, (4,80) }lax; (-2,-28) Min;
- (1,26) Inflexion.
(i) Reason obvious from sketch
(ii) 3 real roots (iii) x < 1
(iv) 27x-y • 1
[quation is x 3 -3x2 +3x-l • 0 i.e.
-2 �.
3
_. nin; 0 u
aaax
3
Greatest value is� when x • .12

-
and least value is3 -3 l when x
-3 3
• 2
!• (O,l) horiz.inflex.; (1,0) Min;
(2/3, 11/27) inflex
y
-�· X

X • 0, 3/5, 1 (O,O) ho�iz.inflex;


X 3 108) ,•1ax,
<s,Jl25
(1,0) Min.

XLI
ANSWERS
(i) (ii) (iii)
I y
o
-,
2 x I

I
r I
����.P:..�---.,t:.-.!-..
-1 0 I 2 !
Greatest is 1 when greatest 3 when x•-1
x•3; least is -3 least O when x • 2 greatest 5 when x • 3
when x • -1 least -4 when x • 0
O
l: (i) x , 0; (1/4, 4) (ii) (8, l� ) Max
9
1
0 �

(1,0), (0,-1); x • -1; 1; le:st valu: -3�
when x • -1/2, greatest value 1/3 when
_..,+-
,...'_,__,...___
X • 2,

:..\,.
.,i, x a O,l; (1/2, -4) l-!ax
(2, -3) Max; (-2, -1/3) Hin
12· /I fY

�I' "
11\
I ;
11.
(1, 1/2) Max; (-1, -1/2) Min;
(O,Q) inflex, (/3, /3/4) inflexn,
(-/3, -13/4) inflexn. Limit is 0

(6) a • 3, b • 2
(ii) A• -3, B • -9, C • 0 (iii) A•-3,
1, b • -3, C • ·-9, d • 5
.U· (i)

l· x • 2 (least Sllll is 3); when x • 5, greatest


Sllll is 129/25. See sketch.
3, x • 2/7 _!. h • 4a,(r • 8cm) � 6kli
I, (i) 1000n cu c.a; least vol· is 29611 cu � when r • 2; greatest
vol. is lOOOn cu.ca when r • 10.
01) 300n sq ca; greatest S.A. ie 100411 sq ca when r • 2; least
S .A. is 30011 sq C1I when r • 10.
l· (i) greatest vol is when x • 2; least vol (here 0) when x • 6,
(ii) v .=:.. - - � - - Greatest vol when x • 6, i.e.
I all material used for cube.
I Lea•t vol, when x • O; no boxes

IX
0 .. • ?
,l!,, (i) x • 8kll ; 2� min (ii) (a) He aoe• direct from A to Q;
tille 15 min (b) He isain goe• direct fr011 A to q; time
3/i;o min+ 9•5 min
2

l· lE-y • 1, x+Jy+l • o; (1/5, -2/5) ,!, (i) x+4y+40 • o [Point


on curve 1• (4,-11) �. 2x+y • 4; (4/3, 41/12) � Q(0,3)
XLII
A'.ISWERS
� (2,2 ) i• 4ax-y • 4a; 2bx+y • llb+9; a • 1/2, b • -1

w
(Constants omitted in question 1)
1
•· (a) l4 x% 5(b) 1 x21J (c) 118 (d) .:. ...!6
6x (e) x (f) l!. 5x
3 _ 2 •· - 7x
8 3

(g)4 x 3 lx 3'x 2 l' 2 o/ I,


33 - 52 t 2 (h) - 2 x 2 - + 3x 3 (i) 3 x 2 + 5 x 2 - Bx
2
2
1 4 7
x
x x 7 1
(j) 3 + 2 - 6x (k) X + X - x3
'
x2 I, 1 s
ct> 2 - 4x - ;
1,, (i) C • 0 (ii) C • 4 (iii) C • 3
(iv) C • 9
.l_, (i) y • X 3+x+l (ii) y • x 3+7 (k•J)
(iii) y • x 2 -4x (a•2, b•-4)
1 2 1
(iv) y • 2 x + x (Hint let point on curve be (-l,Y1) and find
equation of normal with y, in it, and note normal passes
through origin. I
4. (i) (y • 1 - 2/x); 3/5 (ii) y • 3x 2 +2 x+4 (iii) k • 3
's. (1) 13 (ii) y • 5x 2 + 3/x - 2 x + 9; 49
r. (i) f(x) • x 3-Jx 2 +4x-l
2
(11) y • x 3�2x 2 +x+5; 139/27
180.
J..· y • 4x o/ + Sx (k•l); ; 26; 1, This curve passes through
point (9, 180:) and is increasing (gradient 26) and is concave
upwards then,
TOPIC 9 (page 460)
2, (1) 2, 762 8 (ii) 2, 7335 )Integral +2 2 ·7622
(a) 7•912 (b) 7•918 Integral • (616 - 212) + 7·911
3
l,
:_. (i) 16•652 (ii) 16·52 3 (iii) �ntegral • 12 log 4 + 16-636
(t) Constants omitted
x l + 2x - -
1 9x 5 3 1 1
S - 2X + X (c) - z2 2 3x 3 -
(f 2 - l3O x t'2 + 6x
1, (a) (b)
- (d) 2x - l!. -\2x (e) x l o/2 _ l /h ) l!.
)
2 3 5
2

(g) l /!2 + 4x� - 10 x f2 (h) l!. - l!. (i) l!. - l xl + l!.


3

2 4 2 4

10 3 21 (k) 5x 2 + 711 + .!!.


1
x + x +
3 . 2 4 2 3 4
(j)

-2, (a) �x+l)IO (b) l(4x+3)3h (c) - l(2x+3)-4 (d)-...!(3-4x) 6


3 2 X X

10 24
(e) �l-4x)�3 (f) - ...!c1x 2 -1)-1 (g) �Jx4-5)4(h) 2 (x l -4x-1)�
6 8

16 14 48
3
(y) l· (a) -6 (b) � (c) -2 (d) 9 (e) % (f) (g) 16 (h) �! -!
* (j) � l'. (i) 4 (ii) 256
t ce> o (f> rlc3 1...:2 1 > • 61
(1)

,1. <•> 1 ! (b) 1 cc> (d) .1f H <s> o

l?.· l· �11(g(y)J 2 dy. !• (i) (a) ( x'la dx • 4�

Jc
(b)
1 2
y 3dy • !1 (11) (c) he ,,
,rx 73 dx • .!}!1 (d) !
2
'II'/ dy • 12
6
1 7w
f�·cu cu
l 4 5 I
o> � 1q cu> 5
(b) a.
(-2, 0),(0, � ( �
� ) !,�!

l��;l' cutj

(ii)

Xllll
l· Y+.

o
ti 10
(i)
4 q;
2
� sq (ii)
A: ,S\·!EilS
5
:�
n
cu b. t
9 - 4 • 12. q
sq; (1)
1
� - 2 • i sq

- 2 • (iii)
I (ii)
3 3
s 2 3 6 s
:z.. �
3
�n
(iii) (a) _- - (b
)
3011

1-111- 1
(i) (a) 18n (S) J6n (ii) 5


3
(i) ; +
6
� sq c:i) (0,0),(2, 4 );
3 1!

�� y cu. t· sq;;� cu

!�: y • 3x 2
- x
3
; 2r sq �-l ��
I
_1�. Max lj,<\ Y. CB) _
20 15
- - Sq
3 I
62511
""T2' CU �
;;.o
/
/ ?- �
.. �\
12. Height is a �\5 )( ' ... -
87311 - - -
!,f
1 1 83 158
Area 2 + 10 • 12-j sq; Vol • 11( + - -) • cu
3 15 3 15

TOPIC 10 (Page 466)

"'C. 2.
x'
2x
(1) !. 2e , 1. 5x �i' •
' x' '2x /logx
1 ·l ex /2
'2
x
(ii) -1, - 12; 3x 2 (1+1og(l+x 3 )J; (1-llogx)/x�; x2 e (3-x);(l-2x)/e
-x

1 2 6x+4 3 7 40
(iii) 2x-7 ' 2x-7 + 3x2+4x-1' 3x+4 - 7 x-2' 5x+9
2x
2x . _l_. x · l e (2 log 3x - 1/x). _1 __
e (! + og x).
x4-1' l+ eX '
(iv)
x ' (log 3x)2 ' x logx
(v)
X 1 -x 2 1
3 log 3; ; -2x • 10 log 1o, (1 + log x)
x log 3 e 10g 10
e e

(vi)
x x
x Cl + log x); (log x) [log(log x) +
1
g )
r;;--;;
x- x-2 3(1-Sx) l
( 2) (1 + log(x-2)); Z - )' ;
(l X (l-Jx)'i ¥,
(1-x) 2 (l+x)lj
1· (i) l; 6; 12 •.Curve passes through (0,1); is rising there and is
concave up. (ii) 6 (iii) e-1
4. (ii) l or 3
e l a a 2a
x-y • l, x+y • l; l sq unit; ( e-1' e-1
) §_. y-e • e (x -a); e

J· x<l
5
!.•
�- ...,..
(1) (a) X > -1 (ii) (-1, - !) �in; (-2 • - 12) inflexn
e e
(iii) (a) none (S) one (y) two
2 )
(iv) x+e y+4 • O; (-4,0

lC

y' • 1-l'tfiX
Y" •
2 log X - 3, (e !) 'f (2 •7Z
x • XJ Hu:;
j
• 3Ji • ' e '
Inflexion at (e , .- ti)+ (4·48, •335).
(Cuts x axis at (l,O)

'Y I(

XLIV
AN SHERS
U: i,n,en x • 1, y = 2 . Curve passes through Y
point (1, 2 ) and is rising there,
�I /I
0 14 ><

(Constants of integration omitted in question 1)


l 3x l x/ 2 + l e2x e-x
(a) Se + 4 log x (b) � - 3 log x (c) 2e -
x

l l 2 2
(d) Jx - 2 log x (e) - ; + log x + z;zi (f) x - 4x + 4 log x
2x
2
(g) e _ e-x _ l e- (h) l e _ 2x _ l e-2x ·
x 2x
2 2 x-3
(i) l xl + 2 log x - l (j) e l e5x+2
2
+ log(x-3) (k) ,
3
7 log(S 1 3x
3
l 2 -9X 1 4x-J'
+ . e
D
5 x -2 ) (�) - 2 log(3-4x) +
9e
(m) f 4
log(2x -3) - t<2x-3)-l (n) log(x 2 +8) (o) -log(4-x ) 3

(p) log(x2 -3x 5) (q) log(x 5-2x +8x-l) (r) ex (s) ex -7x+l
2 2
3

x3+x-7
- ' e 1 x 6-3 1 - 3
(t) e 8
(u) log(x +2) + 6 e (v) -
3e
x

1 5 1
(w) 5 log(l+x) (x) 2 log(l+ 2 e )
X

l_. (i) 1 + 2 log 3 + 3·20 (ii) 2 log 2 - log 3 = log(1) � •287


x - -x l .
1· (i) ex + ee-x; 0 (ii)�; loe(l+ Ii 2 ) � •88
e
i· (a) log(J/2( + •406 (b) t<e-1) + •859 (c) f<e 4 +e-4-e2 -e-2 ) + 11•77
(d) log(S/3) + ·511 (e) flog(S/3) f · 2 55 (f) fci-e-2 ) + •43 2
1
(g) f1-e
-210g2J • f (h) t log 2 � ·2 97 (i) f<e2 -e-2 -4)fl·627
(j) -3·5 + 6 log 2 � ·659 (k) flog 3 � •549 (!) fci-e-1)�·316
(m) flog 3 + ·549 (n) f11og(2e+l)�log3) + •38 2 (o) log 2 + ·693
+
(p) 9/logelC 3•909
5. (i) y =e + 2x - e- + 2 (ii) (3, 4-6log3)
x x

I· (i) (a) 4 log 4 � 5·545 sq units (b) 7·5 - 4 log 4,t::.•S55 sq units

t�
(ii) (ai 12n cu units (b) 2 lw-12 w•9w cu units (iii) 4log4,t 5.545 sq
units [Sam� as (i)(a))
(i) e _ 1,218 sq

(11) }<e 2 -l) - } = fC3e2 -5) cu.

x w 2a - 28 1 a -a
(ii) -8(e + 4a - e ) cu (iii) -2 (e -e )
units
i: (1,2) )!in; (-1,-2) },ax
---�:....C�--+.- (1) f<e 2+1) sq
-x "
e3 l 4.
(ii) w[ + 2e - ; - J cu.
3 3
-'I
10. (i) y • 2e - 3e-x + 5; 7 [Note when y • O, 2ex - 3e-x + 5 • 0
x

and let v • ex. Prove ex • 1/2) (ii) f(x) • log x - 2/x - x - 6


11. (i) y • l �x (ii) y • e-Z (iii) y • log(2xH) (iv) y • •12x+l
x

396 •
[:,ote if Y • e' , , . log Y • ·396 and • •. Y • 1'486)
1·0986 ·6931,
(1) 1,486,000 (ii) t > •0198 �• 55•4 yrs (iii) t • • 0198�35•0 yrs
-
585 lb [ll • 1000 e · 02231t I 14. B•!lo e , where k • l:.log 3.
kC

t • 4 loglO/log 3 � 6•38, i.e. time is 8·23 p.m. approx.

XLV
AilSWERS
TOPIC 11 TRIG.FU�CTIONS (Page 474)
3. (i) l'.:iOSm (ii) 35·8 sq m
- i• (i) 3.:a,3.'Jm
1511 r: 128'2 11
(ii) (- - - 9,3) � 7•98 sq Cit 1: 4811 sq cm; 19 °28 1 ; - - cc
2 3
727!0km (the figure 4 is unreliable because of the use of 4-
figure tables) l• (a) 105•4 m (Hint: through centre of smaller
circle, draw a lineparallel to the straight portion; Use Trig and
Pythagoras' Theorem) (b) 117•3cm (II int: use similar triangles, or
the method of (a)]
(a) 1;, 1!11 (b) 9 1 i• 4
1;, ;, :11 (c) ; (d) }• ;, ;, ;
2 4 5
.
(ii) (a) ·4637, 3"6053 (b) 1!,11
2 (c) 0,11 ,211,--3114, 4
711

(iii) (a) 611, 511


6 (b) ·8410; 5•4422; 1·2311; 5·052\

.!!.· U (a) Y

/-1 •I
/'
all real x; all real x;

r
lsin xi .:: l; 211 lcos xi � 1;111 11
(d) (e)

,
.,
0 !!:
a "'Y air )( 0 ,r
'f :ttr

/
x.; i<11;
V\1 I 01
X ,j, (2k+l) 2; X ,j, k11;
lcosec xi � l; 211 -a, f: cot X < co; 11
y 1sec xi � l; 211
s
l: +

XLVI
Af:SWErlS
1.· (i) y

Solutions - 54 • ,18·0, 90 °
y Solutions

I
Solutions O,
ff 11 · ..1!. < x < .!!.
11
or 3ff < x < - _or
4, 2 10 6' - 2 2
7ff < 911
< 511 < < 11"
10 X 6 .2!'. 10 X
,Y
8.
=i

o
ff
(i) ?fax value is 2 when x • 6
(ii) 1711 fApprox. solutions 21 ° 28', 90 ° , 158 ° 32'
24 • 127\
0

l.P(cosa, sina); cos(a+y) • cosa cosy - sina siny;


- sin(6+8) • sin6 cosB + cos6 sinB ; sin(c-y) • sine cosy -
cos£ siny (tanu•tanB
)
-
2. tan(a•B) • i -;tana.tan.fi_ 3. (a) sin60 ° /cos60 ° • /j
- =
(b) tan45 • 1 (c) \(lf+«i) (d) 2-/S (e) -J..(16-12)
°

-4.(i) 63/65 (ii) 3/5; 5/13; 63/65 5. (i) 7ff/4 (ii) -7/26
l
,g. (1) 2/3 (ii) 4/7 (iii) 6/17 J..•
72(sin x + cos x);
1(cos tan x + 1
72 x - sin x); 1 _ tan x (i) ri(sin 2 x-cos 2 x)
(ii) \(cos 2 x - sin 2 x) (iii) -1 8. (a) 0 (l) tan x
-9. (iii) -cos28 10. (ii) tanA • 12;- A� 54 ° 44'
-;;-
_!!. (i) R • 2, a• 6; 2 when 8 • � (11) (a) Am2, B • �
lb llff
211 411
(b) B • 2, y • ""} (c) C • 2, 6 • ""}
ll· (i) R • lie, a • 341° 34' (ii) R • 25, a • 73 ° 44'
cot B • .!
J.· (ii) m1m2 • -1 (iii) tan�• tan(1 2 - a>• m2
2. (i) 70 ° 34' (ii) 71•34• 3. 85 ° 14' approx.
i- 3x-y • 3; x+3y • 11 l• (2,3) 4 24.';(l,1), 45
° °
�·Y �t
i• 90 ° at (0,0); 29°45' at (1,1) and at (-1,1)� !I·
0 )(

l· (i) sinx < x < tanx < cosx < coaecx


(ii) sin46 ° � -fio
+ : > + • 72 i· (a) (i) •0279
1 0 (ii) •00349
(b) i •21 1 l: 61..om !· (a) 2 (b) 1/7 (c) 3 (d) o (e) 6

XLVII
ANSWERS
{f) 5/4 . (g) 3/2 + 5/2 = 4 (h) 2 + 1/3
.2,, (i) ?;ot c ontinuous (ii) not cts (iii) cts
F, l: ( ii) cosx (iii) -sinx
�3, (i) 2 cos 2 x · 12 sin 1 . 2x sec (x ); e xcos(ex); -sinx ,e cosx
x'
2

2(log' x )X
2

(ii) sec ; 5 cos (Sx - ..!!.).' - �"� s in x '• cos(sin x) .cosx 0

X 3 180
-sin(2x +l)
{iii) sin6x cos x; ; 12 tan 24x sec24x ;
lco s(Zx+d 3 COS
-6 cos 2 (3 - 2 sin 2) (iv) ; - 7 Sin X
l+ 3 sin x 1 + c os x;
X X 5 X

-ax{b sin bx + a c o s bx);


x (3 sin x + x cos x )
2
(v) -e
s in J x Jx
�:� ;: + log(tan 3x );
2
+ 3 cos 3x log 2x;
x
-(2x sin 2x + cos 2x ) (vi) s 2 + sx 2; -3sin3x - 4cot4x;
x2 ( inx c o ) l+ cos3x
2 COS X + 3 ·
2; 2 x cos 2x (cos 2x - 2x s in 2x ); 13e c os 3x .
2X
(Z+Jcosx)
l· l; 1/2; curve rising 45 ° ) at x = O, and c onc ave up-
_wards then. (1) y = x 0 !: x .:: "/6 or
Sn < X < 2Tl.
�.. {i) Whe� � = 0, X = -1;
6
cos
Same values for
x • n, Jn. ••• However f(x) to be conc ave up.
1; doe not change s ign thr ough
s

these points, ,', curve has horiz. inflex ions at each,


(ii) Stationary points are (3, 13 :tax; (2n
" 3" + 2) 13 �fin;
3' 12!.
3 _ 2)
4n 4Tl 13 , Sn Sn 13) Min; Inflexions are {O,O),
(3, 3 + 2) .!a x; <3 2
• 3 •
ff ff
(2, )T! )T!
2 ), (n,n), (2 2 , ), (2Tl, 2n), y

n n 3
l.· Stati onary points <2 , 1) Nin; (6, -) Max. l

3 n
Greates t value is
2 when x = 6; least
value is 1 when x • 0 or .
2 0

A • 6r-r 2 [Note 2r+re • 12; A = ',r 2 e]; 9 sq,_."-

{Co ns tants omitted in ques tions 1,2)


(a) - 1 a cos ax (ii) l a sin ax (iii) la tan ax (iv) l a
tan ax - x
{a) - 2 cos x + 3 s in x + 4 tan x; {b) 13 s in 3x + l cos 4x+ ;t 1 an8x
4
( c) 3 s in L ff l
3 + 2 cos 2 (r.) 2 cos (3 - 2x) - 3 tan{3x -1)
X X .

f
{e) tan 2x - x l· {a) 1/2 {b) 2 ( c) l - n/4 {d) -1/3
{e) ,'2/2 (f) 0-,'2)/lS (g) 1/0 {h) + sin 2a f
(i) 0 t
{i) ..!!. + l {ii) ..!!. _s. Ci) .!!. - 1 (ii) 1
i: 8 4
{iii) l
4 4 2 3
1· (1) l log{l) {ii) lee " 12 - l}

l· (1) y
4
3
•·l;c o s2x + )sin
2
1 + S tan Sx +· 2
l 3
(ii) y • cos2x+sin3x-2x+3;
1-11, 8. {a) (1) 1 (ii) 4; definite integral • 0
f; 5
:, .. ,; (,'2-1) sq 9. � (1) (n+2) sq
t
(B)

- � :ii) (311+8) cu

.!2· (11) [O, 1 +


fl (111) (a) �log2 sq (B) n{l - 4) cu units •

XLVIII
ANSWERS
TOPIC 12 (page 483)
l· See text, page 3, 4 l.· (� See text , page 9
(.!?) See text, page 13,14 J.• See te!(t, pages 5,10,14
511
i: (a) .! (b) .! (c) !!. (d' - !!. (e) • - .! • (f' !!.
-4-J-6!:.'2- 6 6� - 3
II 11 11 U S11
( ) 0 + 0 •
2 (!!) (- 6) - (ll - 3> • - 6
.& 2 -
l.· ( 1) (41 °
49 1
) +
•730 (.!!) (180 ° - 53 °8') f 2·214
(ill) (- 26 34') °
+-
464
i• ;!,-xz; �
0

'nte derivatives of cos - 1x, cos-I x/s, cos-lax, tan-1x, tan-1 x/a,
- 1 1 -a 1 a
tan 1 ax are respectively - 7i=ic"<• - -,:r=;:r, �· ltt2• 82+x2•
a 3 1 10
( (.£)
l+a2x 2• !• � 7l-'9'x7 (!!) - � 2s+4x2
x cos - 1x
e 1
l2· (� � (!!_) - 7r-x2 (.£) - '+l
x
(&) x [.zt� 2 + 2 sin -Ix]
x
{1-e•A
1 12x
(4' -[x+ri:i7cos - x] lli.&l!_ tan- 1
(!) 1+4icZ + - -- (861 -i,7s- !'i � ,.,l2l"" -. "'cil"" ""4 �2 "°f
x2. Jt:'i2
,
!., x in- x - x
- 4 2 sec 2
x
(.1) 2 tan-I x.
(!!) -------
c oa- 1 I,
/4 -x2
(1)
1 + 4 tan2x

11 •
Y + 6 • 73(x+l) (.!!) y - 4 • -l(x)
(1) 1
(Constante omitted in questions 12,13,14)
(� ain-lx or · -cos - lx (b) - tan-lx (.£) sin-I .!!. a
or - cos-I .!!.
a
(&) ! tan-1 .!!. (!) ! sin-1 ax or - ! cos-I ax (!) ! tan -I ax
a a a a a
i'! sin -I
ll· C:./
(a)
b
sin-
bx or_ 1 os-1 bx
a b
c
a ab
1 .!!. or - cos-I .! (b' 31tan-1 .l!. 'c' lttan-1 2x
=
(ii' ...1 tan-1 x.
b
a

- 2 2!:.' 2-..::1
(d' lsein- 1 2x or
- ',,coa- 1 2x (e' ! sin -1 .h or - 1 coa -1 3x
!!/
:::.,3 S 3 5
(f ' ...1 tan-1 3x (8\ i.tan-l(x-2) (h' s1n-1cx-2) or -cos-ic x -2)
:./ 15 S IV 4 !!/ 4 4
• 11 11 711 211 7fl 11 •
,a. <� 6 <!v
TI 2 <.£> C <&>
TI W 9 D 36 <&.> 12 <!!) i
• " 13•• •
( ) ( (
.!, 2 .1) i <!!.>72 !.) - 12
(� s1n -1<f> • (S3"8') •927 (.!?) tan-12 - +(63 °26' --45° ) f•
, • 322 (.£) sin-I f-f• (48 °35' - 30 °) + •
324
(&) i.(-36 °52'+26 °34') + -·0450 (!) lt(2.dn41 °49') + •730
2
(!) �(78 °41' - 68 °12') i •0183 J.1· 1> ; sq unite
1 �
(.!!) (71 °34' - 45 °) •464 sq units�,. +
.12· '1 .!) 3
sq unite (!!) ain-1cf>-a1n-tcf> + .390 sq unitl

ll· (1) 11 + 3 log 2 (!!) i


.ll· (.!) rt<f + 1) • ·� (!!) !late: put a • o in

w: -·=:=-=
XLIX
ANSHERS
TOPIC 13° (page 491)
.!· (!) 7
3 2 -
<ill 7' 7 ' ,; 3 :2:-6 (!!!) (!) <t, 1, -3)
- 3
(
!!)
(2,
1, -4)
b) sin-1( i) + S8 °S9'
(iv) ( !) cos 1( 7) i 64 37' (-
°
7
3•!•
(:'!) (_!) (2_) �6 (� X • 3t, Y • 2t, Z • -6t
x ..i.__ z ,, �\ x-3 z+6
(!!) 'i; • • •Ji N.B. cosy•·� (£, - - • cl• -S-
-1/12 1 2
(!!) (a) y • 2 (!!) 3 x + 2 y - 6 z • 7 i.e. 3x+2y- 6 z • 49
7 7 7
(£) Sx+7y-3z+29 • 0 (:'!ill x2+y 2 +z 2 • 49
r.= 1 -6 13 7
l· (.!) ..206 (.ill 1:-6:13, -m• 7ior• � (iii) (!) < , 1, )
2 - li
18 -8 9
<!!..> <� <-5- ,
5, 5> <!> < 6,-16 , 33) <.£.> c1,-6,13>
38
(� cose • (where e acute) (� (!) x • 3+SA, y • 2H, · z•-6-8>.,
63
For A• 0,1,2; points are (3,2,-6); (8,3,-14); (13,4,-22).
(b) l!:l .. cl• z+6 (c) .!:! • ili • z-7 (vi) (a) 4x-4y+7z•81
6 3 2 -
(� 4!-4y+7; • -3 � ci x-�y+l 3z • 16 �vii)(!) ;ose "' �
9
(!!) sin,j, • �� (viii) (x- 3) 2 +(y-2) 2 +(z+6) 2 • 206 l• Ji/66
1 x - 1 y +1 z • 2 • 2 units• 1 - 1' .!. (!) x-S • ill • .!±!.
3 3 3 ' ' 3' 3 3 2 -2 - 91 '
(13,-11,0) (ii) 2x-2y+z • 12, No. .§., (!) (!) 2 (!?_) 11 (.ill(!) 2
3

(!!) - n l• ( !) °j (ii) °j !• X • -1+3 A, y•.;.3+9>., z • 2-6 >..


[Other forms are possib le,]; (1,3,-2).
(a) tt t t
(b) =�,�.....,·--- (c)

,t -:t
plane parallel to a pair of parallel
,t

xy- plane planes.


(f) ,. t

'I:
curved· surface
f h•iaphere
(h)

,J,d,
right circular parabolic ri&ht circular
cyliqder cylf.nder cylinder.
l
E
(j) (k) (!,m)

'cl
" 't .,,
paraboloid of revo- Circle cut off by
lution. sphere on plane z•2. (!) line of intersection
of planes x•2, y-3
(a) Surface and non-origin of plane (m) surface of either
See 9(a) plane.
(b) Interior of sphere.
See

::; '"�"::"c:"'-'t,,,C--f---1-.
Interior of cylinder, Interior and surface

�il
between planes of sphere, above plane /
z•l, z•2. z•2 and below plane Shaded quadrantal

.e
2 region.
(g ; ··; (i) '3'

Interior :nd surface "'t


of paraboloid
H.
origin side of plane
y•S-2x, above xy- plane. bolic cylinder,
an11d�
\ ��ll!IIW!.� to right of plane
y•-2.
.U.· Jl• (!) x-Sy-llz+lS • O
(!!) 2x+2y-3z+3 • 0 (.ll!) x+2y+Sz • 13
(.!!) 7x-8y+3z • o (y) 4x+3y-Sz • 10
13. 16x+l7y+l3z+l • 0
....i.---.;-.-++ll• (!) x+2y+3z • 10 (plane bisecting join AB at
right an!lee)
(!!) x2+y 2 +z -sx-6y-z+l2 • o.
(Sphere on diameter AB).
Interior and sur-
face of rectangular pri11111.

TCPIC. 14
_VELOCITY ANO ACCHERATI� (page 494)
A· A.• (i) 400Dl (ii) 5041a; 616m; 56•/eec (iii) t(616 - 400) • 7'J;D/eec
(iv) after 2 aec, 51, sec; one ill on way up and the other on way
down. (v) after 71:; sec (vi) 400H20(t+6t) - 16(t+6t) 2 ; ·
(120-32t-16(6t)1'i/•ec; (120-32t)�/aec
(vii) 8811/aec; 24Ji/aec ; - 3lm/aec 2 (viii) 12<*il/aec; -3211/aec:2
(ix) · 120-32(t+6t); -32-/aec 2 ; -Jz-/eec 2 (x)-after Y.-
4 HC"'•1 62511l·
LI
ANSWERS
(xi) after 10 oec; ,lOO'fl':/aec

,-
(xii)(a) after 2 sec; X • 576m'
(b) after SJ: sec; x • 576m. (xiii) (a )
,;
-
120 'loo
10 t � t
I � 3 4

( )
o�
I!.
-200 ____ -+
y §
(xiv) \Then v-88, t•l, eqn is x-504 • 88(t-l)

u
(xv) "v" represents gradien t of tangent; "a" represen ts con-
cavi ty of curve at point where v-1, a•-32, t he curve is rising
(wi th tangent inclined at angle of 11/4 wi th x axis), and curve
is concave down.
(xvi) greatest s'peed 200:n/sec when hi ts grouhd; leas t speed is 0
when at highes t poin t on path.
.l: (1) v • 3 t2-12 t +9, f c 6t-12 (ii) 9m/sec; 12,,jsec2 q
(iii) af t er 1 sec, 3 sec; when x•60m; 5 6m I t
(iv) -'jtfSec respectively (v)/'
(a) i<t<3 (a) 0 � t < l or t > 3
(vi) (a) t > 2 (a) 0 s t < 2
(vii) 6�/sec2 , after 3 sec;
x • 56 (viii) I
-------... �
Eqn inflexional tangent is
X - 58 = -3(t-2) 0�
� ;
> 5 • 3 4 5
�:; _:: :�';. ' x ] :un vel is� when t • 1,J
1 2 �t !-lax vel 24m/sec when t = 5
�'....__,..3 4 s' Greatest neg vel is -3m/sec when t•2
s
(xi) !lis t = ( v d t • lf>r.>; l6m/sec
_§.,a • �24, ,b = \; -24; -10 7. (i) 2 hrs (ii) 40•-tn
(iii) 20km h (iv) 30km h =
.!.· (i) ft= O, n/2, J,r/2 (ii) l;O;l (vii) n/2-1 a 11-2
lf a 1, -,r/2', 3,r/2 11/2 fl

l_. a•4, b•3; 5\; -3/3; -42 li)., A=7, B•5


f • - i + sec2 t; O; l; 0
( +t)2

!. J.· (i) [x • 3t 2 -S t +2); .o (ii) [x • t +2cost-9); t • w/6, x = f+/3-9


(iii) [x • t2 /2 + 1.og t + 3); x • � when t = l
(iv) [x • e t +e-t-2), 2 (v) [x = tan t - 4t + l); x • 13 -
4
+l -1
l· ·( i) {x • t L3t2 +6 t-l); 7 (ii) [x • t2 -5t+6); when,t•2, .,.,-1 and
when t=3, v-1 (iii) [x•t4 -Jt2 -4 • (t2 -4)(t2 +1)); x • 20 when t"2
(iv) [x • sin2t - cos3t]; /3/2 (v) [v • -4/t + J log t
+ t2 ],
v • 2 + 3 log 2 � 4•079 (vi) x • �e2 -2e+7/2
i:k • 8; 128/3inlsecr 160/3 m

t•
5. x • a sin n t
-
V
& (1) 0 .S t < -
11

4
or
311
4
< t � 11

er,,
11

(ii) 4 vdt • 8

!• Ci) a;:er J s·ec; 32/:7.111 (ii) after 2° sec b.V


(iii) 4mlsec when t•O
!• A • 1/80; B • 1/6400 (i) X • 6400 log(l+t/80) 0
(ii) Start v dv • - -1- v2,' v • 80e-x/6400
dx 6400
-4 £t

l· x • 0 LII
ANSWERS
(i) _L_,
21t+li'
t cos t (iii) 2/,;+I; - 4
-2t
A • 10, k • 2; v • lOe (i) when t • �log 2 f •347 sec
(ii) lOe-4 +
•183 cm/sec (iii) x a 5[1-e-2t]
(iv) x = 5(1-e- 2 ) +
4•324 cm; t = -',log 5 f •805 sec
(v) 5 c:,, (when t -+ =) (vi) v = 10-2x (vii) ''; cm.

12. v • -0m/sec 13, A. (i) 13 cro (ii) e cm B. (i) Hint:


3 4= =-i. - - 2 192
v 2 - 24 - x ; 2.3 er" (ii) Hint: v �7- -48; 11
3.cm
2
Show v 2 • 4[log(x+ 2)-log2); v = 2/iog 11 3· 1 cm/sec +
Hint: v 2 • �(80x-x 2 �9;40m; 5 m/sec J.2., (iii) For P : 3 or -2;
1
1 1
For Q: - ce 3 -l� 17, Note �v 2 • 9 .000 - 5;
3 - X

-
When x • 8000, v .}si/2�3· 5 7 km/sec·; 18200-6400:,11 800 kin
11----
5 " Tl Tl
c. l,, (.!.!) (n•l 6); sec; a • cm�6 4l1lJI ,Time • 2 • = sec
8 8 96 48
l· (a) Note when t•O, v-nk: (b) When x•O, cos nt + sin nt = O,
i�e. tan nt • -1, i.e. t --�; v • -12 kn. approx 12•34m/sec 2 i·
Tl Tl
in+ ·.l: (n • , £ • ). Max, vel. is. 2 /2,mt
2 sec; initial vel. is
.2 4
v 2
o vo
2m/sec in+ i· •v • n ;;:r - x2 ; x • • ii""" sin nt.
(ii • 4); 10 cm/sec; f cm . ]., In last part,
easiest to put V • nb in formula previously derived in the question,

l• (!.!.!) (_! ) x• 12.x0cos(t -� t) (�) x• 12 xocos(t + � t).

l· Hint in (ii), when length is


5f, x •
5
f - f. t •

2.f�
I:_ 2. (�) (j) 24 sec (i.t) 3360 m (.!!_) (_!) 18 sec (ii) 150 m/s
1... (�) (j) 50 m/s (ii) 31\ m (ill.) 251!/2 m (.!!_) (j) 100/! m
(ii) 4 sec (not 1 sec)
4. t = 5/7 sec; x = 15 m, y 17\ m
TOPIC 15 (page 505)
4 . 13
�· J., (i) - )
3 U! - 9
r= r.:c (iv)-3
{ll!) 3+,10 (not 3-,10) (v) 225 °
{tan(x+y) • 1J W> - -35 �> - -53 l· CJ> 13 cu> - 84
ir (iii)
- 5
!!.
13 24
W
1
?Ill' W> s
3
<.ill 1 slio lli> E
(!) 3/4 (1!) -2 (not� (iii) 24/25 (iv) 7/25 (v) -120/169
(y!) -11/16 5. 12-1; other'"root is tan""ll2�0 since tan 225 ° • 1

§.,
also.
/&+12
;
-4- --4-
ii-If
7, Other root is tan 97� 0 ; since
tan(l80 0+15 0) • 2-IJ
- 1
7u; 7u;
5 1
!.•
-7
.2.· �) i; s
Jg_, See answers to Set 4B, lZ.· � •.13 17'
°

!· ,!, l,, l· See answers to Set 4D, 4F, 4G,


If 21311
S.· J.,U.) y-2 • 4 ,J(x
,:
- ) At point [O, 2 - - -J
6 3
If 1!
(ill y-1 • �(x - ); - 2, ,i, (ii) f' (x) • .sec x tan x,
4 4
f"(x) • sec x (sec2 x + tan 2x) f"' (x) • sec x tan x(Ssec2x-ttan 2 x)
LUI
ANSWERS
§.• (!!) f'(x) • -cosec 2 x 7. See Answers to Set 41.
-2·
,,.
l· (!) l; )s(,2-1);
I sin2a
2 (.!!) 6(ff-3); �[a + - -J; -;
2
1 <b-tsnb) 2 -
i-
1 Sff ff
(£) 6(2ff+3); 4 l· (.!) TI' (!!) k • !.;, t • 43; 12 S"2

:.• 2
(£) log(2+/j) (� �log3 (y) log(7j) • log2 - l,log3
ff ,,. ff
Ci) �log 3

12 !• 2(2-,2) J,, (!!) (!) ·P(4 , 1); log 2 sq units


f>
(.!!) ff(2 - cu units (£) acute angle • tan-1<t> • S3 ° 8'
8. (.!) 1 sq unit (!!) /Jff cu units 2,, log( /2+1) units,
12· y • ,nn x + l, sin 2x
(.!) .(-0,0), (11,0),(2ff,0)
(!!.!>-:1,
( II J/j
.!!) (3, 4) Max; (11,0) horiz,
S11 -3/i
inflex; ( , - -) Max.,
3 4 0
(.!l!) area 2 sq unit
S11 2
(y) vol 8 cu unit.

TOPIC 16 (page 510)


1. (.!) 3 Qu -2x 3 (£) 7 @ S, Not monic. £• Only (a).
3. (.!) k•ll (.!!) k•3, 4,s; i i k is any integer 2:. 3,
!, (.!) t•S; m,n arbitrary (ill l•4, ·m�-6, n arbitrary
i..
(jj.!) l,-4, m--6, no/,7 (1J!) 4, m--6, n•7. 5. See StlDIDary in text,

-rrcJ
(Rationals have a multiplic4tive inverse; :i.ich the integers and

.... ···;�
�':'.\
:I�J
g,.

I
f.-+' J.. w!��,-
®::=!
(i) x•-2, 1,3
f. (i) x•l,1,-3 (1) x-0,0,l, l
,.,
(ii) -2,1,3 (ii) 1,1,-3 (ii) 0,0,1,1
(iii) -2SxSl or x?.3 (iii) x 2 -3 (iii) all x
@
wrr . 4!�)xc�
(iii) X :1:2
O "
-,.,'------''"""---4-,c

-� -i
1. (a) P(x) • A(x).(Sx2 -12x+20)f(-23) (b) P(x)•A(x).(5x-12)+(2Sx-15)
!, Q(x)•Jx 2-ex-l; R(x)•l4x-7. 3x�-8x 3+sx 2-2x-9 • (x2 +2)(3x2 -8x-l)+
(14x-7). a-16, b•-6; (x 2+2)(3x2 -sx-l).
9, (i) -9 (ii) -25 (iii) O,(xtl) is facto� p(x)•(x+1)(2x-l)(x-3)
12· (i) (a) (x-l)(x2 +x+6) (b) (x-2)(x-7)(x+3) (c) (x+3)(x2 -x-l)
(ii) (a) x-1 only; other roots unreal (b) x-2, 7,-3 (c) x•-3, ls) to•
11. Roota are x • �. 1*15 12. (i) a • 2 (ii) a • -2, b • 8
it:
11'.
a•2, b--7, c•5. 14. p('ir·• (x+l)(x�3)(x-2)(x-5); -1,3,2,5
(i) (x-l)(irt2)(ic-1r2 (ii) k(2x+1)(3x-2)(x-4); k is a constant
(i) a•2, b--7, c•8 (ii) a•3, b•S, c•6, d•-9

I:
17'. (i) 2 (ii) -3 (iii) -4 (iv) 3/4 (v) -4 (vi) 0 (vii) 10
(1) O (ii) -� (iii) 5/2 (�v) -3/2 (v) 1/3
k--6; a1111 • -2, product• -3. l2•
a•2; x-2, l•l:i
LIV
ANSWERS
x 3�+2 • (x+l)(x2 -x+2); only·real zero is -1.
Eqn of tan at P is y • 4(x+l)
Coords of Q are (1,4) [Note 3x2 +1•4 at P,......,...,....�-..,�-��-
Coords of Rare (-2,-8) �
11 - 2) • 3 sq unit.
Area is C-i;
4 3
-:,i� :<.cu+�
(ii) between -3 and -2.
Eqn of tan is y - P(z1) • P'(z1)[x-z1l
(-1) See text for sketches. (W z1 • z 2 • z 3 • all equal r.
1i· (I) (.!!) Successive approximations are 2•5, 2°-25, 2•125
W 2,19 approx. (W (.!!) Successive approximations are
1. 5, l•25, 1°125 (_ID l• 17 approx.

TOP IC 17 (page 512)


�· .!.· l-7x+21x2 ; 1 + 9x + 1�5 x2 ; 1 + 2x - 3� x 2
2, a•l , b•-1 c•-6, d•lO 3. (1) ka-1400 C.ii) W 2 02) 6
4. Tu 1-5x+1oxL1ox 3+5x4 -x s;-O•904 (W 1 + x + 3x 2 /8; 1 • 104
(!) ·9988 (1!) 1•07214 7, (!) a • -2, b • 1 � a • 11/20,
b • -1/10; -11/50 ..!!.· (!)--4 (W 1+9x+3ox2 +45x +30x 4+9x 5+x 6
.l!.· See answers to Set 68• ---------�
c. 1, -5:8 2, 1•3439; $343•9; by logs $343
!, (!) 462a6b-s (!!l 310,5,11 U!!) 15360k 3 ; 1//J
20 r-1 u 20 r
i• (!) ur • C r-l x , r+l • Cr x ; r:(21-r); 16th, 17th terms
0 r r-1 19-r (x)r; 1 th• 17th
(.!.!) ur • 19 C�l(i_/ - (x) u
r+l
• 19 Cr(i_) 6
terms. 5. n•7 (i'2) (.!.!) n•2 or 3 - 6•.(!) n 8
s
(.!.!) n • To [Note 5 • n-15].

.
TOPIC 18 PRCBABILITY (page 514)
!·t·
"
See text l· (5,6,7,8,9} t! � I ' q (a) 1/5 (b) 0 (c)2/5
10
I
0 0 0 0 0
J· (i) 8 0 0 0 0
o A a ((6,6),(6,8),(8,6),(8,8)}; P(A) • 4/25
B • {(5,6);(5,8);(6,5),...,(9,8)}
7 0 0 0 0 0 P(!I) • 12/25
6 0 o C s ((5,5),(6;6) •., • ,(9,9)}. P(C) • 5/25
'
0 0 0
0 D • {(5,9);(6,8);(7,7);(8,6),(9,5)}
0 0 0 0
P (D) a 5/25
1 E • ((7,5)). P(E) • 1/25
(il)'l 0 0 0 0 A • ((6,8),(8,6)}, P(A) • 2/2C
s 0 0 0 0 B • ((5,6),(5,8),(6,5),.•. (9,8)}.
1 0 0 0
P(B) • 12/20
0
C • {�}. P(C) • C/25 /CD)
6 0 0 0 0 D • ((5,9),(6,8),(8,6),(9,5)} �4/20
5 0 0 0 0 E • {(7,5)} P(E) • 1/20

6
5 '!
{(O,O,O); (0,0,1),...,(l,1,1)} P(A) ·• 1/8; P(B) • 3/8; P(C) • 3/8;
P(D) • 3/8 + 1/8 • 1/2; P(E) • 1 - 1/8 • 7/8
(a) 6/13 (b) 7/13 (c) 4/13 (d) 4/13 (e) 6/13 (1 is not a prime)
(f) O (g) 7/13 (h) 2/13 (1) 8/13 (j) 10/13 (k) 7/13
See text, 7. (1) (a) 1/4 (B) 1 - 1/4 • 3/4
(ii) 18/40 +
13/40 • 31/40 (iii) 18/40 + 13/40 - 7/40 • 3/5
(i).6/36 • 1/6 (ii) 2/36 • 1/18 (111) O (iv) 2/9 (v) 7/9
(vi) 5/6 2,, (1) 19/30 (11) · 7
LV
ANSWERS
10. (i) 15/32 (ii) 7/32 (iii) 13/16 11. (i) 1/5 (ii) 7/20
(iii) 11/2 0 (iv) 9/2 0 1 2. (i) 1/4 (ii) 1/4 (iii) 1/2 (iv) 3/13
(v) 11/ 2 6 (vi) 16/52 = 4/13 (vii) 8/52 = 2/13 (viii) 12/5 2 = 3/13
(ix) 2 8/5 2 • 7/13 (x) 28/52 = 7/13

5+�::
(i), (ii) See text (iii) 5/9 . 1/6 = 5/54 (ii) 1/6 . 1/18 = 1/108
(a) (i) 2 5/144 (ii) 5/48 (iii) 2 9 = ;� (ll) (i) 1/64
(ii) 1�
4
(iii)
1 25 +
��;! =
7
i 15. W (i) 1/16 (ii) 1/8
(iii) 1/16 (iv) 4, 1/16 = 1/4 (v) 1/2 , 1/ 2 • 1/4
(vi) 2 , 1/4 = 1/ 2 (vii) 1/4 (viii) 2 . 1/4 • 1/ 2 (ix) 1/2704
(x) 1/1352 (xi) 1/169 (xii) 2/16 9 (xiii) 1/169 (xiv) 1/52
(xv) 1/13 (xvi) 1/2 08 (xvii) 1/104 (xviii) 3/4
{E_) (i) 13/ 2 04 (ii) 13/102 (iii) 13/204 (iv) 4 , 13/204 = 13/51
(v) 13/51 (vi) 2 6/51 (vii) 25/10 2 (viii) 2 5/51 (1x) 1/2 652
(x) 1/13 2 6 (xi) 4 /663 (xii) 8/663 (xiii) 1/2 2 1 (xiv) O
(xv) 1/17 (xvi) 1/204 (xvii) 1/102 (xviii) 13/17
.ll· (i) \[l + l�l = �� (ii) (a) \[(,£/ + (1) 2 1 a 3;� (ll) 841/1600
.
(iii) (a) \(8 3 5 + 4 6 = 449 (a) 449
' 7 10' 91 1680 840

�-
17. (i) 47/90 (ii) 43/90 �- Y; probabilities are 2 8/55; 30/55
(a) 36/169 (b) 49/169 (c) 42/169 (d) 84/169 (e) 9/169
(f) 21/169 (g) 9/169 (h) 1/169 (1) 12 /169 (j) 1/13
(k) 1 - 81/169 • 88/169 20, (i) 56/60 • 14 /15 (ii) 7/60 (iii) ..l
��- (i) _1.!! • 8 (ii) 6 • 2 (iii) 1 2 = 35
105 35 105 35 105
4 12+16 +18
(iv) 105
46
• 105
15

(v) _ll • _!!. ( i) 8 +9 +1 2 = � ( ii) 1 2+16+18 +24 • I


105 35 V 105 105 V 105 3
(viii) 1 - 6 = 33 22• (i) 6 (ii) 19 (iii) 262
75
105 35 125 375
899 1 4 5 3 397 • 1 4 3 5 4
(B) 1890 3 2
(a) 1890 = 3[7 + 9 + 101 3[7' 6 + 9' 8 10· 91
+
(i) 81/361 (ii) 1/19 (iii) 9/361. (iv) 2 (6/19 , 8/19) a 96/361
(i) 3/8 (ii) 1/2 (iii) 1 - 1/8 • 7/8
(i) 1/16 (ii) 1/4 = 4 . 1/16 (iii) 15/16
(i) 1/ 27 (ii) 8/27 (iii) 19/27 (iv) 26/27 (v) 6/ 2 7
(i) 27/125 (ii) 12/1 25 (iii) 36/1 2 5 (iv) 81/1 2 5
29
(1) a 72 :% • \(lltli) (ii) (a) ....il = s-!-%
1
� • W � 8
(B) \(1 + ..ll + \[ 37
l • 9 a 4�%
42+9
8 16 100. 800 8
B,
T. {!) 126 {!!) 120 l• ill 105 (ill 720 U!!) 105 X -720 • 75600
l• {!) 7920 (ill 330 {!) {!) 120 (!!) 5 ill {!) (5,3,2,l)x41 • 720
(.!!) 30 4, {!) 220 (,!.!) 13 20 {!) (i) 1 ill) 6 {Ju (!) 60
ill) (4 .3.!) x 31 • 360, 5. {!) {!) 24 {!u 48 (il) W 35 (hl 20
{!) 60 (il) 126 ]., {!) 40 {g) 2 880 • 5 I 41 .!!,, {!) 462
(!!) 3,(4!) • 72 2.· 7c x 6 c • 700; 61 • 720
4 3
10. W 360 (ill 6c a 15; S.4.3.1 • 60; 4-c = 4·(Note 1 cannot be used)
3
{!) 6 c • 15 ill) 216141 • 15 No, of ways for k•0,1,2 ...,6 is 1,6,
4
11, 2
15,20,15,6,1 12, U) 210 ill) 24 13 • .(& (!) 7 (!!) 6 (p -5)
(iii) 2 il!) 7- 0!) 17 (.y!) S or 16 '""&ill
10 (p 33) ill r•2, n•4
1s. {!) c4 + c5 + c + r; • 372 (!!) c0+ c1+ •• + c4•163,
2s200
9 9 9 9 8 8 8
14, 6 7
assuming she may decide not to take any books at all; otherwise 162
if &he takes at least 1 .!!b�oo"-'k,.,_.______
LVI
AtlSlo!ERS
2 l J. (!.) l 3 - 3
.£: 1. 2. 86400; TI · 55 ill 44 {El TI i· 91
33
51 l 51SI S 90 3
W 9i • 3024 <.!!> 9T • 126 1.· 1&+72 • 9o; TI • i;
(1) 1/330 (ii) 7/66 (!_!!) 21/55 (.!:t) 4/11 l· s+2o+6o+48 • 133;
1+4+12+12 ..1..2.
133 • 133 - 8. (1) - 4/9 (ii). -- 1/120 - 9, (1) 1/30 (11)- l _ l.
1
6 6
10 9°
(1) 9.9.8.7.6 • 27216 (!!) c5 • 252 (iii) c5 • 126
1 1 - 107 - 215
P(A) • 108 ; P(H) • 216; P(A) • 108; P(B)• 216
.ll· ='
('i\ 6· .! . .! r1n1 ..fil. • 180,' .!
s6'
2
6
c + sc
� 2121
4 15
6
.L fi\ (415121)31
2 ,_ �
111
1
1
• 1155
l
<.!!) 5 S S 5 5 a 31 Jl, {!) (!} JY • 21Q7
c+ c+ c+ c+ c
1 2 3 4 5
,, 3 2 1 13 1
ill s2 • s1 • 5o 5525 (.!!) W <s2>
• ill 52.51.50
t 3 1 l3 12 11 11
ill!) (!) (':,) • 64 ill 52 ' 51 . 50 = 850
I3 l 26 25 2 4
(1Y2 (!) (�) • 8 ill 5 2 ' 51 ' 50 a 17
2

1 l 1 l 12 11 11 22
{_y) W 4 · 4 or· 4 • 64 • 16 ill 51 • 50 or 4 ' 850 • 425
39 26 3 39 26 169
(_Y!) w 52 ' 52 a 8 ill 51 ' sci 425 D

5 3 5 7 4 7 4
c x c + c x c
(1) c 3 x c1 + c4 1 (_!!) 3 2 4 1 25
•2 • 33
ac Uc
4 5
30 2 0 10 l 30 20 10 50 1 1
(!_} {.!) 60'60'60 • 36 @ 60'59'58 m 1711 (!.!) (!) 3l•36 6
300 303 l 30 2 9 28 7
ill 31 'Tm.
50
1711 (iii) (!) <60> • 8 ill 60'59' 58 • 59
l l i. l .. �0.29.28+20.19.18 30 2 20 l
U0 �27..216 8 6.!.E.l +10.9,8 {_y) W <60> x <60> a IT
30 29 zo · 60. �.
ru
5 58 l 1 15
ill 60 • 59 ' 58 • 59 {_y!) w 3 '
12 • 4
5
3 ' 59 59
D

D, 108o+81 o+2 43 2133 243 2 540 135


• 312 5 (11)
- 312 5 -· (1) - 4096 • 1024
(<\
'>.:.I
5
154 77 54 2 7 810 162 1053
<.!!) 4096 • 2048 l• 64 • 32 .i• (9 S° 62 5 (_!!) 3125 C

2 882 2 133 405 90 45


(iii) l - 243 • 3lli � 3125 . .l· ill 102 4 (ii) 102 4 • 512
3125
648 81 16 1 l 343 • 386 2133
(.!ill 10 2 4 • 12 8 (!_y} 10 2 4 • 64 i· - 72 9 72 9 l· 3125
1 64 n
(1) • •00001% (11) 1 - W • 9M19936:t .,2., (.!) 1-(� ;
W
Hin t � + '1, i.e. 2 + 10, Le n + 3 ,
n
2 5n 6n n
(_!!) 1 - (6) > �' i,e, (5) > 2 i,e, (1•2) > 2 and by COIIIIOD
log10 2 5.9�+9 5 10'91854 ,
logs, n > > 3, i.e. n • 4. l51,, � • 91854
1 1,2 •
5+1o+21 2,:0810 02; qlO+loq 9 45q8 2 , where g
180() • • • p+ p • •96,,-1!-:_:02
"' 1 1 59049 32805 /'!10 45 1
.!, ill np • 5 10 • 2 <.!!.> ---rii'S• ---rii'S•
(iii) 59049; 32805; 7290; 810; 45; 1 i, (!) 5 X f • •5
w• w•Tci5'
'liQO
7
o
(!!) 811111! answers as 1 (11) (!ill W 59049 �) 40951

LVII
.I\NSWERS
TOPIC i9 (Page 527)
l:

TOPIC 26 (page 527)


B
, l 1 1 l l . 1 2 3 4 5 6
-· � 11'4'9'16' 25' 36,... ); cges to O
T (]?) 13'5'7'9'll'l3''");

cges to 1/2 W (0, lo8102,log103,


· log104,log105,log106,... }; dges
3 a 15 24 35
@ {O,-l,O,l,0,-1,... ); oscillates W {o, ,10, ,26 , ,... }; cges
5 17 37
to 1 {!) (3,1,-1,-3,-5,-7,... ); dges
5 13 23 49 95 193
W 12,4,8,16, 32,64,... ); cges to 3 (hl (3,5,3,5,3,5,... };
-1 1
oscillates (1) {l,O, ,o, ,o,... }; cges to 0
3 5
e e 2 e 3 e� � e 6
W 11,2 ,3 ,4 ,5 ,6 ,... ); dges l· (!) a.. 3, b•O, u, • 48;11mit is
+ '" (ti) a•O, b•B; u, • 1/2;. limit is O (!ill a•5, b•4; u2 • SOI.I;
limit is ..... 3. (e) is correct 4. W 2/3 ill 11/2 W 8
W -1 < x < 1 -(ti) x > l or x < -'i U!!) x • -1; when x•l,O seq,
is (1), (0) respectively.
U • 5/3 t!!} from 16th term onwards {Ju. least value of N is 1556.

LVlll

Das könnte Ihnen auch gefallen